[ { "question": "You are a helpful medical expert, and your task is to answer a multi-choice medical question using the relevant documents. Please choose the answer from the provided options. Your responses will be used for research purposes only, so please have a definite answer.\nHere are the relevant documents:\n A new family with a bleeding diathesis and FVIII deficiency secondary to abnormal binding of von Willebrand factor (vWF) to factor VIII (FVIII) is described. Two propositi of this family, an 18-year-old male and a 33-year-old female, both with a history of epistaxis, bruising, bleeding from the gums, epistaxis, hemarthrosis, and hematoma, were analyzed. Also additional members of the same family with no bleeding history were also studied. The propositi showed normal vWF activities, low FVIII activity; one of them had been diagnosed as having hemophilia A and the other was a hemophilia A carrier. Both showed a very poor response to treatment with FVIII concentrates and desmopressin (DDAVP) but a good clinical response to cryoprecipitate. APTT was prolonged and no inhibitory activity was noticeable in their plasmas. Thirty-five units per kilogram body weight of Hemofil M was infused to both propositi and FVIII reached basal level within 60 minutes of the infusion. No FVIII response at all was observed in the female after intravenous DDAVP administration. However, the male who received the infusion of 35 U/kg body weight of Humate-P achieved a normal FVIII level that was maintained for 12 hours. Multimeric analysis of vWF was normal in all the members studied. Von Willebrand factor domain for FVIII binding was assayed in the two propositi and in six other members of the same family by using a non-isotopic and sensitive method, a modification of the one previously described, using the Hemofil M concentrate as exogenous FVIII. The data obtained showed that both propositi had similar binding to that observed by using plasma of a patient with severe von Willebrand disease. Furthermore, five siblings had a decreased binding of vWF to FVIII, when compared with plasma from normal individuals or patients with hemophilia A. We also observed that, for screening purpose, the ratio of bound FVIII/immobilized vWF (at saturation of the anti-vWF and offering of 1 U/ml of exogenous FVIII) distinguished two levels of abnormality (normal range 0.70-1.15, propositi 0.004-0.007, and remaining members affected 0.25-0.42). The most probable explanation is that the propositi are homozygous or double heterozygous, the other five siblings affected being heterozygous for a recessive vWF defect. This more accessible assay presented here may be of help in routine analysis for diagnosing this type of von Willebrand disease, which has important implications for therapy and genetic counseling. To assess the frequency and characteristics of children with inherited bleeding disorders that were initially misdiagnosed, leading to inappropriate disease management. This study was conducted at the Haematology/Pathology Department of Fauji Foundation Hospital, Rawalpindi, Pakistan, from August 2014 to August 2018. Children who were diagnosed with an inherited bleeding disorder but did not respond to initial therapy were reevaluated. In total, 62 children were diagnosed with a bleeding disorder. Of these, 27 were diagnosed with an inherited bleeding disorder and 35 with an acquired bleeding disorder. Of the 27 children with inherited bleeding disorders, 18% (n = 5) were misdiagnosed and treated inappropriately. The median age of the misdiagnosed patients was 9 years (range, 5-13 years). Three patients with Bernard-Soulier syndrome had been misdiagnosed as having immune thrombocytopenic purpura, 1 patient with von Willebrand disease had been misdiagnosed as having hemophilia A, and 1 patient with haemophilia B had been misdiagnosed as having hemophilia A. There are chances of misdiagnosis and improper or invasive management if comprehensive laboratory evaluation and a thorough clinical evaluation are not performed in children with congenital bleeding disorders. Congenital bleeding disorders account for approximately one in 10,000 births. Dentists are often anxious about delivering treatment to this special group of patients. In the Irish Republic, patients with inherited bleeding disorders have their dental care co-ordinated centrally at the National Centre for Hereditary Coagulation Disorders (NCHCD), St James's Hospital, Dublin. Dental care is normally integrated with routine outpatient haematological appointments. This ensures regular monitoring of oral health and the early treatment of any hard/soft tissue pathology. This article describes, in simple diagrammatic form, the normal coagulation mechanism (Figures 1 and 2), explains common coagulation terms (Appendix 1), and examines the three most common congenital bleeding disorders: haemophilia A, haemophilia B, and von Willebrand disease. General recommendations based on the current literature are provided with respect to procedures that are appropriate to perform in a general dental practice setting. Although not discussed in this article, it is important to note that non-coagulation bleeding disorders also exist. These include: hereditary haemorrhagic telangiectasia; blood vessel wall defects resulting from connective tissue disorders such as Marfan syndrome and Ehlers-Danlos syndrome; and, platelet disorders such as Bernard-Soulier syndrome, resulting in defective platelet adhesion. Chronic menorrhagia causes anemia and impairment of life quality. In this study the aim was the screening of bleeding disorders in adolescents and young women with menorrhagia. The study was performed prospectively by pediatric hematologists. A form including demographic characteristics of the patients, bleedings other than menorrhagia, familial bleeding history, characteristics of the menorrhagia, and impairment of life quality due to menorrhagia was filled out by the researcher during a face-to-face interview with the patient. A pictorial blood assessment chart was also used for evaluation of blood loss. All patients underwent pelvic ultrasound sonography testing and women also received pelvic examination by gynecologists. Whole blood count, peripheral blood smear, blood group, serum transaminases, urea, creatinine, ferritin, PFA-100, PT, aPTT, INR, TT, fibrinogen, VWF:Ag, VWF:RCo, FVIII, and platelet aggregation assays were performed. Platelet aggregations were studied by lumiaggregometer. Out of 75 patients enrolled, 60 patients completed the study. The mean age was 20.68±10.34 (range: 10-48) years and 65% (n=39) of the patients were younger than 18 years. In 18 (46%) of the adolescents, menorrhagia subsided spontaneously. In 20% (n=12) of the patients, a bleeding disorder was detected (1 case of type 3 von Willebrand disease, 2 patients with low VWF:Ag, 1 case of probable von Willebrand disease, 3 cases of Bernard-Soulier syndrome, 2 cases of Glanzmann thrombasthenia, 2 cases of immune thrombocytopenic purpura, 1 case of congenital factor VII deficiency). In patients with menorrhagia, at least complete blood count, peripheral smear, aPTT, PT, VWF:Ag, VWF:RCo, FVIII, and fibrinogen assays must be performed. When there is history of nose and gum bleeding, platelet function assay by lumiaggregometer must also be performed. In nearly 50% of adolescents, menorrhagia is dysfunctional and transient. Detailed coagulation assays can be postponed in adolescents if bleeding history other than menorrhagia and/or family history of bleeding and/or parental consanguinity is absent. All subjects with menorrhagia must consult with gynecologists and hematologists. None declared. Ehlers-Danlos syndrome includes 11 distinct entities. The diversity of this collagen dysplasia and its combination with other abnormalities make it difficult to understand physiopathologically. A case of Ehlers-Danlos syndrome is reported, which is novel owing to its combination with clotting abnormalities and especially with muscular dystrophy. To our knowledge this has not previously been reported. The patient was a young man aged 16 years who presented with Ehlers-Danlos syndrome satisfying Perelman's diagnostic criteria. His father and two brothers had comparable clinical symptoms, but his mother and sister were healthy. The four male subjects had an increased cephalin-kaolin time, reduced levels of factor VIII and Willebrand's factor (but without haemophilia A or Willebrand's disease), and, especially, an abnormal platelet ATP secretion. The proband alone had muscular disease with bilateral quadriceps fatigability and amyotrophy. The muscle enzyme levels were greatly increased, the electromyographic trace was myogenic, and the biopsy showed severe muscular dystrophy. This new observation poses the problem of the relation between clotting abnormalities and collagen abnormalities in the Ehlers-Danlos syndrome. It is difficult to classify this case within any of the 11 known types because of its muscular manifestations. It may perhaps be a fortuitous combination or an extension of the nosological framework of this syndrome. Differential diagnosis The differential diagnosis for Bernard–Soulier syndrome includes both Glanzmann thrombasthenia and pediatric Von Willebrand disease. BSS platelets do not aggregate to ristocetin, and this defect is not corrected by the addition of normal plasma, distinguishing it from von Willebrand disease. Following is a table comparing its result with other platelet aggregation disorders: Treatment Bleeding events can be controlled by platelet transfusion. Most heterozygotes, with few exceptions, do not have a bleeding diathesis. BSS presents as a bleeding disorder due to the inability of platelets to bind and aggregate at sites of vascular endothelial injury. In the event of an individual with mucosal bleeding tranexamic acid can be given. The affected individual may need to avoid contact sports and medications such as aspirin, which can increase the possibility of bleeding. A potential complication is the possibility of the individual producing anti-platelet antibodies. He published a Swedish-language article in 1926 about the disease, titled Hereditär pseudohemofili (\"Hereditary pseudohemophilia\"). He referenced six previous publications from the years of 1876 to 1922, totalling 19 cases on families with bleeding diatheses. The earlier authors attributed the condition to hemophilia (even in the cases of females) or to thrombopathy, which was discovered shortly before as the cause of what had previously been known as purpura hemorrhagica or Werlhof's disease. Von Willebrand also conducted hematological examinations on Hjördis and some of her family members. He recorded a normal or slightly reduced number of platelets and an undisturbed clot retraction, unlike Glanzmann's thrombasthenia. The bleeding time (Duke) was greatly prolonged, extending to more than 2 hours in some cases, while the clotting time was within the normal range. He concluded that the disease was either a new form of thrombopathy or a condition of the capillary endothelium. – hemorrhagic disorders – afibrinogenemia – bernard-soulier syndrome – disseminated intravascular coagulation – factor v deficiency – factor vii deficiency – factor x deficiency – factor xi deficiency – factor xii deficiency – factor xiii deficiency – hemophilia a – hemophilia b – hypoprothrombinemias – platelet storage pool deficiency – hermanski-pudlak syndrome – purpura, thrombocytopenic, idiopathic – thrombasthenia – thrombocythemia, hemorrhagic – vascular hemostatic disorders – cryoglobulinemia – ehlers-danlos syndrome – hemangioma, cavernous – hemangioma, cavernous, central nervous system – multiple myeloma – pseudoxanthoma elasticum – purpura, hyperglobulinemic – purpura, schoenlein-henoch – scurvy – shwartzman phenomenon – telangiectasia, hereditary hemorrhagic – waldenstrom macroglobulinemia – vitamin k deficiency – hemorrhagic disease of newborn – von willebrand disease – waterhouse-friderichsen syndrome Epistaxis, superficial and deep hematomas, hemarthrosis, gastrointestinal bleeding, hematuria represent the most frequent hemorrhagic events in congenital coagulation disorders. Occasionally, bleeding manifestations occur in unusual sites or are peculiar. A clotting defect may alter the clinical aspect of skin conditions or infections (hemorrhagic scabies or varicella). Hemobilia may occur as a complication of transjugular liver biopsy in hemophilia or Bernard-Soulier syndrome. Hemarthrosis of small joints of feet and hands occur in patients with hemophilia treated with protease inhibitors. Intramedullary hematomas of long bones have been described in α2-plasmin inhibitor or fibrinogen deficiencies. Spleen fracture with consequent hemoperitoneum has been reported in patients with fibrinogen deficiency. Rectus muscle sheath hematoma may occur in patients with factor VII (FVII)or FX deficiency. Acute or subacute intestinal obstruction may be caused by intramural wall hematomas in hemophilia and von Willebrand (vW)-disease. Physicians should always keep in mind that a congenital hemorrhagic disorder may cause bleeding in any tissue of the body and therefore alter the normal clinical features of a given disease. Factor X deficiency Factor XI deficiency Hemophilia A Hemophilia B Bernard-Soulier syndrome Platelet function defects Antiplatelet drug ingestion Fibrinolytic defects The objective of the present study was to determine the pattern of inherited bleeding disorders in southern Iran and evaluate the effect of a comprehensive coagulation laboratory and related efforts. A total of 545 patients with inherited bleeding disorders were evaluated during 1992-2007 by a cross-sectional study. Data were collected by a data-gathering form. Statistical analysis was done using Statistical Package for the Social Sciences version 15. A P value less than 0.05 was considered statistically significant. Overall 411 patients had common bleeding disorders including 326 hemophilia A, 46 hemophilia B, and 39 von Willebrand disease. Seventy-nine patients had rare coagulation disorders including deficiency of factor VII (n = 26), factor X (n = 18), factor XIII (n = 9), factor I (n = 9), factor XI (n = 7), factor V (n = 4), combined factor VIII and factor V (n = 4), and combined factor X and factor VII (n = 2). Fifty-five patients had platelet disorders including 23 with Glanzmann's thrombasthenia, 15 with Bernard-Soulier syndrome, and 17 with other platelet disorders, most of which (45) were diagnosed after the establishment of the comprehensive coagulation laboratory. Annual mean number of new diagnosed patients with common and rare bleeding disorders increased from 29 +/- 4 to 38 +/- 17. The ratio of the patients diagnosed with rare bleeding disorders to common bleeding disorders significantly increased after the establishment of the comprehensive diagnosis laboratory (P < 0.001).It seems that implementation of collaborative projects by the Shiraz Hemophilia Society and the establishment of the comprehensive coagulation laboratory and treatment centers have been successful in increasing diagnosis of the inherited bleeding disorders and consequently better management of the patients. – blood coagulation disorders – coagulation protein disorders – activated protein c resistance – afibrinogenemia – factor v deficiency – factor vii deficiency – factor x deficiency – factor xi deficiency – factor xii deficiency – factor xiii deficiency – hemophilia a – hemophilia b – hypoprothrombinemias – von willebrand disease – disseminated intravascular coagulation – blood coagulation disorders, inherited – activated protein c resistance – afibrinogenemia – antithrombin iii deficiency – bernard-soulier syndrome – factor v deficiency – factor vii deficiency – factor x deficiency – factor xi deficiency – factor xii deficiency – factor xiii deficiency – hemophilia a – hemophilia b – hermanski-pudlak syndrome – hypoprothrombinemias – protein c deficiency – thrombasthenia – von willebrand disease – wiskott-aldrich syndrome – platelet storage pool deficiency – hermanski-pudlak syndrome – protein s deficiency Diagnosis Basic tests performed in any patient with bleeding problems are a complete blood count-CBC (especially platelet counts), activated partial thromboplastin time-APTT, prothrombin time with International Normalized Ratio-PTINR, thrombin time-TT, and fibrinogen level. Patients with abnormal tests typically undergo further testing for hemophilias. Other coagulation factor assays may be performed depending on the results of a coagulation screen. Patients with von Willebrand disease typically display a normal prothrombin time and a variable prolongation of partial thromboplastin time. To determine the frequency of inherited bleeding disorders, its complications, and treatment modalities available for its treatment. Cross-sectional study. Patients with a history of bleeding tendency were tested for confirmation of the diagnosis. History and clinical findings were recorded. Laboratory analysis included prothrombin time (PT), activated partial thromboplastin time (APTT), bleeding time (BT), and fibrinogen assay. Patients with prolonged APTT were tested for factors VIII (FVIII) and IX (FIX). If FVIII was low, von Willebrand factor: antigen (vWF:Ag) and von Willebrand factor:ristocetin cofactor (vWF:RCo) were performed. When PT and APTT both were prolonged, FV, FX, and FII were tested. Platelet aggregation studies were done when there was isolated prolonged BT. Urea clot solubility test was done when all coagulation tests were normal. All patients with hemophilia A and B were evaluated for inhibitors. Of the 376 patients, inherited bleeding disorder was diagnosed in 318 (85%) cases. Median age of patients was 16.4 years. Hemophilia A was the commonest inherited bleeding disorder that was observed in 140 (37.2%) followed by vWD 68 (18.0%), platelet function disorders 48 (12.8%), and hemophilia B in 33 (8.8%) cases. We also found rare congenital factor deficiencies in 13 (3.4%), low VWF in 11 (3.0%) participants and 5 (1.3%) in female hemophilia carriers. Hemarthrosis was the most frequent symptom in hemophilia A and B (79.7%) involving knee joint. Inhibitor was detected in 21 (15%) cases. Fresh frozen plasma/cryoprecipitate were the most common modality of treatment. In 58 patients, no abnormality was detected in coagulation profile. Hemophilia A and vWD are the most common congenital bleeding disorders in this study. Hemarthrosis involving knee joint was the most common complication. Inhibitor was detected in a significant number of patients. Plasma is still the most common modality of treatment. Hemophilia is widely distributed all over the world, but little is known about its clinical profile in resource-limited regions. An insight into its clinical spectrum will help in the formulation of policies to improve the situation in these areas. To study the clinical profile of hemophiliacs (age <18 years) in Jodhpur region and screen them for transfusion-transmitted infections. A cross-sectional study conducted in the Department of Pediatrics, Umaid Hospital, Dr. S. N. Medical College, Jodhpur, over a period of 12 months. Out of a total of 56 cases enrolled, 51 (91%) cases were diagnosed as hemophilia A while 5 (9%) were diagnosed as hemophilia B. Positive family history was found in 26 (46%) cases. According to their factor levels, 25 (44%) cases had severe disease, 20 (36%) had moderate disease, and 11 (20%) had mild disease. The mean age of onset of symptoms and diagnosis was 1.73 ± 1.43 and 3.87 ± 3.84 years, respectively. First clinical presentation was posttraumatic bleed in 20 (36%), gum bleeds in 17 (30%), epistaxis in 4 (7%), joint bleeds in 4 (7%), skin bleeds in 4 (7%), and circumcision bleed in 3 (5%) cases. Knee joint was the predominant joint affected by hemarthrosis in 38 (68%), followed by ankle in 29 (52%), elbow in 20 (36%), and hip joint in 7 (13%) cases. All patients had a negative screening test for transfusion-transmitted infections. Occurrence of posttraumatic bleeds and gum bleeds in an otherwise normal child should warn the clinician for evaluation of hemophilia. Acquired hemophilia Ehlers-Danlos syndrome Factor XI deficiency Glanzmann thrombasthenia Haemophilia C Haemophilia type B Physical child abuse Platelet disorders Von Willebrand disease Normal values fall between 3 – 10 minutes depending on the method used. A disadvantage of Ivy's method is closure of puncture wound before stoppage of bleeding. Duke's method With the Duke;s method, the patient is pricked with a special needle or lancet, preferably on the earlobe or fingertip, after having been swabbed with alcohol. The prick is about 3–4 mm deep. The patient then wipes the blood every 30 seconds with a filter paper. The test ceases when bleeding ceases. The usual time is about 2–5 minutes. This method is not recommended and cannot be standardized because it can cause a large local hematoma. Interpretation Bleeding time is affected by platelet function, certain vascular disorders and von Willebrand Disease—not by other coagulation factors such as haemophilia. Diseases that cause prolonged bleeding time include thrombocytopenia, disseminated intravascular coagulation (DIC), Bernard-Soulier disease, and Glanzmann's thrombasthenia. Keeping an updated registry of bleeding disorders is crucial for planning care and documenting prevalence. We aimed to assess the prevalence of various bleeding disorders including rare inherited coagulation and platelet disorders concerning their clinico-epidemiological, diagnostic data and bleeding manifestations severity. Patients suffering from manifestations of bleeding or coagulation disorders presented to Hematology Clinic during 16 years were included and prospectively followed up. Demographics, clinical characteristics, complete blood count, bleeding, prothrombin and activated partial thromboplastin times, platelet aggregation tests and bone marrow aspiration were recorded. Overall 687 patients with bleeding disorders from total 2949 patients were identified. Inherited coagulation defects were found in 27.2%; hemophilia A (70.6%), hemophilia B (13.9%), factor I deficiency (2.3%), factor V deficiency (1.6%), factor X deficiency (4.2%), factor VII deficiency (2.6%), factor XIII deficiency (1.1%), combined factor deficiency (2.1%) and unclassified coagulation disorders in 1.6% of studied patients. Overall 72.7% had diagnosed with platelet disorders; immune thrombocytopenia was the commonest (74.8%), and inherited conditions represent (25.2%) in the following order: Glanzman's thrombasthenia (11.2%), von Willebrand disease (6.6%), Bernard-Soulier syndrome (1%) and Chediak Higashi in 0.4% and unclassified in 6%. Median age of diagnosis of coagulation and platelet disorders were 33 and 72 months. Presenting symptoms of coagulation disorders were: 25.1% post circumcision bleeding, 22.5% ecchymosis, 20.9% hemoarthrosis and 15% epistaxis. Symptoms of rare coagulation disorders were postcircumcision bleeding (20%), bleeding umbilical stump (20%), epistaxis (12%), hemoarthrosis (8%) and hematomas (4%). Presenting symptoms in rare inherited platelet disorders were purpura, ecchymosis, epistaxis and bleeding gums, respectively. Analysis of the clinico-epidemiological data of patients with bleeding disorders is a useful tool for monitoring and improving their quality of care. Causes other than coagulation Bleeding diathesis may also be caused by impaired wound healing (as in scurvy), or by thinning of the skin, such as in Cushing's syndrome. Genetic Some people lack genes that typically produce the protein coagulation factors that allow normal clotting. Various types of hemophilia and von Willebrand disease are the major genetic disorders associated with coagulopathy. Rare examples are Bernard–Soulier syndrome, Wiskott–Aldrich syndrome and Glanzmann's thrombasthenia. Gene therapy treatments may be a solution as they involve in the insertion of normal genes to replace defective genes causing for the genetic disorder. Gene therapy is a source of active research that hold promise for the future. Diagnosis Comparing coagulation tests Other conditions can also present similarly with bleeding after minor trauma or spontaneous bleeds and require exclusion before confirming the diagnosis of hemophilia. Some of these conditions include von Willebrand disease, scurvy, diseases of platelet dysfunction, deficiency of other coagulation factors like V, VII, X, or fibrinogen, Ehlers-Danlos syndrome, Fabry disease, disseminated intravascular coagulation, and child abuse. In von Willebrand disease, bleeding symptoms can be similar to mild hemophilia, but patients with von Willebrand disease have more mucosal bleeding compared to musculoskeletal bleeding seen in hemophilia. Von Willebrand disease is diagnosed by checking for von Willebrand factor antigen or von Willebrand factor multimers. [40] Similarly, in scurvy, Ehlers-Danlos syndrome, and Fabry disease; also, the bleeding is usually mucosal, unlike hemophilia, where it is musculoskeletal. In scurvy, there is a deficiency of vitamin C. [41] In Ehlers-Danlos syndrome, the skin is hyperextensible, and joints are hypermobile. The diagnosis is usually through clinical features, genetic testing, and tissue biopsy. [42] Similarly, in Fabry disease, patients may also have other organs being affected, including kidneys and heart, and have skin lesions called angiokeratomas. They also have pain in the extremities. Fabry disease is usually diagnosed with clinical findings and genetic testing. [43] In cases of platelet dysfunction disorders, bleeding is usually mucocutaneous, unlike hemophilia. Usually, these disorders are diagnosed by platelet aggregation studies or platelet electron microscopy. [44] In the deficiency of other coagulation factors, musculoskeletal bleeding is uncommon. In fact, sometimes thrombosis can occur, especially in patients with factor VII or fibrinogen deficiency or in patients with combined factor V and VIII deficiency. Specific coagulation factor assays usually confirm the diagnosis. Disseminated intravascular coagulation (DIC) that mimics hemophilia is hard to differentiate, but usually, there is an underlying condition in DIC, for example, acute promyelocytic leukemia. Diagnosis is usually carried out by blood tests that show decreased platelet count and the absence of factor VIII autoantibodies. Child abuse can sometimes be misidentified and confused with hemophilia, and it is essential to find inconsistencies in the history of how trauma has occurred. Other signs of malnourishment require vigilance, and x-rays may reveal evidence of fractures of different ages. [45] [46] – anemia, hypoplastic, congenital – anemia, Diamond–Blackfan – fanconi anemia – ataxia telangiectasia – blood coagulation disorders, inherited – activated protein C resistance – afibrinogenemia – antithrombin III deficiency – Bernard–Soulier syndrome – factor V deficiency – factor VII deficiency – factor X deficiency – factor XI deficiency – factor XII deficiency – factor XIII deficiency – hemophilia A – hemophilia B – Hermansky–Pudlak syndrome – hypoprothrombinemias – protein C deficiency – thrombasthenia – Von Willebrand disease – Wiskott–Aldrich syndrome – CADASIL – cardiomyopathy, hypertrophic, familial – cherubism The International Society on Thrombosis and Haemostasis Bleeding Assessment Tool (ISTH-BAT) is useful for assessing bleeding disorders. Its utility was tested in a small study, including patients with known inherited platelet disorders. The study demonstrated a specificity of 100%, a positive predictive value of 90%, and a negative predictive value of 100% using this assessment tool. [17] In the setting of von Willebrand disease, a BAT score >6 repeatedly correlates to a 99% probability of an inherited platelet defect such as BSS. [8] ] Similarly, other bleeding assessment tools like Molecular and Clinical Markers for the Diagnosis and Management (MCMDM) of type 1 von Willebrand's disease (VWD) and the World Health Organization Bleeding Assessment Tool are similar tools. An electronic version of MCMDM-type-1 VWD was developed in 2010. [17] Hemophilia is a sex-linked recessive genetic disorder characterized by the absence or deficiency of factor VIII (hemophilia A, orclassic hemophilia) or factor IX (hemophilia B, or Christmas disease) (Chap. 141). Hemophilia A constitutes 85% of cases. Spontaneous hemarthrosis is a common problem with both types of hemophilia and can lead to a deforming arthritis. The frequency and severity of hemarthrosis are related to the degree of clotting factor deficiency. Hemarthrosis is not common in other disorders of coagulation such as von Willebrand disease, factor V deficiency, warfarin therapy, or thrombocytopenia. Hemarthrosis occurs after 1 year of age, when a child begins to walk 2241 and run. In order of frequency, the joints most commonly affected are theknees,ankles,elbows,shoulders,andhips.Smalljointsofthehands and feet are occasionally involved. The authors report a case of hemarthrosis complicated by severe anemia related to a congenital connective tissue disease: Ehlers-Danlos syndrome. A boy fell down and suffered tumefaction of both knees with bilateral rupture of the rotula tendon. He underwent surgical reinsertion of each tendon on the rotula. He later showed an unexpected ongoing hematic effusion, with severe anemia. He was screened for coagulation disorders with no results. On taking a more detailed history and investigating the patient's phenotypical features, the authors diagnosed Ehlers-Danlos syndrome, hypermobile variant. The hemarthrosis and anemia were thus concluded to be consequences of excessive tissue fragility due to a congenital connective tissue disease.\nHere is the question:\nPEDIATRICS: An 18-month-old boy, with complete immunization schedule to date, who consults the Emergency Department for right knee swelling after playing in the park, without obvious trauma. In the directed anamnesis, the mother refers that an uncle of hers had similar problems. The ultrasound examination is compatible with hemarthrosis and in the analytical analysis only an APTT lengthening of 52'' (normal 25-35'') stands out. What is the most probable diagnostic hypothesis?\nHere are the potential choices:\n1. Marfan syndrome.\n2. Von-Willebrand's disease.\n3. Ehlers-Danlos disease.\n4. Hemophilia A.\n5. Bernard-Soulier disease.\nThe correct answer is: ", "gold_answer": "4 Hemophilia A.", "prediction": "You are a helpful medical expert, and your task is to answer a multi-choice medical question using the relevant documents. Please choose the answer from the provided options. Your responses will be used for research purposes only, so please have a definite answer.\nHere are the relevant documents:\n A new family with a bleeding diathesis and FVIII deficiency secondary to abnormal binding of von Willebrand factor (vWF) to factor VIII (FVIII) is described. Two propositi of this family, an 18-year-old male and a 33-year-old female, both with a history of epistaxis, bruising, bleeding from the gums, epistaxis, hemarthrosis, and hematoma, were analyzed. Also additional members of the same family with no bleeding history were also studied. The propositi showed normal vWF activities, low FVIII activity; one of them had been diagnosed as having hemophilia A and the other was a hemophilia A carrier. Both showed a very poor response to treatment with FVIII concentrates and desmopressin (DDAVP) but a good clinical response to cryoprecipitate. APTT was prolonged and no inhibitory activity was noticeable in their plasmas. Thirty-five units per kilogram body weight of Hemofil M was infused to both propositi and FVIII reached basal level within 60 minutes of the infusion. No FVIII response at all was observed in the female after intravenous DDAVP administration. However, the male who received the infusion of 35 U/kg body weight of Humate-P achieved a normal FVIII level that was maintained for 12 hours. Multimeric analysis of vWF was normal in all the members studied. Von Willebrand factor domain for FVIII binding was assayed in the two propositi and in six other members of the same family by using a non-isotopic and sensitive method, a modification of the one previously described, using the Hemofil M concentrate as exogenous FVIII. The data obtained showed that both propositi had similar binding to that observed by using plasma of a patient with severe von Willebrand disease. Furthermore, five siblings had a decreased binding of vWF to FVIII, when compared with plasma from normal individuals or patients with hemophilia A. We also observed that, for screening purpose, the ratio of bound FVIII/immobilized vWF (at saturation of the anti-vWF and offering of 1 U/ml of exogenous FVIII) distinguished two levels of abnormality (normal range 0.70-1.15, propositi 0.004-0.007, and remaining members affected 0.25-0.42). The most probable explanation is that the propositi are homozygous or double heterozygous, the other five siblings affected being heterozygous for a recessive vWF defect. This more accessible assay presented here may be of help in routine analysis for diagnosing this type of von Willebrand disease, which has important implications for therapy and genetic counseling. To assess the frequency and characteristics of children with inherited bleeding disorders that were initially misdiagnosed, leading to inappropriate disease management. This study was conducted at the Haematology/Pathology Department of Fauji Foundation Hospital, Rawalpindi, Pakistan, from August 2014 to August 2018. Children who were diagnosed with an inherited bleeding disorder but did not respond to initial therapy were reevaluated. In total, 62 children were diagnosed with a bleeding disorder. Of these, 27 were diagnosed with an inherited bleeding disorder and 35 with an acquired bleeding disorder. Of the 27 children with inherited bleeding disorders, 18% (n = 5) were misdiagnosed and treated inappropriately. The median age of the misdiagnosed patients was 9 years (range, 5-13 years). Three patients with Bernard-Soulier syndrome had been misdiagnosed as having immune thrombocytopenic purpura, 1 patient with von Willebrand disease had been misdiagnosed as having hemophilia A, and 1 patient with haemophilia B had been misdiagnosed as having hemophilia A. There are chances of misdiagnosis and improper or invasive management if comprehensive laboratory evaluation and a thorough clinical evaluation are not performed in children with congenital bleeding disorders. Congenital bleeding disorders account for approximately one in 10,000 births. Dentists are often anxious about delivering treatment to this special group of patients. In the Irish Republic, patients with inherited bleeding disorders have their dental care co-ordinated centrally at the National Centre for Hereditary Coagulation Disorders (NCHCD), St James's Hospital, Dublin. Dental care is normally integrated with routine outpatient haematological appointments. This ensures regular monitoring of oral health and the early treatment of any hard/soft tissue pathology. This article describes, in simple diagrammatic form, the normal coagulation mechanism (Figures 1 and 2), explains common coagulation terms (Appendix 1), and examines the three most common congenital bleeding disorders: haemophilia A, haemophilia B, and von Willebrand disease. General recommendations based on the current literature are provided with respect to procedures that are appropriate to perform in a general dental practice setting. Although not discussed in this article, it is important to note that non-coagulation bleeding disorders also exist. These include: hereditary haemorrhagic telangiectasia; blood vessel wall defects resulting from connective tissue disorders such as Marfan syndrome and Ehlers-Danlos syndrome; and, platelet disorders such as Bernard-Soulier syndrome, resulting in defective platelet adhesion. Chronic menorrhagia causes anemia and impairment of life quality. In this study the aim was the screening of bleeding disorders in adolescents and young women with menorrhagia. The study was performed prospectively by pediatric hematologists. A form including demographic characteristics of the patients, bleedings other than menorrhagia, familial bleeding history, characteristics of the menorrhagia, and impairment of life quality due to menorrhagia was filled out by the researcher during a face-to-face interview with the patient. A pictorial blood assessment chart was also used for evaluation of blood loss. All patients underwent pelvic ultrasound sonography testing and women also received pelvic examination by gynecologists. Whole blood count, peripheral blood smear, blood group, serum transaminases, urea, creatinine, ferritin, PFA-100, PT, aPTT, INR, TT, fibrinogen, VWF:Ag, VWF:RCo, FVIII, and platelet aggregation assays were performed. Platelet aggregations were studied by lumiaggregometer. Out of 75 patients enrolled, 60 patients completed the study. The mean age was 20.68±10.34 (range: 10-48) years and 65% (n=39) of the patients were younger than 18 years. In 18 (46%) of the adolescents, menorrhagia subsided spontaneously. In 20% (n=12) of the patients, a bleeding disorder was detected (1 case of type 3 von Willebrand disease, 2 patients with low VWF:Ag, 1 case of probable von Willebrand disease, 3 cases of Bernard-Soulier syndrome, 2 cases of Glanzmann thrombasthenia, 2 cases of immune thrombocytopenic purpura, 1 case of congenital factor VII deficiency). In patients with menorrhagia, at least complete blood count, peripheral smear, aPTT, PT, VWF:Ag, VWF:RCo, FVIII, and fibrinogen assays must be performed. When there is history of nose and gum bleeding, platelet function assay by lumiaggregometer must also be performed. In nearly 50% of adolescents, menorrhagia is dysfunctional and transient. Detailed coagulation assays can be postponed in adolescents if bleeding history other than menorrhagia and/or family history of bleeding and/or parental consanguinity is absent. All subjects with menorrhagia must consult with gynecologists and hematologists. None declared. Ehlers-Danlos syndrome includes 11 distinct entities. The diversity of this collagen dysplasia and its combination with other abnormalities make it difficult to understand physiopathologically. A case of Ehlers-Danlos syndrome is reported, which is novel owing to its combination with clotting abnormalities and especially with muscular dystrophy. To our knowledge this has not previously been reported. The patient was a young man aged 16 years who presented with Ehlers-Danlos syndrome satisfying Perelman's diagnostic criteria. His father and two brothers had comparable clinical symptoms, but his mother and sister were healthy. The four male subjects had an increased cephalin-kaolin time, reduced levels of factor VIII and Willebrand's factor (but without haemophilia A or Willebrand's disease), and, especially, an abnormal platelet ATP secretion. The proband alone had muscular disease with bilateral quadriceps fatigability and amyotrophy. The muscle enzyme levels were greatly increased, the electromyographic trace was myogenic, and the biopsy showed severe muscular dystrophy. This new observation poses the problem of the relation between clotting abnormalities and collagen abnormalities in the Ehlers-Danlos syndrome. It is difficult to classify this case within any of the 11 known types because of its muscular manifestations. It may perhaps be a fortuitous combination or an extension of the nosological framework of this syndrome. Differential diagnosis The differential diagnosis for Bernard–Soulier syndrome includes both Glanzmann thrombasthenia and pediatric Von Willebrand disease. BSS platelets do not aggregate to ristocetin, and this defect is not corrected by the addition of normal plasma, distinguishing it from von Willebrand disease. Following is a table comparing its result with other platelet aggregation disorders: Treatment Bleeding events can be controlled by platelet transfusion. Most heterozygotes, with few exceptions, do not have a bleeding diathesis. BSS presents as a bleeding disorder due to the inability of platelets to bind and aggregate at sites of vascular endothelial injury. In the event of an individual with mucosal bleeding tranexamic acid can be given. The affected individual may need to avoid contact sports and medications such as aspirin, which can increase the possibility of bleeding. A potential complication is the possibility of the individual producing anti-platelet antibodies. He published a Swedish-language article in 1926 about the disease, titled Hereditär pseudohemofili (\"Hereditary pseudohemophilia\"). He referenced six previous publications from the years of 1876 to 1922, totalling 19 cases on families with bleeding diatheses. The earlier authors attributed the condition to hemophilia (even in the cases of females) or to thrombopathy, which was discovered shortly before as the cause of what had previously been known as purpura hemorrhagica or Werlhof's disease. Von Willebrand also conducted hematological examinations on Hjördis and some of her family members. He recorded a normal or slightly reduced number of platelets and an undisturbed clot retraction, unlike Glanzmann's thrombasthenia. The bleeding time (Duke) was greatly prolonged, extending to more than 2 hours in some cases, while the clotting time was within the normal range. He concluded that the disease was either a new form of thrombopathy or a condition of the capillary endothelium. – hemorrhagic disorders – afibrinogenemia – bernard-soulier syndrome – disseminated intravascular coagulation – factor v deficiency – factor vii deficiency – factor x deficiency – factor xi deficiency – factor xii deficiency – factor xiii deficiency – hemophilia a – hemophilia b – hypoprothrombinemias – platelet storage pool deficiency – hermanski-pudlak syndrome – purpura, thrombocytopenic, idiopathic – thrombasthenia – thrombocythemia, hemorrhagic – vascular hemostatic disorders – cryoglobulinemia – ehlers-danlos syndrome – hemangioma, cavernous – hemangioma, cavernous, central nervous system – multiple myeloma – pseudoxanthoma elasticum – purpura, hyperglobulinemic – purpura, schoenlein-henoch – scurvy – shwartzman phenomenon – telangiectasia, hereditary hemorrhagic – waldenstrom macroglobulinemia – vitamin k deficiency – hemorrhagic disease of newborn – von willebrand disease – waterhouse-friderichsen syndrome Epistaxis, superficial and deep hematomas, hemarthrosis, gastrointestinal bleeding, hematuria represent the most frequent hemorrhagic events in congenital coagulation disorders. Occasionally, bleeding manifestations occur in unusual sites or are peculiar. A clotting defect may alter the clinical aspect of skin conditions or infections (hemorrhagic scabies or varicella). Hemobilia may occur as a complication of transjugular liver biopsy in hemophilia or Bernard-Soulier syndrome. Hemarthrosis of small joints of feet and hands occur in patients with hemophilia treated with protease inhibitors. Intramedullary hematomas of long bones have been described in α2-plasmin inhibitor or fibrinogen deficiencies. Spleen fracture with consequent hemoperitoneum has been reported in patients with fibrinogen deficiency. Rectus muscle sheath hematoma may occur in patients with factor VII (FVII)or FX deficiency. Acute or subacute intestinal obstruction may be caused by intramural wall hematomas in hemophilia and von Willebrand (vW)-disease. Physicians should always keep in mind that a congenital hemorrhagic disorder may cause bleeding in any tissue of the body and therefore alter the normal clinical features of a given disease. Factor X deficiency Factor XI deficiency Hemophilia A Hemophilia B Bernard-Soulier syndrome Platelet function defects Antiplatelet drug ingestion Fibrinolytic defects The objective of the present study was to determine the pattern of inherited bleeding disorders in southern Iran and evaluate the effect of a comprehensive coagulation laboratory and related efforts. A total of 545 patients with inherited bleeding disorders were evaluated during 1992-2007 by a cross-sectional study. Data were collected by a data-gathering form. Statistical analysis was done using Statistical Package for the Social Sciences version 15. A P value less than 0.05 was considered statistically significant. Overall 411 patients had common bleeding disorders including 326 hemophilia A, 46 hemophilia B, and 39 von Willebrand disease. Seventy-nine patients had rare coagulation disorders including deficiency of factor VII (n = 26), factor X (n = 18), factor XIII (n = 9), factor I (n = 9), factor XI (n = 7), factor V (n = 4), combined factor VIII and factor V (n = 4), and combined factor X and factor VII (n = 2). Fifty-five patients had platelet disorders including 23 with Glanzmann's thrombasthenia, 15 with Bernard-Soulier syndrome, and 17 with other platelet disorders, most of which (45) were diagnosed after the establishment of the comprehensive coagulation laboratory. Annual mean number of new diagnosed patients with common and rare bleeding disorders increased from 29 +/- 4 to 38 +/- 17. The ratio of the patients diagnosed with rare bleeding disorders to common bleeding disorders significantly increased after the establishment of the comprehensive diagnosis laboratory (P < 0.001).It seems that implementation of collaborative projects by the Shiraz Hemophilia Society and the establishment of the comprehensive coagulation laboratory and treatment centers have been successful in increasing diagnosis of the inherited bleeding disorders and consequently better management of the patients. – blood coagulation disorders – coagulation protein disorders – activated protein c resistance – afibrinogenemia – factor v deficiency – factor vii deficiency – factor x deficiency – factor xi deficiency – factor xii deficiency – factor xiii deficiency – hemophilia a – hemophilia b – hypoprothrombinemias – von willebrand disease – disseminated intravascular coagulation – blood coagulation disorders, inherited – activated protein c resistance – afibrinogenemia – antithrombin iii deficiency – bernard-soulier syndrome – factor v deficiency – factor vii deficiency – factor x deficiency – factor xi deficiency – factor xii deficiency – factor xiii deficiency – hemophilia a – hemophilia b – hermanski-pudlak syndrome – hypoprothrombinemias – protein c deficiency – thrombasthenia – von willebrand disease – wiskott-aldrich syndrome – platelet storage pool deficiency – hermanski-pudlak syndrome – protein s deficiency Diagnosis Basic tests performed in any patient with bleeding problems are a complete blood count-CBC (especially platelet counts), activated partial thromboplastin time-APTT, prothrombin time with International Normalized Ratio-PTINR, thrombin time-TT, and fibrinogen level. Patients with abnormal tests typically undergo further testing for hemophilias. Other coagulation factor assays may be performed depending on the results of a coagulation screen. Patients with von Willebrand disease typically display a normal prothrombin time and a variable prolongation of partial thromboplastin time. To determine the frequency of inherited bleeding disorders, its complications, and treatment modalities available for its treatment. Cross-sectional study. Patients with a history of bleeding tendency were tested for confirmation of the diagnosis. History and clinical findings were recorded. Laboratory analysis included prothrombin time (PT), activated partial thromboplastin time (APTT), bleeding time (BT), and fibrinogen assay. Patients with prolonged APTT were tested for factors VIII (FVIII) and IX (FIX). If FVIII was low, von Willebrand factor: antigen (vWF:Ag) and von Willebrand factor:ristocetin cofactor (vWF:RCo) were performed. When PT and APTT both were prolonged, FV, FX, and FII were tested. Platelet aggregation studies were done when there was isolated prolonged BT. Urea clot solubility test was done when all coagulation tests were normal. All patients with hemophilia A and B were evaluated for inhibitors. Of the 376 patients, inherited bleeding disorder was diagnosed in 318 (85%) cases. Median age of patients was 16.4 years. Hemophilia A was the commonest inherited bleeding disorder that was observed in 140 (37.2%) followed by vWD 68 (18.0%), platelet function disorders 48 (12.8%), and hemophilia B in 33 (8.8%) cases. We also found rare congenital factor deficiencies in 13 (3.4%), low VWF in 11 (3.0%) participants and 5 (1.3%) in female hemophilia carriers. Hemarthrosis was the most frequent symptom in hemophilia A and B (79.7%) involving knee joint. Inhibitor was detected in 21 (15%) cases. Fresh frozen plasma/cryoprecipitate were the most common modality of treatment. In 58 patients, no abnormality was detected in coagulation profile. Hemophilia A and vWD are the most common congenital bleeding disorders in this study. Hemarthrosis involving knee joint was the most common complication. Inhibitor was detected in a significant number of patients. Plasma is still the most common modality of treatment. Hemophilia is widely distributed all over the world, but little is known about its clinical profile in resource-limited regions. An insight into its clinical spectrum will help in the formulation of policies to improve the situation in these areas. To study the clinical profile of hemophiliacs (age <18 years) in Jodhpur region and screen them for transfusion-transmitted infections. A cross-sectional study conducted in the Department of Pediatrics, Umaid Hospital, Dr. S. N. Medical College, Jodhpur, over a period of 12 months. Out of a total of 56 cases enrolled, 51 (91%) cases were diagnosed as hemophilia A while 5 (9%) were diagnosed as hemophilia B. Positive family history was found in 26 (46%) cases. According to their factor levels, 25 (44%) cases had severe disease, 20 (36%) had moderate disease, and 11 (20%) had mild disease. The mean age of onset of symptoms and diagnosis was 1.73 ± 1.43 and 3.87 ± 3.84 years, respectively. First clinical presentation was posttraumatic bleed in 20 (36%), gum bleeds in 17 (30%), epistaxis in 4 (7%), joint bleeds in 4 (7%), skin bleeds in 4 (7%), and circumcision bleed in 3 (5%) cases. Knee joint was the predominant joint affected by hemarthrosis in 38 (68%), followed by ankle in 29 (52%), elbow in 20 (36%), and hip joint in 7 (13%) cases. All patients had a negative screening test for transfusion-transmitted infections. Occurrence of posttraumatic bleeds and gum bleeds in an otherwise normal child should warn the clinician for evaluation of hemophilia. Acquired hemophilia Ehlers-Danlos syndrome Factor XI deficiency Glanzmann thrombasthenia Haemophilia C Haemophilia type B Physical child abuse Platelet disorders Von Willebrand disease Normal values fall between 3 – 10 minutes depending on the method used. A disadvantage of Ivy's method is closure of puncture wound before stoppage of bleeding. Duke's method With the Duke;s method, the patient is pricked with a special needle or lancet, preferably on the earlobe or fingertip, after having been swabbed with alcohol. The prick is about 3–4 mm deep. The patient then wipes the blood every 30 seconds with a filter paper. The test ceases when bleeding ceases. The usual time is about 2–5 minutes. This method is not recommended and cannot be standardized because it can cause a large local hematoma. Interpretation Bleeding time is affected by platelet function, certain vascular disorders and von Willebrand Disease—not by other coagulation factors such as haemophilia. Diseases that cause prolonged bleeding time include thrombocytopenia, disseminated intravascular coagulation (DIC), Bernard-Soulier disease, and Glanzmann's thrombasthenia. Keeping an updated registry of bleeding disorders is crucial for planning care and documenting prevalence. We aimed to assess the prevalence of various bleeding disorders including rare inherited coagulation and platelet disorders concerning their clinico-epidemiological, diagnostic data and bleeding manifestations severity. Patients suffering from manifestations of bleeding or coagulation disorders presented to Hematology Clinic during 16 years were included and prospectively followed up. Demographics, clinical characteristics, complete blood count, bleeding, prothrombin and activated partial thromboplastin times, platelet aggregation tests and bone marrow aspiration were recorded. Overall 687 patients with bleeding disorders from total 2949 patients were identified. Inherited coagulation defects were found in 27.2%; hemophilia A (70.6%), hemophilia B (13.9%), factor I deficiency (2.3%), factor V deficiency (1.6%), factor X deficiency (4.2%), factor VII deficiency (2.6%), factor XIII deficiency (1.1%), combined factor deficiency (2.1%) and unclassified coagulation disorders in 1.6% of studied patients. Overall 72.7% had diagnosed with platelet disorders; immune thrombocytopenia was the commonest (74.8%), and inherited conditions represent (25.2%) in the following order: Glanzman's thrombasthenia (11.2%), von Willebrand disease (6.6%), Bernard-Soulier syndrome (1%) and Chediak Higashi in 0.4% and unclassified in 6%. Median age of diagnosis of coagulation and platelet disorders were 33 and 72 months. Presenting symptoms of coagulation disorders were: 25.1% post circumcision bleeding, 22.5% ecchymosis, 20.9% hemoarthrosis and 15% epistaxis. Symptoms of rare coagulation disorders were postcircumcision bleeding (20%), bleeding umbilical stump (20%), epistaxis (12%), hemoarthrosis (8%) and hematomas (4%). Presenting symptoms in rare inherited platelet disorders were purpura, ecchymosis, epistaxis and bleeding gums, respectively. Analysis of the clinico-epidemiological data of patients with bleeding disorders is a useful tool for monitoring and improving their quality of care. Causes other than coagulation Bleeding diathesis may also be caused by impaired wound healing (as in scurvy), or by thinning of the skin, such as in Cushing's syndrome. Genetic Some people lack genes that typically produce the protein coagulation factors that allow normal clotting. Various types of hemophilia and von Willebrand disease are the major genetic disorders associated with coagulopathy. Rare examples are Bernard–Soulier syndrome, Wiskott–Aldrich syndrome and Glanzmann's thrombasthenia. Gene therapy treatments may be a solution as they involve in the insertion of normal genes to replace defective genes causing for the genetic disorder. Gene therapy is a source of active research that hold promise for the future. Diagnosis Comparing coagulation tests Other conditions can also present similarly with bleeding after minor trauma or spontaneous bleeds and require exclusion before confirming the diagnosis of hemophilia. Some of these conditions include von Willebrand disease, scurvy, diseases of platelet dysfunction, deficiency of other coagulation factors like V, VII, X, or fibrinogen, Ehlers-Danlos syndrome, Fabry disease, disseminated intravascular coagulation, and child abuse. In von Willebrand disease, bleeding symptoms can be similar to mild hemophilia, but patients with von Willebrand disease have more mucosal bleeding compared to musculoskeletal bleeding seen in hemophilia. Von Willebrand disease is diagnosed by checking for von Willebrand factor antigen or von Willebrand factor multimers. [40] Similarly, in scurvy, Ehlers-Danlos syndrome, and Fabry disease; also, the bleeding is usually mucosal, unlike hemophilia, where it is musculoskeletal. In scurvy, there is a deficiency of vitamin C. [41] In Ehlers-Danlos syndrome, the skin is hyperextensible, and joints are hypermobile. The diagnosis is usually through clinical features, genetic testing, and tissue biopsy. [42] Similarly, in Fabry disease, patients may also have other organs being affected, including kidneys and heart, and have skin lesions called angiokeratomas. They also have pain in the extremities. Fabry disease is usually diagnosed with clinical findings and genetic testing. [43] In cases of platelet dysfunction disorders, bleeding is usually mucocutaneous, unlike hemophilia. Usually, these disorders are diagnosed by platelet aggregation studies or platelet electron microscopy. [44] In the deficiency of other coagulation factors, musculoskeletal bleeding is uncommon. In fact, sometimes thrombosis can occur, especially in patients with factor VII or fibrinogen deficiency or in patients with combined factor V and VIII deficiency. Specific coagulation factor assays usually confirm the diagnosis. Disseminated intravascular coagulation (DIC) that mimics hemophilia is hard to differentiate, but usually, there is an underlying condition in DIC, for example, acute promyelocytic leukemia. Diagnosis is usually carried out by blood tests that show decreased platelet count and the absence of factor VIII autoantibodies. Child abuse can sometimes be misidentified and confused with hemophilia, and it is essential to find inconsistencies in the history of how trauma has occurred. Other signs of malnourishment require vigilance, and x-rays may reveal evidence of fractures of different ages. [45] [46] – anemia, hypoplastic, congenital – anemia, Diamond–Blackfan – fanconi anemia – ataxia telangiectasia – blood coagulation disorders, inherited – activated protein C resistance – afibrinogenemia – antithrombin III deficiency – Bernard–Soulier syndrome – factor V deficiency – factor VII deficiency – factor X deficiency – factor XI deficiency – factor XII deficiency – factor XIII deficiency – hemophilia A – hemophilia B – Hermansky–Pudlak syndrome – hypoprothrombinemias – protein C deficiency – thrombasthenia – Von Willebrand disease – Wiskott–Aldrich syndrome – CADASIL – cardiomyopathy, hypertrophic, familial – cherubism The International Society on Thrombosis and Haemostasis Bleeding Assessment Tool (ISTH-BAT) is useful for assessing bleeding disorders. Its utility was tested in a small study, including patients with known inherited platelet disorders. The study demonstrated a specificity of 100%, a positive predictive value of 90%, and a negative predictive value of 100% using this assessment tool. [17] In the setting of von Willebrand disease, a BAT score >6 repeatedly correlates to a 99% probability of an inherited platelet defect such as BSS. [8] ] Similarly, other bleeding assessment tools like Molecular and Clinical Markers for the Diagnosis and Management (MCMDM) of type 1 von Willebrand's disease (VWD) and the World Health Organization Bleeding Assessment Tool are similar tools. An electronic version of MCMDM-type-1 VWD was developed in 2010. [17] Hemophilia is a sex-linked recessive genetic disorder characterized by the absence or deficiency of factor VIII (hemophilia A, orclassic hemophilia) or factor IX (hemophilia B, or Christmas disease) (Chap. 141). Hemophilia A constitutes 85% of cases. Spontaneous hemarthrosis is a common problem with both types of hemophilia and can lead to a deforming arthritis. The frequency and severity of hemarthrosis are related to the degree of clotting factor deficiency. Hemarthrosis is not common in other disorders of coagulation such as von Willebrand disease, factor V deficiency, warfarin therapy, or thrombocytopenia. Hemarthrosis occurs after 1 year of age, when a child begins to walk 2241 and run. In order of frequency, the joints most commonly affected are theknees,ankles,elbows,shoulders,andhips.Smalljointsofthehands and feet are occasionally involved. The authors report a case of hemarthrosis complicated by severe anemia related to a congenital connective tissue disease: Ehlers-Danlos syndrome. A boy fell down and suffered tumefaction of both knees with bilateral rupture of the rotula tendon. He underwent surgical reinsertion of each tendon on the rotula. He later showed an unexpected ongoing hematic effusion, with severe anemia. He was screened for coagulation disorders with no results. On taking a more detailed history and investigating the patient's phenotypical features, the authors diagnosed Ehlers-Danlos syndrome, hypermobile variant. The hemarthrosis and anemia were thus concluded to be consequences of excessive tissue fragility due to a congenital connective tissue disease.\nHere is the question:\nPEDIATRICS: An 18-month-old boy, with complete immunization schedule to date, who consults the Emergency Department for right knee swelling after playing in the park, without obvious trauma. In the directed anamnesis, the mother refers that an uncle of hers had similar problems. The ultrasound examination is compatible with hemarthrosis and in the analytical analysis only an APTT lengthening of 52'' (normal 25-35'') stands out. What is the most probable diagnostic hypothesis?\nHere are the potential choices:\n1. Marfan syndrome.\n2. Von-Willebrand's disease.\n3. Ehlers-Danlos disease.\n4. Hemophilia A.\n5. Bernard-Soulier disease.\nThe correct answer is: 4. Hemophilia A." }, { "question": "You are a helpful medical expert, and your task is to answer a multi-choice medical question using the relevant documents. Please choose the answer from the provided options. Your responses will be used for research purposes only, so please have a definite answer.\nHere are the relevant documents:\n The aim of this study is to evaluate the efficiency of a treatment prescribed, in the course of an hospital consultation for sleep pathology, to patients suffering from chronic insomnia not improved by longstanding and sustained medication with hypnotic drugs. The basis of the treatment is a progressive but total withdrawal of hypnotics in so far taken regularly. The withdrawal of hypnotics was prescribed to 79 patients: 33 aged 17 to 39 years (group 1, mean age 30) and 46 aged 40 to 70 years (group 2, mean age 51). 41 showed primary psychophysiological insomnia and 28 showed insomnia associated with psychiatric disorders. In patients of group 1, the average durations were 8 years for insomnia and 3 years for sustained hypnotic use; these durations were 15 and 5 years respectively in patients of group 2. Hypnotic drug withdrawal was achieved without placebos in 3 months in group 1 patients and 5 months in group 2 patients. 65 patients completely stopped the continual use of hypnotics. Subjective improvement of insomnia was reported by 51 of these patients (as well as by 6 patients who were given simultaneous antidepressant therapy). 16 of the 51 improved patients have resorted to hypnotics occasionally (at intervals of 10 days or more). After complete withdrawal, patients went on consulting for various lengths of time: 5 months average for group 1, 14 months average for group 2. This study of a fairly large group of insomniacs shows the frequent ineffectiveness of a sustained use of currently available hypnotics. It also shows that two times out of three the complete stop of sustained hypnotic medication proved beneficial to the patient. To carry out an observational epidemiological survey (Studio Morfeo 2) in order to define the management procedures of insomnia in a large Italian population presenting directly to the general practitioner (GP). Each GP recruited five insomniac subjects in the course of 1 week or 5 consecutive office days over a period of 2 weeks. On each office day, a brief questionnaire (Q1) including five questions investigating insomnia symptoms and current use of treatment was administered to the first 10 patients who referred to the GP office for reasons associated with their own health. The first patient of each day classified as insomniac underwent a second investigation based on a more detailed questionnaire (Q2) including demographic variables, socio-economic status, general medical conditions, severity, duration and clinical features of insomnia, daytime dysfunction, sleep satisfaction and therapeutic management. In a primary care setting, insomnia symptoms are often persistent (>1 year), recurrent (>1/week) and accompanied by daytime consequences. Two out of three patients with insomnia symptoms are dissatisfied with their sleep. In most cases, insomnia symptoms are underrated both by the patients, who cover the problem or reject treatment, and by the GP, who limits intervention on the sleep disorder (scarcely modifying ongoing therapy both in responders and in non-responders). In responders, treatment was confirmed in 91% of cases and discontinued in only 2%. When there was no improvement, or if insomnia symptoms became worse (non-responders), treatment was nevertheless continued in 74.5% of cases, either maintaining the same ineffective dose, increasing the dose, or adding another drug or a non-pharmacological procedure. Regardless of specific medication, the Italian GP privileges the pharmacological approach, which is fourfold more frequent than non-pharmacological therapy (78.6 versus 18.2%). Non-benzodiazepine hypnotic drugs are mostly prescribed when the GP decides to apply medication in previously untreated patients with insomnia symptoms. Self-administration is not unusual among the patients with insomnia symptoms and is more common among non-responders. Italian GPs tend to confirm the ongoing therapy and avoid re-evaluation of the treatment regimen. Limited use of non-pharmacological treatment in the Italian primary care setting is in line with this conservative approach of the GPs who tend to be problem-solvers rather than problem-seekers. Insomnia is the most prevalent sleep disorder in the general population, and one of the most frequent reasons for consultation in the Sleep Units. Perampanel is an antiepileptic also effective on the structure of sleep, and in restless legs syndrome. We describe the first study that evaluates perampanel in patients with chronic insomnia. Observational retrospective cohorts study of 66 patients with chronic resistant insomnia, 33 exposed to perampanel, other 33 as non-exposed group. All patients attended in Neurology or Psychiatry Consultation, from November 2017 to November 2018. Patients included had been treated with more than 4 different drugs in the previous 4 years. We reviewed age, sex, insomnia etiology, years of evolution, number of previously used drugs, and the results of perampanel for insomnia after 3 months of treatment in the exposed cohort, measured by the improvement of 3 or more points in the ISI and Pittsburgh scales, as well as the average of hours of sleep gained. Non-exposed patients were matched with this variables, but never treated with perampanel. We have included 66 patients. In the exposed cohort: we describe 33 patients with chronic resistant insomnia, 20 women (60 %), 13 men (40 %). Average age 53.48 years, average time of evolution: 11.25 years. Main etiology: depression 13 cases (40 %). After the combination of perampanel 2-4 mg (100 %) with antidepressants (17 cases, 51.5 %) or anxiolytics (12 cases, 36.36 %) along 3 months: the total number of hours of sleep improves in 2.5 h, the scale ISI improves by 6 points (± 2.1 SD, p = 0.02), and Pittsburgh scale improves in 4 points (± 1.7, p = 0.04). In non-exposed cohort, the improvement of the ISI scale was 2.2 points (±0.8, p = 0.06), on the Pittsburgh scale was 1.6 points (± 0.5, p = 0.01). The main adverse effect was irritability in 3 patients, without withdrawal perampanel. The treatment was abandoned by 4 patients (12.12%): 1 due to persistent irritability (3%), 2 due to lack of efficacy (6 %), 1 due to pregnancy wish (3 %). The combination of Perampanel with an antidepressant, or an anxiolytic, improves the quality of sleep measured by ISI and Pittsburgh scales (statistically significant), probably due to its antagonistic action on glutamate. A clinical trial compared with placebo would be necessary to corroborate these results. Tolerance Chronic use of benzodiazepines, such as chlordiazepoxide, leads to the development of tolerance, with a decrease in number of benzodiazepine binding sites in mouse forebrain. The Committee of Review of Medicines, who carried out an extensive review of benzodiazepines including chlordiazepoxide, found—and were in agreement with the Institute of Medicine (USA) and the conclusions of a study carried out by the White House Office of Drug Policy and the National Institute on Drug Abuse (USA)—that there was little evidence that long-term use of benzodiazepines were beneficial in the treatment of insomnia due to the development of tolerance. Benzodiazepines tended to lose their sleep-promoting properties within 3–14 days of continuous use, and in the treatment of anxiety the committee found that there was little convincing evidence that benzodiazepines retained efficacy in the treatment of anxiety after 4 months' continuous use due to the development of tolerance. Insomnia Barbiturates were introduced as hypnotics for patients with schizophrenia. It induced a state of deep and prolonged sleep. But this was not used for long because of adverse side effects. Anticonvulsant Phenobarbital was the first truly effective drug against epilepsy. It was discovered by accident when given to epileptic patients to help them sleep. The positive side effects were anticonvulsant properties that reduced seizure number and intensity. Sedation Pentobarbital is used as a hypnotic when analgesia is not required. It´s often used in CT imaging when sedation is needed. It is efficient, safe and the recovery time is short. In 2013 the barbiturates phenobarbital and butabarbital are still used as sedatives in certain cases as well as to antagonize the effects of drugs as ephedrine and theophylline. Phenobarbital is used in cases of drug withdrawal syndromes. It is used as normal and emergency treatment in some cases of epilepsy. Generally, these treatments are given after the behavioral treatment has failed. Drugs such as tranquilizers, though they may work well in treating insomnia, have a risk of abuse which is why these treatments are not the first resort. Some sleep disorders such as narcolepsy do require pharmacological treatment. See also Sleep disorder Sleep medicine Snoring References External links Sleep disorders sv:Somnologi Pharmacological treatments Pharmacological treatments are used to chemically treat sleep disturbances such as insomnia or excessive daytime sleepiness. The kinds of drugs used to treat sleep disorders include: anticonvulsants, anti-narcoleptics, anti-Parkinsonian drugs, benzodiazepines, non-benzodiazepine hypnotics, and opiates as well as the hormone melatonin and melatonin receptor stimulators. Anticonvulsants, opiates, and anti-Parkinsonian drugs are often used to treat restless legs syndrome. Furthermore, melatonin, benzodiazepines hypnotics, and non-benzodiazepine hypnotics may be used to treat insomnia. Finally, anti-narcoleptics help treat narcolepsy and excessive daytime sleepiness. Of particular interest are the benzodiazepine drugs which reduce insomnia by increasing the efficiency of GABA. GABA decreases the excitability of neurons by increasing the firing threshold. Benzodiazepine causes the GABA receptor to better bind to GABA, allowing the medication to induce sleep. In India, clobazam is approved for use as an adjunctive therapy in epilepsy, and in acute and chronic anxiety. In Japan, clobazam is approved for adjunctive therapy in treatment-resistant epilepsy featuring complex partial seizures. In New Zealand, clobazam is marketed as Frisium In the United Kingdom clobazam (Frisium) is approved for short-term (2–4 weeks) relief of acute anxiety in patients who have not responded to other drugs, with or without insomnia and without uncontrolled clinical depression. It was not approved in the United States until October 25, 2011, when it was approved for the adjunctive treatment of seizures associated with Lennox-Gastaut syndrome in patients 2 years of age or older. Contraindications Clobazam should be used with great care in patients with the following disorders: Myasthenia gravis. Sleep apnea. Severe liver diseases such as cirrhosis and hepatitis. Severe respiratory failure. Good sleep is an important index of the quality of life in people and above all in old subjects. Among all the symptoms reported to general practitioner, insomnia is at the 3(rd) place and this is present in particular in the elderly. In elderly people high comorbidity and polytreatment are often present. We have studied 60 elderly people with history of insomnia and concomitant diseases: depression, dementia and behavioral disturbances. All the patients of the present study were visited in our outpatients' department. Three hypnotic drugs were used for the treatment of insomnia: zolpidem, or triazolam, or oxazepam, respectively at doses of 10mg/day, 0.125-0.25mg/day and 15.0mg/day. All the three drugs showed to be effective and safe; no paradoxical effects were observed. Sam is a 6-year-old boy with a diagnosis of autism spectrum disorder (ASD) who recently relocated and has an appointment with you, his new pediatric clinician, to establish care. He was previously followed by a psychiatrist for 2 years for additional diagnoses of insomnia, bipolar disorder, anxiety, attention deficit hyperactivity disorder, and intellectual disability. He has tried and (apparently) failed multiple psychotropic trials including stimulants, nonstimulants, mood stabilizers, atypical antipsychotics, and nonbenzodiazepine hypnotics. He has a delayed sleep onset and frequent night awakenings each night for the past 3 months, during which he \"screams, cries, and thrashes and can stay up for over an hour.\" His behaviors are described as irritable, self-injurious, and aggressive with no clear pattern of triggers according to his mother. He is nonverbal and communicates by leading and rarely pointing. The patient's current medication regimen includes clonidine 0.2 mg at night, lorazepam 1.5 mg as needed at night, olanzapine 5 mg twice daily, and diphenhydramine as needed for sleep/agitation. His mother is concerned that he is developing \"tolerance\" to the regimen and wants to wean him off some of the medications. His mother is struggling to take care of the patient given his worsening behavior and body habitus (body mass index >99%; z = 3.41).There is a family history of depression, anxiety, bipolar disorder, and autism. He has a 3-year-old sister, who is also diagnosed with ASD, though she is not as severely impacted. His mother's partner recently moved in along with 2 children of his own, aged 3 and 4 years. Sam attends a specialized school, where he receives behavior therapy and occupational therapy. He has undergone inpatient pediatric hospitalization twice, 1 time for 3 weeks and the other for 6 days, for aggressive behavior, and in both instances, he was discharged before inpatient psychiatric placement because of a lack of available beds.After urgent consultation with your local developmental and behavioral pediatrician, a slight reduction was made in the lorazepam because of concerns about tolerance and side effects. However, within a week of this, he was brought to the emergency department for continued self-injurious behavior and increased trouble with sleeping. His mother voiced concerns about his safety in the home, which were particularly related to aggression toward his younger sister. He was admitted to the pediatric inpatient floor for observation, and medication adjustment (increasing olanzapine), which was initially helpful in improving behavior, but mostly behavioral/environmental strategies were used to soothe him, including frequent wagon rides through the hospital corridors.Despite the patient being stable from the medical standpoint, Sam's mother did not feel comfortable taking him home. Social work contacted local community mental health services to pursue outpatient resources and respite care options and sought inpatient pediatric psychiatry. After several failed attempts to find placement, he remained in pediatric inpatient care for 1 and a half months with no acute medical interventions other than his oral medications.He was finally accepted to the in-state pediatric psychiatric facility when a bed was available. During his week-long stay, he had further medication adjustments with a decrease in olanzapine and optimization of his clonidine dose. During his psychiatric hospital stay, care coordination succeeded in arranging center-based applied behavior analysis interventions and respite care and parent training for his family. Sam began to show improvement in his overall agitation and aggression, requiring less clonazepam, and his mother then maintained outpatient follow-up.The day before discharge, you visit him in the hospital, and a medical student asks you why he was in the hospital for so long. How would you answer the question? The management of sleep disorders in elderly patients with internal diseases consists in the first line in rectifying pathophysiological disregulations. Only in the second line, proper hypnotics are to be prescribed. When considered as indispensable, these medicaments are selected according to their toxicity and side effects. In present time, Benzodiazepines are definitely preferred, whereas neuroleptic, anti-depressant and the older drugs are to be taken secondarly in account. We report on the case of a 45-year old female with chronic insomnia and refractory to hypnotics, who also has a - polygraphically documented - tolerance to the imidazopyridine \"zolpidem\". We discuss the main differential diagnosis and demonstrate a therapeutic regimen which allows a step-by-step replacement of hypnotics by sedative antidepressants. This interval replacement treatment reduces on the one hand the risk of developing a severe withdrawal syndrome. On the other hand the replacement by sedative antidepressants improves insomnia and insomnia-associated depressive symptoms. Finally, the clinical implications and rationale of a therapeutic approach with sedative antidepressants in chronic insomnia are discussed. Chlordiazepoxide, trade name Librium among others, is a sedative and hypnotic medication of the benzodiazepine class; it is used to treat anxiety, insomnia and symptoms of withdrawal from alcohol and other drugs. Chlordiazepoxide has a medium to long half-life but its active metabolite has a very long half-life. The drug has amnesic, anticonvulsant, anxiolytic, hypnotic, sedative and skeletal muscle relaxant properties. Chlordiazepoxide was patented in 1958 and approved for medical use in 1960. It was the first benzodiazepine to be synthesized and the discovery of chlordiazepoxide was by pure chance. Chlordiazepoxide and other benzodiazepines were initially accepted with widespread public approval but were followed with widespread public disapproval and recommendations for more restrictive medical guidelines for its use. Psychiatric patients often have residual intractable insomnia as a serious problem. Forty-eight psychiatrically ill patients (DSM-IV diagnoses) who had failed to respond to medicinal treatment for chronic insomnia were referred for and completed behavioral therapy as an adjunct to the pharmacologic treatment of their insomnia. The behavioral treatments included structured sleep hygiene, progressive muscle relaxation, stimulus control, and sleep restriction. The treatment program was accomplished in 6 sessions over 2 months. Follow-up evaluations were completed at 2, 6, and 12 months from the beginning of the treatment program. The outcome of the treatment program was evaluated in terms of the change in (1) self-reported specific sleep parameters, (2) self-ratings of sleep-related day-time state, (3) self-rating of quality of sleep, (4) the use of sleep medication, and (5) the therapist's global rating of improvement. There was a statistically significant change from the baseline in all self-reported specific sleep parameters after 2 months that was sustained after 6 and 12 months. Sleep-related characteristics of daytime state showed statistically significant changes after 2 and 6 months that were maintained after 12 months. Sleep quality had a statistically significant change after 2 months, continued to improve statistically after 6 months, and was maximum after 12 months. Over half the patients (52.7%; 20 of 38) either reduced their sleep medication by half or stopped it completely. The therapist's global rating showed an improvement in 29.2% (N = 14) of patients after 2 months, 56.2% (N = 27) after 6 months, and 68.7% (N = 33) after 12 months. The use of concomitant behavioral and pharmacologic treatment of chronic insomnia in psychiatrically ill patients results in improving sleep and sleep-related state and reduces the risk of return of insomnia for 10 months after finishing active treatment. Clinical trials conducted in general practice are more especially interesting as they enable to test a drug in the real conditions of use. On the other hand, these trials are beneficial to the G.P. (new image, rupture of his loneliness, change in his prescription habits, contact with hospitals). The methodology, as for hospital studies, must be rigorous. As a matter of fact, these two types of studies are additional and the cooperation between the G.P.'s and the pharmaceutical industry can conduct to the solution of specific problems: drug interaction--long-term therapeutic effect--new indications. This double blind cross over study comparing triazolam and nitrazepam conducted by G.P.'s on insomniacs is the first French clinical study intended for the registration application and done according to this methodology. This work is an exemple of the new opportunity offered to G.P.'s in future. The results have shown that: on 54 patients (23 male and 31 female) of an average age: 48, 32 have preferred triazolam, 13 have preferred nitrazepam, 9 have had no preference. Contraindications and interactions Cabotegravir/rilpivirine must not be combined with drugs that induce the liver enzyme CYP3A4, because they accelerate the inactivation of rilpivirine, and/or the enzmye UGT1A1, because they accelerate the inactivation of cabotegravir. These mechanisms potentially result in loss of effectiveness. Examples for such drugs are rifampicin, rifapentine, carbamazepine, oxcarbazepine, phenytoin and phenobarbital. Adverse effects The most common side effects of the injectable combination therapy with rilpivirine are reactions at the injection site (in up to 84% of patients) such as pain and swelling, as well as headache (up to 12%) and fever or feeling hot (in 10%). Less common side effects (under 10%) are depressive disorders, insomnia, rashes, fatigue, musculoskeletal pain, nausea, sleep disorders, and dizziness. Pharmacology Insomnia, a more or less chronic sleep disturbance, is a very common symptom in psychiatric patients but also relatively freguent in the general population to a lesser degree. Two broad types of insomnia may often be distinguished: (1) difficulty falling asleep and frequent wakening, characteristic of anxiety states or obsessive worrying; and (2) early morning wakening, sometimes in a panic, suggestive of endogenous depression. The first group of patients respond well to minor tranquilizers and psychotherapy, whereas the second do well with tricyclic anti-depressants. Many studies in sleep laboratories have declineated the stages and cycles of sleep physiology and pathology, especially the importance of REM or dreaming sleep. The clinician should be cautious in the use of hypnotics like barbiturates which suppress REM sleep and produce a rebound increase on withdrawal, as well as problems of dependence of habituation. Flurazepam and chloral hydrate are considerably safer in this respect. Understanding sleep neurophysiology and biochemistry permits appropriate individual clinical management for both psychiatric patients and medical patients with conditions like peptic ulcer and nocturnal angina pectoris. for anesthetic purposes, and are also sometimes prescribed for anxiety or insomnia. This is not a common practice anymore, however, owing to the dangers of long-term use of barbiturates; they have been replaced by the benzodiazepines and Z-drugs such as zolpidem, zaleplon and eszopiclone for sleep. The final class of barbiturates are known as long-acting barbiturates (the most notable one being phenobarbital, which has a half-life of roughly 92 hours). This class of barbiturates is used almost exclusively as anticonvulsants, although on rare occasions they are prescribed for daytime sedation. Barbiturates in this class are not used for insomnia, because, owing to their extremely long half-life, patients would awake with a residual \"hang-over\" effect and feel groggy. The impossibility of treating patients with sleep disorders adequately means that, as specialists, we have to look for new pharmacological treatments and for this reason we examined the information in the paper by Salin Pascual (1999) about the increase in deep sleep when olanzapine is used as an antipsychotic drug. We decided to use this medication in six females and three males who were suffering from different sleep disorders that conditioned their chronic insomnia. The dosages of olanzapine used ranged from 2.5 and 10 mg in a single dose. The clinical history and progress were used to elaborate the results and conclusions. The result was positive in eight of the nine patients, five who were administered the medication as monotherapy and three as polytherapy. The population studied is insufficient to prove the effectiveness of the drug, but the fact that in eight of our patients the treatment clearly improved their symptoms leads us to think that this line of research must be continued. Non medication based strategies provide long lasting improvements to insomnia and are recommended as a first line and long-term strategy of management. Behavioral sleep medicine (BSM) tries to address insomnia with non-pharmacological treatments. The BSM strategies used to address chronic insomnia include attention to sleep hygiene, stimulus control, behavioral interventions, sleep-restriction therapy, paradoxical intention, patient education, and relaxation therapy. Some examples are keeping a journal, restricting the time spent awake in bed, practicing relaxation techniques, and maintaining a regular sleep schedule and a wake-up time. Behavioral therapy can assist a patient in developing new sleep behaviors to improve sleep quality and consolidation. Behavioral therapy may include, learning healthy sleep habits to promote sleep relaxation, undergoing light therapy to help with worry-reduction strategies and regulating the circadian clock. An enquiry into the handling by medical practitioners of sleeping problems among elderly patients was conducted in southern Lower Saxony by personal interview, combined with a standard questionnaire. A typical case report had been drafted concerning a 70-year-old, previously healthy widow: her complaints were \"nonspecific\" and could be classified as an example of either depression, of the onset of senile dementia or as within normal limits for age. This case report was presented by two interviewers to 145 general practitioners (GPs) and 14 neurologists in private practice (response rate of 83.2%) who were asked how they would have treated the patient's sleeping disorder. 30.3% of the GPs and 14.3% of the neurologists would initially not have prescribed medication. Only GPs (19.5%) mentioned possible herbal medication. Sedative neuroleptics were preferred by 57.1% of neurologists and 26.2% of GPs, while benzodiazepines would have been given by 14% of both groups. Antidepressive drugs and chloral hydrate were chosen less often (5.7% and 2.5%, respectively). These data support the finding of a high frequency of neuroleptic prescriptions given to the elderly. They also make clear that the possibility of treatment without drugs is usually not sufficiently explored. There is support showing positive sleep outcomes for people who follow more than one sleep hygiene recommendation. There is however no evidence that poor sleep hygiene can contribute to insomnia. While there is inconclusive evidence that sleep hygiene alone is effective as a treatment for insomnia, some research studies have shown improvement in insomnia for patients who receive sleep hygiene education in combination with cognitive behavioral therapy practices. Benzodiazepines and related drugs are the hypnotics of first choice. They shorten sleep latency, enhance sleep continuity and may prolong sleep duration. Their undesired effects include a persistent day-time sedation and ataxia when getting up at night. There is some risk of habit formation and dependence. For treating an acute insomnia, the prescription of hypnotics should be limited to a short duration (smallest package size), for treating chronic forms of insomnia they should have only an adjuvant role in therapy. Patients in general practice complaining of insomnia of recent origin have been studied in order to ascertain which factors may be of value in the detection of those more susceptible to drug dependence. The type of sleep disturbance was found to be of importance and a personal disturbance scale was found useful as a screening test in two-thirds of the patients. No difference was found in the development of dependence on amylobarbitone and nitrazepam. One of the most important factors in the prevention of drug dependence seems to be frequent review by the doctor after the first prescription and his cautionary advice to the patient. Antipsychotics Chlorpromazine, an antipsychotic and antiemetic drug which is classed as a \"major\" tranquilizer, may cause paradoxical effects such as agitation, excitement, insomnia, bizarre dreams, aggravation of psychotic symptoms and toxic confusional states. Barbiturates Phenobarbital can cause hyperactivity in children. This may follow after a small dose of 20 mg, on condition of no phenobarbital administered in previous days. Prerequisity for this reaction is a continued sense of tension. The mechanism of action is not known, but it may be started by the anxiolytic action of the phenobarbital. Barbiturates such as pentobarbital have been shown to cause paradoxical hyperactivity in an estimated 1% of children, who display symptoms similar to the hyperactive-impulsive subtype of attention deficit hyperactivity disorder. Intravenous caffeine administration can return these patients' behaviour to baseline levels. Psychological and behavioral therapies should be considered the first line treatment for chronic insomnia. Although cognitive behavioral therapy for insomnia (CBT-I) is considered the standard of care [1], several monotherapies, including sleep restriction therapy, stimulus control therapy, and relaxation training are also recommended in the treatment of chronic insomnia [2]. CBT-I is a multimodal intervention comprised of a combination of behavioral (eg, sleep restriction, stimulus control) and cognitive therapy strategies, and psychoeducation delivered in 4 to 10 weekly or biweekly sessions [3]. Given that insomnia is thought to be maintained by an interaction between unhelpful sleep-related beliefs and behaviors, the goal of CBT-I is to modify the maladaptive cognitions (eg, worry about the consequences of poor sleep), behaviors (eg, extended time in bed), and arousal (ie, physiological and mental hyperarousal) perpetuating the insomnia. CBT-I is efficacious when implemented alone or in combination with a pharmacologic agent. However, because of the potential for relapse upon discontinuation, CBT-I should be extended throughout drug tapering [4]. Although the treatment options should be guided by the available evidence supporting both psychological therapies and short-term hypnotic treatment, as well as treatment feasibility and availability, treatment selection should ultimately be guided by patient preference [5]. Despite its widespread use among treatment providers [6], the use of sleep hygiene education as a primary intervention for insomnia should be avoided. Sleep hygiene may be a necessary, but insufficient condition for promoting good sleep and should be considered an adjunct to another empirically supported treatment. The first-line drugs for treatment of status epilepticus are benzodiazepines, such as lorazepam or diazepam. If these fail, then phenytoin may be used, with phenobarbital being an alternative in the US, but used only third-line in the UK. Failing that, the only treatment is anaesthesia in intensive care. The World Health Organization (WHO) gives phenobarbital a first-line recommendation in the developing world and it is commonly used there. Phenobarbital is the first-line choice for the treatment of neonatal seizures. Concerns that neonatal seizures in themselves could be harmful make most physicians treat them aggressively. No reliable evidence, though, supports this approach. Phenobarbital is sometimes used for alcohol detoxification and benzodiazepine detoxification for its sedative and anti-convulsant properties. The benzodiazepines chlordiazepoxide (Librium) and oxazepam (Serax) have largely replaced phenobarbital for detoxification. Insomnia is a symptom that should be treated according to the underlying etiology. It is more common in elderly individuals and in women. Common causes of insomnia include acute situational factors, psychiatric disorders, use of various medications and illicit drugs, and medical disorders that cause pain, dyspnea or nausea. Pharmacotherapy should be generally restricted to use of the benzodiazepines, imidazopyridines (zolpidem) and occasionally tricyclic antidepressants. As a rule, hypnotic drugs should be used for less than two weeks to one month.\nHere is the question:\nPSYCHIATRY: We are consulted by an 84-year-old woman for insomnia of conciliation. After failing sleep hygiene measures, it is decided to initiate pharmacological treatment. Which of the following drugs would you select for the patient?\nHere are the potential choices:\n1. Diacepam.\n2. Lormetacepam.\n3. Phenobarbital.\n4. Chlordiazepoxide.\n5. Chloracepate.\nThe correct answer is: ", "gold_answer": "2 Lormetacepam.", "prediction": "You are a helpful medical expert, and your task is to answer a multi-choice medical question using the relevant documents. Please choose the answer from the provided options. Your responses will be used for research purposes only, so please have a definite answer.\nHere are the relevant documents:\n The aim of this study is to evaluate the efficiency of a treatment prescribed, in the course of an hospital consultation for sleep pathology, to patients suffering from chronic insomnia not improved by longstanding and sustained medication with hypnotic drugs. The basis of the treatment is a progressive but total withdrawal of hypnotics in so far taken regularly. The withdrawal of hypnotics was prescribed to 79 patients: 33 aged 17 to 39 years (group 1, mean age 30) and 46 aged 40 to 70 years (group 2, mean age 51). 41 showed primary psychophysiological insomnia and 28 showed insomnia associated with psychiatric disorders. In patients of group 1, the average durations were 8 years for insomnia and 3 years for sustained hypnotic use; these durations were 15 and 5 years respectively in patients of group 2. Hypnotic drug withdrawal was achieved without placebos in 3 months in group 1 patients and 5 months in group 2 patients. 65 patients completely stopped the continual use of hypnotics. Subjective improvement of insomnia was reported by 51 of these patients (as well as by 6 patients who were given simultaneous antidepressant therapy). 16 of the 51 improved patients have resorted to hypnotics occasionally (at intervals of 10 days or more). After complete withdrawal, patients went on consulting for various lengths of time: 5 months average for group 1, 14 months average for group 2. This study of a fairly large group of insomniacs shows the frequent ineffectiveness of a sustained use of currently available hypnotics. It also shows that two times out of three the complete stop of sustained hypnotic medication proved beneficial to the patient. To carry out an observational epidemiological survey (Studio Morfeo 2) in order to define the management procedures of insomnia in a large Italian population presenting directly to the general practitioner (GP). Each GP recruited five insomniac subjects in the course of 1 week or 5 consecutive office days over a period of 2 weeks. On each office day, a brief questionnaire (Q1) including five questions investigating insomnia symptoms and current use of treatment was administered to the first 10 patients who referred to the GP office for reasons associated with their own health. The first patient of each day classified as insomniac underwent a second investigation based on a more detailed questionnaire (Q2) including demographic variables, socio-economic status, general medical conditions, severity, duration and clinical features of insomnia, daytime dysfunction, sleep satisfaction and therapeutic management. In a primary care setting, insomnia symptoms are often persistent (>1 year), recurrent (>1/week) and accompanied by daytime consequences. Two out of three patients with insomnia symptoms are dissatisfied with their sleep. In most cases, insomnia symptoms are underrated both by the patients, who cover the problem or reject treatment, and by the GP, who limits intervention on the sleep disorder (scarcely modifying ongoing therapy both in responders and in non-responders). In responders, treatment was confirmed in 91% of cases and discontinued in only 2%. When there was no improvement, or if insomnia symptoms became worse (non-responders), treatment was nevertheless continued in 74.5% of cases, either maintaining the same ineffective dose, increasing the dose, or adding another drug or a non-pharmacological procedure. Regardless of specific medication, the Italian GP privileges the pharmacological approach, which is fourfold more frequent than non-pharmacological therapy (78.6 versus 18.2%). Non-benzodiazepine hypnotic drugs are mostly prescribed when the GP decides to apply medication in previously untreated patients with insomnia symptoms. Self-administration is not unusual among the patients with insomnia symptoms and is more common among non-responders. Italian GPs tend to confirm the ongoing therapy and avoid re-evaluation of the treatment regimen. Limited use of non-pharmacological treatment in the Italian primary care setting is in line with this conservative approach of the GPs who tend to be problem-solvers rather than problem-seekers. Insomnia is the most prevalent sleep disorder in the general population, and one of the most frequent reasons for consultation in the Sleep Units. Perampanel is an antiepileptic also effective on the structure of sleep, and in restless legs syndrome. We describe the first study that evaluates perampanel in patients with chronic insomnia. Observational retrospective cohorts study of 66 patients with chronic resistant insomnia, 33 exposed to perampanel, other 33 as non-exposed group. All patients attended in Neurology or Psychiatry Consultation, from November 2017 to November 2018. Patients included had been treated with more than 4 different drugs in the previous 4 years. We reviewed age, sex, insomnia etiology, years of evolution, number of previously used drugs, and the results of perampanel for insomnia after 3 months of treatment in the exposed cohort, measured by the improvement of 3 or more points in the ISI and Pittsburgh scales, as well as the average of hours of sleep gained. Non-exposed patients were matched with this variables, but never treated with perampanel. We have included 66 patients. In the exposed cohort: we describe 33 patients with chronic resistant insomnia, 20 women (60 %), 13 men (40 %). Average age 53.48 years, average time of evolution: 11.25 years. Main etiology: depression 13 cases (40 %). After the combination of perampanel 2-4 mg (100 %) with antidepressants (17 cases, 51.5 %) or anxiolytics (12 cases, 36.36 %) along 3 months: the total number of hours of sleep improves in 2.5 h, the scale ISI improves by 6 points (± 2.1 SD, p = 0.02), and Pittsburgh scale improves in 4 points (± 1.7, p = 0.04). In non-exposed cohort, the improvement of the ISI scale was 2.2 points (±0.8, p = 0.06), on the Pittsburgh scale was 1.6 points (± 0.5, p = 0.01). The main adverse effect was irritability in 3 patients, without withdrawal perampanel. The treatment was abandoned by 4 patients (12.12%): 1 due to persistent irritability (3%), 2 due to lack of efficacy (6 %), 1 due to pregnancy wish (3 %). The combination of Perampanel with an antidepressant, or an anxiolytic, improves the quality of sleep measured by ISI and Pittsburgh scales (statistically significant), probably due to its antagonistic action on glutamate. A clinical trial compared with placebo would be necessary to corroborate these results. Tolerance Chronic use of benzodiazepines, such as chlordiazepoxide, leads to the development of tolerance, with a decrease in number of benzodiazepine binding sites in mouse forebrain. The Committee of Review of Medicines, who carried out an extensive review of benzodiazepines including chlordiazepoxide, found—and were in agreement with the Institute of Medicine (USA) and the conclusions of a study carried out by the White House Office of Drug Policy and the National Institute on Drug Abuse (USA)—that there was little evidence that long-term use of benzodiazepines were beneficial in the treatment of insomnia due to the development of tolerance. Benzodiazepines tended to lose their sleep-promoting properties within 3–14 days of continuous use, and in the treatment of anxiety the committee found that there was little convincing evidence that benzodiazepines retained efficacy in the treatment of anxiety after 4 months' continuous use due to the development of tolerance. Insomnia Barbiturates were introduced as hypnotics for patients with schizophrenia. It induced a state of deep and prolonged sleep. But this was not used for long because of adverse side effects. Anticonvulsant Phenobarbital was the first truly effective drug against epilepsy. It was discovered by accident when given to epileptic patients to help them sleep. The positive side effects were anticonvulsant properties that reduced seizure number and intensity. Sedation Pentobarbital is used as a hypnotic when analgesia is not required. It´s often used in CT imaging when sedation is needed. It is efficient, safe and the recovery time is short. In 2013 the barbiturates phenobarbital and butabarbital are still used as sedatives in certain cases as well as to antagonize the effects of drugs as ephedrine and theophylline. Phenobarbital is used in cases of drug withdrawal syndromes. It is used as normal and emergency treatment in some cases of epilepsy. Generally, these treatments are given after the behavioral treatment has failed. Drugs such as tranquilizers, though they may work well in treating insomnia, have a risk of abuse which is why these treatments are not the first resort. Some sleep disorders such as narcolepsy do require pharmacological treatment. See also Sleep disorder Sleep medicine Snoring References External links Sleep disorders sv:Somnologi Pharmacological treatments Pharmacological treatments are used to chemically treat sleep disturbances such as insomnia or excessive daytime sleepiness. The kinds of drugs used to treat sleep disorders include: anticonvulsants, anti-narcoleptics, anti-Parkinsonian drugs, benzodiazepines, non-benzodiazepine hypnotics, and opiates as well as the hormone melatonin and melatonin receptor stimulators. Anticonvulsants, opiates, and anti-Parkinsonian drugs are often used to treat restless legs syndrome. Furthermore, melatonin, benzodiazepines hypnotics, and non-benzodiazepine hypnotics may be used to treat insomnia. Finally, anti-narcoleptics help treat narcolepsy and excessive daytime sleepiness. Of particular interest are the benzodiazepine drugs which reduce insomnia by increasing the efficiency of GABA. GABA decreases the excitability of neurons by increasing the firing threshold. Benzodiazepine causes the GABA receptor to better bind to GABA, allowing the medication to induce sleep. In India, clobazam is approved for use as an adjunctive therapy in epilepsy, and in acute and chronic anxiety. In Japan, clobazam is approved for adjunctive therapy in treatment-resistant epilepsy featuring complex partial seizures. In New Zealand, clobazam is marketed as Frisium In the United Kingdom clobazam (Frisium) is approved for short-term (2–4 weeks) relief of acute anxiety in patients who have not responded to other drugs, with or without insomnia and without uncontrolled clinical depression. It was not approved in the United States until October 25, 2011, when it was approved for the adjunctive treatment of seizures associated with Lennox-Gastaut syndrome in patients 2 years of age or older. Contraindications Clobazam should be used with great care in patients with the following disorders: Myasthenia gravis. Sleep apnea. Severe liver diseases such as cirrhosis and hepatitis. Severe respiratory failure. Good sleep is an important index of the quality of life in people and above all in old subjects. Among all the symptoms reported to general practitioner, insomnia is at the 3(rd) place and this is present in particular in the elderly. In elderly people high comorbidity and polytreatment are often present. We have studied 60 elderly people with history of insomnia and concomitant diseases: depression, dementia and behavioral disturbances. All the patients of the present study were visited in our outpatients' department. Three hypnotic drugs were used for the treatment of insomnia: zolpidem, or triazolam, or oxazepam, respectively at doses of 10mg/day, 0.125-0.25mg/day and 15.0mg/day. All the three drugs showed to be effective and safe; no paradoxical effects were observed. Sam is a 6-year-old boy with a diagnosis of autism spectrum disorder (ASD) who recently relocated and has an appointment with you, his new pediatric clinician, to establish care. He was previously followed by a psychiatrist for 2 years for additional diagnoses of insomnia, bipolar disorder, anxiety, attention deficit hyperactivity disorder, and intellectual disability. He has tried and (apparently) failed multiple psychotropic trials including stimulants, nonstimulants, mood stabilizers, atypical antipsychotics, and nonbenzodiazepine hypnotics. He has a delayed sleep onset and frequent night awakenings each night for the past 3 months, during which he \"screams, cries, and thrashes and can stay up for over an hour.\" His behaviors are described as irritable, self-injurious, and aggressive with no clear pattern of triggers according to his mother. He is nonverbal and communicates by leading and rarely pointing. The patient's current medication regimen includes clonidine 0.2 mg at night, lorazepam 1.5 mg as needed at night, olanzapine 5 mg twice daily, and diphenhydramine as needed for sleep/agitation. His mother is concerned that he is developing \"tolerance\" to the regimen and wants to wean him off some of the medications. His mother is struggling to take care of the patient given his worsening behavior and body habitus (body mass index >99%; z = 3.41).There is a family history of depression, anxiety, bipolar disorder, and autism. He has a 3-year-old sister, who is also diagnosed with ASD, though she is not as severely impacted. His mother's partner recently moved in along with 2 children of his own, aged 3 and 4 years. Sam attends a specialized school, where he receives behavior therapy and occupational therapy. He has undergone inpatient pediatric hospitalization twice, 1 time for 3 weeks and the other for 6 days, for aggressive behavior, and in both instances, he was discharged before inpatient psychiatric placement because of a lack of available beds.After urgent consultation with your local developmental and behavioral pediatrician, a slight reduction was made in the lorazepam because of concerns about tolerance and side effects. However, within a week of this, he was brought to the emergency department for continued self-injurious behavior and increased trouble with sleeping. His mother voiced concerns about his safety in the home, which were particularly related to aggression toward his younger sister. He was admitted to the pediatric inpatient floor for observation, and medication adjustment (increasing olanzapine), which was initially helpful in improving behavior, but mostly behavioral/environmental strategies were used to soothe him, including frequent wagon rides through the hospital corridors.Despite the patient being stable from the medical standpoint, Sam's mother did not feel comfortable taking him home. Social work contacted local community mental health services to pursue outpatient resources and respite care options and sought inpatient pediatric psychiatry. After several failed attempts to find placement, he remained in pediatric inpatient care for 1 and a half months with no acute medical interventions other than his oral medications.He was finally accepted to the in-state pediatric psychiatric facility when a bed was available. During his week-long stay, he had further medication adjustments with a decrease in olanzapine and optimization of his clonidine dose. During his psychiatric hospital stay, care coordination succeeded in arranging center-based applied behavior analysis interventions and respite care and parent training for his family. Sam began to show improvement in his overall agitation and aggression, requiring less clonazepam, and his mother then maintained outpatient follow-up.The day before discharge, you visit him in the hospital, and a medical student asks you why he was in the hospital for so long. How would you answer the question? The management of sleep disorders in elderly patients with internal diseases consists in the first line in rectifying pathophysiological disregulations. Only in the second line, proper hypnotics are to be prescribed. When considered as indispensable, these medicaments are selected according to their toxicity and side effects. In present time, Benzodiazepines are definitely preferred, whereas neuroleptic, anti-depressant and the older drugs are to be taken secondarly in account. We report on the case of a 45-year old female with chronic insomnia and refractory to hypnotics, who also has a - polygraphically documented - tolerance to the imidazopyridine \"zolpidem\". We discuss the main differential diagnosis and demonstrate a therapeutic regimen which allows a step-by-step replacement of hypnotics by sedative antidepressants. This interval replacement treatment reduces on the one hand the risk of developing a severe withdrawal syndrome. On the other hand the replacement by sedative antidepressants improves insomnia and insomnia-associated depressive symptoms. Finally, the clinical implications and rationale of a therapeutic approach with sedative antidepressants in chronic insomnia are discussed. Chlordiazepoxide, trade name Librium among others, is a sedative and hypnotic medication of the benzodiazepine class; it is used to treat anxiety, insomnia and symptoms of withdrawal from alcohol and other drugs. Chlordiazepoxide has a medium to long half-life but its active metabolite has a very long half-life. The drug has amnesic, anticonvulsant, anxiolytic, hypnotic, sedative and skeletal muscle relaxant properties. Chlordiazepoxide was patented in 1958 and approved for medical use in 1960. It was the first benzodiazepine to be synthesized and the discovery of chlordiazepoxide was by pure chance. Chlordiazepoxide and other benzodiazepines were initially accepted with widespread public approval but were followed with widespread public disapproval and recommendations for more restrictive medical guidelines for its use. Psychiatric patients often have residual intractable insomnia as a serious problem. Forty-eight psychiatrically ill patients (DSM-IV diagnoses) who had failed to respond to medicinal treatment for chronic insomnia were referred for and completed behavioral therapy as an adjunct to the pharmacologic treatment of their insomnia. The behavioral treatments included structured sleep hygiene, progressive muscle relaxation, stimulus control, and sleep restriction. The treatment program was accomplished in 6 sessions over 2 months. Follow-up evaluations were completed at 2, 6, and 12 months from the beginning of the treatment program. The outcome of the treatment program was evaluated in terms of the change in (1) self-reported specific sleep parameters, (2) self-ratings of sleep-related day-time state, (3) self-rating of quality of sleep, (4) the use of sleep medication, and (5) the therapist's global rating of improvement. There was a statistically significant change from the baseline in all self-reported specific sleep parameters after 2 months that was sustained after 6 and 12 months. Sleep-related characteristics of daytime state showed statistically significant changes after 2 and 6 months that were maintained after 12 months. Sleep quality had a statistically significant change after 2 months, continued to improve statistically after 6 months, and was maximum after 12 months. Over half the patients (52.7%; 20 of 38) either reduced their sleep medication by half or stopped it completely. The therapist's global rating showed an improvement in 29.2% (N = 14) of patients after 2 months, 56.2% (N = 27) after 6 months, and 68.7% (N = 33) after 12 months. The use of concomitant behavioral and pharmacologic treatment of chronic insomnia in psychiatrically ill patients results in improving sleep and sleep-related state and reduces the risk of return of insomnia for 10 months after finishing active treatment. Clinical trials conducted in general practice are more especially interesting as they enable to test a drug in the real conditions of use. On the other hand, these trials are beneficial to the G.P. (new image, rupture of his loneliness, change in his prescription habits, contact with hospitals). The methodology, as for hospital studies, must be rigorous. As a matter of fact, these two types of studies are additional and the cooperation between the G.P.'s and the pharmaceutical industry can conduct to the solution of specific problems: drug interaction--long-term therapeutic effect--new indications. This double blind cross over study comparing triazolam and nitrazepam conducted by G.P.'s on insomniacs is the first French clinical study intended for the registration application and done according to this methodology. This work is an exemple of the new opportunity offered to G.P.'s in future. The results have shown that: on 54 patients (23 male and 31 female) of an average age: 48, 32 have preferred triazolam, 13 have preferred nitrazepam, 9 have had no preference. Contraindications and interactions Cabotegravir/rilpivirine must not be combined with drugs that induce the liver enzyme CYP3A4, because they accelerate the inactivation of rilpivirine, and/or the enzmye UGT1A1, because they accelerate the inactivation of cabotegravir. These mechanisms potentially result in loss of effectiveness. Examples for such drugs are rifampicin, rifapentine, carbamazepine, oxcarbazepine, phenytoin and phenobarbital. Adverse effects The most common side effects of the injectable combination therapy with rilpivirine are reactions at the injection site (in up to 84% of patients) such as pain and swelling, as well as headache (up to 12%) and fever or feeling hot (in 10%). Less common side effects (under 10%) are depressive disorders, insomnia, rashes, fatigue, musculoskeletal pain, nausea, sleep disorders, and dizziness. Pharmacology Insomnia, a more or less chronic sleep disturbance, is a very common symptom in psychiatric patients but also relatively freguent in the general population to a lesser degree. Two broad types of insomnia may often be distinguished: (1) difficulty falling asleep and frequent wakening, characteristic of anxiety states or obsessive worrying; and (2) early morning wakening, sometimes in a panic, suggestive of endogenous depression. The first group of patients respond well to minor tranquilizers and psychotherapy, whereas the second do well with tricyclic anti-depressants. Many studies in sleep laboratories have declineated the stages and cycles of sleep physiology and pathology, especially the importance of REM or dreaming sleep. The clinician should be cautious in the use of hypnotics like barbiturates which suppress REM sleep and produce a rebound increase on withdrawal, as well as problems of dependence of habituation. Flurazepam and chloral hydrate are considerably safer in this respect. Understanding sleep neurophysiology and biochemistry permits appropriate individual clinical management for both psychiatric patients and medical patients with conditions like peptic ulcer and nocturnal angina pectoris. for anesthetic purposes, and are also sometimes prescribed for anxiety or insomnia. This is not a common practice anymore, however, owing to the dangers of long-term use of barbiturates; they have been replaced by the benzodiazepines and Z-drugs such as zolpidem, zaleplon and eszopiclone for sleep. The final class of barbiturates are known as long-acting barbiturates (the most notable one being phenobarbital, which has a half-life of roughly 92 hours). This class of barbiturates is used almost exclusively as anticonvulsants, although on rare occasions they are prescribed for daytime sedation. Barbiturates in this class are not used for insomnia, because, owing to their extremely long half-life, patients would awake with a residual \"hang-over\" effect and feel groggy. The impossibility of treating patients with sleep disorders adequately means that, as specialists, we have to look for new pharmacological treatments and for this reason we examined the information in the paper by Salin Pascual (1999) about the increase in deep sleep when olanzapine is used as an antipsychotic drug. We decided to use this medication in six females and three males who were suffering from different sleep disorders that conditioned their chronic insomnia. The dosages of olanzapine used ranged from 2.5 and 10 mg in a single dose. The clinical history and progress were used to elaborate the results and conclusions. The result was positive in eight of the nine patients, five who were administered the medication as monotherapy and three as polytherapy. The population studied is insufficient to prove the effectiveness of the drug, but the fact that in eight of our patients the treatment clearly improved their symptoms leads us to think that this line of research must be continued. Non medication based strategies provide long lasting improvements to insomnia and are recommended as a first line and long-term strategy of management. Behavioral sleep medicine (BSM) tries to address insomnia with non-pharmacological treatments. The BSM strategies used to address chronic insomnia include attention to sleep hygiene, stimulus control, behavioral interventions, sleep-restriction therapy, paradoxical intention, patient education, and relaxation therapy. Some examples are keeping a journal, restricting the time spent awake in bed, practicing relaxation techniques, and maintaining a regular sleep schedule and a wake-up time. Behavioral therapy can assist a patient in developing new sleep behaviors to improve sleep quality and consolidation. Behavioral therapy may include, learning healthy sleep habits to promote sleep relaxation, undergoing light therapy to help with worry-reduction strategies and regulating the circadian clock. An enquiry into the handling by medical practitioners of sleeping problems among elderly patients was conducted in southern Lower Saxony by personal interview, combined with a standard questionnaire. A typical case report had been drafted concerning a 70-year-old, previously healthy widow: her complaints were \"nonspecific\" and could be classified as an example of either depression, of the onset of senile dementia or as within normal limits for age. This case report was presented by two interviewers to 145 general practitioners (GPs) and 14 neurologists in private practice (response rate of 83.2%) who were asked how they would have treated the patient's sleeping disorder. 30.3% of the GPs and 14.3% of the neurologists would initially not have prescribed medication. Only GPs (19.5%) mentioned possible herbal medication. Sedative neuroleptics were preferred by 57.1% of neurologists and 26.2% of GPs, while benzodiazepines would have been given by 14% of both groups. Antidepressive drugs and chloral hydrate were chosen less often (5.7% and 2.5%, respectively). These data support the finding of a high frequency of neuroleptic prescriptions given to the elderly. They also make clear that the possibility of treatment without drugs is usually not sufficiently explored. There is support showing positive sleep outcomes for people who follow more than one sleep hygiene recommendation. There is however no evidence that poor sleep hygiene can contribute to insomnia. While there is inconclusive evidence that sleep hygiene alone is effective as a treatment for insomnia, some research studies have shown improvement in insomnia for patients who receive sleep hygiene education in combination with cognitive behavioral therapy practices. Benzodiazepines and related drugs are the hypnotics of first choice. They shorten sleep latency, enhance sleep continuity and may prolong sleep duration. Their undesired effects include a persistent day-time sedation and ataxia when getting up at night. There is some risk of habit formation and dependence. For treating an acute insomnia, the prescription of hypnotics should be limited to a short duration (smallest package size), for treating chronic forms of insomnia they should have only an adjuvant role in therapy. Patients in general practice complaining of insomnia of recent origin have been studied in order to ascertain which factors may be of value in the detection of those more susceptible to drug dependence. The type of sleep disturbance was found to be of importance and a personal disturbance scale was found useful as a screening test in two-thirds of the patients. No difference was found in the development of dependence on amylobarbitone and nitrazepam. One of the most important factors in the prevention of drug dependence seems to be frequent review by the doctor after the first prescription and his cautionary advice to the patient. Antipsychotics Chlorpromazine, an antipsychotic and antiemetic drug which is classed as a \"major\" tranquilizer, may cause paradoxical effects such as agitation, excitement, insomnia, bizarre dreams, aggravation of psychotic symptoms and toxic confusional states. Barbiturates Phenobarbital can cause hyperactivity in children. This may follow after a small dose of 20 mg, on condition of no phenobarbital administered in previous days. Prerequisity for this reaction is a continued sense of tension. The mechanism of action is not known, but it may be started by the anxiolytic action of the phenobarbital. Barbiturates such as pentobarbital have been shown to cause paradoxical hyperactivity in an estimated 1% of children, who display symptoms similar to the hyperactive-impulsive subtype of attention deficit hyperactivity disorder. Intravenous caffeine administration can return these patients' behaviour to baseline levels. Psychological and behavioral therapies should be considered the first line treatment for chronic insomnia. Although cognitive behavioral therapy for insomnia (CBT-I) is considered the standard of care [1], several monotherapies, including sleep restriction therapy, stimulus control therapy, and relaxation training are also recommended in the treatment of chronic insomnia [2]. CBT-I is a multimodal intervention comprised of a combination of behavioral (eg, sleep restriction, stimulus control) and cognitive therapy strategies, and psychoeducation delivered in 4 to 10 weekly or biweekly sessions [3]. Given that insomnia is thought to be maintained by an interaction between unhelpful sleep-related beliefs and behaviors, the goal of CBT-I is to modify the maladaptive cognitions (eg, worry about the consequences of poor sleep), behaviors (eg, extended time in bed), and arousal (ie, physiological and mental hyperarousal) perpetuating the insomnia. CBT-I is efficacious when implemented alone or in combination with a pharmacologic agent. However, because of the potential for relapse upon discontinuation, CBT-I should be extended throughout drug tapering [4]. Although the treatment options should be guided by the available evidence supporting both psychological therapies and short-term hypnotic treatment, as well as treatment feasibility and availability, treatment selection should ultimately be guided by patient preference [5]. Despite its widespread use among treatment providers [6], the use of sleep hygiene education as a primary intervention for insomnia should be avoided. Sleep hygiene may be a necessary, but insufficient condition for promoting good sleep and should be considered an adjunct to another empirically supported treatment. The first-line drugs for treatment of status epilepticus are benzodiazepines, such as lorazepam or diazepam. If these fail, then phenytoin may be used, with phenobarbital being an alternative in the US, but used only third-line in the UK. Failing that, the only treatment is anaesthesia in intensive care. The World Health Organization (WHO) gives phenobarbital a first-line recommendation in the developing world and it is commonly used there. Phenobarbital is the first-line choice for the treatment of neonatal seizures. Concerns that neonatal seizures in themselves could be harmful make most physicians treat them aggressively. No reliable evidence, though, supports this approach. Phenobarbital is sometimes used for alcohol detoxification and benzodiazepine detoxification for its sedative and anti-convulsant properties. The benzodiazepines chlordiazepoxide (Librium) and oxazepam (Serax) have largely replaced phenobarbital for detoxification. Insomnia is a symptom that should be treated according to the underlying etiology. It is more common in elderly individuals and in women. Common causes of insomnia include acute situational factors, psychiatric disorders, use of various medications and illicit drugs, and medical disorders that cause pain, dyspnea or nausea. Pharmacotherapy should be generally restricted to use of the benzodiazepines, imidazopyridines (zolpidem) and occasionally tricyclic antidepressants. As a rule, hypnotic drugs should be used for less than two weeks to one month.\nHere is the question:\nPSYCHIATRY: We are consulted by an 84-year-old woman for insomnia of conciliation. After failing sleep hygiene measures, it is decided to initiate pharmacological treatment. Which of the following drugs would you select for the patient?\nHere are the potential choices:\n1. Diacepam.\n2. Lormetacepam.\n3. Phenobarbital.\n4. Chlordiazepoxide.\n5. Chloracepate.\nThe correct answer is: 2. Lormetacepam." }, { "question": "You are a helpful medical expert, and your task is to answer a multi-choice medical question using the relevant documents. Please choose the answer from the provided options. Your responses will be used for research purposes only, so please have a definite answer.\nHere are the relevant documents:\n A 64-year-old man was admitted to our department because of muscle cramp, atrophy and weakness of the limbs together with difficulty in walking, which had gradually progressed from age 60. About 1 year prior to admission, he had noticed hand tremor and gynecomastia. On admission, neurological examination revealed diffuse muscle atrophy and weakness of the extremities, which were more obvious on the right side with preponderance in the right leg. Bilateral postural hand tremor was also more prominent on the right hand. Fasciculations were observed both in the extremities and tongue. The remaining cranial nerves and cerebellar functions were intact. Sensation was normal except for slightly decreased vibratory sense in the distal part of the legs. Deep tendon reflexes including jaw jerk were increased with the exception of hyporeflexia of the right leg. Babinski sign was negative bilaterally. Blood examination disclosed slight elevation of CK and fasting glucose level of 110 mg/dl. Glucose tolerance test showed a diabetic pattern. CSF examination showed total protein of 74 mg/dl and IgG of 12 mg/dl. On a series of endocrinological studies, there was no significant elevation of androgen and estrogen both in serum and urine except for slight elevation of serum E1 level. Serum LH and FSH, however, were markedly high, which responded far beyond the normal range following to 0.1 mg injection of LH-RH. These results suggested that gynecomastia might be caused by dysfunction of the hypothalamus-hypophysis system. Brain CT and spine MRI showed no abnormality. Muscle biopsy obtained from the right quadriceps femoris revealed neurogenic abnormalities.(ABSTRACT TRUNCATED AT 250 WORDS) Signs and symptoms As a result of lower motor neuron degeneration, the symptoms of PMA include: muscle weakness muscle atrophy fasciculations Some patients have symptoms restricted only to the arms or legs (or in some cases just one of either). These cases are referred to as flail limb (either flail arm or flail leg) and are associated with a better prognosis. Diagnosis PMA is a diagnosis of exclusion, there is no specific test which can conclusively establish whether a patient has the condition. Instead, a number of other possibilities have to be ruled out, such as multifocal motor neuropathy or spinal muscular atrophy. Tests used in the diagnostic process include MRI, clinical examination, and EMG. EMG tests in patients who do have PMA usually show denervation (neuron death) in most affected body parts, and in some unaffected parts too. It typically takes longer to be diagnosed with PMA than ALS, an average of 20 months for PMA vs 15 months in ALS. A 57-year-old man developed severe muscle weakness and atrophy of the upper extremities within a five-month period. Neurological examination revealed severe weakness and atrophy in the scapular muscles and proximal and distal muscles of the upper extremities. Fasciculations were also observed in the various muscles of the upper extremities. There was neither muscle weakness, atrophy nor fasciculation in either his face, neck muscles or lower extremities. He had no pseudobulbar or bulbar signs. Tendon reflexes were mildly hyperactive in the jaw and lower extremities, and normal in the upper extremities. There were no pathological reflexes, spasticity or sensory disturbances. The needle EMG study revealed denervation potentials in all muscles of the upper extremities examined. The nerve conduction study revealed no findings of the conduction block. Cervical spine X-rays revealed the narrowing of the spinal foramens at the left C3/C4 and bilateral C4/C5, C5/C6, and C6/C7 intervertebral levels. In addition, magnetic resonance imaging (MRI) revealed compressions of the cervical cord at C4/C5 and C5/C6 intervertebral levels. These clinical and neuroradiological findings resembled those of the cervical spondylotic amyotrophy (CSA). However, the motor evoked potential (MEP) study revealed the pyramidal tract dysfunction above the levels of the pyramidal decussation. Furthermore, brain MRI revealed abnormal foci in both internal capsules which were characterized by hyperintense relative to cortical gray matter on T2-weighted images and still hyperintense to white matter on proton-density-weighted images. In addition, T2-weighted images demonstrated a low signal within the motor cortex and hyperintense lesions in the white matter of the precentral gyri. These MRI findings indicated the degeneration of the pyramidal tract and corresponded to those found in the patients with amyotrophic lateral sclerosis (ALS) which have been recently reported. It has been difficult to distinguish ALS from CSA. However, MEP and brain MRI studies were useful for distinguishing these two diseases in this patient. In addition, this patient showed typical MRI findings suggesting the degeneration of the pyramidal tract, although this patient had a relatively short course of illness and did not show obvious physical findings suggesting pyramidal tract dysfunction. We report an 80-year-old woman with progressive muscular atrophy predominantly involving her right lower extremity. She was well until 1992 (75 years of age) when she noted an onset of weakness in her right leg which had got progressively worse. She was admitted to our service in July 1994. On admission, general physical examination was unremarkable. She was alert and well oriented without dementia. Higher cerebral functions were normal. Cranial nerves also appeared intact. She dragged her right leg in walking. Mild to moderate weakness (2/5 to 4/5) was noted in muscles in her right lower extremity more in the distal part. Deep tendon reflexes were within normal limits, and the plantar response was flexor bilaterally. Sensation was intact. Laboratory examinations were also unremarkable except for slight increase in CK which was 470 IU/l. CSF was also normal. EMG revealed neurogenic changes in the lower extremities. She was admitted to Aoki Hospital on October 21, 1994, by that time, her weakness in the right lower extremity had gotten worse in that the muscle strength of the right extensor hallucis longus was 0 and tibialis anterior 2; muscle atrophy was also prominent in her right leg; the right ankle jerk could not be elicited. In the subsequent course, weakness and atrophy appeared in her left lower extremity, however, upper extremities and cranial nerves had never been affected. Babinski sign was always negative. In February 1996, she developed delusional ideation of self persecution, and showed difficulty in communication with medical staffs. She developed fever of 38.7 degrees C on June 13, 1996 expired on the next day. The patient was discussed in a neurological CPC, and the chief discussant arrived at the conclusion that the patient had a form of spinal muscular atrophy. Opinions were divided between ALS and spinal muscular atrophy. Post-mortem examination revealed marked loss of anterior horn neurons in the lumbar area with astrogliosis. Bunina bodies were seen in some of the remaining neurons. No myelin pallor was noted in the pyramidal tracts, however, atrophy and loss of Betz cells were noted in the motor cortex. Other cortical areas were unremarkable. The neuropathologist arrived at the conclusion that the patient had ALS. This patient was unique in that she had asymmetric atrophy and weakness limited to the lower extremities. This is quite unusual as ALS of four years duration. In addition, the patient developed some mental change which was thought to represent dementia by some participants. But no clear morphologic changes were seen to account for her mental change. A 70 year old woman had suffered from diabetes mellitus type 2 since she was 52. Three years before the surgery she had begun to experience weakness together with altered sensitivity in the right leg, which was regarded as having been caused by diabetic polyneuropathy. During the admission examination the level for algesia on the right-hand side was at about D 11, a distal paraparesis of the leg (3-4 degrees, Janda's classification), more intense on the right, hyperactive deep tendon reflexes, Babinski's reflex on both sides, and depressed abdominal cutaneous reflexes. The sensitivity to vibrations on the Malleolus medialis on both sides was 0/8. The patient could walk only with the help of a Rollator. Over the three-year period following onset of symptoms the following tests were carried out: motor nerve conduction speeds of the N. tibialis and N. peronaeus, electromyogram of the N. tibialis anterior and the M. gastrocnemius, somatosensory evoked potentials (SSEP) of the N. tibialis, which indicated a lesion in the peripheral nerves or nerve roots. Cranial computed tomography (CCT), CT scan of the lumbar spine (L3-S1) and angiological investigation elicited no significant pathological findings. An MRI of the thoracal spine showed a vertebra-sized dorsal tumor pressing on the spinal cord from left to right. By means of microsurgery the spinal tumor was completely removed. Suspected meningeoma was confirmed by histological analysis. During the post-surgical period, the incomplete paraplegia quickly regressed, and 7 weeks after the removal of the spinal meningeoma the patient was able to climb stairs. In case of slowly-developing paresis of the legs in diabetic patients, diabetic polyneuropathy should not be diagnosed without careful consideration, and rare spinal tumors should be considered as part of the differential diagnosis, especially if the blood glucose level is normal, and intensive physiotherapy brings no improvement in the patient's condition. Familial spastic paraplegia (FSP) with severe muscular atrophy of hands and feet is exceptional. Autosomal dominant forms were initially described by Silver in 1966. We report two cases, from the same Tunisian family, presenting FSP with severe amyotrophy of the hands. A brother and his sister, aged respectively 37 and 36 years old, presented practically the same clinical picture. Their parents were cousins. The female patient was hospitalized. Both patients developed gait disorders around the age of three years. Muscular atrophy of the hands arose much later, around the age of 20 years. The neurological examination disclosed a spastic gait with distal amyotrophy, severe in the hands and moderate in the feet. Sensitivity was preserved and there was no fasciculation. The spinal cord and cerebral MRI was normal. Electromyography (EMG) showed a neurogenic pattern in the distal muscles. Stimulation of the median, ulnar and sciatica nerves was ineffective. The somatosensory evoked potentials (EP) were delayed (upper limb) or desynchronised (lower limb). The auditory and visual EP were normal. The cerebrospinal fluid contained 1 mononuclear cell/mm3 and 10 mg protein/100 ml. Abnormalities of the cranio-vertebral junction, Arnold-Chiari malformation, syringomyelia and familial juvenile amyotrophic lateral sclerosis (ALS) were excluded and the diagnosis of Silver's syndrome was evoked. We report a patient with motor neuron syndrome similar to amyotrophic lateral sclerosis (ALS) and with spontaneous recovery. At the age 40, the woman developed progressive muscular weakness, atrophy and fasciculation in extremities. She also noted a dyspnea, tongue atrophy and dysphagia. A neurological examination 6 months after onset revealed i) a tongue atrophy and fasciculation, ii) diffuse muscule weakness and atrophy in face, neck and extremities, and iii) marked hyperreflexia in the four limbs and bilateral Babinski reflex, but iv) neither sensory disturbance nor ophthalmoplegia. Electromyogram (EMG) detected such denervation potentials as fibrillation potentials, fasciculation potentials, positive sharp waves and polyphasic or giant MUPs diffusely in the limb muscles. Peripheral nerve conduction study detected neither conduction block nor delay. Thus, she was diagnosed as suffering from ALS. However, since approximate 1 year after onset, her muscle weakness has gradually been getting better. Simultaneously, the dyspnea and dysphagia gradually improved. Two years after onset, an EMG examination detected chronic denervation potentials in the left musculus sternocleidomastoideus and a few on-going denervation potentials in the left musculus extensor carpi radialis, but no denervation potentials in other limb muscles. Fasciculation potentials were found in tongue muscles. Thus, the present case was thought to have a reversible motor neuron syndrome clinically quite similar to ALS. A mild increase in IgE (346 U/ml) and a low-titer IgM-class anti-GM1 antibody were found in her serum though its pathological significance was uncertain. Any immunological aberrance may account for the pathogenesis. It should be noted that clinically diagnosed cases of ALS may rarely recover spontaneously. Corticobasal degeneration (CBD) is an uncommon, sporadic, neurodegenerative disorder of mid- to late-adult life. We describe a further example of the pathologic heterogeneity of this condition. A 71-year-old woman initially presented dysarthria, clumsiness, progressive asymmetric bradykinesia, and rigidity in left arm. Rigidity gradually involved ipsilateral leg; postural instability with falls, blepharospasm, and dysphagia subsequently developed. She has been previously diagnosed as unresponsive Parkinson's Disease. At our clinical examination, she presented left upper-arm-fixed-dystonia, spasticity in left lower limb and pyramidal signs (Babinski and Hoffmann). Brain MRI showed asymmetric cortical atrophy in the right frontotemporal cortex. Neuropsychological examination showed an impairment in visuospatial functioning, frontal-executive dysfunction, and hemineglect. This case demonstrates that association of asymmetrical focal cortical and subcortical features remains the clinical hallmark of this condition. There are no absolute markers for the clinical diagnosis that is complicated by the variability of presentation involving also cognitive symptoms that are reviewed in the paper. Despite the difficulty of diagnosing CBD, somatosensory evoked potentials, motor evoked potentials, long latency reflexes, and correlations between results on electroencephalography (EEG) and electromyography (EMG) provide further support for a CBD diagnosis. These techniques are also used to identify neurophysiological correlates of the neurological signs of the disease. Hoover's sign (63% Sensitivity & 100% specificity): This test is commonly used to separate organic from the nonorganic cause of weakness or paralysis. An examiner's hand is placed below the heel of the affected leg, and the patient is asked to flex the hip of the normal leg against resistance. In organic disorders, there should not be any pressure on the examiner's hand on the affected side, while pressure is felt in patients with FNSD/CD. [20] Variable Strength (63% sensitivity and 97% specificity): The weakness is inconsistent with variable force at different locations. In patients with neurologic signs, nerve conduction studies disclose reduced amplitude of the ulnar sensory potentials. There may be decreased amplitude of the median motor evoked potentials as well, a mild but uniform slowing of the median motor conduction velocity, and a prolongation of the F-wave latency. Concentric needle examination of affected hand muscles reveals large-amplitude motor units, suggesting collateral reinnervation. Somatosensory evoked potentials may be a useful adjunct to the conventional nerve conduction and EMG studies (Yiannikas and Walsh). Brachial artery MR angiography is usually reserved for patients with a suspected arterial occlusion, an aneurysm, or an obvious cervical rib. The place of venography in the diagnostic workup is uncertain, for a number of otherwise normal individuals can occlude the subclavian vein by fully abducting the arm. The biceps and brachioradial reflexes on one or both sides may be depressed, sometimes in association with an increase in the triceps and finger reflexes. The hand or forearm muscles may undergo atrophy; in a few cases, the atrophy of hand muscles is severe. In such cases, the spondylotic compression, as judged by MRI or CT myelography, may be confined to the high cervical cord, well above the levels of the motor neurons that innervate these muscles. In patients with sensory loss, pain and thermal sensation often appear to be affected more than tactile sense. An unexpected Babinski sign has already been mentioned and a few fasciculations may be seen, especially in proximal arm muscles. Another unusual feature in advanced stages of cervical cord compression is the appearance of mirror movements of the hands, in which effortful attempts to make refined movements of the fingers of one hand, causes the opposite hand to move similarly. Monoplegia with Muscular Atrophy This is more frequent than monoplegia without muscular atrophy. Long-continued disuse of one limb may lead to atrophy, but it is usually of lesser degree than atrophy caused by lower motor neuron disease (denervation atrophy). In disuse atrophy, the tendon reflexes are retained and nerve conduction studies are normal. With denervation of muscles, there may be visible fasciculations and reduced or abolished tendon reflexes in addition to paralysis. The location of the lesion (in nerves, spinal roots, or spinal cord) can usually be determined by the pattern of weakness, by the associated neurologic symptoms and signs, and by special tests—MRI of the spine, examination of the cerebrospinal fluid (CSF), and electrical studies of nerve and muscle. If the limb is partially denervated, the EMG shows reduced numbers of motor unit potentials (often of large size) as well as fasciculations and fibrillations. Though various textbooks describe clinical manoeuvres that help detect subtle motor deficits, their sensitivity, specificity and predictive values have not been determined. We investigated the sensitivity, specificity and predictive values of various manoeuvres in order to determine the most sensitive and reliable test or combination thereof. Straight arm raising (Barré), pronator drift, Mingazzini's manoeuvre, finger tap, forearm roll, segmental strength and deep tendon reflexes were tested in 170 patients with (86) and without (84) a proven lesion in the motor areas confirmed by computed tomography. Segmental motor strength bad good specificity (97.5%) but poor sensitivity (38.9%) and negative predictive value (NPV) (58.7%). The forearm roll had a similar profile. Finger tap had a sensitivity of 73.3% and a specificity of 87.5%. Barré and pronator testing had a sensitivity and specificity of 92.2% and 90.0% respectively. Hyperreflexia had a sensitivity of 68.9% and a specificity of 87.5%. An abnormality of pronator, reflexes or finger tap had a sensitivity of 97%, and when these three tests were positive, specificity was 97%. When all six tests were positive, the positive predictive value was 100%, when all six tests were negative the NPV was 100%. The detailed segmental examination has very good specificity for detecting motor deficits, but the sensitivity and NPV are unacceptably low. Pronator drift with finger tap and reflexes is the most reliable and time-effective combination of tests for the detection of subtle motor lesions, and could replace the segmental motor examination as a screening for motor lesions. Examination of the motor system of a limb includes checking for muscle bulk and fasciculation, muscle tone at joints, the power of muscle groups, deep tendon reflexes, clonus, plantar response, and coordination. In cases of a lower motor neuron type weakness, there is early muscle wasting, fasciculations, hypotonia, hyporeflexia, and a normal plantar response. On the other hand, the upper motor neuron type of weakness is characterized by normal muscle bulk, hypertonia, hyperreflexia, clonus, and an extensor plantar response (positive Babinski’s sign). Furthermore, the preservation of deep tendon reflexes distinguishes myopathy from neuropathy. Hirayama's disease (HD), is a benign, self-limited, motor neuron disease, characterized by asymmetric weakness and atrophy of one or both distal upper extremities. In the present study we report the clinical, electrophysiological and MRI features of a group of Italian patients, with review of the literature. Moreover we propose an optimized MRI protocol for patients with suspected or diagnosed HD in order to make an early diagnosis and a standardized follow up. Eight patients with clinical suspicion of Hirayama disease underwent evaluation between January 2007 and November 2013. All patients underwent standard nerve conduction studies (NCS), electromyography (EMG) and motor/sensory evoked potentials (MEP/SEP). Cervical spine MRI studies were conducted with a 1.5 Tesla MRI scanner in neutral and flexion position, including sagittal T1-weighted sequences and sagittal and axial T2-weighted sequences. The following diagnostic features were evaluated: abnormal cervical curvature, localized cervical cord atrophy in the lower tract (C4-C7), presence of cord flattening (CF), intramedullary signal hyperintensity on T2 weighted sequences, anterior shifting of the posterior wall of the cervical dural sac (ASD) and presence of flow voids (EFV) in the posterior epidural space during flexion. All patients complained of weakness in hand muscles as initial symptoms, associated with hand tremor in three of them and abnormal sweating of the hand palm in two of them. No sensory deficits and weakness at lower limbs were reported by any patients. Distal deep tendon reflexes at upper limbs were absent in all patients with the absence of the right tricipital reflex in one of them. Deep tendon reflexes at lower limbs were normal and no signs of pyramidal tract involvement were present. The clinical involvement at onset was unilateral in six patients (three left-sided and three right-sided) and bilateral asymmetric in two of them, with the right side more affected. With the progression of the disease all patients but one experienced weakness and wasting of hand muscles and forearm bilaterally, but still asymmetric. The duration of the progression phase of the disease ranged from eight months to three years. In all patients, NCS and EMG findings were consistent with a spinal metameric disorder involving the C7-T1 myotomes bilaterally; sensory conduction and electrophysiologic features at lower limbs were normal. MEP and SEP were normal and we did not observe the disappearance of the spinal potential during the neck flexion in any of the patients. MRI is the best diagnostic tool in the diagnosis of HD; it can confirm clinical diagnosis and exclude other conditions responsible for the neurological deficits leading to a correct patient management and therapy, limiting arm impairment. On MRI all patients had loss of the normal cervical lordosis (100%). Five patients had loss of attachment of posterior dural sac and anterior dural shift on flexion MRI with presence of flow voids from venous plexus congestion (62.5%); three patients had no anterior dislocation of the dural sac and no epidural vein congestion. Two patients showed localized cord atrophy, one at C5-C6 and the other at C6-C7 level (25%). Three patients had T2 intramedullary hyperintensities (37.5%) and cord flattening (CF) was present in 5 patients of 8 (62.5%). HD is a rare entity and a self-limited condition, but it has to be early differentiated from other diseases that may determine myelopathy and amyotrophy to establish a correct therapy and limit arm impairment. MRI is very important to confirm the clinical suspect of HD and a standardized MRI protocol using axial and sagittal images in both neutral and flexing position is needed, in order to diagnose and follow up affected patients. We wish to discuss the value of the clinical history and examination in orientation of the diagnosis of probable multiple sclerosis (MS). We report the two year study of a woman who over the previous ten years had had three episodes of paraesthesia, with pins and needles in her left leg and other parts of the left side of her body, although never affecting head or neck. She also complained of tiring more than usual. In an outpatient clinic she was found to have a syndrome affecting the upper segments of the spinal cord, mainly involving the right side and resembling an incomplete Brown Sequard type syndrome. There were increased clinical muscle and deep reflexes. The most marked was that of the right deltoid (C5), bilateral fanning of the toes when the Babinski reflex was tested, Barré positive in the right leg, pins and needles and dysaesthesia on the left to an undetermined level. Function was well preserved when compared with the clinical signs found. The case was considered to be of monotopical MS. Spinal magnetic resonance findings confirmed the clinical diagnosis. We emphasise the value of careful clinical investigation directed towards the diagnosis of probable MS. We draw attention to the diagnostic value of the dissociation between the severe clinical alterations and the functional performance, which was surprisingly well maintained. Also we report the originality of the presence of bilateral fanning sign supporting the diagnosis of MS, occurring in a disease of adult life. We report a 54-year-old man with progressive proximal muscle atrophy and gynecomastia. The patient had an insidious onset of weakness in his lower extremities at age 14, in that he noted a difficulty in standing up from a chair. Soon after he noted some difficulty in climbing up stairs. At age 35, he noted weakness in his arms; his weakness slowly progressed in that he became unable to walk or stand alone before 40 years of age. He also noted gynecomastia at that age. He was admitted to our hospital for the work up on September 16, 1993, when he was 54-year-old. On admission, he was alert and oriented; his BP was 150/70 mmHg; he had bilateral gynecomastia, however, no other skeletal deformities were found. On neurologic examination, he was mentally sound without dementia, and his higher cerebral functions were normal. Cranial nerves also appeared intact without facial atrophy, dysarthria, or dysphagia; no atrophy was noted in the tongue. He had marked muscle atrophy in both upper and lower extremities more marked in the proximal portions; muscle strength was approximately in the range of 2/5 to 3/5 in the proximal parts, and 4/5 in the distal parts in both upper and lower extremities. No fasciculation was noted; muscle tone was flaccid; no ataxia was present. Deep reflexes were either lost or markedly diminished. No Babinski sign was noted. Sensation was intact. Laboratory examination revealed normal blood counts; serum CK was slightly increased to 131 IU/l; ECG showed complete right bundle branch block; EMG revealed no active units in the right biceps brachii, deltoid, quadriceps femoris, and triceps surae muscles; in other muscles tested, motor unit potentials of low amplitude and short duration were seen; in the right tibialis anterior muscle, however, motor unit potentials with an amplitude up to 6 m V were also seen. Nerve conduction velocities were normal. A diagnostic procedure was performed. He was discussed in the neurological CPC, and the chief discussant arrived at the conclusion that this patient had Becker type of progressive muscular dystrophy. In her differential diagnosis, the possibility of Kennedy-Alter-Sung syndrome was discussed because this patient had gynecomastia. However, the discussant excluded that possibility because of absence of both bulbar symptoms and typical neurogenic changes in his EMG. The diagnostic procedure was a muscle biopsy on the left tibialis anterior muscle. Histologic observation on HE stained specimens revealed marked inequality in the muscle fiber diameters, increase in endomysial nuclei, proliferation of connective tissue, and fiber splitting.(ABSTRACT TRUNCATED AT 400 WORDS) 30,000/μL appear not to have increased mortality related to the thrombocytopenia. Pregnancy Anti-platelet autoantibodies in a pregnant woman with ITP will attack the patient's own platelets and will also cross the placenta and react against fetal platelets. Therefore, ITP is a significant cause of fetal and neonatal immune thrombocytopenia. Approximately 10% of newborns affected by ITP will have platelet counts <50,000/uL and 1% to 2% will have a risk of intracerebral hemorrhage comparable to infants with neonatal alloimmune thrombocytopenia (NAIT). No lab test can reliably predict if neonatal thrombocytopenia will occur. The risk of neonatal thrombocytopenia is increased with: Mothers with a history of splenectomy for ITP Mothers who had a previous infant affected with ITP Gestational (maternal) platelet count less than 100,000/uL Disorders Adapted from: The three broad categories of platelet disorders are \"not enough\"; \"dysfunctional\"; and \"too many\". Thrombocytopenia Immune thrombocytopenias (ITP) – formerly known as immune thrombocytopenic purpura and idiopathic thrombocytopenic purpura Splenomegaly Gaucher's disease Familial thrombocytopenia Chemotherapy Babesiosis Dengue fever Onyalai Thrombotic thrombocytopenic purpura HELLP syndrome Hemolytic–uremic syndrome Drug-induced thrombocytopenic purpura (five known drugs – most problematic is heparin-induced thrombocytopenia (HIT) Pregnancy-associated Neonatal alloimmune associated Aplastic anemia Transfusion-associated Pseudothrombocytopenia idiopathic thrombocytopenic purpura Vaccine induced immune thrombocytopenia Gilbert's syndrome Causes Although Evans syndrome seems to be a disorder of immune regulation, the exact pathophysiology is unknown, but a gradual loss of self-tolerance is postulated. Autoantibodies targeted at different antigenic determinants on red cells and platelets are assumed to cause isolated episodes of hemolytic anemia and thrombocytopenia, respectively. Diagnosis The diagnosis of primary Evans syndrome is made upon blood tests to confirm not only hemolytic anemia and immune thrombocytopenic purpura, but also a positive direct antiglobulin test (DAT) and an absence of any known underlying cause. In 27% to 50% of cases there is an associated malignancy or a predisposing autoimmune disease (e.g. systemic lupus erythematosus), it is then common to denote it as secondary Evans syndrome. Other antibodies may occur directed against neutrophils and lymphocytes, and \"immunopancytopenia\" has been suggested as a term for this syndrome. In patients with isolated thrombocytopenia, but without significant dysplasia, diagnosis of idiopathic thrombocytopenic purpura (ITP) rather than myelodysplastic syndrome (MDS) may be taken into account. It is important to make an accurate diagnosis because different treatments are used for ITP and MDS. The purpose of this study was to investigate the clinical and hematologic features of patients who were initially diagnosed as ITP but had cytogenetic abnormalities. We retrospectively reviewed cytogenetic studies of 100 patients who were diagnosed as ITP from 2004 to 2009 at Mokdong Hospital of Ewha Womans University based on clinical features and hematologic studies. Bone marrow pathology was re-evaluated based on 2008 WHO classification. Cytogenetic analysis was performed by 24-48 hr culture of bone marrow aspirates without using mitogens and 20 metaphases were analyzed. Of the 100 patients diagnosed as ITP initially, three patients (3%) had cytogenetic abnormalities. They had no thrombocytopenia-related symptoms and thrombocytopenia was found accidentally. The numbers of megakaryocytes in bone marrow were increased and dysplasia was not found in megakaryocyte, erythroid, and myeloid cell lineages. The proportion of blasts was within normal limits. Clonal chromosomal abnormalities found were der(1;7)(q10;p10), add(9)(q12), or t(7;11)(p22;q12). Presumptive diagnosis of MDS or diagnosis of idiopathic cytopenia of undetermined significance (ICUS) was made according to 2008 WHO classification. During the follow up, disease progression was not found. In patients with suspected ITP, cytogenetic analysis should be done. If specific clonal chromosomal abnormality is found, presumptive diagnosis of MDS has to be considered and close follow up is needed. The Harrington–Hollingsworth experiment was an experiment that established the autoimmune nature of the blood disorder immune thrombocytopenic purpura. It was performed in 1950 by the academic staff of Barnes-Jewish Hospital in St. Louis, Missouri. Experiment The experiment was undertaken in 1950 by William J. Harrington and James W. Hollingsworth, who postulated that in patients with idiopathic thrombocytopenic purpura (ITP), it was a blood factor that caused the destruction of platelets. To test this hypothesis, Harrington received 500 ml of blood from a patient with ITP. Within three hours, his platelets dropped to dangerously low levels and he experienced a seizure. His platelet count remained extremely low for four days, finally returning to normal levels by the fifth day. Bone marrow biopsy from Harrington's sternum demonstrated normal megakaryocytes, the cells necessary for platelet production. Diagnosis. The diagnosis of ITP usually is based on clinical presentation and the platelet count and does not often require a bone marrow examination. If atypical findings are noted, however, marrow examination is indicated to rule out an infiltrative disorder (leukemia) or an aplastic process (aplastic anemia). In ITP, an examination of the bone marrow reveals increased megakaryocytes and normal erythroid and myeloid elements.\nHere is the question:\nHEMATOLOGY: A 33-year-old woman consults for repeated epistaxis, petechiae and ecchymosis. Laboratory tests show thrombocytopenia with a platelet count of 4000 platelets/microliter. The initial presumptive diagnosis is chronic immune thrombocytopenic purpura (ITP). Which of the following statements is FALSE regarding the diagnosis of ITP?\nHere are the potential choices:\n1. The presence of lymphadenopathy or splenomegaly in the physical examination suggests a different diagnosis of ITP.\n2. Bone marrow analysis shows a decreased number of megakaryocytes without other alterations.\n3. Complete blood count shows isolated thrombocytopenia with often large platelets, without anemia unless there is significant bleeding or associated autoimmune hemolysis (Evans syndrome).\n4. The diagnosis of ITP is established by exclusion of other processes causing thrombocytopenia.\n5. The determination of antiplatelet antibodies is not accurate to establish the diagnosis.\nThe correct answer is: ", "gold_answer": "2 Bone marrow analysis shows a decreased number of megakaryocytes without other alterations.", "prediction": "You are a helpful medical expert, and your task is to answer a multi-choice medical question using the relevant documents. Please choose the answer from the provided options. Your responses will be used for research purposes only, so please have a definite answer.\nHere are the relevant documents:\n Despite the destruction of platelets by splenic macrophages, the spleen is normally not enlarged. In fact, an enlarged spleen should lead to a search for other possible causes for the thrombocytopenia. Bleeding time is usually prolonged in ITP patients. However, the use of bleeding time in diagnosis is discouraged by the American Society of Hematology practice guidelines and a normal bleeding time does not exclude a platelet disorder. Bone marrow examination may be performed on patients over the age of 60 and those who do not respond to treatment, or when the diagnosis is in doubt. On examination of the marrow, an increase in the production of megakaryocytes may be observed and may help in establishing a diagnosis of ITP. An analysis for anti-platelet antibodies is a matter of clinician's preference, as there is disagreement on whether the 80 percent specificity of this test is sufficient to be clinically useful. ITP is an autoimmune disease with antibodies detectable against several platelet surface structures. ITP is diagnosed by identifying a low platelet count on a complete blood count (a common blood test). However, since the diagnosis depends on the exclusion of other causes of a low platelet count, additional investigations (such as a bone marrow biopsy) may be necessary in some cases. In mild cases, only careful observation may be required but very low counts or significant bleeding may prompt treatment with corticosteroids, intravenous immunoglobulin, anti-D immunoglobulin, or immunosuppressive medications. Refractory ITP (not responsive to conventional treatment or constant relapsing after splenectomy) requires treatment to reduce the risk of clinically significant bleeding. Platelet transfusions may be used in severe cases with very low platelet counts in people who are bleeding. Sometimes the body may compensate by making abnormally large platelets. Idiopathic (immune) thrombocytopenic purpura (ITP) is the most frequent hemorrhagic disease in children. It represents the acquired megakaryocytic thrombocytopenia with the shortened life of platelets because of immunologic damage (antibodies absorbed by platelets). In the case of this acquired hemorrhagic disorder, in spite of compensatory increased function of the bone marrow, there is a reduced number of platelets because of their increased destruction by the reticuloendothelial system (destructive thrombocytopenia). There are three forms of ITP: acute, chronic and intermittent. The acute form occurs in 80-90% of cases with bleeding episodes lasting a few days or weeks, but no longer than 6 months. The chronic form occurs in 10-15% of children, while the rarest-intermittent form is characterized by periods of normalization in regard to the number of platelets but also with relapse in intervals of 1-3 months. The disease is caused by an immunological disorder in the sense of an imbalanced immune response. Immunologic damages of platelets cause shortening of the opsonized platelets life span. The most frequent platelet opsonins are the immumoglobulin G (IgG) antibodies directed at the platelet membrane in the form of autoantibodies, alloantibodies or possibly absorbed antigen caused by microorganism infection or drug intake. It is typical for the phenomenon of bleeding that it starts suddenly and without any other sign of illness. The most frequent acute form appears between the second and fourth year, and is characterized by seasonal occurrence usually after acute viral infections. Children older than 10 years of age, like adults, often have the chronic form associated with other immunologic disorders. The disease affects girls more often than boys (about three times more often) with moderate and constant increase of antiplatelet antibodies. Hemorrhagic manifestations include: petechiae, purpura, epistaxis, gastrointestinal and genitourinary bleeding. They depend on the grade of thrombocytopenia, although there is no strict correlation between the number of platelets and volume of bleeding. Low mortality of the acute ITP is almost exclusively due to intracranial hemorrhage. LABORATORY STUDIES: Thrombocytopenia represents a decrease in the number of blood platelets being a basic abnormality of the blood count. The half-life of platelets in ITP is shortened. Detection of antiplatelet antibodies is connected with technical difficulties, so they are established in about 30% of cases. Bleeding time is prolonged and so is the coagulum retraction which may be completely missed. The Rumpel-Leede test is positive. Clinical differentiation of drug-induced thrombocytopenia is not possible. However, other differential diagnostic possibilities are thrombotic-thrombocytopenic purpura and hemolytic-uremic syndrome. A child with aplastic anemia or acute leukemia, beside thrombocytopenia, has a characteristic finding of white and red blood cell count. Thrombocytopenia with absent radii syndrome is associated with skeletal system abnormalities. New knowledge about the role of the immune system in ITP determines the modern therapeutic modalities. In cases of acute ITP in children, there are two therapeutic options or therapies of choice: corticosteroids and high doses of intravenous immunoglobulin. Immunosupressive therapy means anti Rh(D) immunoglobulin, cyclosporine, cytostatics, danazol, loaded platelets. In cases of distinctive hemorrhagic syndrome there are also indications for platelet transfusion. Nowadays splenectomy is more restricted, because one third of cases is unsuccessful, whereas plasmapheresis is rarely used in children because of possible complications. ITP is the most frequent hemorrhagic disease in children. The disease is basically caused by an immunologic disorder with platelet destruction due to increased immunoglobulin on their membrane. (ABSTRACT TRUNCATED) To identify initial laboratory findings useful for the later diagnosis of idiopathic thrombocytopenic purpura (ITP) in adult patients with thrombocytopenia. We studied 62 consecutive adult patients who had thrombocytopenia and whose peripheral blood film was normal except for thrombocytopenia at presentation. Each patient underwent physical examination and routine laboratory tests and was prospectively followed for 22.5 +/- 9.8 months (range, 8 to 41 months). The frequency of antiglycoprotein (GP) IIb/IIIa antibody-producing B cells, the presence of platelet-associated and plasma anti-GPIIb/IIIa antibodies, the percentage of reticulated platelets, and the plasma thrombopoietin level were examined at the first visit. The final diagnosis was based on the clinical history, physical examination, complete blood test, bone marrow findings, and the clinical course at last observation. Forty-six patients were diagnosed as having ITP and 16 as having another disorder, including myelodysplastic syndrome, aplastic anemia, amegakaryocytic thrombocytopenia, and reduced platelet production, with or without other cytopenias, and without dysplasia or evidence for destruction. Six initial laboratory findings discriminated ITP from other diagnoses: the absence of anemia, absence of leukocytopenia, increased frequency of anti-GPIIb/IIIa antibody-producing B cells, increased platelet-associated anti-GPIIb/IIIa antibodies, elevated percentage of reticulated platelets, and a normal or slightly increased plasma thrombopoietin level. Three or more of these ITP-associated findings were found at presentation in 44 patients (96%) with thrombocytopenia later diagnosed as ITP, compared with only 1 patient (6%) whose disorder was non-ITP. Initial laboratory findings can well predict future diagnosis of ITP. Further studies prospectively evaluating these same diagnostic criteria on another, independent set of patients are necessary. The need for a bone marrow examination was assessed in patients with clinical and laboratory features consistent with ITP; the literature was reviewed. The records of all patients undergoing a bone marrow examination between January 1988 to January 1998 were retrospectively reviewed to determine which were motivated by the suspicion of ITP. Data were collected from hospital and outpatient medical and pathology records. Eighty-six patients with isolated thrombocytopenia (i.e., normal white blood cell count, hemoglobin, peripheral smear and clotting studies) were studied. The bone marrow was consistent with ITP in 82 patients, (i.e., normal or increased megakaryocytes and other hemopoietic lineages normal.) Four patients had decreased megakaryocytes, but all patients responded to corticosteroids. All 86 patients were followed up for a median of 22 months after bone marrow aspiration (range, 2-76 months.) During that time, none of the patients developed features to suggest an alternative diagnosis to ITP. The initial clinical and laboratory findings of 99 patients with acute leukemia were also reviewed; all had features atypical of ITP. These data suggest that routine performance of a bone marrow examination for the diagnosis of ITP is not necessary, provided that a thorough history and physical examination are performed and that the complete blood cell count, peripheral blood smear, and routine clotting studies show no abnormalities apart from thrombocytopenia. The findings of seven prior retrospective studies, two in adults and five in children are consistent with the previous findings. However, the value of marrow investigation in ITP remains unresolved and data from a large prospective study would be helpful. Immune thrombocytopenic purpura Many cases of immune thrombocytopenic purpura (ITP) also known as idiopathic thrombocytopenic purpura, can be left untreated, and spontaneous remission (especially in children) is not uncommon. However, counts under 50,000 are usually monitored with regular blood tests, and those with counts under 10,000 are usually treated, as the risk of serious spontaneous bleeding is high with such low platelet counts. Any patient experiencing severe bleeding symptoms is also usually treated. The threshold for treating ITP has decreased since the 1990s; hematologists recognize that patients rarely spontaneously bleed with platelet counts greater than 10,000, although exceptions to this observation have been documented. Immune thrombocytopenia (ITP) is characterized by a platelet count less than 100 × 10^9/L without anemia or leukopenia. Patients with ITP may be asymptomatic, or they may have mild bleeding like petechiae, purpura, or epistaxis. In rare cases, they may present to the emergency department (ED) with life-threatening bleeding as a result of their thrombocytopenia. The emergency physician should thus be prepared to diagnose ITP and treat the bleeding that can result from it. The diagnosis of ITP requires excluding secondary causes of thrombocytopenia, and in the ED, the bare minimum workup for ITP includes a complete blood count and a peripheral blood smear. The peripheral blood smear should show a small number of large platelets with normal morphology, and there should not be an increased number of schistocytes. Many patients with ITP require no emergent treatment. However, if a patient with suspected ITP presents to the ED with critical hemorrhage, the emergency physician should initiate treatment with a platelet transfusion, corticosteroids, and intravenous immune globulin (IVIG) as soon as possible. For less severe bleeding, platelet transfusions are not recommended, and the treatment consists of corticosteroids by themselves or in conjunction with IVIG. Idiopathic thrombocytopenic purpura(ITP) is a hematologic disorder which causes thrombocytopenia. The diagnosis of ITP is based on the history, physical examination and, complete blood count, and examination of the peripheral smear. The diagnostic criteria of ITP established by the Ministry of Health, Welfare, and Labor in Japan requires the bone marrow examination and the measurement of platelet associated IgG, but those tests are not always necessary according to the guidelines developed by the American Society of Hematology. The appropriate strategies for the diagnosis of ITP need to be established. In this paper, some new examinations which may help the diagnosis of ITP are also demonstrated. Immune thrombocytopenic purpura (ITP), also known as idiopathic thrombocytopenic purpura or immune thrombocytopenia, is a type of thrombocytopenic purpura defined as an isolated low platelet count with a normal bone marrow in the absence of other causes of low platelets. It causes a characteristic red or purple bruise-like rash and an increased tendency to bleed. Two distinct clinical syndromes manifest as an acute condition in children and a chronic condition in adults. The acute form often follows an infection and spontaneously resolves within two months. Chronic immune thrombocytopenia persists longer than six months with a specific cause being unknown. ITP is an autoimmune disease with antibodies detectable against several platelet surface structures. Immune thrombocytopenic purpura (ITP) is a bleeding disorder characterized by isolated thrombocytopenia (platelet count <150,000 u/L), which is not associated with a systemic illness. ITP is reported in approximately 2 per 100,000 adults. The mean age of diagnosis is 50 years. ITP is more common in females of childbearing age and in pregnancy. In adults, the course is more chronic although spontaneous remission can also occur within months of initial diagnosis. A thorough and timely workup of thrombocytopenia is imperative to rule out other differentials of ITP as it is considered a diagnosis of exclusion. Primary care physicians encounter patients who exhibit signs of thrombocytopenia such as petechiae or purpura on a regular basis. A high index of clinical suspicion is required to accurately diagnose ITP and commence the appropriate treatment including glucocorticoids to increase the chances of a favorable prognosis as described by the authors. This report illustrates the importance of a detailed history and physical exam and careful analysis of hematologic parameters when diagnosing ITP. This case demonstrates that even with subtle deviations from typical ITP findings one must promptly reevaluate the diagnosis. This case also highlights the importance of peripheral smear review by an expert in pediatric hematopathology. A previously healthy 10 year-old Asian boy presented with 2 months of easy bruising. Review of systems was negative for any constitutional symptoms. On examination, he appeared well but had numerous large ecchymoses. He had no appreciable lymphadenopathy or splenomegaly. The liver was palpable 1.5 cm below the costal margin. A complete blood count (CBC) showed: platelets = 17 × 109/L, hemoglobin = 128 g/L, white blood cell count = 5.43 × 109/L, and neutrophils = 1.63 × 109/L. A blood smear was reported as normal. Urate was 370 umol/L and lactate dehydrogenase (LDH) was 803 U/L. The child was admitted with a presumptive diagnosis of immune thrombocytopenic purpura (ITP) and treated with intravenous immunoglobulin. The following day, the blood smear was reviewed by a hematopathologist who identified blasts. A bone marrow aspiration (BMA) confirmed the diagnosis of precursor B-cell acute lymphoblastic leukemia. In children presenting with suspected ITP, leukemia should always be considered. A BMA was historically performed on all patients with presumed ITP to rule out leukemia. In 2011, the American Society of Hematology (ASH) stopped recommending routine BMA in patients suspected of having ITP. ASH advises in cases with unusual findings on history, physical examination or CBC, it is reasonable to perform a BMA. Our patient had mild hepatomegaly, which may have qualified him for a BMA. He also had an elevated LDH and urate, which are not listed as criteria for BMA by ASH but were considered atypical for ITP by the clinical team. A literature search did not reveal any primary data assessing these markers. While corticosteroids are a first line treatment in ITP, they must be reserved for when clinicians are confident that the patient does not have leukemia. Steroid administration prior to diagnosing leukemia results in delayed diagnosis and may increase the risk of complications and decrease survival. A review is made of the data in the literature and of the authors' experience regarding the etiology and pathogenesis, diagnosis and therapy of the chronic form of idiopathic thrombocytopenic purpura (ITP). The mechanisms of production (after McMillan et al.) are presented schematically and the five criteria of ITP diagnosis suggested by Karpatkin are discussed: 1) decreased blood platelet count with direct or indirect signs of thrombocytolysis; 2) increased number of megakaryocytes in the bone marrow and/or signs of intramedullary thrombocytolysis; 3) direct or indirect signs of antiplatelet autoantibody presence in the plasma; 4) exclusion of a primary disorder and 5) absence of splenomegaly. The results of corticotherapy, splenectomy, platelet transfusion and immunosuppression (including \"target\" immunosuppressive therapy) in 188 patients with ITP, admitted to the clinic of Hematology--Bucharest between 1966 and 1978, are presented and analysed. It is recommended that pregnant women with thrombocytopenia or a previous diagnosis of ITP should be tested for serum antiplatelet antibodies. A woman with symptomatic thrombocytopenia and an identifiable antiplatelet antibody should be started on therapy for their ITP which may include steroids or IVIG. Fetal blood analysis to determine the platelet count is not generally performed as ITP-induced thrombocytopenia in the fetus is generally less severe than NAIT. Platelet transfusions may be performed in newborns, depending on the degree of thrombocytopenia. It is recommended that neonates be followed with serial platelet counts for the first few days after birth. ITP is associated with a range of conditions, including lymphoma, leukemia, SLE, HIV, and HCV. The clinical presentation is as follows: Acute: Abrupt onset of hemorrhagic complications following a viral illness. Commonly affects children 2–6 years of age, with males and females affected equally. Chronic: Insidious onset that is unrelated to infection. Most often affects adults 20–40 years of age; women are three times more likely to be affected than men. A diagnosis of exclusion, as the test for platelet-associated antibodies is a poor one. Once other causes of thrombocytopenia have been ruled out, a diagnosis can be made on the basis of the history and physical, a CBC, and a peripheral blood smear showing normal RBC morphology. Most patients do not require bone marrow biopsy, which would show ↑ megakaryocytes as the only abnormality. Most patients with acute childhood ITP spontaneously remit, but this is rarely the case in chronic ITP. Diagnosis The diagnosis of ITP is a process of exclusion. First, it has to be determined that there are no blood abnormalities other than a low platelet count, and no physical signs other than bleeding. Then, secondary causes (5–10 percent of suspected ITP cases) should be excluded. Such secondary causes include leukemia, medications (e.g., quinine, heparin), lupus erythematosus, cirrhosis, HIV, hepatitis C, congenital causes, antiphospholipid syndrome, von Willebrand factor deficiency, onyalai and others. All patients with presumed ITP should be tested for HIV and hepatitis C virus, as platelet counts may be corrected by treating the underlying disease. In approximately 2.7 to 5 percent of cases, autoimmune hemolytic anemia and ITP coexist, a condition referred to as Evans syndrome. Immune thrombocytopenic purpura (ITP) is an immune-mediated acquired disease found in both adults and children. It is characterized by transient or persistent decreases in the platelet count. We report a case of ITP detected based on oral hemorrhagic symptoms. The patient was a 79-year-old female with no significant past medical history. She presented with sudden onset of gingival bleeding and hemorrhagic bullae on the buccal mucosa. Gingival bleeding was difficult to control. Laboratory tests revealed severe thrombocytopenia with a platelet count as low as 2000/μL. Under a provisional diagnosis of a hematological disorder, she was referred to a hematologist. A peripheral smear showed normal-sized platelets. A bone marrow examination revealed increased numbers of megakaryocytes without morphologic abnormalities. The patient was diagnosed with ITP and treated with a combination of pulsed steroid therapy and high-dose immunoglobulin therapy. However, her severe thrombocytopenia was refractory to these treatments. Then, a thrombopoietin receptor agonist was begun as a second-line treatment. Her platelets rapidly increased, and no bleeding complications were reported. Because oral symptoms can be one of the initial manifestations of ITP, dentists should be familiar with the clinical appearance of ITP, and attention must be paid to detect and diagnose unidentified cases. Immune thrombocytopenia (ITP) is a diagnosis of exclusion that can resemble other thrombocytopenic disorders. To develop a clinical prediction model (CPM) for the diagnosis of ITP to aid hematogists in investigating patients presenting with undifferentiated thrombocytopenia. We designed a CPM for ITP diagnosis at the time of the initial hematology consultation using penalized logistic regression based on data from patients with thrombocytopenia enrolled in the McMaster ITP registry (n = 523) called the Predict-ITP Tool. The case definition for ITP was a platelet count less than 100 × 1030,000/μL appear not to have increased mortality related to the thrombocytopenia. Pregnancy Anti-platelet autoantibodies in a pregnant woman with ITP will attack the patient's own platelets and will also cross the placenta and react against fetal platelets. Therefore, ITP is a significant cause of fetal and neonatal immune thrombocytopenia. Approximately 10% of newborns affected by ITP will have platelet counts <50,000/uL and 1% to 2% will have a risk of intracerebral hemorrhage comparable to infants with neonatal alloimmune thrombocytopenia (NAIT). No lab test can reliably predict if neonatal thrombocytopenia will occur. The risk of neonatal thrombocytopenia is increased with: Mothers with a history of splenectomy for ITP Mothers who had a previous infant affected with ITP Gestational (maternal) platelet count less than 100,000/uL Disorders Adapted from: The three broad categories of platelet disorders are \"not enough\"; \"dysfunctional\"; and \"too many\". Thrombocytopenia Immune thrombocytopenias (ITP) – formerly known as immune thrombocytopenic purpura and idiopathic thrombocytopenic purpura Splenomegaly Gaucher's disease Familial thrombocytopenia Chemotherapy Babesiosis Dengue fever Onyalai Thrombotic thrombocytopenic purpura HELLP syndrome Hemolytic–uremic syndrome Drug-induced thrombocytopenic purpura (five known drugs – most problematic is heparin-induced thrombocytopenia (HIT) Pregnancy-associated Neonatal alloimmune associated Aplastic anemia Transfusion-associated Pseudothrombocytopenia idiopathic thrombocytopenic purpura Vaccine induced immune thrombocytopenia Gilbert's syndrome Causes Although Evans syndrome seems to be a disorder of immune regulation, the exact pathophysiology is unknown, but a gradual loss of self-tolerance is postulated. Autoantibodies targeted at different antigenic determinants on red cells and platelets are assumed to cause isolated episodes of hemolytic anemia and thrombocytopenia, respectively. Diagnosis The diagnosis of primary Evans syndrome is made upon blood tests to confirm not only hemolytic anemia and immune thrombocytopenic purpura, but also a positive direct antiglobulin test (DAT) and an absence of any known underlying cause. In 27% to 50% of cases there is an associated malignancy or a predisposing autoimmune disease (e.g. systemic lupus erythematosus), it is then common to denote it as secondary Evans syndrome. Other antibodies may occur directed against neutrophils and lymphocytes, and \"immunopancytopenia\" has been suggested as a term for this syndrome. In patients with isolated thrombocytopenia, but without significant dysplasia, diagnosis of idiopathic thrombocytopenic purpura (ITP) rather than myelodysplastic syndrome (MDS) may be taken into account. It is important to make an accurate diagnosis because different treatments are used for ITP and MDS. The purpose of this study was to investigate the clinical and hematologic features of patients who were initially diagnosed as ITP but had cytogenetic abnormalities. We retrospectively reviewed cytogenetic studies of 100 patients who were diagnosed as ITP from 2004 to 2009 at Mokdong Hospital of Ewha Womans University based on clinical features and hematologic studies. Bone marrow pathology was re-evaluated based on 2008 WHO classification. Cytogenetic analysis was performed by 24-48 hr culture of bone marrow aspirates without using mitogens and 20 metaphases were analyzed. Of the 100 patients diagnosed as ITP initially, three patients (3%) had cytogenetic abnormalities. They had no thrombocytopenia-related symptoms and thrombocytopenia was found accidentally. The numbers of megakaryocytes in bone marrow were increased and dysplasia was not found in megakaryocyte, erythroid, and myeloid cell lineages. The proportion of blasts was within normal limits. Clonal chromosomal abnormalities found were der(1;7)(q10;p10), add(9)(q12), or t(7;11)(p22;q12). Presumptive diagnosis of MDS or diagnosis of idiopathic cytopenia of undetermined significance (ICUS) was made according to 2008 WHO classification. During the follow up, disease progression was not found. In patients with suspected ITP, cytogenetic analysis should be done. If specific clonal chromosomal abnormality is found, presumptive diagnosis of MDS has to be considered and close follow up is needed. The Harrington–Hollingsworth experiment was an experiment that established the autoimmune nature of the blood disorder immune thrombocytopenic purpura. It was performed in 1950 by the academic staff of Barnes-Jewish Hospital in St. Louis, Missouri. Experiment The experiment was undertaken in 1950 by William J. Harrington and James W. Hollingsworth, who postulated that in patients with idiopathic thrombocytopenic purpura (ITP), it was a blood factor that caused the destruction of platelets. To test this hypothesis, Harrington received 500 ml of blood from a patient with ITP. Within three hours, his platelets dropped to dangerously low levels and he experienced a seizure. His platelet count remained extremely low for four days, finally returning to normal levels by the fifth day. Bone marrow biopsy from Harrington's sternum demonstrated normal megakaryocytes, the cells necessary for platelet production. Diagnosis. The diagnosis of ITP usually is based on clinical presentation and the platelet count and does not often require a bone marrow examination. If atypical findings are noted, however, marrow examination is indicated to rule out an infiltrative disorder (leukemia) or an aplastic process (aplastic anemia). In ITP, an examination of the bone marrow reveals increased megakaryocytes and normal erythroid and myeloid elements.\nHere is the question:\nHEMATOLOGY: A 33-year-old woman consults for repeated epistaxis, petechiae and ecchymosis. Laboratory tests show thrombocytopenia with a platelet count of 4000 platelets/microliter. The initial presumptive diagnosis is chronic immune thrombocytopenic purpura (ITP). Which of the following statements is FALSE regarding the diagnosis of ITP?\nHere are the potential choices:\n1. The presence of lymphadenopathy or splenomegaly in the physical examination suggests a different diagnosis of ITP.\n2. Bone marrow analysis shows a decreased number of megakaryocytes without other alterations.\n3. Complete blood count shows isolated thrombocytopenia with often large platelets, without anemia unless there is significant bleeding or associated autoimmune hemolysis (Evans syndrome).\n4. The diagnosis of ITP is established by exclusion of other processes causing thrombocytopenia.\n5. The determination of antiplatelet antibodies is not accurate to establish the diagnosis.\nThe correct answer is: 2. Bone marrow analysis shows a decreased number of megakaryocytes without other alterations." }, { "question": "You are a helpful medical expert, and your task is to answer a multi-choice medical question using the relevant documents. Please choose the answer from the provided options. Your responses will be used for research purposes only, so please have a definite answer.\nHere are the relevant documents:\n To show how the evaluation for endometriosis can be included in the routine pelvic ultrasound examination. Stepwise narrated video demonstration of the sonographic evaluation for endometriosis in routine pelvic ultrasound following the recommended sonographic approach published in the 2016 consensus paper by the International Deep Endometriosis Analysis (IDEA) group [1]. Endometriosis is a common and often debilitating gynecological disorder that affects 5-10% of women [2]. The prevalence is even higher among women with symptoms of endometriosis [2], which include chronic pelvic pain, acquired dysmenorrhea, dyspareunia, dyschezia, menorrhagia, abnormal bleeding, and infertility. Approximately 80% of women who have endometriosis have superficial lesions, whereas 20% have deep infiltrating endometriosis (DIE; [3]). Laparoscopy is the gold standard for diagnosing endometriosis, because it allows the diagnosis of all forms of endometriosis and often immediate removal of superficial endometriosis. The removal of DIE is considerably more complicated and usually cannot be completed unless it was diagnosed preoperatively. The technique to diagnose DIE with transvaginal ultrasound (TVUS) was first described in detail in 2009 [4]. Since then, the accuracy of TVUS for the prediction of DIE has been well established in the literature [5-7]. TVUS is widely used as a first-line investigation for women with gynecological symptoms. The inclusion of an assessment for endometriosis in the routine pelvic ultrasound allows earlier diagnosis and better surgical outcomes for all women with DIE. The evaluation for endometriosis in routine pelvic ultrasound based on the IDEA consensus promotes a 4-step dynamic ultrasound approach [1]: (1) routine evaluation of uterus and adnexa with particular attention for sonographic signs of adenomyosis and the presence or absence of endometriomas; (2) evaluation of transvaginal sonographic 'soft markers' such as site-specific tenderness and ovarian mobility; (3) assessment of status of pouch of Douglas using the real-time ultrasound-based \"sliding sign;\" and (4) assessment of DIE nodules in the anterior and posterior compartments, which involves assessment of the bladder, vaginal vault, uterosacral ligaments, and bowel, including rectum, rectosigmoid junction, and sigmoid colon. Because 5-10% of women with DIE also have ureteric endometriosis, it is useful to assess the kidneys. Silent hydronephrosis is easily identified in 50-60% of patients with ureteric involvement. Although it is possible to identify DIE involving the ureters more directly, this requires more advanced skills, and further studies are still needed to better define the accuracy of ureteric DIE detection by TVUS [8-10]. Traditionally, only pathologies of the uterus and ovaries are assessed during a routine pelvic ultrasound. Here we demonstrate that the routine ultrasound examination can easily be extended beyond the uterus and ovaries into the posterior and anterior pelvic compartments to evaluate structural mobility and to look for deep infiltrating endometriotic nodules, wherewith women suffering from DIE can benefit from a preoperative diagnosis and subsequently, a single, well-planned procedure in the hands of a well-prepared team. Adolescent Patients A pelvic examination may be less revealing in an adolescent than in an older woman, particularly if it is the patient’s first examination or if it takes place on an emergency basis. An adolescent who presents with excessive bleeding should have a pelvic examination if she had intercourse, if the results of a pregnancy test are positive, if she has abdominal pain, if she is markedly anemic, or if she is bleeding heavily enough to compromise hemodynamic stability. The pelvic examination occasionally may be deferred in young teenagers who have a classic history of irregular cycles soon after menarche, who have normal hematocrit levels, who deny sexual activity, and who will reliably return for follow-up. A pelvic examination may be deferred in adolescents who present to the office requesting oral contraceptives before the initiation of intercourse or at the patient’s request, even if she has had intercourse. Newer testing methods using DNA amplification techniques To describe the management and the fertility-enhancing potential of surgery in an infertile patient with deep-infiltrating endometriosis and adenomyosis externa. Video case report. Minimally invasive and robotic gynecologic surgery unit of a university hospital. A 31-year-old nulliparous patient with dysmenorrhea, dysuria, dyspareunia, and primary infertility. Bimanual examination, transvaginal ultrasound, and magnetic resonance imaging (MRI) were performed as a comprehensive preoperative workup. The findings were consistent with bladder endometriosis and a 4-cm right pararectal cystic mass suggestive of adenomyosis externa. Laparoscopic excision of all visible endometriosis was performed. A pararectal lesion was found, completely developing in the retroperitoneal spaces, from the right medial pararectal space to the rectovaginal space, reaching the pelvic floor fascia without infiltration of the levator ani muscle. According to Koninckx classification, this kind of lesion corresponds to type III endometriosis or adenomyosis externa. Nerve-sparing eradication of the nodule was performed. The decision to use these techniques was taken with the intention to treat the patient, and not with the aim of testing the procedures performed. Therefore, as a common clinical practice in our institution and for the above reasons, there was no need for consultation of the institutional review board for approval. Improvement of symptoms and spontaneous conception after surgical removal of all endometriotic implants. There were no intraoperative or postoperative complications, and the patient was discharged after 3 days. She discontinued postoperative hormone therapy with gonadotropin-releasing hormone analogue after 3 months because she desired fertility. She conceived spontaneously after 2 months of attempting. She delivered vaginally and had no complications during pregnancy and labor. Neither recurrence of pain symptoms nor voiding or rectal dysfunctions were reported by the patient. In the management of a case of deep endometriosis, the preoperative assessment should be carefully carried out to give the surgeon the most accurate information about the extent of the disease and the patient's main objectives. Imaging techniques such as ultrasound and MRI play a fundamental role along with the clinical evaluation in also detecting lesions that are not visible at first laparoscopic inspection. In this case of a young woman without any detectable fertility issues except for endometriosis, the laparoscopic excision of endometriosis was feasible, safe, and effective in improving the patient's fertility and pain symptoms. The fertility-enhancing potential of complete eradication of pelvic endometriosis, including removal of deep posterior localizations such those presented in this case, has been hypothesized by various investigators. It has been suggested that skilled surgical management for symptomatic deep endometriosis may be followed by a high pregnancy rate, with most pregnancies resulting from postoperative natural conception even in patients with primary infertility. The uterus is typically diffusely enlarged, although usually less than 14 cm in size, and is often soft and tender, particularly at the time of menses. Mobility of the uterus is not restricted, and there is no associated adnexal pathology (48). Adenomyosis is a clinical diagnosis. Imaging studies including pelvic ultrasound or MRI, although helpful, are not definitive. Because of the cost of MRI and negligible improvement in diagnostic accuracy, this study is not recommended routinely. In women with diffuse uterine enlargement and negative pregnancy test results, secondary dysmenorrhea may be attributed to adenomyosis; however, the pathologic confirmation of suspected adenomyosis can be made only at the time of hysterectomy. To demonstrate the technique of laparoscopic approach in a rare case of rectovaginal splenosis with severe dyspareunia and dyschesia. A step-by-step explanation of the patient's condition, diagnosis, surgical technique, and postoperative results (Canadian Task Force classification II-3). Splenosis consists of ectopic functioning splenic tissue that can be located anywhere within the abdomen or pelvis. Fragments are often multiple and range in diameter from a few millimeters to a few centimeters. They are reddish-blue and are sessile or pedunculated. Their appearance can mimic that of neoplasms or endometriosis, which are the main differential diagnoses. Trauma and subsequent splenectomy is the cause in most cases. Splenosis is a benign condition usually found incidentally and is usually asymptomatic. The need for therapy is controversial, and treatment is suggested only in symptomatic cases, primarily those related to pelvic or abdominal lesions, as in our patient. The diagnosis of splenosis in a woman complaining of pelvic pain may present diagnostic difficulties. The splenic tissue has the macroscopic appearance of endometriosis, and its position in the pelvis also may suggest this diagnosis. Where excision of splenosis is considered necessary, the approach should be laparoscopic, unless this is considered too risky owing to the proximity of vital structures. A 40-year-old woman was referred to our department for severe dyspareunia and dyschezia. The gynecologic examination revealed a painfull nodularity on the posterior vaginal cul de sac. Further evaluation with 2- and 3-dimensional ultrasound and magnetic resonance imaging revealed several soft tissue nodules in the pouch of Douglas (POD), which were enhanced on contrast administration. She had undergone a splenectomy 15 years earlier after a car accident. A laparoscopic approach to a rectovaginal nodularity was performed. Under general anesthesia, the patient was placed in the dorsal decubitus position with her arms alongside her body and her legs in abduction. Pneumoperitoneum was achieved using a Veres needle placed at the umbilicus. Four trocars were placed: a 10-mm trocar at the umbilicus for the 0-degree laparoscope; a 5-mm trocar at the right anterosuperior iliac spine; a 5-mm trocar in the midline between the umbilicus and the pubic symphysis, approximately 8 to 10 cm inferior to the umbilical trocar; and a 5-mm trocar at the left anterosuperior iliac spine. The entire pelvis was inspected for endometriotic lesions. In the pelvis, hypervascular and bluish nodules were visible with extension from the POD into the deep rectovaginal space. The macroscopic appearance was atypical for endometriotic implants. The nodularities were carefully dissected and excised, and histological assessment revealed splenic tissue. At the time of this report, the patient had been asymptomatic for 6 months after surgery. Rectovaginal splenosis may mimic endometriosis. The laparoscopic approach to rectovaginal splenosis avoids an abdominal incision, with its associated pain and possible adhesion formation. It also provides a better view for dissection. In this patient, the splenosis was removed by laparoscopy, with no postoperative dyspareunia or dyschesia. The efficacy of medical and surgical treatment of endometriosis and pelvic pain is a source of questions and controversies. Complete resolution of endometriosis is not yet possible but therapy has essentially three main objectives: 1) to reduce pain; 2) to increase the possibility of pregnancy; 3) to delay recurrence for as long as possible. In case of moderate and severe endometriosis, operative laparoscopy must be considered as first line treatment. The mean pregnancy rate of 50% reported in the literature following surgery provides scientific proof that operative treatment should first be undertaken to give our patients the best chance of conceiving naturally. In case of rectovaginal adenomyotic nodules, surgery must also be considered as first line therapy, medical therapy being relatively inefficacious. Careful preoperative examination is mandatory (transrectal sonography, magnetic resonance imaging, bowel barium enema or intravenous pyelography) to evaluate potentially severe complications of the disease. Endometriosis is defined as the presence of endometrial tissue outside the uterine cavity. It generally involves the peritoneum, ovaries and rectovaginal septum. Its characteristic symptoms include dysmenorrhea, pelvic pain, deep dyspareunia and infertility. It may also involve the gastrointestinal tract, urinary tract or extra abdominal sites, giving rise to a wide variety of clinical symptoms such as bloody stools, renal haemorrhage, hemoptysis and pleural effusion during menstruation. Recurrent hemorrhagic ascites secondary to endometriosis is an unusual occurrence, 41 cases have been reported since 1954. Here we report an additional case, in order to draw attention to this condition. A 28 years-old black nulligravida woman was seen for the first time in april 2000 with a chief complaint of infertility. Her past medical history was unremarkable. She had regular menses but associated with severe dysmenorrhea. She also recalled abdominal and pelvic pain for several years. She underwent an ovulation induction with gonadotrophin, which resulted in a progressive increase of pelvic pain. A first laparoscopy was performed, revealing voluminous ascites (10 I). Two years later the ascites recurred spontaneously. Ultrasound examination revealed suspect \"para uterine masses\". A second exploratory laparoscopy showed a voluminous bloody ascites (71), and extensive adhesions. On histologic examination all specimens (peritoneal biopsies) were compatible with endometriosis and ruled out malignancy. Treatment with Gn RH analog was performed and full remission was obtained after 6 months. One year later the ascites recurred again spontaneously, leading to a third laparoscopy in an other medical institution. Histologic examination showed endometrial stromal tissue and fibrous proliferation. Later she became pregnant after in vitro fertilization. In the first trimester of pregnancy, the pelvic ultrasound showed only a small effusion in the pouch of Douglas. Still, the ascites did not progress during pregnancy. The patient was hospitalized from 27 to 33 weeks of gestational age for threatened labor, but she finally had a normal vaginal delivery at 36 weeks of gestational age. Four months later, she had no complaint, but the pelvic ultrasound showed the recurrence of the ascites. She will have a drainage. The future treatement will consists of GnRH analog for about six months, which will be relayed by a long term progestative therapy. A diagnosis of endometriosis should always be considered in middle-age women who presents with bloody ascites. Long follow-up is advisable for patients who undergo conservative treatment because of thehigh risk of recurrence. Although gastrointestinal endometriosis is an uncommon and often unexpected finding, the best treatment requires removal of all endometriotic lesions. The purpose of our study was to report our experience with the diagnosis and treatment of bowel endometriosis. From January 1997 to January 2004, 13 patients (mean 35.7y ; range 21-55y) were operated for bowel endometriosis. We noted: age, history of endometriosis, previous pregnancies, preoperative investigations and symptoms, operative procedure and intraoperative findings. Follow-up varied between one month postoperative examination and seven years. Presenting symptoms of the cases were: acute appendicitis (3), dysmenorrhoea (7), constipation (6), pelvic pain (2), rectal bleeding (3) and dyspareunia (2). Operative management was performed in accordance with the anatomical distribution. Seven patients had a history of previous operations and multifocal involvement was present in 61.5% of cases. At a median follow-up of 12.2 months, 83.3% had complete relief of their initial complaints, with only one reoperation needed. The pregnancy rate after surgery was 66.6%. Preoperative tests were: ultrasound for ovarian endometriomas, coloscopy, barium enema, vaginal palpation for detecting rectovaginal involvement, MRI and CT scan. These tests predicted the extension of endometriotic process correctly in 50% of the cases. Endometriosis of the sigmoid and rectum is rare but can give rise to severe gastrointestinal and pelvic symptoms. Preoperative investigations are not infallible in predicting the extent of the disease, sometimes placing the surgeon before a dilemma, because it involves mostly young women in the reproductive phase of life. The colorectal surgeon, therefore, should seek the advice of an experienced gynaecologist and vice versa. Removal of all endometriotic lesions is mandatory for obtaining an optimal relief of symptoms. To describe a multidisciplinary approach for the resection of deeply infiltrative endometriosis using the robotic platform. A technical video showing a step-by-step approach for the resection of deeply infiltrative endometriosis (Canadian Task Force classification level III). Institutional review board approval was not required for this study. There is considerable involvement of the bowel and bladder with deeply infiltrative endometriosis [1-3]. The need for operative procedures involving multiple organs while performing a complete resection is common. The benefits of minimally invasive surgery for a gynecologic pathology have been documented in numerous studies. Patients had fewer medical and surgical complications postoperatively, better cosmesis, and better quality of life [4-6]. We believe that deeply infiltrative endometriosis does not preclude patients from having a minimally invasive resection procedure. In this video, we describe how the robotic platform was used for a seamless transition between surgical specialties including gynecology, colorectal, and urology to ensure complete resection of endometriosis lesions involving multiple organs. A 47-year-old woman with a 4-year history of severe pelvic pain, dysuria, dyspareunia, dyschezia, and dysmenorrhea failing multiple medical therapies presented to our clinic to discuss surgical options. After thorough counseling, the decision was made to proceed with definitive surgical management. Postoperatively, the patient was admitted for 2 days of postoperative inpatient care. After meeting all immediate postoperative milestones, she was discharged with an indwelling Foley catheter and instructed to follow up in the clinic with all 3 surgical specialties. At the 1-week interval, she was seen by the urology team; her indwelling catheter was removed after a cystoscopy was performed documenting adequate healing. Two weeks postoperatively, the patient was seen by the gynecology and colorectal teams and was noted to be healing adequately from the procedure. Her six-week visit was also unremarkable. She continued to follow up with the gynecology team for her yearly well-woman examinations and has been symptom free for 2 years after the surgery. She takes norethindrone daily to minimize recurrence. Preoperative pelvic magnetic resonance imaging (MRI) showed bladder endometriosis and extensive rectovaginal endometriosis. We describe the multidisciplinary approach used for surgery and the procedures performed by each specialty. The urology team performed a cystoscopy preoperatively to assess for full-thickness erosions and the location of those lesions in that event. The urology team also reviewed the magnetic resonance images with the radiology team, and the endometriosis lesions were suspected to be close to the bladder trigone, keeping in mind that this finding could be overestimated given that the bladder was deflated at the time the imaging was obtained. Accordingly, at the time of surgery, the decision was made to proceed with cystoscopy and the placement of ureteral stents as a prophylactic measure. An intentional cystotomy and resection of the bladder section involved with endometriosis were performed followed by watertight closure. The trigone area of the bladder was not involved, and ureteral reimplantation was not needed in this case. The gynecology team operated second and performed an extensive dissection of the retroperitoneal space with the development of the pararectal and paravesical spaces. They also ligated the uterine artery at its origin followed by dissection of the uterovesical space, effectively reflecting the bladder off of the lower uterine segment. At this point, they proceeded with a total hysterectomy, and the specimen was removed from the pelvis through the vaginal cuff. Preoperatively, the colorectal surgeon ordered a colonoscopy to determine if full-thickness erosions were present and reviewed the magnetic resonance images with the radiology team. Based on the MRI and colonoscopy, all patients are counseled and consented for the possibility of a low anterior resection and loop ileostomy to protect the anastomosis. Based on the understanding that colorectal and gynecologic surgeries have a different approach when dissecting the pararectal space at our institution, a discussion between the 2 teams is initiated at the multidisciplinary session for surgery planning. In the case we present, the colorectal surgeon opted for the removal of the uterus before his dissection was initiated given that he dissects this space presacrally and not retroperitoneally like the gynecology counterpart. He would also benefit from the extra space for dissection with the uterus out of the pelvis. The colorectal part of the case was initiated by mobilization of the rectum and dissecting the obliterated rectovaginal space. The presacral space was then opened followed by mobilization of the rectosigmoid from its attachment. The case was concluded with full transection and reanastomosis of the rectum section involved with endometriosis. The specimen was also removed from the pelvis through the vaginal cuff. Complete resection of deeply infiltrative endometriosis spanning beyond the scope of 1 surgical specialty. No immediate intraoperative, perioperative, or long-term complications from surgery. Complete resolution of endometriosis symptoms. We encourage collaborative care for planning and performing comprehensive and safe resection of deeply infiltrative endometriosis. 2. Loose stools are rarely present without the use of laxatives, and there are insufficient criteria for IBS. 3. a Criteria fulfilled for the last 3 months with symptom onset at least 6 months prior to diagnosis. From Drossman DA, Corazziari E, Talley NJ, et al., eds. Rome III: the functional gastrointestinal disorders. 3nd ed. McLean, VA: Degnon Associates, 2006:885–897, Appendix A, with permission. Table 28.6 Functional Defecation Disorders 1. The patient must satisfy diagnostic criteria for functional constipation (Table 28.5) 2. During repeated attempts to defecate must have at least two of the following: a. Evidence of impaired evacuation, based on balloon expulsion test or imaging b. Inappropriate contraction of the pelvic floor muscles (i.e., anal sphincter or puborectalis) or less than 20% relaxation of basal resting sphincter pressure by manometry, imaging, or EMG c. 27 year old Melinda presents to you with increasingly severe dysmenorrhoea. She has been using condoms for contraception and is no longer able to control the pain with the anti inflammatory tablets you suggested at the last consultation 6 months ago. She also complains of the recent development of deep dyspareunia. She has been in her current relationship for the last 5 years. Examination illicits similar pain and tenderness to that which she feels during intercourse. A Pap smear and STD screen are normal. You refer her to a gynaecologist who undertakes a laparoscopy on Melinda. What is seen? Ovarian adhesions secondary to endometriosis (Figure 1). Classic endometriosis (Figure 2). The aim of the authors is to show the data for the reception diagnosis, age, histological results and the conduct after the performed diagnostic hysteroscopies in Gynecological clinic of UMBAL-Pleven. For the fulfillment of this aim was made a prospective study for 7 years' period: from 01/01/1997 to 31/01/2003. The objects of observation were 74 women of age from 16 to 65 years, with performed hysteroscopies for gynecologic complaints. There were performed 74 diagnostic hysteroscopies for the studied period. The hysteroscopic findings were 20 cases with endometrial polyposis, 14--submucosal myoamatic nodes, deforming the uterine cavity, 4--cervical polyp, 19--increased endometrium, 9--Asherman syndrome, 1--bicomous uterus, 1--a suspected section for endometrial carcinoma and 6 cases without pathologic findings. There were performed 59 trial abrasions and the removed materials were sent for histological examination The performed comparative analysis between the hysteroscopic presentation and histological findings showed a coincidence of the diagnosis. It was made the conclusion, that the hysteroscopy is an easy, accessible and inexpensive diagnostic method, which must take its place as one of the basic contemporary diagnostic methods in gynecology. To describe how a hydronephrosis can lead to a difficult differential diagnosis between endometriosis and retroperitoneal fibrosis. Case report. Department of Obstetrics and Gynecology, Sacro Cuore Don Calabria General Hospital, Negrar, Verona, Italy. The history of a 34-year-old woman revealed the appearance of hydroureteronephrosis on the right side at the 35th week of pregnancy. She had an magnetic resonance imaging scan and was diagnosed with a spread retroperitoneal fibrosis. After 2 months, the patient reported the occurrence of pelvic pain, dyspareunia and dysmenorrhea. She was treated with corticosteroids and tamoxifen with no results. Laparoscopic surgery. A complete retroperitoneal extirpation was done of an endometriotic nodule of the right broad ligament, near the right ureter (without stenosis). Reduction of pelvic pain. She noticed an important decrease of pain. The cause of hydronephrosis could be a physiologic hydroureteronephrosis, which is the most common cause of dilatation of the urinary tract in pregnancy. The pain symptoms of the patients seemed to be linked to endometriosis and not to retroperitoneal fibrosis. Magnetic resonance imaging sometimes does not enable a correct diagnosis between these two pathologies. Fertile women with suspected fibrosis should undergo a diagnostic laparoscopy by an expert surgeon in retroperitoneal surgery. Pelvic congestion affects women of reproductive age. Typical symptoms include bilateral lower abdominal and back pain that is increased with standing for long periods, secondary dysmenorrhea, dyspareunia, abnormal uterine bleeding, chronic fatigue, and irritable bowel symptoms (97). Pain usually begins with ovulation and lasts until the end of menses. The uterus is often bulky, and the ovaries are enlarged with multiple functional cysts. The uterus, parametria, and uterosacral ligaments are tender. Transuterine venography is the primary method for diagnosis, although other modalities, such as pelvic ultrasound, magnetic resonance imaging, and laparoscopy, may disclose varicosities (93). Because of the cost and possible side effects of treatment, further management should be based on related symptoms and not simply on the presence of varicosities. Heidi is an almost 6-year-old girl presenting to your primary care office to establish care because of a change in insurance status. You review her previous medical records before seeing her.She was diagnosed with autism spectrum disorder (ASD) when she was 25 months old. Her parents were initially concerned about language delay. Through a comprehensive evaluation by a developmental-behavioral pediatrician and a child psychologist, including administration of the Bayley Scales of Infant and Toddler Development and the Autism Diagnostic Observation Schedule, she was diagnosed with ASD. Her cognitive skills were reported to be within the average range. Soon after the diagnosis, she began receiving 20 hours of applied behavioral analysis (ABA) per week, as well as music therapy, occupational therapy, and a toddler playgroup through early intervention. Four months after the initial diagnosis, her parents reported that she had started making small improvements in her behavior, used more eye contact, and seemed more socially engaged. Approximately 1 year after the diagnosis, she was receiving 6 hours of ABA per week in addition to starting preschool with an Individualized Education Program. She reportedly continued to show progress with social communication and pretend play skills.At the age of 3 years, 8 months, neuropsychological testing was completed at her parent's request, and her cognitive skills and adaptive skills were reported to be within the average range. She continued to meet the diagnostic criteria for ASD, given her challenges with social awareness, communication, delayed play skills, decreased flexibility, and tendency toward subtle self-direction. She continued to receive speech/language therapy and attended an integrated preschool program within the school district because of her social and communication challenges. She also received ABA 4 hours weekly at home.During your first visit with Heidi, her parents report that she has continued to make progress in all areas, including social skills. She can engage in imaginary play with her friends, ask strangers questions, and comprehend the perspective of others, and she is no longer \"rigid.\" She is not receiving services outside of school and is only receiving once weekly speech/language therapy in school. Her parents no longer believe that she meets the criteria for ASD, and they are interested in further evaluation. Her parents ask if it is possible to \"lose\" the diagnosis of ASD. They also want to know if there are other things to be concerned about for her future. How do you respond? Imaging Tests The role of imaging techniques in studying female urinary incontinence is not yet established. Researchers are evaluating the potential roles of ultrasonography, fluoroscopy, functional neuroimaging, and magnetic resonance imaging (MRI). These tests should not be done routinely but are useful in certain conditions. If the patient’s symptoms (easily remembered by the three Ds: dysuria, dribbling, and dyspareunia) or examination suggests a urethral diverticulum, MRI is the test of choice (44).\nHere is the question:\nGYNECOLOGY AND OBSTETRICS: A 27-year-old woman referred to the gynecology office for evaluation referring dyspareunia for about 8 months, along with dyschezia and occasional rectorrhagia coinciding with menstruation for 3-4 months. She also reports dysmenorrhea for years, which she controls well with ibuprofen. She has been trying to get pregnant for 16 months without success. In the gynecological examination she only has pain when pressing on the posterior vaginal fornix. Which test do you consider would allow you to reach a diagnosis of certainty of her pathology?\nHere are the potential choices:\n1. Transvaginal ultrasound.\n2. Diagnostic laparoscopy.\n3. Magnetic resonance imaging.\n4. Colonoscopy.\nThe correct answer is: ", "gold_answer": "2 Diagnostic laparoscopy.", "prediction": "You are a helpful medical expert, and your task is to answer a multi-choice medical question using the relevant documents. Please choose the answer from the provided options. Your responses will be used for research purposes only, so please have a definite answer.\nHere are the relevant documents:\n To show how the evaluation for endometriosis can be included in the routine pelvic ultrasound examination. Stepwise narrated video demonstration of the sonographic evaluation for endometriosis in routine pelvic ultrasound following the recommended sonographic approach published in the 2016 consensus paper by the International Deep Endometriosis Analysis (IDEA) group [1]. Endometriosis is a common and often debilitating gynecological disorder that affects 5-10% of women [2]. The prevalence is even higher among women with symptoms of endometriosis [2], which include chronic pelvic pain, acquired dysmenorrhea, dyspareunia, dyschezia, menorrhagia, abnormal bleeding, and infertility. Approximately 80% of women who have endometriosis have superficial lesions, whereas 20% have deep infiltrating endometriosis (DIE; [3]). Laparoscopy is the gold standard for diagnosing endometriosis, because it allows the diagnosis of all forms of endometriosis and often immediate removal of superficial endometriosis. The removal of DIE is considerably more complicated and usually cannot be completed unless it was diagnosed preoperatively. The technique to diagnose DIE with transvaginal ultrasound (TVUS) was first described in detail in 2009 [4]. Since then, the accuracy of TVUS for the prediction of DIE has been well established in the literature [5-7]. TVUS is widely used as a first-line investigation for women with gynecological symptoms. The inclusion of an assessment for endometriosis in the routine pelvic ultrasound allows earlier diagnosis and better surgical outcomes for all women with DIE. The evaluation for endometriosis in routine pelvic ultrasound based on the IDEA consensus promotes a 4-step dynamic ultrasound approach [1]: (1) routine evaluation of uterus and adnexa with particular attention for sonographic signs of adenomyosis and the presence or absence of endometriomas; (2) evaluation of transvaginal sonographic 'soft markers' such as site-specific tenderness and ovarian mobility; (3) assessment of status of pouch of Douglas using the real-time ultrasound-based \"sliding sign;\" and (4) assessment of DIE nodules in the anterior and posterior compartments, which involves assessment of the bladder, vaginal vault, uterosacral ligaments, and bowel, including rectum, rectosigmoid junction, and sigmoid colon. Because 5-10% of women with DIE also have ureteric endometriosis, it is useful to assess the kidneys. Silent hydronephrosis is easily identified in 50-60% of patients with ureteric involvement. Although it is possible to identify DIE involving the ureters more directly, this requires more advanced skills, and further studies are still needed to better define the accuracy of ureteric DIE detection by TVUS [8-10]. Traditionally, only pathologies of the uterus and ovaries are assessed during a routine pelvic ultrasound. Here we demonstrate that the routine ultrasound examination can easily be extended beyond the uterus and ovaries into the posterior and anterior pelvic compartments to evaluate structural mobility and to look for deep infiltrating endometriotic nodules, wherewith women suffering from DIE can benefit from a preoperative diagnosis and subsequently, a single, well-planned procedure in the hands of a well-prepared team. Adolescent Patients A pelvic examination may be less revealing in an adolescent than in an older woman, particularly if it is the patient’s first examination or if it takes place on an emergency basis. An adolescent who presents with excessive bleeding should have a pelvic examination if she had intercourse, if the results of a pregnancy test are positive, if she has abdominal pain, if she is markedly anemic, or if she is bleeding heavily enough to compromise hemodynamic stability. The pelvic examination occasionally may be deferred in young teenagers who have a classic history of irregular cycles soon after menarche, who have normal hematocrit levels, who deny sexual activity, and who will reliably return for follow-up. A pelvic examination may be deferred in adolescents who present to the office requesting oral contraceptives before the initiation of intercourse or at the patient’s request, even if she has had intercourse. Newer testing methods using DNA amplification techniques To describe the management and the fertility-enhancing potential of surgery in an infertile patient with deep-infiltrating endometriosis and adenomyosis externa. Video case report. Minimally invasive and robotic gynecologic surgery unit of a university hospital. A 31-year-old nulliparous patient with dysmenorrhea, dysuria, dyspareunia, and primary infertility. Bimanual examination, transvaginal ultrasound, and magnetic resonance imaging (MRI) were performed as a comprehensive preoperative workup. The findings were consistent with bladder endometriosis and a 4-cm right pararectal cystic mass suggestive of adenomyosis externa. Laparoscopic excision of all visible endometriosis was performed. A pararectal lesion was found, completely developing in the retroperitoneal spaces, from the right medial pararectal space to the rectovaginal space, reaching the pelvic floor fascia without infiltration of the levator ani muscle. According to Koninckx classification, this kind of lesion corresponds to type III endometriosis or adenomyosis externa. Nerve-sparing eradication of the nodule was performed. The decision to use these techniques was taken with the intention to treat the patient, and not with the aim of testing the procedures performed. Therefore, as a common clinical practice in our institution and for the above reasons, there was no need for consultation of the institutional review board for approval. Improvement of symptoms and spontaneous conception after surgical removal of all endometriotic implants. There were no intraoperative or postoperative complications, and the patient was discharged after 3 days. She discontinued postoperative hormone therapy with gonadotropin-releasing hormone analogue after 3 months because she desired fertility. She conceived spontaneously after 2 months of attempting. She delivered vaginally and had no complications during pregnancy and labor. Neither recurrence of pain symptoms nor voiding or rectal dysfunctions were reported by the patient. In the management of a case of deep endometriosis, the preoperative assessment should be carefully carried out to give the surgeon the most accurate information about the extent of the disease and the patient's main objectives. Imaging techniques such as ultrasound and MRI play a fundamental role along with the clinical evaluation in also detecting lesions that are not visible at first laparoscopic inspection. In this case of a young woman without any detectable fertility issues except for endometriosis, the laparoscopic excision of endometriosis was feasible, safe, and effective in improving the patient's fertility and pain symptoms. The fertility-enhancing potential of complete eradication of pelvic endometriosis, including removal of deep posterior localizations such those presented in this case, has been hypothesized by various investigators. It has been suggested that skilled surgical management for symptomatic deep endometriosis may be followed by a high pregnancy rate, with most pregnancies resulting from postoperative natural conception even in patients with primary infertility. The uterus is typically diffusely enlarged, although usually less than 14 cm in size, and is often soft and tender, particularly at the time of menses. Mobility of the uterus is not restricted, and there is no associated adnexal pathology (48). Adenomyosis is a clinical diagnosis. Imaging studies including pelvic ultrasound or MRI, although helpful, are not definitive. Because of the cost of MRI and negligible improvement in diagnostic accuracy, this study is not recommended routinely. In women with diffuse uterine enlargement and negative pregnancy test results, secondary dysmenorrhea may be attributed to adenomyosis; however, the pathologic confirmation of suspected adenomyosis can be made only at the time of hysterectomy. To demonstrate the technique of laparoscopic approach in a rare case of rectovaginal splenosis with severe dyspareunia and dyschesia. A step-by-step explanation of the patient's condition, diagnosis, surgical technique, and postoperative results (Canadian Task Force classification II-3). Splenosis consists of ectopic functioning splenic tissue that can be located anywhere within the abdomen or pelvis. Fragments are often multiple and range in diameter from a few millimeters to a few centimeters. They are reddish-blue and are sessile or pedunculated. Their appearance can mimic that of neoplasms or endometriosis, which are the main differential diagnoses. Trauma and subsequent splenectomy is the cause in most cases. Splenosis is a benign condition usually found incidentally and is usually asymptomatic. The need for therapy is controversial, and treatment is suggested only in symptomatic cases, primarily those related to pelvic or abdominal lesions, as in our patient. The diagnosis of splenosis in a woman complaining of pelvic pain may present diagnostic difficulties. The splenic tissue has the macroscopic appearance of endometriosis, and its position in the pelvis also may suggest this diagnosis. Where excision of splenosis is considered necessary, the approach should be laparoscopic, unless this is considered too risky owing to the proximity of vital structures. A 40-year-old woman was referred to our department for severe dyspareunia and dyschezia. The gynecologic examination revealed a painfull nodularity on the posterior vaginal cul de sac. Further evaluation with 2- and 3-dimensional ultrasound and magnetic resonance imaging revealed several soft tissue nodules in the pouch of Douglas (POD), which were enhanced on contrast administration. She had undergone a splenectomy 15 years earlier after a car accident. A laparoscopic approach to a rectovaginal nodularity was performed. Under general anesthesia, the patient was placed in the dorsal decubitus position with her arms alongside her body and her legs in abduction. Pneumoperitoneum was achieved using a Veres needle placed at the umbilicus. Four trocars were placed: a 10-mm trocar at the umbilicus for the 0-degree laparoscope; a 5-mm trocar at the right anterosuperior iliac spine; a 5-mm trocar in the midline between the umbilicus and the pubic symphysis, approximately 8 to 10 cm inferior to the umbilical trocar; and a 5-mm trocar at the left anterosuperior iliac spine. The entire pelvis was inspected for endometriotic lesions. In the pelvis, hypervascular and bluish nodules were visible with extension from the POD into the deep rectovaginal space. The macroscopic appearance was atypical for endometriotic implants. The nodularities were carefully dissected and excised, and histological assessment revealed splenic tissue. At the time of this report, the patient had been asymptomatic for 6 months after surgery. Rectovaginal splenosis may mimic endometriosis. The laparoscopic approach to rectovaginal splenosis avoids an abdominal incision, with its associated pain and possible adhesion formation. It also provides a better view for dissection. In this patient, the splenosis was removed by laparoscopy, with no postoperative dyspareunia or dyschesia. The efficacy of medical and surgical treatment of endometriosis and pelvic pain is a source of questions and controversies. Complete resolution of endometriosis is not yet possible but therapy has essentially three main objectives: 1) to reduce pain; 2) to increase the possibility of pregnancy; 3) to delay recurrence for as long as possible. In case of moderate and severe endometriosis, operative laparoscopy must be considered as first line treatment. The mean pregnancy rate of 50% reported in the literature following surgery provides scientific proof that operative treatment should first be undertaken to give our patients the best chance of conceiving naturally. In case of rectovaginal adenomyotic nodules, surgery must also be considered as first line therapy, medical therapy being relatively inefficacious. Careful preoperative examination is mandatory (transrectal sonography, magnetic resonance imaging, bowel barium enema or intravenous pyelography) to evaluate potentially severe complications of the disease. Endometriosis is defined as the presence of endometrial tissue outside the uterine cavity. It generally involves the peritoneum, ovaries and rectovaginal septum. Its characteristic symptoms include dysmenorrhea, pelvic pain, deep dyspareunia and infertility. It may also involve the gastrointestinal tract, urinary tract or extra abdominal sites, giving rise to a wide variety of clinical symptoms such as bloody stools, renal haemorrhage, hemoptysis and pleural effusion during menstruation. Recurrent hemorrhagic ascites secondary to endometriosis is an unusual occurrence, 41 cases have been reported since 1954. Here we report an additional case, in order to draw attention to this condition. A 28 years-old black nulligravida woman was seen for the first time in april 2000 with a chief complaint of infertility. Her past medical history was unremarkable. She had regular menses but associated with severe dysmenorrhea. She also recalled abdominal and pelvic pain for several years. She underwent an ovulation induction with gonadotrophin, which resulted in a progressive increase of pelvic pain. A first laparoscopy was performed, revealing voluminous ascites (10 I). Two years later the ascites recurred spontaneously. Ultrasound examination revealed suspect \"para uterine masses\". A second exploratory laparoscopy showed a voluminous bloody ascites (71), and extensive adhesions. On histologic examination all specimens (peritoneal biopsies) were compatible with endometriosis and ruled out malignancy. Treatment with Gn RH analog was performed and full remission was obtained after 6 months. One year later the ascites recurred again spontaneously, leading to a third laparoscopy in an other medical institution. Histologic examination showed endometrial stromal tissue and fibrous proliferation. Later she became pregnant after in vitro fertilization. In the first trimester of pregnancy, the pelvic ultrasound showed only a small effusion in the pouch of Douglas. Still, the ascites did not progress during pregnancy. The patient was hospitalized from 27 to 33 weeks of gestational age for threatened labor, but she finally had a normal vaginal delivery at 36 weeks of gestational age. Four months later, she had no complaint, but the pelvic ultrasound showed the recurrence of the ascites. She will have a drainage. The future treatement will consists of GnRH analog for about six months, which will be relayed by a long term progestative therapy. A diagnosis of endometriosis should always be considered in middle-age women who presents with bloody ascites. Long follow-up is advisable for patients who undergo conservative treatment because of thehigh risk of recurrence. Although gastrointestinal endometriosis is an uncommon and often unexpected finding, the best treatment requires removal of all endometriotic lesions. The purpose of our study was to report our experience with the diagnosis and treatment of bowel endometriosis. From January 1997 to January 2004, 13 patients (mean 35.7y ; range 21-55y) were operated for bowel endometriosis. We noted: age, history of endometriosis, previous pregnancies, preoperative investigations and symptoms, operative procedure and intraoperative findings. Follow-up varied between one month postoperative examination and seven years. Presenting symptoms of the cases were: acute appendicitis (3), dysmenorrhoea (7), constipation (6), pelvic pain (2), rectal bleeding (3) and dyspareunia (2). Operative management was performed in accordance with the anatomical distribution. Seven patients had a history of previous operations and multifocal involvement was present in 61.5% of cases. At a median follow-up of 12.2 months, 83.3% had complete relief of their initial complaints, with only one reoperation needed. The pregnancy rate after surgery was 66.6%. Preoperative tests were: ultrasound for ovarian endometriomas, coloscopy, barium enema, vaginal palpation for detecting rectovaginal involvement, MRI and CT scan. These tests predicted the extension of endometriotic process correctly in 50% of the cases. Endometriosis of the sigmoid and rectum is rare but can give rise to severe gastrointestinal and pelvic symptoms. Preoperative investigations are not infallible in predicting the extent of the disease, sometimes placing the surgeon before a dilemma, because it involves mostly young women in the reproductive phase of life. The colorectal surgeon, therefore, should seek the advice of an experienced gynaecologist and vice versa. Removal of all endometriotic lesions is mandatory for obtaining an optimal relief of symptoms. To describe a multidisciplinary approach for the resection of deeply infiltrative endometriosis using the robotic platform. A technical video showing a step-by-step approach for the resection of deeply infiltrative endometriosis (Canadian Task Force classification level III). Institutional review board approval was not required for this study. There is considerable involvement of the bowel and bladder with deeply infiltrative endometriosis [1-3]. The need for operative procedures involving multiple organs while performing a complete resection is common. The benefits of minimally invasive surgery for a gynecologic pathology have been documented in numerous studies. Patients had fewer medical and surgical complications postoperatively, better cosmesis, and better quality of life [4-6]. We believe that deeply infiltrative endometriosis does not preclude patients from having a minimally invasive resection procedure. In this video, we describe how the robotic platform was used for a seamless transition between surgical specialties including gynecology, colorectal, and urology to ensure complete resection of endometriosis lesions involving multiple organs. A 47-year-old woman with a 4-year history of severe pelvic pain, dysuria, dyspareunia, dyschezia, and dysmenorrhea failing multiple medical therapies presented to our clinic to discuss surgical options. After thorough counseling, the decision was made to proceed with definitive surgical management. Postoperatively, the patient was admitted for 2 days of postoperative inpatient care. After meeting all immediate postoperative milestones, she was discharged with an indwelling Foley catheter and instructed to follow up in the clinic with all 3 surgical specialties. At the 1-week interval, she was seen by the urology team; her indwelling catheter was removed after a cystoscopy was performed documenting adequate healing. Two weeks postoperatively, the patient was seen by the gynecology and colorectal teams and was noted to be healing adequately from the procedure. Her six-week visit was also unremarkable. She continued to follow up with the gynecology team for her yearly well-woman examinations and has been symptom free for 2 years after the surgery. She takes norethindrone daily to minimize recurrence. Preoperative pelvic magnetic resonance imaging (MRI) showed bladder endometriosis and extensive rectovaginal endometriosis. We describe the multidisciplinary approach used for surgery and the procedures performed by each specialty. The urology team performed a cystoscopy preoperatively to assess for full-thickness erosions and the location of those lesions in that event. The urology team also reviewed the magnetic resonance images with the radiology team, and the endometriosis lesions were suspected to be close to the bladder trigone, keeping in mind that this finding could be overestimated given that the bladder was deflated at the time the imaging was obtained. Accordingly, at the time of surgery, the decision was made to proceed with cystoscopy and the placement of ureteral stents as a prophylactic measure. An intentional cystotomy and resection of the bladder section involved with endometriosis were performed followed by watertight closure. The trigone area of the bladder was not involved, and ureteral reimplantation was not needed in this case. The gynecology team operated second and performed an extensive dissection of the retroperitoneal space with the development of the pararectal and paravesical spaces. They also ligated the uterine artery at its origin followed by dissection of the uterovesical space, effectively reflecting the bladder off of the lower uterine segment. At this point, they proceeded with a total hysterectomy, and the specimen was removed from the pelvis through the vaginal cuff. Preoperatively, the colorectal surgeon ordered a colonoscopy to determine if full-thickness erosions were present and reviewed the magnetic resonance images with the radiology team. Based on the MRI and colonoscopy, all patients are counseled and consented for the possibility of a low anterior resection and loop ileostomy to protect the anastomosis. Based on the understanding that colorectal and gynecologic surgeries have a different approach when dissecting the pararectal space at our institution, a discussion between the 2 teams is initiated at the multidisciplinary session for surgery planning. In the case we present, the colorectal surgeon opted for the removal of the uterus before his dissection was initiated given that he dissects this space presacrally and not retroperitoneally like the gynecology counterpart. He would also benefit from the extra space for dissection with the uterus out of the pelvis. The colorectal part of the case was initiated by mobilization of the rectum and dissecting the obliterated rectovaginal space. The presacral space was then opened followed by mobilization of the rectosigmoid from its attachment. The case was concluded with full transection and reanastomosis of the rectum section involved with endometriosis. The specimen was also removed from the pelvis through the vaginal cuff. Complete resection of deeply infiltrative endometriosis spanning beyond the scope of 1 surgical specialty. No immediate intraoperative, perioperative, or long-term complications from surgery. Complete resolution of endometriosis symptoms. We encourage collaborative care for planning and performing comprehensive and safe resection of deeply infiltrative endometriosis. 2. Loose stools are rarely present without the use of laxatives, and there are insufficient criteria for IBS. 3. a Criteria fulfilled for the last 3 months with symptom onset at least 6 months prior to diagnosis. From Drossman DA, Corazziari E, Talley NJ, et al., eds. Rome III: the functional gastrointestinal disorders. 3nd ed. McLean, VA: Degnon Associates, 2006:885–897, Appendix A, with permission. Table 28.6 Functional Defecation Disorders 1. The patient must satisfy diagnostic criteria for functional constipation (Table 28.5) 2. During repeated attempts to defecate must have at least two of the following: a. Evidence of impaired evacuation, based on balloon expulsion test or imaging b. Inappropriate contraction of the pelvic floor muscles (i.e., anal sphincter or puborectalis) or less than 20% relaxation of basal resting sphincter pressure by manometry, imaging, or EMG c. 27 year old Melinda presents to you with increasingly severe dysmenorrhoea. She has been using condoms for contraception and is no longer able to control the pain with the anti inflammatory tablets you suggested at the last consultation 6 months ago. She also complains of the recent development of deep dyspareunia. She has been in her current relationship for the last 5 years. Examination illicits similar pain and tenderness to that which she feels during intercourse. A Pap smear and STD screen are normal. You refer her to a gynaecologist who undertakes a laparoscopy on Melinda. What is seen? Ovarian adhesions secondary to endometriosis (Figure 1). Classic endometriosis (Figure 2). The aim of the authors is to show the data for the reception diagnosis, age, histological results and the conduct after the performed diagnostic hysteroscopies in Gynecological clinic of UMBAL-Pleven. For the fulfillment of this aim was made a prospective study for 7 years' period: from 01/01/1997 to 31/01/2003. The objects of observation were 74 women of age from 16 to 65 years, with performed hysteroscopies for gynecologic complaints. There were performed 74 diagnostic hysteroscopies for the studied period. The hysteroscopic findings were 20 cases with endometrial polyposis, 14--submucosal myoamatic nodes, deforming the uterine cavity, 4--cervical polyp, 19--increased endometrium, 9--Asherman syndrome, 1--bicomous uterus, 1--a suspected section for endometrial carcinoma and 6 cases without pathologic findings. There were performed 59 trial abrasions and the removed materials were sent for histological examination The performed comparative analysis between the hysteroscopic presentation and histological findings showed a coincidence of the diagnosis. It was made the conclusion, that the hysteroscopy is an easy, accessible and inexpensive diagnostic method, which must take its place as one of the basic contemporary diagnostic methods in gynecology. To describe how a hydronephrosis can lead to a difficult differential diagnosis between endometriosis and retroperitoneal fibrosis. Case report. Department of Obstetrics and Gynecology, Sacro Cuore Don Calabria General Hospital, Negrar, Verona, Italy. The history of a 34-year-old woman revealed the appearance of hydroureteronephrosis on the right side at the 35th week of pregnancy. She had an magnetic resonance imaging scan and was diagnosed with a spread retroperitoneal fibrosis. After 2 months, the patient reported the occurrence of pelvic pain, dyspareunia and dysmenorrhea. She was treated with corticosteroids and tamoxifen with no results. Laparoscopic surgery. A complete retroperitoneal extirpation was done of an endometriotic nodule of the right broad ligament, near the right ureter (without stenosis). Reduction of pelvic pain. She noticed an important decrease of pain. The cause of hydronephrosis could be a physiologic hydroureteronephrosis, which is the most common cause of dilatation of the urinary tract in pregnancy. The pain symptoms of the patients seemed to be linked to endometriosis and not to retroperitoneal fibrosis. Magnetic resonance imaging sometimes does not enable a correct diagnosis between these two pathologies. Fertile women with suspected fibrosis should undergo a diagnostic laparoscopy by an expert surgeon in retroperitoneal surgery. Pelvic congestion affects women of reproductive age. Typical symptoms include bilateral lower abdominal and back pain that is increased with standing for long periods, secondary dysmenorrhea, dyspareunia, abnormal uterine bleeding, chronic fatigue, and irritable bowel symptoms (97). Pain usually begins with ovulation and lasts until the end of menses. The uterus is often bulky, and the ovaries are enlarged with multiple functional cysts. The uterus, parametria, and uterosacral ligaments are tender. Transuterine venography is the primary method for diagnosis, although other modalities, such as pelvic ultrasound, magnetic resonance imaging, and laparoscopy, may disclose varicosities (93). Because of the cost and possible side effects of treatment, further management should be based on related symptoms and not simply on the presence of varicosities. Heidi is an almost 6-year-old girl presenting to your primary care office to establish care because of a change in insurance status. You review her previous medical records before seeing her.She was diagnosed with autism spectrum disorder (ASD) when she was 25 months old. Her parents were initially concerned about language delay. Through a comprehensive evaluation by a developmental-behavioral pediatrician and a child psychologist, including administration of the Bayley Scales of Infant and Toddler Development and the Autism Diagnostic Observation Schedule, she was diagnosed with ASD. Her cognitive skills were reported to be within the average range. Soon after the diagnosis, she began receiving 20 hours of applied behavioral analysis (ABA) per week, as well as music therapy, occupational therapy, and a toddler playgroup through early intervention. Four months after the initial diagnosis, her parents reported that she had started making small improvements in her behavior, used more eye contact, and seemed more socially engaged. Approximately 1 year after the diagnosis, she was receiving 6 hours of ABA per week in addition to starting preschool with an Individualized Education Program. She reportedly continued to show progress with social communication and pretend play skills.At the age of 3 years, 8 months, neuropsychological testing was completed at her parent's request, and her cognitive skills and adaptive skills were reported to be within the average range. She continued to meet the diagnostic criteria for ASD, given her challenges with social awareness, communication, delayed play skills, decreased flexibility, and tendency toward subtle self-direction. She continued to receive speech/language therapy and attended an integrated preschool program within the school district because of her social and communication challenges. She also received ABA 4 hours weekly at home.During your first visit with Heidi, her parents report that she has continued to make progress in all areas, including social skills. She can engage in imaginary play with her friends, ask strangers questions, and comprehend the perspective of others, and she is no longer \"rigid.\" She is not receiving services outside of school and is only receiving once weekly speech/language therapy in school. Her parents no longer believe that she meets the criteria for ASD, and they are interested in further evaluation. Her parents ask if it is possible to \"lose\" the diagnosis of ASD. They also want to know if there are other things to be concerned about for her future. How do you respond? Imaging Tests The role of imaging techniques in studying female urinary incontinence is not yet established. Researchers are evaluating the potential roles of ultrasonography, fluoroscopy, functional neuroimaging, and magnetic resonance imaging (MRI). These tests should not be done routinely but are useful in certain conditions. If the patient’s symptoms (easily remembered by the three Ds: dysuria, dribbling, and dyspareunia) or examination suggests a urethral diverticulum, MRI is the test of choice (44).\nHere is the question:\nGYNECOLOGY AND OBSTETRICS: A 27-year-old woman referred to the gynecology office for evaluation referring dyspareunia for about 8 months, along with dyschezia and occasional rectorrhagia coinciding with menstruation for 3-4 months. She also reports dysmenorrhea for years, which she controls well with ibuprofen. She has been trying to get pregnant for 16 months without success. In the gynecological examination she only has pain when pressing on the posterior vaginal fornix. Which test do you consider would allow you to reach a diagnosis of certainty of her pathology?\nHere are the potential choices:\n1. Transvaginal ultrasound.\n2. Diagnostic laparoscopy.\n3. Magnetic resonance imaging.\n4. Colonoscopy.\nThe correct answer is: 2. Diagnostic laparoscopy." }, { "question": "You are a helpful medical expert, and your task is to answer a multi-choice medical question using the relevant documents. Please choose the answer from the provided options. Your responses will be used for research purposes only, so please have a definite answer.\nHere are the relevant documents:\n Psittacosis, caused by the bacteria Chlamydia psittaci, is primarily a disease of birds that can be transmitted to humans. The clinical manifestations of the disease are wide, ranging from asymptomatic illness to fulminant psittacosis with multi-organ failure. The organism gets attached to the upper respiratory mucosa after inhalation and the majority remain asymptomatic. However, some people may develop symptoms of atypical pneumonia. Psittacosis usually presents with sudden onset fever with chills and rigor, headache, and myalgia. Here we present a case of a 35 years old female with a history of close contact with parrots who presented to the ER with complaints of high-grade fever and headache for 2 weeks which started 2 days after her parrots died. The disease usually manifests as flu-like symptoms or pneumonia and is included in the differential diagnosis of community-acquired pneumonia. Investigations reveal neutrophilia, raised erythrocyte sedimentation rate, C-reactive protein, and elevated liver enzymes which were consistent with the findings of our patient. Chest X-ray showed ill-defined consolidation in the right middle and lower lobes which were inconclusive. Hence, a CT chest was done which revealed patchy ground glass opacities with surrounding consolidation giving a reverse halo sign. Due to her contact with birds and CT findings which were suggestive of psittacosis, she was started on doxycycline and her condition improved thereafter. We highlight the importance of proper history taking and awareness on zoonotic diseases to the general public to prevent, diagnose and treat the disease effectively. Three familial cases of psittacosis are reported. The first case was a 46-year-old woman, the second case, her 18-year-old daughter. Both of them often visited the house of the third case, a 49-year-old women, who was the elder sister of case 1 and who took care of the chick of a budgerigar which she kept in the house. Case 1 came to our hospital with abrupt onset of fever, headache, nausea and general malaise. Because she was suspected to have meningitis, she was admitted to the Department of Neurology. On admission, her chest X-ray film showed bilateral ground glass shadows. She also had hypoxemia and liver dysfunction. On learning of her history of contact with the chick, psittacosis was suspected. Case 2 suffered from fever and headache. Her chest X-ray film revealed opaque infiltration in the right lower lung field. Case 3 complained of fever, headache and vomiting. Her chest X-ray film showed fan-shaped faint shadows in the left upper, middle and lower lung fields. We interpreted these findings as showing psittacosis based on anamnesis. The result of the complement fixation (CF) antibody titer against chlamydia was 1:32 in cases 2 and 3, enabling a serological diagnosis of psittacosis. The corresponding result was 1:16 in case 1. Although the CF antibody titer showed no increase, we diagnosed the case clinically as psittacosis. It is difficult to correctly diagnose psittacosis only from the physical findings and chest X-ray films. Detailed anamnesis, in particular taking a history of exposure to birds, is an important clue for diagnosis.(ABSTRACT TRUNCATED AT 250 WORDS) Coxiella burnetii (C.b.) is a strictly intracellular, Gram-negative bacterium. It causes Q fever in humans and animals worldwide. The animal Q fever is sometimes designated \"coxiellosis\". This infection has many different reservoirs including arthropods, birds and mammals. Domestic animals and pets, are the most frequent source of human infections. Q fever may appear basically in two forms, acute and chronic (persistent). The latter form of Q fever in animals is characteristic by shedding C.b. into the environment during parturition or abortion. Human Q fever results usually from inhalation of contaminated aerosols originating mostly from tissue and body fluids of infected animals. Q fever may appear in humans either in an acute form accompanied mainly by fever (pneumonia, flu-like disease, hepatitis) or in a chronic form (mainly endocarditis). Diagnosis of Q fever is based on isolation of the agent in cell culture, its direct detection, namely by PCR, and serology. Detection of high phase II antibodies titers 1-3 weeks after the onset of symptoms and identification of IgM antibodies are indicative to acute infection. High phase I IgG antibody titers >800 as revealed by microimmunofluorescence offer evidence of chronic C.b. infection. For acute Q fever, a two-weeks-treatment with doxycycline is recommended as the first-line therapy. In the case of Q fever endocarditis a long-term combined antibiotic therapy is necessary to prevent relapses. Application of Q fever vaccines containing or prepared from phase I C.b. corpuscles should be considered at least for professionally exposed groups of the population. Infections caused by C.b. are spread worldwide and may pose serious and often underestimated health problems in human but also in veterinary medicine. Though during the last decades substantial progress in investigation of C.b. has been achieved and many data concerning this pathogen has been accumulated, some questions, namely those related to the pathogenesis of the disease, remain open. Q fever which is caused by Coxiella burnetii, is a worldwide zoonosis. Many species of wild and domestic mammals, birds, and arthropods, are reservoirs of C.burnetii in nature, however farm animals are the most frequent sources of human infection. The most frequent way of transmission is by inhalation of contaminated aerosols. The clinical presentation of Q fever is polymorphic and nonspecific. Q fever may present as acute or chronic disease. In acute cases, the most common clinical syndromes are selflimited febrile illness, granulomatous hepatitis, and pneumonia, but it can also be asymptomatic. Fever with hepatitis associated with Q fever has rarely been described in the literature. Herein we report two cases of C.burnetii hepatitis presented with jaundice. In May 2011, two male cases, who inhabited in Malkara village of Tekirdag province (located at Trace region of Turkey), were admitted to the hospital with the complaints of persistent high grade fever, chills and sweats, icterus, disseminated myalgia and headache. Physical examination revealed fever, icterus and the patient appeared to be mildly ill but had no localizing signs of infection. Radiological findings of the patients were in normal limits. Laboratory findings revealed leukocytosis, increased hepatic and cholestatic enzyme levels, and moderate hyperbilirubinemia- mainly direct bilirubin, whereas serum C-reactive protein and erythrocyte sedimentation rate were found normal. Blood and urine cultures of the patients yielded no bacterial growth. Serological markers for acute viral hepatitis, citomegalovirus and Epstein-Barr virus infections, brucellosis, salmonellosis, toxoplasmosis and leptospirosis were found negative. Acute Q fever diagnosis of the cases were based on the positive results obtained by C.burnetii Phase II IgM and IgG ELISA (Vircell SL, Spain) test, and the serological diagnosis were confirmed by Phase I and II immunofluorescence (Vircell SL, Spain) method. Both cases were treated with doxycycline for 14 days and became afebrile within four days. These cases were presented to emphasize that C.burnetii infection should be considered in the differential diagnosis of patients with fever and elevated serum transaminase levels, irrespective of the presence of abdominal pain and exposure to potentially infected animals. A 58-year-old man was admitted because of respiratory failure, episodic fever with chilling, cough, malaise, fatigue, myalgia and weight loss lasting for at least one month. Chest x-rays and CT scan of the chest showed bilateral pulmonary consolidations in upper lobes, the left lower lobe, and mediastinal lymphadenopathy. Bronchoscopy with cytology was unremarkable. A needle CT-guided lung biopsy documented an inflammatory pseudotumor, lymphoplasmacytic type. Serology showed high titer antibodies to phase II Coxiella burnetii infection. Therapy with doxycycline and hydroxychloroquine for three months led to a complete resolution of symptoms and radiological findings, and a marked decrease in titers to Q fever. Q fever is a bacterial infection affecting mainly the lungs, liver, and heart. It is found around the world and is caused by the bacteria Coxiella burnetii. The bacteria affects sheep, goats, cattle, dogs, cats, birds, rodents, and ticks. Infected animals shed this bacteria in birth products, feces, milk, and urine. Humans usually get Q fever by breathing in contaminated droplets released by infected animals and drinking raw milk. People at highest risk for this infection are farmers, laboratory workers, sheep and dairy workers, and veterinarians. Chronic Q fever develops in people who have been infected for more than 6 months. It usually takes about 20 days after exposure to the bacteria for symptoms to occur. Most cases are mild, yet some severe cases have been reported. Symptoms of acute Q fever may include: chest pain with breathing, cough, fever, headache, jaundice, muscle pains, and shortness of breath. Symptoms of chronic Q fever may include chills, fatigue, night sweats, prolonged fever, and shortness of breath. Q fever is diagnosed with a blood antibody test. The main treatment for the disease is with antibiotics. For acute Q fever, doxycycline is recommended. For chronic Q fever, a combination of doxycycline and hydroxychloroquine is often used long term. Complications are cirrhosis, hepatitis, encephalitis, endocarditis, pericarditis, myocarditis, interstitial pulmonary fibrosis, meningitis, and pneumonia. People at risk should always: carefully dispose of animal products that may be infected, disinfect any contaminated areas, and thoroughly wash their hands. Pasteurizing milk can also help prevent Q fever. The brucellosis and Q-fever coinfection is very rarely reported. To our knowledge, this is the first case report of concomitant brucellosis and Q-fever, most likely imported in Croatia. A 30-year-old male agricultural worker was hospitalized on 22 April 2017 after a ten days fever up to 40°C with chills, shivering, excessive sweating, general weakness, loss of appetite and headache. A month and a half prior to the hospitalization he lost 18 kg of body weight. Three weeks before hospitalization the patient returned from Kupres (Bosnia and Herzegovina) where he was working for the past year on a sheep farm and consumed unpasteurized dairy products of sheep origin. At admission, his condition was moderately severe due to pronounced dehydration. Routine laboratory tests showed slightly elevated erythrocyte sedimentation rate, anemia, thrombocytopenia and elevated liver transaminases. The chest X-ray showed an inhomogeneous infiltrate of the lower right lung. Three sets of blood culture were cultivated. After 48 hours incubation, bacterial growth was detected in aerobic bottles. Gram-stained smear revealed small, gram-negative coccobacilli. Specimens were subcultured on blood and chocolate agar plates. Using a Vitek GN identification card, the isolated organism was identified as Brucella melitensis. 16S rRNA gene sequencing of the isolate confirmed it as a Brucella sp. Rose-Bengal test was positive, while Wright agglutination test showed a significant increase in antibody titer from 80 to 640 in paired sera. Using indirect immunofluorescence assay (IFA), Coxiella burnetii phase II IgM/IgG titers were 50 and 1024, respectively indicating acute Q-fever. The patient was treated with doxycycline and rifampicin. So far, there has been no relapse or signs of chronic infection. Diagnosis The diagnosis of acute chest syndrome is made difficult by its similarity in presentation with pneumonia. Both may present with a new opacification of the lung on chest x-ray. The presence of fevers, low oxygen levels in the blood, increased respiratory rate, chest pain, and cough are also common in acute chest syndrome. Diagnostic workup includes chest x-ray, complete cell count, reticulocyte count, ECG, and blood and sputum cultures. Patients may also require additional blood tests or imaging (e.g. a CT scan) to exclude a heart attack or other pulmonary pathology. Prevention Hydroxyurea is a medication that can help to prevent acute chest syndrome. It may cause a low white blood cell count, which can predispose the person to some types of infection. Treatment Broad spectrum antibiotics to cover common infections such as Streptococcus pneumoniae and mycoplasma, pain control, and blood transfusion. Acute chest syndrome is an indication for exchange transfusion. Signs and symptoms Initially, HPS has an incubation phase of 2–4 weeks, in which patients remain asymptomatic. Subsequently, patients can experience 3–5 days of flu-like prodromal phase symptoms, including fever, cough, muscle pain, headache, lethargy, shortness of breath, nausea, vomiting and diarrhea. In the following 5–7 day cardiopulmonary phase, the patient's condition rapidly deteriorates into acute respiratory failure, characterized by the sudden onset of shortness of breath with rapidly evolving pulmonary edema, as well as cardiac failure, with hypotension, tachycardia and shock. In this phase, patients may develop acute respiratory distress syndrome. It is often fatal despite mechanical ventilation and intervention with diuretics. After the cardiopulmonary phase, patients can enter a diuretic phase of 2–3 days characterized by symptom improvement and diuresis. Subsequent convalescence can last months to years. As of 2017, patient mortality in the USA from HPS is 36%. Brucellosis is a zoonotic disease caused by a Gram-negative bacillus of the Brucella genus with multisystem involvement, primarily affecting the reticuloendothelial system, joints, heart and kidneys. Although the disease can be spread by inhalation, pulmonary involvement is rare. To report a case of brucellosis with pulmonary involvement. CASE PRESENTAION: A previously healthy 36-year-old woman was admitted with complaints of fever, weakness, night sweats, dry cough and bilateral chest pain. She hed been diagnosed with pneumonia 20 days previously and was started on a course of ampicillin for 14 days, with no response. Her chest auscultation revealed diminished breath sounds and scattered crackles and rhonchi over the inferior zone of the right hemithorax. Wright and Coombs testing resulted in titres of 1:1280 and 1:640, respectively. Chest radiography revealed an area of confluent lobar consolidation in the right lower lobe. Treatment was started with a six-week course of oral doxycycline 200 mg/day and rifampicin 600 mg/day. This treatment regimen rapidly improved the patient's condition. Follow-up after one year showed no residual effects from the infection. Pulmonary involvement is a rare event in the course of brucellosis, but the rate could be higher than currently estimated. In endemic regions, brucellosis should be considered as a causative agent in patients with pulmonary symptoms. Brucellosis is a zoonotic disease caused by a Gram-negative bacillus of the 1.6 mEq/L). Which of the following therapeutic options would be most indicated?\nHere are the potential choices:\n1. Aminophylline associated with a cathartic.\n2. Activated charcoal.\n3. Hemodialysis.\n4. Forced diuresis.\nThe correct answer is: ", "gold_answer": "3 Hemodialysis.", "prediction": "You are a helpful medical expert, and your task is to answer a multi-choice medical question using the relevant documents. Please choose the answer from the provided options. Your responses will be used for research purposes only, so please have a definite answer.\nHere are the relevant documents:\n To describe the case of a patient who ingested 50 sustained release lithium carbonate 400 mg tablets, and reached a late peak concentration above 3 mEq/L. A 32-year-old male with bipolar mood disorder ingested 50 sustained-release lithium carbonate tablets. Upon admission to the emergency room, a gastric wash was performed,from which several tablet remnants were obtained, as well as an intestinal lavage using activated carbon. good general status, no fever, blood pressure 160/90 mm Hg, no edemas. Neurologic, pulmonary, and cardiac examinations were normal. CBC and the chemistry panel were normal. The patient's psychopathological examination suggested a stable status with no apparent manifestations arising from a decompensated mood disorder. Five hours after his massive lithium ingestion the drug's plasma levels were 0.75 mEq/L. At 22 hours post-ingestion a chemistry panel was obtained, which showed serum creatinin at 1.38 mg/dL and a lithium plasma concentration of 3.15 mEq/L. A hemodyalisis trial was attempted for 4 hours. At 73 hours post-ingestion, lithium plasma levels were 0.6 mEq/L, that is, within therapeutic range. The patient was hemodynamically stable and serial blood tests were normal; he was discharged. Acute lithium intoxication with plasma levels above 3 mEq/l can be fatal or result in irreversible neurologicsequelae in almost one third of cases, with persistent cerebellar dysfunction in association with dementia of variable degree, andrenal, blood, and liver disturbances. Sustained-release tablets may prolong absorption and delay peak plasma concentrations. In such cases, therefore, it is recommended that drug plasma concentrations be monitored during 48-72 hours post-ingestion. Lithium has been used to treat bipolar disorder. Lithium has a narrow therapeutic index, with a therapeutic level between 0.6 and 1.5 mEq/L. The possible complications of lithium overdose include altered mental status, hand tremor, muscle weakness, nausea, vomiting, diarrhea, seizure, syncope, and arrhythmia. Lithium intoxication can be fatal and is difficult to diagnose in patients without a history of lithium intake. The occurrence of serious cardiac arrhythmias is rare in lithium intoxication. An 81-year-old man was brought to the emergency department because of consciousness disturbance for 2 days. According to his daughter, he had a history of hypertension and diabetes. Recently, his family also observed slurring of speech and easy choking. The physical examination findings were unremarkable. Blood examination only revealed impaired renal function. Twelve-lead electrocardiography revealed sinus rhythm with first-degree atrioventricular block. Chest radiography revealed mediastinal widening. The blood pressures obtained from the 4 limbs showed no significant differences. Subsequently, brain computed tomography revealed no obvious intracranial lesion. A neurologist was consulted, and a recent ischemic stroke could not be ruled out. While in the observation area, his systolic blood pressure decreased to <90 mm Hg and he showed bradycardia, and 12-lead electrocardiography revealed an AV block and long pulse. Contrast-enhanced chest computed tomography revealed no evidence of aortic dissection. Another family member reported a history of lithium intake for bipolar disorder for >30 years. Blood examination revealed a lithium concentration of 2.65 mEq/L. A nephrologist was consulted, and emergency hemodialysis was indicated. Dopamine was administered for his shock status via a right neck central venous catheter. His lithium level gradually declined after the hemodialysis, and blood pressure and consciousness level improved subsequently. The patient was discharged 9 days later in a stable condition. If an emergency physician encounters a patient with altered consciousness and arrhythmia with cardiogenic shock, the patient's drug intake history should be carefully reviewed to rule out cardiovascular problems on the basis of the patient's clinical condition. A 53-year-old woman was brought to the Emergency Department for a persistent state of stupor, tremors, fever and oliguria. The patient had been under treatment for depression. The electrocardiogram showed a wider QRS complex; laboratory tests were as follows: urea 110 mg/dL, creatinine 3 mg/dL, sodium 135 mEq/L, potassium 4.5 mEq/L, and lithium 8.0 mEq/L. Renal replacement was initiated to normalize plasma lithium levels; both stupor and speech impairment persisted for several days after dialysis. Complete recovery was achieved several days afterwards. The patient was a 26-year-old woman who ingested a total of 230 tablets, including 160 lithium carbonate tablets (200 mg), in a suicide attempt, and was brought to our hospital 5 hr later. After arriving at the hospital, her only complaint was mild nausea, and no neurological abnormalities or renal dysfunction was observed. We were unable to learn the blood concentration of lithium immediately. A forced diuresis was performed after admission, but the nausea persisted. Continuous hemodialysis (CHD) was performed for 23.5 hr starting 19 hr after ingesting the tablets because the patient had ingested a large amount of lithium carbonate, 32,000 mg. Since the nausea resolved after the CHD was started and no manifestations of lithium intoxication had developed as of 91 hr after ingestion, the patient was discharged. The blood lithium concentrations (mEq/L) revealed at a later date showed that the concentration 5 hr (at the time of the initial examination), 19 hr (start of CHD), 44.5 hr (end of CHD), and 91 hr after ingestion (at the time of discharge) was 4.08, 3.30, 1.09, and 0.38, respectively. Blood purification is said to be effective in treating serious lithium intoxication, but it is difficult to judge when to stop. A favorable outcome of treatment of acute lithium intoxication in a patient with normal renal function appears to have been achieved by performing CHD guided by the clinical manifestations, intravascular redistribution times, etc. Lithium is widely used to treat bipolar disorders. Lithium toxicity is generally caused by inappropriately high doses of lithium or impaired lithium excretion. Most lithium is eliminated via the kidneys and, since thyroid hormone increases tubular reabsorption of lithium, thyrotoxicosis could contribute to the development of lithium toxicity. We report a case of severe lithium toxicity that was apparently precipitated by the onset of thyrotoxicosis resulting from silent thyroiditis and dehydration. The patient was a 64-year-old woman who was admitted for muscle weakness in the lower extremities, diarrhea, and palpitations. She had bipolar disorder and was being treated with lithium carbonate, which she discontinued one week before admission. Her circulating lithium levels had been monitored yearly. Early in her admission she was dehydrated and had febrile episodes, paroxysmal atrial fibrillation, and muscle weakness. Initially, fluid therapy was started, but she lost consciousness and had a cardiac arrest for 2 minutes due to prolonged sinus arrest. Chest compression and manual artificial ventilation were performed, and body surface pacing was started. Serum lithium was markedly elevated to 3.81 mEq/L (therapeutic range, 0.4-1.0 mEq/L), and thyroid hormone levels were increased (free triiodothyronine, 8.12 pg/mL; free thyroxine, 4.45 ng/dL), while thyrotropin (TSH) was suppressed (<0.01 μIU/mL). Hemodialysis was performed, and a temporary pacemaker was inserted for severe sinus bradycardia. The serum thyroglobulin was 4680 ng/mL (reference range, <32.7 ng/mL). A TSH receptor antibody test was negative. Glucocorticoid therapy and inorganic iodine (100 mg) were administered and continued until day 11. However, her neurological symptoms deteriorated with floppy quadriplegia and deep coma. She gradually recovered. On day 36, she was discharged without any neurological symptoms or thyrotoxicosis. A 64-year-old woman taking lithium for bipolar disorder developed lithium toxicity in the setting of what seemed likely to be a recent onset of thyrotoxicosis due to silent thyroiditis. Thyrotoxicosis may be a contributing cause of lithium toxicity, particularly if it is abrupt in onset and even with cessation of lithium therapy if renal function is compromised. Thyroid function should be assessed immediately in patients with suspected lithium toxicity. Several reports have been published in the literature of choreoathetosis associated with lithium intoxication, but little is known about choreoathetosis without concurrent antipsychotic treatment. We report a 65-year-old woman with lithium intoxication whose choreoathetosis completely recovered without sequela following decrease of her serum lithium level. She had been treated elsewhere for bipolar II disorder and also for hypertension, chronic hepatitis type C and diabetes mellitus. As she became hypomanic, lithium carbonate at 600 mg/day was commenced, which was increased to 1200 mg/day due to unfavorable therapeutic response. She began to manifest disorientation and abnormal involuntary movement and was therefore referred to our Department of Psychiatry. Her clinical symptoms at admission included consciousness disturbance with marked bilateral symmetrical slow-wave activity in her EEG and choreoathetosis was observed in her face and upper and lower extremities. Cerebellar symptoms were minimal with only mild ataxic gait and finger-to-nose test did not show dysmetria or intention tremor. Her serum lithium level was 3.52 mEq/L, which was clearly in the toxic range. She demonstrated no metabolic abnormalities including hyperglycemia, and was diagnosed with lithium intoxication and treated with water loading and mannitol for forced diuresis. On the 14th day after admission her consciousness disturbance and choreoathetosis resolved, but EEG abnormalities still persisted. On the 23rd day after admission, she was discharged with clinical remission and normal EEG background activity. Although she developed mild renal dysfunction, hemodialysis was not indicated. Hypersensitivity of dopamine receptor in the nigrostriatal pathways may contribute to choreoathetosis in association with the patient's vulnerability. Choreoathetosis can be a sign of lithium intoxication and prompt treatment is required following careful differential diagnosis. Lithium is the milestone of psychiatric patients' therapy, in particular in bipolar disorder. Despite its high therapeutic efficacy, there are several side effects (renal, thyroid, parathyroid, dermatological) and management problems linked to its narrow therapeutic range, which exposes patients to a high risk of toxicity. We describe the case of a male patient with bipolar disorder in therapy with lithium sulfate who developed a severe acute-on-chronic intoxication. He came to our attention in a somnolent state with lithemia >3 mEq/L and therefore underwent hemodialysis. In view of the high toxicity of lithium, a timely and correct therapeutic choice is important to improve the patient's outcome. In this context, considering lithemia, but also kidney function and the patient's clinical status, it is necessary to consider extracorporeal treatments, of which hemodialysis is the most preferable. Gastric lavage and whole bowel irrigation may be useful if done early. Activated charcoal is not effective. For severe toxicity hemodialysis is recommended. The risk of death is generally low. Acute toxicity generally has better outcomes than chronic toxicity. In the United States about 5,000 cases are reported to poison control centers a year. Lithium toxicity was first described in 1898. Signs and symptoms Symptoms of lithium toxicity can be mild, moderate, or severe. Mild symptoms include nausea, feeling tired, and tremor occur at a level of 1.5 to 2.5 mEq/L. Moderate symptoms include confusion, an increased heart rate, and low muscle tone occur at a level of 2.5 to 3.5 mEq/L. Severe symptoms include coma, seizures, low blood pressure and increased body temperature which occur at a lithium concentration greater than 3.5 mEq/L. When lithium overdoses produce neurological deficits or cardiac toxicity, the symptoms are considered serious and can be fatal. Potential side effects from lithium include gastrointestinal upset, tremor, sedation, excessive thirst, frequent urination, cognitive problems, impaired motor coordination, hair loss, and acne. Excessive levels of lithium can be harmful to the kidneys, and increase the risk of side effects in general. As a result, kidney function and blood levels of lithium are monitored in patients being treated with lithium. Therapeutic plasma levels of lithium range from 0.5 to 1.5 mEq/L, with levels of 0.8 or higher being desirable in acute mania. Lithium levels should be above 0.6 mEq/L to reduce both manic and depressive episodes in patients. A recent review concludes that the standard lithium serum level should be 0.60-0.80 mmol/L with optional reduction to 0.40-0.60 mmol/L in case of good response but poor tolerance or an increase to 0.80-1.00 mmol/L in case of insufficient response and good tolerance. To report a case of severe hypermagnesemia caused by magnesium hydroxide in a woman with normal renal function. A 42-year-old Hispanic woman with schizophrenia and bipolar affective disorder was transported from jail to the emergency department with confusion, abdominal pain, vomiting, and constipation. She had been treated in jail with magnesium hydroxide, ordered as milk of magnesia 30 mL po each night and Maalox 30 mL po three times daily. Additional medications included lithium carbonate 300 mg po three times daily, chlorpromazine 150 mg po three times daily, benztropine mesylate 1 mg po twice daily, and docusate sodium 100 mg po each morning. Her temperature was 35.1 degrees C, blood pressure 108/58 mm Hg, heart rate 112 beats/min, and respiratory rate 24 breaths/min. She would respond only briefly to voice or painful stimuli. Her abdomen was distended and diffusely tender. Laboratory tests included serum magnesium concentration 9.1 mEq/L (normal 1.3-2), blood urea nitrogen 16 mg/dL (8-22), creatinine 0.9 mg/dL (0.5-1.1), calcium 3.9 mEq/L (4.2-5.2), and lithium 1.0 mEq/L. A laparotomy was performed, and an adhesive band from a previous oophorectomy was found to be compressing the sigmoid colon. Hypermagnesemia, hypothermia, and hypotension continued in the intensive care unit. Despite successful treatment of the hypermagnesemia with calcium, intravenous fluids, and furosemide, the patient's cardiac rhythm degenerated into fatal, pulseless electrical activity on postoperative day 2. This case of severe hypermagnesemia from magnesium hydroxide ingestion illustrates many of the risk factors for hypermagnesemia in patients with normal renal function. People using magnesium-containing medications for relief of gastrointestinal distress may be at increased risk for hypermagnesemia. A brief review of magnesium physiology, clinical effects, and treatment is provided. Frequent use of the laboratory to identify hypermagnesemia is encouraged because it is often a clinically unexpected finding and responds well to early treatment. We describe here the case of a 54-year-old bipolar woman, followed in psychiatry and treated with lithium and a selective serotonin reuptake inhibitor (escitalopram) and lamotrigine, presenting a lithium poisoning with an altered state of consciousness caused by a supposed mismanagement of her treatment. Lithium poisoning was suggested based on neurological clinical features, but the blood test brought out a lithium concentration within the therapeutic values at 1,2 mmol/L (N: 0,6-1,2 mmol/L). The classic biological complications related to lithium poisoning (hypercalcemia, diabetes insipidus) confirmed the diagnosis. The patient has been transferred to our nephrology department where she got two hemodialysis sessions conducting to clinical and biological improvement, confirming the diagnosis of lithium poisoning despite the normal blood levels. Later, she was transferred to the psychiatry department for follow-up and for treatment adjustment. A 63-year-old man was admitted to the intensive care unit (ICU) with a severe aspiration pneumonia. Past medical history included schizophrenia, for which he required institutional care; treatment had included neuroleptics and intermittent lithium, the latter restarted 6 months before admission. The patient was treated with antibiotics and intubated for several days, with the development of polyuria (3–5 L/d), hypernatremia, and acute renal insufficiency; the peak plasma Na+ concentration was 156 meq/L, and peak creatinine was 2.6 mg/dL. Urine osmolality was measured once and reported as 157 mOsm/kg, with a coincident plasma osmolality of 318 mOsm/kg. Lithium was stopped on admission to the ICU. On physical examination, the patient was alert, extubated, and thirsty. Weight was 97.5 kg. Urine output for the previous 24 h had been 3.4 L, with an IV intake of 2 L/d of D5W. To describe a patient with lithium intoxication presenting as acute parkinsonism, adverse metabolic effects and nephrogenic diabetes insipidus (DI). We report a case of a 67-year-old woman with a bipolar affective disorder who was treated with lithium for 10 years. Under concomitant renal insufficiency and urinary tract infarction, she experienced progressive hand tremor, bradykinesia, and unsteady gait. Laboratory results revealed hypercalcemia and hypermagnesiemia. A high serum lithium level (3.6 mEq/L) was found; thus lithium was discontinued. She was found to have a high serum level of intact parathyroid hormone: 135.0 pg/ml and a suspicious parathyroid adenoma. Polyuria with hypernatremia was also noted. A water deprivation test confirmed nephrogenic diabetes insipitces. After correction of electrolyte imbalance and reduction of lithium level, her consciousness recovered. Her parkinsonian features were responsive to levodopa 400 mg/day in 2 divided doses. One month later, apart from the residual extrapyramidal symptoms and mania, her condition was otherwise stationary. Tremor is the most frequent movement disorder associated with lithium therapy, while severe parkinsonism has been rarely reported. It should be kept in mind in differential diagnosis of acute parkinsonism especially in elder patients who receive a chronic lithium carbonate therapy. Treatment with lithium can cause several neurological side effects, even at therapeutic levels. We report the case of a 49-year-old woman, with bipolar disorder and depression, undergoing treatment with lithium, antidepressants and antipsychotics, who was admitted to hospital due to a clinical picture of visual hallucinations with an elevated lithaemia of 2.1 mEq/L (therapeutic range: 0.6-1.2 mEq/L). The patient developed a severe encephalopathy that required the use of assisted ventilation in the intensive care unit. Initial magnetic resonance imaging showed a reversible bilateral symmetrical hyperintensity in the dentate nuclei in T2 and T2-FLAIR sequences. Over the following months she gradually developed a pancerebellar syndrome with evidence of a marked loss of bilateral volume in the cerebellum, above all at the expense of the vermis, which was accompanied by a permanent and disabling cerebellar syndrome. Although treatment with lithium can cause a variety of neurological side effects, they are usually reversible. However, they occasionally give rise to permanent and disabling sequelae, as in the case of the patient reported here, with a marked and progressive cerebellar atrophy, accompanied by permanent sequelae in the form of a disabling cerebellar syndrome. The cerebellar neurotoxicity of lithium must be taken into account in the broad differential diagnosis of cerebellar ataxia in adults. A 35-year-old woman intentionally took 40,000 mg of lithium carbonate, and she was transferred to our hospital with nausea, vomiting, and diarrhea. She was diagnosed as having bipolar disorder 10 years ago and was receiving oral lithium therapy. Blood test results on arrival were remarkable for a negative anion gap of -2.1 and later, the serum lithium level turned out to be as high as 15.4 mEq/L. Intubation was required because of disrupted consciousness, and continuous hemodiafiltration (CHDF) was immediately started in the intensive care unit to obtain constant removal of lithium. After adding intermittent hemodialysis (IHD) twice during the daytime to accelerate the lithium clearance, CHDF became unnecessary on day 4, and she was extubated on day 6 with complete recovery of consciousness. Close monitoring of the patient data showed recovery of the decreased anion gap as indicator of the serum lithium level reduction. On day 36, she was discharged without any complication and sequela. The current case highlighted the effective use of CHDF between IHD sessions to prevent the rebound elevation of lithium and the role of the anion gap as a surrogate marker of serum lithium concentration during the treatment. Mineral Lithium – Lithium is the \"classic\" mood stabilizer, the first to be approved by the US FDA, and still popular in treatment. Therapeutic drug monitoring is required to ensure lithium levels remain in the therapeutic range: 0.6 or 0.8-1.2 mEq/L (or millimolar). Signs and symptoms of toxicity include nausea, vomiting, diarrhea, and ataxia. The most common side effects are lethargy and weight gain. The less common side effects of using lithium are blurred vision, a slight tremble in the hands, and a feeling of being mildly ill. In general, these side effects occur in the first few weeks after commencing lithium treatment. These symptoms can often be improved by lowering the dose. Lithium concentrations in whole blood, plasma, serum or urine may be measured using instrumental techniques as a guide to therapy, to confirm the diagnosis in potential poisoning victims or to assist in the forensic investigation in a case of fatal overdosage. Serum lithium concentrations are usually in the range of 0.5–1.3 mmol/L (0.5–1.3 mEq/L) in well-controlled people, but may increase to 1.8–2.5 mmol/L in those who accumulate the drug over time and to 3–10 mmol/L in acute overdose. Lithium salts have a narrow therapeutic/toxic ratio, so should not be prescribed unless facilities for monitoring plasma concentrations are available. Doses are adjusted to achieve plasma concentrations of 0.4 to 1.2 mmol /L on samples taken 12 hours after the preceding dose. Given the rates of thyroid dysfunction, thyroid parameters should be checked before lithium is instituted and monitored after 3–6 months and then every 6–12 months. Gastric lavage. A tube is placed through the nose or mouth into the stomach. The tube is used to remove lithium that has not been digested yet. It may also be used to put medicines directly into the stomach to help stop lithium from being absorbed. Use of an artificial kidney to clean the blood (dialysis). This is usually done only in the most severe cases. Diuretic medications such as furosemide and hydration via intravenous normal saline appear to be effective in speeding the removal of lithium and also rehydrate patients who've lost fluids. Hemodialysis. Hemodialysis is widely advocated as a means of reducing the risk of permanent neurological sequelae following lithium poisoning. Although hemodialysis clearly enhances the elimination of lithium, it is unclear whether this translates into improved patient outcomes. People may be sent home once their lithium level is less than 1.5 mEq/L and they have no symptoms. See also A stuporous 22-year-old man was admitted with a history of behaving strangely. His friends indicated he experienced recent emotional problems stemming from a failed relationship and had threatened suicide. There was a history of alcohol abuse, but his friends were unaware of recent alcohol consumption. The patient was obtunded on admission, with no evident focal neurologic deficits. The remainder of the physical examination was unremarkable. Na+ 140 meq/L K+ 5 meq/L Cl− 95 meq/L HCO3− 10 meq/L Glucose 125 mg/dL BUN 15 mg/dL Creatinine 0.9 mg/dL Ionized calcium 4.0 mg/dL Plasma osmolality 325 mOsm kg/H2O Urinalysis revealed crystalluria, with a mixture of envelope-shaped and needle-shaped crystals. With blood levels of lithium in the upper therapeutic range (therapeutic 0.6 to 1.2 mEq/L), it is not uncommon to observe a fast-frequency action tremor or asterixis, together with nausea, loose stools, fatigue, polydipsia, and polyuria. These symptoms usually subside with time. Above a level of 1.5 to 2 mEq/L, particularly in patients with impaired renal function or in those taking a thiazide diuretic, serious intoxication becomes manifest—clouding of consciousness, confusion, delirium, dizziness, nystagmus, ataxia, stammering, diffuse myoclonic twitching, and nephrogenic diabetes insipidus. Vertical (downbeating) nystagmus and opsoclonus (see Chap. 13) may also be prominent. A variety of skin problems is common including worsening of acne vulgaris. An uncommon toxic effect is the development of goiter but most patients remain euthyroid although the thyroid-stimulating hormone (TSH) levels may increase slightly. The goiter usually requires no treatment but it is possible to becomes effective, usually a matter of 4 or 5 days. The usual dosage of lithium is 1,200 to 2,400 mg daily in divided oral doses, which produces a desired serum level of 0.9 to 1.4 mEq/L. The serum level of lithium must be checked frequently, both to ensure that a therapeutic dose is being taken and to guard against toxicity (see later). In the treatment of acute mania, lithium is initiated at 300 mg bid or tid, and the dose is then increased by 300 mg every 2–3 days to achieve blood levels of 0.8–1.2 meq/L. Because the therapeutic effect of lithium may not appear until after 7–10 days of treatment, adjunctive usage of lorazepam (1–2 mg every 4 h) or clonazepam (0.5–1 mg every 4 h) may be beneficial to control agitation. Antipsychotics are indicated in patients with severe agitation who respond only partially to benzodiazepines. Patients using lithium should be monitored closely, since the blood levels required to achieve a therapeutic benefit are close to those associated with toxicity. Valproic acid may be better than lithium for patients who experience rapid cycling (i.e., more than four episodes a year) or who present with a mixed or dysphoric mania. Tremor and weight gain are the most common side effects; hepatotoxicity and pancreatitis are rare toxicities. A 32-year-old man was admitted to the hospital with weakness and hypokalemia. The patient had been very healthy until 2 months previously when he developed intermittent leg weakness. His review of systems was otherwise negative. He denied drug or laxative abuse and was on no medications. Past medical history was unremarkable, with no history of neuromuscular disease. Family history was notable for a sister with thyroid disease. Physical examination was notable only for reduced deep tendon reflexes. Sodium 139 143 meq/L Potassium 2.0 3.8 meq/L Chloride 105 107 meq/L Bicarbonate 26 29 meq/L BUN 11 16 mg/dL Creatinine 0.6 1.0 mg/dL Calcium 8.8 8.8 mg/dL Phosphate 1.2 mg/dL Albumin 3.8 meq/L TSH 0.08 μIU/L (normal 0.2–5.39) Free T4 41 pmol/L (normal 10–27) The abuse of laxatives and a prolonged treatment with diuretics has to be brought into consideration as the most common cause for renal or intestinal loss of potassium. Besides characteristical alterations at the E.C.G. and besides intestinal disturbances there do occur again and again acute, life-threatening aspects of cases connected with tetraplegias and a respiratory failure. By means of 3 cases from our hospital and 27 casuistics in literature symptomatology and dynamic in the development of hypokaliemia is discussed. The mean of potassium in the serum of the 16 patients, those having quadriplegias, ran up to 1,7 mval/l (range 1,4-2,5). Paralysis develops peracutely in 4 of the cases within hours and in 12 of the cases within days. In the anamnesis symptoms of adynamia could be traced with nearly every patient. A functional psychosis (reversible physically founded psychosis) couldn't be detected in any of the cases. With the help of a administration of potassium one could achieve a total retrogression of the symptoms. Besides this a normalisation of the acide-base equilibrium is required because of a metabolic alkalosis, detectable in most of the cases.\nHere is the question:\nPSYCHIATRY: A 46-year-old man with bipolar disorder is brought to the emergency department after an over-ingestion of lithium carbonate. Examination reveals severe tremor, ataxia, dysarthria, myoclonus and fasciculations. Lithemia is 4.1 mEq/L (toxicity > 1.6 mEq/L). Which of the following therapeutic options would be most indicated?\nHere are the potential choices:\n1. Aminophylline associated with a cathartic.\n2. Activated charcoal.\n3. Hemodialysis.\n4. Forced diuresis.\nThe correct answer is: 3. Hemodialysis." }, { "question": "You are a helpful medical expert, and your task is to answer a multi-choice medical question using the relevant documents. Please choose the answer from the provided options. Your responses will be used for research purposes only, so please have a definite answer.\nHere are the relevant documents:\n The present report describes the severe evolution of Kawasaki disease in a three-month-old infant. The ailment was initially atypical in its presentation, with the patient exhibiting only persistent fever in association with a progressive lethargy and maculopapular rash on the face, trunk and limbs erroneously diagnosed as roseola infantum. On the 10th day of the condition, mainly due to the unexplained persistence of fever, the infant was admitted to a local hospital. The typical features of KD appeared only on the 14th day of illness with the relapse of the maculopapular rash in association with non-purulent conjunctivitis; dry, reddish and fissured lips; tongue with reddish and hypertrophic papillae; erythema and edema of the palms and soles. During the following days, the ailment rapidly evolved to a catastrophic clinical picture characterized by generalized vasculitis, splenic infarction, pulmonary thrombosis, giant right and left coronary aneurysms, dilatation of common and internal iliac arteries and progressive ischemia of the distal third of the feet resulting in necrotic lesions of both halluces. Appropriate therapy was initiated, but repeated administration of intravenous immunoglobulin G (IVIG) followed by three days of administration of methylprednisolone did not abate the intense inflammatory activity. The remission of inflammation and regression of vascular lesions were only achieved during the following five weeks after the introduction of methotrexate associated with etanercept. The report of this case aims to draw attention to severe forms of KD that exhibit an unfavorable evolution and can be extremely refractory to the conventional therapy. While the diagnosis of typical form of Kawasaki disease (KD) is obvious, this multifaceted disease continues to surprise us. We report the case of a recurrent Kawasaki disease in an infant. At the age of 13 months, the infant was diagnosed with complete Kawasaki disease; he presented with prolonged fever, bilateral conjunctivitis, enanthem, exanthema, edema of the lower limb, peeling, and biological inflammatory syndrome. He was treated with intravenous immunoglobulin (IVIG) associated with a high dose of aspirin and then an antiplatelet dose with a good clinical-biological evolution. The echocardiography was normal. Seven months later, the patient was again admitted, in a similar picture: a prolonged fever evolving for 7 days, bilateral conjunctivitis, enanthem, cervical adenopathy of 1.5 cm/1 cm, scarlatiniform erythema, pruriginous of the trunk and limb, and peeling of the toes, with indurated edema of the hands and feet. The rest of the examination was normal except the irritability. The diagnosis of recurrent KD was made according the five criteria of the American Heart Association. The echocardiography was normal again. The infant received IVIG with good outcome. Despite its rarity, the possibility of recurrence of KD should be known by clinicians, so as not to delay the specific management of vasculitis whose stakes in terms of prevention of coronary artery lesions are well known. Our case confirms the possibility of this recurrence. The cutaneous eruption in Kawasaki disease (Chap. 385) is polymorphous, but the two most common forms are morbilliform and scarlatiniform. Additional mucocutaneous findings include bilateral conjunctival injection; erythema and edema of the hands and feet followed by desquamation; and diffuse erythema of the oropharynx, red strawberry tongue, and dry fissured lips. This clinical picture can resemble TSS and scarlet fever, but clues to the diagnosis of Kawasaki disease are cervical lymphadenopathy, cheilitis, and thrombocytosis. The most serious associated systemic finding in this disease is coronary aneurysms secondary to arteritis. Scarlatiniform eruptions are also seen in the early phase of SSSS (see “Vesicles/Bullae,” above), in young adults with Arcanobacterium haemolyticum infection, and as reactions to drugs. Kawasaki disease is an acute inflammatory condition characterized by various combinations of features but renal involvement is rare. This report is of a case of Kawasaki disease complicated by acute kidney failure. A 10 year-old girl was admitted because of acute renal failure with fever. She developed a high fever, and her general condition was poor; she had developed a macular erythematous rash 10 days earlier for which she was given cefadroxil. At admission, she remained febrile and had strawberry tongue, pharyngitis, dry erythematous lips, bilateral conjunctivitis, cervical lymphadenopathy and desquamation of the skin on her hands. She was anemic (hemoglobin = 9.6 g%), leukocytotic (33,100/mm3), but with no burr, fragmented red blood cells or thrombocytopenia. Her plasma C-reactive protein level was 236 mg/l; her blood urea was 9.5 mmol/l, her creatininemia 288 mumol/l and proteinuria was 0.5 g/l without hematuria. Urine cultures did not grow. Her blood transaminase and gammaglutamyltransferase activities were elevated. Ultrasonography of kidneys and coronary arteries was normal. Kidney biopsy performed one day after admission showed no vascular or glomerular changes, but renal tubular necrosis, indicating urinary excretion of pigments. Tests for myoglobinemia, myoglobinuria and blood muscle enzyme activities were all positive. The renal failure disappeared within 10 days but the fever and inflammatory manifestations persisted for 1 1/2-2 months despite two treatments of intravenous gammaglobulins and continuous salicylate administration. The patient developed arthralgias at the end of the first month of disease, but recovered without renal or vascular complications. Several cases of renal involvement have been reported during the course of Kawasaki disease. They have been rarely documented by histological examination so that the vascular origin of changes has not been demonstrated. Myoglobinuria, as seen in muscular crush injury, and in our case possibly due to malignant hyperthermia, may be responsible for the transient acute renal failure. In 2014, the American Heart Association (AHA) published the criteria needed to establish a diagnosis. [1] However, it is important to note that children who fall short of the full criteria but have cardiac abnormalities on echocardiogram meet the diagnosis of KD. [1] The patient must have fevers for five or more days, with at least four of the following criteria (either all at once or over a series of days): Bilateral painless bulbar conjunctival injection without exudate Erythematous mouth and pharynx, strawberry tongue or red, cracked lips Polymorphous exanthem (morbilliform, maculopapular, or scarlatiniform) Swelling of hands and feet with erythema of the palms and soles Cervical lymphadenopathy (over 1.5 cm in diameter) Arthritis is reported in one-third of cases with Kawasaki syndrome. It may have an early or a late onset form. We present a 15-month-old-girl who had been referred with complaints of pain and swelling in her left shoulder. Physical examination revealed bulbar conjunctival injection, erythematous lips and pharynx, strawberry tongue, erythematous rash, edema and erythema of the left shoulder, left knee, right elbow and right wrist, and moderate distress in the left shoulder and left hip. She was diagnosed with Kawasaki syndrome, and intravenous immunoglobulin infusion (IVIG) 2 g/kg and aspirin (100 mg/kg/day) were instituted. The patient had two additional episodes of arthritis involving the hip joint on the 8th day, and the shoulder and metacarpophalangeal (MCP) and interphalangeal (IP) joints of her right hand on the 15th day. Turbid material was aspirated in both instances; Gram and Wright's staining of this material showed many leukocytes but no bacteria. A second dose of IVIG (1 g/kg) was given. At the end of the third week all extremities were painless, with a normal range of motion. Arthritis in our patient was the presenting sign, having a 'septic arthritis mimicking' and 'biphasic' pattern. Although the patient presented with severe and recurrent arthritis, which is significantly correlated with severe multisystem disease and the presence or development of coronary artery aneurysm, the response to IVIG was excellent. Kawasaki disease (KD) is a systemic vasculitis of unknown cause and is associated with various digestive disorders, although only a few cases of intussusception associated with KD have been reported. We describe a case of intussusception followed by KD in a 3-year-old boy. The patient was admitted to our hospital for evaluation of severe abdominal pain. Because the target sign was seen on ultrasonography, intussusception was diagnosed and hydrostatic reduction was performed. On the second day after admission, he developed a high fever (38°C) and an irregular rash over his whole body. On the fourth day after admission, the high fever continued, and bilateral nonexudative conjunctivitis, erythema of the lips and oral mucosa, strawberry tongue, indurated edema of the dorsa of the hands and feet, and diffuse erythema of the palms and soles appeared, and KD was ultimately diagnosed. He was treated with intravenous immunoglobulin 2 g/kg, aspirin 30 mg/kg/day, and prednisolone 2 mg/kg/day. The high fever and other clinical symptoms resolved immediately after the start of treatment. There was no relapse of KD symptoms after initial treatment, and periungual desquamation was observed on the 10th day after admission. He was discharged on the 15th day, without abnormalities such as coronary dilatation, 3 months after the onset of KD symptoms. Patients with intussusception and KD were older (≥3 years vs <3 years) than those with intussusception alone. In addition, the site of intussusception in KD was mainly colonic rather than ileocolic. If intussusception precedes development of the characteristic clinical symptoms of KD, diagnosis of KD may be delayed. KD should be considered in children older than 3 years with intussusception at a colonic site. A 14-month-old girl presented with a 4-d history of fever and generalized exanthema. Four characteristic symptoms of incomplete Kawasaki disease (KD) were present on admission (fever, rash, non-purulent conjunctival injection, oropharyngeal changes) and then followed by oedema of the hands and feet and mild plantar desquamation. The typical laboratory features of KD, such as elevated erythrocyte sedimentation rate, leukocytosis, thrombocytosis, and positive C-reactive protein were also seen. Ultrasound examination of the mediastinum revealed the presence of a lymph node, 30 mm in diameter, below the tracheal carina. Thoracic CT scan confirmed the mediastinal lymph node. The patient was treated with aspirin and intravenous gamma-globulin. Ultrasound study of the mediastinum, which was carried out 6 weeks after hospital discharge, showed that the lymph node had disappeared. This case illustrates that lymph nodes other than cervical lymphadenopathy should be sought when the diagnosis of classical or atypical KD is suspected. We report a 3-year-old girl with Kawasaki disease who presented with retropharyngeal edema and shock syndrome. This is the first reported case in Taiwan. The patient initially presented with fever, cough, and pyuria followed by rapidly progressive enlarged bilateral cervical lymphadenopathy. On the third day of the fever, computed tomography for airway compression sign found widening of the retropharyngeal space mimicking a retropharyngeal abscess. Later, an endotracheal tube was inserted for respiratory distress. A skin rash over her trunk was also noted. On the fifth day of the fever, the clinical course progressed to hypotension and shock syndrome. Because of more swelling of bilateral neck lymph nodes, computed tomography was arranged again and revealed partial resolution of the edematous changes in the retropharyngeal space. Edema of the hands and feet, bilateral bulbar conjunctivitis, and fissured lips were subsequently found. The diagnosis of Kawasaki disease was confirmed on the eighth day of fever. There was no evidence of bacterial infection. She was administered intravenous immunoglobulin (2 mg/kg) and high dose aspirin (100 mg/kg/day). One day later, the fever subsided, and her blood pressure gradually became stable. Heart echocardiography on the Day 13 revealed dilated left coronary artery and mitral regurgitation. Follow-up echocardiography six months later showed normal coronary arteries. To date, the patient has not experienced any complications. This case illustrates that retropharyngeal edema and shock syndrome can be present in the same clinical course of Kawasaki disease. Clinicians and those who work in intensive care units should be aware of unusual presentations of Kawasaki disease to decrease rates of cardiovascular complications. A 9-year-old girl developing fever and hyperemia of both bulbar conjunctiva 5 days before admission to the Saitama Children's Medical Center after antibiotics proved ineffective was found on admission to have general fatigue and a temperature of 39 degrees C. Physical examination showed hyperemia of the bulbar conjunctiva, fissures of the lips, redness of the pharynx, and swelling of the cervical lymph nodes. Laboratory tests detected neutrophilia (11,200/microL), mild anemia (11.4g/dL), thrombocytopenia (110,000/microL), and elevated serum aspartate aminotransferase (242IU/L), alanine aminotransferase (328IU/L), and C-rective protein (25.2 mg/dL). Autoantibodies such as anti-nuclear, anti-SS-A/Ro, and anti-Jo-1 were also found. Echocardiography showed no abnormality of the coronary arteries. She was diagnosed as having incomplete Kawasaki disease on day 7 of illness, necessitating that a high dose of immunoglobulin be given intravenously. Her temperature dropped temporarily to 37 degrees C, but she developed erythema of the cheek and fever. Intravenous immunoglobulin was restarted, and minocycline introduced because her daily contact with a pet cat indicated richettsial infection such as Q fever. Mild fever, muscle pain, and elevated C-reactive protein did not improve, but clinical signs and symptoms gradually lessened after ibuprofen was given, then disappeared. A definitive diagnosis of Q fever was made through an over 4-fold rise in phase II IgG antibody titers against Coxiella burnetii, titer of less than 1 : 16 on day 14 of illness, and titer of 1 : 256 on day 34. This case study describes on atypical case of Q fever with clinical manifestations mimicking Kawasaki disease. Erythema multiforme (EM) is an immune-mediated disease with mucocutaneous localization and plurietiologic determinism. The term \"multiforme\" refers to the variety of aspects that the lesions can take from patient to patient and during evolution in a single patient. We have selected 2 cases of small children diagnosed with different etiology of EM to illustrate the importance of a correct and fast diagnosis. Case 1 involves a 2-year-old girl from a rural area who presented with fever and pruritic erythematous papular eruption. The onset of the symptoms was 3 days before presentation with fever and ulcerative lesions on the oral and labial mucosa, followed by the appearance of erythematous macular lesions, with progressive confluence to intense pruritic patches. The 2nd involves a 2-year-old boy with fever, loss of appetite, productive cough, and petechiae. He had corticosensible immune thrombocytopenia from the age of 6 months, with many recurrences. The patient received treatment with ampicillin/sulbactam and symptomatics for an erythemato-pultaceous angina. During the 2nd day of treatment the patient developed an erythematous macular eruption on the face, scalp, trunk, and limbs, with bullae formation. The 1st patient was diagnosed based on biologic findings: positive inflammatory syndrome, elevated level of anti-Mycoplasma pneumoniae immunoglobulin M antibodies and immunoglobulin E. Histopathologic examination described papillary dermal edema, inflammatory infiltrate, and lymphocyte exocytosis. In the 2nd case, the hemoleucogram identified 12,000/mm platelets and the medulogram aspect was normal. Serology for Epstein-Barr virus was negative. The diagnosis was EM secondary to M pneumoniae infection in case 1 and secondary to administration of ampicillin/sulbactam in case 2. In both cases, etiopathogenic treatment consisting of steroidal antiinflammatory drugs, antihistamines was administered. Because of specific etiology, the 1st case received antibiotics. The evolution was favorable in 10 to 14 days; the patients were discharged after etiopathogenic treatment consisting of steroidal antiinflammatory drugs, antihistamines, and/or antibiotics. Performing a detailed clinical examination, medical history of drug use, infection or general diseases can establish a good diagnosis of EM. Histopathologic examination can help. The treatment is etiologic, pathogenic, and symptomatic. EM usually has a self-limited evolution. A previously healthy 11-month-old girl presented with fever and rash for 6 days. Physical examination revealed an irritable infant with a high fever, injected conjunctivae, red cracked lips, posterior auricular lymphadenopathy, hepatomegaly, generalized erythematous maculopapular rash and petechial hemorrhage on trunk, face and extremities. Complete blood count showed atypical lymphocytosis and thrombocytopenia. Dengue infection was initially diagnosed. The persistent fever and clinical manifestations of Kawasaki disease (KD) were observed on day 8 with high erythrocyte sedimentation rate (56 mm/hr). Treatment of KD included intravenous immunoglobulin on day 9 of the illness. Desquamation of the fingers was found on day 15 of the illness. Ectasia of left coronary artery with small aneurysmal dilatation was detected by echocardiography on day 15 of the illness. Hemagglutination-inhibition test and enzyme-linked immunosorbent assay for dengue virus eventually showed a four-fold rising. According to the literature review, this is the second reported case of dengue infection concomitant with KD. The natural course of each disease may be modified and causes some difficulties in diagnosis and management. Cervical lymphadenopathy (often painful and unilateral, with at least one node > 1.5 cm). Diffuse mucous membrane erythema (e.g., “strawberry tongue”); dry, red, chapped lips. Erythema of the palms and soles; indurative edema of the hands and feet; late desquamation of the fingertips (in the subacute phase). Other manifestations include sterile pyuria, gallbladder hydrops, hepatitis, and arthritis. Untreated Kawasaki disease can lead to coronary aneurysms and even myocardial infarction! Conjunctivitis Rash Adenopathy Strawberry tongue Hands and feet (red, swollen, f aky skin) BURN (fever > 40°C for ≥ 5 days) Subacute phase: Begins after the abatement of fever and typically lasts for an additional 2–3 weeks. Manifestations are thrombocytosis and elevated ESR. Untreated children may begin to develop coronary artery aneurysms (40%); all patients should be assessed by echocardiography at diagnosis. Kawasaki disease is an idiopathic medium-sized vasculitis that occurs primarily in infants and children younger than 5 years of age. Atypical Kawasaki disease applies to patients who do not fulfill the complete criteria of fever of 5 days or more with at least four of five features: bilateral conjunctival injection, changes in the lips and oral cavity, cervical lymphadenopathy, extremity changes, and polymorphous rash. Acute kidney injury is defined as a sudden decline in kidney function within hours, including structural injuries and loss of function. Acute kidney injury is extremely common in hospitalized pediatric patients. However, it is rarely documented in Kawasaki disease. Acute kidney injury is underestimated in Kawasaki disease due to the lack of a clear definition of age-specific normal serum creatinine levels and routine renal functions. This report describes a case who presented with clinical features suggestive of atypical Kawasaki disease and developed acute kidney injury. A 2-year-old Saudi girl had a history of high-grade fever for 5 days, moderate dehydration, dry cracked lips, poor appetite, and generalized erythematous rash; therefore, she was diagnosed to have incomplete Kawasaki disease. Laboratory investigations revealed normochromic normocytic anemia, leukocytosis, thrombocytosis, high inflammatory markers, and acute kidney injury stage III. An echocardiogram showed a 4-mm dilatation on the left main coronary artery and a 3-mm dilatation on the right. A renal biopsy was not performed to identify the cause of the injury as it showed improvements after the start of the specific therapy for Kawasaki disease; intravenous immune globulin at a dose of 2 g/kg, aspirin at a high dosage of 80 mg/kg/day, and prednisolone at 2 mg/kg. In addition to the acute kidney injury management, normal saline boluses were followed by furosemide at a 2 mg/kg dose. Her urine output increased, and her renal functions normalized. She was discharged in good condition after 10 days. It is valuable to check renal function tests in a confirmed case of Kawasaki disease to reduce the negative consequences of late acute kidney injury discovery. Early detection and intervention make a substantial difference in acute kidney injury management. Measles should be distinguished from similar presenting exanthemic diseases of childhood, autoimmune processes, and adverse drug reactions. Rubella causes a rash similar to measles with head to caudal distribution, mild respiratory symptoms, the absence of conjunctivitis. Still, it is accompanied by the presence of adenopathies - which is characteristic of this disease. Roseola is characterized by an illness beginning with a high fever, which subsides after a few days, accompanied by the appearance of a rash in the central part of the body, without the presence of Koplik's points. Mononucleosis is a febrile viral disease, a characteristic course with few symptoms during childhood, contrary to what happens in more advanced ages.  Mononucleosis manifests itself by pharyngeal compromise, polyadenopathy, and hepatosplenomegaly, and the rash can have different forms of presentation. In Kawasaki disease, there is an ocular compromise with the presence of conjunctivitis without exudate, and the respiratory compromise is not part of this pathology. Group A Streptococcus (particularly Scarlet fever) may present with a similar rash (a coarse, sandpaper-like, blanching, erythematous) to measles in association with pharyngitis. [5] [14] Kawasaki disease (KD) is a common vasculitic disorder usually seen in children below 5 years of age. The disease can present with protean clinical manifestations which include high grade fever (for at least 5 days), rash, redness of the lips and a typical strawberry tongue, cervical lymph node enlargement (often unilateral), swelling over the hands/feet and, later a characteristic peripheral desquamation over the fingers and toes. These clinical features appear sequentially and the findings may change from day-to-day. Thus, all these features may not be seen together at any one point of time. The diagnosis rests on the recognition of this characteristic temporal sequence of clinical events, none of which are, by themselves, pathognomonic. Establishing a diagnosis of KD may be further complicated by the occurrence of several other, seemingly unrelated, clinical features. These include irritability, neck stiffness, sterile pyuria, pneumonitis, hydrops of the gallbladder and hepatitis among many others. There is no laboratory test that can help in confirming a diagnosis of KD. Left untreated, up to 20% of children with KD can develop coronary aneurysms with catastrophic long term sequelae. It is important to diagnose KD in the first 10 days of the illness so that appropriate therapy with intravenous immunoglobulin and aspirin can be Initiated. All paediatricians, and physicians looking after children, need to be aware of this condition which is now being increasingly recognized in India. Kawasaki syndrome, also known as mucocutaneous lymph node syndrome, is an acute vasculitis of infants and young children. We describe a four-year-old girl who presented with fever, a diffuse erythematous maculopapular rash, bilateral nonpurulent bulbar conjunctivitis, dry, red, fissured lips, a tongue with a strawberry \"appearance\", an erythematous pharynx, indurative erythema, and edema and desquamation of the face, hands and feet. She probably developed mitral valve prolapse during the course of the disease. The diagnosis of Kawasaki syndrome was arrived at by excluding other diseases and by the presence of all the clinical criteria for Kawasaki syndrome. Since this syndrome is rarely encountered in Turkey, this case is presented and the literature regarding the syndrome is reviewed. Classically, five days of fever plus four of five diagnostic criteria must be met to establish the diagnosis. The criteria are: erythema of the lips or oral cavity or cracking of the lips rash on the trunk swelling or erythema of the hands or feet red eyes (conjunctival injection) swollen lymph node in the neck of at least 15 mm Many children, especially infants, eventually diagnosed with Kawasaki disease, do not exhibit all of the above criteria. In fact, many experts now recommend treating for Kawasaki disease even if only three days of fever have passed and at least three diagnostic criteria are present, especially if other tests reveal abnormalities consistent with Kawasaki disease. In addition, the diagnosis can be made purely by the detection of coronary artery aneurysms in the proper clinical setting. Investigations A physical examination will demonstrate many of the features listed above. This report describes a 26-year-old woman who fulfills the criteria for the diagnosis of Kawasaki disease or mucocutaneous lymph node syndrome, an acute febrile illness that usually afflicts young children. The diagnosis is made in persons with fever lasting 5 or more days when four of the following criteria are met: bilateral injection of ocular conjunctivae; the involvement of the mucous membranes of the upper respiratory tract consisting of any combination of the following--redness and fissuring of lips; \"strawberry tongue,\" or erythema of the pharynx; involvement of the peripheral extremities characterized in the early stages by an indurative erythematous rash of palms and soles followed by membranous desquamation; polymorphous nonvesicular truncal exanthem; and acute nonsuppurative enlargement of cervical lymph nodes. An added stipulation is that the illness must not be attributable to a known disease process. A 4-year-old boy experienced sudden fever, followed by a rash on the trunk and extremities and erythema of the pharynx. Five days later, the fever remained and erythema appeared on the oropharynx, tongue, and lips. The skin of the palms and soles became erythematous and indurated, and both conjunctivae became injected. Desquamation of the skin occurred on both thumbs and one finger, and an anterior cervical lymph node was found to be enlarged. The patient was diagnosed as having Kawasaki disease, and treatment with aspirin was started. The desquamation progressed to involve the entire surface of the palms and soles, and then symptoms resolved. Twenty years after recognition of Kawasaki disease, this enigmatic illness continues to defy attempts to understand its etiology and pathogenesis. Most experts agree that the cause is either an environmental toxin or an infectious agent, but other possible causative agents may need to be proposed and investigated. We describe adult-onset Kawasaki disease (KD) and review clinical manifestations and treatment guidelines. Our patient is a 20-year-old female who initially presented to an outside hospital for fever, cervical lymphadenopathy, malaise, exudative tonsillitis, and skin eruption. She received antibiotics for suspected exudative pharyngitis, but experienced continued fevers and presented to the UCLA emergency room one week later. She had diffuse petechial macules coalescing into reticulated patches, fingertip peeling, conjunctival injection, oral erosions, and tongue swelling. Despite her age, given her constellation of symptoms, a diagnosis of typical KD was favored. She was started on high dose aspirin and IVIG, with improvement of rash and conjunctivitis. She was discharged on 325mg of aspirin daily with close follow-up. This case highlights the challenge of diagnosing KD in adults. Although this patient had classic symptoms, she was likely misdiagnosed because KD is rare in adults and without validated criteria. Our patient met the pediatric criteria, suggesting these should be considered when clinical suspicion for adult-onset KD is high. Adult-onset KD is most commonly misdiagnosed as toxic shock syndrome or drug-induced hypersensitivity syndrome and these are important to rule-out. Treatment with high-dose aspirin and IVIG is well established and should be initiated promptly.\nHere is the question:\nDERMATOLOGY: A 4-year-old girl presenting with a high fever of 6 days' evolution. On clinical examination she presents an erythematous maculopapular rash on the trunk and genital area, with a tendency to confluence, without becoming scarlatiniform; conjunctival injection without secretions and red lips with raspberry tongue. She also presents erythema with edema in hands and feet and a unilateral cervical adenopathy of 2 cm in diameter. The most likely clinical diagnosis of suspicion is:\nHere are the potential choices:\n1. Kawasaki disease.\n2. Measles.\n3. Rubella.\n4. Scarlet fever.\nThe correct answer is: ", "gold_answer": "1 Kawasaki disease.", "prediction": "You are a helpful medical expert, and your task is to answer a multi-choice medical question using the relevant documents. Please choose the answer from the provided options. Your responses will be used for research purposes only, so please have a definite answer.\nHere are the relevant documents:\n The present report describes the severe evolution of Kawasaki disease in a three-month-old infant. The ailment was initially atypical in its presentation, with the patient exhibiting only persistent fever in association with a progressive lethargy and maculopapular rash on the face, trunk and limbs erroneously diagnosed as roseola infantum. On the 10th day of the condition, mainly due to the unexplained persistence of fever, the infant was admitted to a local hospital. The typical features of KD appeared only on the 14th day of illness with the relapse of the maculopapular rash in association with non-purulent conjunctivitis; dry, reddish and fissured lips; tongue with reddish and hypertrophic papillae; erythema and edema of the palms and soles. During the following days, the ailment rapidly evolved to a catastrophic clinical picture characterized by generalized vasculitis, splenic infarction, pulmonary thrombosis, giant right and left coronary aneurysms, dilatation of common and internal iliac arteries and progressive ischemia of the distal third of the feet resulting in necrotic lesions of both halluces. Appropriate therapy was initiated, but repeated administration of intravenous immunoglobulin G (IVIG) followed by three days of administration of methylprednisolone did not abate the intense inflammatory activity. The remission of inflammation and regression of vascular lesions were only achieved during the following five weeks after the introduction of methotrexate associated with etanercept. The report of this case aims to draw attention to severe forms of KD that exhibit an unfavorable evolution and can be extremely refractory to the conventional therapy. While the diagnosis of typical form of Kawasaki disease (KD) is obvious, this multifaceted disease continues to surprise us. We report the case of a recurrent Kawasaki disease in an infant. At the age of 13 months, the infant was diagnosed with complete Kawasaki disease; he presented with prolonged fever, bilateral conjunctivitis, enanthem, exanthema, edema of the lower limb, peeling, and biological inflammatory syndrome. He was treated with intravenous immunoglobulin (IVIG) associated with a high dose of aspirin and then an antiplatelet dose with a good clinical-biological evolution. The echocardiography was normal. Seven months later, the patient was again admitted, in a similar picture: a prolonged fever evolving for 7 days, bilateral conjunctivitis, enanthem, cervical adenopathy of 1.5 cm/1 cm, scarlatiniform erythema, pruriginous of the trunk and limb, and peeling of the toes, with indurated edema of the hands and feet. The rest of the examination was normal except the irritability. The diagnosis of recurrent KD was made according the five criteria of the American Heart Association. The echocardiography was normal again. The infant received IVIG with good outcome. Despite its rarity, the possibility of recurrence of KD should be known by clinicians, so as not to delay the specific management of vasculitis whose stakes in terms of prevention of coronary artery lesions are well known. Our case confirms the possibility of this recurrence. The cutaneous eruption in Kawasaki disease (Chap. 385) is polymorphous, but the two most common forms are morbilliform and scarlatiniform. Additional mucocutaneous findings include bilateral conjunctival injection; erythema and edema of the hands and feet followed by desquamation; and diffuse erythema of the oropharynx, red strawberry tongue, and dry fissured lips. This clinical picture can resemble TSS and scarlet fever, but clues to the diagnosis of Kawasaki disease are cervical lymphadenopathy, cheilitis, and thrombocytosis. The most serious associated systemic finding in this disease is coronary aneurysms secondary to arteritis. Scarlatiniform eruptions are also seen in the early phase of SSSS (see “Vesicles/Bullae,” above), in young adults with Arcanobacterium haemolyticum infection, and as reactions to drugs. Kawasaki disease is an acute inflammatory condition characterized by various combinations of features but renal involvement is rare. This report is of a case of Kawasaki disease complicated by acute kidney failure. A 10 year-old girl was admitted because of acute renal failure with fever. She developed a high fever, and her general condition was poor; she had developed a macular erythematous rash 10 days earlier for which she was given cefadroxil. At admission, she remained febrile and had strawberry tongue, pharyngitis, dry erythematous lips, bilateral conjunctivitis, cervical lymphadenopathy and desquamation of the skin on her hands. She was anemic (hemoglobin = 9.6 g%), leukocytotic (33,100/mm3), but with no burr, fragmented red blood cells or thrombocytopenia. Her plasma C-reactive protein level was 236 mg/l; her blood urea was 9.5 mmol/l, her creatininemia 288 mumol/l and proteinuria was 0.5 g/l without hematuria. Urine cultures did not grow. Her blood transaminase and gammaglutamyltransferase activities were elevated. Ultrasonography of kidneys and coronary arteries was normal. Kidney biopsy performed one day after admission showed no vascular or glomerular changes, but renal tubular necrosis, indicating urinary excretion of pigments. Tests for myoglobinemia, myoglobinuria and blood muscle enzyme activities were all positive. The renal failure disappeared within 10 days but the fever and inflammatory manifestations persisted for 1 1/2-2 months despite two treatments of intravenous gammaglobulins and continuous salicylate administration. The patient developed arthralgias at the end of the first month of disease, but recovered without renal or vascular complications. Several cases of renal involvement have been reported during the course of Kawasaki disease. They have been rarely documented by histological examination so that the vascular origin of changes has not been demonstrated. Myoglobinuria, as seen in muscular crush injury, and in our case possibly due to malignant hyperthermia, may be responsible for the transient acute renal failure. In 2014, the American Heart Association (AHA) published the criteria needed to establish a diagnosis. [1] However, it is important to note that children who fall short of the full criteria but have cardiac abnormalities on echocardiogram meet the diagnosis of KD. [1] The patient must have fevers for five or more days, with at least four of the following criteria (either all at once or over a series of days): Bilateral painless bulbar conjunctival injection without exudate Erythematous mouth and pharynx, strawberry tongue or red, cracked lips Polymorphous exanthem (morbilliform, maculopapular, or scarlatiniform) Swelling of hands and feet with erythema of the palms and soles Cervical lymphadenopathy (over 1.5 cm in diameter) Arthritis is reported in one-third of cases with Kawasaki syndrome. It may have an early or a late onset form. We present a 15-month-old-girl who had been referred with complaints of pain and swelling in her left shoulder. Physical examination revealed bulbar conjunctival injection, erythematous lips and pharynx, strawberry tongue, erythematous rash, edema and erythema of the left shoulder, left knee, right elbow and right wrist, and moderate distress in the left shoulder and left hip. She was diagnosed with Kawasaki syndrome, and intravenous immunoglobulin infusion (IVIG) 2 g/kg and aspirin (100 mg/kg/day) were instituted. The patient had two additional episodes of arthritis involving the hip joint on the 8th day, and the shoulder and metacarpophalangeal (MCP) and interphalangeal (IP) joints of her right hand on the 15th day. Turbid material was aspirated in both instances; Gram and Wright's staining of this material showed many leukocytes but no bacteria. A second dose of IVIG (1 g/kg) was given. At the end of the third week all extremities were painless, with a normal range of motion. Arthritis in our patient was the presenting sign, having a 'septic arthritis mimicking' and 'biphasic' pattern. Although the patient presented with severe and recurrent arthritis, which is significantly correlated with severe multisystem disease and the presence or development of coronary artery aneurysm, the response to IVIG was excellent. Kawasaki disease (KD) is a systemic vasculitis of unknown cause and is associated with various digestive disorders, although only a few cases of intussusception associated with KD have been reported. We describe a case of intussusception followed by KD in a 3-year-old boy. The patient was admitted to our hospital for evaluation of severe abdominal pain. Because the target sign was seen on ultrasonography, intussusception was diagnosed and hydrostatic reduction was performed. On the second day after admission, he developed a high fever (38°C) and an irregular rash over his whole body. On the fourth day after admission, the high fever continued, and bilateral nonexudative conjunctivitis, erythema of the lips and oral mucosa, strawberry tongue, indurated edema of the dorsa of the hands and feet, and diffuse erythema of the palms and soles appeared, and KD was ultimately diagnosed. He was treated with intravenous immunoglobulin 2 g/kg, aspirin 30 mg/kg/day, and prednisolone 2 mg/kg/day. The high fever and other clinical symptoms resolved immediately after the start of treatment. There was no relapse of KD symptoms after initial treatment, and periungual desquamation was observed on the 10th day after admission. He was discharged on the 15th day, without abnormalities such as coronary dilatation, 3 months after the onset of KD symptoms. Patients with intussusception and KD were older (≥3 years vs <3 years) than those with intussusception alone. In addition, the site of intussusception in KD was mainly colonic rather than ileocolic. If intussusception precedes development of the characteristic clinical symptoms of KD, diagnosis of KD may be delayed. KD should be considered in children older than 3 years with intussusception at a colonic site. A 14-month-old girl presented with a 4-d history of fever and generalized exanthema. Four characteristic symptoms of incomplete Kawasaki disease (KD) were present on admission (fever, rash, non-purulent conjunctival injection, oropharyngeal changes) and then followed by oedema of the hands and feet and mild plantar desquamation. The typical laboratory features of KD, such as elevated erythrocyte sedimentation rate, leukocytosis, thrombocytosis, and positive C-reactive protein were also seen. Ultrasound examination of the mediastinum revealed the presence of a lymph node, 30 mm in diameter, below the tracheal carina. Thoracic CT scan confirmed the mediastinal lymph node. The patient was treated with aspirin and intravenous gamma-globulin. Ultrasound study of the mediastinum, which was carried out 6 weeks after hospital discharge, showed that the lymph node had disappeared. This case illustrates that lymph nodes other than cervical lymphadenopathy should be sought when the diagnosis of classical or atypical KD is suspected. We report a 3-year-old girl with Kawasaki disease who presented with retropharyngeal edema and shock syndrome. This is the first reported case in Taiwan. The patient initially presented with fever, cough, and pyuria followed by rapidly progressive enlarged bilateral cervical lymphadenopathy. On the third day of the fever, computed tomography for airway compression sign found widening of the retropharyngeal space mimicking a retropharyngeal abscess. Later, an endotracheal tube was inserted for respiratory distress. A skin rash over her trunk was also noted. On the fifth day of the fever, the clinical course progressed to hypotension and shock syndrome. Because of more swelling of bilateral neck lymph nodes, computed tomography was arranged again and revealed partial resolution of the edematous changes in the retropharyngeal space. Edema of the hands and feet, bilateral bulbar conjunctivitis, and fissured lips were subsequently found. The diagnosis of Kawasaki disease was confirmed on the eighth day of fever. There was no evidence of bacterial infection. She was administered intravenous immunoglobulin (2 mg/kg) and high dose aspirin (100 mg/kg/day). One day later, the fever subsided, and her blood pressure gradually became stable. Heart echocardiography on the Day 13 revealed dilated left coronary artery and mitral regurgitation. Follow-up echocardiography six months later showed normal coronary arteries. To date, the patient has not experienced any complications. This case illustrates that retropharyngeal edema and shock syndrome can be present in the same clinical course of Kawasaki disease. Clinicians and those who work in intensive care units should be aware of unusual presentations of Kawasaki disease to decrease rates of cardiovascular complications. A 9-year-old girl developing fever and hyperemia of both bulbar conjunctiva 5 days before admission to the Saitama Children's Medical Center after antibiotics proved ineffective was found on admission to have general fatigue and a temperature of 39 degrees C. Physical examination showed hyperemia of the bulbar conjunctiva, fissures of the lips, redness of the pharynx, and swelling of the cervical lymph nodes. Laboratory tests detected neutrophilia (11,200/microL), mild anemia (11.4g/dL), thrombocytopenia (110,000/microL), and elevated serum aspartate aminotransferase (242IU/L), alanine aminotransferase (328IU/L), and C-rective protein (25.2 mg/dL). Autoantibodies such as anti-nuclear, anti-SS-A/Ro, and anti-Jo-1 were also found. Echocardiography showed no abnormality of the coronary arteries. She was diagnosed as having incomplete Kawasaki disease on day 7 of illness, necessitating that a high dose of immunoglobulin be given intravenously. Her temperature dropped temporarily to 37 degrees C, but she developed erythema of the cheek and fever. Intravenous immunoglobulin was restarted, and minocycline introduced because her daily contact with a pet cat indicated richettsial infection such as Q fever. Mild fever, muscle pain, and elevated C-reactive protein did not improve, but clinical signs and symptoms gradually lessened after ibuprofen was given, then disappeared. A definitive diagnosis of Q fever was made through an over 4-fold rise in phase II IgG antibody titers against Coxiella burnetii, titer of less than 1 : 16 on day 14 of illness, and titer of 1 : 256 on day 34. This case study describes on atypical case of Q fever with clinical manifestations mimicking Kawasaki disease. Erythema multiforme (EM) is an immune-mediated disease with mucocutaneous localization and plurietiologic determinism. The term \"multiforme\" refers to the variety of aspects that the lesions can take from patient to patient and during evolution in a single patient. We have selected 2 cases of small children diagnosed with different etiology of EM to illustrate the importance of a correct and fast diagnosis. Case 1 involves a 2-year-old girl from a rural area who presented with fever and pruritic erythematous papular eruption. The onset of the symptoms was 3 days before presentation with fever and ulcerative lesions on the oral and labial mucosa, followed by the appearance of erythematous macular lesions, with progressive confluence to intense pruritic patches. The 2nd involves a 2-year-old boy with fever, loss of appetite, productive cough, and petechiae. He had corticosensible immune thrombocytopenia from the age of 6 months, with many recurrences. The patient received treatment with ampicillin/sulbactam and symptomatics for an erythemato-pultaceous angina. During the 2nd day of treatment the patient developed an erythematous macular eruption on the face, scalp, trunk, and limbs, with bullae formation. The 1st patient was diagnosed based on biologic findings: positive inflammatory syndrome, elevated level of anti-Mycoplasma pneumoniae immunoglobulin M antibodies and immunoglobulin E. Histopathologic examination described papillary dermal edema, inflammatory infiltrate, and lymphocyte exocytosis. In the 2nd case, the hemoleucogram identified 12,000/mm platelets and the medulogram aspect was normal. Serology for Epstein-Barr virus was negative. The diagnosis was EM secondary to M pneumoniae infection in case 1 and secondary to administration of ampicillin/sulbactam in case 2. In both cases, etiopathogenic treatment consisting of steroidal antiinflammatory drugs, antihistamines was administered. Because of specific etiology, the 1st case received antibiotics. The evolution was favorable in 10 to 14 days; the patients were discharged after etiopathogenic treatment consisting of steroidal antiinflammatory drugs, antihistamines, and/or antibiotics. Performing a detailed clinical examination, medical history of drug use, infection or general diseases can establish a good diagnosis of EM. Histopathologic examination can help. The treatment is etiologic, pathogenic, and symptomatic. EM usually has a self-limited evolution. A previously healthy 11-month-old girl presented with fever and rash for 6 days. Physical examination revealed an irritable infant with a high fever, injected conjunctivae, red cracked lips, posterior auricular lymphadenopathy, hepatomegaly, generalized erythematous maculopapular rash and petechial hemorrhage on trunk, face and extremities. Complete blood count showed atypical lymphocytosis and thrombocytopenia. Dengue infection was initially diagnosed. The persistent fever and clinical manifestations of Kawasaki disease (KD) were observed on day 8 with high erythrocyte sedimentation rate (56 mm/hr). Treatment of KD included intravenous immunoglobulin on day 9 of the illness. Desquamation of the fingers was found on day 15 of the illness. Ectasia of left coronary artery with small aneurysmal dilatation was detected by echocardiography on day 15 of the illness. Hemagglutination-inhibition test and enzyme-linked immunosorbent assay for dengue virus eventually showed a four-fold rising. According to the literature review, this is the second reported case of dengue infection concomitant with KD. The natural course of each disease may be modified and causes some difficulties in diagnosis and management. Cervical lymphadenopathy (often painful and unilateral, with at least one node > 1.5 cm). Diffuse mucous membrane erythema (e.g., “strawberry tongue”); dry, red, chapped lips. Erythema of the palms and soles; indurative edema of the hands and feet; late desquamation of the fingertips (in the subacute phase). Other manifestations include sterile pyuria, gallbladder hydrops, hepatitis, and arthritis. Untreated Kawasaki disease can lead to coronary aneurysms and even myocardial infarction! Conjunctivitis Rash Adenopathy Strawberry tongue Hands and feet (red, swollen, f aky skin) BURN (fever > 40°C for ≥ 5 days) Subacute phase: Begins after the abatement of fever and typically lasts for an additional 2–3 weeks. Manifestations are thrombocytosis and elevated ESR. Untreated children may begin to develop coronary artery aneurysms (40%); all patients should be assessed by echocardiography at diagnosis. Kawasaki disease is an idiopathic medium-sized vasculitis that occurs primarily in infants and children younger than 5 years of age. Atypical Kawasaki disease applies to patients who do not fulfill the complete criteria of fever of 5 days or more with at least four of five features: bilateral conjunctival injection, changes in the lips and oral cavity, cervical lymphadenopathy, extremity changes, and polymorphous rash. Acute kidney injury is defined as a sudden decline in kidney function within hours, including structural injuries and loss of function. Acute kidney injury is extremely common in hospitalized pediatric patients. However, it is rarely documented in Kawasaki disease. Acute kidney injury is underestimated in Kawasaki disease due to the lack of a clear definition of age-specific normal serum creatinine levels and routine renal functions. This report describes a case who presented with clinical features suggestive of atypical Kawasaki disease and developed acute kidney injury. A 2-year-old Saudi girl had a history of high-grade fever for 5 days, moderate dehydration, dry cracked lips, poor appetite, and generalized erythematous rash; therefore, she was diagnosed to have incomplete Kawasaki disease. Laboratory investigations revealed normochromic normocytic anemia, leukocytosis, thrombocytosis, high inflammatory markers, and acute kidney injury stage III. An echocardiogram showed a 4-mm dilatation on the left main coronary artery and a 3-mm dilatation on the right. A renal biopsy was not performed to identify the cause of the injury as it showed improvements after the start of the specific therapy for Kawasaki disease; intravenous immune globulin at a dose of 2 g/kg, aspirin at a high dosage of 80 mg/kg/day, and prednisolone at 2 mg/kg. In addition to the acute kidney injury management, normal saline boluses were followed by furosemide at a 2 mg/kg dose. Her urine output increased, and her renal functions normalized. She was discharged in good condition after 10 days. It is valuable to check renal function tests in a confirmed case of Kawasaki disease to reduce the negative consequences of late acute kidney injury discovery. Early detection and intervention make a substantial difference in acute kidney injury management. Measles should be distinguished from similar presenting exanthemic diseases of childhood, autoimmune processes, and adverse drug reactions. Rubella causes a rash similar to measles with head to caudal distribution, mild respiratory symptoms, the absence of conjunctivitis. Still, it is accompanied by the presence of adenopathies - which is characteristic of this disease. Roseola is characterized by an illness beginning with a high fever, which subsides after a few days, accompanied by the appearance of a rash in the central part of the body, without the presence of Koplik's points. Mononucleosis is a febrile viral disease, a characteristic course with few symptoms during childhood, contrary to what happens in more advanced ages.  Mononucleosis manifests itself by pharyngeal compromise, polyadenopathy, and hepatosplenomegaly, and the rash can have different forms of presentation. In Kawasaki disease, there is an ocular compromise with the presence of conjunctivitis without exudate, and the respiratory compromise is not part of this pathology. Group A Streptococcus (particularly Scarlet fever) may present with a similar rash (a coarse, sandpaper-like, blanching, erythematous) to measles in association with pharyngitis. [5] [14] Kawasaki disease (KD) is a common vasculitic disorder usually seen in children below 5 years of age. The disease can present with protean clinical manifestations which include high grade fever (for at least 5 days), rash, redness of the lips and a typical strawberry tongue, cervical lymph node enlargement (often unilateral), swelling over the hands/feet and, later a characteristic peripheral desquamation over the fingers and toes. These clinical features appear sequentially and the findings may change from day-to-day. Thus, all these features may not be seen together at any one point of time. The diagnosis rests on the recognition of this characteristic temporal sequence of clinical events, none of which are, by themselves, pathognomonic. Establishing a diagnosis of KD may be further complicated by the occurrence of several other, seemingly unrelated, clinical features. These include irritability, neck stiffness, sterile pyuria, pneumonitis, hydrops of the gallbladder and hepatitis among many others. There is no laboratory test that can help in confirming a diagnosis of KD. Left untreated, up to 20% of children with KD can develop coronary aneurysms with catastrophic long term sequelae. It is important to diagnose KD in the first 10 days of the illness so that appropriate therapy with intravenous immunoglobulin and aspirin can be Initiated. All paediatricians, and physicians looking after children, need to be aware of this condition which is now being increasingly recognized in India. Kawasaki syndrome, also known as mucocutaneous lymph node syndrome, is an acute vasculitis of infants and young children. We describe a four-year-old girl who presented with fever, a diffuse erythematous maculopapular rash, bilateral nonpurulent bulbar conjunctivitis, dry, red, fissured lips, a tongue with a strawberry \"appearance\", an erythematous pharynx, indurative erythema, and edema and desquamation of the face, hands and feet. She probably developed mitral valve prolapse during the course of the disease. The diagnosis of Kawasaki syndrome was arrived at by excluding other diseases and by the presence of all the clinical criteria for Kawasaki syndrome. Since this syndrome is rarely encountered in Turkey, this case is presented and the literature regarding the syndrome is reviewed. Classically, five days of fever plus four of five diagnostic criteria must be met to establish the diagnosis. The criteria are: erythema of the lips or oral cavity or cracking of the lips rash on the trunk swelling or erythema of the hands or feet red eyes (conjunctival injection) swollen lymph node in the neck of at least 15 mm Many children, especially infants, eventually diagnosed with Kawasaki disease, do not exhibit all of the above criteria. In fact, many experts now recommend treating for Kawasaki disease even if only three days of fever have passed and at least three diagnostic criteria are present, especially if other tests reveal abnormalities consistent with Kawasaki disease. In addition, the diagnosis can be made purely by the detection of coronary artery aneurysms in the proper clinical setting. Investigations A physical examination will demonstrate many of the features listed above. This report describes a 26-year-old woman who fulfills the criteria for the diagnosis of Kawasaki disease or mucocutaneous lymph node syndrome, an acute febrile illness that usually afflicts young children. The diagnosis is made in persons with fever lasting 5 or more days when four of the following criteria are met: bilateral injection of ocular conjunctivae; the involvement of the mucous membranes of the upper respiratory tract consisting of any combination of the following--redness and fissuring of lips; \"strawberry tongue,\" or erythema of the pharynx; involvement of the peripheral extremities characterized in the early stages by an indurative erythematous rash of palms and soles followed by membranous desquamation; polymorphous nonvesicular truncal exanthem; and acute nonsuppurative enlargement of cervical lymph nodes. An added stipulation is that the illness must not be attributable to a known disease process. A 4-year-old boy experienced sudden fever, followed by a rash on the trunk and extremities and erythema of the pharynx. Five days later, the fever remained and erythema appeared on the oropharynx, tongue, and lips. The skin of the palms and soles became erythematous and indurated, and both conjunctivae became injected. Desquamation of the skin occurred on both thumbs and one finger, and an anterior cervical lymph node was found to be enlarged. The patient was diagnosed as having Kawasaki disease, and treatment with aspirin was started. The desquamation progressed to involve the entire surface of the palms and soles, and then symptoms resolved. Twenty years after recognition of Kawasaki disease, this enigmatic illness continues to defy attempts to understand its etiology and pathogenesis. Most experts agree that the cause is either an environmental toxin or an infectious agent, but other possible causative agents may need to be proposed and investigated. We describe adult-onset Kawasaki disease (KD) and review clinical manifestations and treatment guidelines. Our patient is a 20-year-old female who initially presented to an outside hospital for fever, cervical lymphadenopathy, malaise, exudative tonsillitis, and skin eruption. She received antibiotics for suspected exudative pharyngitis, but experienced continued fevers and presented to the UCLA emergency room one week later. She had diffuse petechial macules coalescing into reticulated patches, fingertip peeling, conjunctival injection, oral erosions, and tongue swelling. Despite her age, given her constellation of symptoms, a diagnosis of typical KD was favored. She was started on high dose aspirin and IVIG, with improvement of rash and conjunctivitis. She was discharged on 325mg of aspirin daily with close follow-up. This case highlights the challenge of diagnosing KD in adults. Although this patient had classic symptoms, she was likely misdiagnosed because KD is rare in adults and without validated criteria. Our patient met the pediatric criteria, suggesting these should be considered when clinical suspicion for adult-onset KD is high. Adult-onset KD is most commonly misdiagnosed as toxic shock syndrome or drug-induced hypersensitivity syndrome and these are important to rule-out. Treatment with high-dose aspirin and IVIG is well established and should be initiated promptly.\nHere is the question:\nDERMATOLOGY: A 4-year-old girl presenting with a high fever of 6 days' evolution. On clinical examination she presents an erythematous maculopapular rash on the trunk and genital area, with a tendency to confluence, without becoming scarlatiniform; conjunctival injection without secretions and red lips with raspberry tongue. She also presents erythema with edema in hands and feet and a unilateral cervical adenopathy of 2 cm in diameter. The most likely clinical diagnosis of suspicion is:\nHere are the potential choices:\n1. Kawasaki disease.\n2. Measles.\n3. Rubella.\n4. Scarlet fever.\nThe correct answer is: 1. Kawasaki disease." }, { "question": "You are a helpful medical expert, and your task is to answer a multi-choice medical question using the relevant documents. Please choose the answer from the provided options. Your responses will be used for research purposes only, so please have a definite answer.\nHere are the relevant documents:\n Medium vessel vasculitis These conditions are sometimes considered together with the small vessel vasculitides. Polyarteritis nodosa (PAN). Systemic necrotizing vasculitis and aneurysm formation affecting both medium and small arteries. If only small vessels are affected, it is called microscopic polyangiitis, although it is more associated with granulomatosis with polyangiitis than to classic PAN. At least 3 out of 10 criteria yields sensitivity and specificity of 82 and 87%: Unexplained weight loss > 4 kg Livedo reticularis Testicular pain Myalgias, weakness Abdominal pain, diarrhea, and GI bleeding Mononeuropathy or polyneuropathy New onset diastolic blood pressure > 90 mmHg Elevated serum blood urea nitrogen (> 40 mg/dL) or serum creatinine (> 1.5 mg/dL) Hepatitis B infection Arteriographic abnormalities Arterial biopsy showing polymorphonuclear cells Eosinophilic granulomatosis with polyangiitis (EGPA, Churg-Strauss syndrome) is a rare multisystem necrotizing vasculitis that involves small- to medium-sized blood vessels. We report a rare case of syndrome of the inappropriate antidiuretic hormone (ADH) secretion (SIADH) secondary to EGPA. A 53-year-old man applied with complaints of pain in the large joints and morning stiffness in knee for 2 months. The patient had the history of impaired fasting glucose, asthma, nasal polyps, and urticaria. Physical examination revealed intrinsic muscle atrophy and weakness in the right hand. Peripheral eosinophil count was 9.78 × 109/L (0.02-0.5), erythrocyte sedimentation rate 39 mm/h (0-20), and C-reactive protein 5.77 mg/dL (0-0.5). Migratory ground-glass pulmonary opacities had been reported in previous chest computed tomography scans. Echocardiography revealed findings compatible with eosinophilic involvement. Electroneuromyographic evaluation showed acute distal axonal neuropathy of right ulnar nerve. EGPA was considered. Oral methylprednisolone treatment was initiated. Intravenous immunoglobulin (IVIG) and cyclophosphamide treatment and gradual tapering of oral steroids were planned. In 24-h urine analysis, sodium was 387 mEq, creatinine was 1156 mg, and volume was 3000 mL. When his medical records were investigated, it was observed that hyponatremia was present for nearly 2 years. While serum osmolality was 270, urine osmolality was 604 mOsm/kg H1.58 mg/dl or 140 μmol/l) Proteinuria (>1 g/24h) Gastrointestinal hemorrhage, infarction, or pancreatitis Involvement of the central nervous system Cardiomyopathy Having none of these factors indicates milder case, with a five-year mortality rate of 11.9%. The presence of one factor indicates severe disease, with a five-year mortality rate of 26%, and three or more indicate very severe disease: 46% five-year mortality rate. In the setting of systemic vasculitis as described above, proliferative nephritis is associated with antineutrophil cytoplasmic antibodies (ANCA). Because of this, an ANCA test should always follow a negative immunofluorescence result to have the highest accuracy for confirming pauci-immune vasculitis-driven proliferative nephritis. Some cases of pauci-immune proliferative nephritis have no explanation and are thus deemed \"idiopathic.\" Peak incidences in 50- to 60-year-olds symptoms include intermittent fever / weight loss / shortness of breath / joint pain. See also Systemic vasculitis#Pauci-immune Goodpasture Syndrome and Poststrep Glomerulonephritis Microscopic polyangiitis, Eosinophilic granulomatosis with polyangiitis or Granulomatosis with polyangiitis References External links wikt:paucity Vascular-related cutaneous conditions Upon presentation, renal involvement is noted in only 10%-20%, but glomerulonephritis eventually develops in 80% of patients within two years of disease onset. The most common manifestation is rapidly progressive crescentic glomerulonephritis leading to chronic kidney disease or end-stage renal disease. Eye involvement: A 55-year-old male was admitted with non productive cough and fever which had continued for 6 weeks. The patient had symptoms of peripheral neuralgia. Chest X-ray revealed bilateral hilar lymphadenopathy (BHL) and reticular shadows in both lung fields. Other laboratory abnormalities included hematuria, RBC cast, high BUN, leukocytosis and thrombocytosis. Destruction of the internal membrane of arterioles was observed in a livedo reticularis on the right lower extremity. Renal angiography showed irregularity in the diameter, discontinuation and narrowing of peripheral arteries of both kidneys. These findings suggested the existence of \"angiitis\". These data were compatible with the diagnosis of polyarteritis nodosa (PN). Prednisolone (60 mg/day) administration resulted in the improvement of his symptoms and laboratory findings. A case of PN with lymph node swelling has been reported, however PN with BHL has not yet been reported. This is the first report of PN with BHL.\nHere is the question:\nRHEUMATOLOGY: A 67-year-old man presents with 3 months of asthenia and febrile fever, with nasal obstruction and mucus emission with some clots in the last month. In the last few days she noticed pain in the right eye and asymmetry with respect to the contralateral eye. Physical examination reveals proptosis of the right eyeball and inspection of the nostrils reveals an erythematous mucosa with serohematic crusts. The rest of the examination was normal. Blood tests (hemogram, renal and hepatic function) are normal, except for an ESR of 65 mm/h; urinalysis shows microhematuria and proteinuria of 520 mg/24h. What is the most probable initial diagnosis?\nHere are the potential choices:\n1. Eosinophilic granulomatosis with polyangiitis.\n2. Microscopic polyangiitis.\n3. Granulomatosis with polyangiitis.\n4. Polyarteritis nodosa.\nThe correct answer is: ", "gold_answer": "3 Granulomatosis with polyangiitis.", "prediction": "You are a helpful medical expert, and your task is to answer a multi-choice medical question using the relevant documents. Please choose the answer from the provided options. Your responses will be used for research purposes only, so please have a definite answer.\nHere are the relevant documents:\n Medium vessel vasculitis These conditions are sometimes considered together with the small vessel vasculitides. Polyarteritis nodosa (PAN). Systemic necrotizing vasculitis and aneurysm formation affecting both medium and small arteries. If only small vessels are affected, it is called microscopic polyangiitis, although it is more associated with granulomatosis with polyangiitis than to classic PAN. At least 3 out of 10 criteria yields sensitivity and specificity of 82 and 87%: Unexplained weight loss > 4 kg Livedo reticularis Testicular pain Myalgias, weakness Abdominal pain, diarrhea, and GI bleeding Mononeuropathy or polyneuropathy New onset diastolic blood pressure > 90 mmHg Elevated serum blood urea nitrogen (> 40 mg/dL) or serum creatinine (> 1.5 mg/dL) Hepatitis B infection Arteriographic abnormalities Arterial biopsy showing polymorphonuclear cells Eosinophilic granulomatosis with polyangiitis (EGPA, Churg-Strauss syndrome) is a rare multisystem necrotizing vasculitis that involves small- to medium-sized blood vessels. We report a rare case of syndrome of the inappropriate antidiuretic hormone (ADH) secretion (SIADH) secondary to EGPA. A 53-year-old man applied with complaints of pain in the large joints and morning stiffness in knee for 2 months. The patient had the history of impaired fasting glucose, asthma, nasal polyps, and urticaria. Physical examination revealed intrinsic muscle atrophy and weakness in the right hand. Peripheral eosinophil count was 9.78 × 109/L (0.02-0.5), erythrocyte sedimentation rate 39 mm/h (0-20), and C-reactive protein 5.77 mg/dL (0-0.5). Migratory ground-glass pulmonary opacities had been reported in previous chest computed tomography scans. Echocardiography revealed findings compatible with eosinophilic involvement. Electroneuromyographic evaluation showed acute distal axonal neuropathy of right ulnar nerve. EGPA was considered. Oral methylprednisolone treatment was initiated. Intravenous immunoglobulin (IVIG) and cyclophosphamide treatment and gradual tapering of oral steroids were planned. In 24-h urine analysis, sodium was 387 mEq, creatinine was 1156 mg, and volume was 3000 mL. When his medical records were investigated, it was observed that hyponatremia was present for nearly 2 years. While serum osmolality was 270, urine osmolality was 604 mOsm/kg H1.58 mg/dl or 140 μmol/l) Proteinuria (>1 g/24h) Gastrointestinal hemorrhage, infarction, or pancreatitis Involvement of the central nervous system Cardiomyopathy Having none of these factors indicates milder case, with a five-year mortality rate of 11.9%. The presence of one factor indicates severe disease, with a five-year mortality rate of 26%, and three or more indicate very severe disease: 46% five-year mortality rate. In the setting of systemic vasculitis as described above, proliferative nephritis is associated with antineutrophil cytoplasmic antibodies (ANCA). Because of this, an ANCA test should always follow a negative immunofluorescence result to have the highest accuracy for confirming pauci-immune vasculitis-driven proliferative nephritis. Some cases of pauci-immune proliferative nephritis have no explanation and are thus deemed \"idiopathic.\" Peak incidences in 50- to 60-year-olds symptoms include intermittent fever / weight loss / shortness of breath / joint pain. See also Systemic vasculitis#Pauci-immune Goodpasture Syndrome and Poststrep Glomerulonephritis Microscopic polyangiitis, Eosinophilic granulomatosis with polyangiitis or Granulomatosis with polyangiitis References External links wikt:paucity Vascular-related cutaneous conditions Upon presentation, renal involvement is noted in only 10%-20%, but glomerulonephritis eventually develops in 80% of patients within two years of disease onset. The most common manifestation is rapidly progressive crescentic glomerulonephritis leading to chronic kidney disease or end-stage renal disease. Eye involvement: A 55-year-old male was admitted with non productive cough and fever which had continued for 6 weeks. The patient had symptoms of peripheral neuralgia. Chest X-ray revealed bilateral hilar lymphadenopathy (BHL) and reticular shadows in both lung fields. Other laboratory abnormalities included hematuria, RBC cast, high BUN, leukocytosis and thrombocytosis. Destruction of the internal membrane of arterioles was observed in a livedo reticularis on the right lower extremity. Renal angiography showed irregularity in the diameter, discontinuation and narrowing of peripheral arteries of both kidneys. These findings suggested the existence of \"angiitis\". These data were compatible with the diagnosis of polyarteritis nodosa (PN). Prednisolone (60 mg/day) administration resulted in the improvement of his symptoms and laboratory findings. A case of PN with lymph node swelling has been reported, however PN with BHL has not yet been reported. This is the first report of PN with BHL.\nHere is the question:\nRHEUMATOLOGY: A 67-year-old man presents with 3 months of asthenia and febrile fever, with nasal obstruction and mucus emission with some clots in the last month. In the last few days she noticed pain in the right eye and asymmetry with respect to the contralateral eye. Physical examination reveals proptosis of the right eyeball and inspection of the nostrils reveals an erythematous mucosa with serohematic crusts. The rest of the examination was normal. Blood tests (hemogram, renal and hepatic function) are normal, except for an ESR of 65 mm/h; urinalysis shows microhematuria and proteinuria of 520 mg/24h. What is the most probable initial diagnosis?\nHere are the potential choices:\n1. Eosinophilic granulomatosis with polyangiitis.\n2. Microscopic polyangiitis.\n3. Granulomatosis with polyangiitis.\n4. Polyarteritis nodosa.\nThe correct answer is: 3. Granulomatosis with polyangiitis." }, { "question": "You are a helpful medical expert, and your task is to answer a multi-choice medical question using the relevant documents. Please choose the answer from the provided options. Your responses will be used for research purposes only, so please have a definite answer.\nHere are the relevant documents:\n Macular diseases in the elderly, such as age-related macular degeneration, idiopathic senile macular hole and epiretinal membrane of the macular area were studied. In 75 normal subjects aged from 20 to 78 years, retinal sensitivity in the central 10 degree visual field were examined using automated static quantitative perimetry. For background luminance of 31.5 asb, a significant reciprocal correlation was demonstrated between individual mean sensitivity and age. The influence of age on the decrease in sensitivity was proved to differ according to different test locations. To enhance contrast, an image processing procedure was applied for fluorescein angiographs of age-related pathologies which resulted in better recognition of age-related RPE pathologies were recognized. The senile disciform macular degeneration (SDMD) study group sponsored by the Ministry of Health and Welfare performed an epidemiological survey to estimate the number of patients with SDMD. The epidemiological estimation was 6,000 to 13,000 patients in the entire Japanese population. 133 eyes of uni- or bi-lateral senile macular degeneration without choroidal neovascularization and 156 opposite eyes of patients with unilateral SDMD were followed-up for choroidal neovascularization development. Choroidal neovascularization development was confirmed in 15 eyes, 5.2%. In 13 of the 15 eyes, choroidal neovascularization was proved to develop through serous RPE detachment. Also, serous drusen were shown to be to predisposed to choroidal neovascularization through serous RPE detachment. Therefore, it was concluded that senile macular degeneration should be classified into the atrophic form, predisciform or intermediate form and disciform form. In the author's previous paper, it was reported that the navel-like lesion would be a macular lesion predisposing to a senile macular hole. 49 opposite eye of patients with one eye affected by a macular hole were follow-up for macular hole development. At the initial examination, the navel-like lesion was observed in 5 of the 49 eyes. During the course of observation, navel-like lesions developed in one of the other 27 eyes with other abnormalities and in 4 of the 17 eyes without any abnormality. Finally, macular holes developed in 11 of the 49 eyes; in 10 eyes with a navel-like lesion and one eye with another abnormality. We found a 17 year old female Japanese monkey with pre-macular holes in both eyes and clinicopathological correlative study was carried out. In her right eye, photoreceptor cell loss at the foveola, circumferential retinal detachment around the area of cell loss, cystoid spaces in the detached retina, and very thin residual tissue covering the foveolar lesion were observed.(ABSTRACT TRUNCATED AT 400 WORDS) Drusen are important risk factor for neovascular age-related macular degeneration (AMD) and have a dynamic nature as they can enlarge, newly form, or disappear over time. There have been few reports on drusen regression or choroidal neovascularization (CNV) development after macular hole surgery. We report, to our knowledge, the first case of both drusen regression and subsequent CNV development within 7 months of successful macular hole surgery. A 73-year-old woman presented with a stage 3 full-thickness macular hole and large, confluent soft macular drusen in the right eye and a neovascular age-related macular degeneration (AMD) in the fellow eye. Four months after the successful macular hole surgery, significant regression of drusen was seen, especially in the temporal area to the fovea. Three months later, the patient developed CNV and her best-corrected visual acuity decreased to 20/100, despite further regression of macular drusen. Macular hole patients with macular soft drusen need to be carefully followed up after surgery for possible drusen regression and CNV development. To describe functional and anatomic results obtained by treatment with photodynamic therapy (PDT) or intravitreal bevacizumab (Avastin, Roche) in macular serous retinal detachment associated with tilted disk syndrome. Three eyes of 3 patients with symptomatic macular serous detachment associated with tilted disc syndrome (optic disc with an oblique axis, inferonasal crescent, and inferior staphyloma) were treated. In all patients, best-corrected visual acuity (BCVA) was tested and fluorescein angiography (FA) and optical coherence tomography (OCT) were performed before and about 45 days after treatment. All patients underwent a complete ophthalmologic examination including OCT at least 6 months after treatment. The first patient was treated with one low fluence (300 mW/cm2 for 83 seconds) PDT (6 months follow-up). The second patient was treated with 3 intravitreal injections of bevacizumab 1.25 mg (33 months follow-up) and the third patient was treated with 2 low fluence PDTs at 4 months and, after 1 year, 3 intravitreal injections of bevacizumab 1.25 mg (37 months follow-up). Before treatment, all patients complained of visual loss and metamorphopsia. The OCT showed in the macular area a focal neurosensory detachment with foveal involvement. The FA showed in the macular area multiple focal areas of hyperfluorescence due to pigment epithelium atrophy and in the second and third patient also a hyperfluorescent pinpoint with minimal leakage. After treatment in all eyes, symptoms did not change, BCVA remained stable, and in OCT the foveal neuroretinal detachment was changeless. In FA, no noticeable variation of the hyperfluorescence areas was appreciated. In the second patient, the hyperfluorescent point remained unvaried, and the same occurred in the third patient after the first PDT, while after the second PDT a new leaking dot disappeared. Macular serous retinal detachment was first described in 1998 as an uncommon complication of tilted disc syndrome showing angiographic and OCT features similar to a chronic central serous chorioretinopathy. In contrast to this pathology, in our patients treatment with PDT or intravitreal bevacizumab did not succeed, probably because of a different pathogenesis of macular serous detachment. Further investigations are needed to clarify the proper therapy of this disease. Acquired vitelliform lesions (AVLs) are associated with age-related macular degeneration and other variable macular disorders. AVLs often lead to outer retinal atrophy, sometimes accompanying a macular hole and choroidal neovascularization. The purpose of this study was to report a rare case with bilateral AVLs, in which one eye had accompanied a macular hole and the second eye a serous pigment epithelial detachment (sPED). A 66-year-old woman complained of bilateral metamorphopsia. AVLs were observed in the right eye and a flat sPED in the left eye. The best-corrected visual acuity (BCVA) was 20/17 in both eyes. Fluorescein angiography revealed local leakage in the right eye and pattern dystrophy-like hypofluorescence in both eyes. The sPED progressed with AVLs in the left eye and was treated with a combination therapy of intravitreal aflibercept, a sub-Tenon's injection of triamcinolone acetonide, and photodynamic therapy (IVA/STTA/PDT), which successfully flattened the sPED and sustained good vision for 4 years. The right eye was treated with intravitreal ranibizumab and tissue plasminogen activator, which enhanced absorption of the vitelliform material. However, 14 months later, a macular hole with typical metamorphopsia formed above a subretinal fibrotic scar at the vitelliruptive stage. Although pars plana vitrectomy closed the macular hole, enlargement of the outer retinal atrophy worsened the BCVA to 20/100. We successfully treated one eye with a sPED with AVLs using the combination therapy of IVA/STTA/PDT, while the second eye with a macular hole secondary to AVLs ultimately developed outer retinal atrophy with visual loss. Clinical significance Amsler grid can be used in detecting central visual field defects in following conditions: Age-related macular degeneration: The grid will help detecting the progression of AMD from dry form to wet form. Chance of metamorphopsia is more in wet AMD compared to dry form. Choroidal neovascular membranes: Choroidal neovascular membranes cause scotoma and metamorphopsia. It may be associated with many diseases like macular degeneration, POHS, myopic macular degeneration, trauma etc. Central serous chorioretinopathy: CSCR Causes round or oval scotoma. Macular pucker: Macular pucker also known as an epiretinal membrane cause metamorphopsia and distortions in central field of vision. Cystoid macular edema: Due to macular edema, micropsia may occur. Glaucoma: Amsler grid is useful in detecting central field defects in moderate to severe glaucoma. Macular sparing: Amsler Grid can be used to detect and accurately measure macular sparing. Types Recent advancement in OCT has made it simpler to differentiate a macular hole from other similar types of smaller diameter macular pathologies of old age. From history and physical examination, round central small reddish lesions with dimness of central vision, probable differential diagnosis include: Epiretinal membrane (ERM) with a macular pseudo hole Central foveal dot hemorrhage Lamellar (aborted) macular hole Vitreomacular traction syndrome (VMTS) Foveal drusen Central areolar pigment epitheliopathy Solar retinopathy Small choroidal neovascular membrane involving center Small central serous chorioretinopathy involving center Cystoid macular edema (CME) Other types There are a few other (rare) kinds of macular degeneration with similar symptoms but unrelated in etiology to Wet or Dry age-related macular degeneration. They are all genetic disorders that may occur in childhood or middle age. Vitelliform macular dystrophy Sorsby's fundus dystrophy is an autosomal dominant, retinal disease characterized by sudden acuity loss resulting from untreatable submacular neovascularisation Stargardt's disease (juvenile macular degeneration, STGD) is an autosomal recessive retinal disorder characterized by juvenile-onset macular dystrophy, alterations of the peripheral retina, and subretinal deposition of lipofuscin-like material. Similar symptoms with a very different etiology and different treatment can be caused by epiretinal membrane or macular pucker or any other condition affecting the macula, such as central serous retinopathy. Notable cases Judi Dench Joan Plowright Peter Sallis June Brown S. Robert Morgan See also Treatment and Prognosis Metamorphopsia is a symptom of several common retinal and macular diseases, therefore treating the underlying disorder can improve symptoms. For people who have conditions such as Epiretinal membrane (ERM), Macular Holes and Retinal Detachment, decreased metamorphopsia is associated with an increase in visual acuity. Quantitative evaluation of metamorphopsia is an important step in understanding visual functions of individuals with macular disorders and is an essential tool for physicians in evaluating treatment results. Types Dry (non-exudative, > 80%)—deposition of yellowish extracellular material in and between Bruch's membrane and retinal pigment epithelium (“drusen”) with gradual loss in vision. Wet (exudative, 10–15%)—rapid loss of vision due to bleeding secondary to choroidal neovascularization. Etymology Gk, meta + morphe, form, opsis, sight See also Dysmorphopsia Hallucination References Visual disturbances and blindness Purpose. Myopic macular holes can be difficult to close with surgery and are frequently associated with retinal detachment. We report on a case of a macular hole in a severely myopic eye that underwent spontaneous closure. Methods. An observational case study. Results. A 55-year-old female was referred to Ophthalmology for a central scotoma and metamorphopsia in the right eye. Visual acuity was 1/20 in both eyes. Fundus examination showed loss of the foveal depression, with a small yellow ring in the center of the fovea in the right eye, and a tilted optic disc and peripapillary staphyloma bilaterally. Spectral domain optical coherence tomography (SD-OCT) revealed a fully developed macular hole with a rim of thickened and slightly elevated retina in the right eye. The patient refused surgery. After 4 years of follow-up, her visual acuity improved to 20/40 in the right eye, and SD-OCT revealed spontaneous sealing of the macular hole without bare retinal pigment epithelium. Conclusions. Myopic macular holes represent a challenge regarding their management, and the prognosis is often poor. To characterize retinal and fluorescein angiographic findings of Asian patients with symptoms secondary to age-related macular degeneration (ARMD). We retrospectively reviewed 453 consecutive medical records corresponding to fluorescein angiograms performed between November 1992 and November 1995 to identify Asian patients with symptomatic ARMD. Presenting visual symptoms, best-corrected Snellen visual acuities, and retinal examination findings were determined from the medical records. Fundus photos and fluorescein angiograms were reviewed. There were 26 symptomatic eyes in 19 Asian patients with a median age of 73 years. Presenting visual symptoms included decreased visual acuity (19 eyes), metamorphopsia (5 eyes), or scotoma (2 eyes). Retinal findings included occult choroidal neovascularization (CNV) in 5 (19%) eyes, serous pigment epithelial detachment (PED) in 8 (31%) eyes, PED with CNV in 5 (19%) eyes, drusen in 5 (19%) eyes, retinal pigment epithelial atrophy in 1 (4%) eye, vitreous hemorrhage in 1 (4%) eye, and a disciform scar in 1 (4%) eye. In this cohort of Asian patients with ARMD, the majority of symptomatic eyes had either CNV (46%) or serous PED (31%). To study the clinical features and the pathogenesis of macular hole with and without retina detachment (RD) in high myopic eyes. It was a retrospective series case study. The charts of high myopic patients with macular hole at our hospital from June 2006 to February 2007 were retrospectively reviewed and analyzed. Patients were divided into two groups (the RD group and non-RD group) depending on the presence of RD or not. Their clinical data and optic coherence tomography (OCT) results were further analyzed. SPSS 13.0 was used for the statistic analysis. When comparing the quantitative aspects like age, axial length and refraction, t-test was used. Categorical data, such as sex ratio, occurrence of vitreous traction, posterior staphyloma and retinoschisis were compared by using χ(2) test. Fisher's test was used in comparing eye laterality, incidence of white hole, visual acuity and posterior vitreous detachment (PVD). During this period, there were 43 patients fitting the including criteria. Among them, 36 patents (37 eyes) were in the RD group and 7 patients (7 eyes) in the no-RD group. In the RD group, the average age was 56.1, 24.3% of them (9/37) were male; percentage of left and right eyes was (11/37) and 70.3% (26/37), respectively; average refraction was (-8.9 ± 2.2) D; average axial length was (28.7 ± 2.0) mm. Visual acuity was ≤ 0.05 (72.2%) in 26 eyes and 0.05 - 0.2 (27.8%) in 10 patients. The incidence of complete and non-complete PVD was 89.2% (33/37) and 10.8% (4/37), respectively. White hole presented in 35.1% (3/37) patients. Vitreous traction and retinoschisis presented in 27.0% (10/37) and 35.1% (13/37) patients, respectively. In the non-RD group, the average age was 47.6; 16.7% of them (1/7) were male; left and right eyes were involved in 42.9% (3/7) and 57.1% (4/7), respectively. Average refraction was (-9.0 ± 1.9) D; average axial length was (28.9 ± 1.5) mm. Vision acuity was ≤ 0.05 in 3 patients (42.9%); between 0.05 - 0.2 in 3 eyes (42.9%) and ≥ 0.2 in 1 eye (14.3%). Incidence of complete and non-complete PVD was 85.7% (6/7) and 14.3% (1/7), respectively. White hole was observed in 14.3% (1/7) patients; 42.9% (3/7) patients were accompanied with vitreous traction and 71.4% (5/7) with retinoschisis. B-scan ultrasonography showed posterior staphyloma in all 44 eyes. The results of statistical analysis showed that the gender (χ(2) = 0.008) and eye laterality (χ(2) = 0.449) as well as refraction (t = 0.193), axial length (t = -0.25) and visual acuity (χ(2) = 4.509) of these two groups were similar (P > 0.05). The incidences of vitreous traction (χ(2) = 0.709), white hole (χ(2) = 1.179), PVD (χ(2) = 0.071) and retinoschisis (χ(2) = 3.207) were also similar (P > 0.05). But the age of the non-RD group is significantly younger than the RD group (t = 1.66, P < 0.05). Various pathogenesis may involved in the occurrence of retinal detachment in highly myopic eyes with macular hole. Further study is required to improve our understanding of this entity. The aim of this case report is to describe a case of atypical central serous chorioretinopathy (CSCR) definitively diagnosed after 8 years. A 44-year-old woman presented with reduced visual acuity in her left eye. Her visual acuity was light perception with projection in the right eye and 0.15 in the left. She described a similar decline in vision in her right eye 8 years ago. At that time, she had exudative retinal detachment and was treated with systemic immunosuppressive therapy for a presumed diagnosis of Vogt-Koyanagi-Harada disease. Despite resolution of the exudative retinal detachment, macular scarring developed. Eight years later, she developed inferior exudative retinal detachment in the left eye. A diagnosis of atypical CSCR was made with the help of multimodal imaging and her left eye was successfully treated with eplerenone and half-fluence photodynamic therapy (hf-PDT). In conclusion, early diagnosis and treatment of atypical CSCR may prevent subretinal fibrosis formation and permanent vision loss. Hf-PDT and eplerenone are successful treatment options for atypical CSCR. Self-separation or peeling of an idiopathic epiretinal membrane (ERM) in an eye with partial posterior vitreous detachment (PVD) is a rare event. A 56-year-old woman presented to our clinic with complaints of floaters in her right eye. Best-corrected visual acuity (BCVA) was 9/10 in this eye. Fundus examination and Spectral domain optical coherence tomography (SD-OCT) revealed an idiopathic ERM and Grade 3 PVD in this eye. Four months later, she had complaints of metamorphopsia in her right eye. BCVA was 7/10, while SD-OCT images of the right macula were similar to previous images. One week after the last visit, she presented again due to the sudden disappearance of her metamorphopsia complaints. BCVA had improved to 10/10. Fundus examination demonstrated that the ERM had spontaneously separated from the retinal surface as a flap floating in the vitreous and the foveal contour had returned to normal. The etiologic mechanism may be explained as the contracting forces within an immature ERM being stronger than its adhesion to the retina. The pathogenesis of the macular serous retinal detachment (SRD) associated with congenital optic disc pit remains controversial. The treatment is also discussed. Through this study, which includes the majority of the techniques available, we report our experiment in the treatment of this pathology. This was a retrospective single-centre study of 20 patients who presented with macular SRD associated with optic disc pit between 1983 and 2009. Various treatments were provided. At the beginning of the study, patients were treated only by juxtapapillary laser photocoagulation. After laser failure then as first-line treatment, laser photocoagulation was associated with intravitreal gas (C3F8) injection with postoperative facedown positioning for 2 weeks. During the past few years, all patients have been systematically treated with vitrectomy with or without internal limiting membrane (ILM) peeling, laser, and gas (C2F6) tamponade. This series consisted of 20 patients: nine men and 11 women. The patients' mean age at presentation was 29 years (range, 9-60 years). The mean time between the onset of the decrease in visual acuity (VA) and treatment was 6.1 months. None of these patients had a posterior vitreous detachment at the time of diagnosis. Six patients were treated by laser photocoagulation alone, which was successful only in two cases. Eleven patients (with laser treatment failure in three) were treated by laser and intravitreal gas injection, with a 72% success rate. We performed vitrectomy with posterior hyaloid dissection, laser, and gas tamponade in eight cases (with laser-gas treatment failure in two) with 87% success rate and no recurrence. Five of these patients had ILM peeling during the vitrectomy. The mean follow-up period was 60 months (range, 2 months to 17 years). This study shows that early treatment of macular SRD associated with optic disc pit by vitrectomy, ILM peeling, juxtapapillary photocoagulation, and gas tamponade is followed by good anatomical and functional results. This treatment is superior to the other less invasive procedures. Optical coherence tomography is an important exam for diagnosis and postoperative follow-up of patients. Central serous choroidopathy is a macular disease, usually with a self-limited and benign course, and predominantly affects male patients between 20 and 45 years old. A 68 year-old female patient complained of decreased visual acuity of her right eye of approximately 3 weeks of onset. Best corrected visual acuity in her right eye was 20/100. Fundus examination revealed a macular serous detachment involving its centre, as well as the presence of multiple calcified drusen. Fluorescein angiography showed late parafoveal leakage in a \"smokestack\" pattern in the right macular area. Optical coherence tomography showed a dome-shape macular detachment, also in the right eye. The patient was observed every 2 weeks and spontaneous resolution of the macular detachment was seen a month later. Based on these clinical features, a diagnosis was made of central serous choroidopathy of atypical presentation. Atypical presentation cases of serous central choroidopathy might be seen occasionally. Hence, it is an important differential diagnosis of age related macular degeneration in patients older than 60 years. To evaluate a macular buckle for exudative choroidal neovascularization secondary to age-related macular degeneration (ARMD). Forty-two eyes with choroidal neovascular membranes (CNVM) secondary to ARMD underwent surgical placement of a macular buckle. A Gore-Tex strip (2.0-2.5 mm wide) was button-holed through a 5 mm diameter silicone sponge (9 mm long) and placed behind the macula underneath the CNVM by the same surgeon (Dr Peyman) in all cases. Follow-up ranged from 7-76 months (mean, 20.9 months). Of 12 eyes with classic subfoveal CNVM: 4 (33%) gained 2 or more lines of Snellen visual acuity; 3 (25%) gained 1 line, remained the same, or lost 1 line; and 5 (42%) lost 2 or more lines (range + 6 to - 6 lines). Of 22 eyes with ill-defined subfoveal CNVM: 12 (54%) gained 1 line, remained the same, or lost 1 line; and 10 (46%) lost 2 or more lines (range + 1 to - 8 lines). Eight eyes with ill-defined juxtafoveal CNVM had the following visual acuity outcomes: 5 eyes (62%) maintained the same level of Snellen visual acuity (gained 1, 0, or lost 1 line); and 3 (38%) got worse (lost 2 or more lines of Snellen visual acuity, range + 1 to - 6 lines). Ten eyes (24%) bled subretinally during the follow-up period (average 11.5 months, range 14 days to 27 months), all outside the area of indentation of the macular buckle. The macular buckle treatment for exudative subretinal choroidal neovascular membranes in ARMD stabilized visual decline and displaced significant subfoveal hemorrhage. To report the pathogenic factors that account for cystoid macular edema and cystoid macular degeneration in chronic central serous chorioretinopathy (CSC). The clinical course and multimodal imaging findings, including fundus color photography, fundus autofluorescence, spectral-domain optical coherence tomography, and fluorescein angiography, of one eye with cystoid macular edema due to chronic CSC was documented. A 44-year old woman with a history of chronic CSC presented with progressive visual decline in the right eye. Best-corrected visual acuity was 20/40. Funduscopic examination revealed diffuse retinal pigment epithelial changes and macular edema. Fluorescein angiography demonstrated perifoveal microaneurysms and leakage in a petaloid configuration. Spectral-domain optical coherence tomography demonstrated cysts at the level of the inner nuclear layer, an epiretinal membrane, vitreomacular traction, and an attenuated retinal pigment epithelial band. Central subfield thickness was 486 μm. Three intravitreal injections of aflibercept were administered over 16 weeks following which there was resolution of leakage, release of vitreomacular traction, and resolution of microaneurysms. Central subfield thickness reduced to 379 μm, but persistent intraretinal cysts were observed. There was subjective improvement in visual symptoms, but Snellen acuity remained at 20/40. Intraretinal cystic changes in chronic CSC may be the result of multifactorial pathogenic factors and may represent the coexistence of cystoid macular edema and cystoid macular degeneration. Anti-vascular endothelial growth factor may play an important role in the treatment of cystoid macular edema caused by CSC. A patient developed choroidal neovascularization (CNV) in one eye during treatment for bilateral recurrent central serous chorioretinopathy (CSC) and was intravitreously injected with bevacizumab; she developed multiple evanescent white dots and serous retinal detachment(SRD). A 50-year-old women had a history of CSC OD at the age of 29 years. On initial examination, CSC OD was noted, and multiple detachments of the retinal pigment epithelium OU were observed. While the CSC in the right eye was successfully treated by laser photocoagulation, it spread to both eyes following this episode. Examination of the right eye by optical coherence tomography (OCT) following the recurrence of the CSC showed slight elevation of the retinal pigment epithelial layer in the central fovea, but this finding disappeared with the resolution of the CSC. However, as the CSC combined with CNV (Gass type 2) recurred within 1 year, the patient was intravitreously injected with bevacizumab. On the day following the injection, SRD OD occurred, and on the 7th day following the injection many white lesions varying in size appeared in the deep layer of the retina, but they healed 3 weeks later, leaving only the CNV. The CNV was cured later by additional photodynamic therapy. Since the lesions of the fundus observed immediately after the bevacizumab administration resolved spontaneously without sequelae, they were retrospectively diagnosed as a white dot syndrome-like disease. The white dot syndrome-like disease is suggested as a rare complication of bevacizumab. Central serous chorioretinopathy (CSCR) is an idiopathic condition characterized by serous detachment of the neurosensory retina in the macular region. It is relatively uncommon in Africans and though pregnancy is a known risk factor, there are no previous reports of CSCR in pregnant African women. We report the case of a 35-year-old pregnant woman who presented to our clinic at gestational age of 29 weeks with a 4 months history of blurring of vision in her left eye. Examination revealed visual acuity of 6/4 on the right eye and 6/9 on the left eye. She had normal anterior segments bilaterally and a normal posterior segment on the right. However, she had left macular edema with exudates. There was no significant refractive error. Her blood pressure was normal. Investigations including electrolytes and urea, urinalysis, and blood sugar profile were all normal. She was managed conservatively, and symptoms resolved 2 weeks prior to delivery. This is a case report of CSCR in a pregnant Nigerian woman with spontaneous resolution of symptoms prior to delivery. Pregnant women should be educated about the possibility of visual problems accompanying pregnancy. \nHere is the question:\nOPHTHALMOLOGY: 75-year-old woman diagnosed 3 years ago with soft drusen in the fundus. She reports presenting, since 2 weeks ago, metamorphopsia and significant visual loss in her right eye that prevents her from reading. Indicate the most probable diagnosis:\nHere are the potential choices:\n1. Macular epiretinal membrane.\n2. Macular hole.\n3. Thrombosis of the central retinal vein.\n4. Senile macular degeneration.\n5. Central serous chorioretinopathy.\nThe correct answer is: ", "gold_answer": "4 Senile macular degeneration.", "prediction": "You are a helpful medical expert, and your task is to answer a multi-choice medical question using the relevant documents. Please choose the answer from the provided options. Your responses will be used for research purposes only, so please have a definite answer.\nHere are the relevant documents:\n Macular diseases in the elderly, such as age-related macular degeneration, idiopathic senile macular hole and epiretinal membrane of the macular area were studied. In 75 normal subjects aged from 20 to 78 years, retinal sensitivity in the central 10 degree visual field were examined using automated static quantitative perimetry. For background luminance of 31.5 asb, a significant reciprocal correlation was demonstrated between individual mean sensitivity and age. The influence of age on the decrease in sensitivity was proved to differ according to different test locations. To enhance contrast, an image processing procedure was applied for fluorescein angiographs of age-related pathologies which resulted in better recognition of age-related RPE pathologies were recognized. The senile disciform macular degeneration (SDMD) study group sponsored by the Ministry of Health and Welfare performed an epidemiological survey to estimate the number of patients with SDMD. The epidemiological estimation was 6,000 to 13,000 patients in the entire Japanese population. 133 eyes of uni- or bi-lateral senile macular degeneration without choroidal neovascularization and 156 opposite eyes of patients with unilateral SDMD were followed-up for choroidal neovascularization development. Choroidal neovascularization development was confirmed in 15 eyes, 5.2%. In 13 of the 15 eyes, choroidal neovascularization was proved to develop through serous RPE detachment. Also, serous drusen were shown to be to predisposed to choroidal neovascularization through serous RPE detachment. Therefore, it was concluded that senile macular degeneration should be classified into the atrophic form, predisciform or intermediate form and disciform form. In the author's previous paper, it was reported that the navel-like lesion would be a macular lesion predisposing to a senile macular hole. 49 opposite eye of patients with one eye affected by a macular hole were follow-up for macular hole development. At the initial examination, the navel-like lesion was observed in 5 of the 49 eyes. During the course of observation, navel-like lesions developed in one of the other 27 eyes with other abnormalities and in 4 of the 17 eyes without any abnormality. Finally, macular holes developed in 11 of the 49 eyes; in 10 eyes with a navel-like lesion and one eye with another abnormality. We found a 17 year old female Japanese monkey with pre-macular holes in both eyes and clinicopathological correlative study was carried out. In her right eye, photoreceptor cell loss at the foveola, circumferential retinal detachment around the area of cell loss, cystoid spaces in the detached retina, and very thin residual tissue covering the foveolar lesion were observed.(ABSTRACT TRUNCATED AT 400 WORDS) Drusen are important risk factor for neovascular age-related macular degeneration (AMD) and have a dynamic nature as they can enlarge, newly form, or disappear over time. There have been few reports on drusen regression or choroidal neovascularization (CNV) development after macular hole surgery. We report, to our knowledge, the first case of both drusen regression and subsequent CNV development within 7 months of successful macular hole surgery. A 73-year-old woman presented with a stage 3 full-thickness macular hole and large, confluent soft macular drusen in the right eye and a neovascular age-related macular degeneration (AMD) in the fellow eye. Four months after the successful macular hole surgery, significant regression of drusen was seen, especially in the temporal area to the fovea. Three months later, the patient developed CNV and her best-corrected visual acuity decreased to 20/100, despite further regression of macular drusen. Macular hole patients with macular soft drusen need to be carefully followed up after surgery for possible drusen regression and CNV development. To describe functional and anatomic results obtained by treatment with photodynamic therapy (PDT) or intravitreal bevacizumab (Avastin, Roche) in macular serous retinal detachment associated with tilted disk syndrome. Three eyes of 3 patients with symptomatic macular serous detachment associated with tilted disc syndrome (optic disc with an oblique axis, inferonasal crescent, and inferior staphyloma) were treated. In all patients, best-corrected visual acuity (BCVA) was tested and fluorescein angiography (FA) and optical coherence tomography (OCT) were performed before and about 45 days after treatment. All patients underwent a complete ophthalmologic examination including OCT at least 6 months after treatment. The first patient was treated with one low fluence (300 mW/cm2 for 83 seconds) PDT (6 months follow-up). The second patient was treated with 3 intravitreal injections of bevacizumab 1.25 mg (33 months follow-up) and the third patient was treated with 2 low fluence PDTs at 4 months and, after 1 year, 3 intravitreal injections of bevacizumab 1.25 mg (37 months follow-up). Before treatment, all patients complained of visual loss and metamorphopsia. The OCT showed in the macular area a focal neurosensory detachment with foveal involvement. The FA showed in the macular area multiple focal areas of hyperfluorescence due to pigment epithelium atrophy and in the second and third patient also a hyperfluorescent pinpoint with minimal leakage. After treatment in all eyes, symptoms did not change, BCVA remained stable, and in OCT the foveal neuroretinal detachment was changeless. In FA, no noticeable variation of the hyperfluorescence areas was appreciated. In the second patient, the hyperfluorescent point remained unvaried, and the same occurred in the third patient after the first PDT, while after the second PDT a new leaking dot disappeared. Macular serous retinal detachment was first described in 1998 as an uncommon complication of tilted disc syndrome showing angiographic and OCT features similar to a chronic central serous chorioretinopathy. In contrast to this pathology, in our patients treatment with PDT or intravitreal bevacizumab did not succeed, probably because of a different pathogenesis of macular serous detachment. Further investigations are needed to clarify the proper therapy of this disease. Acquired vitelliform lesions (AVLs) are associated with age-related macular degeneration and other variable macular disorders. AVLs often lead to outer retinal atrophy, sometimes accompanying a macular hole and choroidal neovascularization. The purpose of this study was to report a rare case with bilateral AVLs, in which one eye had accompanied a macular hole and the second eye a serous pigment epithelial detachment (sPED). A 66-year-old woman complained of bilateral metamorphopsia. AVLs were observed in the right eye and a flat sPED in the left eye. The best-corrected visual acuity (BCVA) was 20/17 in both eyes. Fluorescein angiography revealed local leakage in the right eye and pattern dystrophy-like hypofluorescence in both eyes. The sPED progressed with AVLs in the left eye and was treated with a combination therapy of intravitreal aflibercept, a sub-Tenon's injection of triamcinolone acetonide, and photodynamic therapy (IVA/STTA/PDT), which successfully flattened the sPED and sustained good vision for 4 years. The right eye was treated with intravitreal ranibizumab and tissue plasminogen activator, which enhanced absorption of the vitelliform material. However, 14 months later, a macular hole with typical metamorphopsia formed above a subretinal fibrotic scar at the vitelliruptive stage. Although pars plana vitrectomy closed the macular hole, enlargement of the outer retinal atrophy worsened the BCVA to 20/100. We successfully treated one eye with a sPED with AVLs using the combination therapy of IVA/STTA/PDT, while the second eye with a macular hole secondary to AVLs ultimately developed outer retinal atrophy with visual loss. Clinical significance Amsler grid can be used in detecting central visual field defects in following conditions: Age-related macular degeneration: The grid will help detecting the progression of AMD from dry form to wet form. Chance of metamorphopsia is more in wet AMD compared to dry form. Choroidal neovascular membranes: Choroidal neovascular membranes cause scotoma and metamorphopsia. It may be associated with many diseases like macular degeneration, POHS, myopic macular degeneration, trauma etc. Central serous chorioretinopathy: CSCR Causes round or oval scotoma. Macular pucker: Macular pucker also known as an epiretinal membrane cause metamorphopsia and distortions in central field of vision. Cystoid macular edema: Due to macular edema, micropsia may occur. Glaucoma: Amsler grid is useful in detecting central field defects in moderate to severe glaucoma. Macular sparing: Amsler Grid can be used to detect and accurately measure macular sparing. Types Recent advancement in OCT has made it simpler to differentiate a macular hole from other similar types of smaller diameter macular pathologies of old age. From history and physical examination, round central small reddish lesions with dimness of central vision, probable differential diagnosis include: Epiretinal membrane (ERM) with a macular pseudo hole Central foveal dot hemorrhage Lamellar (aborted) macular hole Vitreomacular traction syndrome (VMTS) Foveal drusen Central areolar pigment epitheliopathy Solar retinopathy Small choroidal neovascular membrane involving center Small central serous chorioretinopathy involving center Cystoid macular edema (CME) Other types There are a few other (rare) kinds of macular degeneration with similar symptoms but unrelated in etiology to Wet or Dry age-related macular degeneration. They are all genetic disorders that may occur in childhood or middle age. Vitelliform macular dystrophy Sorsby's fundus dystrophy is an autosomal dominant, retinal disease characterized by sudden acuity loss resulting from untreatable submacular neovascularisation Stargardt's disease (juvenile macular degeneration, STGD) is an autosomal recessive retinal disorder characterized by juvenile-onset macular dystrophy, alterations of the peripheral retina, and subretinal deposition of lipofuscin-like material. Similar symptoms with a very different etiology and different treatment can be caused by epiretinal membrane or macular pucker or any other condition affecting the macula, such as central serous retinopathy. Notable cases Judi Dench Joan Plowright Peter Sallis June Brown S. Robert Morgan See also Treatment and Prognosis Metamorphopsia is a symptom of several common retinal and macular diseases, therefore treating the underlying disorder can improve symptoms. For people who have conditions such as Epiretinal membrane (ERM), Macular Holes and Retinal Detachment, decreased metamorphopsia is associated with an increase in visual acuity. Quantitative evaluation of metamorphopsia is an important step in understanding visual functions of individuals with macular disorders and is an essential tool for physicians in evaluating treatment results. Types Dry (non-exudative, > 80%)—deposition of yellowish extracellular material in and between Bruch's membrane and retinal pigment epithelium (“drusen”) with gradual loss in vision. Wet (exudative, 10–15%)—rapid loss of vision due to bleeding secondary to choroidal neovascularization. Etymology Gk, meta + morphe, form, opsis, sight See also Dysmorphopsia Hallucination References Visual disturbances and blindness Purpose. Myopic macular holes can be difficult to close with surgery and are frequently associated with retinal detachment. We report on a case of a macular hole in a severely myopic eye that underwent spontaneous closure. Methods. An observational case study. Results. A 55-year-old female was referred to Ophthalmology for a central scotoma and metamorphopsia in the right eye. Visual acuity was 1/20 in both eyes. Fundus examination showed loss of the foveal depression, with a small yellow ring in the center of the fovea in the right eye, and a tilted optic disc and peripapillary staphyloma bilaterally. Spectral domain optical coherence tomography (SD-OCT) revealed a fully developed macular hole with a rim of thickened and slightly elevated retina in the right eye. The patient refused surgery. After 4 years of follow-up, her visual acuity improved to 20/40 in the right eye, and SD-OCT revealed spontaneous sealing of the macular hole without bare retinal pigment epithelium. Conclusions. Myopic macular holes represent a challenge regarding their management, and the prognosis is often poor. To characterize retinal and fluorescein angiographic findings of Asian patients with symptoms secondary to age-related macular degeneration (ARMD). We retrospectively reviewed 453 consecutive medical records corresponding to fluorescein angiograms performed between November 1992 and November 1995 to identify Asian patients with symptomatic ARMD. Presenting visual symptoms, best-corrected Snellen visual acuities, and retinal examination findings were determined from the medical records. Fundus photos and fluorescein angiograms were reviewed. There were 26 symptomatic eyes in 19 Asian patients with a median age of 73 years. Presenting visual symptoms included decreased visual acuity (19 eyes), metamorphopsia (5 eyes), or scotoma (2 eyes). Retinal findings included occult choroidal neovascularization (CNV) in 5 (19%) eyes, serous pigment epithelial detachment (PED) in 8 (31%) eyes, PED with CNV in 5 (19%) eyes, drusen in 5 (19%) eyes, retinal pigment epithelial atrophy in 1 (4%) eye, vitreous hemorrhage in 1 (4%) eye, and a disciform scar in 1 (4%) eye. In this cohort of Asian patients with ARMD, the majority of symptomatic eyes had either CNV (46%) or serous PED (31%). To study the clinical features and the pathogenesis of macular hole with and without retina detachment (RD) in high myopic eyes. It was a retrospective series case study. The charts of high myopic patients with macular hole at our hospital from June 2006 to February 2007 were retrospectively reviewed and analyzed. Patients were divided into two groups (the RD group and non-RD group) depending on the presence of RD or not. Their clinical data and optic coherence tomography (OCT) results were further analyzed. SPSS 13.0 was used for the statistic analysis. When comparing the quantitative aspects like age, axial length and refraction, t-test was used. Categorical data, such as sex ratio, occurrence of vitreous traction, posterior staphyloma and retinoschisis were compared by using χ(2) test. Fisher's test was used in comparing eye laterality, incidence of white hole, visual acuity and posterior vitreous detachment (PVD). During this period, there were 43 patients fitting the including criteria. Among them, 36 patents (37 eyes) were in the RD group and 7 patients (7 eyes) in the no-RD group. In the RD group, the average age was 56.1, 24.3% of them (9/37) were male; percentage of left and right eyes was (11/37) and 70.3% (26/37), respectively; average refraction was (-8.9 ± 2.2) D; average axial length was (28.7 ± 2.0) mm. Visual acuity was ≤ 0.05 (72.2%) in 26 eyes and 0.05 - 0.2 (27.8%) in 10 patients. The incidence of complete and non-complete PVD was 89.2% (33/37) and 10.8% (4/37), respectively. White hole presented in 35.1% (3/37) patients. Vitreous traction and retinoschisis presented in 27.0% (10/37) and 35.1% (13/37) patients, respectively. In the non-RD group, the average age was 47.6; 16.7% of them (1/7) were male; left and right eyes were involved in 42.9% (3/7) and 57.1% (4/7), respectively. Average refraction was (-9.0 ± 1.9) D; average axial length was (28.9 ± 1.5) mm. Vision acuity was ≤ 0.05 in 3 patients (42.9%); between 0.05 - 0.2 in 3 eyes (42.9%) and ≥ 0.2 in 1 eye (14.3%). Incidence of complete and non-complete PVD was 85.7% (6/7) and 14.3% (1/7), respectively. White hole was observed in 14.3% (1/7) patients; 42.9% (3/7) patients were accompanied with vitreous traction and 71.4% (5/7) with retinoschisis. B-scan ultrasonography showed posterior staphyloma in all 44 eyes. The results of statistical analysis showed that the gender (χ(2) = 0.008) and eye laterality (χ(2) = 0.449) as well as refraction (t = 0.193), axial length (t = -0.25) and visual acuity (χ(2) = 4.509) of these two groups were similar (P > 0.05). The incidences of vitreous traction (χ(2) = 0.709), white hole (χ(2) = 1.179), PVD (χ(2) = 0.071) and retinoschisis (χ(2) = 3.207) were also similar (P > 0.05). But the age of the non-RD group is significantly younger than the RD group (t = 1.66, P < 0.05). Various pathogenesis may involved in the occurrence of retinal detachment in highly myopic eyes with macular hole. Further study is required to improve our understanding of this entity. The aim of this case report is to describe a case of atypical central serous chorioretinopathy (CSCR) definitively diagnosed after 8 years. A 44-year-old woman presented with reduced visual acuity in her left eye. Her visual acuity was light perception with projection in the right eye and 0.15 in the left. She described a similar decline in vision in her right eye 8 years ago. At that time, she had exudative retinal detachment and was treated with systemic immunosuppressive therapy for a presumed diagnosis of Vogt-Koyanagi-Harada disease. Despite resolution of the exudative retinal detachment, macular scarring developed. Eight years later, she developed inferior exudative retinal detachment in the left eye. A diagnosis of atypical CSCR was made with the help of multimodal imaging and her left eye was successfully treated with eplerenone and half-fluence photodynamic therapy (hf-PDT). In conclusion, early diagnosis and treatment of atypical CSCR may prevent subretinal fibrosis formation and permanent vision loss. Hf-PDT and eplerenone are successful treatment options for atypical CSCR. Self-separation or peeling of an idiopathic epiretinal membrane (ERM) in an eye with partial posterior vitreous detachment (PVD) is a rare event. A 56-year-old woman presented to our clinic with complaints of floaters in her right eye. Best-corrected visual acuity (BCVA) was 9/10 in this eye. Fundus examination and Spectral domain optical coherence tomography (SD-OCT) revealed an idiopathic ERM and Grade 3 PVD in this eye. Four months later, she had complaints of metamorphopsia in her right eye. BCVA was 7/10, while SD-OCT images of the right macula were similar to previous images. One week after the last visit, she presented again due to the sudden disappearance of her metamorphopsia complaints. BCVA had improved to 10/10. Fundus examination demonstrated that the ERM had spontaneously separated from the retinal surface as a flap floating in the vitreous and the foveal contour had returned to normal. The etiologic mechanism may be explained as the contracting forces within an immature ERM being stronger than its adhesion to the retina. The pathogenesis of the macular serous retinal detachment (SRD) associated with congenital optic disc pit remains controversial. The treatment is also discussed. Through this study, which includes the majority of the techniques available, we report our experiment in the treatment of this pathology. This was a retrospective single-centre study of 20 patients who presented with macular SRD associated with optic disc pit between 1983 and 2009. Various treatments were provided. At the beginning of the study, patients were treated only by juxtapapillary laser photocoagulation. After laser failure then as first-line treatment, laser photocoagulation was associated with intravitreal gas (C3F8) injection with postoperative facedown positioning for 2 weeks. During the past few years, all patients have been systematically treated with vitrectomy with or without internal limiting membrane (ILM) peeling, laser, and gas (C2F6) tamponade. This series consisted of 20 patients: nine men and 11 women. The patients' mean age at presentation was 29 years (range, 9-60 years). The mean time between the onset of the decrease in visual acuity (VA) and treatment was 6.1 months. None of these patients had a posterior vitreous detachment at the time of diagnosis. Six patients were treated by laser photocoagulation alone, which was successful only in two cases. Eleven patients (with laser treatment failure in three) were treated by laser and intravitreal gas injection, with a 72% success rate. We performed vitrectomy with posterior hyaloid dissection, laser, and gas tamponade in eight cases (with laser-gas treatment failure in two) with 87% success rate and no recurrence. Five of these patients had ILM peeling during the vitrectomy. The mean follow-up period was 60 months (range, 2 months to 17 years). This study shows that early treatment of macular SRD associated with optic disc pit by vitrectomy, ILM peeling, juxtapapillary photocoagulation, and gas tamponade is followed by good anatomical and functional results. This treatment is superior to the other less invasive procedures. Optical coherence tomography is an important exam for diagnosis and postoperative follow-up of patients. Central serous choroidopathy is a macular disease, usually with a self-limited and benign course, and predominantly affects male patients between 20 and 45 years old. A 68 year-old female patient complained of decreased visual acuity of her right eye of approximately 3 weeks of onset. Best corrected visual acuity in her right eye was 20/100. Fundus examination revealed a macular serous detachment involving its centre, as well as the presence of multiple calcified drusen. Fluorescein angiography showed late parafoveal leakage in a \"smokestack\" pattern in the right macular area. Optical coherence tomography showed a dome-shape macular detachment, also in the right eye. The patient was observed every 2 weeks and spontaneous resolution of the macular detachment was seen a month later. Based on these clinical features, a diagnosis was made of central serous choroidopathy of atypical presentation. Atypical presentation cases of serous central choroidopathy might be seen occasionally. Hence, it is an important differential diagnosis of age related macular degeneration in patients older than 60 years. To evaluate a macular buckle for exudative choroidal neovascularization secondary to age-related macular degeneration (ARMD). Forty-two eyes with choroidal neovascular membranes (CNVM) secondary to ARMD underwent surgical placement of a macular buckle. A Gore-Tex strip (2.0-2.5 mm wide) was button-holed through a 5 mm diameter silicone sponge (9 mm long) and placed behind the macula underneath the CNVM by the same surgeon (Dr Peyman) in all cases. Follow-up ranged from 7-76 months (mean, 20.9 months). Of 12 eyes with classic subfoveal CNVM: 4 (33%) gained 2 or more lines of Snellen visual acuity; 3 (25%) gained 1 line, remained the same, or lost 1 line; and 5 (42%) lost 2 or more lines (range + 6 to - 6 lines). Of 22 eyes with ill-defined subfoveal CNVM: 12 (54%) gained 1 line, remained the same, or lost 1 line; and 10 (46%) lost 2 or more lines (range + 1 to - 8 lines). Eight eyes with ill-defined juxtafoveal CNVM had the following visual acuity outcomes: 5 eyes (62%) maintained the same level of Snellen visual acuity (gained 1, 0, or lost 1 line); and 3 (38%) got worse (lost 2 or more lines of Snellen visual acuity, range + 1 to - 6 lines). Ten eyes (24%) bled subretinally during the follow-up period (average 11.5 months, range 14 days to 27 months), all outside the area of indentation of the macular buckle. The macular buckle treatment for exudative subretinal choroidal neovascular membranes in ARMD stabilized visual decline and displaced significant subfoveal hemorrhage. To report the pathogenic factors that account for cystoid macular edema and cystoid macular degeneration in chronic central serous chorioretinopathy (CSC). The clinical course and multimodal imaging findings, including fundus color photography, fundus autofluorescence, spectral-domain optical coherence tomography, and fluorescein angiography, of one eye with cystoid macular edema due to chronic CSC was documented. A 44-year old woman with a history of chronic CSC presented with progressive visual decline in the right eye. Best-corrected visual acuity was 20/40. Funduscopic examination revealed diffuse retinal pigment epithelial changes and macular edema. Fluorescein angiography demonstrated perifoveal microaneurysms and leakage in a petaloid configuration. Spectral-domain optical coherence tomography demonstrated cysts at the level of the inner nuclear layer, an epiretinal membrane, vitreomacular traction, and an attenuated retinal pigment epithelial band. Central subfield thickness was 486 μm. Three intravitreal injections of aflibercept were administered over 16 weeks following which there was resolution of leakage, release of vitreomacular traction, and resolution of microaneurysms. Central subfield thickness reduced to 379 μm, but persistent intraretinal cysts were observed. There was subjective improvement in visual symptoms, but Snellen acuity remained at 20/40. Intraretinal cystic changes in chronic CSC may be the result of multifactorial pathogenic factors and may represent the coexistence of cystoid macular edema and cystoid macular degeneration. Anti-vascular endothelial growth factor may play an important role in the treatment of cystoid macular edema caused by CSC. A patient developed choroidal neovascularization (CNV) in one eye during treatment for bilateral recurrent central serous chorioretinopathy (CSC) and was intravitreously injected with bevacizumab; she developed multiple evanescent white dots and serous retinal detachment(SRD). A 50-year-old women had a history of CSC OD at the age of 29 years. On initial examination, CSC OD was noted, and multiple detachments of the retinal pigment epithelium OU were observed. While the CSC in the right eye was successfully treated by laser photocoagulation, it spread to both eyes following this episode. Examination of the right eye by optical coherence tomography (OCT) following the recurrence of the CSC showed slight elevation of the retinal pigment epithelial layer in the central fovea, but this finding disappeared with the resolution of the CSC. However, as the CSC combined with CNV (Gass type 2) recurred within 1 year, the patient was intravitreously injected with bevacizumab. On the day following the injection, SRD OD occurred, and on the 7th day following the injection many white lesions varying in size appeared in the deep layer of the retina, but they healed 3 weeks later, leaving only the CNV. The CNV was cured later by additional photodynamic therapy. Since the lesions of the fundus observed immediately after the bevacizumab administration resolved spontaneously without sequelae, they were retrospectively diagnosed as a white dot syndrome-like disease. The white dot syndrome-like disease is suggested as a rare complication of bevacizumab. Central serous chorioretinopathy (CSCR) is an idiopathic condition characterized by serous detachment of the neurosensory retina in the macular region. It is relatively uncommon in Africans and though pregnancy is a known risk factor, there are no previous reports of CSCR in pregnant African women. We report the case of a 35-year-old pregnant woman who presented to our clinic at gestational age of 29 weeks with a 4 months history of blurring of vision in her left eye. Examination revealed visual acuity of 6/4 on the right eye and 6/9 on the left eye. She had normal anterior segments bilaterally and a normal posterior segment on the right. However, she had left macular edema with exudates. There was no significant refractive error. Her blood pressure was normal. Investigations including electrolytes and urea, urinalysis, and blood sugar profile were all normal. She was managed conservatively, and symptoms resolved 2 weeks prior to delivery. This is a case report of CSCR in a pregnant Nigerian woman with spontaneous resolution of symptoms prior to delivery. Pregnant women should be educated about the possibility of visual problems accompanying pregnancy. \nHere is the question:\nOPHTHALMOLOGY: 75-year-old woman diagnosed 3 years ago with soft drusen in the fundus. She reports presenting, since 2 weeks ago, metamorphopsia and significant visual loss in her right eye that prevents her from reading. Indicate the most probable diagnosis:\nHere are the potential choices:\n1. Macular epiretinal membrane.\n2. Macular hole.\n3. Thrombosis of the central retinal vein.\n4. Senile macular degeneration.\n5. Central serous chorioretinopathy.\nThe correct answer is: 2. Macular hole." }, { "question": "You are a helpful medical expert, and your task is to answer a multi-choice medical question using the relevant documents. Please choose the answer from the provided options. Your responses will be used for research purposes only, so please have a definite answer.\nHere are the relevant documents:\n Classic Cushing's syndrome is a rare disease with an estimated incidence of 1 case per 100,000 persons. With routine use of imaging techniques such as ultrasound and CT, adrenal masses are being detected with increased frequency. A substantial percentage of these incidentalomas are hormonally active, with 5% to 20% of the tumors producing glucocorticoids. Autonomous glucocorticoid production without specific signs and symptoms of Cushing's syndrome is termed subclinical Cushing's syndrome. With an estimated prevalence of 79 cases per 100,000 persons, subclinical Cushing's syndrome is much more common than classic Cushing's syndrome. Depending on the amounts of glucocorticoids secreted by the tumor, the clinical spectrum ranges from slightly attenuated diurnal cortisol rhythm to complete atrophy of the contralateral adrenal gland with lasting adrenal insufficiency after unilateral adrenalectomy. Patients with subclinical Cushing's syndrome lack the classical stigmata of hypercortisolism but have a high prevalence of obesity, hypertension, and type 2 diabetes. All patients with incidentally detected adrenal masses scheduled for surgery must undergo testing for subclinical Cushing's syndrome to avoid postoperative adrenal crisis. The best screening test to uncover autonomous cortisol secretion is the short dexamethasone suppression test. Because the adrenal origin of a pathologic cortisol secretion is anticipated, the author prefers a higher dexamethasone dose (3 mg instead of 1 mg) to reduce false-positive results. A suppressed serum cortisol level of less than 3 micrograms/dL (80 nmol/L) after dexamethasone excludes significant cortisol secretion by the tumor. A serum cortisol level greater than 3 micrograms/dL requires further investigation, including confirmation by high-dose dexamethasone (8 mg) suppression testing, a CRH test, and analysis of diurnal rhythm. Determination of urinary free cortisol is less useful because increased values are a late finding usually associated with emerging clinical signs of Cushing's syndrome. Patients with suppressed plasma ACTH in response to CRH generally have adrenal insufficiency after surgery and require adequate perioperative and postoperative substitution therapy. Whether patients with subclinical Cushing's syndrome should undergo adrenalectomy is a matter of debate. The author performs surgery in young patients (< 50 years), in patients with suppressed plasma ACTH, and in patients with a recent history of weight gain, substantial obesity, arterial hypertension, diabetes mellitus, and osteopenia. In completely asymptomatic patients with normal plasma ACTH concentrations and in patients older than 75 years, the author recommends a nonsurgical approach. A large prospective randomized study is necessary to evaluate the benefits of surgery versus conservative treatment in patients with subclinical Cushing's syndrome. Cushing's syndrome (CS) is a rare disease characterized by a collection of signs and symptoms, also common in the general population without elevated cortisol secretion. During the last years more patients with CS are identified earlier and with milder disease. Many of these patients are diagnosed during screening efforts performed for certain or isolated complaints like weight gain, diabetes mellitus (DM), hypertension, osteoporosis, elevated white blood cell counts and more. In this review article the most popular screening test performed in the studies cited was the 1-mg dexamethasone suppression test. Cushing is not frequent enough to support the use of routine screening in patients with morbid obesity and type 2 DM. Also only 1% of hypertensive patients have secondary hypertension due to CS. However, screening should be considered in young patients with resistant DM and/or hypertension. Among patients with osteoporosis and vertebral fractures up to 5% were diagnosed with subclinical hypercortisolism; most of these had adrenal adenoma. Screening for CS is important in subjects with adrenal incidentaloma, and many studies show a high prevalence (~10%) of Cushing or subclinical CS in these patients. A 64 year old lady, with a background history of type 2 diabetes mellitus and hypertension, presented with general deterioration of general health, poor glycemic control, difficulty in controlling blood pressure and difficulty in walking. She had past medical history of adenocarcinoma of the oesophagus, treated with surgery and subsequent chemotherapy. General examination revealed high blood glucose and blood pressure and a Cushingoid facies. Overnight dexamethasone suppression test and urinary free cortisol levels confirmed Cushing's syndrome and High dose dexamethasone suppression test showed partial suppression. CT scan of the abdomen showed bilateral hyperplasia of the adrenals with nodularity on the left side, raising the possibility of an adrenal adenoma. ACTH levels were elevated thereby ruling out autonomously functioning adrenal nodule, however increasing the possibility of ectopic ACTH secretion due to the previous medical history. MRI of the pituitary confirmed the presence of an adenoma, thereby pointing to the diagnosis of pituitary dependant Cushing's disease. The patient could not undergo further invasive investigation or surgery due to septicaemia. Medical management of Cushing's syndrome was resorted to in the interim with Ketoconazole, showing excellent response. This case depicts the need for a high index of suspicion for the diagnosis, the importance of organizing specific investigations in the appropriate order to arrive at a diagnosis and an effective management plan. Endogenous Cushing's syndrome is a rare endocrine disorder that is fatal if left untreated. It can be distinguished into adrenocorticotropic hormone (ACTH)-dependent (central and ectopic Cushing's syndrome) and ACTH-independent subtypes (unilateral or bilateral adrenal adenomas). The clinical presentation of patients includes typical stigmata of cortisol excess with physical symptoms of catabolic metabolism (myopathy, striae, parchment skin, osteoporosis) and components of metabolic syndrome (diabetes mellitus, obesity, arterial hypertension, hypercholesterolemia). Biochemical diagnosis is performed in three steps: 1. Confirmation of the diagnosis by 1‑mg dexamethasone suppression test, 24‑h urine free cortisol, and measurement of late-night salivary cortisol. 2. Differentiation of ACTH-dependent Cushing's syndrome from ACTH-independent adrenal Cushing's syndrome by measurement of plasma ACTH. 3. Further subtyping by corticotropin-releasing hormone (CRH) test, inferior petrosal sinus sampling, and imaging modalities. Therapeutic decisions are made on an interdisciplinary basis. First-line therapy for all subtypes is surgery when possible; additional options for all forms include drug therapy and bilateral adrenalectomy. Despite adequate treatment, Cushing's syndrome is associated with increased long-term morbidity and mortality. Interdisciplinary and multimodal therapy management is necessary in the long term to positively influence mortality and reduced quality of life. We report a 49-year-old woman who had minimal features of Cushing syndrome and an incidentally discovered adrenal adenoma. She was subsequently diagnosed with pituitary-dependent Cushing syndrome. Laboratory and imaging studies including serum cortisol, plasma adrenocorticotrophic hormone (ACTH), high dose dexamethasone test, corticotropin-releasing hormone test, computed tomography (CT) scan, and magnetic resonance imaging were performed. A 49-year-old woman was admitted for urosepsis. An abdominal CT scan performed during the urosepsis workup showed a 2.7-cm right adrenal adenoma. She denied any abdominal striae or other symptoms. Physical examination showed normal vital signs, minimal facial fullness without central obesity, and striae. Laboratory results were as follows: 24-hour-urine cortisol 294 μg (reference 4.0-50.0), midnight serum cortisol 23.0 μg/dL (reference < 7.5), and plasma ACTH level 39 pg/mL (reference 5-27). A corticotropin-releasing hormone stimulation test showed >20% rise in serum cortisol and >35% rise in ACTH levels. A pituitary magnetic resonance image showed a 5 mm pituitary lesion. The patient underwent transsphenoidal pituitary surgery, which confirmed an ACTH-secreting lesion. Postoperatively, she required hydrocortisone replacement for the next 10 months. A follow-up adrenal CT performed 6 months later showed a decrease in the size of the adrenal adenoma (1.8 cm). This case highlights the importance of recognizing the coexistence of ACTH-dependent Cushing disease with an adrenal adenoma and partial ACTH dependency of the adrenal adenoma. Endogenous Cushing's syndrome is a very rare entity, with an incidence of 2-4 cases per million inhabitants per year. Cases caused by ectopic ACTH secretion are under-diagnosed. Cushing's disease is the most frequent cause of endogenous Cushing's syndrome, which is 5 or 6 times more frequent than adrenal Cushing's syndrome, with an incidence of between 1.2 and 2.4 cases per million inhabitants per year. Cushing's disease is 3-8 times higher in women than in men. The frequency of adrenal tumors is 3 times higher in women, while that of Cushing's syndrome due to adrenal tumors is 3-5 times higher. Age at diagnosis of Cushing's syndrome varies according to the etiology. Most cases of Cushing's disease are due to a pituitary adenoma, although the tumor may not be visible on the available imaging techniques. ACTH-independent Cushing's syndrome is found in 20% of cases and is most frequently due to adenomas (10%) or adrenal carcinomas (8). Bilateral micronodular hyperplasia and macronodular hyperplasia are infrequent entities, representing less than 10% of all cases of ACTH-independent Cushing's syndrome. Both familial and sporadic forms exist: the familial form, or Carney complex, and ACTH-independent bilateral macronodular hyperplasia, in which the size of the adrenal glands is considerably enlarged. The signs and symptoms of Cushing's syndrome are a direct result of long-term exposure to excessive glucocorticoids. Most signs and symptoms are highly prevalent in the general population (hypertension, central obesity, diabetes mellitus or carbohydrate intolerance, osteoporosis, and characteristic phenotypical alterations). The diagnosis of Cushing's syndrome is one of the most perplexing and controversial problems in endocrinology. However, significant advances in the diagnosis procedures have been made in the past decade. The diagnostic studies involved in the evaluation of patients with suspected Cushing's syndrome fall into two categories: confirming the presence of true hypercortisolism and establishing the precise aetiology. Diagnosis of Cushing's syndrome: ambulatory screening relies on the overnight 1 mg dexamethasone test. Negative tests are confirmed by measuring cortisol in two 24-hour urine samples. If cortisol excretion is slightly above normal, a 48-hour low-dose dexamethasone suppression test or an intravenous infusion dexamethasone suppression test are required. Diagnosis of the aetiology of Cushing's syndrome: the first step is to establish if the hypercortisolism is ACTH-dependent or not. This step is solved by measuring plasma ACTH and cortisol in the late afternoon. Computed tomography scanning of the adrenal glands is required in ACTH-independent Cushing's syndrome. A unilateral tumour will be demonstrated in most of cases. If bilateral lesions are found, dynamic testing using cortisol releasing factor and/or metyrapone must be performed to confirm the ACTH-independency of the syndrome. In ACTH-dependent Cushing's syndrome, the major difficulty is to distinguish between a pituitary source and an ectopic source of ACTH secretion. Magnetic resonance imaging of the pituitary with gadolinium enhancement must be preferred to computed tomography scanning but its sensitivity is not better than 70-80% and false positives can occur. When no macroscopic pituitary lesion can be detected, bilateral inferior petrosal sinus sampling coupled to CRH injection for ACTH measurement will indicate the source of ACTH secretion. If this test indicates the patient has Cushing's disease, pituitary trans-sphenoidal surgery can be performed. If the test indicates the patient has ectopic ACTH-secretion, a cervico-thoraco-abdominal scanning is necessary to identify the tumour. In the case of occult tumour the hypercortisolism must be controlled by pharmacological agents and the imaging investigations must be repeated at appropriate intervals. Cushing's syndrome is a rare but frequently considered disease. Its diagnosis can lead to some difficulties, including confirming the effective endogenous hypercortisolism and determining its cause. The severity of this disease, the diversity of its complications and the multiple therapeutic options make its management challenging. The aim of this review is to present the most recent data about management of Cushing's syndrome, especially diagnostic approaches and therapeutic options. Our references were obtained by screening MEDLINE database from 1996 to 2006. We also included some anterior reviews and consensus statements. We retained the following points: midnight salivary cortisol is a useful tool in the diagnosis of Cushing's syndrome; the desmopressin test can help to distinguish between Cushing's syndrome and \"pseudoCushing's\" due to alcohol consumption or psychiatric disorders; cavernous sinus and inferior petrosal sinus sampling is indicated in the evaluation of ACTH-dependent Cushing's syndromes when pituitary imaging is normal or equivocal or when dynamic tests are contradictory; multislice computed-tomography of the chest and the abdomen and somatostatin analogue scintigraphy, eventually combined, are the best imaging procedures in occult ectopic ACTH syndromes; patients with Cushing's disease should be referred to a neurosurgeon experienced in corticotroph adenomas surgery; metabolic consequences of Cushing's syndrome, such as cardiovascular risk factors and osteoporosis need an aggressive treatment. The incidence of Cushing's syndrome is only 1/100000 per year. However, hypercortisolism is diagnosed by systematic evaluation in 2 to 5% of patients with poorly controlled type 2 diabetes and adrenal incidentalomas. Endocrinological management of the disease improves metabolic disorders in these patients. If these results are confirmed, screening for Cushing's syndrome should be systematically performed in these populations. When Cushing's syndrome is suspected, either a dexamethasone suppression test (administration of dexamethasone and frequent determination of cortisol and ACTH level), or a 24-hour urinary measurement for cortisol offers equal detection rates. Dexamethasone is a glucocorticoid and simulates the effects of cortisol, including negative feedback on the pituitary gland. When dexamethasone is administered and a blood sample is tested, cortisol levels >50 nmol/l (1.81 μg/dl) would be indicative of Cushing's syndrome because an ectopic source of cortisol or ACTH (such as adrenal adenoma) exists which is not inhibited by the dexamethasone. A novel approach, recently cleared by the US FDA, is sampling cortisol in saliva over 24 hours, which may be equally sensitive, as late-night levels of salivary cortisol are high in cushingoid patients. Other pituitary hormone levels may need to be ascertained. Performing a physical examination to determine any visual field defect may be necessary if a ACTH blood test Once Cushing's syndrome has been diagnosed, the first step towards finding the cause is measuring plasma adrenocorticotropic hormone (ACTH) concentration. A concentration consistently below 1.1 pmol/L is classified as corticotropin-independent and does not lead to a diagnosis of Cushing's disease. In such cases, the next step is adrenal imaging with CT. If plasma corticotropin concentrations are consistently above 3.3 pmol/L, then corticotropin-dependent Cushing's syndrome is most likely. Any intermediate values need to be cautiously interpreted and a corticotropin-releasing hormone (CRH) test is advised in order to confirm corticotropin dependency. If corticotropin-dependent Cushing's syndrome is determined then the next step is to distinguish between Cushing's disease and ectopic corticotropin syndrome. This is done via a combination of techniques including CRH, high-dose DST, BIPSS, and pituitary MRI. A 49-year-old man was referred to our hospital for the treatment of gallstones in 1993. Bilateral adrenal nodular masses were detected incidentally by abdominal computed tomography. He had no clinical signs of Cushing's syndrome such as central obesity, striae of skin and diabetes mellitus. We performed cholecystectomy and partial adrenalectomy of right adrenal gland as a biopsy, and diagnosed him as preclinical Cushing's syndrome due to adrenocorticotropin-independent bilateral adrenal macronodular hyperplasia (AIMAH) based on endocrinological and histological examinations. We followed him up for 7 years. During the observation period, the sizes of both adrenal glands increased gradually, and finally serum cortisol level increased beyond normal range, and he showed a Cushingoid appearance such as moon face and central obesity. His skin became atrophic and very fragile, and the bone mineral density of his lumbar spine was extremely low. Serum cortisol level was elevated, and plasma ACTH level was always suppressed. Urinary excretion of 17-hydroxycorticosteroid and free cortisol were increased. Diurnal rhythm of cortisol and ACTH was completely lost and high dose (8 mg/day) dexamethasone did not suppress urinary 17-hydroxycorticosteroid excretion. He became clinically overt Cushing's syndrome. We recommended total adrenalectomy, but he refused it. It is important to know the natural history of preclinical Cushing's syndrome due to AIMAH when choosing an adequate treatment. Corticosteroid overproduction Cushing's syndrome Cushing's syndrome is the manifestation of glucocorticoid excess. It can be the result of a prolonged treatment with glucocorticoids or be caused by an underlying disease which produces alterations in the HPA axis or the production of cortisol. Causes can be further classified into ACTH-dependent or ACTH-independent. The most common cause of endogenous Cushing's syndrome is a pituitary adenoma which causes an excessive production of ACTH. The disease produces a wide variety of signs and symptoms which include obesity, diabetes, increased blood pressure, excessive body hair (hirsutism), osteoporosis, depression, and most distinctively, stretch marks in the skin, caused by its progressive thinning. To review the challenges encountered in the diagnostic work-up and management of patients with subclinical Cushing's syndrome (SCS) and bilateral adrenal masses to aid in the case description of a patient with SCS and adrenocorticotropic hormone (ACTH)-independent macronodular adrenal hyperplasia (AIMAH). We describe our experience managing a patient with AIMAH and SCS. This case report is followed by an extensive review of the literature regarding differential diagnoses, work-up including adrenal venous sampling (AVS), and treatment of SCS with bilateral adrenal masses. A 51-year-old female who was diagnosed with recent onset hypertension and diabetes mellitus type 2 was evaluated for adrenal masses discovered incidentally on computed tomography (CT). She did not have any Cushingoid features. Magnetic resonance imaging (MRI) of abdomen was performed for further evaluation. Hormonal evaluation came back consistent with SCS. The AVS results were consistent with bilateral autonomous cortisol hypersecretion without lateralization. Collectively, the findings favored the diagnosis of bilateral AIMAH. A left adrenalectomy was performed, and the patient's clinical response was favorable with improvement in blood pressure (BP) accompanied by significant weight loss. Follow-up hormonal testing for autonomous cortisol hypersecretion was within the target range. AIMAH is a rare cause of SCS. AVS is a useful diagnostic tool that helps localize the source of autonomous cortisol hypersecretion in ACTH-independent SCS with bilateral adrenal masses, especially if radiological features are inconclusive. Patients undergoing unilateral adrenalectomy should be followed for monitoring of clinical response, as well as progression of AIMAH in the contralateral adrenal gland. Associated diseases Cushing's Disease Corticotropic cells can have detrimental effects on the body if they express too much or too little ACTH. One such example is Cushing's disease, which can result from overproduction of ACTH in the corticotropes due to pituitary tumors known as corticotroph adenomas; this is the cause for roughly two-thirds of those diagnosed with Cushing's disease. It is also possible that this disease can result from production of ACTH in a non-pituitary tumor, known as ectopic production, or the adrenal glands can overproduce cortisol due to an adrenal tumor. This overproduction of ACTH causes an increase in cortisol levels due to increased glucocorticoid synthesis in the adrenal cortex resulting in several associated symptoms. Symptoms of Cushing's disease include: Fatty deposits in the neck or back Stretch marks (striae) Fatigue Osteoporosis Weakened immune system Hypertension When any of these tests is positive, CT scanning of the adrenal gland and MRI of the pituitary gland are performed to detect the presence of any adrenal or pituitary adenomas or incidentalomas (the incidental discovery of harmless lesions). Scintigraphy of the adrenal gland with iodocholesterol scan is occasionally necessary. Occasionally, determining the ACTH levels in various veins in the body by venous catheterization, working towards the pituitary (petrosal sinus sampling) is necessary. In many cases, the tumors causing Cushing's disease are less than 2 mm in size and difficult to detect using MRI or CT imaging. In one study of 261 patients with confirmed pituitary Cushing's disease, only 48% of pituitary lesions were identified using MRI prior to surgery. Plasma CRH levels are inadequate at diagnosis (with the possible exception of tumors secreting CRH) because of peripheral dilution and binding to CRHBP. Cushing's disease, i.e., pituitary ACTH-secreting adenoma causing excess glucocorticoid secretion, is a rare disease with significant mortality and morbidity. Timely diagnosis and appropriate treatment can alter the course of the disease and are therefore mandatory. First step of the diagnostic work-up is the endogenous glucocorticoid excess by measurement of urinary free cortisol, cortisol circadian rhythmicity or suppression by low doses of dexamethasone. In patients with equivocal results, second line tests, such as the dexamethasone-suppressed CRH test and desmopressin stimulation, usually enable the diagnosis to be confirmed. Measurement of plasma ACTH then allows the distinction between ACTH-dependent (e.g., pituitary or extrapituitary neuroendocrine tumors) and ACTH-independent causes (e.g., adrenal tumors). The last step in the diagnostic algorithm is often the most fraught with problems as the distinction between Cushing's disease and ectopic ACTH secretion relies on judicious interpretation of several diagnostic procedures. Positive responses to stimulation with CRH and inhibition by high doses of dexamethasone, if concurrent, enable a pituitary origin to be established whereas conflicting results call for inferior petrosal sinus sampling, the latter to be performed in experienced centres only. Visualisation of the tumor at pituitary imaging is helpful but not required for the diagnosis, as microadenomas often remain undectected by MRI and/or CT scan and, on the other hand, visualisation of a non-secreting incidentaloma may be misleading. Surgical removal of the pituitary tumor is the optimal treatment choice and should be attempted in every patient. Surgical failures as well as relapses can be treated by radiotherapy, medical therapy or, if necessary, bilateral adrenalectomy. Finally, patients cured of Cushing's disease require long-term monitoring given the risk of relapse and clinical burden of associated ailments. Cushing's syndrome, including its mild form/state of adrenal-dependent subset (subclinical Cushing's syndrome; subCS), is known to enhance glucose intolerance, hypertension and obesity. Recently, subclinical Cushing's disease (subCD) has been identified, but its prevalence and the extent of consequent metabolic derangement are unclear. We screened 90 type 2 diabetic patients hospitalized in our department for subCD, according to the diagnostic guideline proposed by the working group of Japanese Ministry of Health, Welfare and Labor in 2006. Plasma ACTH and cortisol levels in the morning and at midnight were determined, and overnight 0.5 mg dexamethasone suppression test (DST) was performed. Those who showed poor cortisol suppression in DST underwent the desmopressin (DDAVP) test. Fifty-seven patients (63.3%) demonstrated abnormally high midnight cortisol levels (>or=2.5 microg/dL), while only nine of them failed to suppress plasma cortisol levels to <3 microg/dL after DST. Although none of the eight patients who underwent the DDAVP test demonstrated the anticipated paradoxical rise in plasma ACTH, these eight patients (8.9%) endocrinologically met the screening criteria of subCD. Since a considerable percentage of pituitary adenomas causing overt Cushing's disease are not identifiable in magnetic resonance imaging, many of those causing subCD may also be unidentifiable. Further follow-up studies including confirmatory testing and pituitary imaging are necessary. The most accurate test used to differentiate a pituitary adenoma from ectopic or adrenal Cushing syndrome is inferior petrosal sinus sampling. [12] [18] This invasive method measures the difference in ACTH level found in the inferior petrosal sinus (where the pituitary gland drains) compared to the periphery. [12] [18] A basal central to the peripheral ratio of over 3:1 when CRH is administered confirms the diagnosis of Cushing disease. [18] This test is considered the gold standard in diagnosing Cushing disease because it has a sensitivity and specificity of nearly 94%. Still, it is rarely used in clinical practice due to its high cost, invasiveness, rare but serious complications, and the required special expertise to perform. is diagnostic 1) Overnight DST2) 24-hour urinary free cortisol3) 11:00 pm salivary cortisol1) Plasma ACTH2) High-dose DST and urinary cortisolConfirm the diagnosisACTH gradient?Determine source of hypercortisolismDecreased ACTHLack of suppressionCT scan adrenalsIncreased ACTHPositive Increased ACTHLack of suppressionEquivocalresultsFurther testingBilateral petrosalvein samplingAdrenalsourcePituitarysourceEctopic ACTHsourceSTEPS IN DIAGNOSISDIAGNOSTIC STUDIESYesNoFigure 38-44. Diagnosis of Cushing’s syndrome. ACTH = adrenocorticotropic hormone; CT = computed tomography; DST = dexamethasone suppression test.Brunicardi_Ch38_p1625-p1704.indd 169001/03/19 11:22 AM 1691THYROID, PARATHYROID, AND ADRENALCHAPTER 38of a pituitary tumor. In patients suspected of having ectopic ACTH production, CT or MRI scans of the chest and anterior mediastinum are performed first, followed by imaging of the neck, abdomen, and pelvis if the initial studies are negative.Treatment Laparoscopic Serum ACTH levels can differentiate ACTH-dependent Cushing syndrome (elevated ACTH or inappropriately normal ACTH) from ACTH-independent (low ACTH level) Cushing syndrome. In patients with ACTH-dependent Cushing syndrome, the high-dose dexamethasone suppression test done by giving 8 mg dexamethasone by mouth at 2300 h and checking cortisol the next day at 0800 h, can differentiate pituitary ACTH from an ectopic ACTH source. A high-dose dexamethasone suppression test will decrease cortisol level by 50 % if the ACTH source is a pituitary adenoma, but not if ACTH is secreted by an ectopic tumor (e.g., oat cell lung carcinoma). Pituitary MRI, unenhanced CT scan of the adrenals, and chest X-ray and CT are also useful to localize the pathology. Cushing's syndrome is caused by either excessive cortisol-like medication, such as prednisone, or a tumor that either produces or results in the production of excessive cortisol by the adrenal glands. Cases due to a pituitary adenoma are known as Cushing's disease, which is the second most common cause of Cushing's syndrome after medication. A number of other tumors, often referred to as ectopic due to their placement outside the pituitary, may also cause Cushing's. Some of these are associated with inherited disorders such as multiple endocrine neoplasia type 1 and Carney complex. Diagnosis requires a number of steps. The first step is to check the medications a person takes. The second step is to measure levels of cortisol in the urine, saliva or in the blood after taking dexamethasone. If this test is abnormal, the cortisol may be measured late at night. If the cortisol remains high, a blood test for ACTH may be done.\nHere is the question:\nENDOCRINOLOGY: A 56-year-old female patient consulted for dorso-lumbar spine pain and progressive difficulty in performing usual tasks. In the last 5 years she gained weight, she has ecchymosis easily and arterial hypertension was detected. Physical examination: Obesity of central predominance, rounded facies, increased supraclavicular fat, decreased proximal muscle strength and some reddish striae in the abdomen. She has a blood glucose of 136 mg/dL and the radiological study showed osteoporosis and vertebral crushing. What do you think is the most coherent interpretation and attitude?\nHere are the potential choices:\n1. Postmenopausal osteoporosis, type 2 diabetes mellitus and essential hypertension, with decreased strength due to diabetic polyneuropathy.\n2. It is necessary to rule out Cushing's disease by dexamethasone suppression test and perform a cranial CT scan.\n3. Suggest Cushing's. Determine urinary free cortisol and basal ACTH, which serves to orient its etiology and select the most appropriate imaging technique.\n4. It looks like Cushing's. If basal ACTH is high, it may be due to corticosteroid use or an adrenal tumor, and an MRI should be performed.\n5. Probably has Cushing's. If the basal ACTH is low, he probably has a pituitary micro-adenoma, and a cranial CT scan should be performed.\nThe correct answer is: ", "gold_answer": "3 Suggest Cushing's. Determine urinary free cortisol and basal ACTH, which serves to orient its etiology and select the most appropriate imaging technique.", "prediction": "You are a helpful medical expert, and your task is to answer a multi-choice medical question using the relevant documents. Please choose the answer from the provided options. Your responses will be used for research purposes only, so please have a definite answer.\nHere are the relevant documents:\n Classic Cushing's syndrome is a rare disease with an estimated incidence of 1 case per 100,000 persons. With routine use of imaging techniques such as ultrasound and CT, adrenal masses are being detected with increased frequency. A substantial percentage of these incidentalomas are hormonally active, with 5% to 20% of the tumors producing glucocorticoids. Autonomous glucocorticoid production without specific signs and symptoms of Cushing's syndrome is termed subclinical Cushing's syndrome. With an estimated prevalence of 79 cases per 100,000 persons, subclinical Cushing's syndrome is much more common than classic Cushing's syndrome. Depending on the amounts of glucocorticoids secreted by the tumor, the clinical spectrum ranges from slightly attenuated diurnal cortisol rhythm to complete atrophy of the contralateral adrenal gland with lasting adrenal insufficiency after unilateral adrenalectomy. Patients with subclinical Cushing's syndrome lack the classical stigmata of hypercortisolism but have a high prevalence of obesity, hypertension, and type 2 diabetes. All patients with incidentally detected adrenal masses scheduled for surgery must undergo testing for subclinical Cushing's syndrome to avoid postoperative adrenal crisis. The best screening test to uncover autonomous cortisol secretion is the short dexamethasone suppression test. Because the adrenal origin of a pathologic cortisol secretion is anticipated, the author prefers a higher dexamethasone dose (3 mg instead of 1 mg) to reduce false-positive results. A suppressed serum cortisol level of less than 3 micrograms/dL (80 nmol/L) after dexamethasone excludes significant cortisol secretion by the tumor. A serum cortisol level greater than 3 micrograms/dL requires further investigation, including confirmation by high-dose dexamethasone (8 mg) suppression testing, a CRH test, and analysis of diurnal rhythm. Determination of urinary free cortisol is less useful because increased values are a late finding usually associated with emerging clinical signs of Cushing's syndrome. Patients with suppressed plasma ACTH in response to CRH generally have adrenal insufficiency after surgery and require adequate perioperative and postoperative substitution therapy. Whether patients with subclinical Cushing's syndrome should undergo adrenalectomy is a matter of debate. The author performs surgery in young patients (< 50 years), in patients with suppressed plasma ACTH, and in patients with a recent history of weight gain, substantial obesity, arterial hypertension, diabetes mellitus, and osteopenia. In completely asymptomatic patients with normal plasma ACTH concentrations and in patients older than 75 years, the author recommends a nonsurgical approach. A large prospective randomized study is necessary to evaluate the benefits of surgery versus conservative treatment in patients with subclinical Cushing's syndrome. Cushing's syndrome (CS) is a rare disease characterized by a collection of signs and symptoms, also common in the general population without elevated cortisol secretion. During the last years more patients with CS are identified earlier and with milder disease. Many of these patients are diagnosed during screening efforts performed for certain or isolated complaints like weight gain, diabetes mellitus (DM), hypertension, osteoporosis, elevated white blood cell counts and more. In this review article the most popular screening test performed in the studies cited was the 1-mg dexamethasone suppression test. Cushing is not frequent enough to support the use of routine screening in patients with morbid obesity and type 2 DM. Also only 1% of hypertensive patients have secondary hypertension due to CS. However, screening should be considered in young patients with resistant DM and/or hypertension. Among patients with osteoporosis and vertebral fractures up to 5% were diagnosed with subclinical hypercortisolism; most of these had adrenal adenoma. Screening for CS is important in subjects with adrenal incidentaloma, and many studies show a high prevalence (~10%) of Cushing or subclinical CS in these patients. A 64 year old lady, with a background history of type 2 diabetes mellitus and hypertension, presented with general deterioration of general health, poor glycemic control, difficulty in controlling blood pressure and difficulty in walking. She had past medical history of adenocarcinoma of the oesophagus, treated with surgery and subsequent chemotherapy. General examination revealed high blood glucose and blood pressure and a Cushingoid facies. Overnight dexamethasone suppression test and urinary free cortisol levels confirmed Cushing's syndrome and High dose dexamethasone suppression test showed partial suppression. CT scan of the abdomen showed bilateral hyperplasia of the adrenals with nodularity on the left side, raising the possibility of an adrenal adenoma. ACTH levels were elevated thereby ruling out autonomously functioning adrenal nodule, however increasing the possibility of ectopic ACTH secretion due to the previous medical history. MRI of the pituitary confirmed the presence of an adenoma, thereby pointing to the diagnosis of pituitary dependant Cushing's disease. The patient could not undergo further invasive investigation or surgery due to septicaemia. Medical management of Cushing's syndrome was resorted to in the interim with Ketoconazole, showing excellent response. This case depicts the need for a high index of suspicion for the diagnosis, the importance of organizing specific investigations in the appropriate order to arrive at a diagnosis and an effective management plan. Endogenous Cushing's syndrome is a rare endocrine disorder that is fatal if left untreated. It can be distinguished into adrenocorticotropic hormone (ACTH)-dependent (central and ectopic Cushing's syndrome) and ACTH-independent subtypes (unilateral or bilateral adrenal adenomas). The clinical presentation of patients includes typical stigmata of cortisol excess with physical symptoms of catabolic metabolism (myopathy, striae, parchment skin, osteoporosis) and components of metabolic syndrome (diabetes mellitus, obesity, arterial hypertension, hypercholesterolemia). Biochemical diagnosis is performed in three steps: 1. Confirmation of the diagnosis by 1‑mg dexamethasone suppression test, 24‑h urine free cortisol, and measurement of late-night salivary cortisol. 2. Differentiation of ACTH-dependent Cushing's syndrome from ACTH-independent adrenal Cushing's syndrome by measurement of plasma ACTH. 3. Further subtyping by corticotropin-releasing hormone (CRH) test, inferior petrosal sinus sampling, and imaging modalities. Therapeutic decisions are made on an interdisciplinary basis. First-line therapy for all subtypes is surgery when possible; additional options for all forms include drug therapy and bilateral adrenalectomy. Despite adequate treatment, Cushing's syndrome is associated with increased long-term morbidity and mortality. Interdisciplinary and multimodal therapy management is necessary in the long term to positively influence mortality and reduced quality of life. We report a 49-year-old woman who had minimal features of Cushing syndrome and an incidentally discovered adrenal adenoma. She was subsequently diagnosed with pituitary-dependent Cushing syndrome. Laboratory and imaging studies including serum cortisol, plasma adrenocorticotrophic hormone (ACTH), high dose dexamethasone test, corticotropin-releasing hormone test, computed tomography (CT) scan, and magnetic resonance imaging were performed. A 49-year-old woman was admitted for urosepsis. An abdominal CT scan performed during the urosepsis workup showed a 2.7-cm right adrenal adenoma. She denied any abdominal striae or other symptoms. Physical examination showed normal vital signs, minimal facial fullness without central obesity, and striae. Laboratory results were as follows: 24-hour-urine cortisol 294 μg (reference 4.0-50.0), midnight serum cortisol 23.0 μg/dL (reference < 7.5), and plasma ACTH level 39 pg/mL (reference 5-27). A corticotropin-releasing hormone stimulation test showed >20% rise in serum cortisol and >35% rise in ACTH levels. A pituitary magnetic resonance image showed a 5 mm pituitary lesion. The patient underwent transsphenoidal pituitary surgery, which confirmed an ACTH-secreting lesion. Postoperatively, she required hydrocortisone replacement for the next 10 months. A follow-up adrenal CT performed 6 months later showed a decrease in the size of the adrenal adenoma (1.8 cm). This case highlights the importance of recognizing the coexistence of ACTH-dependent Cushing disease with an adrenal adenoma and partial ACTH dependency of the adrenal adenoma. Endogenous Cushing's syndrome is a very rare entity, with an incidence of 2-4 cases per million inhabitants per year. Cases caused by ectopic ACTH secretion are under-diagnosed. Cushing's disease is the most frequent cause of endogenous Cushing's syndrome, which is 5 or 6 times more frequent than adrenal Cushing's syndrome, with an incidence of between 1.2 and 2.4 cases per million inhabitants per year. Cushing's disease is 3-8 times higher in women than in men. The frequency of adrenal tumors is 3 times higher in women, while that of Cushing's syndrome due to adrenal tumors is 3-5 times higher. Age at diagnosis of Cushing's syndrome varies according to the etiology. Most cases of Cushing's disease are due to a pituitary adenoma, although the tumor may not be visible on the available imaging techniques. ACTH-independent Cushing's syndrome is found in 20% of cases and is most frequently due to adenomas (10%) or adrenal carcinomas (8). Bilateral micronodular hyperplasia and macronodular hyperplasia are infrequent entities, representing less than 10% of all cases of ACTH-independent Cushing's syndrome. Both familial and sporadic forms exist: the familial form, or Carney complex, and ACTH-independent bilateral macronodular hyperplasia, in which the size of the adrenal glands is considerably enlarged. The signs and symptoms of Cushing's syndrome are a direct result of long-term exposure to excessive glucocorticoids. Most signs and symptoms are highly prevalent in the general population (hypertension, central obesity, diabetes mellitus or carbohydrate intolerance, osteoporosis, and characteristic phenotypical alterations). The diagnosis of Cushing's syndrome is one of the most perplexing and controversial problems in endocrinology. However, significant advances in the diagnosis procedures have been made in the past decade. The diagnostic studies involved in the evaluation of patients with suspected Cushing's syndrome fall into two categories: confirming the presence of true hypercortisolism and establishing the precise aetiology. Diagnosis of Cushing's syndrome: ambulatory screening relies on the overnight 1 mg dexamethasone test. Negative tests are confirmed by measuring cortisol in two 24-hour urine samples. If cortisol excretion is slightly above normal, a 48-hour low-dose dexamethasone suppression test or an intravenous infusion dexamethasone suppression test are required. Diagnosis of the aetiology of Cushing's syndrome: the first step is to establish if the hypercortisolism is ACTH-dependent or not. This step is solved by measuring plasma ACTH and cortisol in the late afternoon. Computed tomography scanning of the adrenal glands is required in ACTH-independent Cushing's syndrome. A unilateral tumour will be demonstrated in most of cases. If bilateral lesions are found, dynamic testing using cortisol releasing factor and/or metyrapone must be performed to confirm the ACTH-independency of the syndrome. In ACTH-dependent Cushing's syndrome, the major difficulty is to distinguish between a pituitary source and an ectopic source of ACTH secretion. Magnetic resonance imaging of the pituitary with gadolinium enhancement must be preferred to computed tomography scanning but its sensitivity is not better than 70-80% and false positives can occur. When no macroscopic pituitary lesion can be detected, bilateral inferior petrosal sinus sampling coupled to CRH injection for ACTH measurement will indicate the source of ACTH secretion. If this test indicates the patient has Cushing's disease, pituitary trans-sphenoidal surgery can be performed. If the test indicates the patient has ectopic ACTH-secretion, a cervico-thoraco-abdominal scanning is necessary to identify the tumour. In the case of occult tumour the hypercortisolism must be controlled by pharmacological agents and the imaging investigations must be repeated at appropriate intervals. Cushing's syndrome is a rare but frequently considered disease. Its diagnosis can lead to some difficulties, including confirming the effective endogenous hypercortisolism and determining its cause. The severity of this disease, the diversity of its complications and the multiple therapeutic options make its management challenging. The aim of this review is to present the most recent data about management of Cushing's syndrome, especially diagnostic approaches and therapeutic options. Our references were obtained by screening MEDLINE database from 1996 to 2006. We also included some anterior reviews and consensus statements. We retained the following points: midnight salivary cortisol is a useful tool in the diagnosis of Cushing's syndrome; the desmopressin test can help to distinguish between Cushing's syndrome and \"pseudoCushing's\" due to alcohol consumption or psychiatric disorders; cavernous sinus and inferior petrosal sinus sampling is indicated in the evaluation of ACTH-dependent Cushing's syndromes when pituitary imaging is normal or equivocal or when dynamic tests are contradictory; multislice computed-tomography of the chest and the abdomen and somatostatin analogue scintigraphy, eventually combined, are the best imaging procedures in occult ectopic ACTH syndromes; patients with Cushing's disease should be referred to a neurosurgeon experienced in corticotroph adenomas surgery; metabolic consequences of Cushing's syndrome, such as cardiovascular risk factors and osteoporosis need an aggressive treatment. The incidence of Cushing's syndrome is only 1/100000 per year. However, hypercortisolism is diagnosed by systematic evaluation in 2 to 5% of patients with poorly controlled type 2 diabetes and adrenal incidentalomas. Endocrinological management of the disease improves metabolic disorders in these patients. If these results are confirmed, screening for Cushing's syndrome should be systematically performed in these populations. When Cushing's syndrome is suspected, either a dexamethasone suppression test (administration of dexamethasone and frequent determination of cortisol and ACTH level), or a 24-hour urinary measurement for cortisol offers equal detection rates. Dexamethasone is a glucocorticoid and simulates the effects of cortisol, including negative feedback on the pituitary gland. When dexamethasone is administered and a blood sample is tested, cortisol levels >50 nmol/l (1.81 μg/dl) would be indicative of Cushing's syndrome because an ectopic source of cortisol or ACTH (such as adrenal adenoma) exists which is not inhibited by the dexamethasone. A novel approach, recently cleared by the US FDA, is sampling cortisol in saliva over 24 hours, which may be equally sensitive, as late-night levels of salivary cortisol are high in cushingoid patients. Other pituitary hormone levels may need to be ascertained. Performing a physical examination to determine any visual field defect may be necessary if a ACTH blood test Once Cushing's syndrome has been diagnosed, the first step towards finding the cause is measuring plasma adrenocorticotropic hormone (ACTH) concentration. A concentration consistently below 1.1 pmol/L is classified as corticotropin-independent and does not lead to a diagnosis of Cushing's disease. In such cases, the next step is adrenal imaging with CT. If plasma corticotropin concentrations are consistently above 3.3 pmol/L, then corticotropin-dependent Cushing's syndrome is most likely. Any intermediate values need to be cautiously interpreted and a corticotropin-releasing hormone (CRH) test is advised in order to confirm corticotropin dependency. If corticotropin-dependent Cushing's syndrome is determined then the next step is to distinguish between Cushing's disease and ectopic corticotropin syndrome. This is done via a combination of techniques including CRH, high-dose DST, BIPSS, and pituitary MRI. A 49-year-old man was referred to our hospital for the treatment of gallstones in 1993. Bilateral adrenal nodular masses were detected incidentally by abdominal computed tomography. He had no clinical signs of Cushing's syndrome such as central obesity, striae of skin and diabetes mellitus. We performed cholecystectomy and partial adrenalectomy of right adrenal gland as a biopsy, and diagnosed him as preclinical Cushing's syndrome due to adrenocorticotropin-independent bilateral adrenal macronodular hyperplasia (AIMAH) based on endocrinological and histological examinations. We followed him up for 7 years. During the observation period, the sizes of both adrenal glands increased gradually, and finally serum cortisol level increased beyond normal range, and he showed a Cushingoid appearance such as moon face and central obesity. His skin became atrophic and very fragile, and the bone mineral density of his lumbar spine was extremely low. Serum cortisol level was elevated, and plasma ACTH level was always suppressed. Urinary excretion of 17-hydroxycorticosteroid and free cortisol were increased. Diurnal rhythm of cortisol and ACTH was completely lost and high dose (8 mg/day) dexamethasone did not suppress urinary 17-hydroxycorticosteroid excretion. He became clinically overt Cushing's syndrome. We recommended total adrenalectomy, but he refused it. It is important to know the natural history of preclinical Cushing's syndrome due to AIMAH when choosing an adequate treatment. Corticosteroid overproduction Cushing's syndrome Cushing's syndrome is the manifestation of glucocorticoid excess. It can be the result of a prolonged treatment with glucocorticoids or be caused by an underlying disease which produces alterations in the HPA axis or the production of cortisol. Causes can be further classified into ACTH-dependent or ACTH-independent. The most common cause of endogenous Cushing's syndrome is a pituitary adenoma which causes an excessive production of ACTH. The disease produces a wide variety of signs and symptoms which include obesity, diabetes, increased blood pressure, excessive body hair (hirsutism), osteoporosis, depression, and most distinctively, stretch marks in the skin, caused by its progressive thinning. To review the challenges encountered in the diagnostic work-up and management of patients with subclinical Cushing's syndrome (SCS) and bilateral adrenal masses to aid in the case description of a patient with SCS and adrenocorticotropic hormone (ACTH)-independent macronodular adrenal hyperplasia (AIMAH). We describe our experience managing a patient with AIMAH and SCS. This case report is followed by an extensive review of the literature regarding differential diagnoses, work-up including adrenal venous sampling (AVS), and treatment of SCS with bilateral adrenal masses. A 51-year-old female who was diagnosed with recent onset hypertension and diabetes mellitus type 2 was evaluated for adrenal masses discovered incidentally on computed tomography (CT). She did not have any Cushingoid features. Magnetic resonance imaging (MRI) of abdomen was performed for further evaluation. Hormonal evaluation came back consistent with SCS. The AVS results were consistent with bilateral autonomous cortisol hypersecretion without lateralization. Collectively, the findings favored the diagnosis of bilateral AIMAH. A left adrenalectomy was performed, and the patient's clinical response was favorable with improvement in blood pressure (BP) accompanied by significant weight loss. Follow-up hormonal testing for autonomous cortisol hypersecretion was within the target range. AIMAH is a rare cause of SCS. AVS is a useful diagnostic tool that helps localize the source of autonomous cortisol hypersecretion in ACTH-independent SCS with bilateral adrenal masses, especially if radiological features are inconclusive. Patients undergoing unilateral adrenalectomy should be followed for monitoring of clinical response, as well as progression of AIMAH in the contralateral adrenal gland. Associated diseases Cushing's Disease Corticotropic cells can have detrimental effects on the body if they express too much or too little ACTH. One such example is Cushing's disease, which can result from overproduction of ACTH in the corticotropes due to pituitary tumors known as corticotroph adenomas; this is the cause for roughly two-thirds of those diagnosed with Cushing's disease. It is also possible that this disease can result from production of ACTH in a non-pituitary tumor, known as ectopic production, or the adrenal glands can overproduce cortisol due to an adrenal tumor. This overproduction of ACTH causes an increase in cortisol levels due to increased glucocorticoid synthesis in the adrenal cortex resulting in several associated symptoms. Symptoms of Cushing's disease include: Fatty deposits in the neck or back Stretch marks (striae) Fatigue Osteoporosis Weakened immune system Hypertension When any of these tests is positive, CT scanning of the adrenal gland and MRI of the pituitary gland are performed to detect the presence of any adrenal or pituitary adenomas or incidentalomas (the incidental discovery of harmless lesions). Scintigraphy of the adrenal gland with iodocholesterol scan is occasionally necessary. Occasionally, determining the ACTH levels in various veins in the body by venous catheterization, working towards the pituitary (petrosal sinus sampling) is necessary. In many cases, the tumors causing Cushing's disease are less than 2 mm in size and difficult to detect using MRI or CT imaging. In one study of 261 patients with confirmed pituitary Cushing's disease, only 48% of pituitary lesions were identified using MRI prior to surgery. Plasma CRH levels are inadequate at diagnosis (with the possible exception of tumors secreting CRH) because of peripheral dilution and binding to CRHBP. Cushing's disease, i.e., pituitary ACTH-secreting adenoma causing excess glucocorticoid secretion, is a rare disease with significant mortality and morbidity. Timely diagnosis and appropriate treatment can alter the course of the disease and are therefore mandatory. First step of the diagnostic work-up is the endogenous glucocorticoid excess by measurement of urinary free cortisol, cortisol circadian rhythmicity or suppression by low doses of dexamethasone. In patients with equivocal results, second line tests, such as the dexamethasone-suppressed CRH test and desmopressin stimulation, usually enable the diagnosis to be confirmed. Measurement of plasma ACTH then allows the distinction between ACTH-dependent (e.g., pituitary or extrapituitary neuroendocrine tumors) and ACTH-independent causes (e.g., adrenal tumors). The last step in the diagnostic algorithm is often the most fraught with problems as the distinction between Cushing's disease and ectopic ACTH secretion relies on judicious interpretation of several diagnostic procedures. Positive responses to stimulation with CRH and inhibition by high doses of dexamethasone, if concurrent, enable a pituitary origin to be established whereas conflicting results call for inferior petrosal sinus sampling, the latter to be performed in experienced centres only. Visualisation of the tumor at pituitary imaging is helpful but not required for the diagnosis, as microadenomas often remain undectected by MRI and/or CT scan and, on the other hand, visualisation of a non-secreting incidentaloma may be misleading. Surgical removal of the pituitary tumor is the optimal treatment choice and should be attempted in every patient. Surgical failures as well as relapses can be treated by radiotherapy, medical therapy or, if necessary, bilateral adrenalectomy. Finally, patients cured of Cushing's disease require long-term monitoring given the risk of relapse and clinical burden of associated ailments. Cushing's syndrome, including its mild form/state of adrenal-dependent subset (subclinical Cushing's syndrome; subCS), is known to enhance glucose intolerance, hypertension and obesity. Recently, subclinical Cushing's disease (subCD) has been identified, but its prevalence and the extent of consequent metabolic derangement are unclear. We screened 90 type 2 diabetic patients hospitalized in our department for subCD, according to the diagnostic guideline proposed by the working group of Japanese Ministry of Health, Welfare and Labor in 2006. Plasma ACTH and cortisol levels in the morning and at midnight were determined, and overnight 0.5 mg dexamethasone suppression test (DST) was performed. Those who showed poor cortisol suppression in DST underwent the desmopressin (DDAVP) test. Fifty-seven patients (63.3%) demonstrated abnormally high midnight cortisol levels (>or=2.5 microg/dL), while only nine of them failed to suppress plasma cortisol levels to <3 microg/dL after DST. Although none of the eight patients who underwent the DDAVP test demonstrated the anticipated paradoxical rise in plasma ACTH, these eight patients (8.9%) endocrinologically met the screening criteria of subCD. Since a considerable percentage of pituitary adenomas causing overt Cushing's disease are not identifiable in magnetic resonance imaging, many of those causing subCD may also be unidentifiable. Further follow-up studies including confirmatory testing and pituitary imaging are necessary. The most accurate test used to differentiate a pituitary adenoma from ectopic or adrenal Cushing syndrome is inferior petrosal sinus sampling. [12] [18] This invasive method measures the difference in ACTH level found in the inferior petrosal sinus (where the pituitary gland drains) compared to the periphery. [12] [18] A basal central to the peripheral ratio of over 3:1 when CRH is administered confirms the diagnosis of Cushing disease. [18] This test is considered the gold standard in diagnosing Cushing disease because it has a sensitivity and specificity of nearly 94%. Still, it is rarely used in clinical practice due to its high cost, invasiveness, rare but serious complications, and the required special expertise to perform. is diagnostic 1) Overnight DST2) 24-hour urinary free cortisol3) 11:00 pm salivary cortisol1) Plasma ACTH2) High-dose DST and urinary cortisolConfirm the diagnosisACTH gradient?Determine source of hypercortisolismDecreased ACTHLack of suppressionCT scan adrenalsIncreased ACTHPositive Increased ACTHLack of suppressionEquivocalresultsFurther testingBilateral petrosalvein samplingAdrenalsourcePituitarysourceEctopic ACTHsourceSTEPS IN DIAGNOSISDIAGNOSTIC STUDIESYesNoFigure 38-44. Diagnosis of Cushing’s syndrome. ACTH = adrenocorticotropic hormone; CT = computed tomography; DST = dexamethasone suppression test.Brunicardi_Ch38_p1625-p1704.indd 169001/03/19 11:22 AM 1691THYROID, PARATHYROID, AND ADRENALCHAPTER 38of a pituitary tumor. In patients suspected of having ectopic ACTH production, CT or MRI scans of the chest and anterior mediastinum are performed first, followed by imaging of the neck, abdomen, and pelvis if the initial studies are negative.Treatment Laparoscopic Serum ACTH levels can differentiate ACTH-dependent Cushing syndrome (elevated ACTH or inappropriately normal ACTH) from ACTH-independent (low ACTH level) Cushing syndrome. In patients with ACTH-dependent Cushing syndrome, the high-dose dexamethasone suppression test done by giving 8 mg dexamethasone by mouth at 2300 h and checking cortisol the next day at 0800 h, can differentiate pituitary ACTH from an ectopic ACTH source. A high-dose dexamethasone suppression test will decrease cortisol level by 50 % if the ACTH source is a pituitary adenoma, but not if ACTH is secreted by an ectopic tumor (e.g., oat cell lung carcinoma). Pituitary MRI, unenhanced CT scan of the adrenals, and chest X-ray and CT are also useful to localize the pathology. Cushing's syndrome is caused by either excessive cortisol-like medication, such as prednisone, or a tumor that either produces or results in the production of excessive cortisol by the adrenal glands. Cases due to a pituitary adenoma are known as Cushing's disease, which is the second most common cause of Cushing's syndrome after medication. A number of other tumors, often referred to as ectopic due to their placement outside the pituitary, may also cause Cushing's. Some of these are associated with inherited disorders such as multiple endocrine neoplasia type 1 and Carney complex. Diagnosis requires a number of steps. The first step is to check the medications a person takes. The second step is to measure levels of cortisol in the urine, saliva or in the blood after taking dexamethasone. If this test is abnormal, the cortisol may be measured late at night. If the cortisol remains high, a blood test for ACTH may be done.\nHere is the question:\nENDOCRINOLOGY: A 56-year-old female patient consulted for dorso-lumbar spine pain and progressive difficulty in performing usual tasks. In the last 5 years she gained weight, she has ecchymosis easily and arterial hypertension was detected. Physical examination: Obesity of central predominance, rounded facies, increased supraclavicular fat, decreased proximal muscle strength and some reddish striae in the abdomen. She has a blood glucose of 136 mg/dL and the radiological study showed osteoporosis and vertebral crushing. What do you think is the most coherent interpretation and attitude?\nHere are the potential choices:\n1. Postmenopausal osteoporosis, type 2 diabetes mellitus and essential hypertension, with decreased strength due to diabetic polyneuropathy.\n2. It is necessary to rule out Cushing's disease by dexamethasone suppression test and perform a cranial CT scan.\n3. Suggest Cushing's. Determine urinary free cortisol and basal ACTH, which serves to orient its etiology and select the most appropriate imaging technique.\n4. It looks like Cushing's. If basal ACTH is high, it may be due to corticosteroid use or an adrenal tumor, and an MRI should be performed.\n5. Probably has Cushing's. If the basal ACTH is low, he probably has a pituitary micro-adenoma, and a cranial CT scan should be performed.\nThe correct answer is: 3. Suggest Cushing's. Determine urinary free cortisol and basal ACTH, which serves to orient its etiology and select the most appropriate imaging technique." }, { "question": "You are a helpful medical expert, and your task is to answer a multi-choice medical question using the relevant documents. Please choose the answer from the provided options. Your responses will be used for research purposes only, so please have a definite answer.\nHere are the relevant documents:\n Colorectal Primary signet ring cell carcinoma of the colon and rectum (PSRCCR) is rare, with a reported incidence of less than 1 percent. It has a poor prognosis because symptoms often develop late and it is usually diagnosed at an advanced stage. Five-year survival rates in previous studies ranged from nine to 30 percent. Average survival was between 20 and 45 months. It tends to affect younger adults with higher likelihood of lymphovascular invasion. It is worth noting that the overall survival rate of patients with SRCC was significantly poorer than that of patients with mucinous or poorly differentiated adenocarcinoma. In advanced gastric cancers, the prognosis for patients with the SRCCs was significantly worse than for the other histological types, which can be explained by the finding that advanced SRCC gastric cancers have a larger tumor size, more lymph node metastasis, a deeper invasive depth and more Borrmann type 4 lesions than other types. Stomach The U.S. National Comprehensive Cancer Network and American Society of Clinical Oncology provide guidelines for the follow-up of colon cancer. A medical history and physical examination are recommended every 3 to 6 months for 2 years, then every 6 months for 5 years. Carcinoembryonic antigen blood level measurements follow the same timing, but are only advised for people with T2 or greater lesions who are candidates for intervention. A CT-scan of the chest, abdomen and pelvis can be considered annually for the first 3 years for people who are at high risk of recurrence (for example, those who had poorly differentiated tumors or venous or lymphatic invasion) and are candidates for curative surgery (with the aim to cure). A colonoscopy can be done after 1 year, except if it could not be done during the initial staging because of an obstructing mass, in which case it should be performed after 3 to 6 months. If a villous polyp, a polyp >1 centimeter or high-grade dysplasia is found, it Microscopy Adenocarcinoma is a malignant epithelial tumor, originating from superficial glandular epithelial cells lining the colon and rectum. It invades the wall, infiltrating the muscularis mucosae layer, the submucosa, and then the muscularis propria. Tumor cells describe irregular tubular structures, harboring pluristratification, multiple lumens, reduced stroma (\"back to back\" aspect). Sometimes, tumor cells are discohesive and secrete mucus, which invades the interstitium producing large pools of mucus. This occurs in mucinous adenocarcinoma, in which cells are poorly differentiated. If the mucus remains inside the tumor cell, it pushes the nucleus at the periphery, this occurs in \"signet-ring cell.\" Depending on glandular architecture, cellular pleomorphism, and mucosecretion of the predominant pattern, adenocarcinoma may present three degrees of differentiation: well, moderately, and poorly differentiated. Micrographs (H&E stain) Microscopic criteria To investigate the clinicopathological features and prognosis of colorectal synchronous multiple primary cancer(SMPC). From January 2008 to June 2011, 51 patients diagnosed with colorectal SMPC underwent surgery at Department of General Surgery of Peking University First Hospital. Their clinicopathological features, diagnosis, treatment and prognosis were summarized and analyzed. SMPC was diagnosed according to the following criteria: each tumor must have a definite pathologic picture of malignancy; metastasis or recurrence from another colorectal cancer was excluded; tumors must be distinctly separated by at least 5 cm of all intact bowel wall from each other; SMPC has abnormal cells between tumor and normal mucosa and abnormal gland of transitional zone; each cancer is infiltrating carcinoma except the carcinoma in situ; all the cancers are detected at the same time or within 6 months. Multiple primary colorectal cancer originated from familial colonic polyposis or ulcerative colitis was excluded. These 51 colorectal SMPC patients accounted for 3.5% of 1 452 colorectal cancer patients in the same period at our hospital, with 32 males and 19 females, and mean age of (63±13)(29 to 82) years. Of 51 cases, 46(90.2%) had 2 original carcinoma, 3(5.9%) had 3 original carcinoma and 2(3.9%) had 4 carcinoma; 23(45.1%) complicated with colon polyps, 4(7.8%) complicated with malignancy outside the colorectum. In TNM staging, 7(13.7%), 15(29.4%), 24(47.1%) and 5(9.8%) patients were stage I(, II(, III( and IIII( respectively. Among 51 patients undergoing surgery by different procedures, 16 were subtotal colon resection, 8 were extended right colon resection, 5 were extended left hemicolon resection, 8 were right hemicolon resection plus Dixon procedure, 10 were Dixon, and 4 were right hemicolon resection plus sigmoid colon resection. Adjuvant chemotherapy and support treatment were given according to the condition after operation. A total of 105 tumors were found, including 25(23.8%) tumors in sigmoid colon, 24(22.9%) in rectum, 22(21.0%) in ascending colon and 4 in organs outside the colorectum. Tubular adenocarcinoma (86/105, 81.9%) was the main pathological type in these colorectal SMPC patients. During the follow-up of median 43.5 months, 10 cases presented local recurrence and 6 cases had liver metastasis. Multivariable analysis showed that ≤65 years old (OR=22.757, 95%CI: 1.562-331.543, P=0.002),undifferentiated carcinoma or mucous adenocarcinoma (OR=27.174, 95%CI: 2.834-260.512, P=0.004), stage III(-IIII( (OR=29.626, 95%CI: 3.216-272.884, P=0.003) were independent risk factors of postoperative 5-year recurrence and metastasis, but the number of SMPC lesions and the surgical method were not associated with postoperative 5-year recurrence and metastasis (P=0.564, P=0.513). The 3-year and 5-year survival rates of colorectal SMPC patients were 76.5% and 64.7%. Two-original carcinoma is the most common in colorectal SMPC patients, which mainly distributes in sigmoid colon and rectum. Postoperative monitoring should be strengthened for those patients with younger age, poor pathological types and advanced staging to prevent recurrence and metastasis. In Stage I colon cancer, no chemotherapy is offered, and surgery is the definitive treatment. The role of chemotherapy in Stage II colon cancer is debatable, and is usually not offered unless risk factors such as T4 tumor, undifferentiated tumor, vascular and perineural invasion or inadequate lymph node sampling is identified. It is also known that the people who carry abnormalities of the mismatch repair genes do not benefit from chemotherapy. For stage III and Stage IV colon cancer, chemotherapy is an integral part of treatment. Direct and indirect mechanisms contribute to chemotherapy resistance. Direct mechanisms include pathways that metabolize the drug, while indirect mechanisms include pathways that respond to the chemotherapy treatment. The NER DNA repair pathway plays a substantial role in reversing cell damage caused by chemotherapeutic agents such as 5-FU. Discoveries since 2010 In May 2017 the FDA approved an immunotherapeutic called Keytruda (pembrolizumab) (PD-1 inhibitor) for patients with unresectable or metastatic microsatellite instability-high (MSI-H) or mismatch repair deficient (dMMR) solid tumors that have progressed following prior treatment. This indication is independent of PD-L1 expression assessment, tissue type and tumor location. Researchers have found another MSI, called elevated microsatellite alterations at selected tetranucleotide repeats (EMAST). However, EMAST is unique in that it is not derived from MMR, and it is commonly associated with TP53 mutations. It is a common opinion that the more often and the more rigorously the colon is examined, the more lesions will be discovered and diagnosed. However it has not been shown which methods of colonic examination and which regimen of surveillance should be used. Chart review was conducted on 481 patients who underwent curative resection for colorectal cancer between 1980 and 1990. Colonoscopy was performed preoperatively, after 12-15 months from surgical treatment, and then at an interval of 12-24 months, or when symptoms appeared. About ten percent of patients developed intraluminal recurrences, and more than 25% adenomatous polyps. More than one half of the metachronous lesions arise within the first 24 months. The median time to diagnosis was 25 months for intraluminal recurrences and 22 months for adenomatous polyps. Patients with left sited tumor at an advanced stage run a higher risk of developing recurrent intraluminal disease, and patients who presented associated polyps at the time of the operation for the index cancer have a higher risk of developing new polyps. About 50% of recurrences were detected when patients were asymptomatic. Colonoscopy must be performed within the first 12-15 months after operation, while an interval of 24 months between each examination seems sufficient to guarantee an early detection of metachronous lesion. Asymptomatic patients are more frequently reoperated for cure and thus have a better survival rate. Bowel cancer may be diagnosed by obtaining a sample of the colon during a sigmoidoscopy or colonoscopy. This is then followed by medical imaging to determine whether the disease has spread. Screening is effective for preventing and decreasing deaths from colorectal cancer. Screening, by one of a number of methods, is recommended starting from the age of 50 to 75. During colonoscopy, small polyps may be removed if found. If a large polyp or tumor is found, a biopsy may be performed to check if it is cancerous. Aspirin and other non-steroidal anti-inflammatory drugs decrease the risk. Their general use is not recommended for this purpose, however, due to side effects. There are two types of liquid biopsy (which is not really a biopsy as they are blood tests that do not require a biopsy of tissue): circulating tumor cell assays or cell-free circulating tumor DNA tests. These methods provide a non-invasive alternative to repeat invasive biopsies to monitor cancer treatment, test available drugs against the circulating tumor cells, evaluate the mutations in cancer and plan individualized treatments. In addition, because cancer is a heterogeneous genetic disease, and excisional biopsies provide only a snapshot in time of some of the rapid, dynamic genetic changes occurring in tumors, liquid biopsies provide some advantages over tissue biopsy-based genomic testing. In addition, excisional biopsies are invasive, can’t be used repeatedly, and are ineffective in understanding the dynamics of tumor progression and metastasis. By detecting, quantifying and characterisation of vital circulating tumor cells or genomic alterations in CTCs and cell-free DNA in Colorectal cancer is predominantly a disease of older population, but occasionally it affects younger patients, in whom very often diagnosis is overseen and treatment begins late. The aim of our study was to compare localization, clinical and pathological characteristics and survival of sporadic colorectal cancer patients aged up to 40 and over 65 years. The first group (group I) included 19 patients under 40 years and the second group (group II) 28 patients aged over 65 years, treated during 1997-2001. Patients with family history of colon cancer and inflammatory disease of the colon were not included. Arithmetic mean, standard deviation, Fisher's test, Student 's t test, x(2) test and the Kaplan-Meier method were used in the statistical analysis of the results. There was no difference among the tested groups regarding tumor localization. The most frequent localization was in the rectum and left colon. At presentation, in group I patients, besides the metastases in the liver and lymph nodes, colorectal cancer infiltrated also the duodenum, stomach, right kidney capsule in one patient, and adnexa in two patients. In group II patients we registered only liver and lymph node metastases. Pathologically, tubular and mucinous forms were present in all of the patients up to 40 years of age, while only one patient over 65 had tumor with mucinous component. In group I, Astler-Coller stage B was found in 1.5% of the patients, stage C in 72.5% and stage D in 26%; in group II, stage B was found in 1.5%, stage C in 84.5% and stage D in 14%. Grade III was 36.8% in group I and 17.8% in group II. No statistical differences were found in stage distribution (p=0.36) and grade (p=0.06) between group I and II. Five-year overall survival was 57.8% and 28.5% in younger and older patients, respectively (p=0.053). The results obtained showed no difference in clinical symptomatology and tumor localization in both groups. The incidence of more aggressive tumors was higher in younger persons. However, early detection combined with more aggressive therapeutic approach, could enable significant improvement of the 5-year survival of younger patients with colon cancer. Microsatellite instability diagnostics MSI is a good marker for determining Lynch syndrome and determining a prognosis for cancer treatments. In 1996, the National Cancer Institute (NCI) hosted an international workshop on Lynch Syndrome, which led to the development of the “Bethesda Guidelines” and loci for MSI testing. During this first workshop the NCI has agreed on five microsatellite markers necessary to determine MSI presence: two mononucleotides, BAT25 and BAT26, and three dinucleotide repeats, D2S123, D5S346, and D17S250. MSI-H tumors result from MSI of greater than 30% of unstable MSI loci (>2 or more of the 5 loci). MSI-L tumors result from less than 30% of unstable MSI biomarkers. MSI-L tumors are classified as tumors of alternative etiologies. Several studies demonstrate that MSI-H patients respond best to surgery alone, rather than chemotherapy and surgery, thus preventing patients from needlessly experiencing chemotherapy. There has been an unremitting rise in incidence of colonic cancer in this country with no recent improvement in cure rate. As a result the evolution of colorectal cancer has been the focus of considerable attention with an enlarging body of evidence pointing to the common neoplastic polyp as a precursor to malignancy. \"Neoplastic\" polyps include \"adenomatous polyps,\" \"villous adenomas\" and, lately recognized, \"villo-glandular polyps.\" Experience with endoscopic removal of over 2,000 colonic polyps (with no mortality) has introduced two questions of prime concern to the surgeon: (1) What constitutes clinical malignancy in a polyp? AND, (2) When should laparatomy supplant or follow endoscopic removal? Eight hundred and ninety-two consecutive adenomatous (tubular), villous, villoglandular (villo-tubular) and \"polypoid cancer\" polyps are analyzed, 855 of which have been followed for 6 months to 4 years. Support is offered to the concept that villous and tubular growth patterns are merely variants of a similar base disturbance in cell renewal. Superficial cancer (carcinoma-in-situ) occurred in 6.6% of neoplastic polyps and represents no threat if the polyp is completely removed. Only when the cancer penetrates the muscularis mucosae should it be regarded as \"invasive.\" The term \"malignant polyp\" should be reserved for this form. Invasive cancer was found in 5.0% of neoplastic polyps in this series. Only in this group need the question of further surgical intervention be raised. Major considerations influencing a decision for subsequent laparotomy are polyp size and gross morphology (i.e. sessile or pedunculated), histologic type (of the polyp and of the cancer itself), adequacy of clearance between depth of invasion and plane of polyp resection, and the patient's age and general condition. These are analyzed. Twenty-five of 46 patients with \"malignant polyps\" were subjected to abdominal exploration: 17 showed no residual cancer, whereas 8 (5 with recognized incomplete endoscopic removal) had tumor in the bowel wall. Of the remaining 21 patients, for whom endoscopic polypectomy alone was deemed appropriate, none have shown residual or recurrent cancer on clinical and endoscopic followup. Colonoscopy appears to be a most promising approach in terms of the goals of cancer programs, offering both prophylaxis and opportunity for treatment at a favorable stage of disease. Treatment The specific treatment will depend on the tumor's type, location, size, and whether the cancer has spread to other organs. Surgical removal of the tumor remains the standard treatment of choice, but additional forms of therapy such as radiation therapy, chemotherapy, or immunotherapy exist. When detected early, skin cancer in cats and dogs can often be treated successfully. In many cases, a biopsy can remove the whole tumor, as long as the healthy tissues removed from just outside the tumor area do not contain any cancer cells. References External links Skin Cancer in Cats and Dogs from Pet Cancer Center Skin Cancer in Dogs from CanineCancer.com' Types of animal cancers Cancer in dogs Cancer in cats Integumentary neoplasia The authors examined the five-years postoperative survival rate of fifty patients who suffered from colorectal cancer along with the fact that the large bowel one of the neighbouring organs were resected. The subjects were divided into four groups: the colorectal resection was associated with (1) stomach resection (13 patients); (2) liver metastasectomy (14 patients); (3) small bowel resection (10 patients); (4) the resection of other organs (13 patients). In the first two years of the study they were examined once in every three months, in the next two years once in every six months and then yearly. The following tests were carried out: chest X ray, abdominal sonography, irrigography or colonoscopy and CEA. On condition that the colonoscope reached the caecum and the result was negative, the test was repeated only a year later. The patients were operated on between 1985 and 1997. The statistical analysis was made with the help of the Kaplan-Meier method. During this period fifty-six complex resections were performed. Out of fifty-six patients fifty were followed. Compliance 89%. In group 1, where the average age of patients was sixty-two years, one patient died in the forty-first and the other in the fifty-second month after the surgery. Survival rate: 11/13 (83%). The survival rates for the other groups were as follows: group 2 (average age 64) twelve patients died within five years. Survival rate 2/14 (14%). The difference between the survival rates in the first two groups in significant (P = 0.0001). Group 3 (average age 67) seven died and only three survived. Survival rate: 3/10 (30%). The difference between group 1 and group 3 is significant (P = 0.0022). Group 4 (average age 64) seven patients died. Survival rate 6/13 (46%). Comparing this rate to that of the group 1, the difference is not significant (P > 0.01). Having analysed the results of the four groups it can be concluded that the patients of group 1 lived the longest (stomach resection) and those of group 2 (liver metastasectomy) died the earliest after the operation. It is surprising that the patients of group 3 lived significantly shorter than the ones of group 1 in spite of the fact that they belong by far the greatest number to stage Dukes B (group 1: 12/13 = 92%; group 3: 7/10 = 70%). The authors assume that the partial or the total absence of the stomach keeps back the growth of the tumour (gastro-colic tumour growing dependency). They think that in case of colon cancer which infiltrates the stomach surgeons experienced in gastric and colorectal surgery should be encouraged to take the risk of the double resection providing the fact that the operation is accomplishable. in understanding the dynamics of tumor progression and metastasis. By detecting, quantifying and characterisation of vital circulating tumor cells or genomic alterations in CTCs and cell-free DNA in blood, liquid biopsy can provide real-time information on the stage of tumor progression, treatment effectiveness, and cancer metastasis risk. This technological development could make it possible to diagnose and manage cancer from repeated blood tests rather than from a traditional biopsy. Colo-rectal adenoms occur more frequently in the elderly and should be considered as precancerous. The structural changes of the glandular epithelium are known as dysplasia or atypia and are classified into three grades of severity; the \"severe epithelial dysplasia\" has all the histological characteristics of a malignant tumor which however has not infiltrated the muscularis mucosa and so has not gained access to the lymphatic system. Whenever these structural changes were present the terms focal carcinoma or carcinoma in situ were used. However in 1976 the WHO accepted to change the nomenclature to \"severe epithelial dysplasia\", as Morson had proposed. Their aim was to avoid superfluous radical surgical intervention. Whenever severe dysplasia is present in an adenoma, the necessary therapy is the local excision of the adenoma together with its pedick. An exact complete histological examination is necessary. Between 1976 and 1980 we saw 201 cases of adenoma of the colon or rectum at the Surgical Clinic, University of Düsseldorf. 27 of these cases showed severe epithelial dysplasia. As described in the literature there was a correlation between the size of the adenoma, the histological picture and the risk of malignancy. The reexamination of 105 patients showed that there was a significant percentage of recurrency at the site of excision or new polyps at a different site. Therefore, regular checkups are a must for all those patients in whom polyps of the large bowel have been removed. The diagnosis of LPF depends on its clinical presentation almost exclusively in newborn and young children and, most importantly, its histopathology as determined on biopsied intact tissue or fine-needle aspiration to obtain a sampling of the tumor's cells. Intact tissue samples typical show abundant mature-appearing adipose (i.e. fat) tissue mixed with a minor component of oval-shaped or spindle-shaped fibroblast-like cells some of which have a pseudolipoblast-like morphology. Needle biopsies should show these cells. However, LPF histopathology can vary widely between cases. The cited gene abnormalities in the above section are insufficient to support a diagnosis of LPF although further study of these and discoveries of other gene abnormalities may do so. The histopathology of lipofibromatosis-like neural tumors (LPF-NT) can closely resemble LPF tumors. Unlike LPF tumors, however, LPF-NT tumors have been diagnosed in adults in more than 27% of cases with the remaining cases diagnosed Three hundred and seventy-two patients with colorectal tumours treated by curative resection between January 1982 and January 1992 were reviewed in order to determine the role of colonoscopy and the outcome of patients with multiple tumours. Thirty (8.1%) of them with a mean age of 57 (35-79) years (20 males, 10 females) had synchronous (19 cases) or metachronous (11 cases) lesions. Rectum and sigmoid colon were the most frequent site of multiple lesions, accounting for 73% of all lesions. Accurate pre-operative diagnosis was performed in 14 of the 19 patients with synchronous lesions, and in the remaining 5 cases failure to perform an intra-operative colonoscopy was the cause of missing the lesions. Three of them had over-looked lesions on the previous barium enema. Synchronous lesions has the tendency to be less invasive as compared to metachronous ones. Five-year survival rates (Kaplan-Meier method) were 45% and 58% for patients with multiple and single lesions respectively (not significant). For patients with colorectal carcinoma a thorough examination of the whole colon by intra-operative colonoscopy should be accomplished in order to rule out the possibility of associated lesions as well as to decrease the incidence of \"early\" metachronous lesions. Tumors are formed by carcinogenesis, a process in which cellular alterations lead to the formation of cancer. Multistage carcinogenesis involves the sequential genetic or epigenetic changes to a cell's DNA, where each step produces a more advanced tumor. It is often broken down into three stages; initiation, promotion and progression, and several mutations may occur at each stage. Initiation is where the first genetic mutation occurs in a cell. Promotion is the clonal expansion (repeated division) of this transformed cell into a visible tumor that is usually benign. Following promotion, progression may take place where more genetic mutations are acquired in a sub-population of tumor cells. Progression changes the benign tumor into a malignant tumor. A prominent and well studied example of this phenomenon is the tubular adenoma, a common type of colon polyp which is an important precursor to colon cancer. The cells in tubular adenomas, like most tumors that frequently progress to The emergence of drug-resistant tumors due to intra- and inter-tumoral heterogeneity an issue in treatment efficacy. A minor genetic clone within the tumor can expand after treatment if it carries a drug-resistant mutation. Initial biopsies can miss these clones due to low frequency or spatial separation of cells within the tumor. For example, since a biopsy only samples a small part of the tumor, clones that resides in a different location may go unnoticed. This can mislead research that focuses on studying the role of tumor heterogeneity in cancer progression and relapse. The use of ctDNA in research can alleviate these concerns because it could provide a more representative 'screenshot' of the genetic diversity of cancer at both primary and metastatic sites. For example, ctDNA has been shown to be useful in studying the clonal evolution of a patient’s cancer before and after treatment regimens. Early detection of cancer is still challenging but recent progress in the analysis of Flexible sigmoidoscopy is indicated for colorectal cancer screening. The decision about who needs total colonoscopy based on distal findings is still controversial because of the uncertainty of the associations between distal and proximal findings. The purpose of the study was to characterize distal findings in patients with total colonoscopy, to investigate its importance as markers of advanced proximal lesions and to evaluate the usefulness of a clinical Predictive Index, already published in the literature, in the identification of these lesions. Retrospective analysis of the patients submitted to total colonoscopy between January 2006 and February 2007, with selection of 1000 consecutive cases with reference to polyps. We analysed demographic data, indication for the exam and morphological and histological characteristics of the polyps. Advanced lesion was defined as any adenoma larger than 10 mm or any polyp with villous characteristics, high grade dysplasia or cancer. The Predictive Index was obtained through the assignment of points to 3 categories: sex, age and distal findings, which result in 3 groups: low, intermediate and high risk. The mean age of patients was 64,69 years and 65,1% were male. Distal and proximal polyps were identified in 829 (82,9%) and 369 (36,9%) patients, respectively. Advanced distal lesion was found in 342 patients (34,2%) and advanced proximal lesion in 98 (9,8%). 587 patients (58,7%) were in the high risk group. In the group of patients with advanced proximal lesion, a third presented low and intermediate risk, 52% had no distal polyps, 88,7% had less than three distal polyps and 71,4% had no advanced distal lesion. Sensitivity values for these four categories ranged between 11,2% and 66,6%. If the decision to perform total colonoscopy is based on distal colonic findings or on the Predictive Index, the ability to identify advanced proximal lesion is markedly reduced, endangering the aim of a screening program. For the esophagous Endoscopic mucosal resection has been advocated for early esophageal cancers (that is, those that are superficial and confined to the mucosa only) and has been shown to be a less invasive, safe, and effective therapy for early squamous cell carcinoma. It has also been shown to be safe and effective for early adenocarcinoma arising in Barrett’s esophagus. The prognosis after treatment with this method is comparable to surgical resection. This technique can be attempted in patients who have no evidence of nodal or distant metastases, with differentiated tumors that are slightly raised and less than 2 cm in diameter, or in differentiated tumors that are ulcerated and less than 1 cm in diameter. The most commonly employed modalities of endoscopic mucosal resection include strip biopsy, double-snare polypectomy, resection with combined use of highly concentrated saline and epinephrine, and resection using a cap. In contrast to earlier studies that suggested that colon carcinoma is unusually lethal in the young, 69 patients, ages 20 to 39 years, had a relatively good prognosis. Fifty-nine percent lived over 5 years after diagnosis, and 51% were cured. Furthermore, 67% were cured if they did not have distant spread of the carcinoma at the time of the initial operation. Neither age, sex, tumor size, location, mere presence of lymph node metastases, depth of tumor invasion, nor predisposing disease of the colon was a strong prognostic factor. Metastases to six or more lymph nodes and distant spread of the tumor at the time of initial surgery were ominous findings. Mucinous carcinoma was relatively frequent (28%) and was also an ominous feature (only 5 of 20 patients cured as opposed to 26 of 43 with classical adenocarcinoma).\nHere is the question:\nDIGESTIVE: During a colonoscopy, a 5-cm tumor located in the right colon is detected in a 48-year-old man. No other lesions were found. His maternal grandmother also suffered from colon cancer. The biopsies are superficial and show a poorly differentiated tumor with abundant inflammatory cells in the stroma that is diagnosed as a medullary type carcinoma.\nHere are the potential choices:\n1. Chemotherapy is the treatment of choice.\n2. Since the biopsy is superficial, it should be repeated before proceeding with treatment.\n3. The prognosis of the tumor depends mainly on its high degree of anaplasia.\n4. It is unlikely that this tumor has developed over a previous adenoma.\n5. Microsatellite instability and DNA error repair genes should be studied.\nThe correct answer is: ", "gold_answer": "5 Microsatellite instability and DNA error repair genes should be studied.", "prediction": "You are a helpful medical expert, and your task is to answer a multi-choice medical question using the relevant documents. Please choose the answer from the provided options. Your responses will be used for research purposes only, so please have a definite answer.\nHere are the relevant documents:\n Colorectal Primary signet ring cell carcinoma of the colon and rectum (PSRCCR) is rare, with a reported incidence of less than 1 percent. It has a poor prognosis because symptoms often develop late and it is usually diagnosed at an advanced stage. Five-year survival rates in previous studies ranged from nine to 30 percent. Average survival was between 20 and 45 months. It tends to affect younger adults with higher likelihood of lymphovascular invasion. It is worth noting that the overall survival rate of patients with SRCC was significantly poorer than that of patients with mucinous or poorly differentiated adenocarcinoma. In advanced gastric cancers, the prognosis for patients with the SRCCs was significantly worse than for the other histological types, which can be explained by the finding that advanced SRCC gastric cancers have a larger tumor size, more lymph node metastasis, a deeper invasive depth and more Borrmann type 4 lesions than other types. Stomach The U.S. National Comprehensive Cancer Network and American Society of Clinical Oncology provide guidelines for the follow-up of colon cancer. A medical history and physical examination are recommended every 3 to 6 months for 2 years, then every 6 months for 5 years. Carcinoembryonic antigen blood level measurements follow the same timing, but are only advised for people with T2 or greater lesions who are candidates for intervention. A CT-scan of the chest, abdomen and pelvis can be considered annually for the first 3 years for people who are at high risk of recurrence (for example, those who had poorly differentiated tumors or venous or lymphatic invasion) and are candidates for curative surgery (with the aim to cure). A colonoscopy can be done after 1 year, except if it could not be done during the initial staging because of an obstructing mass, in which case it should be performed after 3 to 6 months. If a villous polyp, a polyp >1 centimeter or high-grade dysplasia is found, it Microscopy Adenocarcinoma is a malignant epithelial tumor, originating from superficial glandular epithelial cells lining the colon and rectum. It invades the wall, infiltrating the muscularis mucosae layer, the submucosa, and then the muscularis propria. Tumor cells describe irregular tubular structures, harboring pluristratification, multiple lumens, reduced stroma (\"back to back\" aspect). Sometimes, tumor cells are discohesive and secrete mucus, which invades the interstitium producing large pools of mucus. This occurs in mucinous adenocarcinoma, in which cells are poorly differentiated. If the mucus remains inside the tumor cell, it pushes the nucleus at the periphery, this occurs in \"signet-ring cell.\" Depending on glandular architecture, cellular pleomorphism, and mucosecretion of the predominant pattern, adenocarcinoma may present three degrees of differentiation: well, moderately, and poorly differentiated. Micrographs (H&E stain) Microscopic criteria To investigate the clinicopathological features and prognosis of colorectal synchronous multiple primary cancer(SMPC). From January 2008 to June 2011, 51 patients diagnosed with colorectal SMPC underwent surgery at Department of General Surgery of Peking University First Hospital. Their clinicopathological features, diagnosis, treatment and prognosis were summarized and analyzed. SMPC was diagnosed according to the following criteria: each tumor must have a definite pathologic picture of malignancy; metastasis or recurrence from another colorectal cancer was excluded; tumors must be distinctly separated by at least 5 cm of all intact bowel wall from each other; SMPC has abnormal cells between tumor and normal mucosa and abnormal gland of transitional zone; each cancer is infiltrating carcinoma except the carcinoma in situ; all the cancers are detected at the same time or within 6 months. Multiple primary colorectal cancer originated from familial colonic polyposis or ulcerative colitis was excluded. These 51 colorectal SMPC patients accounted for 3.5% of 1 452 colorectal cancer patients in the same period at our hospital, with 32 males and 19 females, and mean age of (63±13)(29 to 82) years. Of 51 cases, 46(90.2%) had 2 original carcinoma, 3(5.9%) had 3 original carcinoma and 2(3.9%) had 4 carcinoma; 23(45.1%) complicated with colon polyps, 4(7.8%) complicated with malignancy outside the colorectum. In TNM staging, 7(13.7%), 15(29.4%), 24(47.1%) and 5(9.8%) patients were stage I(, II(, III( and IIII( respectively. Among 51 patients undergoing surgery by different procedures, 16 were subtotal colon resection, 8 were extended right colon resection, 5 were extended left hemicolon resection, 8 were right hemicolon resection plus Dixon procedure, 10 were Dixon, and 4 were right hemicolon resection plus sigmoid colon resection. Adjuvant chemotherapy and support treatment were given according to the condition after operation. A total of 105 tumors were found, including 25(23.8%) tumors in sigmoid colon, 24(22.9%) in rectum, 22(21.0%) in ascending colon and 4 in organs outside the colorectum. Tubular adenocarcinoma (86/105, 81.9%) was the main pathological type in these colorectal SMPC patients. During the follow-up of median 43.5 months, 10 cases presented local recurrence and 6 cases had liver metastasis. Multivariable analysis showed that ≤65 years old (OR=22.757, 95%CI: 1.562-331.543, P=0.002),undifferentiated carcinoma or mucous adenocarcinoma (OR=27.174, 95%CI: 2.834-260.512, P=0.004), stage III(-IIII( (OR=29.626, 95%CI: 3.216-272.884, P=0.003) were independent risk factors of postoperative 5-year recurrence and metastasis, but the number of SMPC lesions and the surgical method were not associated with postoperative 5-year recurrence and metastasis (P=0.564, P=0.513). The 3-year and 5-year survival rates of colorectal SMPC patients were 76.5% and 64.7%. Two-original carcinoma is the most common in colorectal SMPC patients, which mainly distributes in sigmoid colon and rectum. Postoperative monitoring should be strengthened for those patients with younger age, poor pathological types and advanced staging to prevent recurrence and metastasis. In Stage I colon cancer, no chemotherapy is offered, and surgery is the definitive treatment. The role of chemotherapy in Stage II colon cancer is debatable, and is usually not offered unless risk factors such as T4 tumor, undifferentiated tumor, vascular and perineural invasion or inadequate lymph node sampling is identified. It is also known that the people who carry abnormalities of the mismatch repair genes do not benefit from chemotherapy. For stage III and Stage IV colon cancer, chemotherapy is an integral part of treatment. Direct and indirect mechanisms contribute to chemotherapy resistance. Direct mechanisms include pathways that metabolize the drug, while indirect mechanisms include pathways that respond to the chemotherapy treatment. The NER DNA repair pathway plays a substantial role in reversing cell damage caused by chemotherapeutic agents such as 5-FU. Discoveries since 2010 In May 2017 the FDA approved an immunotherapeutic called Keytruda (pembrolizumab) (PD-1 inhibitor) for patients with unresectable or metastatic microsatellite instability-high (MSI-H) or mismatch repair deficient (dMMR) solid tumors that have progressed following prior treatment. This indication is independent of PD-L1 expression assessment, tissue type and tumor location. Researchers have found another MSI, called elevated microsatellite alterations at selected tetranucleotide repeats (EMAST). However, EMAST is unique in that it is not derived from MMR, and it is commonly associated with TP53 mutations. It is a common opinion that the more often and the more rigorously the colon is examined, the more lesions will be discovered and diagnosed. However it has not been shown which methods of colonic examination and which regimen of surveillance should be used. Chart review was conducted on 481 patients who underwent curative resection for colorectal cancer between 1980 and 1990. Colonoscopy was performed preoperatively, after 12-15 months from surgical treatment, and then at an interval of 12-24 months, or when symptoms appeared. About ten percent of patients developed intraluminal recurrences, and more than 25% adenomatous polyps. More than one half of the metachronous lesions arise within the first 24 months. The median time to diagnosis was 25 months for intraluminal recurrences and 22 months for adenomatous polyps. Patients with left sited tumor at an advanced stage run a higher risk of developing recurrent intraluminal disease, and patients who presented associated polyps at the time of the operation for the index cancer have a higher risk of developing new polyps. About 50% of recurrences were detected when patients were asymptomatic. Colonoscopy must be performed within the first 12-15 months after operation, while an interval of 24 months between each examination seems sufficient to guarantee an early detection of metachronous lesion. Asymptomatic patients are more frequently reoperated for cure and thus have a better survival rate. Bowel cancer may be diagnosed by obtaining a sample of the colon during a sigmoidoscopy or colonoscopy. This is then followed by medical imaging to determine whether the disease has spread. Screening is effective for preventing and decreasing deaths from colorectal cancer. Screening, by one of a number of methods, is recommended starting from the age of 50 to 75. During colonoscopy, small polyps may be removed if found. If a large polyp or tumor is found, a biopsy may be performed to check if it is cancerous. Aspirin and other non-steroidal anti-inflammatory drugs decrease the risk. Their general use is not recommended for this purpose, however, due to side effects. There are two types of liquid biopsy (which is not really a biopsy as they are blood tests that do not require a biopsy of tissue): circulating tumor cell assays or cell-free circulating tumor DNA tests. These methods provide a non-invasive alternative to repeat invasive biopsies to monitor cancer treatment, test available drugs against the circulating tumor cells, evaluate the mutations in cancer and plan individualized treatments. In addition, because cancer is a heterogeneous genetic disease, and excisional biopsies provide only a snapshot in time of some of the rapid, dynamic genetic changes occurring in tumors, liquid biopsies provide some advantages over tissue biopsy-based genomic testing. In addition, excisional biopsies are invasive, can’t be used repeatedly, and are ineffective in understanding the dynamics of tumor progression and metastasis. By detecting, quantifying and characterisation of vital circulating tumor cells or genomic alterations in CTCs and cell-free DNA in Colorectal cancer is predominantly a disease of older population, but occasionally it affects younger patients, in whom very often diagnosis is overseen and treatment begins late. The aim of our study was to compare localization, clinical and pathological characteristics and survival of sporadic colorectal cancer patients aged up to 40 and over 65 years. The first group (group I) included 19 patients under 40 years and the second group (group II) 28 patients aged over 65 years, treated during 1997-2001. Patients with family history of colon cancer and inflammatory disease of the colon were not included. Arithmetic mean, standard deviation, Fisher's test, Student 's t test, x(2) test and the Kaplan-Meier method were used in the statistical analysis of the results. There was no difference among the tested groups regarding tumor localization. The most frequent localization was in the rectum and left colon. At presentation, in group I patients, besides the metastases in the liver and lymph nodes, colorectal cancer infiltrated also the duodenum, stomach, right kidney capsule in one patient, and adnexa in two patients. In group II patients we registered only liver and lymph node metastases. Pathologically, tubular and mucinous forms were present in all of the patients up to 40 years of age, while only one patient over 65 had tumor with mucinous component. In group I, Astler-Coller stage B was found in 1.5% of the patients, stage C in 72.5% and stage D in 26%; in group II, stage B was found in 1.5%, stage C in 84.5% and stage D in 14%. Grade III was 36.8% in group I and 17.8% in group II. No statistical differences were found in stage distribution (p=0.36) and grade (p=0.06) between group I and II. Five-year overall survival was 57.8% and 28.5% in younger and older patients, respectively (p=0.053). The results obtained showed no difference in clinical symptomatology and tumor localization in both groups. The incidence of more aggressive tumors was higher in younger persons. However, early detection combined with more aggressive therapeutic approach, could enable significant improvement of the 5-year survival of younger patients with colon cancer. Microsatellite instability diagnostics MSI is a good marker for determining Lynch syndrome and determining a prognosis for cancer treatments. In 1996, the National Cancer Institute (NCI) hosted an international workshop on Lynch Syndrome, which led to the development of the “Bethesda Guidelines” and loci for MSI testing. During this first workshop the NCI has agreed on five microsatellite markers necessary to determine MSI presence: two mononucleotides, BAT25 and BAT26, and three dinucleotide repeats, D2S123, D5S346, and D17S250. MSI-H tumors result from MSI of greater than 30% of unstable MSI loci (>2 or more of the 5 loci). MSI-L tumors result from less than 30% of unstable MSI biomarkers. MSI-L tumors are classified as tumors of alternative etiologies. Several studies demonstrate that MSI-H patients respond best to surgery alone, rather than chemotherapy and surgery, thus preventing patients from needlessly experiencing chemotherapy. There has been an unremitting rise in incidence of colonic cancer in this country with no recent improvement in cure rate. As a result the evolution of colorectal cancer has been the focus of considerable attention with an enlarging body of evidence pointing to the common neoplastic polyp as a precursor to malignancy. \"Neoplastic\" polyps include \"adenomatous polyps,\" \"villous adenomas\" and, lately recognized, \"villo-glandular polyps.\" Experience with endoscopic removal of over 2,000 colonic polyps (with no mortality) has introduced two questions of prime concern to the surgeon: (1) What constitutes clinical malignancy in a polyp? AND, (2) When should laparatomy supplant or follow endoscopic removal? Eight hundred and ninety-two consecutive adenomatous (tubular), villous, villoglandular (villo-tubular) and \"polypoid cancer\" polyps are analyzed, 855 of which have been followed for 6 months to 4 years. Support is offered to the concept that villous and tubular growth patterns are merely variants of a similar base disturbance in cell renewal. Superficial cancer (carcinoma-in-situ) occurred in 6.6% of neoplastic polyps and represents no threat if the polyp is completely removed. Only when the cancer penetrates the muscularis mucosae should it be regarded as \"invasive.\" The term \"malignant polyp\" should be reserved for this form. Invasive cancer was found in 5.0% of neoplastic polyps in this series. Only in this group need the question of further surgical intervention be raised. Major considerations influencing a decision for subsequent laparotomy are polyp size and gross morphology (i.e. sessile or pedunculated), histologic type (of the polyp and of the cancer itself), adequacy of clearance between depth of invasion and plane of polyp resection, and the patient's age and general condition. These are analyzed. Twenty-five of 46 patients with \"malignant polyps\" were subjected to abdominal exploration: 17 showed no residual cancer, whereas 8 (5 with recognized incomplete endoscopic removal) had tumor in the bowel wall. Of the remaining 21 patients, for whom endoscopic polypectomy alone was deemed appropriate, none have shown residual or recurrent cancer on clinical and endoscopic followup. Colonoscopy appears to be a most promising approach in terms of the goals of cancer programs, offering both prophylaxis and opportunity for treatment at a favorable stage of disease. Treatment The specific treatment will depend on the tumor's type, location, size, and whether the cancer has spread to other organs. Surgical removal of the tumor remains the standard treatment of choice, but additional forms of therapy such as radiation therapy, chemotherapy, or immunotherapy exist. When detected early, skin cancer in cats and dogs can often be treated successfully. In many cases, a biopsy can remove the whole tumor, as long as the healthy tissues removed from just outside the tumor area do not contain any cancer cells. References External links Skin Cancer in Cats and Dogs from Pet Cancer Center Skin Cancer in Dogs from CanineCancer.com' Types of animal cancers Cancer in dogs Cancer in cats Integumentary neoplasia The authors examined the five-years postoperative survival rate of fifty patients who suffered from colorectal cancer along with the fact that the large bowel one of the neighbouring organs were resected. The subjects were divided into four groups: the colorectal resection was associated with (1) stomach resection (13 patients); (2) liver metastasectomy (14 patients); (3) small bowel resection (10 patients); (4) the resection of other organs (13 patients). In the first two years of the study they were examined once in every three months, in the next two years once in every six months and then yearly. The following tests were carried out: chest X ray, abdominal sonography, irrigography or colonoscopy and CEA. On condition that the colonoscope reached the caecum and the result was negative, the test was repeated only a year later. The patients were operated on between 1985 and 1997. The statistical analysis was made with the help of the Kaplan-Meier method. During this period fifty-six complex resections were performed. Out of fifty-six patients fifty were followed. Compliance 89%. In group 1, where the average age of patients was sixty-two years, one patient died in the forty-first and the other in the fifty-second month after the surgery. Survival rate: 11/13 (83%). The survival rates for the other groups were as follows: group 2 (average age 64) twelve patients died within five years. Survival rate 2/14 (14%). The difference between the survival rates in the first two groups in significant (P = 0.0001). Group 3 (average age 67) seven died and only three survived. Survival rate: 3/10 (30%). The difference between group 1 and group 3 is significant (P = 0.0022). Group 4 (average age 64) seven patients died. Survival rate 6/13 (46%). Comparing this rate to that of the group 1, the difference is not significant (P > 0.01). Having analysed the results of the four groups it can be concluded that the patients of group 1 lived the longest (stomach resection) and those of group 2 (liver metastasectomy) died the earliest after the operation. It is surprising that the patients of group 3 lived significantly shorter than the ones of group 1 in spite of the fact that they belong by far the greatest number to stage Dukes B (group 1: 12/13 = 92%; group 3: 7/10 = 70%). The authors assume that the partial or the total absence of the stomach keeps back the growth of the tumour (gastro-colic tumour growing dependency). They think that in case of colon cancer which infiltrates the stomach surgeons experienced in gastric and colorectal surgery should be encouraged to take the risk of the double resection providing the fact that the operation is accomplishable. in understanding the dynamics of tumor progression and metastasis. By detecting, quantifying and characterisation of vital circulating tumor cells or genomic alterations in CTCs and cell-free DNA in blood, liquid biopsy can provide real-time information on the stage of tumor progression, treatment effectiveness, and cancer metastasis risk. This technological development could make it possible to diagnose and manage cancer from repeated blood tests rather than from a traditional biopsy. Colo-rectal adenoms occur more frequently in the elderly and should be considered as precancerous. The structural changes of the glandular epithelium are known as dysplasia or atypia and are classified into three grades of severity; the \"severe epithelial dysplasia\" has all the histological characteristics of a malignant tumor which however has not infiltrated the muscularis mucosa and so has not gained access to the lymphatic system. Whenever these structural changes were present the terms focal carcinoma or carcinoma in situ were used. However in 1976 the WHO accepted to change the nomenclature to \"severe epithelial dysplasia\", as Morson had proposed. Their aim was to avoid superfluous radical surgical intervention. Whenever severe dysplasia is present in an adenoma, the necessary therapy is the local excision of the adenoma together with its pedick. An exact complete histological examination is necessary. Between 1976 and 1980 we saw 201 cases of adenoma of the colon or rectum at the Surgical Clinic, University of Düsseldorf. 27 of these cases showed severe epithelial dysplasia. As described in the literature there was a correlation between the size of the adenoma, the histological picture and the risk of malignancy. The reexamination of 105 patients showed that there was a significant percentage of recurrency at the site of excision or new polyps at a different site. Therefore, regular checkups are a must for all those patients in whom polyps of the large bowel have been removed. The diagnosis of LPF depends on its clinical presentation almost exclusively in newborn and young children and, most importantly, its histopathology as determined on biopsied intact tissue or fine-needle aspiration to obtain a sampling of the tumor's cells. Intact tissue samples typical show abundant mature-appearing adipose (i.e. fat) tissue mixed with a minor component of oval-shaped or spindle-shaped fibroblast-like cells some of which have a pseudolipoblast-like morphology. Needle biopsies should show these cells. However, LPF histopathology can vary widely between cases. The cited gene abnormalities in the above section are insufficient to support a diagnosis of LPF although further study of these and discoveries of other gene abnormalities may do so. The histopathology of lipofibromatosis-like neural tumors (LPF-NT) can closely resemble LPF tumors. Unlike LPF tumors, however, LPF-NT tumors have been diagnosed in adults in more than 27% of cases with the remaining cases diagnosed Three hundred and seventy-two patients with colorectal tumours treated by curative resection between January 1982 and January 1992 were reviewed in order to determine the role of colonoscopy and the outcome of patients with multiple tumours. Thirty (8.1%) of them with a mean age of 57 (35-79) years (20 males, 10 females) had synchronous (19 cases) or metachronous (11 cases) lesions. Rectum and sigmoid colon were the most frequent site of multiple lesions, accounting for 73% of all lesions. Accurate pre-operative diagnosis was performed in 14 of the 19 patients with synchronous lesions, and in the remaining 5 cases failure to perform an intra-operative colonoscopy was the cause of missing the lesions. Three of them had over-looked lesions on the previous barium enema. Synchronous lesions has the tendency to be less invasive as compared to metachronous ones. Five-year survival rates (Kaplan-Meier method) were 45% and 58% for patients with multiple and single lesions respectively (not significant). For patients with colorectal carcinoma a thorough examination of the whole colon by intra-operative colonoscopy should be accomplished in order to rule out the possibility of associated lesions as well as to decrease the incidence of \"early\" metachronous lesions. Tumors are formed by carcinogenesis, a process in which cellular alterations lead to the formation of cancer. Multistage carcinogenesis involves the sequential genetic or epigenetic changes to a cell's DNA, where each step produces a more advanced tumor. It is often broken down into three stages; initiation, promotion and progression, and several mutations may occur at each stage. Initiation is where the first genetic mutation occurs in a cell. Promotion is the clonal expansion (repeated division) of this transformed cell into a visible tumor that is usually benign. Following promotion, progression may take place where more genetic mutations are acquired in a sub-population of tumor cells. Progression changes the benign tumor into a malignant tumor. A prominent and well studied example of this phenomenon is the tubular adenoma, a common type of colon polyp which is an important precursor to colon cancer. The cells in tubular adenomas, like most tumors that frequently progress to The emergence of drug-resistant tumors due to intra- and inter-tumoral heterogeneity an issue in treatment efficacy. A minor genetic clone within the tumor can expand after treatment if it carries a drug-resistant mutation. Initial biopsies can miss these clones due to low frequency or spatial separation of cells within the tumor. For example, since a biopsy only samples a small part of the tumor, clones that resides in a different location may go unnoticed. This can mislead research that focuses on studying the role of tumor heterogeneity in cancer progression and relapse. The use of ctDNA in research can alleviate these concerns because it could provide a more representative 'screenshot' of the genetic diversity of cancer at both primary and metastatic sites. For example, ctDNA has been shown to be useful in studying the clonal evolution of a patient’s cancer before and after treatment regimens. Early detection of cancer is still challenging but recent progress in the analysis of Flexible sigmoidoscopy is indicated for colorectal cancer screening. The decision about who needs total colonoscopy based on distal findings is still controversial because of the uncertainty of the associations between distal and proximal findings. The purpose of the study was to characterize distal findings in patients with total colonoscopy, to investigate its importance as markers of advanced proximal lesions and to evaluate the usefulness of a clinical Predictive Index, already published in the literature, in the identification of these lesions. Retrospective analysis of the patients submitted to total colonoscopy between January 2006 and February 2007, with selection of 1000 consecutive cases with reference to polyps. We analysed demographic data, indication for the exam and morphological and histological characteristics of the polyps. Advanced lesion was defined as any adenoma larger than 10 mm or any polyp with villous characteristics, high grade dysplasia or cancer. The Predictive Index was obtained through the assignment of points to 3 categories: sex, age and distal findings, which result in 3 groups: low, intermediate and high risk. The mean age of patients was 64,69 years and 65,1% were male. Distal and proximal polyps were identified in 829 (82,9%) and 369 (36,9%) patients, respectively. Advanced distal lesion was found in 342 patients (34,2%) and advanced proximal lesion in 98 (9,8%). 587 patients (58,7%) were in the high risk group. In the group of patients with advanced proximal lesion, a third presented low and intermediate risk, 52% had no distal polyps, 88,7% had less than three distal polyps and 71,4% had no advanced distal lesion. Sensitivity values for these four categories ranged between 11,2% and 66,6%. If the decision to perform total colonoscopy is based on distal colonic findings or on the Predictive Index, the ability to identify advanced proximal lesion is markedly reduced, endangering the aim of a screening program. For the esophagous Endoscopic mucosal resection has been advocated for early esophageal cancers (that is, those that are superficial and confined to the mucosa only) and has been shown to be a less invasive, safe, and effective therapy for early squamous cell carcinoma. It has also been shown to be safe and effective for early adenocarcinoma arising in Barrett’s esophagus. The prognosis after treatment with this method is comparable to surgical resection. This technique can be attempted in patients who have no evidence of nodal or distant metastases, with differentiated tumors that are slightly raised and less than 2 cm in diameter, or in differentiated tumors that are ulcerated and less than 1 cm in diameter. The most commonly employed modalities of endoscopic mucosal resection include strip biopsy, double-snare polypectomy, resection with combined use of highly concentrated saline and epinephrine, and resection using a cap. In contrast to earlier studies that suggested that colon carcinoma is unusually lethal in the young, 69 patients, ages 20 to 39 years, had a relatively good prognosis. Fifty-nine percent lived over 5 years after diagnosis, and 51% were cured. Furthermore, 67% were cured if they did not have distant spread of the carcinoma at the time of the initial operation. Neither age, sex, tumor size, location, mere presence of lymph node metastases, depth of tumor invasion, nor predisposing disease of the colon was a strong prognostic factor. Metastases to six or more lymph nodes and distant spread of the tumor at the time of initial surgery were ominous findings. Mucinous carcinoma was relatively frequent (28%) and was also an ominous feature (only 5 of 20 patients cured as opposed to 26 of 43 with classical adenocarcinoma).\nHere is the question:\nDIGESTIVE: During a colonoscopy, a 5-cm tumor located in the right colon is detected in a 48-year-old man. No other lesions were found. His maternal grandmother also suffered from colon cancer. The biopsies are superficial and show a poorly differentiated tumor with abundant inflammatory cells in the stroma that is diagnosed as a medullary type carcinoma.\nHere are the potential choices:\n1. Chemotherapy is the treatment of choice.\n2. Since the biopsy is superficial, it should be repeated before proceeding with treatment.\n3. The prognosis of the tumor depends mainly on its high degree of anaplasia.\n4. It is unlikely that this tumor has developed over a previous adenoma.\n5. Microsatellite instability and DNA error repair genes should be studied.\nThe correct answer is: 5. Microsatellite instability and DNA error repair genes should be studied." }, { "question": "You are a helpful medical expert, and your task is to answer a multi-choice medical question using the relevant documents. Please choose the answer from the provided options. Your responses will be used for research purposes only, so please have a definite answer.\nHere are the relevant documents:\n Traditionally, allopurinol is not initiated during an acute gout attack to avoid prolonging the painful arthritis. The 2012 American College of Rheumatology Guidelines for the Management of Gout suggest that urate-lowering therapy can be started during an acute attack, based on \"consensus opinion of experts, case studies, or standard of care.\" The aim of this study was to determine whether initiating allopurinol will adversely affect the resolution of acute, treated gout. We conducted a 28-day, placebo-controlled, double-blind study of allopurinol initiation in patients with acute gout. Patients with crystal-proven gout by arthrocentesis were enrolled if they presented to the rheumatology clinic with an acute gout attack within 72 hours from initial therapy. The patients were also required to meet at least 1 additional criterion for urate-lowering therapy including (1) the presence of gouty tophi, (2) more than 1 acute gout attack per year, (3) a history of nephrolithiasis, or (4) urate overproduction (>1000 mg in 24-hour urine collection). Patients were excluded from the study if they had a glomerular filtration rate of less than 50 or liver function test of greater than 1.25 times the upper limit of normal. The treating physician determined therapy for the acute gout attack. Standard prophylaxis, with colchicine or nonsteroidal anti-inflammatory drugs, was prescribed. Allopurinol or placebo was initiated at 100 mg daily for the first 14 days and then increased to 200 mg daily for the next 14 days. The primary end point was protocol defined days to resolution of acute gout, incorporating patient-rated joint pain and physician examination. Secondary measures included Physician Global Assessment, patient-rated pain, adverse effects of therapy, and serum uric acid. Thirty-one patients (17 on placebo, 14 on allopurinol) completed the study. Both intent-to-treat and completer analyses showed only a statistically insignificant difference in days to resolution (15.4 days in the allopurinol group completers vs 13.4 days in the placebo group; P = 0.5). The secondary measures revealed that the acute phase of pain rapidly improved in both groups. We initiated allopurinol at low doses during an acute gout attack in patients who met criteria for starting urate-lowering therapy and did not have abnormal kidney or liver function. In this cohort, allopurinol did not prolong the acute, treated attack. Difficulties and errors in the treatment of patients with the gout arise, mainly, during urate-lowering therapy. The article discusses possible medical errors in acute gouty arthritis and during chronic tophaceous gout in the light of the updated international recommendations of the American College of Rheumatology (ACR) and the European Antirheumatic League (EULAR 2018). As an example of inadequate treatment, the authors describe a case of a patient with chronic tophaceous gout. Errors in the diagnosis and treatment of the patient caused various complications and unjustified surgical intervention - amputation of the right finger and removal of a large tophus in the left forearm. Based on the analysis of mistakes made in the diagnosis and treatment of gout, the authors propose an algorithm for therapeutic tactics in different periods of the disease. So, for the relief of exacerbation in acute gouty arthritis, it is recommended to take the following drugs at starting doses: colchicine at a dose of 1.8 mg/day (1.2 mg immediately followed by 0.6 mg 1 hour later during 7-10 days or until complete relief of the gout attack), non-steroidal anti-inflammatory drugs (nimesulide up to 200 mg/day) or glucocorticosteroids (prednisolone at a dose of 30 mg/day for 3-5 days with subsequent withdrawal). The first-line urate-lowering drugs for chronic tofaceous gout are xanthine oxidase inhibitors - allopurinol and febuxostat. Allopurinol is prescribed no earlier than 2 weeks after the arthritis attack has stopped at a starting dose of no more than 100 mg/day, the dose is gradually increased to the minimum effective. The starting dose of febuxostat is 40 mg/day. Also, together with allopurinol or febuxostat, it is recommended to take uricosuric drugs (probenecid 500 mg/day or benzbromarone 50-200 mg/day). At the same time, the authors draw attention to the inadmissibility of the combination of allopurinol and febuxostat. In case of gout that does not respond to the main methods of therapy, treatment with pegloticase is recommended. When prescribing urate-lowering therapy, dose titration is necessary, to avoid the development of toxic effects. Streamlining the initiation of allopurinol could result in a cost benefit for a common medical problem and obviate the perception that no treatment is required once acute attacks have resolved. Our objective was to test the hypothesis that there is no difference in patient daily pain or subsequent attacks with early versus delayed initiation of allopurinol for an acute gout attack. A total of 57 men with crystal-proven gout were randomized to allopurinol 300 mg daily or matching placebo for 10 days. All subjects received indomethacin 50 mg 3 times per day for 10 days, a prophylactic dose of colchicine 0.6 mg 2 times per day for 90 days, and open-label allopurinol starting at day 11. Primary outcome measures were pain on visual analogue scale (VAS) for the primary joint on days 1 to 10 and self-reported flares in any joint through day 30. On the basis of 51 evaluable subjects (allopurinol in 26, placebo in 25), mean daily VAS pain scores did not differ significantly between study groups at any point between days 1 and 10. Initial VAS pain scores for allopurinol and placebo arms were 6.72 versus 6.28 (P=.37), declining to 0.18 versus 0.27 (P=.54) at day 10, with neither group consistently having more daily pain. Subsequent flares occurred in 2 subjects taking allopurinol and 3 subjects taking placebo (P=.60). Although urate levels decreased rapidly in the allopurinol group (from 7.8 mg/dL at baseline to 5.9 mg/dL at day 3), sedimentation rates and C-reactive protein levels did not differ between groups at any point. Allopurinol initiation during an acute gout attack caused no significant difference in daily pain, recurrent flares, or inflammatory markers. If Dr. Sydenham could have benefited from today's therapy, he likely would not have had to endure thirty years of \"violent ... torture\" that gave birth to his most elegant and classic description of acute gout. The five key points to remember in management of the gouty spectrum are: (1) Establish the diagnosis as clearly as possible or as clearly as seems necessary under the clinical circumstances (i.e. arthrocentesis with crystal analysis to establish diagnosis is not always necessary with reliable patients when septic joint seems highly unlikely). (2) Treat acute attacks with NSAIDs alone or perhaps steroids--or rarely IV colchicine under special circumstances. (3) DO NOT START ALLOPURINOL OR PROBENECID DURING AN ACUTE FLARE OF GOUT--IT MAY MAKE THE EPISODE WORSE. (4) The pattern of disease over time (frequency and severity of attacks) determines whether or not one decides to use an agent such as allopurinol, probenecid, or prophylactic colchicine chronically once a patient is over the acute attack--the mere presence of increased uric acid and a single or rare gouty attack would not usually require any other than the appropriate acute therapy. (5) The presence of visible tophi, uric acid renal calculi and destructive gouty arthritis nearly always warrant uric acid lowering therapy. For the management of acute gouty arthritis, non-steroidal anti-inflammatory drugs (NSAIDs) are the drugs of choice. In recent years, the use of colchicine has declined because of its frequent adverse reactions, and its reduced efficacy when administered more than 24 hours after onset of an acute attack. Intra-articular corticosteroid therapy (e.g. methylprednisolone acetate) is indicated for the treatment of acute mono or oligoarticular gouty arthritis in aged patients, and in those with co-morbid conditions contraindicating therapy with either NSAIDs or colchicine. Oral corticosteroids (e.g. prednisone), and both parenteral corticotrophin (ACTH) and corticosteroids (e.g. intramuscular triamcinolone acetonide) are valuable, relatively safe alternate treatment modalities in those with polyarticular attacks. For the treatment of hyperuricaemia and chronic gouty arthritis, allopurinol is the preferred urate-lowering drug. Its toxicity in elderly individuals, those with renal impairment, and in cyclosporine-treated transplant patients can be minimised by adjusting the initial dose according to the patient's creatinine clearance. In those experiencing cutaneous reactions to allopurinol, cautious desensitisation to the drug can be achieved using a schedule of gradually increasing doses. The therapeutic usefulness of uricosuric drugs is limited by the presence of renal impairment, occurrence of intolerable side-effects, or concomitant intake of salicylates. They are particularly indicated in patients allergic to allopurinol and in those with massive tophi requiring combined therapy with both allopurinol and a uricosuric. The treatment of gout is based on several principles. Symptom control and termination of the inflammatory process are important early goals, whereas the urate level should be lowered in the long term to prevent further gout attacks and complications. The non-pharmacological approach is based on individually informing the patient on dietary measures and changes of life style. Besides physical measures, such as cold applications on the affected joint, various medications are available for treatment of an acute gout attack. The choice of drug depends on the individual risk profile. If non-steroidal anti-inflammatory drugs (NSAID) and coxibs are chosen it should be taken into account that the use is restricted in patients with renal insufficiency. Moreover, these drugs may have gastrointestinal side effects and are associated with increased cardiovascular morbidity and mortality. Colchicine has gastrointestinal side effects at high dosages but can also be used for differential diagnostics if there is a quick response to treatment. Steroids are an effective alternative and can be given orally or parenterally in patients with dysphagia. Moreover, steroids can be used in cases of renal insufficiency. After symptoms of the acute attack have subsided, urate lowering therapy should be initiated to prevent further attacks. Low-dose urate lowering therapy can be started during an acute gout attack when acute therapy is initiated. Allopurinol is still the medication of choice but its use is restricted in patients with renal insufficiency. A rare but serious side effect is allopurinol hypersensitivity syndrome. Febuxostat can be an alternative in patients who do not tolerate allopurinol. In February 2016, lesinurad, an URAT-1 and OAT-4 inhibitor, was approved in combination with allopurinol or febuxostat. Data on the effectiveness and safety of synthetic uricases and biologicals are still sparse for elderly patients. These substances are reserved for severe cases of gout. Allopurinol, a xanthine oxidase inhibitor, is considered one of the most effective urate-lowering drugs and is frequently used in the treatment of chronic gout. To assess the efficacy and safety of allopurinol compared with placebo and other urate-lowering therapies for treating chronic gout. We searched the Cochrane Central Register of Controlled Trials (CENTRAL), MEDLINE and EMBASE on 14 January 2014. We also handsearched the 2011 to 2012 American College of Rheumatology (ACR) and European League against Rheumatism (EULAR) abstracts, trial registers and regulatory agency drug safety databases. All randomised controlled trials (RCTs) or quasi-randomised controlled clinical trials (CCTs) that compared allopurinol with a placebo or an active therapy in adults with chronic gout. We extracted and analysed data using standard methods for Cochrane reviews. The major outcomes of interest were frequency of acute gout attacks, serum urate normalisation, pain, function, tophus regression, study participant withdrawal due to adverse events (AE) and serious adverse events (SAE). We assessed the quality of the body of evidence for these outcomes using the GRADE approach. We included 11 trials (4531 participants) that compared allopurinol (various doses) with placebo (two trials); febuxostat (four trials); benzbromarone (two trials); colchicine (one trial); probenecid (one trial); continuous versus intermittent allopurinol (one trial) and different doses of allopurinol (one trial). Only one trial was at low risk of bias in all domains. We deemed allopurinol versus placebo the main comparison, and allopurinol versus febuxostat and versus benzbromarone as the most clinically relevant active comparisons and restricted reporting to these comparisons here.Moderate-quality evidence from one trial (57 participants) indicated allopurinol 300 mg daily probably does not reduce the rate of gout attacks (2/26 with allopurinol versus 3/25 with placebo; risk ratio (RR) 0.64, 95% confidence interval (CI) 0.12 to 3.52) but increases the proportion of participants achieving a target serum urate over 30 days (25/26 with allopurinol versus 0/25 with placebo, RR 49.11, 95% CI 3.15 to 765.58; number needed to treat for an additional beneficial outcome (NNTB) 1). In two studies (453 participants), there was no significant increase in withdrawals due to AE (6% with allopurinol versus 4% with placebo, RR 1.36, 95% CI 0.61 to 3.08) or SAE (2% with allopurinol versus 1% with placebo, RR 1.93, 95% CI 0.48 to 7.80). One trial reported no difference in pain reduction or tophus regression, but did not report outcome data or measures of variance sufficiently and we could not calculate the differences between groups. Neither trial reported function.Low-quality evidence from three trials (1136 participants) indicated there may be no difference in the incidence of acute gout attacks with allopurinol up to 300 mg daily versus febuxostat 80 mg daily over eight to 24 weeks (21% with allopurinol versus 23% with febuxostat, RR 0.89, 95% CI 0.71 to 1.1); however more participants may achieve target serum urate level (four trials; 2618 participants) with febuxostat 80 mg daily versus allopurinol 300 mg daily (38% with allopurinol versus 70% with febuxostat, RR 0.56, 95% CI 0.48 to 0.65, NNTB with febuxostat 4). Two trials reported no difference in tophus regression between allopurinol and febuxostat over a 28- to 52-week period; but as the trialists did not provide variance, we could not calculate the mean difference between groups. The trials did not report pain reduction or function. Moderate-quality evidence from pooled data from three trials (2555 participants) comparing allopurinol up to 300 mg daily versus febuxostat 80 mg daily indicated no difference in the number of withdrawals due to AE (7% with allopurinol versus 8% with febuxostat, RR 0.89, 95% CI 0.62 to 1.26) or SAE (4% with allopurinol versus 4% with febuxostat, RR 1.13, 95% CI 0.71 to 1.82) over a 24- to 52-week period.Low-quality evidence from one trial (65 participants) indicated there may be no difference in the incidence of acute gout attacks with allopurinol up to 600 mg daily compared with benzbromarone up to 200 mg daily over a four-month period (0/30 with allopurinol versus 1/25 with benzbromarone, RR 0.28, 95% CI 0.01 to 6.58). Based on the pooled results of two trials (102 participants), there was moderate-quality evidence of no probable difference in the proportion of participants achieving a target serum urate level with allopurinol versus benzbromarone (58% with allopurinol versus 74% with benzbromarone, RR 0.79, 95% CI 0.56 to 1.11). Low-quality evidence from two studies indicated there may be no difference in the number of participants who withdrew due to AE with allopurinol versus benzbromarone over a four- to nine-month period (6% with allopurinol versus 7% with benzbromarone, pooled RR 0.80, 95% CI 0.18 to 3.58). There were no SAEs. They did not report tophi regression, pain and function.All other comparisons were supported by small, single studies only, limiting conclusions. Our review found low- to moderate-quality evidence indicating similar effects on withdrawals due to AEs and SAEs and incidence of acute gout attacks when allopurinol (100 to 600 mg daily) was compared with placebo, benzbromarone (100 to 200 mg daily) or febuxostat (80 mg daily). There was moderate-quality evidence of little or no difference in the proportion of participants achieving target serum urate when allopurinol was compared with benzbromarone. However, allopurinol seemed more successful than placebo and may be less successful than febuxostat (80 mg daily) in achieving a target serum urate level (6 mg/dL or less; 0.36 mmol/L or less) based on moderate- to low-quality evidence. Single studies reported no difference in pain reduction when allopurinol (300 mg daily) was compared with placebo over 10 days, and no difference in tophus regression when allopurinol (200 to 300 mg daily) was compared with febuxostat (80 mg daily). None of the trials reported on function, health-related quality of life or participant global assessment of treatment success, where further research would be useful. This is a case report of a 36-year-old male with tophaceous gout for 16 years. He started therapy with 300 mg/day of allopurinol. He had received variable dexamethasone doses by self-prescription for 16 years. When allopurinol was initiated, he had hyperuricemia and normal renal function. Twenty days after starting allopurinol, he presented diffuse maculopapular rash, conjunctivitis, increase in serum creatinine values, leukocytosis and eosinophilia and the diagnosis of allopurinol hypersensitivity (AH) syndrome was made. He completely recovered from the AH and renal function normalized. However, the gout worsened over the following years in spite of treatment with benzobromarone, low doses of prednisone, and colchicine. Allopurinol desensitization was successful beginning with an oral low dose scheme (6.5 mug/day) until we reached 300mg/day. Today the patient receives allopurinol with no side effects. We believe that this is the first reported example of successful desensitization in full-blown AH with renal involvement. Our cautious regimen might be tried in other such patients. Medical uses Gout Colchicine is an alternative for those unable to tolerate NSAIDs when treating gout. Low doses appear to be well tolerated and may reduce gout symptoms and pain (1.2 mg in one hour, followed by 0.6 mg an hour later). This low dose may have a similar effectiveness to NSAIDS (low quality evidence). At high doses, side effects (primarily diarrhea, nausea, or vomiting) limit its use, however may be effective against pain. In addition, there is preliminary evidence that daily colchicine (0.6 mg twice daily) may be effective as a long-term prophylaxis when used with allopurinol to reduce the risk of increased uric acid levels and acute gout flares, although adverse gastrointestinal effects may occur. For treating gout symptoms, colchicine is used orally with or without food, as symptoms first appear. Subsequent doses may be needed if symptoms worsen. Treatment The initial aim of treatment is to settle the symptoms of an acute attack. Repeated attacks can be prevented by medications that reduce serum uric acid levels. Tentative evidence supports the application of ice for 20 to 30 minutes several times a day to decrease pain. Options for acute treatment include nonsteroidal anti-inflammatory drugs (NSAIDs), colchicine, and Glucocorticoids. While glucocorticoids and NSAIDs work equally well, glucocorticoids may be safer. Options for prevention include allopurinol, febuxostat, and probenecid. Lowering uric acid levels can cure the disease. Treatment of associated health problems is also important. Lifestyle interventions have been poorly studied. It is unclear whether dietary supplements have an effect in people with gout. While it has been recommended that urate-lowering measures should be increased until serum uric acid levels are below 300–360 µmol/l (5.0–6.0 mg/dl), there is little evidence to support this practice over simple putting people on a standard dose of allopurinol. If these medications are in chronic use at the time of an attack, it is recommended that they be continued. Levels that cannot be brought below 6.0 mg/dl while attacks continue indicates refractory gout. While historically it is not recommended to start allopurinol during an acute attack of gout, this practice appears acceptable. Allopurinol blocks uric acid production, and is the most commonly used agent. Long term therapy is safe and well-tolerated and can be used in people with renal impairment or urate stones, although hypersensitivity occurs in a small number of individuals. This study aimed at determining whether lowering serum urate (SU) to less than 6 mg/dl in patients with gout affects ultrasonographic findings. Seven joints in five patients with monosodium urate (MSU) crystal proven gout and hyperuricemia were examined over time with serial ultrasonography. Four of the five patients were treated with urate lowering drugs (ULDs) (allopurinol, n = 3; probenecid, n = 1). One patient was treated with colchicine alone. Attention was given to changes in a hyperechoic, irregular coating of the hyaline cartilage in the examined joints (double contour sign or \"urate icing\"). This coating was considered to represent precipitate of MSU crystals. Index joints included metacarpophalangeal (MCP) joints (n = 2), knee joints (n = 3), and first metatarsophalangeal (MTP) joints (n = 2). The interval between baseline and follow-up images ranged from 7 to 18 months. Serial SU levels were obtained during the follow-up period. During the follow-up period, three patients treated with ULD (allopurinol, n = 2; probenecid, n = 1) achieved a SU level of <6 mg/dl. In two patients, SU levels remained above 6 mg/dl (treated with allopurinol, n = 1; treated with colchicine, n = 1). At baseline, the double contour sign was seen in all patients. In those patients who achieved SU levels of <6 ml/dl, this sign had disappeared at follow-up. Disappearance of the double contour sign was seen in two knee joints, two first MTP joints, and one MCP joint. In contrast, disappearance of the double contour sign was not seen in patients who maintained a SU level > or =7 mg/dl. In one patient treated with allopurinol, SU levels improved from 13 to 7 mg/dl during the follow-up period. Decrease, but not resolution of the hyperechoic coating was seen in this patient. In the patient treated with colchicine alone, SU levels remained >8 mg/dl, and no sonographic change was observed. In our patients, sonographic signs of deposition of MSU crystals on the surface of hyaline cartilage disappeared completely if sustained normouricemia was achieved. This is the first report showing that characteristic sonographic changes are influenced by ULDs once SU levels remain < or =6 mg/dl for 7 months or more. Sonographic changes of gout correlate with SU levels and may be a non-invasive means to track changes in the uric acid pool. Larger prospective studies are needed to further assess these potentially important findings. Lesinurad/allopurinol (trade name Duzallo) is a fixed-dose combination drug for the treatment of gout. It contains 200 mg of lesinurad and 300 mg of allopurinol. In August 2017, the US Food and Drug Administration approved it for the treatment of hyperuricemia associated with gout in patients for whom target serum uric acid levels have not been achieved with allopurinol alone. It was approved for medical use in the European Union in August 2018. In February 2019, it was discontinued by its manufacturer for business reasons and is no longer available. References Antigout agents Combination drugs AstraZeneca brands Withdrawn drugs There are three stages in the management of gout: (i) treating the acute attack; (ii) lowering excess stores of uric acid to prevent flares of gouty arthritis and to prevent tissue deposition of urate; and (iii) providing prophylaxis to prevent acute flares. It is important to distinguish between therapy to reduce acute inflammation in acute gout and therapy to manage hyperuricaemia in patients with chronic gouty arthritis. During the acute gouty attack nonpharmacological treatments such as topical ice and rest of the inflamed joint are useful. NSAIDs are the preferred treatment in acute gout. The most important determinant of therapeutic success is not which NSAID is chosen, but rather how soon NSAID therapy is initiated. Other treatments include oral and intravenous colchicine, intra-articular and systemic corticosteroids, and intramuscular corticotropin. Optimal treatment of chronic gout requires long-standing reduction in serum uric acid. The urate-lowering drugs used to treat chronic gout are the uricosuric drugs, the uricostatic drugs, which are xanthine oxidase inhibitors, and the uricolytic drugs. Xanthine oxidase inhibitors such as allopurinol, oxipurinol and febuxastat should be used as first-line treatment in patients with renal calculi, renal insufficiency, concomitant diuretic therapy and ciclosporin (cyclosporine) therapy, and urate overproduction. Uricosuric drugs include probenecid, benzbromarone, micronised fenofibrate and losartan. They are the urate-lowering drugs of choice in allopurinol-allergic patients and underexcretors with normal renal function and no history of urolithiasis. The use of recombinant urate oxidase in patients with chronic gout is limited by the need for parenteral administration, the potential antigenicity and production of anti-urate oxidase antibodies, and declining efficacy. The effectiveness of colchicine prophylaxis as an isolated therapy is still to be confirmed by placebo-controlled trials. Another issue is prophylaxis with NSAIDs. There are no comparative studies with colchicine. To describe the current pharmacological approach to gout treatment reported by rheumatologists in Brazil. We performed a cross-sectional survey study using an online questionnaire e-mailed to 395 rheumatologists, randomly selected, from among the members of the Brazilian Society of Rheumatology. Three hundred and nine rheumatologists (78.2%) responded to the survey. For acute gout attacks, combination therapy (NSAIDs or steroid + colchicine) was often used, even in monoarticular involvement, and colchicine was commonly started as monotherapy after 36 hours or more from onset of attack. During an acute attack, urate-lowering therapy (ULT) was withdrawn by approximately a third of rheumatologists. Anti-inflammatory prophylaxis (98% colchicine) was initiated when ULT was started in most cases (92.4%), but its duration was varied. Most (70%) respondents considered the target serum uric acid level to be less than 6 mg/dl. Approximately 50% of rheumatologists reported starting allopurinol at doses of 100 mg daily or less and 42% reported the initial dose to be 300 mg daily in patients with normal renal function. ULT was maintained indefinitely in 76% of gout patients with tophi whereas in gout patients without tophi its use was kept indefinitely in 39.6%. This is the first study evaluating gout treatment in a representative, random sample of Brazilian rheumatologists describing common treatment practices among these specialists. We identified several gaps in reported gout management, mainly concerning the use of colchicine and ULT and the duration of anti-inflammatory prophylaxis and ULT. Since rheumatologists are considered as opinion leaders in this disease, a program for improving quality of care for gout patients should focus on increasing their knowledge in this common disease.\nHere is the question:\nRHEUMATOLOGY: A hyperuricemic patient who usually takes 100 mg of allopurinol daily comes to the ED with acute pain and inflammatory signs in the right knee. Arthrocentesis is performed and polarized light microscopy shows intracellular crystals with negative birefringence. Which of the following therapeutic approaches is the most appropriate in this case?\nHere are the potential choices:\n1. Discontinue allopurinol and start colchicine treatment.\n2. Discontinue allopurinol and start NSAIDs.\n3. Add an NSAID until the crisis remits.\n4. Increase the dose of allopurinol to 300 mg/day.\n5. Substitute allopurinol for uricosuric acid.\nThe correct answer is: ", "gold_answer": "3 Add an NSAID until the crisis remits.", "prediction": "You are a helpful medical expert, and your task is to answer a multi-choice medical question using the relevant documents. Please choose the answer from the provided options. Your responses will be used for research purposes only, so please have a definite answer.\nHere are the relevant documents:\n Traditionally, allopurinol is not initiated during an acute gout attack to avoid prolonging the painful arthritis. The 2012 American College of Rheumatology Guidelines for the Management of Gout suggest that urate-lowering therapy can be started during an acute attack, based on \"consensus opinion of experts, case studies, or standard of care.\" The aim of this study was to determine whether initiating allopurinol will adversely affect the resolution of acute, treated gout. We conducted a 28-day, placebo-controlled, double-blind study of allopurinol initiation in patients with acute gout. Patients with crystal-proven gout by arthrocentesis were enrolled if they presented to the rheumatology clinic with an acute gout attack within 72 hours from initial therapy. The patients were also required to meet at least 1 additional criterion for urate-lowering therapy including (1) the presence of gouty tophi, (2) more than 1 acute gout attack per year, (3) a history of nephrolithiasis, or (4) urate overproduction (>1000 mg in 24-hour urine collection). Patients were excluded from the study if they had a glomerular filtration rate of less than 50 or liver function test of greater than 1.25 times the upper limit of normal. The treating physician determined therapy for the acute gout attack. Standard prophylaxis, with colchicine or nonsteroidal anti-inflammatory drugs, was prescribed. Allopurinol or placebo was initiated at 100 mg daily for the first 14 days and then increased to 200 mg daily for the next 14 days. The primary end point was protocol defined days to resolution of acute gout, incorporating patient-rated joint pain and physician examination. Secondary measures included Physician Global Assessment, patient-rated pain, adverse effects of therapy, and serum uric acid. Thirty-one patients (17 on placebo, 14 on allopurinol) completed the study. Both intent-to-treat and completer analyses showed only a statistically insignificant difference in days to resolution (15.4 days in the allopurinol group completers vs 13.4 days in the placebo group; P = 0.5). The secondary measures revealed that the acute phase of pain rapidly improved in both groups. We initiated allopurinol at low doses during an acute gout attack in patients who met criteria for starting urate-lowering therapy and did not have abnormal kidney or liver function. In this cohort, allopurinol did not prolong the acute, treated attack. Difficulties and errors in the treatment of patients with the gout arise, mainly, during urate-lowering therapy. The article discusses possible medical errors in acute gouty arthritis and during chronic tophaceous gout in the light of the updated international recommendations of the American College of Rheumatology (ACR) and the European Antirheumatic League (EULAR 2018). As an example of inadequate treatment, the authors describe a case of a patient with chronic tophaceous gout. Errors in the diagnosis and treatment of the patient caused various complications and unjustified surgical intervention - amputation of the right finger and removal of a large tophus in the left forearm. Based on the analysis of mistakes made in the diagnosis and treatment of gout, the authors propose an algorithm for therapeutic tactics in different periods of the disease. So, for the relief of exacerbation in acute gouty arthritis, it is recommended to take the following drugs at starting doses: colchicine at a dose of 1.8 mg/day (1.2 mg immediately followed by 0.6 mg 1 hour later during 7-10 days or until complete relief of the gout attack), non-steroidal anti-inflammatory drugs (nimesulide up to 200 mg/day) or glucocorticosteroids (prednisolone at a dose of 30 mg/day for 3-5 days with subsequent withdrawal). The first-line urate-lowering drugs for chronic tofaceous gout are xanthine oxidase inhibitors - allopurinol and febuxostat. Allopurinol is prescribed no earlier than 2 weeks after the arthritis attack has stopped at a starting dose of no more than 100 mg/day, the dose is gradually increased to the minimum effective. The starting dose of febuxostat is 40 mg/day. Also, together with allopurinol or febuxostat, it is recommended to take uricosuric drugs (probenecid 500 mg/day or benzbromarone 50-200 mg/day). At the same time, the authors draw attention to the inadmissibility of the combination of allopurinol and febuxostat. In case of gout that does not respond to the main methods of therapy, treatment with pegloticase is recommended. When prescribing urate-lowering therapy, dose titration is necessary, to avoid the development of toxic effects. Streamlining the initiation of allopurinol could result in a cost benefit for a common medical problem and obviate the perception that no treatment is required once acute attacks have resolved. Our objective was to test the hypothesis that there is no difference in patient daily pain or subsequent attacks with early versus delayed initiation of allopurinol for an acute gout attack. A total of 57 men with crystal-proven gout were randomized to allopurinol 300 mg daily or matching placebo for 10 days. All subjects received indomethacin 50 mg 3 times per day for 10 days, a prophylactic dose of colchicine 0.6 mg 2 times per day for 90 days, and open-label allopurinol starting at day 11. Primary outcome measures were pain on visual analogue scale (VAS) for the primary joint on days 1 to 10 and self-reported flares in any joint through day 30. On the basis of 51 evaluable subjects (allopurinol in 26, placebo in 25), mean daily VAS pain scores did not differ significantly between study groups at any point between days 1 and 10. Initial VAS pain scores for allopurinol and placebo arms were 6.72 versus 6.28 (P=.37), declining to 0.18 versus 0.27 (P=.54) at day 10, with neither group consistently having more daily pain. Subsequent flares occurred in 2 subjects taking allopurinol and 3 subjects taking placebo (P=.60). Although urate levels decreased rapidly in the allopurinol group (from 7.8 mg/dL at baseline to 5.9 mg/dL at day 3), sedimentation rates and C-reactive protein levels did not differ between groups at any point. Allopurinol initiation during an acute gout attack caused no significant difference in daily pain, recurrent flares, or inflammatory markers. If Dr. Sydenham could have benefited from today's therapy, he likely would not have had to endure thirty years of \"violent ... torture\" that gave birth to his most elegant and classic description of acute gout. The five key points to remember in management of the gouty spectrum are: (1) Establish the diagnosis as clearly as possible or as clearly as seems necessary under the clinical circumstances (i.e. arthrocentesis with crystal analysis to establish diagnosis is not always necessary with reliable patients when septic joint seems highly unlikely). (2) Treat acute attacks with NSAIDs alone or perhaps steroids--or rarely IV colchicine under special circumstances. (3) DO NOT START ALLOPURINOL OR PROBENECID DURING AN ACUTE FLARE OF GOUT--IT MAY MAKE THE EPISODE WORSE. (4) The pattern of disease over time (frequency and severity of attacks) determines whether or not one decides to use an agent such as allopurinol, probenecid, or prophylactic colchicine chronically once a patient is over the acute attack--the mere presence of increased uric acid and a single or rare gouty attack would not usually require any other than the appropriate acute therapy. (5) The presence of visible tophi, uric acid renal calculi and destructive gouty arthritis nearly always warrant uric acid lowering therapy. For the management of acute gouty arthritis, non-steroidal anti-inflammatory drugs (NSAIDs) are the drugs of choice. In recent years, the use of colchicine has declined because of its frequent adverse reactions, and its reduced efficacy when administered more than 24 hours after onset of an acute attack. Intra-articular corticosteroid therapy (e.g. methylprednisolone acetate) is indicated for the treatment of acute mono or oligoarticular gouty arthritis in aged patients, and in those with co-morbid conditions contraindicating therapy with either NSAIDs or colchicine. Oral corticosteroids (e.g. prednisone), and both parenteral corticotrophin (ACTH) and corticosteroids (e.g. intramuscular triamcinolone acetonide) are valuable, relatively safe alternate treatment modalities in those with polyarticular attacks. For the treatment of hyperuricaemia and chronic gouty arthritis, allopurinol is the preferred urate-lowering drug. Its toxicity in elderly individuals, those with renal impairment, and in cyclosporine-treated transplant patients can be minimised by adjusting the initial dose according to the patient's creatinine clearance. In those experiencing cutaneous reactions to allopurinol, cautious desensitisation to the drug can be achieved using a schedule of gradually increasing doses. The therapeutic usefulness of uricosuric drugs is limited by the presence of renal impairment, occurrence of intolerable side-effects, or concomitant intake of salicylates. They are particularly indicated in patients allergic to allopurinol and in those with massive tophi requiring combined therapy with both allopurinol and a uricosuric. The treatment of gout is based on several principles. Symptom control and termination of the inflammatory process are important early goals, whereas the urate level should be lowered in the long term to prevent further gout attacks and complications. The non-pharmacological approach is based on individually informing the patient on dietary measures and changes of life style. Besides physical measures, such as cold applications on the affected joint, various medications are available for treatment of an acute gout attack. The choice of drug depends on the individual risk profile. If non-steroidal anti-inflammatory drugs (NSAID) and coxibs are chosen it should be taken into account that the use is restricted in patients with renal insufficiency. Moreover, these drugs may have gastrointestinal side effects and are associated with increased cardiovascular morbidity and mortality. Colchicine has gastrointestinal side effects at high dosages but can also be used for differential diagnostics if there is a quick response to treatment. Steroids are an effective alternative and can be given orally or parenterally in patients with dysphagia. Moreover, steroids can be used in cases of renal insufficiency. After symptoms of the acute attack have subsided, urate lowering therapy should be initiated to prevent further attacks. Low-dose urate lowering therapy can be started during an acute gout attack when acute therapy is initiated. Allopurinol is still the medication of choice but its use is restricted in patients with renal insufficiency. A rare but serious side effect is allopurinol hypersensitivity syndrome. Febuxostat can be an alternative in patients who do not tolerate allopurinol. In February 2016, lesinurad, an URAT-1 and OAT-4 inhibitor, was approved in combination with allopurinol or febuxostat. Data on the effectiveness and safety of synthetic uricases and biologicals are still sparse for elderly patients. These substances are reserved for severe cases of gout. Allopurinol, a xanthine oxidase inhibitor, is considered one of the most effective urate-lowering drugs and is frequently used in the treatment of chronic gout. To assess the efficacy and safety of allopurinol compared with placebo and other urate-lowering therapies for treating chronic gout. We searched the Cochrane Central Register of Controlled Trials (CENTRAL), MEDLINE and EMBASE on 14 January 2014. We also handsearched the 2011 to 2012 American College of Rheumatology (ACR) and European League against Rheumatism (EULAR) abstracts, trial registers and regulatory agency drug safety databases. All randomised controlled trials (RCTs) or quasi-randomised controlled clinical trials (CCTs) that compared allopurinol with a placebo or an active therapy in adults with chronic gout. We extracted and analysed data using standard methods for Cochrane reviews. The major outcomes of interest were frequency of acute gout attacks, serum urate normalisation, pain, function, tophus regression, study participant withdrawal due to adverse events (AE) and serious adverse events (SAE). We assessed the quality of the body of evidence for these outcomes using the GRADE approach. We included 11 trials (4531 participants) that compared allopurinol (various doses) with placebo (two trials); febuxostat (four trials); benzbromarone (two trials); colchicine (one trial); probenecid (one trial); continuous versus intermittent allopurinol (one trial) and different doses of allopurinol (one trial). Only one trial was at low risk of bias in all domains. We deemed allopurinol versus placebo the main comparison, and allopurinol versus febuxostat and versus benzbromarone as the most clinically relevant active comparisons and restricted reporting to these comparisons here.Moderate-quality evidence from one trial (57 participants) indicated allopurinol 300 mg daily probably does not reduce the rate of gout attacks (2/26 with allopurinol versus 3/25 with placebo; risk ratio (RR) 0.64, 95% confidence interval (CI) 0.12 to 3.52) but increases the proportion of participants achieving a target serum urate over 30 days (25/26 with allopurinol versus 0/25 with placebo, RR 49.11, 95% CI 3.15 to 765.58; number needed to treat for an additional beneficial outcome (NNTB) 1). In two studies (453 participants), there was no significant increase in withdrawals due to AE (6% with allopurinol versus 4% with placebo, RR 1.36, 95% CI 0.61 to 3.08) or SAE (2% with allopurinol versus 1% with placebo, RR 1.93, 95% CI 0.48 to 7.80). One trial reported no difference in pain reduction or tophus regression, but did not report outcome data or measures of variance sufficiently and we could not calculate the differences between groups. Neither trial reported function.Low-quality evidence from three trials (1136 participants) indicated there may be no difference in the incidence of acute gout attacks with allopurinol up to 300 mg daily versus febuxostat 80 mg daily over eight to 24 weeks (21% with allopurinol versus 23% with febuxostat, RR 0.89, 95% CI 0.71 to 1.1); however more participants may achieve target serum urate level (four trials; 2618 participants) with febuxostat 80 mg daily versus allopurinol 300 mg daily (38% with allopurinol versus 70% with febuxostat, RR 0.56, 95% CI 0.48 to 0.65, NNTB with febuxostat 4). Two trials reported no difference in tophus regression between allopurinol and febuxostat over a 28- to 52-week period; but as the trialists did not provide variance, we could not calculate the mean difference between groups. The trials did not report pain reduction or function. Moderate-quality evidence from pooled data from three trials (2555 participants) comparing allopurinol up to 300 mg daily versus febuxostat 80 mg daily indicated no difference in the number of withdrawals due to AE (7% with allopurinol versus 8% with febuxostat, RR 0.89, 95% CI 0.62 to 1.26) or SAE (4% with allopurinol versus 4% with febuxostat, RR 1.13, 95% CI 0.71 to 1.82) over a 24- to 52-week period.Low-quality evidence from one trial (65 participants) indicated there may be no difference in the incidence of acute gout attacks with allopurinol up to 600 mg daily compared with benzbromarone up to 200 mg daily over a four-month period (0/30 with allopurinol versus 1/25 with benzbromarone, RR 0.28, 95% CI 0.01 to 6.58). Based on the pooled results of two trials (102 participants), there was moderate-quality evidence of no probable difference in the proportion of participants achieving a target serum urate level with allopurinol versus benzbromarone (58% with allopurinol versus 74% with benzbromarone, RR 0.79, 95% CI 0.56 to 1.11). Low-quality evidence from two studies indicated there may be no difference in the number of participants who withdrew due to AE with allopurinol versus benzbromarone over a four- to nine-month period (6% with allopurinol versus 7% with benzbromarone, pooled RR 0.80, 95% CI 0.18 to 3.58). There were no SAEs. They did not report tophi regression, pain and function.All other comparisons were supported by small, single studies only, limiting conclusions. Our review found low- to moderate-quality evidence indicating similar effects on withdrawals due to AEs and SAEs and incidence of acute gout attacks when allopurinol (100 to 600 mg daily) was compared with placebo, benzbromarone (100 to 200 mg daily) or febuxostat (80 mg daily). There was moderate-quality evidence of little or no difference in the proportion of participants achieving target serum urate when allopurinol was compared with benzbromarone. However, allopurinol seemed more successful than placebo and may be less successful than febuxostat (80 mg daily) in achieving a target serum urate level (6 mg/dL or less; 0.36 mmol/L or less) based on moderate- to low-quality evidence. Single studies reported no difference in pain reduction when allopurinol (300 mg daily) was compared with placebo over 10 days, and no difference in tophus regression when allopurinol (200 to 300 mg daily) was compared with febuxostat (80 mg daily). None of the trials reported on function, health-related quality of life or participant global assessment of treatment success, where further research would be useful. This is a case report of a 36-year-old male with tophaceous gout for 16 years. He started therapy with 300 mg/day of allopurinol. He had received variable dexamethasone doses by self-prescription for 16 years. When allopurinol was initiated, he had hyperuricemia and normal renal function. Twenty days after starting allopurinol, he presented diffuse maculopapular rash, conjunctivitis, increase in serum creatinine values, leukocytosis and eosinophilia and the diagnosis of allopurinol hypersensitivity (AH) syndrome was made. He completely recovered from the AH and renal function normalized. However, the gout worsened over the following years in spite of treatment with benzobromarone, low doses of prednisone, and colchicine. Allopurinol desensitization was successful beginning with an oral low dose scheme (6.5 mug/day) until we reached 300mg/day. Today the patient receives allopurinol with no side effects. We believe that this is the first reported example of successful desensitization in full-blown AH with renal involvement. Our cautious regimen might be tried in other such patients. Medical uses Gout Colchicine is an alternative for those unable to tolerate NSAIDs when treating gout. Low doses appear to be well tolerated and may reduce gout symptoms and pain (1.2 mg in one hour, followed by 0.6 mg an hour later). This low dose may have a similar effectiveness to NSAIDS (low quality evidence). At high doses, side effects (primarily diarrhea, nausea, or vomiting) limit its use, however may be effective against pain. In addition, there is preliminary evidence that daily colchicine (0.6 mg twice daily) may be effective as a long-term prophylaxis when used with allopurinol to reduce the risk of increased uric acid levels and acute gout flares, although adverse gastrointestinal effects may occur. For treating gout symptoms, colchicine is used orally with or without food, as symptoms first appear. Subsequent doses may be needed if symptoms worsen. Treatment The initial aim of treatment is to settle the symptoms of an acute attack. Repeated attacks can be prevented by medications that reduce serum uric acid levels. Tentative evidence supports the application of ice for 20 to 30 minutes several times a day to decrease pain. Options for acute treatment include nonsteroidal anti-inflammatory drugs (NSAIDs), colchicine, and Glucocorticoids. While glucocorticoids and NSAIDs work equally well, glucocorticoids may be safer. Options for prevention include allopurinol, febuxostat, and probenecid. Lowering uric acid levels can cure the disease. Treatment of associated health problems is also important. Lifestyle interventions have been poorly studied. It is unclear whether dietary supplements have an effect in people with gout. While it has been recommended that urate-lowering measures should be increased until serum uric acid levels are below 300–360 µmol/l (5.0–6.0 mg/dl), there is little evidence to support this practice over simple putting people on a standard dose of allopurinol. If these medications are in chronic use at the time of an attack, it is recommended that they be continued. Levels that cannot be brought below 6.0 mg/dl while attacks continue indicates refractory gout. While historically it is not recommended to start allopurinol during an acute attack of gout, this practice appears acceptable. Allopurinol blocks uric acid production, and is the most commonly used agent. Long term therapy is safe and well-tolerated and can be used in people with renal impairment or urate stones, although hypersensitivity occurs in a small number of individuals. This study aimed at determining whether lowering serum urate (SU) to less than 6 mg/dl in patients with gout affects ultrasonographic findings. Seven joints in five patients with monosodium urate (MSU) crystal proven gout and hyperuricemia were examined over time with serial ultrasonography. Four of the five patients were treated with urate lowering drugs (ULDs) (allopurinol, n = 3; probenecid, n = 1). One patient was treated with colchicine alone. Attention was given to changes in a hyperechoic, irregular coating of the hyaline cartilage in the examined joints (double contour sign or \"urate icing\"). This coating was considered to represent precipitate of MSU crystals. Index joints included metacarpophalangeal (MCP) joints (n = 2), knee joints (n = 3), and first metatarsophalangeal (MTP) joints (n = 2). The interval between baseline and follow-up images ranged from 7 to 18 months. Serial SU levels were obtained during the follow-up period. During the follow-up period, three patients treated with ULD (allopurinol, n = 2; probenecid, n = 1) achieved a SU level of <6 mg/dl. In two patients, SU levels remained above 6 mg/dl (treated with allopurinol, n = 1; treated with colchicine, n = 1). At baseline, the double contour sign was seen in all patients. In those patients who achieved SU levels of <6 ml/dl, this sign had disappeared at follow-up. Disappearance of the double contour sign was seen in two knee joints, two first MTP joints, and one MCP joint. In contrast, disappearance of the double contour sign was not seen in patients who maintained a SU level > or =7 mg/dl. In one patient treated with allopurinol, SU levels improved from 13 to 7 mg/dl during the follow-up period. Decrease, but not resolution of the hyperechoic coating was seen in this patient. In the patient treated with colchicine alone, SU levels remained >8 mg/dl, and no sonographic change was observed. In our patients, sonographic signs of deposition of MSU crystals on the surface of hyaline cartilage disappeared completely if sustained normouricemia was achieved. This is the first report showing that characteristic sonographic changes are influenced by ULDs once SU levels remain < or =6 mg/dl for 7 months or more. Sonographic changes of gout correlate with SU levels and may be a non-invasive means to track changes in the uric acid pool. Larger prospective studies are needed to further assess these potentially important findings. Lesinurad/allopurinol (trade name Duzallo) is a fixed-dose combination drug for the treatment of gout. It contains 200 mg of lesinurad and 300 mg of allopurinol. In August 2017, the US Food and Drug Administration approved it for the treatment of hyperuricemia associated with gout in patients for whom target serum uric acid levels have not been achieved with allopurinol alone. It was approved for medical use in the European Union in August 2018. In February 2019, it was discontinued by its manufacturer for business reasons and is no longer available. References Antigout agents Combination drugs AstraZeneca brands Withdrawn drugs There are three stages in the management of gout: (i) treating the acute attack; (ii) lowering excess stores of uric acid to prevent flares of gouty arthritis and to prevent tissue deposition of urate; and (iii) providing prophylaxis to prevent acute flares. It is important to distinguish between therapy to reduce acute inflammation in acute gout and therapy to manage hyperuricaemia in patients with chronic gouty arthritis. During the acute gouty attack nonpharmacological treatments such as topical ice and rest of the inflamed joint are useful. NSAIDs are the preferred treatment in acute gout. The most important determinant of therapeutic success is not which NSAID is chosen, but rather how soon NSAID therapy is initiated. Other treatments include oral and intravenous colchicine, intra-articular and systemic corticosteroids, and intramuscular corticotropin. Optimal treatment of chronic gout requires long-standing reduction in serum uric acid. The urate-lowering drugs used to treat chronic gout are the uricosuric drugs, the uricostatic drugs, which are xanthine oxidase inhibitors, and the uricolytic drugs. Xanthine oxidase inhibitors such as allopurinol, oxipurinol and febuxastat should be used as first-line treatment in patients with renal calculi, renal insufficiency, concomitant diuretic therapy and ciclosporin (cyclosporine) therapy, and urate overproduction. Uricosuric drugs include probenecid, benzbromarone, micronised fenofibrate and losartan. They are the urate-lowering drugs of choice in allopurinol-allergic patients and underexcretors with normal renal function and no history of urolithiasis. The use of recombinant urate oxidase in patients with chronic gout is limited by the need for parenteral administration, the potential antigenicity and production of anti-urate oxidase antibodies, and declining efficacy. The effectiveness of colchicine prophylaxis as an isolated therapy is still to be confirmed by placebo-controlled trials. Another issue is prophylaxis with NSAIDs. There are no comparative studies with colchicine. To describe the current pharmacological approach to gout treatment reported by rheumatologists in Brazil. We performed a cross-sectional survey study using an online questionnaire e-mailed to 395 rheumatologists, randomly selected, from among the members of the Brazilian Society of Rheumatology. Three hundred and nine rheumatologists (78.2%) responded to the survey. For acute gout attacks, combination therapy (NSAIDs or steroid + colchicine) was often used, even in monoarticular involvement, and colchicine was commonly started as monotherapy after 36 hours or more from onset of attack. During an acute attack, urate-lowering therapy (ULT) was withdrawn by approximately a third of rheumatologists. Anti-inflammatory prophylaxis (98% colchicine) was initiated when ULT was started in most cases (92.4%), but its duration was varied. Most (70%) respondents considered the target serum uric acid level to be less than 6 mg/dl. Approximately 50% of rheumatologists reported starting allopurinol at doses of 100 mg daily or less and 42% reported the initial dose to be 300 mg daily in patients with normal renal function. ULT was maintained indefinitely in 76% of gout patients with tophi whereas in gout patients without tophi its use was kept indefinitely in 39.6%. This is the first study evaluating gout treatment in a representative, random sample of Brazilian rheumatologists describing common treatment practices among these specialists. We identified several gaps in reported gout management, mainly concerning the use of colchicine and ULT and the duration of anti-inflammatory prophylaxis and ULT. Since rheumatologists are considered as opinion leaders in this disease, a program for improving quality of care for gout patients should focus on increasing their knowledge in this common disease.\nHere is the question:\nRHEUMATOLOGY: A hyperuricemic patient who usually takes 100 mg of allopurinol daily comes to the ED with acute pain and inflammatory signs in the right knee. Arthrocentesis is performed and polarized light microscopy shows intracellular crystals with negative birefringence. Which of the following therapeutic approaches is the most appropriate in this case?\nHere are the potential choices:\n1. Discontinue allopurinol and start colchicine treatment.\n2. Discontinue allopurinol and start NSAIDs.\n3. Add an NSAID until the crisis remits.\n4. Increase the dose of allopurinol to 300 mg/day.\n5. Substitute allopurinol for uricosuric acid.\nThe correct answer is: 1. Discontinue allopurinol and start colchicine treatment." }, { "question": "You are a helpful medical expert, and your task is to answer a multi-choice medical question using the relevant documents. Please choose the answer from the provided options. Your responses will be used for research purposes only, so please have a definite answer.\nHere are the relevant documents:\n The ELISA antibody tests were developed to provide a high level of confidence that donated blood was not infected with HIV. It is therefore not possible to conclude that blood rejected for transfusion because of a positive ELISA antibody test is in fact infected with HIV. Sometimes, retesting the donor in several months will produce a negative ELISA antibody test. This is why a confirmatory western blot is always used before reporting a \"positive\" HIV test result. If no antibodies to HIV are detected, this does not mean the person has not been infected with HIV. It may take several months after HIV infection for the antibody response to reach detectable levels, during which time rapid testing for antibodies to HIV will not be indicative of true infection status. For most people, HIV antibodies reach a detectable level after two to six weeks. Although these tests have high specificity, false positives do occur. Any positive test result should be confirmed by a lab using the western blot. Interpreting antibody tests ELISA testing alone cannot be used to diagnose HIV, even if the test suggests a high probability that antibody to HIV-1 is present. In the United States, such ELISA results are not reported as \"positive\" unless confirmed by a western blot. HIV antibody testing is unreliable Diagnosis of infection using antibody testing is a well-established technique in medicine. HIV antibody tests exceed the performance of most other infectious disease tests in both sensitivity (the ability of the screening test to give a positive finding when the person tested truly has the disease) and specificity (the ability of the test to give a negative finding when the subjects tested are free of the disease under study). Many current HIV antibody tests have sensitivity and specificity in excess of 96% and are therefore extremely reliable. While most patients with HIV show an antibody response after six weeks, window periods vary and may occasionally be as long as three months. The specificity rate given here for the inexpensive enzyme immunoassay screening tests indicates that, in 1,000 HIV test results of healthy individuals, about 15 of these results will be a false positive. Confirming the test result (i.e., by repeating the test, if this option is available) could reduce the ultimate likelihood of a false positive to about 1 result in 250,000 tests given. The sensitivity rating, likewise, indicates that, in 1,000 test results of HIV infected people, 3 will actually be a false negative result. However, based upon the HIV prevalence rates at most testing centers within the United States, the negative predictive value of these tests is extremely high, meaning that a negative test result will be correct more than 9,997 times in 10,000 (99.97% of the time). The very high negative predictive value of these tests is why the CDC recommends that a negative test result be considered conclusive evidence that an individual does not have HIV. Antibody tests may give false negative (no antibodies were detected despite the presence of HIV) results during the window period, an interval of three weeks to six months between the time of HIV infection and the production of measurable antibodies to HIV seroconversion. Most people develop detectable antibodies approximately 30 days after infection, although some seroconvert later. The vast majority of people (97%) have detectable antibodies by three months after HIV infection; a six-month window is extremely rare with modern antibody testing. During the window period, an infected person can transmit HIV to others although their HIV infection may not be detectable with an antibody test. Antiretroviral therapy during the window period can delay the formation of antibodies and extend the window period beyond 12 months. This was not the case with patients that underwent treatment with post-exposure prophylaxis (PEP). Those patients must take ELISA tests at various intervals after the According the WHO there are about 14,000 new HIV infections a day. However, in a few cases the diagnosis will be made in the acute phase of the disease. Acute HIV infection is the period between infection with the virus and complete seroconversion, defined by a positive Western blot test. This period lasts approximately 30 days and most patients (40-90%) have mild clinical manifestations (fever, rash, pharyngitis, mucosal ulcers, among others) for 2 weeks which, because they are nonspecific, can be confused with other community-acquired infections. Microbiological diagnosis is based on the absence of serum antibodies (negative ELISA test) together with a positive HIV viral load in plasma (> 10,000 copies/ml). Diagnosis of acute HIV infection is important for several reasons: firstly, from the epidemiological point of view, this is the period with the highest rates of HIV transmission and identification of new HIV infections reveals the growth of the epidemic and the transmission rates of resistant HIV strains, which in Spain is about 10%; secondly, from the immunopathological point of view, this period provides a unique opportunity to study the virological, immunological and genetic mechanisms that play a role in the transmission and pathogenesis of this disease; and thirdly, therapeutically, starting antiretroviral therapy during this phase could alter the natural history of the disease. However, this is a controversial issue and currently most guidelines recommend treatment only if these patients can be included in clinical trials or if they show lasting or severe clinical manifestations. Three instances of delayed HIV seroconversion occurring in health-care workers have been reported; in these instances, the health-care workers tested negative for HIV antibodies greater than 6 months postexposure but were seropositive within 12 months after the exposure. DNA sequencing confirmed the source of infection in one instance. Two of the delayed seroconversions were associated with simultaneous exposure to hepatitis C virus (HCV). In one case, co-infection was associated with a rapidly fatal HCV disease course; however, it is not known whether HCV directly influences the risk for or course of HIV infection or is a marker for other exposure-related factors. ELISA The enzyme-linked immunosorbent assay (ELISA), or enzyme immunoassay (EIA), was the first screening test commonly employed for HIV. It has a high sensitivity. Viral load tests can also be used to diagnose HIV infection, especially in children under 18 months born to mothers with HIV, where the presence of maternal antibodies prevents the use of antibody-based (ELISA) diagnostic tests. Pooled viral RNA testing shortens the window period to a median of 17 days (95% CI, 13-28 Days). Although it is not the standard of care to use this test for diagnosis, in communities with high HIV prevalence, this test has a significantly improved negative predictive value over 3rd and 4th generation tests for detecting acute HIV infections. HIV-1 testing is initially done using an enzyme-linked immunosorbent assay (ELISA) to detect antibodies to HIV-1. Specimens with a non-reactive result from the initial ELISA are considered HIV-negative, unless new exposure to an infected partner or partner of unknown HIV status has occurred. Specimens with a reactive ELISA result are retested in duplicate. If the result of either duplicate test is reactive, the specimen is reported as repeatedly reactive and undergoes confirmatory testing with a more specific supplemental test (e.g., a polymerase chain reaction (PCR), western blot or, less commonly, an immunofluorescence assay (IFA)). Only specimens that are repeatedly reactive by ELISA and positive by IFA or PCR or reactive by western blot are considered HIV-positive and indicative of HIV infection. Specimens that are repeatedly ELISA-reactive occasionally provide an indeterminate western blot result, which may be either an incomplete antibody response to HIV in an infected person or Diagnosis HIV-2 diagnosis can be made when a patient has no symptoms but positive blood work indicating the individual has HIV. The Multispot HIV-1/HIV-2 Rapid Test is currently the only FDA approved method for such differentiation between the two viruses. Recommendations for the screening and diagnosis of HIV has always been to use enzyme immunoassays that detect HIV-1, HIV-1 group O, and HIV-2. When screening the combination, if the test is positive followed by an indeterminate HIV-1 western blot, a follow up test, such as amino acid testing, must be performed to distinguish which infection is present. According to the NIH, a differential diagnosis of HIV-2 should be considered when a person is of West African descent or has had sexual contact or shared needles with such a person. West Africa is at the highest risk as it is the origin of the virus. The HIV set point is the viral load or number of virions in the blood of a person infected with HIV. HIV infections are broken down into three stages: acute infection, asymptomatic infection, and AIDS. The acute infection stage refers to the first weeks after infection, where the majority of infected individuals display severe flu-like symptoms such as fever, myalgia, sore throat, swollen lymph nodes, arthralgia, fatigue, headache, and sometimes rash. At this stage, viral loads reach high levels and the number of CD4 helper T cells in the blood begins to drop. At this point, seroconversion, the development of antibodies, occurs and the CD4 T cell counts begin to recover as the immune system attempts to fight the virus, marking the HIV set point. The higher the viral load at the set point, the faster the virus will progress to AIDS; the lower the viral load at the set point, the longer the patient will remain in clinical latency, the next stage of the infection. To diagnose an individual with HIV, a test must be taken to determine if the virus is present in their system. There are several test options including ELISA, at-home, saliva, viral load, and western blot. To establish the presence of the HIV virus, some tests measure the level of HIV antibodies in the blood and/or saliva or the level of both HIV antigens and antibodies in the blood. Other tests can detect the presence of the HIV virus by calculating the amount of actual virus present in the blood. None of the tests available can determine if a person is positive immediately after they believe they have been exposed to the virus. Each test has a window of time after the initial exposure occurred until the test can accurately tell if an individual has been infected or not. One reason for this is because the focus of some of the tests are antibodies. After the initial exposure to the virus, it usually takes 3–4 weeks but it can take up to six months for antibodies to be produced by the The most updated HIV testing protocols recommend using the HIV-1 and HIV-2 antigen/antibody combination immunoassay as the initial screening test for an HIV infection. This blood test assesses whether or not the mother has created antibodies, which are disease-fighting proteins of the immune system, against the HIV-1 and HIV-2 viruses. These antibodies will only be present if the patient has been exposed to HIV, therefore, they act as a marker of infection. This test also detects a protein called p24 in maternal blood, which is a specific component of the HIV virus itself and also acts as an early marker of an HIV infection. If this test is positive, the CDC recommends performing follow-up testing using a test called the HIV-1/HIV-2 antibody differentiation immunoassay that both confirms the diagnosis and determines the specific type of HIV infection the patient has to specifically tailor further management of the patient. HIV antibody tests are highly sensitive, meaning they react preferentially with HIV antibodies, but not all positive or inconclusive HIV ELISA tests mean the person is infected by HIV. Risk history, and clinical judgement should be included in the assessment, and a confirmation test (western blot) should be administered. An individual with an inconclusive test should be re-tested at a later date. HIV tests are used to detect the presence of the human immunodeficiency virus (HIV), the virus that causes acquired immunodeficiency syndrome (AIDS), in serum, saliva, or urine. Such tests may detect antibodies, antigens, or RNA. AIDS diagnosis AIDS is diagnosed separately from HIV. Terminology The window period is the time from infection until a test can detect any change. The average window period with HIV-1 antibody tests is 25 days for subtype B. Antigen testing cuts the window period to approximately 16 days and nucleic acid testing (NAT) further reduces this period to 12 days. Performance of medical tests is often described in terms of: Sensitivity: The percentage of the results that will be positive when HIV is present Specificity: The percentage of the results that will be negative when HIV is not present. Diagnosis of HIV infection Tests used for the diagnosis of HIV infection in a particular person require a high degree of both sensitivity and specificity. In the United States, this is achieved using an algorithm combining two tests for HIV antibodies. If antibodies are detected by an initial test based on the ELISA method, then a second test using the western blot procedure determines the size of the antigens in the test kit binding to the antibodies. The combination of these two methods is highly accurate Human rights The UNAIDS/WHO policy statement on HIV Testing states that conditions under which people undergo HIV testing must be anchored in a human rights approach that pays due respect to ethical principles. According to these principles, the conduct of HIV testing of individuals must be Confidential; Accompanied by counseling (for those who test positive); Conducted with the informed consent of the person being tested. Confidentiality Serology is the primary means for identifying patients with HIV infection and Acquired Immunodeficiency Syndrome (AIDS). Testing of serum by serologic methods has been extensively used since 1985, not only for clinical diagnosis but also for epidemiological surveillance and donor screening in blood banks. Fast serological diagnostic techniques are now being developed, using urine and oral fluid, as an alternative for anti-HIV antibody screening, and many parallel studies are proving its accuracy. The purpose of this study was to evaluate the sensitivity, specificity, accuracy, positive predictive value (PPV) and negative predictive value (NPV) of the ImmunoComb II HIV 1&2 Saliva((R)) test from Orgenics (Dot-ELISA) compared to the routine exams (ELISA and Western Blot) of HIV positive/AIDS patients, undergoing antiretroviral treatment or not, in different stages of the disease's evolution, and compared to serologic testing of known HIV negative patients by the use of serum ELISA (blood donors). To accomplish this, patients of the Immunogenic Deficiencies Control Center (CCDI) and voluntary blood donors of the Blood Bank Center of the Medical School of S&atilde:o Paulo/Federal University of São Paulo (EPM-UN I FESP) were evaluated. Sensitivity of Dot-ELISA in oral fluid was 100%, specificity 97.08%, PPV 96.66% and NPV 100%. The method used in this case study was shown to be highly sensitive and specific, being useful particularly in epidemiological surveillance and screening. In the non-clinical setting, oral swab tests are still primarily enzyme-linked immunosorbent assay (ELISA) antibody tests performed as rapid tests but need to be confirmed with a serum western blot. An advantage of rapid tests is that they can be offered in a non-clinical setting such as community health fairs, places of worship, HIV service centers, and other locations outside healthcare facilities. Results are presented within 20 minutes, which decreases the number of patients who do not know the outcome of their test because they do not follow up for a return appointment, as can occur in the clinical setting. ELISA tests are Ab-only tests and detect HIV as early as 3 weeks after transmission. [5] In 1985, early tests using the ELISA method looked for antibodies, which are the immune system's response to the virus. However, there is a window period when using this method in which a person who has been infected with HIV is able to spread the disease but may test negative for the virus. This window period can be as long as three to six months, with an average of 22 days. Tests using the ELISA methods are often still used in developed countries because of their ease-of-use, as well as their fairly high sensitivity, which boasts 100% sensitivity. To cover the window period resultant from the use of these tests, donors are also screened for high risk behaviors, one of which is a history of same-sex sexual activity among male potential donors. Newer tests look for the virus itself, such as the p24 antigen test, which looks for a part of the virus on the surface of infected cells, and Nucleic acid tests (NAT), which look for the genetic material of the virus in HIV-infected cells. Diagnosis HIV/AIDS is diagnosed via laboratory testing and then staged based on the presence of certain signs or symptoms. HIV screening is recommended by the United States Preventive Services Task Force for all people 15 years to 65 years of age, including all pregnant women. Additionally, testing is recommended for those at high risk, which includes anyone diagnosed with a sexually transmitted illness. In many areas of the world, a third of HIV carriers only discover they are infected at an advanced stage of the disease when AIDS or severe immunodeficiency has become apparent. HIV testing Most people infected with HIV develop specific antibodies (i.e. seroconvert) within three to twelve weeks after the initial infection. Diagnosis of primary HIV before seroconversion is done by measuring HIV-RNA or p24 antigen. Positive results obtained by antibody or PCR testing are confirmed either by a different antibody or by PCR. On June 15, 2010, the FDA approved the first diagnostic test capable of detecting HIV antigens and HIV antibodies. The Abbott ARCHITECT HIV Ag/Ab combo test, a fourth-generation test, has an increased sensitivity for detecting infections during the acute phase (when compared to 1st and 3rd generation tests), when the immune system is still developing antibodies and the virus is replicating unchecked, and in one study, was able to detect 83% of such infections. Society and culture A person, who may be unaware of the infection, is highly infectious during this time yet may test negative for HIV using tests that detect anti-HIV antibodies only. Although Nucleic Acid Amplification Testing NAAT is more expensive and can take a week for processing, some have argued that it may still be a preferred way to screen for HIV. There are no universal criteria for interpreting the western blot test: The number of viral bands that must be present may vary. If no viral bands are detected, the result is negative. If at least one viral band for each of the GAG, POL, and ENV gene-product groups are present, the result is positive. The three-gene-product approach to western blot interpretation has not been adopted for public health or clinical practice. Tests in which less than the required number of viral bands are detected are reported as indeterminate: a person who has an indeterminate result should be retested, as later tests may be more conclusive. Almost all HIV-infected persons with indeterminate western blot results will develop a positive result when tested in one month; persistently indeterminate results over a period of six months suggests the results are not due to HIV infection. In a generally healthy low-risk population, indeterminate results on western blot occur on the order of 1 in 5,000 patients. Since the advent of HIV testing of donor blood in the mid/later 1980s, ex. 1985's ELISA, the transmission of HIV during transfusion has dropped dramatically. Prior testing of donor blood only included testing for antibodies to HIV. However, because of latent infection (the \"window period\" in which an individual is infectious, but has not had time to develop antibodies) many cases of HIV seropositive blood were missed. The development of a nucleic acid test for the HIV-1 RNA has dramatically lowered the rate of donor blood seropositivity to about 1 in 3 million units. As transmittance of HIV does not necessarily mean HIV infection, the latter could still occur at an even lower rate. The transmission of hepatitis C via transfusion currently stands at a rate of about 1 in 2 million units. As with HIV, this low rate has been attributed to the ability to screen for both antibodies as well as viral RNA nucleic acid testing in donor blood. To evaluate a WHO testing strategy based on the use of two consecutive enzyme-linked immunosorbent assays (ELISA) as an alternative to ELISA followed by Western blotting (WB) for the serologic diagnosis of HIV infection. The study was of 2069 consecutive serum specimens from patients suspected of HIV infection received for HIV diagnostic testing at the HIV laboratory, Muhimbili Medical Centre, Dar es Salaam. The strategy involved testing all sera with Behring indirect anti-HIV 1 + 2 peptide ELISA, followed by Wellcozyme anti-HIV-1 recombinant competitive ELISA on those sera reactive by the first ELISA. WB was done on a sample of the sera reactive on both ELISAs and on all those giving discordant results on the two ELISAs. Of the 2069 sera tested, 837 (40.5%) were negative on the first ELISA, 1172 (56.6%) were reactive on both ELISAs and 60 (2.9%) were initially reactive on the first test but not on the second assay. Of the 1172 sera reactive on both ELISAs, 329 (28.1%) were tested by WB. The diagnostic accuracy of the WHO alternative testing strategy using WB confirmation as the 'gold' standard was as follows: sensitivity 99.4% (326/328), specificity 99.7%, (893/896), positive predictive value 99.1% (328/331) and negative predictive value 99.8% (893/895). Repeated testing by ELISA of the sera which initially gave discordant results on the two ELISAs increased the sensitivity to 100%. Three sera giving false positive reactions on both ELISAs became negative on both ELISAs after retesting. In order to achieve a specificity and a positive predictive value of 100%, it would have been necessary to subject all sera reacting on both ELISAs to retesting on one ELISA. A second ELISA based on different antigens and a different test principle compared with the first ELISA could be used as an alternative to the WB assay for confirmation of HIV antibodies. However, some modifications of the WHO strategy for diagnostic HIV antibody testing were required in order to maximize the diagnostic accuracy. Contagiousness The higher the viral load value, the more viral elements there are in blood and other body fluids. For example, individuals with HIV are most contagious during the earliest (acute) stages of the infection, sometimes with millions of copies of HIV per centiliter of blood. According to one estimate, the majority of transmissions among gay men in the UK occur during primary infection. This is because, at this phase, the immune response is still developing. Antibody levels against the virus during acute infection are often too low to be detected, meaning that an antibody test for a highly infectious individual can come back negative. The latest recommendations of the US Centers for Disease Control and Prevention (CDC) show that HIV testing must start with an immunoassay combination test for HIV-1 and HIV-2 antibodies and p24 antigen. A negative result rules out HIV exposure, while a positive one must be followed by an HIV-1/2 antibody differentiation immunoassay to detect which antibodies are present. This gives rise to four possible scenarios: 1. HIV-1 (+) & HIV-2 (−): HIV-1 antibodies detected 2. HIV-1 (−) & HIV-2 (+): HIV-2 antibodies detected 3. HIV-1 (+) & HIV-2 (+): both HIV-1 and HIV-2 antibodies detected 4. HIV-1 (−) or indeterminate & HIV-2 (−): Nucleic acid test must be carried out to detect the acute infection of HIV-1 or its absence. Research HIV/AIDS research includes all medical research that attempts to prevent, treat, or cure HIV/AIDS, as well as fundamental research about the nature of HIV as an infectious agent and AIDS as the disease caused by HIV. antibodies to HIV, an series of assay, HIV-1 RNA assay, or HIV-1 DNA PCR and specific serologic point-of-care tests can provide results in 1–60 min. Among the most testing for HIV-2. If the p24 and HIV RNA assays are negative and popular of these is the OraQuick Rapid HIV-1 antibody test that can be run on blood, plasma, or saliva. The sensitivity and specificity of this test is ~99% when run on whole blood. Specificity remains the SEROLOGIC TESTS IN THE DIAGNOSIS OF HIV-1 OR same but sensitivity drops to 98% when the test is run on saliva. While The standard blood screening tests for HIV infection are based on the detection of antibodies to HIV. A common platform is the ELISA, also referred to as an enzyme immunoassay (EIA). This solid-phase assay is an extremely good screening test with a sensitivity of >99.5%. Most diagnostic laboratories use commercial kits that contain antigens from both HIV-1 and HIV-2 and thus are able to detect antibodies to either. These kits use both natural and recombinant antigens and are continuously updated to increase their sensitivity to newly discovered species, such as group O viruses (Fig. 226-1). The fourth-generation EIA tests combine detection of antibodies to HIV with detection of the p24 antigen of HIV. EIA tests are generally scored as positive (highly reactive), negative (nonreactive), or indeterminate (partially reactive). While the EIA is an extremely sensitive test, it is not optimal with regard to specificity. This is particularly true in studies of low-risk individuals, such as The aim of the study was to detect HIV RNA in seropositive patients using RT-PCR method and thus, to establish PCR methodology in the routine laboratory works. The total of 33 examined persons were divided in two groups: 1) 13 persons seropositive for HIV; and 2) 20 healthy persons - randomly selected blood donors that made the case control group. The subjects age was between 25 and 52 years (average 38,5). ELFA test for combined detection of HIV p24 antigen and anti HIV-1+2 IgG and ELISA test for detection of antibodies against HIV-1 and HIV-2, were performed for each examined person. RNA from the whole blood was extracted using a commercial kit based on salt precipitation. Detection of HIV RNA was performed using RT-PCR kit. Following nested PCR, the product was separated by electrophoresis in 1,5 % agarose gel. The result was scored positive if the band of 210bp was visible regardless of intensity. Measures of precaution were taken during all the steps of the work and HIV infected materials were disposed of accordingly. In the group of blood donors ELFA, ELISA and RT-PCR were negative. Assuming that prevalence of HIV infection is zero, the clinical specificity of RT-PCR is 100 %. The analytical specificity of RT-PCR method was tested against Hepatitis C and B, Human Papiloma Virus, Cytomegalovirus, Herpes Simplex Virus, Rubella Virus, Mycobacterium tuberculosis, Chlamydia trachomatis. None of these templates yielded amplicon. In the group of 13 seropositive persons, 33 samples were analyzed. HIV RNA was detected in 15 samples. ELISA and ELFA test were positive in all samples. Different aliquots of the samples were tested independently and showed the same results. After different periods of storing the RNA samples at -70 masculineC, RT-PCR reaction was identical to the one performed initially. The obtained amplicons were maintained frozen at -20 masculineC for a week and the subsequently performed electrophoresis was identical to the previous one. The reaction is fast, simple for manipulation; with low detection level of 60 IU/ml. RT-PCR needs a small amount of RNA, as well as a small volume of sample. HIV RNA was detected in different periods of time with different clinical presentations in patients, with or without antiretroviral therapy. RT-PCR method gives the opportunity for reliable determination of HIV-1 RNA with border of detection of 60 IU/ml. The test is reproducible and has high analytical and clinical specificity. a period of six months suggests the results are not due to HIV infection. In a generally healthy low-risk population, indeterminate results on western blot occur on the order of 1 in 5,000 patients. However, for those individuals who have had high-risk exposures to individuals where HIV-2 is most prevalent, Western Africa, an inconclusive western blot test may prove infection with HIV-2. AIDS denialism HIV tests have been criticized by AIDS denialists (a fringe group whose members believe that HIV either does not exist or is harmless). The accuracy of serologic testing has been verified by isolation and culture of HIV and by detection of HIV RNA by PCR, which are widely accepted \"gold standards\" in microbiology. While AIDS denialists focus on individual components of HIV testing, the combination of ELISA and western blot used for the diagnosis of HIV is remarkably accurate, with very low false-positive and -negative rates as described above. The views of AIDS denialists are based on highly selective analysis of mostly outdated scientific papers; there is broad scientific consensus that HIV is the cause of AIDS. The study aims to evaluate the HIV-1/2 rapid diagnostic test kit is routinely used to screen HIV infection for safe blood transfusion and VCT services in many parts of Ethiopia. A total of 324 sera were collected from consecutive blood donors from February to May 2006. All samples were screened for HIV infection using Determine HIV-1/2 (Abbott Japan) at hospital blood bank laboratory. Blindly, all serums were retested at Regional Health Research Laboratory using 4th generation ELISA (Vironostika HIV Uni-Form II AG/Ab) and Determine HIV-1/2 (Abbott lab). Discordant samples were repeatedly retested using the same ELISA and Determine HIV-1/2 to avoid technical errors. Finally, discordant results were resolved using Western Blot at the National HIV/AIDS Laboratory. Determine HIV-1/2 and ELISA showed 94.4% concordance in HIV antibody testing with fair Cohen's Kappa statistic value (0.68) among blood donors. The sensitivity, specificity, positive and negative predictive values of Determine HIV-1/2 were 60.5%, 98.9%, 88.5% and 94.9% respectively. As a rapid HIV screening test for blood donors, Determine HIV-1/2 showed poor sensitivity. Further evaluation at multiple centres is recommended to test its validity as a routine HIV screening test in blood donors. Use of a combination of rapid assays is also recommended for screening of HIV infection among the donor population.\nHere is the question:\nINFECTIOUS DISEASES: A 20-year-old girl comes to the clinic with an acute fever, cervical lymphadenopathy and skin rash. According to the patient, 3 weeks ago she had a sexual relationship that could be a risk for contracting the HIV virus. Indicate which of the following answers is true:\nHere are the potential choices:\n1. A negative HIV-1/HIV-2 serology performed by ELISA technique rules out the possibility that the patient has been infected with the HIV virus.\n2. The clinical process that the patient suffers from is not consistent with acute HIV infection.\n3. If the patient's ELISA test was positive, nothing further would be necessary for the diagnosis of HIV infection.\n4. The ELISA technique has a high sensitivity for the diagnosis of HIV infection, but its specificity is even higher.\n5. If the ELISA test to diagnose HIV in the patient was negative, we could determine by PCR technique the viral load in blood.\nThe correct answer is: ", "gold_answer": "5 If the ELISA test to diagnose HIV in the patient was negative, we could determine by PCR technique the viral load in blood.", "prediction": "You are a helpful medical expert, and your task is to answer a multi-choice medical question using the relevant documents. Please choose the answer from the provided options. Your responses will be used for research purposes only, so please have a definite answer.\nHere are the relevant documents:\n The ELISA antibody tests were developed to provide a high level of confidence that donated blood was not infected with HIV. It is therefore not possible to conclude that blood rejected for transfusion because of a positive ELISA antibody test is in fact infected with HIV. Sometimes, retesting the donor in several months will produce a negative ELISA antibody test. This is why a confirmatory western blot is always used before reporting a \"positive\" HIV test result. If no antibodies to HIV are detected, this does not mean the person has not been infected with HIV. It may take several months after HIV infection for the antibody response to reach detectable levels, during which time rapid testing for antibodies to HIV will not be indicative of true infection status. For most people, HIV antibodies reach a detectable level after two to six weeks. Although these tests have high specificity, false positives do occur. Any positive test result should be confirmed by a lab using the western blot. Interpreting antibody tests ELISA testing alone cannot be used to diagnose HIV, even if the test suggests a high probability that antibody to HIV-1 is present. In the United States, such ELISA results are not reported as \"positive\" unless confirmed by a western blot. HIV antibody testing is unreliable Diagnosis of infection using antibody testing is a well-established technique in medicine. HIV antibody tests exceed the performance of most other infectious disease tests in both sensitivity (the ability of the screening test to give a positive finding when the person tested truly has the disease) and specificity (the ability of the test to give a negative finding when the subjects tested are free of the disease under study). Many current HIV antibody tests have sensitivity and specificity in excess of 96% and are therefore extremely reliable. While most patients with HIV show an antibody response after six weeks, window periods vary and may occasionally be as long as three months. The specificity rate given here for the inexpensive enzyme immunoassay screening tests indicates that, in 1,000 HIV test results of healthy individuals, about 15 of these results will be a false positive. Confirming the test result (i.e., by repeating the test, if this option is available) could reduce the ultimate likelihood of a false positive to about 1 result in 250,000 tests given. The sensitivity rating, likewise, indicates that, in 1,000 test results of HIV infected people, 3 will actually be a false negative result. However, based upon the HIV prevalence rates at most testing centers within the United States, the negative predictive value of these tests is extremely high, meaning that a negative test result will be correct more than 9,997 times in 10,000 (99.97% of the time). The very high negative predictive value of these tests is why the CDC recommends that a negative test result be considered conclusive evidence that an individual does not have HIV. Antibody tests may give false negative (no antibodies were detected despite the presence of HIV) results during the window period, an interval of three weeks to six months between the time of HIV infection and the production of measurable antibodies to HIV seroconversion. Most people develop detectable antibodies approximately 30 days after infection, although some seroconvert later. The vast majority of people (97%) have detectable antibodies by three months after HIV infection; a six-month window is extremely rare with modern antibody testing. During the window period, an infected person can transmit HIV to others although their HIV infection may not be detectable with an antibody test. Antiretroviral therapy during the window period can delay the formation of antibodies and extend the window period beyond 12 months. This was not the case with patients that underwent treatment with post-exposure prophylaxis (PEP). Those patients must take ELISA tests at various intervals after the According the WHO there are about 14,000 new HIV infections a day. However, in a few cases the diagnosis will be made in the acute phase of the disease. Acute HIV infection is the period between infection with the virus and complete seroconversion, defined by a positive Western blot test. This period lasts approximately 30 days and most patients (40-90%) have mild clinical manifestations (fever, rash, pharyngitis, mucosal ulcers, among others) for 2 weeks which, because they are nonspecific, can be confused with other community-acquired infections. Microbiological diagnosis is based on the absence of serum antibodies (negative ELISA test) together with a positive HIV viral load in plasma (> 10,000 copies/ml). Diagnosis of acute HIV infection is important for several reasons: firstly, from the epidemiological point of view, this is the period with the highest rates of HIV transmission and identification of new HIV infections reveals the growth of the epidemic and the transmission rates of resistant HIV strains, which in Spain is about 10%; secondly, from the immunopathological point of view, this period provides a unique opportunity to study the virological, immunological and genetic mechanisms that play a role in the transmission and pathogenesis of this disease; and thirdly, therapeutically, starting antiretroviral therapy during this phase could alter the natural history of the disease. However, this is a controversial issue and currently most guidelines recommend treatment only if these patients can be included in clinical trials or if they show lasting or severe clinical manifestations. Three instances of delayed HIV seroconversion occurring in health-care workers have been reported; in these instances, the health-care workers tested negative for HIV antibodies greater than 6 months postexposure but were seropositive within 12 months after the exposure. DNA sequencing confirmed the source of infection in one instance. Two of the delayed seroconversions were associated with simultaneous exposure to hepatitis C virus (HCV). In one case, co-infection was associated with a rapidly fatal HCV disease course; however, it is not known whether HCV directly influences the risk for or course of HIV infection or is a marker for other exposure-related factors. ELISA The enzyme-linked immunosorbent assay (ELISA), or enzyme immunoassay (EIA), was the first screening test commonly employed for HIV. It has a high sensitivity. Viral load tests can also be used to diagnose HIV infection, especially in children under 18 months born to mothers with HIV, where the presence of maternal antibodies prevents the use of antibody-based (ELISA) diagnostic tests. Pooled viral RNA testing shortens the window period to a median of 17 days (95% CI, 13-28 Days). Although it is not the standard of care to use this test for diagnosis, in communities with high HIV prevalence, this test has a significantly improved negative predictive value over 3rd and 4th generation tests for detecting acute HIV infections. HIV-1 testing is initially done using an enzyme-linked immunosorbent assay (ELISA) to detect antibodies to HIV-1. Specimens with a non-reactive result from the initial ELISA are considered HIV-negative, unless new exposure to an infected partner or partner of unknown HIV status has occurred. Specimens with a reactive ELISA result are retested in duplicate. If the result of either duplicate test is reactive, the specimen is reported as repeatedly reactive and undergoes confirmatory testing with a more specific supplemental test (e.g., a polymerase chain reaction (PCR), western blot or, less commonly, an immunofluorescence assay (IFA)). Only specimens that are repeatedly reactive by ELISA and positive by IFA or PCR or reactive by western blot are considered HIV-positive and indicative of HIV infection. Specimens that are repeatedly ELISA-reactive occasionally provide an indeterminate western blot result, which may be either an incomplete antibody response to HIV in an infected person or Diagnosis HIV-2 diagnosis can be made when a patient has no symptoms but positive blood work indicating the individual has HIV. The Multispot HIV-1/HIV-2 Rapid Test is currently the only FDA approved method for such differentiation between the two viruses. Recommendations for the screening and diagnosis of HIV has always been to use enzyme immunoassays that detect HIV-1, HIV-1 group O, and HIV-2. When screening the combination, if the test is positive followed by an indeterminate HIV-1 western blot, a follow up test, such as amino acid testing, must be performed to distinguish which infection is present. According to the NIH, a differential diagnosis of HIV-2 should be considered when a person is of West African descent or has had sexual contact or shared needles with such a person. West Africa is at the highest risk as it is the origin of the virus. The HIV set point is the viral load or number of virions in the blood of a person infected with HIV. HIV infections are broken down into three stages: acute infection, asymptomatic infection, and AIDS. The acute infection stage refers to the first weeks after infection, where the majority of infected individuals display severe flu-like symptoms such as fever, myalgia, sore throat, swollen lymph nodes, arthralgia, fatigue, headache, and sometimes rash. At this stage, viral loads reach high levels and the number of CD4 helper T cells in the blood begins to drop. At this point, seroconversion, the development of antibodies, occurs and the CD4 T cell counts begin to recover as the immune system attempts to fight the virus, marking the HIV set point. The higher the viral load at the set point, the faster the virus will progress to AIDS; the lower the viral load at the set point, the longer the patient will remain in clinical latency, the next stage of the infection. To diagnose an individual with HIV, a test must be taken to determine if the virus is present in their system. There are several test options including ELISA, at-home, saliva, viral load, and western blot. To establish the presence of the HIV virus, some tests measure the level of HIV antibodies in the blood and/or saliva or the level of both HIV antigens and antibodies in the blood. Other tests can detect the presence of the HIV virus by calculating the amount of actual virus present in the blood. None of the tests available can determine if a person is positive immediately after they believe they have been exposed to the virus. Each test has a window of time after the initial exposure occurred until the test can accurately tell if an individual has been infected or not. One reason for this is because the focus of some of the tests are antibodies. After the initial exposure to the virus, it usually takes 3–4 weeks but it can take up to six months for antibodies to be produced by the The most updated HIV testing protocols recommend using the HIV-1 and HIV-2 antigen/antibody combination immunoassay as the initial screening test for an HIV infection. This blood test assesses whether or not the mother has created antibodies, which are disease-fighting proteins of the immune system, against the HIV-1 and HIV-2 viruses. These antibodies will only be present if the patient has been exposed to HIV, therefore, they act as a marker of infection. This test also detects a protein called p24 in maternal blood, which is a specific component of the HIV virus itself and also acts as an early marker of an HIV infection. If this test is positive, the CDC recommends performing follow-up testing using a test called the HIV-1/HIV-2 antibody differentiation immunoassay that both confirms the diagnosis and determines the specific type of HIV infection the patient has to specifically tailor further management of the patient. HIV antibody tests are highly sensitive, meaning they react preferentially with HIV antibodies, but not all positive or inconclusive HIV ELISA tests mean the person is infected by HIV. Risk history, and clinical judgement should be included in the assessment, and a confirmation test (western blot) should be administered. An individual with an inconclusive test should be re-tested at a later date. HIV tests are used to detect the presence of the human immunodeficiency virus (HIV), the virus that causes acquired immunodeficiency syndrome (AIDS), in serum, saliva, or urine. Such tests may detect antibodies, antigens, or RNA. AIDS diagnosis AIDS is diagnosed separately from HIV. Terminology The window period is the time from infection until a test can detect any change. The average window period with HIV-1 antibody tests is 25 days for subtype B. Antigen testing cuts the window period to approximately 16 days and nucleic acid testing (NAT) further reduces this period to 12 days. Performance of medical tests is often described in terms of: Sensitivity: The percentage of the results that will be positive when HIV is present Specificity: The percentage of the results that will be negative when HIV is not present. Diagnosis of HIV infection Tests used for the diagnosis of HIV infection in a particular person require a high degree of both sensitivity and specificity. In the United States, this is achieved using an algorithm combining two tests for HIV antibodies. If antibodies are detected by an initial test based on the ELISA method, then a second test using the western blot procedure determines the size of the antigens in the test kit binding to the antibodies. The combination of these two methods is highly accurate Human rights The UNAIDS/WHO policy statement on HIV Testing states that conditions under which people undergo HIV testing must be anchored in a human rights approach that pays due respect to ethical principles. According to these principles, the conduct of HIV testing of individuals must be Confidential; Accompanied by counseling (for those who test positive); Conducted with the informed consent of the person being tested. Confidentiality Serology is the primary means for identifying patients with HIV infection and Acquired Immunodeficiency Syndrome (AIDS). Testing of serum by serologic methods has been extensively used since 1985, not only for clinical diagnosis but also for epidemiological surveillance and donor screening in blood banks. Fast serological diagnostic techniques are now being developed, using urine and oral fluid, as an alternative for anti-HIV antibody screening, and many parallel studies are proving its accuracy. The purpose of this study was to evaluate the sensitivity, specificity, accuracy, positive predictive value (PPV) and negative predictive value (NPV) of the ImmunoComb II HIV 1&2 Saliva((R)) test from Orgenics (Dot-ELISA) compared to the routine exams (ELISA and Western Blot) of HIV positive/AIDS patients, undergoing antiretroviral treatment or not, in different stages of the disease's evolution, and compared to serologic testing of known HIV negative patients by the use of serum ELISA (blood donors). To accomplish this, patients of the Immunogenic Deficiencies Control Center (CCDI) and voluntary blood donors of the Blood Bank Center of the Medical School of S&atilde:o Paulo/Federal University of São Paulo (EPM-UN I FESP) were evaluated. Sensitivity of Dot-ELISA in oral fluid was 100%, specificity 97.08%, PPV 96.66% and NPV 100%. The method used in this case study was shown to be highly sensitive and specific, being useful particularly in epidemiological surveillance and screening. In the non-clinical setting, oral swab tests are still primarily enzyme-linked immunosorbent assay (ELISA) antibody tests performed as rapid tests but need to be confirmed with a serum western blot. An advantage of rapid tests is that they can be offered in a non-clinical setting such as community health fairs, places of worship, HIV service centers, and other locations outside healthcare facilities. Results are presented within 20 minutes, which decreases the number of patients who do not know the outcome of their test because they do not follow up for a return appointment, as can occur in the clinical setting. ELISA tests are Ab-only tests and detect HIV as early as 3 weeks after transmission. [5] In 1985, early tests using the ELISA method looked for antibodies, which are the immune system's response to the virus. However, there is a window period when using this method in which a person who has been infected with HIV is able to spread the disease but may test negative for the virus. This window period can be as long as three to six months, with an average of 22 days. Tests using the ELISA methods are often still used in developed countries because of their ease-of-use, as well as their fairly high sensitivity, which boasts 100% sensitivity. To cover the window period resultant from the use of these tests, donors are also screened for high risk behaviors, one of which is a history of same-sex sexual activity among male potential donors. Newer tests look for the virus itself, such as the p24 antigen test, which looks for a part of the virus on the surface of infected cells, and Nucleic acid tests (NAT), which look for the genetic material of the virus in HIV-infected cells. Diagnosis HIV/AIDS is diagnosed via laboratory testing and then staged based on the presence of certain signs or symptoms. HIV screening is recommended by the United States Preventive Services Task Force for all people 15 years to 65 years of age, including all pregnant women. Additionally, testing is recommended for those at high risk, which includes anyone diagnosed with a sexually transmitted illness. In many areas of the world, a third of HIV carriers only discover they are infected at an advanced stage of the disease when AIDS or severe immunodeficiency has become apparent. HIV testing Most people infected with HIV develop specific antibodies (i.e. seroconvert) within three to twelve weeks after the initial infection. Diagnosis of primary HIV before seroconversion is done by measuring HIV-RNA or p24 antigen. Positive results obtained by antibody or PCR testing are confirmed either by a different antibody or by PCR. On June 15, 2010, the FDA approved the first diagnostic test capable of detecting HIV antigens and HIV antibodies. The Abbott ARCHITECT HIV Ag/Ab combo test, a fourth-generation test, has an increased sensitivity for detecting infections during the acute phase (when compared to 1st and 3rd generation tests), when the immune system is still developing antibodies and the virus is replicating unchecked, and in one study, was able to detect 83% of such infections. Society and culture A person, who may be unaware of the infection, is highly infectious during this time yet may test negative for HIV using tests that detect anti-HIV antibodies only. Although Nucleic Acid Amplification Testing NAAT is more expensive and can take a week for processing, some have argued that it may still be a preferred way to screen for HIV. There are no universal criteria for interpreting the western blot test: The number of viral bands that must be present may vary. If no viral bands are detected, the result is negative. If at least one viral band for each of the GAG, POL, and ENV gene-product groups are present, the result is positive. The three-gene-product approach to western blot interpretation has not been adopted for public health or clinical practice. Tests in which less than the required number of viral bands are detected are reported as indeterminate: a person who has an indeterminate result should be retested, as later tests may be more conclusive. Almost all HIV-infected persons with indeterminate western blot results will develop a positive result when tested in one month; persistently indeterminate results over a period of six months suggests the results are not due to HIV infection. In a generally healthy low-risk population, indeterminate results on western blot occur on the order of 1 in 5,000 patients. Since the advent of HIV testing of donor blood in the mid/later 1980s, ex. 1985's ELISA, the transmission of HIV during transfusion has dropped dramatically. Prior testing of donor blood only included testing for antibodies to HIV. However, because of latent infection (the \"window period\" in which an individual is infectious, but has not had time to develop antibodies) many cases of HIV seropositive blood were missed. The development of a nucleic acid test for the HIV-1 RNA has dramatically lowered the rate of donor blood seropositivity to about 1 in 3 million units. As transmittance of HIV does not necessarily mean HIV infection, the latter could still occur at an even lower rate. The transmission of hepatitis C via transfusion currently stands at a rate of about 1 in 2 million units. As with HIV, this low rate has been attributed to the ability to screen for both antibodies as well as viral RNA nucleic acid testing in donor blood. To evaluate a WHO testing strategy based on the use of two consecutive enzyme-linked immunosorbent assays (ELISA) as an alternative to ELISA followed by Western blotting (WB) for the serologic diagnosis of HIV infection. The study was of 2069 consecutive serum specimens from patients suspected of HIV infection received for HIV diagnostic testing at the HIV laboratory, Muhimbili Medical Centre, Dar es Salaam. The strategy involved testing all sera with Behring indirect anti-HIV 1 + 2 peptide ELISA, followed by Wellcozyme anti-HIV-1 recombinant competitive ELISA on those sera reactive by the first ELISA. WB was done on a sample of the sera reactive on both ELISAs and on all those giving discordant results on the two ELISAs. Of the 2069 sera tested, 837 (40.5%) were negative on the first ELISA, 1172 (56.6%) were reactive on both ELISAs and 60 (2.9%) were initially reactive on the first test but not on the second assay. Of the 1172 sera reactive on both ELISAs, 329 (28.1%) were tested by WB. The diagnostic accuracy of the WHO alternative testing strategy using WB confirmation as the 'gold' standard was as follows: sensitivity 99.4% (326/328), specificity 99.7%, (893/896), positive predictive value 99.1% (328/331) and negative predictive value 99.8% (893/895). Repeated testing by ELISA of the sera which initially gave discordant results on the two ELISAs increased the sensitivity to 100%. Three sera giving false positive reactions on both ELISAs became negative on both ELISAs after retesting. In order to achieve a specificity and a positive predictive value of 100%, it would have been necessary to subject all sera reacting on both ELISAs to retesting on one ELISA. A second ELISA based on different antigens and a different test principle compared with the first ELISA could be used as an alternative to the WB assay for confirmation of HIV antibodies. However, some modifications of the WHO strategy for diagnostic HIV antibody testing were required in order to maximize the diagnostic accuracy. Contagiousness The higher the viral load value, the more viral elements there are in blood and other body fluids. For example, individuals with HIV are most contagious during the earliest (acute) stages of the infection, sometimes with millions of copies of HIV per centiliter of blood. According to one estimate, the majority of transmissions among gay men in the UK occur during primary infection. This is because, at this phase, the immune response is still developing. Antibody levels against the virus during acute infection are often too low to be detected, meaning that an antibody test for a highly infectious individual can come back negative. The latest recommendations of the US Centers for Disease Control and Prevention (CDC) show that HIV testing must start with an immunoassay combination test for HIV-1 and HIV-2 antibodies and p24 antigen. A negative result rules out HIV exposure, while a positive one must be followed by an HIV-1/2 antibody differentiation immunoassay to detect which antibodies are present. This gives rise to four possible scenarios: 1. HIV-1 (+) & HIV-2 (−): HIV-1 antibodies detected 2. HIV-1 (−) & HIV-2 (+): HIV-2 antibodies detected 3. HIV-1 (+) & HIV-2 (+): both HIV-1 and HIV-2 antibodies detected 4. HIV-1 (−) or indeterminate & HIV-2 (−): Nucleic acid test must be carried out to detect the acute infection of HIV-1 or its absence. Research HIV/AIDS research includes all medical research that attempts to prevent, treat, or cure HIV/AIDS, as well as fundamental research about the nature of HIV as an infectious agent and AIDS as the disease caused by HIV. antibodies to HIV, an series of assay, HIV-1 RNA assay, or HIV-1 DNA PCR and specific serologic point-of-care tests can provide results in 1–60 min. Among the most testing for HIV-2. If the p24 and HIV RNA assays are negative and popular of these is the OraQuick Rapid HIV-1 antibody test that can be run on blood, plasma, or saliva. The sensitivity and specificity of this test is ~99% when run on whole blood. Specificity remains the SEROLOGIC TESTS IN THE DIAGNOSIS OF HIV-1 OR same but sensitivity drops to 98% when the test is run on saliva. While The standard blood screening tests for HIV infection are based on the detection of antibodies to HIV. A common platform is the ELISA, also referred to as an enzyme immunoassay (EIA). This solid-phase assay is an extremely good screening test with a sensitivity of >99.5%. Most diagnostic laboratories use commercial kits that contain antigens from both HIV-1 and HIV-2 and thus are able to detect antibodies to either. These kits use both natural and recombinant antigens and are continuously updated to increase their sensitivity to newly discovered species, such as group O viruses (Fig. 226-1). The fourth-generation EIA tests combine detection of antibodies to HIV with detection of the p24 antigen of HIV. EIA tests are generally scored as positive (highly reactive), negative (nonreactive), or indeterminate (partially reactive). While the EIA is an extremely sensitive test, it is not optimal with regard to specificity. This is particularly true in studies of low-risk individuals, such as The aim of the study was to detect HIV RNA in seropositive patients using RT-PCR method and thus, to establish PCR methodology in the routine laboratory works. The total of 33 examined persons were divided in two groups: 1) 13 persons seropositive for HIV; and 2) 20 healthy persons - randomly selected blood donors that made the case control group. The subjects age was between 25 and 52 years (average 38,5). ELFA test for combined detection of HIV p24 antigen and anti HIV-1+2 IgG and ELISA test for detection of antibodies against HIV-1 and HIV-2, were performed for each examined person. RNA from the whole blood was extracted using a commercial kit based on salt precipitation. Detection of HIV RNA was performed using RT-PCR kit. Following nested PCR, the product was separated by electrophoresis in 1,5 % agarose gel. The result was scored positive if the band of 210bp was visible regardless of intensity. Measures of precaution were taken during all the steps of the work and HIV infected materials were disposed of accordingly. In the group of blood donors ELFA, ELISA and RT-PCR were negative. Assuming that prevalence of HIV infection is zero, the clinical specificity of RT-PCR is 100 %. The analytical specificity of RT-PCR method was tested against Hepatitis C and B, Human Papiloma Virus, Cytomegalovirus, Herpes Simplex Virus, Rubella Virus, Mycobacterium tuberculosis, Chlamydia trachomatis. None of these templates yielded amplicon. In the group of 13 seropositive persons, 33 samples were analyzed. HIV RNA was detected in 15 samples. ELISA and ELFA test were positive in all samples. Different aliquots of the samples were tested independently and showed the same results. After different periods of storing the RNA samples at -70 masculineC, RT-PCR reaction was identical to the one performed initially. The obtained amplicons were maintained frozen at -20 masculineC for a week and the subsequently performed electrophoresis was identical to the previous one. The reaction is fast, simple for manipulation; with low detection level of 60 IU/ml. RT-PCR needs a small amount of RNA, as well as a small volume of sample. HIV RNA was detected in different periods of time with different clinical presentations in patients, with or without antiretroviral therapy. RT-PCR method gives the opportunity for reliable determination of HIV-1 RNA with border of detection of 60 IU/ml. The test is reproducible and has high analytical and clinical specificity. a period of six months suggests the results are not due to HIV infection. In a generally healthy low-risk population, indeterminate results on western blot occur on the order of 1 in 5,000 patients. However, for those individuals who have had high-risk exposures to individuals where HIV-2 is most prevalent, Western Africa, an inconclusive western blot test may prove infection with HIV-2. AIDS denialism HIV tests have been criticized by AIDS denialists (a fringe group whose members believe that HIV either does not exist or is harmless). The accuracy of serologic testing has been verified by isolation and culture of HIV and by detection of HIV RNA by PCR, which are widely accepted \"gold standards\" in microbiology. While AIDS denialists focus on individual components of HIV testing, the combination of ELISA and western blot used for the diagnosis of HIV is remarkably accurate, with very low false-positive and -negative rates as described above. The views of AIDS denialists are based on highly selective analysis of mostly outdated scientific papers; there is broad scientific consensus that HIV is the cause of AIDS. The study aims to evaluate the HIV-1/2 rapid diagnostic test kit is routinely used to screen HIV infection for safe blood transfusion and VCT services in many parts of Ethiopia. A total of 324 sera were collected from consecutive blood donors from February to May 2006. All samples were screened for HIV infection using Determine HIV-1/2 (Abbott Japan) at hospital blood bank laboratory. Blindly, all serums were retested at Regional Health Research Laboratory using 4th generation ELISA (Vironostika HIV Uni-Form II AG/Ab) and Determine HIV-1/2 (Abbott lab). Discordant samples were repeatedly retested using the same ELISA and Determine HIV-1/2 to avoid technical errors. Finally, discordant results were resolved using Western Blot at the National HIV/AIDS Laboratory. Determine HIV-1/2 and ELISA showed 94.4% concordance in HIV antibody testing with fair Cohen's Kappa statistic value (0.68) among blood donors. The sensitivity, specificity, positive and negative predictive values of Determine HIV-1/2 were 60.5%, 98.9%, 88.5% and 94.9% respectively. As a rapid HIV screening test for blood donors, Determine HIV-1/2 showed poor sensitivity. Further evaluation at multiple centres is recommended to test its validity as a routine HIV screening test in blood donors. Use of a combination of rapid assays is also recommended for screening of HIV infection among the donor population.\nHere is the question:\nINFECTIOUS DISEASES: A 20-year-old girl comes to the clinic with an acute fever, cervical lymphadenopathy and skin rash. According to the patient, 3 weeks ago she had a sexual relationship that could be a risk for contracting the HIV virus. Indicate which of the following answers is true:\nHere are the potential choices:\n1. A negative HIV-1/HIV-2 serology performed by ELISA technique rules out the possibility that the patient has been infected with the HIV virus.\n2. The clinical process that the patient suffers from is not consistent with acute HIV infection.\n3. If the patient's ELISA test was positive, nothing further would be necessary for the diagnosis of HIV infection.\n4. The ELISA technique has a high sensitivity for the diagnosis of HIV infection, but its specificity is even higher.\n5. If the ELISA test to diagnose HIV in the patient was negative, we could determine by PCR technique the viral load in blood.\nThe correct answer is: 2. The clinical process that the patient suffers from is not consistent with acute HIV infection." }, { "question": "You are a helpful medical expert, and your task is to answer a multi-choice medical question using the relevant documents. Please choose the answer from the provided options. Your responses will be used for research purposes only, so please have a definite answer.\nHere are the relevant documents:\n Most authors agree that surgery is the treatment of choice for benign tumours of the parotid gland. However, the best surgical technique and the extent of surgery remain controversial. This study attempts to establish whether the implementation of a partial superficial parotidectomy (PSP) is appropriate for the treatment of benign parotid gland tumours. We selected 63 patients with benign parotid gland surgery, of whom 43 had a pleomorphic adenoma and 20, a Warthin tumour. Of this group of 63 patients, 6 could not be included. We consequently studied 57 patients, 41 of them diagnosed as pleomorphic adenoma and 16, as Warthin tumours. In all of them, a PSP was performed without intraoperative monitoring of the facial nerve. Transient facial nerve paralysis, 14 patients (24.5%). Ten cases were resolved within the first month after surgery and 4 before the third month, after indicating facial physiotherapy. One patient (1.7%) had a permanent difficulty in keeping one side of the lower lip aligned under pressure-mobility, without altering commissure mobility. None of the patients studied had a recurrence (control, 3-13 years). Although PSP is a technique with a few complications, it has a recurrence rate comparable to or lower than other techniques used for the treatment of pleomorphic adenomas or Warthin parotid tumours. Intraoperative facial nerve monitoring can be helpful during surgery. The lack of monitoring would not be considered a contraindication for surgery. Botulinum toxin has been successfully used to treat Frey's syndrome occurring in a 31-year-old patient following superficial parotidectomy for pleomorphic adenoma. An initial injection of 7.5 U (0.3 ml over 6 cm2 of cheek) resulted in 3 months' resolution of gustatory sweating and flushing and a second injection 12 months' symptomatic improvement. The symptoms recurred after further facial surgery. A case of Frey syndrome (FS) secondary to submaxillar gland exeresis is presented and the results of the treatment with botulinum toxin (BTX) type A. FS is a condition of sweating cheek and preauricular area during realtime as a sequel detected in about 20-60% of patients after parotidectomy. The clinical symptoms include swelling, flushing and hyperhidrosis. The treatment choice for this condition is intracutaneous injection of BTX type A which blocks acetylcholine release at the sweat glands. A 30-year-old man, with thyroid medullar carcinoma diagnosed in 2002 received 6 cicles of cisplatin plus doxorubicin previous to the thyroidectomy with anterolateral neck dissection. During surgery the left ramus marginalis mandibulae was damaged. Two years later the patient referred sweating in submaxillar region during meals. CT scan demonstrated the absence of left submaxillar gland. Minor's test disclosed the affected area and BTX type A was injected (2.5 U/cm2/17 points). A twenty-one-day control showed a 95% reduction of the affected skin area. Persistent efficacy was observed up to one year follow-up time when he was reinjected. The FS, also known as \"gustatory hyperhidrosis\", was probably first reported by M. Duphenix in 1757. Lucja Frey considered its physiopathology as a disorder of both sympathetic and parasympathetic innervation. In our case the FS was caused by a misdirected regeneration of postganglionic parasympathetic nerve fibers that arrised from the nervus lingualis rami ganglionares of the nervus trigeminus. After nerve injury the colinergic parasympathetic fibers seek out colinergic receptors--sympathetic receptors of the skin--innervating sweat glands and small skin vessels. All previous cases were located at masseter region post-parotidectomy. We have not found any description of FS in the submaxillary region. The self-assessed efficacy of the treatment with a hyperhidrosis disease severity scale revealed a very satisfied patient at 20 months follow-up after being injected twice with BTX type A. This is a retrospective study of 10 patients who underwent surgery for a first or multiple recurrence of pleomorphic adenoma (P.A.). Recurrence may come more than 10 years after an initial episode. During initial surgery, the risk of recurrence is related to pre-operative dissemination and to failure to identify tumoral prolongations in the parotid gland. Recurrence is generally multifocal. In 2 cases, carcinoma developed in association with P.A. Treatment of benign recurrence is surgical: it consists of totalizing the previous parotidectomy. If the previous parotidectomy has been total, tumorectomy is carried out. In all cases, surgery preserves the facial nerve trunk. Gustatory sweating is a common complication of parotid surgery. In order to evaluate the incidence of Frey's syndrome following superficial parotidectomy, 69 patients who underwent surgery due to adenoma were studied. Forty-three patients (62%) suffered from gustatory sweating following superficial parotidectomy, and 33 of them requested treatment. Nineteen patients felt that their quality of life had been decreased by the symptoms. Minor's starch iodine test proved that 85% of the patients who did not notice Frey's syndrome after surgery actually had a subclinical manifestation. Eight patients were successfully treated with intracutaneous injections of botulinum toxin A. Within 1 week gustatory sweating disappeared. Frey's syndrome is present in almost all patients following superficial parotidectomy and there is a strong need for treatment. Intracutaneous injection of botulinum toxin A is an effective treatment in severe cases of the syndrome. Complications Complications that may occur due to parotidectomy involve nerve dysfunction, Frey’s syndrome (uncharacteristic sweating near glands), salivation from wound, numbness, facial asymmetry, necrosis (death of skin) near incision, and tumor reappearance. Prognosis There is a 25-50% risk of facial weakness directly after parotidectomy and a 1-2% risk of permanent weakness. Frey’s syndrome may occur in up to 90% of patients. Risk of mortality is very low in reference to the surgery. In a case of benign tumor, such as pleomorphic adenoma, a significant outcome is also the rate of tumor recurrence. Pleomorphic adenomas may recur after a very long time from primary surgery, on average over 7–10 years and up to 24 years. Survival rates due to malignancy depends on the patient and extent of disease. A 10-year survival ranges from 32-83%. Of all cancers, salivary gland tumors account for only 1%. Parotid tumors account for 7% of all head and neck cancers. Etymology To describe the results of parotidectomy for pleomorphic adenoma of the parotid gland in 245 patients, 1974-1994. Descriptive. Academic Medical Hospital, Free University, Amsterdam, the Netherlands. Of all patients follow-up data were obtained by yearly physical diagnostic examination during 10 years, and by a questionnaire (1995) from the general practitioners involved. In the period 1974-1994, 246 primary surgical parotid procedures were performed on 245 patients for pleomorphic adenoma. The surgical procedures included: 131 'partial' and 61 'complete' superficial parotidectomies, 30 partial superficial/ deep lobe parotidectomies, 8 total parotidectomies, and 16 'selective' deep lobe parotidectomies. Eleven patients received postoperative radiotherapy for different reasons. The median follow-up was 95 months. Fourteen patients died without recurrent tumour. Two patients (0.8%) developed a local recurrence, both after total parotidectomy for a deep lobe tumour. None of the patients experienced permanent facial nerve paresis or paralysis. The incidence of auriculotemporal sweating for partial superficial parotidectomy proved to be 6.9% (9/I31) as compared with 13.1% (8/61) for complete superficial parotidectomy. In the later years, in the majority of patients, the posterior branch of the greater auricular nerve was preserved. In the hands of the experienced head and neck surgeon partial parotidectomy is an effective treatment for the great majority of pleomorphic adenomas: local recurrence is rare, while in general morbidity is minimal. Rarely there is a need for prolonged follow-up. Between 1967 and 1994, 344 patients were treated with total conservative parotidectomy for benign pleomorphic adenoma of the parotid gland. Our retrospective study focuses on a sixty-two patients group treated for recurrence after biopsy, enucleation or total parotidectomy. Twenty-two patients underwent a systematic total parotidectomy after biopsy (n = 7) or enucleation (n = 15). Twenty-nine patients were treated with total parotidectomy for local recurrence after enucleation. The mean time before this treatment was 8 years-9 months. In the third group, 11 patients, (7 patients from our institution), were surgically treated for recurrence after total parotidectomy. After enucleation, the recurrence rate was high and insufficient margins were found in 27% of the cases. In this group, a multicentric recurrence was found in 45% of the cases. In our own experience, recurrence after total parotidectomy was noted in 2.4%. The surgical salvage was performed with enucleation after identification of the branches of the facial nerve. The operative microscope was usefull. In 1 case, a second recurrence occured, and in 1 case iterative recurrence was noted. The local control rate after total parotidectomy was 99.6% (292/293). Total conservative parotidectomy is, for us, the treatment of choice for pleomorphic adenoma of the parotid gland. A twenty-year-old woman underwent right superficial parotidectomy for pleomorphic adenoma. On the 10th postoperative day she presented with a salivary fistula, for which repeated aspirations with pressure dressings were applied for a month. Despite decreases in the salivary fluid volume, reaccumulation persisted. Following aspiration of the salivary fluid, 40 units of botulinum toxin was injected into the pouch. On the second day of injection, the discharge ceased and the pouch disappeared. No side effects were observed and the patient remained symptom-free during four-month follow-up. Frey s syndrome or Gustatory sweating was first described by Baillarger in 1853. Lucie Frey had described a patient as \"auriculotemporal syndrome\" in 1923. The explanation for this symptom has been an aberrant regeneration of postganglionic parasympathetic fibers feeding the parotid gland that are severed during parotidectomy. After parotidectomy, these cholinergic parasympathetic fibers regenerate and anastomosis with postganglionic sympathetic fibers that supply vessel and sweat gland of the skin. According to a recent study, the treatment of Frey's syndrome has no treatment of choice. The authors investigated the effectiveness of botulinum toxin type A in the treatment of Frey's syndrome for the first time in Thai patients. The present study was a prospective non-randomized, exploratory study. Nine patients with a median involvement skin area of 4.2 cm2 (1-16.3) were injected intradermal with botulinum toxin type A 2 unit in every 1 cm2 of involved skin. The mean total dose was 10.6 units (range 2-32 unit). All of the patients showed improvement after 4-7 days. Five patients have no Gustatory sweating. In the same way, four patients present with a dramatic decrease in Gustatory sweating. When comparing the skin involvement area, indicated by Minor's iodine starch test and calculated by program ImageJ 1.34s, between before and after injection of botulinum toxin type A using sign test, the result is statistically significant with p = 0.0039. The result lasted for 9.2 months (7-10 months). Intradermal injection of botulinum toxin type A for patients with Frey's syndrome is not only effective with no side effect but also minimally invasive. The present report supports that intradermal injection of botulinum toxin type A should be the treatment of choice for Frey's syndrome. The Frey's syndrome, manifest after parotid trauma, is characterized by head and neck hyperemia and abundant sweating of the hyperemic skin in response to gustatory stimuli. The use of the botulin toxin to treat the symptoms in patients with Frey's syndrome has been described in numerous studies. For some time up until now our Center has achieved excellent results using the group A botulin toxin to overcome the hypertonus of the cricopharyngeal muscle in patients who had undergone laryngectomy and were rehabilitated with voice button. We have sought to extend the use of this toxin to Frey's syndrome, a relatively frequent complication of parotidectomy. A total of 86 patients participated in the study: 41 males (47.6%) and 45 females (52.4%) ranging in age from 25 to 77 years (average age 51 years). Of these patients 7 (8.1%) had undergone post-operative radiotherapy. Of the 86 patients studied, 18 referred significant symptoms in terms of abundance and frequency. The syndrome was considered severe if the symptoms were present at each meal and if the patient indicated a significant worsening of his quality of life. Intermittent episodes were indicated by 22 patients. The remaining 46 (43.5%) did not complain of any symptoms. The exact extension of the cervicofacial gustatory sweating was evaluated using the Minor test and the involved region was divided into 1 square centimeters sections. The amount of skin surface involved ranged from 10 to 80 square centimeters. The type A neurotoxin was frozen and was reconstituted with a sterile saline solution at a final concentration of 2.5 UI/0.1 ml. The intracutaneous infiltration was performed without anesthesia, infiltrating 0.1 ml of solution, containing 2.5 UI of toxin into the center of each 1 square centimeters section. Statistical analysis was performed to evaluate the potential relationship between how long the treatment was effective, incidence of recurrence, seriousness of the crises and the following variables: age, sex, histology, cutaneous surface involved, injected dose of botulin toxin and post-operative radiotherapy. In the group of 18 patients with severe symptoms (20.9%) the benefit was immediate in all cases although the recurrence rate was 50%. The Frey's syndrome symptoms disappeared within 7 days of infiltration. In the group of 22 patients with less severe involvement (25.5%), the treatment gave positive, definitive results in 16 patients (72.7%). Those patients whose symptoms persisted were treated a second time with an infiltration of 2.5 UI per square centimeters. We feel that the use of the type A botulin toxin is the most appropriate treatment for the Frey's syndrome. In fact, such treatment offers the following advantages: it is effective within 7 days, has limited side effects, can be applied on an outpatient basis, is inexpensive and is positively considered by the patients. There have been several approaches for surgery of parotid pleomorphic adenoma in the course of time. Enucleation of the tumor (i.e. intracapsular dissection), a procedure that was common in the early 20th century, is nowadays obsolete due to very high incidence of recurrence. After the time of enucleations, pleomorphic adenomas of parotid gland were recommended to be routinely treated with superficial or total parotidectomy. These procedures combine complete tumor removal and identification of the main trunk of facial nerve during surgery to avoid any lesions to the nerve. However, extensive surgery may cause significant morbidity, such as Frey´s syndrome (excessive sweat while eating) and salivary fistula. Also, aesthetic outcome may be compromised. Therefore, less invasive procedures have been preferred in selected cases during the recent years, and introduction of perioperative neuromonitoring enabled the evolution of several different surgical techniques some twenty years ago. Frey's syndrome, gustatory sweating in the preauricular area, is an unpleasant phenomenon occurring during meals after surgery on the parotid gland. Recently, botulinum toxin A (BTX) has been shown to reduce the symptoms, but the variation in the reported doses is large. To quantify the effect of treatment with low-dose BTX in a case of Frey's syndrome over a period of 6 months. A 56-year-old woman was treated with 10 U Botox given as 20 single, intracutaneous injections of 0.5 U, one for each cm(2), 3 years after resection of the parotid gland. Before treatment and repeatedly during the 6-month period, the sweating was rated subjectively on a 100-mm visual analog scale (VAS) and by a severity index, and objectively by assessment of the extent of the involved skin area using Minor's iodine-starch test, staining the area of sweating dark. The treatment decreased the involved area from 20 to 5 cm(2) and the VAS ratings from 98 to 8 mm. The index showed that treatment affected the sweating intensity, not the frequency. After the 6-month period the patient was still satisfied, but the involved skin area had increased; however, not entirely to pretreatment values. The effect of BTX injections for gustatory sweating obtained in this case was comparable to results reported using higher doses. Low doses of BTX can therefore be used in the treatment of Frey's syndrome, but studies to clarify the dose-response relationship, in terms of both time-course and obtained effect, are needed. Almost all patients who undergo parotidectomy will to some extent develop Frey syndrome (auriculotemporal syndrome or gustatory sweating) after surgery, because of aberrant regeneration of cut parasympathetic fibers between otic ganglion and subcutaneous vessels. However, only the minority of these patients needs treatment. The syndrome consists of gustatory sweating, flushing, and warming over the preauricular and temporal areas. Thick skin flap and partial superficial parotidectomy are the most important techniques to minimize the risk of developing symptomatic Frey syndrome. Intracutaneous injection of botulinum toxin A is an effective, long-lasting, and well-tolerated treatment of Frey syndrome. If recurrence occurs, the treatment can be repeated. Frey’s syndrome: more accurately referred to as gustatory sweating. Patients report facial swelling and sweating at the site of the parotidectomy in occurrence with meals. Etiology is believed to be aberrant innervation of the sweat glands with branches emerging from the auriculotemporal nerve after their division during surgery. This provides parasympathetic innervation to the normally sympathetic-innervated sweat glands [38] . Diagnosis is usually based on patient history, however if there is any doubt an iodine-starch test (Minor test) will confirm the diagnosis, where iodine starch placed on the affected area turns blue signaling sweat secretion. The incidence historically has been reported as high as 50 to 100%, though, with modern techniques and the use of SMAS flaps and thicker skin flaps at the time of initial elevation, this is greatly reduced and is now quite rare. Should this develop, surgical treatment options can be disappointing, with the best results obtained using SMAS and superficial temporal artery flaps as a barrier between the surgical site and the skin. Gold standard treatment now is botulinum toxin injection. Relief of symptoms is obtained for 6 to 36 months. It works at the pre-synaptic level of the neuromuscular and neuroglandular junction by blocking the release of acetylcholine. [37] [39] Currently, the choice of surgical approach for parotid pleomorphic adenoma is mainly based on the size, location, and mobility of the tumor. The recommended main techniques include extracapsular dissection, partial superficial parotidectomy, and lateral or total parotidectomy. Nevertheless, the experience of surgeon plays a key role in the results of these distinct procedures. An important point of view is that recurrent pleomorphic adenomas may occur after a very long time from primary surgery, on average over 7–10 years but up to 24 years afterwards. Thus, it is of utmost importance to evaluate the ultimate results of these different surgical techniques in the future. The benign tumors of the submandibular gland is treated by simple excision with preservation of mandibular branch of the facial nerve, the hypoglossal nerve, and the lingual nerve. Other benign tumors of minor salivary glands are treated similarly. Several authors demand emphatically that the minimal operative procedure in benign parotid gland tumors has to be a superficial parotidectomy. Of a consecutive series of 372 patients with benign parotid tumors treated in our department between 1973-1996 81% of the patients could be followed up 1-24 years. in 10.9% a total parotidectomy was performed, in 16% a lateral parotidectomy and in 73.1% a simple extirpation of the tumor (often taking away a small margin of surrounding parotid parenchyma). The operating microscope and microsurgical techniques were used in all of these operations. Of all the followed-up patients 2.3% developed a recurrence. There were no recurrences of cystadenolymphomas or of rare types of adenomas. Recurrences of primary treated pleomorphic adenomas occurred in 3.0%. In recurrent pleomorphic adenomas a further recurrence could be seen in 7.4% of the cases. The over-all incidence of permanent facial nerve weakness was 2.1%: 0.7% after extirpation, 3.3% after lateral parotidectomy and 9.7% after total parotidectomy. we observed in 6.3% a gustatory sweating. Our data prove that with simple extirpation similar results compared to lateral parotidectomy can be achieved concerning recurrence, function of the facial nerve and the Frey's syndrome. We suggest a surgical management adapted to the extent, the size and the location of the parotid gland tumors. In our opinion lateral or total parotidectomy should be reserved for tumors of larger amount or deep located tumors. Post-Operation After completion of a parotidectomy, patients can expect postoperative hospitalization ranging from one-to-three days, to help ensure the safest and most effective postoperative management. At this time, patients will be administered antibiotics to minimize risk of infection as well as an assessment of pain management throughout their stay. Duration of hospitalization is subject to change from patient to patient, with most patients being discharged within 24 hours after surgery. If a tumor was malignant, many patients are referred to radiation therapy. For benign tumors and slow growing cancers, surgery typically provides a complete cure or remission (no evidence for disease). Patient Care after Discharge Throughout history, many different types and techniques have been developed in order to complete a parotidectomy and consequently, many different names have been associated with each type. However, there are really only two main distinctions to be made in parotidectomies: The specific nerve(s) to be dissected or not dissected The amount of gland excised It is important to note that the specific surgery chosen is based on preservation of the facial nerve in order to avoid significant morbidities (diseases). Furthermore, there are still many controversies regarding the choice of surgery and incidence of cancer recurrence. Below indicates the various and main techniques typically associated with a parotidectomy: Extracapsular dissection - excision of the parotid tumor surrounded by some millimetres of healthy tissue, without searching and exposing the main truck of the facial nerve. This study compared extracapsular dissection (ED) vs superficial parotidectomy (SP) in the treatment of pleomorphic adenoma and benign parotid tumors. The research covered the years 1950-2011 in PubMed, Ovid MEDLINE, the Cochrane Database of Systematic Reviews, and Scopus. Of 1152 articles screened, 123 studies met the inclusion criteria. A review of the nomenclature of the different parotid surgery techniques was done. Recurrence rate, permanent facial nerve paralysis, and Frey syndrome of patients who underwent ED vs those who underwent SP were compared by meta-analysis. Our meta-analysis data comparing ED and SP found that: (1) the recurrence rate is higher in patients treated with SP; (2) SP has a higher incidence of cranial nerve VII paralysis; and (3) Frey syndrome is more common after SP. ED may be a viable option in the treatment of unilateral benign parotid tumors of the superficial lobe, sized less than 4 cm, without involvement of the facial nerve. Treatments Injection of botulinum toxin A Surgical transection of the nerve fibers (a temporary treatment) Application of an ointment containing an anticholinergic drug such as scopolamine Cochrane reviews of interventions to either prevent or treat Frey’s syndrome have found little or no evidence to support their effectiveness or safety, and conclude that further clinical trials are needed. Epidemiology The condition is rare, although the exact incidence is unknown. The disorder most often occurs as a complication of the surgical removal of a parotid gland (parotidectomy). The percentage of individuals who develop Frey syndrome after a parotidectomy is controversial and reported estimates range from 30–50 percent. In follow-up examinations, approximately 15 percent of affected individuals rated their symptoms as severe. Frey syndrome affects males and females in equal numbers. Twenty years experience of lateral parotidectomy as suspical treatment for pleomorphic adenoma are reviewed. All cases were managed at the ORL Clinic of the University of Zürich. 167 patients were followed for the frequency of possible recurrent tumors. Three patients (3/123) operated primarily developed a recurrences. Recurrences appeared after an average of 10 years, ranging from 1-30 years. The follow-up time varied from 1 to 21 years (average, 8 years). 39% (13 of 33) of the patients, who were re-operated for a recurrent tumor, developed another recurrence. The second recurrence appeared after an average of 10 years, ranging from 1-22 years. A persistent partial paresis of the facial nerve was found in 1% of the patients operated primarily and in 9% of the patients operated more than once. No paralysis was seen. We now choose \"en-bloc\" resections of pleomorphic adenomas without intra-operative opening of the tumor capsule as the treatment of choice. This treatment was possible in 83% of all cases, using a lateral parotidectomy. If tumor extends into the medial parotid lobe, total parotidectomy is required. The tumors of the salivary glands are infrequent in children, and parotid gland is involved in 80% of them. When a salivary gland tumor is present, the chance of malignancy is greater in the child than in the adult. We reviewed 8 cases identified in patients aged 14 years and younger in our hospital, analyzing its antecedents, signs and symptoms, histological features, diagnosis, treatment and evolution. All the patients displayed preauricular painless, non-inflammatory and slow-growing masses to an age between 10 months and 14 years. Four or them were pleomorphic adenomas, two haemangiomas, one epidermal cysts and one myoepithelial carcinoma. We emphasize the exceptional nature of the carcinoma for its rareness and for the high degree of malignancy expressed. We made a fine needle aspiration biopsy in four cases but they were conclusive only in three. All were treated by surgical resection of the tumour except for the myoepithelial carcinoma and the recurrent pleomorphic adenoma that were treated by total parotidectomy. The malignant tumours of the parotid gland are clinically indistinguishable of the benign ones, thus when any palpable mass appears in the zone of the parotid gland, an accurate diagnosis should be made without delay. The treatment of choice is the surgical excision with wide margins, being other adjuvant treatments less useful to this age than in the adult age. To evaluate salivary gland chemodenervation with botulinum toxin in chronic parotid sialadenitis. Patients who underwent parotid gland chemodenervation for chronic sialadenitis due to duct stenosis refractory to siaendoscopy were reviewed (case series). Additionally, a systematic review of the literature on botulinum toxin injection for chronic parotid sialadenitis was performed. Inclusion criteria included studies containing original data on botulinum toxin injections in patients with chronic sialadenitis symptoms. Sialadenitis symptoms from 10 patients with 13 affected parotid glands were examined. All had duct stenosis diagnosed on sialendoscopy, refractory sialadenitis symptoms, and received parotid onabotulinum toxin injection(s) (median dose 65U). Of patients with 3-month follow-up, 78% reported significant improvement in symptoms. Mean Chronic Obstructive Sialadenitis Symptoms (COSS) Score improved at 3 months post-injection (47-25.9, 10 WBCs/high power field (hpf) 2. Dipstick-Positive leukocyte esterase (due to pyuria) and nitrites (bacteria convert nitrates to nitrites) 3. Culture-greater than 100,000 colony forming units (gold standard) D. Etiology 1. E coli (80%) 2. Staphylococcus saprophyticus-increased incidence in young, sexually active women (but E coli is still more common in this population) 3. Klebsiella pneumoniae 4. Proteus mirabilis-alkaline urine with ammonia scent Fig. 12.15 Linear IF, Goodpasture syndrome. Fig. 12.16 lgA nephropathy. (Courtesy ofTony (Courtesy ofTony Chang, MD) Chang, MD) 5. Enterococcus faecalis E. Sterile pyuria is the presence of pyuria (> 10 WBCs/hpf and leukocyte esterase) with a negative urine culture. Urinary tract infection (UTI) is a common bacterial infection during pregnancy and a significant cause of perinatal and maternal morbidity and mortality. The causative bacteria have remained virtually the same although with variations in individual prevalence. There has been an increasing resistance by these bacteria to the commonly available antibiotics. To determine the prevalence of UTI, the common causative bacteria, and their antibiotic sensitivity pattern among pregnant women with UTI. This is a descriptive study that was carried out at the Obstetrics Department of two tertiary institutions in Abakaliki, Ebonyi State, Nigeria (Federal Medical Center and Ebonyi State University Teaching Hospital) over a period of 12 months. Midstream urine specimens from selected pregnant women with clinical features of UTI were collected for microscopy, culture, and sensitivity. The results were analyzed with the 2008 Epi Info™ software. A total of 542 pregnant women presented with symptoms of UTI and were recruited for the study over the study period. Of the 542 pregnant women, 252 (46.5%) had significant bacteriuria with positive urine culture and varying antibiotic sensitivity pattern. The prevalence of symptomatic UTI was 3%. Escherichia coli was the most common bacteria isolated with a percentage of 50.8%. Other isolated micro organisms included Stapylococcus aereus (52 cultures, 20.6%), Proteus mirabilis (24 cultures, 9.5%), S. saprophyticus (18 cultures, 7.1%), Streptococcus spp. (14 cultures, 5.6%), Citrobacter spp. (5 cultures, 2.0%), Klebsiella spp. (4 cultures, 1.6%), Enterobacter spp. (4 cultures, 1.6%), and Pseudomonas spp. (3 cultures, 1.2%). Levofloxacin had the highest overall antibiotic sensitivity of 92.5%. Others with overall antibiotic sensitivity pattern greater than 50% included cefpodoxime (87.3%), ofloxacin (77.4%), ciprofloxacin (66.7%), ceftriaxone (66.7%), and gentamicin (50.8%). E. coli was the most common etiological agent of UTI in pregnancy with Enterococcus (Staphylococcus) gaining prominence. Cephalosporin and quinolones were shown to be very effective against the organisms causing UTI in these pregnant women. Uncomplicated urinary tract infections (uUTIs) are common in adult women across the entire age spectrum, with mean annual incidences of approximately 15% and 10% in those aged 15-39 and 40-79 years, respectively. By definition, UTIs in males or pregnant females and those associated with risk factors known to increase the risk of infection or treatment failure (e.g. acquisition in a hospital setting, presence of an indwelling urinary catheter, urinary tract instrumentation/interventions, diabetes mellitus or immunosuppression) are not considered herein. The majority of uUTIs are caused by Escherichia coli (70-95%), with Proteus mirabilis, Klebsiella spp. and Staphylococcus saprophyticus accounting for 1-2%, 1-2% and 5-10% of infections, respectively. If clinical signs and symptoms consistent with uUTI are present (e.g. dysuria, frequency, back pain or costovertebral angle tenderness) and there is no vaginal discharge or irritation present, the likelihood of uUTI is >90-95%. Laboratory testing (i.e. urinary nitrites, leukocyte esterase, culture) is not necessary in this circumstance and empirical treatment can be initiated. The ever-increasing incidence of antimicrobial resistance of the common uropathogens in uUTI has been and is a continuing focus of intensive study. Resistance to cotrimoxazole (trimethoprim/sulfamethoxazole) has made the empirical use of this drug problematic in many geographical areas. If local uropathogen resistance rates to cotrimoxazole exceed 10-25%, empirical cotrimoxazole therapy should not be utilized (fluoroquinolones become the new first-line agents). In a few countries, uropathogen resistance rates to the fluoroquinolones now exceed 10-25%, rendering empirical use of fluoroquinolones problematic. With the exception of fosfomycin (a second-line therapy), single-dose therapy is not recommended because of suboptimal cure rates and high relapse rates. Cotrimoxazole and the fluoroquinolones can be administered in 3-day regimens. Nitrofurantoin, a second-line therapy, should be given for 7 days. beta-Lactams are not recommended because of suboptimal clinical and bacteriological results compared with those of non-beta-lactams. If a beta-lactam is chosen, it should be given for 7 days. Management of uUTIs can frequently be triaged to non-physician healthcare personnel without adverse clinical consequences, resulting in substantial cost savings. It can be anticipated that the optimal approach to the management of uUTIs will change substantially in the future as a consequence of antimicrobial resistance. This report describes a case of urinary tract infection (UTI) due to Shigella sonnei during pregnancy. A 31-year-old pregnant woman was admitted complaining of left-flank tenderness, dysuria, and fever. Following examination, significant laboratory data were collected including increased leukocyte count (10,800/ul with 86% neutrophils) and C-reactive protein (9.6 mg/dl). Urinalysis revealed 30 to 50 leukocytes per high power field while from the quantitative urine culture Shigella sonnei was recovered after 24 h incubation at 37 degrees C. After a two-week course with 750 mg cefuroxime every 8 h, the patient experienced gradual resolution of all symptoms and urinary cultures were negative two weeks and one month, respectively, after completing the therapy. The gestational course was uneventful and the patient delivered a healthy baby girl at term. Shigella sonnei can be responsible for UTI during pregnancy even when no predisposing factors or an apparent source of infection can be identified. To determine the prevalence of outpatient-diagnosed urinary tract infection (UTI) in consecutive febrile neonates ≤ 30 days of age and correlate demographic, laboratory and radiographic imaging results with infectious etiology. Review of medical records of consecutive febrile infants ≤ 30 days of age presenting to an urban pediatric emergency department during a 10-year period, whose policy is to perform a sepsis evaluation (urine culture obtained by bladder catheterization) and hospitalize for parenteral antibiotic therapy pending culture results. Of 670 febrile neonates ≤ 30 days of age evaluated for sepsis, urine culture was obtained in 651 cases (97%). Of 100 patients with UTI (15.4%), 73% were male; the most common uropathogens were Escherichia coli (71%), Enterococcus (10%) and Klebsiella sp. (10%). In all, 39% had a maximum documented fever ≥ 102 °F, and 40% had CBC total white blood cells count ≥ 15,000/mm(3). Urine dipstick test was positive for leukocyte esterase or nitrite in 79%. Renal ultrasound performed in 95 patients (95%) showed anatomic abnormalities in 47%; 5/26 (24%) with hydronephrosis had vesicoureteral reflux on voiding cystourethrogram. Four patients had urosepsis; none had bacterial meningitis and no patients died. UTI affects approximately 1 in 6 febrile neonates ≤ 30 days of age. Males are affected 2.5-times greater than females. E. coli continues to be the predominant uropathogen. Clinical parameters like height of fever, CBC total white blood cell count and urine dipstick test lack sensitivity in identifying UTI risk in the outpatient setting. Only 4 infants had urosepsis (4%). Nearly half of neonates with UTI have a radiographically identified anatomic abnormality. All febrile young infants should receive performance of a urine culture; those with UTI require imaging. If this infection is suspected, a urine sample obtained by catheterization may be preferred to avoid obscuring contamination from the lower genital tract. he urinary sediment contains many leukocytes, frequently in clumps, and numerous bacteria. Bacteremia is demonstrated in 15 to 20 percent of these women. E coli is isolated from urine or blood in 70 to 80 percent of infections, Klebsiela pneumoniae in 3 to 5 percent, Enterobacter or Proteus species in 3 to 5 percent, and gram-positive organisms, including group B Streptococcus and Staphylococcus aureus, in up to 10 percent of cases (Hill, 2005; Wing, 2000). The objective of this study was (1) to determine the reliability of urinalysis (UA) for predicting urinary tract infection (UTI) in febrile children, (2) to determine whether UA findings can predict Escherichia coli versus non-E. coli urinary tract infection, and (3) to determine if empiric antibiotics should be selected based on E. coli versus non-E. coli infection predictions. This was a retrospective chart review of children from 2 months to 2 years of age who presented to the emergency department with fever (rectal temperature >100.4°F) and had a positive urine culture. This study was conducted between January 2004 and December 2007. Negative UA was defined as urine white blood cell count less than 5 per high-power field, negative leukocyte esterase, and negative nitrites. Urine cultures were classified into E. coli and non-E. coli groups. These groups were compared for sex, race, and UA findings. Multivariate forward logistic regression, using the Wald test, was performed to calculate the likelihood ratio (LR) of each variable (eg, sex, race, UA parameters) in predicting UTI. In addition, antibiotic sensitivities between both groups were compared. Of 749 medical records reviewed, 608 were included; negative UA(-) was present in 183 cases, and positive UA(+) was observed in 425 cases. Furthermore, 424 cases were caused by E. coli, and 184 were due to non-E. coli organisms. Among 425 UA(+) cases, E. coli was identified in 349 (82.1%), whereas non-E. coli organisms were present in 76 (17.9%); in contrast, in 183 UA(-) cases, 108 (59%) were due to non-E. coli organisms versus 75 (41%), which were caused by E. coli. Urinalysis results were shown to be associated with organism group (P < 0.001). Positive leukocytes esterase had an LR of 2.5 (95% confidence interval [CI], 1.5-4.2), positive nitrites had an LR of 2.8 (95% CI, 1.4-5.5), and urine white blood cell count had an LR of 1.8 (95% CI, 1.3-2.4) in predicting E. coli versus non-E. coli infections. Antibiotic sensitivity compared between UA groups demonstrated equivalent superiority of cefazolin (94.7% sensitive in UA(+) vs 84.0% in UA(-) group; P < 0.0001), cefuroxime (98.2% vs 91.7%; P < 0.001), and nitrofurantoin (96.1% vs 82.2%; P < 0.0001) in the UA(+) group. In contrast, the UA(-) group showed significant sensitivity to trimethoprim-sulfamethoxazole (82.2% vs 71.3% in UA(+); P = 0.008). Urinalysis is not an accurate predictor of UTI. A positive urine culture in the presence of negative UA most likely grew non-E. coli organisms, whereas most UA(+) results were associated with E. coli. This study also highlighted local patterns of antibiotic resistance between E. coli and non-E. coli groups. Negative UA results in the presence of strong suspicion of a UTI suggest a non-E. coli organism, which may be best treated with trimethoprim-sulfamethoxazole. Conversely, UA(+) results suggest E. coli, which calls for treatment with cefazolin or cefuroxime. Urinary tract infection (UTI) is the most common serious bacterial infection in young infants. Signs and symptoms are often nonspecific. To describe clinical, demographic and laboratory features of UTI in infants ≤ 3 months old. Cross-sectional study of infants ≤ 3 months old with UTI diagnosed in a pediatric emergency department, for the period 2010-2012. UTI was defined as ≥ 50,000 colony-forming units per milliliter of a single uropathogen isolated from bladder catheterization. Paired urinalysis and urine culture from group culture-positive and group culture-negative were used to determine the sensitivity and specificity of pyuria and nitrite tests in detecting UTI. Of 519 urine cultures collected, UTI was diagnosed in 65 cases (prevalence: 12.5%); with male predominance (77%). The most common etiologies were Escherichia coli (56.9%), Klebsiella pneumoniae (18.5%) and Enterococcus faecalis (7.7%). Frequent clinical manifestations were fever (77.8%), irritability (41.4%) and vomiting (25.4%). The median temperature was 38.7°C. The sensitivity of the nitrite test was 30.8% (95%CI:19.9-43.4%), specificity of 100% (95%CI:99.2-100%). Pyuria ≥ 10,000/mL had a sensitivity of 87.7% (95%CI:77.2-94.5%), specificity of 74.9% (95%CI:70.6 -78.8%). The median peripheral white blood cell count was 13,150/mm3; C-reactive protein levels were normal in 30.5% of cases. The male: female ratio for urinary tract infection was 3.3:1. Non-Escherichia coli etiologies should be considered in empirical treatment. Fever was the main symptom. Positive nitrite is highly suggestive of UTI but has low sensitivity; whereas pyuria ≥ 10,000/mL revealed good sensitivity, but low specificity. Peripheral white blood cell count and C-reactive protein concentration have limited usefulness to suggest UTI. Diagnosis In straightforward cases, a diagnosis may be made and treatment given based on symptoms alone without further laboratory confirmation. In complicated or questionable cases, it may be useful to confirm the diagnosis via urinalysis, looking for the presence of urinary nitrites, white blood cells (leukocytes), or leukocyte esterase. Another test, urine microscopy, looks for the presence of red blood cells, white blood cells, or bacteria. Urine culture is deemed positive if it shows a bacterial colony count of greater than or equal to 103 colony-forming units per mL of a typical urinary tract organism. Antibiotic sensitivity can also be tested with these cultures, making them useful in the selection of antibiotic treatment. However, women with negative cultures may still improve with antibiotic treatment. As symptoms can be vague and without reliable tests for urinary tract infections, diagnosis can be difficult in the elderly. Septic arthritis of a lumbar facet joint is a rare condition. We report the case of a 77-year-old diabetic woman who developed fever and back pain 15 days after she had been diagnosed with a genitourinary infection for which she had received ciprofloxacin. Physical examination showed fever (38°C) and pain on pressure over the lower lumbar spinous vertebral apophyses and over the lower left paraspinal musculature. Investigations showed a white cell count of 8.4×10⁹/l, neutrophils 85.3%, erythrocyte sedimentation rate of 125 mm/h, and C-reactive protein of ≥9 mg/dl. Two blood cultures were both positive for Escherichia coli resistant to ciprofloxacin. There was no growth of pathogens from the urine cultures. Scintigraphy with gallium citrate Ga⁶⁷ showed vertical lower lumbar (L4-L5) radionuclide uptake lateralized to the left. Magnetic resonance imaging of the lumbar spine demonstrated signal changes and alteration of the structure at the left interapophyseal L4-L5 joint, an adjacent small collection of 1cm in diameter, and infiltration of the surrounding soft tissues, which extended to the epidural area, left conjunction hole, and paraspinal muscles. The patient was treated with intravenous cefotaxime and gentamicin and bed rest for 21 days, and recovered. This is the first report of interapophyseal arthritis caused by E. coli. Urinary nitrite test A nitrite test is a standard component of a urinary test strip. A positive test for nitrites in the urine is called nitrituria. This test is commonly used in diagnosing urinary tract infections (UTIs). A positive nitrite test indicates that the cause of the UTI is a gram negative organism, most commonly Escherichia coli. The reason for nitrites' existence in the presence of a UTI is due to a bacterial conversion of endogenous nitrates to nitrites. This may be a sign of infection. However, other parameters, such as leukocyte esterase, urine white blood cell count, and symptoms such as dysuria, urinary urgency, fevers, and chills must be correlated to diagnose an infection. Organisms causing UTI in pregnancy are the same uropathogens which commonly cause UTI in non-pregnant patients. Escherichia coli is the most common organism isolated. An 18-year retrospective analysis found E. coli to be the causative agent in 82.5% of cases of pyelonephritis in pregnant patients. [3] Other bacteria which may be seen include Klebsiella pneumoniae, Staphylococcus, Streptococcus, Proteus, and Enterococcus species. It is normal to find up to 3 (occasionally 5) leukocytes per high power field (40X) in a urine sample, with women having slightly higher results owing to vaginal contamination. Higher numbers indicate urinary infection. The urine test strip test for white blood cells detects leukocyte esterase, which is present in azurophilic granules of monocytes and granulocytes (neutrophilic, eosinophilic and basophilic). Bacteria, lymphocytes and epithelial cells from the genitourinary tract do not contain esterases. Neutrophil granulocytes are the leukocytes most commonly associated with urinary infections. A positive test for leukocyte esterase normally indicates the presence of bacteria and a positive nitrite test (although it is not always the case). Infections caused by Trichomonas, Chlamydia and yeasts produce leukocyturia without bacteriuria. The inflammation of the renal tissues (interstitial nephritis) can produce leukocyturia, in particular toxic interstitial nephritis with predominant Pathogenesis The bacteria that cause urinary tract infections typically enter the bladder via the urethra. However, infection may also occur via the blood or lymph. It is believed that the bacteria are usually transmitted to the urethra from the bowel, with females at greater risk due to their anatomy. After gaining entry to the bladder, E. Coli are able to attach to the bladder wall and form a biofilm that resists the body's immune response. Escherichia coli is the single most common microorganism, followed by Klebsiella and Proteus spp., to cause urinary tract infection. Klebsiella and Proteus spp., are frequently associated with stone disease. The presence of Gram positive bacteria such as Enterococcus and Staphylococcus increased. The increased resistance of urinary pathogens to quinolone antibiotics has been reported worldwide and might be the consequence of overuse and misuse of quinolones. Diagnosis We compare the test characteristics of urine dipstick and urinalysis at various test cutoff points in women presenting to emergency departments and an intermediate care center with symptoms of urinary tract infection. This was a prospective, observational study of adult women presenting to 1 of 2 community hospital EDs or an intermediate care center with dysuria, urgency, or urinary frequency on history, or suprapubic or costovertebral angle tenderness on examination. Patients who had taken antibiotics in the past 72 hours, had indwelling Foley catheters, symptomatic vaginal discharge, diabetes mellitus, immunodeficiency disorders, or were unable to provide a reliable history were excluded. The patient's clean-catch or catheterized urine specimen was tested immediately by a nurse using a Multistix 9 SG reagent strip. A second aliquot was sent within 1 hour of collection to the hospital laboratory, where a semiautomated microscopic urinalysis and a urine culture were performed. A positive urine culture was defined as more than 100,000 colonies of 1 or 2 uropathogenic bacteria per mL of urine at 48 hours. Dipstick and urinalysis data were compared with urine culture results. Sensitivity, specificity, and predictive values were calculated at various definitions of a positive test, or \"test cutoff points,\" for combinations of leukocyte esterase, nitrite, and blood on dipstick and for RBCs and WBCs on urinalyses. The probability of an erroneous decision to withhold treatment on the basis of a negative test result was defined as \"undertreatment,\" or 1 minus the negative predictive value. \"Overtreatment\" was defined as 1 minus the positive predictive value. Three hundred forty-three patients were enrolled in this study. Twelve patients were withdrawn because of missing laboratory results. Forty-six percent (152/331) of patients had positive urine cultures. If urine dipstick results are defined as positive when leukocyte esterase or nitrite is positive or blood is more than trace, the overtreatment rate is 47% (156/331) and the undertreatment rate is 13% (43/331). If urinalysis results are defined as positive when WBCs are more than 3 per high-power field or RBCs are more than 5 per high-power field, the overtreatment rate is 44% (146/331) and the undertreatment rate is 11% (36/331). Matched pairs of test characteristics were identified when the analysis was repeated using more than 10,000 colonies per mL as a positive culture. In this patient population, similar overtreatment and undertreatment rates were identified for various test cutoff points for urine dipstick tests and urinalysis. Although a urine dipstick may be equivalent to a urinalysis for the diagnosis of urinary tract infection, the limitations in the diagnostic accuracy of both tests should be incorporated into medical decisionmaking. Urinary tract infection (UTI) is one of the most common infections during pregnancy, which can lead to significant maternal and perinatal morbidity and mortality if left untreated. Challenges when treating UTIs in pregnancy include fetal protection and resistance development of uropathogens. Currently, the Essential Medicines List recommends nitrofurantoin to treat cystitis and ceftriaxone to treat pyelonephritis in pregnant women. To determine common pathogens causing UTI in pregnancy and their antibiotic susceptibility patterns. A retrospective analysis was performed of laboratory data for positive urine specimens from obstetric departments of 6 KwaZulu- Natal Province hospitals during 2011 - 2016. Identification and susceptibility testing were performed using the VITEK 2 system. Results were interpreted according to the breakpoints of the Clinical and Laboratory Standards Institute, USA. From 5 971 positive urine specimens, the most common isolate was Escherichia coli (n=3 236; 54.2%), followed by Klebsiella pneumoniae (n=770; 12.9%). Group B streptococcus (GBS) (n=239; 4.0%) and Enterococcus faecalis (n=251; 4.2%) were the most common Gram-positive pathogens. E. coli displayed significant resistance to trimethoprim-sulfamethoxazole (65.1%), cephalothin (38.3%), cefuroxime (27.3%), ciprofloxacin (16.9%) and amoxicillin-clavulanic acid (17.1%). Resistance to ceftriaxone and nitrofurantoin remained low ‒ 9.1% and 7.7%, respectively. Among Gram-positive pathogens, GBS displayed 100% penicillin susceptibility and E. faecalis showed 92.9% susceptibility to ampicillin. E. coli is unsurprisingly the most common cause of UTI in pregnancy in KwaZulu-Natal. Susceptibility to ceftriaxone and nitrofurantoin remains good. Among Gram positives, GBS is prevalent and susceptible to penicillin, while E. faecalis is susceptible to ampicillin. As antimicrobial resistance evolves, routine surveillance is necessary to modify recommended empirical antibiotic use.\nHere is the question:\nMICROBIOLOGY: Pregnant woman, 27 years old, 30 weeks of gestation. She comes to the emergency room because she noticed pain in the left lumbar region and dysuria since yesterday. She has no febrile sensation. She refers repeated urinary tract infections (UTI). Urinalysis shows Hb 3+, leukocytes 3+, nitrites 2+, sediment: 15-20 leukocytes per field and 5-10 red blood cells per field. Which of the following microorganisms is the most frequent culprit in pregnant women?\nHere are the potential choices:\n1. Escherichia coli.\n2. Enterococcus faecalis.\n3. Streptococcus agalactiae.\n4. Proteus mirabilis.\n5. Satphylococcus saprophyticus.\nThe correct answer is: ", "gold_answer": "1 Escherichia coli.", "prediction": "You are a helpful medical expert, and your task is to answer a multi-choice medical question using the relevant documents. Please choose the answer from the provided options. Your responses will be used for research purposes only, so please have a definite answer.\nHere are the relevant documents:\n Pregnant women are usually at risk of urinary tract infections (UTIs) such as asymptomatic bacteriuria. In the current multidrug-resistance era, appropriate diagnosis and treatment should be provided to avoid complications in pregnant women in developing countries, which have limited facilities, such as Indonesia. The aim of this study was to evaluate in vitro susceptibility tests. Urinary isolates were collected from 715 pregnant women who visited eight Community Health Centers in Jakarta, Indonesia, between 2015 and 2017. We identified bacterial uropathogens from samples that were positive for nitrite/leukocyte esterase (LE), using two types of VITEK cards. Since noncompliance among patients is a major problem, fosfomycin-trometamol 3 g single-dose sachets were given to the patients, and the side effects of the medication and neonatal outcomes were reported. Asymptomatic bacteriuria was found in 10.5% of the 715 pregnant women. 10 WBCs/high power field (hpf) 2. Dipstick-Positive leukocyte esterase (due to pyuria) and nitrites (bacteria convert nitrates to nitrites) 3. Culture-greater than 100,000 colony forming units (gold standard) D. Etiology 1. E coli (80%) 2. Staphylococcus saprophyticus-increased incidence in young, sexually active women (but E coli is still more common in this population) 3. Klebsiella pneumoniae 4. Proteus mirabilis-alkaline urine with ammonia scent Fig. 12.15 Linear IF, Goodpasture syndrome. Fig. 12.16 lgA nephropathy. (Courtesy ofTony (Courtesy ofTony Chang, MD) Chang, MD) 5. Enterococcus faecalis E. Sterile pyuria is the presence of pyuria (> 10 WBCs/hpf and leukocyte esterase) with a negative urine culture. Urinary tract infection (UTI) is a common bacterial infection during pregnancy and a significant cause of perinatal and maternal morbidity and mortality. The causative bacteria have remained virtually the same although with variations in individual prevalence. There has been an increasing resistance by these bacteria to the commonly available antibiotics. To determine the prevalence of UTI, the common causative bacteria, and their antibiotic sensitivity pattern among pregnant women with UTI. This is a descriptive study that was carried out at the Obstetrics Department of two tertiary institutions in Abakaliki, Ebonyi State, Nigeria (Federal Medical Center and Ebonyi State University Teaching Hospital) over a period of 12 months. Midstream urine specimens from selected pregnant women with clinical features of UTI were collected for microscopy, culture, and sensitivity. The results were analyzed with the 2008 Epi Info™ software. A total of 542 pregnant women presented with symptoms of UTI and were recruited for the study over the study period. Of the 542 pregnant women, 252 (46.5%) had significant bacteriuria with positive urine culture and varying antibiotic sensitivity pattern. The prevalence of symptomatic UTI was 3%. Escherichia coli was the most common bacteria isolated with a percentage of 50.8%. Other isolated micro organisms included Stapylococcus aereus (52 cultures, 20.6%), Proteus mirabilis (24 cultures, 9.5%), S. saprophyticus (18 cultures, 7.1%), Streptococcus spp. (14 cultures, 5.6%), Citrobacter spp. (5 cultures, 2.0%), Klebsiella spp. (4 cultures, 1.6%), Enterobacter spp. (4 cultures, 1.6%), and Pseudomonas spp. (3 cultures, 1.2%). Levofloxacin had the highest overall antibiotic sensitivity of 92.5%. Others with overall antibiotic sensitivity pattern greater than 50% included cefpodoxime (87.3%), ofloxacin (77.4%), ciprofloxacin (66.7%), ceftriaxone (66.7%), and gentamicin (50.8%). E. coli was the most common etiological agent of UTI in pregnancy with Enterococcus (Staphylococcus) gaining prominence. Cephalosporin and quinolones were shown to be very effective against the organisms causing UTI in these pregnant women. Uncomplicated urinary tract infections (uUTIs) are common in adult women across the entire age spectrum, with mean annual incidences of approximately 15% and 10% in those aged 15-39 and 40-79 years, respectively. By definition, UTIs in males or pregnant females and those associated with risk factors known to increase the risk of infection or treatment failure (e.g. acquisition in a hospital setting, presence of an indwelling urinary catheter, urinary tract instrumentation/interventions, diabetes mellitus or immunosuppression) are not considered herein. The majority of uUTIs are caused by Escherichia coli (70-95%), with Proteus mirabilis, Klebsiella spp. and Staphylococcus saprophyticus accounting for 1-2%, 1-2% and 5-10% of infections, respectively. If clinical signs and symptoms consistent with uUTI are present (e.g. dysuria, frequency, back pain or costovertebral angle tenderness) and there is no vaginal discharge or irritation present, the likelihood of uUTI is >90-95%. Laboratory testing (i.e. urinary nitrites, leukocyte esterase, culture) is not necessary in this circumstance and empirical treatment can be initiated. The ever-increasing incidence of antimicrobial resistance of the common uropathogens in uUTI has been and is a continuing focus of intensive study. Resistance to cotrimoxazole (trimethoprim/sulfamethoxazole) has made the empirical use of this drug problematic in many geographical areas. If local uropathogen resistance rates to cotrimoxazole exceed 10-25%, empirical cotrimoxazole therapy should not be utilized (fluoroquinolones become the new first-line agents). In a few countries, uropathogen resistance rates to the fluoroquinolones now exceed 10-25%, rendering empirical use of fluoroquinolones problematic. With the exception of fosfomycin (a second-line therapy), single-dose therapy is not recommended because of suboptimal cure rates and high relapse rates. Cotrimoxazole and the fluoroquinolones can be administered in 3-day regimens. Nitrofurantoin, a second-line therapy, should be given for 7 days. beta-Lactams are not recommended because of suboptimal clinical and bacteriological results compared with those of non-beta-lactams. If a beta-lactam is chosen, it should be given for 7 days. Management of uUTIs can frequently be triaged to non-physician healthcare personnel without adverse clinical consequences, resulting in substantial cost savings. It can be anticipated that the optimal approach to the management of uUTIs will change substantially in the future as a consequence of antimicrobial resistance. This report describes a case of urinary tract infection (UTI) due to Shigella sonnei during pregnancy. A 31-year-old pregnant woman was admitted complaining of left-flank tenderness, dysuria, and fever. Following examination, significant laboratory data were collected including increased leukocyte count (10,800/ul with 86% neutrophils) and C-reactive protein (9.6 mg/dl). Urinalysis revealed 30 to 50 leukocytes per high power field while from the quantitative urine culture Shigella sonnei was recovered after 24 h incubation at 37 degrees C. After a two-week course with 750 mg cefuroxime every 8 h, the patient experienced gradual resolution of all symptoms and urinary cultures were negative two weeks and one month, respectively, after completing the therapy. The gestational course was uneventful and the patient delivered a healthy baby girl at term. Shigella sonnei can be responsible for UTI during pregnancy even when no predisposing factors or an apparent source of infection can be identified. To determine the prevalence of outpatient-diagnosed urinary tract infection (UTI) in consecutive febrile neonates ≤ 30 days of age and correlate demographic, laboratory and radiographic imaging results with infectious etiology. Review of medical records of consecutive febrile infants ≤ 30 days of age presenting to an urban pediatric emergency department during a 10-year period, whose policy is to perform a sepsis evaluation (urine culture obtained by bladder catheterization) and hospitalize for parenteral antibiotic therapy pending culture results. Of 670 febrile neonates ≤ 30 days of age evaluated for sepsis, urine culture was obtained in 651 cases (97%). Of 100 patients with UTI (15.4%), 73% were male; the most common uropathogens were Escherichia coli (71%), Enterococcus (10%) and Klebsiella sp. (10%). In all, 39% had a maximum documented fever ≥ 102 °F, and 40% had CBC total white blood cells count ≥ 15,000/mm(3). Urine dipstick test was positive for leukocyte esterase or nitrite in 79%. Renal ultrasound performed in 95 patients (95%) showed anatomic abnormalities in 47%; 5/26 (24%) with hydronephrosis had vesicoureteral reflux on voiding cystourethrogram. Four patients had urosepsis; none had bacterial meningitis and no patients died. UTI affects approximately 1 in 6 febrile neonates ≤ 30 days of age. Males are affected 2.5-times greater than females. E. coli continues to be the predominant uropathogen. Clinical parameters like height of fever, CBC total white blood cell count and urine dipstick test lack sensitivity in identifying UTI risk in the outpatient setting. Only 4 infants had urosepsis (4%). Nearly half of neonates with UTI have a radiographically identified anatomic abnormality. All febrile young infants should receive performance of a urine culture; those with UTI require imaging. If this infection is suspected, a urine sample obtained by catheterization may be preferred to avoid obscuring contamination from the lower genital tract. he urinary sediment contains many leukocytes, frequently in clumps, and numerous bacteria. Bacteremia is demonstrated in 15 to 20 percent of these women. E coli is isolated from urine or blood in 70 to 80 percent of infections, Klebsiela pneumoniae in 3 to 5 percent, Enterobacter or Proteus species in 3 to 5 percent, and gram-positive organisms, including group B Streptococcus and Staphylococcus aureus, in up to 10 percent of cases (Hill, 2005; Wing, 2000). The objective of this study was (1) to determine the reliability of urinalysis (UA) for predicting urinary tract infection (UTI) in febrile children, (2) to determine whether UA findings can predict Escherichia coli versus non-E. coli urinary tract infection, and (3) to determine if empiric antibiotics should be selected based on E. coli versus non-E. coli infection predictions. This was a retrospective chart review of children from 2 months to 2 years of age who presented to the emergency department with fever (rectal temperature >100.4°F) and had a positive urine culture. This study was conducted between January 2004 and December 2007. Negative UA was defined as urine white blood cell count less than 5 per high-power field, negative leukocyte esterase, and negative nitrites. Urine cultures were classified into E. coli and non-E. coli groups. These groups were compared for sex, race, and UA findings. Multivariate forward logistic regression, using the Wald test, was performed to calculate the likelihood ratio (LR) of each variable (eg, sex, race, UA parameters) in predicting UTI. In addition, antibiotic sensitivities between both groups were compared. Of 749 medical records reviewed, 608 were included; negative UA(-) was present in 183 cases, and positive UA(+) was observed in 425 cases. Furthermore, 424 cases were caused by E. coli, and 184 were due to non-E. coli organisms. Among 425 UA(+) cases, E. coli was identified in 349 (82.1%), whereas non-E. coli organisms were present in 76 (17.9%); in contrast, in 183 UA(-) cases, 108 (59%) were due to non-E. coli organisms versus 75 (41%), which were caused by E. coli. Urinalysis results were shown to be associated with organism group (P < 0.001). Positive leukocytes esterase had an LR of 2.5 (95% confidence interval [CI], 1.5-4.2), positive nitrites had an LR of 2.8 (95% CI, 1.4-5.5), and urine white blood cell count had an LR of 1.8 (95% CI, 1.3-2.4) in predicting E. coli versus non-E. coli infections. Antibiotic sensitivity compared between UA groups demonstrated equivalent superiority of cefazolin (94.7% sensitive in UA(+) vs 84.0% in UA(-) group; P < 0.0001), cefuroxime (98.2% vs 91.7%; P < 0.001), and nitrofurantoin (96.1% vs 82.2%; P < 0.0001) in the UA(+) group. In contrast, the UA(-) group showed significant sensitivity to trimethoprim-sulfamethoxazole (82.2% vs 71.3% in UA(+); P = 0.008). Urinalysis is not an accurate predictor of UTI. A positive urine culture in the presence of negative UA most likely grew non-E. coli organisms, whereas most UA(+) results were associated with E. coli. This study also highlighted local patterns of antibiotic resistance between E. coli and non-E. coli groups. Negative UA results in the presence of strong suspicion of a UTI suggest a non-E. coli organism, which may be best treated with trimethoprim-sulfamethoxazole. Conversely, UA(+) results suggest E. coli, which calls for treatment with cefazolin or cefuroxime. Urinary tract infection (UTI) is the most common serious bacterial infection in young infants. Signs and symptoms are often nonspecific. To describe clinical, demographic and laboratory features of UTI in infants ≤ 3 months old. Cross-sectional study of infants ≤ 3 months old with UTI diagnosed in a pediatric emergency department, for the period 2010-2012. UTI was defined as ≥ 50,000 colony-forming units per milliliter of a single uropathogen isolated from bladder catheterization. Paired urinalysis and urine culture from group culture-positive and group culture-negative were used to determine the sensitivity and specificity of pyuria and nitrite tests in detecting UTI. Of 519 urine cultures collected, UTI was diagnosed in 65 cases (prevalence: 12.5%); with male predominance (77%). The most common etiologies were Escherichia coli (56.9%), Klebsiella pneumoniae (18.5%) and Enterococcus faecalis (7.7%). Frequent clinical manifestations were fever (77.8%), irritability (41.4%) and vomiting (25.4%). The median temperature was 38.7°C. The sensitivity of the nitrite test was 30.8% (95%CI:19.9-43.4%), specificity of 100% (95%CI:99.2-100%). Pyuria ≥ 10,000/mL had a sensitivity of 87.7% (95%CI:77.2-94.5%), specificity of 74.9% (95%CI:70.6 -78.8%). The median peripheral white blood cell count was 13,150/mm3; C-reactive protein levels were normal in 30.5% of cases. The male: female ratio for urinary tract infection was 3.3:1. Non-Escherichia coli etiologies should be considered in empirical treatment. Fever was the main symptom. Positive nitrite is highly suggestive of UTI but has low sensitivity; whereas pyuria ≥ 10,000/mL revealed good sensitivity, but low specificity. Peripheral white blood cell count and C-reactive protein concentration have limited usefulness to suggest UTI. Diagnosis In straightforward cases, a diagnosis may be made and treatment given based on symptoms alone without further laboratory confirmation. In complicated or questionable cases, it may be useful to confirm the diagnosis via urinalysis, looking for the presence of urinary nitrites, white blood cells (leukocytes), or leukocyte esterase. Another test, urine microscopy, looks for the presence of red blood cells, white blood cells, or bacteria. Urine culture is deemed positive if it shows a bacterial colony count of greater than or equal to 103 colony-forming units per mL of a typical urinary tract organism. Antibiotic sensitivity can also be tested with these cultures, making them useful in the selection of antibiotic treatment. However, women with negative cultures may still improve with antibiotic treatment. As symptoms can be vague and without reliable tests for urinary tract infections, diagnosis can be difficult in the elderly. Septic arthritis of a lumbar facet joint is a rare condition. We report the case of a 77-year-old diabetic woman who developed fever and back pain 15 days after she had been diagnosed with a genitourinary infection for which she had received ciprofloxacin. Physical examination showed fever (38°C) and pain on pressure over the lower lumbar spinous vertebral apophyses and over the lower left paraspinal musculature. Investigations showed a white cell count of 8.4×10⁹/l, neutrophils 85.3%, erythrocyte sedimentation rate of 125 mm/h, and C-reactive protein of ≥9 mg/dl. Two blood cultures were both positive for Escherichia coli resistant to ciprofloxacin. There was no growth of pathogens from the urine cultures. Scintigraphy with gallium citrate Ga⁶⁷ showed vertical lower lumbar (L4-L5) radionuclide uptake lateralized to the left. Magnetic resonance imaging of the lumbar spine demonstrated signal changes and alteration of the structure at the left interapophyseal L4-L5 joint, an adjacent small collection of 1cm in diameter, and infiltration of the surrounding soft tissues, which extended to the epidural area, left conjunction hole, and paraspinal muscles. The patient was treated with intravenous cefotaxime and gentamicin and bed rest for 21 days, and recovered. This is the first report of interapophyseal arthritis caused by E. coli. Urinary nitrite test A nitrite test is a standard component of a urinary test strip. A positive test for nitrites in the urine is called nitrituria. This test is commonly used in diagnosing urinary tract infections (UTIs). A positive nitrite test indicates that the cause of the UTI is a gram negative organism, most commonly Escherichia coli. The reason for nitrites' existence in the presence of a UTI is due to a bacterial conversion of endogenous nitrates to nitrites. This may be a sign of infection. However, other parameters, such as leukocyte esterase, urine white blood cell count, and symptoms such as dysuria, urinary urgency, fevers, and chills must be correlated to diagnose an infection. Organisms causing UTI in pregnancy are the same uropathogens which commonly cause UTI in non-pregnant patients. Escherichia coli is the most common organism isolated. An 18-year retrospective analysis found E. coli to be the causative agent in 82.5% of cases of pyelonephritis in pregnant patients. [3] Other bacteria which may be seen include Klebsiella pneumoniae, Staphylococcus, Streptococcus, Proteus, and Enterococcus species. It is normal to find up to 3 (occasionally 5) leukocytes per high power field (40X) in a urine sample, with women having slightly higher results owing to vaginal contamination. Higher numbers indicate urinary infection. The urine test strip test for white blood cells detects leukocyte esterase, which is present in azurophilic granules of monocytes and granulocytes (neutrophilic, eosinophilic and basophilic). Bacteria, lymphocytes and epithelial cells from the genitourinary tract do not contain esterases. Neutrophil granulocytes are the leukocytes most commonly associated with urinary infections. A positive test for leukocyte esterase normally indicates the presence of bacteria and a positive nitrite test (although it is not always the case). Infections caused by Trichomonas, Chlamydia and yeasts produce leukocyturia without bacteriuria. The inflammation of the renal tissues (interstitial nephritis) can produce leukocyturia, in particular toxic interstitial nephritis with predominant Pathogenesis The bacteria that cause urinary tract infections typically enter the bladder via the urethra. However, infection may also occur via the blood or lymph. It is believed that the bacteria are usually transmitted to the urethra from the bowel, with females at greater risk due to their anatomy. After gaining entry to the bladder, E. Coli are able to attach to the bladder wall and form a biofilm that resists the body's immune response. Escherichia coli is the single most common microorganism, followed by Klebsiella and Proteus spp., to cause urinary tract infection. Klebsiella and Proteus spp., are frequently associated with stone disease. The presence of Gram positive bacteria such as Enterococcus and Staphylococcus increased. The increased resistance of urinary pathogens to quinolone antibiotics has been reported worldwide and might be the consequence of overuse and misuse of quinolones. Diagnosis We compare the test characteristics of urine dipstick and urinalysis at various test cutoff points in women presenting to emergency departments and an intermediate care center with symptoms of urinary tract infection. This was a prospective, observational study of adult women presenting to 1 of 2 community hospital EDs or an intermediate care center with dysuria, urgency, or urinary frequency on history, or suprapubic or costovertebral angle tenderness on examination. Patients who had taken antibiotics in the past 72 hours, had indwelling Foley catheters, symptomatic vaginal discharge, diabetes mellitus, immunodeficiency disorders, or were unable to provide a reliable history were excluded. The patient's clean-catch or catheterized urine specimen was tested immediately by a nurse using a Multistix 9 SG reagent strip. A second aliquot was sent within 1 hour of collection to the hospital laboratory, where a semiautomated microscopic urinalysis and a urine culture were performed. A positive urine culture was defined as more than 100,000 colonies of 1 or 2 uropathogenic bacteria per mL of urine at 48 hours. Dipstick and urinalysis data were compared with urine culture results. Sensitivity, specificity, and predictive values were calculated at various definitions of a positive test, or \"test cutoff points,\" for combinations of leukocyte esterase, nitrite, and blood on dipstick and for RBCs and WBCs on urinalyses. The probability of an erroneous decision to withhold treatment on the basis of a negative test result was defined as \"undertreatment,\" or 1 minus the negative predictive value. \"Overtreatment\" was defined as 1 minus the positive predictive value. Three hundred forty-three patients were enrolled in this study. Twelve patients were withdrawn because of missing laboratory results. Forty-six percent (152/331) of patients had positive urine cultures. If urine dipstick results are defined as positive when leukocyte esterase or nitrite is positive or blood is more than trace, the overtreatment rate is 47% (156/331) and the undertreatment rate is 13% (43/331). If urinalysis results are defined as positive when WBCs are more than 3 per high-power field or RBCs are more than 5 per high-power field, the overtreatment rate is 44% (146/331) and the undertreatment rate is 11% (36/331). Matched pairs of test characteristics were identified when the analysis was repeated using more than 10,000 colonies per mL as a positive culture. In this patient population, similar overtreatment and undertreatment rates were identified for various test cutoff points for urine dipstick tests and urinalysis. Although a urine dipstick may be equivalent to a urinalysis for the diagnosis of urinary tract infection, the limitations in the diagnostic accuracy of both tests should be incorporated into medical decisionmaking. Urinary tract infection (UTI) is one of the most common infections during pregnancy, which can lead to significant maternal and perinatal morbidity and mortality if left untreated. Challenges when treating UTIs in pregnancy include fetal protection and resistance development of uropathogens. Currently, the Essential Medicines List recommends nitrofurantoin to treat cystitis and ceftriaxone to treat pyelonephritis in pregnant women. To determine common pathogens causing UTI in pregnancy and their antibiotic susceptibility patterns. A retrospective analysis was performed of laboratory data for positive urine specimens from obstetric departments of 6 KwaZulu- Natal Province hospitals during 2011 - 2016. Identification and susceptibility testing were performed using the VITEK 2 system. Results were interpreted according to the breakpoints of the Clinical and Laboratory Standards Institute, USA. From 5 971 positive urine specimens, the most common isolate was Escherichia coli (n=3 236; 54.2%), followed by Klebsiella pneumoniae (n=770; 12.9%). Group B streptococcus (GBS) (n=239; 4.0%) and Enterococcus faecalis (n=251; 4.2%) were the most common Gram-positive pathogens. E. coli displayed significant resistance to trimethoprim-sulfamethoxazole (65.1%), cephalothin (38.3%), cefuroxime (27.3%), ciprofloxacin (16.9%) and amoxicillin-clavulanic acid (17.1%). Resistance to ceftriaxone and nitrofurantoin remained low ‒ 9.1% and 7.7%, respectively. Among Gram-positive pathogens, GBS displayed 100% penicillin susceptibility and E. faecalis showed 92.9% susceptibility to ampicillin. E. coli is unsurprisingly the most common cause of UTI in pregnancy in KwaZulu-Natal. Susceptibility to ceftriaxone and nitrofurantoin remains good. Among Gram positives, GBS is prevalent and susceptible to penicillin, while E. faecalis is susceptible to ampicillin. As antimicrobial resistance evolves, routine surveillance is necessary to modify recommended empirical antibiotic use.\nHere is the question:\nMICROBIOLOGY: Pregnant woman, 27 years old, 30 weeks of gestation. She comes to the emergency room because she noticed pain in the left lumbar region and dysuria since yesterday. She has no febrile sensation. She refers repeated urinary tract infections (UTI). Urinalysis shows Hb 3+, leukocytes 3+, nitrites 2+, sediment: 15-20 leukocytes per field and 5-10 red blood cells per field. Which of the following microorganisms is the most frequent culprit in pregnant women?\nHere are the potential choices:\n1. Escherichia coli.\n2. Enterococcus faecalis.\n3. Streptococcus agalactiae.\n4. Proteus mirabilis.\n5. Satphylococcus saprophyticus.\nThe correct answer is: 1. Escherichia coli." }, { "question": "You are a helpful medical expert, and your task is to answer a multi-choice medical question using the relevant documents. Please choose the answer from the provided options. Your responses will be used for research purposes only, so please have a definite answer.\nHere are the relevant documents:\n This study evaluated the real-world weight loss and glycemic outcomes of multidrug therapy (MDT) according to various combinations of metformin, sodium-glucose cotransporter -2 inhibitor (SGLT2i), glucagon-like peptide-1 receptor analogs (GLP1a), and orlistat in diabesity. Data retrospectively captured from medical records of 2 different centers in New Delhi for patients >35 years-age having prediabetes/diabetes and on at least any one of the 4 above medications with >6-months follow-up was analyzed. In total, 5,336 patient records were screened; 2,442 with prediabetes/diabetes were considered; 1,509 patients who fulfilled all criteria were analyzed. Use of metformin, SGLT2i, sulfonylureas, DPP4i, pioglitazone, orlistat, and GLP1a was 85.35%, 74.95%, 68.32%, 60%, 39.16%, 9.08%, and 4.17%, respectively. However, 365, 970, and 104 patients were on one of 4 concerned medications (Group-1; 24.18%), dual MDT (Group-2; 64.28%), and triple/quadruple MDT (Group-3; 6.89%). Metformin with SGLT2i was most commonly used dual MDT (94.12%). Analysis according to weight-loss quartiles from 558 patients showed 6.9 kg weight-loss in the highest quartile. People losing maximum weight were significantly younger; had higher use of metformin, SGLT2i, GLP1, orlistat, and lower pioglitazone use; greatest HbA1c reduction (-1.3 vs. -0.3; quartile-1 vs. quartile -4; 50 percent even after five losses (Table 18-4). TABLE 18-4. Predicted Success Rate of Subsequent Pregnancy According to Age and Number of Previous Miscarriages The differential diagnosis of third trimester bleeding can range from placenta abruptia to placenta previa to uterine rupture and the placenta accreta spectrum (PAS). However, patients with risk factors such as multiple cesarean sections (c-sections), advanced maternal age (AMA), grand multiparity, and single-layer uterine closure are at greater risk of developing these complications earlier than we would traditionally expect. This case recounts a 38-year-old gravida 6 preterm 3 term 1 abortus 1 live 4 (G6P3114) at 23 weeks and five days gestational age (GA) with a past medical history of preterm pregnancy, pre-eclampsia, chronic abruptia, three previous c-sections, and low-lying placenta who presented to the emergency department (ED) with vaginal bleeding. Initial workup revealed placenta accreta and possible percreta. The patient was placed on intramuscular (IM) corticosteroids in anticipation of preterm delivery. As soon as the patient was stable, she was discharged home. She presented to a different hospital the next day with the same complaints. Imaging was consistent with accreta and her presentation with abruption. During the hospital stay, the patient went into threatened preterm labor (PTL). At first, we suspected preterm premature rupture of membranes (PPROM) due to apparent pooling of amniotic fluid in the vaginal canal. Upon further work up, the diagnosis was consistent with chronic abruption oligohydramnios sequence (CAOS). Before this could be investigated, her hospital course was complicated by acute abruption and Category III/nonreassuring fetal heart rate (FHR) tracing. The patient underwent an emergency c-section at 26 weeks GA as well as a planned supracervical hysterectomy for desired permanent sterilization. During the operation, the patient suffered a postpartum hemorrhage (PPH) of 4500 mL. She was later discharged home on postoperative day (POD) eight. What is it? This question arose early this year during a discussion with my doula partner, who lived in Great Britain for 20 years, and one of the midwives I work with, who attends only homebirths, which is very rare in France. The mom whose case we were discussing was late going into labor but not postdated according to the official pregnancy term here in France (37-42 WA--\"weeks of amenorrhea\"). The midwife expressed her discomfort with waiting. My doula partner and I felt differently, but we knew we were influenced by American and British midwives' practices. I had been shocked the previous December by the position of the chief of the maternity department in a private hospital (a small unit with no residents, in which nurse-midwives attend \"normal\" births and obstetricians are called in case of complications only). This OB explained to my client and me that if she didn't go into labor naturally, she would be called at 41 + 1 for a vaginal exam to check her cervix and would be induced at 41 + 2. Waiting until 42 weeks requires daily checks, for which he has neither the room nor personnel. He clearly stated that it was a matter of management of time and finances. A month later one of our clients reported the story of her brother and sister-in-law's planned homebirth in London. Their doctors had put a lot of pressure on the mother during her pregnancy with gestational diabetes regarding her length of term. They started to talk about induction. The parents didn't feel comfortable with this, and at that point our client had asked me to refer them to someone who could help them there. We referred them to the National Childbirth Trust and to the sweetest doula we know there. This doula (Liliana Lammers) and her famous doctor partner (Dr. Michel Odent) were a good match. The doctor advised waiting, on the condition that the health of the baby and the amount of fluid be checked daily at the local hospital. The mother had already been waiting several weeks past what was supposed to be her term. Finally, she went naturally into labor at home. The doctor and the doula came and, after some hours of observation, decided it would be wiser for the mother to deliver in the hospital. The doctor and doula were not comfortable with the prolonged prelabor, when, at nearly 44 weeks, the health of the baby and the amount of fluid had not been checked for five days. The mother finally had a vaginal birth without drugs at the hospital. After hearing this story, I suggested it would be interesting to collect the official lengths of term and the different routines in other countries as a learning tool and in order to give us something other than French protocol on which to base our practice. So I sent the question to every midwife for whom I had an e-mail address from the last Midwifery Today conference in Paris. Beyond this motivation was my own curiosity regarding the relationship between the official term in each country and its uses and routines. The most significant (because the most unique) answer, in my opinion, is from The Netherlands, where physiology is a priority. Data is now more than ever available to inform couples at risk of second trimester miscarriage or preterm delivery. We are able to give customized information according to the obstetrical history and to the evolution of the cervix during the second trimester although the level of scientific evidence is limited or poor. Elective cerclage can be proposed to patients with a history of at least 3 second trimester miscarriages or preterm deliveries. There is no clear consensus on which patients could benefit from therapeutic cerclage. Indications would have to be motivated by a short cervix on ultrasound measurements and the cerclage performed before 24 weeks of gestation. A diagnosis of cervical cancer during pregnancy poses difficult management and ethical problems. Survival of the patient is the foremost concern, but fetal viability and well-being must also be addressed. Radical trachelectomy (RT) has recently begun to be performed as a possible treatment modality for early stage invasive uterine cervical cancer in pregnant patients who would like to continue their pregnancy. A 32-year-old Japanese woman visited a local hospital for prenatal care, and was diagnosed with a FIGO I B1 adenocarcinoma of the uterine cervix. She had a strong desire to avoid pregnancy termination, so she was admitted to our hospital for fertility-preserving surgery. After extensive counseling, vaginal radical trachelectomy with abdominal pelvic lymphadenectomy was performed in the 16th gestational week. The excised uterine cervix and lymph nodes were pathologically negative for cancer. To maintain her pregnancy, daily vaginal disinfection with povidone iodine, bed rest, and administration of ritodrine and an ulinastatin vaginal suppository were continued until the delivery. At 34 weeks' gestation, an emergency cesarean section was performed because of sudden premature rupture of the membranes. A baby girl was born weighing 2112 g, with Apgar score of 8/9. The mother remains without evidence of recurrence at the time of this report. This is the first case of successful pregnancy and delivery in Japan after vaginal RT. Prevention of preterm birth remains an elusive goal. Still, may be achievable. Of options, cerclage placement may be used to prevent pre term birth in at least three circumstances. First, the procedure may beneit women who have a history of recurrent midtrimester losses and who are diagnosed with cervical insuiciency. A second instance is the woman identiied during sonographic examination to have a short cervix. The third indication is a \"rescue\" cerclage, done emergently when cervical incompetence is recognized in women with threatened preterm labor. A 26-year-old primigravid woman presented with a dichorionic diamniotic twin pregnancy after 7 years of infertility. No formal ultrasound was performed until a morphology check at 19 weeks and 4 days of gestation, at which time a shortened cervix was identified. The patient was already on vaginal progesterone pessaries from conception, as per her infertility specialist, and was advised to change to a rectal route of administration. At 20 weeks and 5 days, progesterone pessaries were increased to twice daily. A repeat scan at 21 weeks and 4 days showed a funnelled cervix 29 mm in length, a closed portion of 4-6 mm and bulging membranes. A speculum examination at this time showed a shortened cervix, 5 mm open, with visible membranes. A cervical cerclage was placed at 21 weeks and 5 days. The patient was given oral antibiotics for 1 week and was continued on progesterone pessaries. The patient was managed through the twins clinic and had serial ultrasound scans throughout the pregnancy. She went on to develop gestational diabetes and pre-eclampsia. She had a caesarean section at 33 weeks and 4 days due to pre-eclampsia, with abnormal doppler scans. Cervical cerclage was removed at the time of the caesarean section. Both twins were admitted to the nursery for prematurity and progressed well. This case report illustrates how a cervical cerclage can be utilised successfully in a primigravid dichorionic diamniotic twin pregnancy. Case Study  Sarah is a 58-year-old breast cancer survivor, social worker, and health-care administrator at a long-term care facility. She lives with her husband and enjoys gardening and reading. She has two grown children and three grandchildren who live approximately 180 miles away. SECOND CANCER DIAGNOSIS  One morning while showering, Sarah detected a painless quarter-sized lump on her inner thigh. While she thought it was unusual, she felt it would probably go away. One month later, she felt the lump again; she thought that it had grown, so she scheduled a visit with her primary care physician. A CT scan revealed a 6.2-cm soft-tissue mass in the left groin. She was referred to an oncologic surgeon and underwent an excision of the groin mass. Pathology revealed a grade 3 malignant melanoma. She was later tested and found to have BRAF-negative status. Following her recovery from surgery, Sarah was further evaluated with an MRI scan of the brain, which was negative, and a PET scan, which revealed two nodules in the left lung. As Sarah had attended a cancer support group during her breast cancer treatment in the past, she decided to go back to the group when she learned of her melanoma diagnosis. While the treatment options for her lung lesions included interleukin-2, ipilimumab (Yervoy), temozolomide, dacarbazine, a clinical trial, or radiosurgery, Sarah's oncologist felt that ipilimumab or radiosurgery would be the best course of action. She shared with her support group that she was ambivalent about this decision, as she had experienced profound fatigue and nausea with chemotherapy during her past treatment for breast cancer. She eventually opted to undergo stereotactic radiosurgery. DISEASE RECURRENCE  After the radiosurgery, Sarah was followed every 2 months. She complained of shortness of breath about 2 weeks prior to each follow-up visit. Each time her chest x-ray was normal, and she eventually believed that her breathlessness was anxiety-related. Unfortunately, Sarah's 1-year follow-up exam revealed a 2 cm × 3 cm mass in her left lung, for which she had a surgical wedge resection. Her complaints of shortness of breath increased following the surgery and occurred most often with anxiety, heat, and gardening activities, especially when she needed to bend over. Sarah also complained of a burning \"pins and needles\" sensation at the surgical chest wall site that was bothersome and would wake her up at night. Sarah met with the nurse practitioner in the symptom management clinic to discuss her concerns. Upon physical examination, observable signs of breathlessness were lacking, and oxygen saturation remained stable at 94%, but Sarah rated her breathlessness as 7 on the 0 to 10 Borg scale. The nurse practitioner prescribed duloxetine to help manage the surgical site neuropathic pain and to assist with anxiety, which in turn could possibly improve Sarah's breathlessness. Several nonpharmacologic modalities for breathlessness were also recommended: using a fan directed toward her face, working in the garden in the early morning when the weather is cooler, gardening in containers that are at eye level to avoid the need to bend down, and performing relaxation exercises with pursed lip breathing to relieve anxiety-provoked breathlessness. One month later, Sarah reported relief of her anxiety; she stated that the fan directed toward her face helped most when she started to feel \"air hungry.\" She rated her breathlessness at 4/10 on the Borg scale. SECOND RECURRENCE: MULTIPLE PULMONARY NODULES  Sarah's chest x-rays remained clear for 6 months, but she developed a chronic cough shortly before the 9-month exam. An x-ray revealed several bilateral lung lesions and growth in the area of the previously resected lung nodule. Systemic therapy was recommended, and she underwent two cycles of ipilimumab. Sarah's cough and breathlessness worsened, she developed colitis, and she decided to stop therapy after the third cycle. In addition, her coughing spells triggered bronchospasms that resulted in severe anxiety, panic attacks, and air hunger. She rated her breathlessness at 10/10 on the Borg scale during these episodes. She found communication difficult due to the cough and began to isolate herself. She continued to attend the support group weekly but had difficulty participating in conversation due to her cough. Sarah was seen in the symptom management clinic every 2 weeks or more often as needed. No acute distress was present at the beginning of each visit, but when Sarah began to talk about her symptoms and fear of dying, her shortness of breath and anxiety increased. The symptom management nurse practitioner treated the suspected underlying cause of the breathlessness and prescribed oral lorazepam (0.5 to 1 mg every 6 hours) for anxiety and codeine cough syrup for the cough. Opioids were initiated for chest wall pain and to control the breathlessness. Controlled-release oxycodone was started at 10 mg every 12 hours with a breakthrough pain (BTP) dose of 5 mg every 2 hours as needed for breathlessness or pain. Sarah noted improvement in her symptoms and reported a Borg scale rating of 5/10. Oxygen therapy was attempted, but subjective improvement in Sarah's breathlessness was lacking. END OF LIFE  Sarah's disease progressed to the liver, and she began experiencing more notable signs of breathlessness: nasal flaring, tachycardia, and restlessness. Opioid doses were titrated over the course of 3 months to oxycodone (40 mg every 12 hours) with a BTP dose of 10 to 15 mg every 2 hours as needed, but her breathlessness caused significant distress, which she rated 8/10. The oxycodone was rotated to IV morphine continuous infusion with patient-controlled analgesia (PCA) that was delivered through her implantable port. This combination allowed Sarah to depress the PCA as needed and achieve immediate control of her dyspneic episodes. Oral lorazepam was also continued as needed. Sarah's daughter moved home to take care of her mother, and hospice became involved for end-of-life care. As Sarah became less responsive, nurses maintained doses of morphine for control of pain and breathlessness and used a respiratory distress observation scale to assess for breathlessness since Sarah could no longer self-report. A bolus PCA dose of morphine was administered by Sarah's daughter if her mother appeared to be in distress. Sarah died peacefully in her home without signs of distress. Cervical cerclage has always been the main treatment option in cases of so-called cervical insufficiency, a condition that is notoriously associated with a high risk of second trimester abortion and/or preterm delivery. We can distinguish between a prophylactic cerclage, to be performed electively, usually at 13-16 weeks gestation, only when the woman has a history extremely suggestive for cervical incompetence (3 or more mid-trimester abortions or preterm deliveries) and a therapeutic cerclage. This last cerclage is recommended either for women who have ultrasonographic changes consistent with a short cervix or the presence of funneling after the 16-20 weeks gestation (urgent cerclage) and for women who present the asymptomatic dilation of the uterine cervix of at least 2 cm and/or a prolapse of the amniochorial membranes (emergent cerclage). So far there is still a lack of controlled and randomized trials that can unquestionably demonstrate the advantages of the cervical cerclage in comparison with a ''wait and see'' aptitude. The cerclage can be performed either transvaginally, usually according to the McDonald technique, or transabdominally. This last approach is recommended when a transvaginal cerclage has to be avoided because of technical difficulties depending on the conditions of the cervix or when the pregnant woman has a history of one or more failed transvaginal cerclages. Interesting perspectives are currently offered by the laparoscopic cerclage, a method that has been effective and unexpectedly safe till now. Healthcare workers (HCWs) are at an increased risk for exposure to infections. 50 percent even after five losses (Table 18-4). TABLE 18-4. Predicted Success Rate of Subsequent Pregnancy According to Age and Number of Previous Miscarriages The differential diagnosis of third trimester bleeding can range from placenta abruptia to placenta previa to uterine rupture and the placenta accreta spectrum (PAS). However, patients with risk factors such as multiple cesarean sections (c-sections), advanced maternal age (AMA), grand multiparity, and single-layer uterine closure are at greater risk of developing these complications earlier than we would traditionally expect. This case recounts a 38-year-old gravida 6 preterm 3 term 1 abortus 1 live 4 (G6P3114) at 23 weeks and five days gestational age (GA) with a past medical history of preterm pregnancy, pre-eclampsia, chronic abruptia, three previous c-sections, and low-lying placenta who presented to the emergency department (ED) with vaginal bleeding. Initial workup revealed placenta accreta and possible percreta. The patient was placed on intramuscular (IM) corticosteroids in anticipation of preterm delivery. As soon as the patient was stable, she was discharged home. She presented to a different hospital the next day with the same complaints. Imaging was consistent with accreta and her presentation with abruption. During the hospital stay, the patient went into threatened preterm labor (PTL). At first, we suspected preterm premature rupture of membranes (PPROM) due to apparent pooling of amniotic fluid in the vaginal canal. Upon further work up, the diagnosis was consistent with chronic abruption oligohydramnios sequence (CAOS). Before this could be investigated, her hospital course was complicated by acute abruption and Category III/nonreassuring fetal heart rate (FHR) tracing. The patient underwent an emergency c-section at 26 weeks GA as well as a planned supracervical hysterectomy for desired permanent sterilization. During the operation, the patient suffered a postpartum hemorrhage (PPH) of 4500 mL. She was later discharged home on postoperative day (POD) eight. What is it? This question arose early this year during a discussion with my doula partner, who lived in Great Britain for 20 years, and one of the midwives I work with, who attends only homebirths, which is very rare in France. The mom whose case we were discussing was late going into labor but not postdated according to the official pregnancy term here in France (37-42 WA--\"weeks of amenorrhea\"). The midwife expressed her discomfort with waiting. My doula partner and I felt differently, but we knew we were influenced by American and British midwives' practices. I had been shocked the previous December by the position of the chief of the maternity department in a private hospital (a small unit with no residents, in which nurse-midwives attend \"normal\" births and obstetricians are called in case of complications only). This OB explained to my client and me that if she didn't go into labor naturally, she would be called at 41 + 1 for a vaginal exam to check her cervix and would be induced at 41 + 2. Waiting until 42 weeks requires daily checks, for which he has neither the room nor personnel. He clearly stated that it was a matter of management of time and finances. A month later one of our clients reported the story of her brother and sister-in-law's planned homebirth in London. Their doctors had put a lot of pressure on the mother during her pregnancy with gestational diabetes regarding her length of term. They started to talk about induction. The parents didn't feel comfortable with this, and at that point our client had asked me to refer them to someone who could help them there. We referred them to the National Childbirth Trust and to the sweetest doula we know there. This doula (Liliana Lammers) and her famous doctor partner (Dr. Michel Odent) were a good match. The doctor advised waiting, on the condition that the health of the baby and the amount of fluid be checked daily at the local hospital. The mother had already been waiting several weeks past what was supposed to be her term. Finally, she went naturally into labor at home. The doctor and the doula came and, after some hours of observation, decided it would be wiser for the mother to deliver in the hospital. The doctor and doula were not comfortable with the prolonged prelabor, when, at nearly 44 weeks, the health of the baby and the amount of fluid had not been checked for five days. The mother finally had a vaginal birth without drugs at the hospital. After hearing this story, I suggested it would be interesting to collect the official lengths of term and the different routines in other countries as a learning tool and in order to give us something other than French protocol on which to base our practice. So I sent the question to every midwife for whom I had an e-mail address from the last Midwifery Today conference in Paris. Beyond this motivation was my own curiosity regarding the relationship between the official term in each country and its uses and routines. The most significant (because the most unique) answer, in my opinion, is from The Netherlands, where physiology is a priority. Data is now more than ever available to inform couples at risk of second trimester miscarriage or preterm delivery. We are able to give customized information according to the obstetrical history and to the evolution of the cervix during the second trimester although the level of scientific evidence is limited or poor. Elective cerclage can be proposed to patients with a history of at least 3 second trimester miscarriages or preterm deliveries. There is no clear consensus on which patients could benefit from therapeutic cerclage. Indications would have to be motivated by a short cervix on ultrasound measurements and the cerclage performed before 24 weeks of gestation. A diagnosis of cervical cancer during pregnancy poses difficult management and ethical problems. Survival of the patient is the foremost concern, but fetal viability and well-being must also be addressed. Radical trachelectomy (RT) has recently begun to be performed as a possible treatment modality for early stage invasive uterine cervical cancer in pregnant patients who would like to continue their pregnancy. A 32-year-old Japanese woman visited a local hospital for prenatal care, and was diagnosed with a FIGO I B1 adenocarcinoma of the uterine cervix. She had a strong desire to avoid pregnancy termination, so she was admitted to our hospital for fertility-preserving surgery. After extensive counseling, vaginal radical trachelectomy with abdominal pelvic lymphadenectomy was performed in the 16th gestational week. The excised uterine cervix and lymph nodes were pathologically negative for cancer. To maintain her pregnancy, daily vaginal disinfection with povidone iodine, bed rest, and administration of ritodrine and an ulinastatin vaginal suppository were continued until the delivery. At 34 weeks' gestation, an emergency cesarean section was performed because of sudden premature rupture of the membranes. A baby girl was born weighing 2112 g, with Apgar score of 8/9. The mother remains without evidence of recurrence at the time of this report. This is the first case of successful pregnancy and delivery in Japan after vaginal RT. Prevention of preterm birth remains an elusive goal. Still, may be achievable. Of options, cerclage placement may be used to prevent pre term birth in at least three circumstances. First, the procedure may beneit women who have a history of recurrent midtrimester losses and who are diagnosed with cervical insuiciency. A second instance is the woman identiied during sonographic examination to have a short cervix. The third indication is a \"rescue\" cerclage, done emergently when cervical incompetence is recognized in women with threatened preterm labor. A 26-year-old primigravid woman presented with a dichorionic diamniotic twin pregnancy after 7 years of infertility. No formal ultrasound was performed until a morphology check at 19 weeks and 4 days of gestation, at which time a shortened cervix was identified. The patient was already on vaginal progesterone pessaries from conception, as per her infertility specialist, and was advised to change to a rectal route of administration. At 20 weeks and 5 days, progesterone pessaries were increased to twice daily. A repeat scan at 21 weeks and 4 days showed a funnelled cervix 29 mm in length, a closed portion of 4-6 mm and bulging membranes. A speculum examination at this time showed a shortened cervix, 5 mm open, with visible membranes. A cervical cerclage was placed at 21 weeks and 5 days. The patient was given oral antibiotics for 1 week and was continued on progesterone pessaries. The patient was managed through the twins clinic and had serial ultrasound scans throughout the pregnancy. She went on to develop gestational diabetes and pre-eclampsia. She had a caesarean section at 33 weeks and 4 days due to pre-eclampsia, with abnormal doppler scans. Cervical cerclage was removed at the time of the caesarean section. Both twins were admitted to the nursery for prematurity and progressed well. This case report illustrates how a cervical cerclage can be utilised successfully in a primigravid dichorionic diamniotic twin pregnancy. Case Study  Sarah is a 58-year-old breast cancer survivor, social worker, and health-care administrator at a long-term care facility. She lives with her husband and enjoys gardening and reading. She has two grown children and three grandchildren who live approximately 180 miles away. SECOND CANCER DIAGNOSIS  One morning while showering, Sarah detected a painless quarter-sized lump on her inner thigh. While she thought it was unusual, she felt it would probably go away. One month later, she felt the lump again; she thought that it had grown, so she scheduled a visit with her primary care physician. A CT scan revealed a 6.2-cm soft-tissue mass in the left groin. She was referred to an oncologic surgeon and underwent an excision of the groin mass. Pathology revealed a grade 3 malignant melanoma. She was later tested and found to have BRAF-negative status. Following her recovery from surgery, Sarah was further evaluated with an MRI scan of the brain, which was negative, and a PET scan, which revealed two nodules in the left lung. As Sarah had attended a cancer support group during her breast cancer treatment in the past, she decided to go back to the group when she learned of her melanoma diagnosis. While the treatment options for her lung lesions included interleukin-2, ipilimumab (Yervoy), temozolomide, dacarbazine, a clinical trial, or radiosurgery, Sarah's oncologist felt that ipilimumab or radiosurgery would be the best course of action. She shared with her support group that she was ambivalent about this decision, as she had experienced profound fatigue and nausea with chemotherapy during her past treatment for breast cancer. She eventually opted to undergo stereotactic radiosurgery. DISEASE RECURRENCE  After the radiosurgery, Sarah was followed every 2 months. She complained of shortness of breath about 2 weeks prior to each follow-up visit. Each time her chest x-ray was normal, and she eventually believed that her breathlessness was anxiety-related. Unfortunately, Sarah's 1-year follow-up exam revealed a 2 cm × 3 cm mass in her left lung, for which she had a surgical wedge resection. Her complaints of shortness of breath increased following the surgery and occurred most often with anxiety, heat, and gardening activities, especially when she needed to bend over. Sarah also complained of a burning \"pins and needles\" sensation at the surgical chest wall site that was bothersome and would wake her up at night. Sarah met with the nurse practitioner in the symptom management clinic to discuss her concerns. Upon physical examination, observable signs of breathlessness were lacking, and oxygen saturation remained stable at 94%, but Sarah rated her breathlessness as 7 on the 0 to 10 Borg scale. The nurse practitioner prescribed duloxetine to help manage the surgical site neuropathic pain and to assist with anxiety, which in turn could possibly improve Sarah's breathlessness. Several nonpharmacologic modalities for breathlessness were also recommended: using a fan directed toward her face, working in the garden in the early morning when the weather is cooler, gardening in containers that are at eye level to avoid the need to bend down, and performing relaxation exercises with pursed lip breathing to relieve anxiety-provoked breathlessness. One month later, Sarah reported relief of her anxiety; she stated that the fan directed toward her face helped most when she started to feel \"air hungry.\" She rated her breathlessness at 4/10 on the Borg scale. SECOND RECURRENCE: MULTIPLE PULMONARY NODULES  Sarah's chest x-rays remained clear for 6 months, but she developed a chronic cough shortly before the 9-month exam. An x-ray revealed several bilateral lung lesions and growth in the area of the previously resected lung nodule. Systemic therapy was recommended, and she underwent two cycles of ipilimumab. Sarah's cough and breathlessness worsened, she developed colitis, and she decided to stop therapy after the third cycle. In addition, her coughing spells triggered bronchospasms that resulted in severe anxiety, panic attacks, and air hunger. She rated her breathlessness at 10/10 on the Borg scale during these episodes. She found communication difficult due to the cough and began to isolate herself. She continued to attend the support group weekly but had difficulty participating in conversation due to her cough. Sarah was seen in the symptom management clinic every 2 weeks or more often as needed. No acute distress was present at the beginning of each visit, but when Sarah began to talk about her symptoms and fear of dying, her shortness of breath and anxiety increased. The symptom management nurse practitioner treated the suspected underlying cause of the breathlessness and prescribed oral lorazepam (0.5 to 1 mg every 6 hours) for anxiety and codeine cough syrup for the cough. Opioids were initiated for chest wall pain and to control the breathlessness. Controlled-release oxycodone was started at 10 mg every 12 hours with a breakthrough pain (BTP) dose of 5 mg every 2 hours as needed for breathlessness or pain. Sarah noted improvement in her symptoms and reported a Borg scale rating of 5/10. Oxygen therapy was attempted, but subjective improvement in Sarah's breathlessness was lacking. END OF LIFE  Sarah's disease progressed to the liver, and she began experiencing more notable signs of breathlessness: nasal flaring, tachycardia, and restlessness. Opioid doses were titrated over the course of 3 months to oxycodone (40 mg every 12 hours) with a BTP dose of 10 to 15 mg every 2 hours as needed, but her breathlessness caused significant distress, which she rated 8/10. The oxycodone was rotated to IV morphine continuous infusion with patient-controlled analgesia (PCA) that was delivered through her implantable port. This combination allowed Sarah to depress the PCA as needed and achieve immediate control of her dyspneic episodes. Oral lorazepam was also continued as needed. Sarah's daughter moved home to take care of her mother, and hospice became involved for end-of-life care. As Sarah became less responsive, nurses maintained doses of morphine for control of pain and breathlessness and used a respiratory distress observation scale to assess for breathlessness since Sarah could no longer self-report. A bolus PCA dose of morphine was administered by Sarah's daughter if her mother appeared to be in distress. Sarah died peacefully in her home without signs of distress. Cervical cerclage has always been the main treatment option in cases of so-called cervical insufficiency, a condition that is notoriously associated with a high risk of second trimester abortion and/or preterm delivery. We can distinguish between a prophylactic cerclage, to be performed electively, usually at 13-16 weeks gestation, only when the woman has a history extremely suggestive for cervical incompetence (3 or more mid-trimester abortions or preterm deliveries) and a therapeutic cerclage. This last cerclage is recommended either for women who have ultrasonographic changes consistent with a short cervix or the presence of funneling after the 16-20 weeks gestation (urgent cerclage) and for women who present the asymptomatic dilation of the uterine cervix of at least 2 cm and/or a prolapse of the amniochorial membranes (emergent cerclage). So far there is still a lack of controlled and randomized trials that can unquestionably demonstrate the advantages of the cervical cerclage in comparison with a ''wait and see'' aptitude. The cerclage can be performed either transvaginally, usually according to the McDonald technique, or transabdominally. This last approach is recommended when a transvaginal cerclage has to be avoided because of technical difficulties depending on the conditions of the cervix or when the pregnant woman has a history of one or more failed transvaginal cerclages. Interesting perspectives are currently offered by the laparoscopic cerclage, a method that has been effective and unexpectedly safe till now. Healthcare workers (HCWs) are at an increased risk for exposure to infections. 50 years of age, immunocompromised, or with preexisting malignancy. If SAH is suspected, obtain a head CT without contrast. If CT is , LP is mandatory. Obtain a CBC. Optic neuritis is a common cause of subacute unilateral vision loss, occurring in 1-5 per 100,000 persons per year. It is more common in Caucasians, women, and those from countries with northern latitudes. Those aged 20-49 years are at greatest risk. The condition arises due to inflammation of the optic nerve. Inflammation may occur due to systemic inflammatory disorders, most commonly multiple sclerosis. A 21-year-old African-American male presented to our emergency department with a complaint of painful unilateral vision loss. On examination he was found to have a relative afferent pupillary defect and red desaturation. A bedside ultrasound suggested pseudopapilledema suggestive of optic neuritis. He was admitted to Neurology for confirmation of and treatment for optic neuritis. Magnetic resonance imaging confirmed optic neuritis. The patient was treated with i.v. steroids and discharged after improvement in visual function. WHY SHOULD AN EMERGENCY PHYSICIAN BE AWARE OF THIS?: Optic neuritis is a clinical diagnosis. The subtle historical components and examination findings make it a diagnostic challenge for the busy emergency physician. Early diagnosis may improve visual outcomes. Discovery of pseudopapilledema on bedside ultrasound may be seen in optic neuritis, and is another finding that emergency physicians may assess for in patient presenting with unilateral vision loss. Giant cell arteritis (GCA) is a polysymptomatic disease which constitutes an ophthalmic emergency because early recognition and management can prevent blindness. There is conflicting information in the literature on the validity, sensitivity, and specificity of various systemic symptoms and signs of GCA. This paper presents a review of our prospective studies on the subject, and our findings are particularly relevant to dentists. We investigated 363 patients in a prospective study. Positive temporal artery biopsy was seen in 106 patients and negative in 257 referred for diagnosis of GCA. Systemic symptoms and signs of GCA and erythrocyte sedimentation rate (Westergren-ESR) and C-reactive protein (CRP) levels were compared in these two groups of patients. The odds of having a positive temporal artery biopsy (i.e., GCA) were 9.1 times greater with jaw claudication (pain in masticatory muscles on eating), 3.4 times with neck pain, 3.2 times with CRP > 2.45 mg/dL, 2.0 times with ESR 47.107 mm/hr, 2.7 times with ESR > 107 mm/hr, and 2.0 times when the patients were aged > or = 75 years. Other signs and symptoms did not show a significant association with a positive biopsy. Our study showed that \"normal\" ESR values do not rule out GCA but that CRP is a more useful test than ESR. Since jaw claudication is one of the most important symptoms of GCA, dentists should keep this possibility in mind when older patients come complaining of jaw pain while eating. To report the case and multimodal imaging findings of a healthy young woman who developed paracentral acute middle maculopathy (PAMM) 9 weeks after COVID-19 disease. Case report. Ultra-widefield fundus photography, macular spectral-domain optical coherence tomography (SD-OCT), fluorescein angiography (FA), and OCT-angiography (OCT-A) were performed. A 36-year-old woman who developed PAMM 9 weeks after SARS-CoV-2 infection. A 36-year-old woman went to the emergency department (ED) with sudden, painless, left eye (LE) vision loss. The only relevant past medical history was COVID-19 disease 9 weeks before. Best corrected visual acuity (BCVA) was 20/200, a LE relative afferent pupillary defect (RAPD) was present and superficial hemorrhages throughout the macular area and peripheral retina were found. Nearly four hours after admission, LE BCVA recovered to 20/20 without RAPD. Five days after presentation in the ED, the patient returned with recurrent LE vision loss, with spontaneous recovery within 12 hours. Macular SD-OCT revealed hyperreflectivity of the inner plexiform and inner nuclear layers and the diagnosis of PAMM was established. The patient started oral acetylsalicylic acid and oral prednisolone. The patient did not report any new episodes of vision loss and there was a progressive resolution of abnormal fundus findings. SARS-CoV-2 infection increases the risk of vascular thrombotic events with possible involvement of the retinal circulation, and PAMM may present as a possible complication. Ophthalmologists should be able to recognize it promptly through multimodal imaging findings. In anterior ischemic optic neuropathy (AION), it is important not to miss the diagnosis of giant cell arteritis (GCA) because this requires immediate steroid treatment to prevent involvement of the second eye and possible blindness. A missed diagnosis also might lead to fatal systemic complications. Observational case report. A 79-year-old woman noticed decreased visual and visual field loss in the right eye. At presentation, right visual acuity was 10/20 (ETDRS chart 2000). There was a right relative afferent pupillary defect of 0.6 log units. Asked for symptoms of GCA she complained about temporal and occipital headache, jaw claudication combined with malaise, and myalgia of the upper limbs. Laboratory tests showed normal inflammatory markers. Repeated tests confirmed ESR and CRP to be within the normal range. GCA being suspected, ultrasound of the superficial temporal arteries and temporal artery biopsy were performed unilaterally on the right side. Histology showed a chronic inflammatory cell infiltrate consistent with active GCA. The patient was treated with high-dose corticosteroids (250 mg methylprednisolone, three times/day, initially) and symptoms rapidly resolved, but visual loss remained unchanged. The case presented here proves that GCA with typical related visual loss (AION) is possible even when both ESR and CRP are in the normal range. Therefore, in the presence of typical symptoms, the clinician must not rely solely on laboratory testing, but start steroid therapy immediately and order a temporal artery biopsy. Cancer should be suspected if there is previous history of it, unexplained weight loss, or unremitting pain. Spinal epidural abscess is more common among those with diabetes mellitus or immunocompromised or who had spinal surgery, injection or catheter; it typically causes fever, leukocytosis and increased erythrocyte sedimentation rate. If cancer or spinal epidural abscess are suspected, urgent magnetic resonance imaging is recommended for confirmation. Proximal diabetic neuropathy typically affects middle aged and older people with well-controlled type-2 diabetes mellitus; onset is sudden causing pain usually in multiple dermatomes quickly followed by weakness. Diagnosis typically involves electromyography and lumbar puncture. Shingles is more common among the elderly and immunocompromised; usually (but not always) pain is followed by appearance of a rash with small blisters along a single dermatome. Acute Lyme radiculopathy may follow a history of outdoor activities during warmer Giant cell arteritis (GCA) is often diagnosed very late, variable \"facets\" of the disease exist render the diagnosis more difficult. Follow-up observations of five very old patients are reported in whom diagnosis was made too late, resulting in blindness of both eyes. Five patients (age 76-84 years, 4 women, one man) with GCA became blind in both eyes because diagnosis had been delayed (two patients) or onset of therapy was too late (three patients). In two patients who also had arterial hypertension, the symptom \"headache\" had been misleading. Symptoms of accompanying general diseases masked the real diagnosis, particularly in the second patient who had renal insufficiency, coronary artery disease, and unilateral obstruction of the internal carotid artery. Symptoms that failed to lead to the correct diagnosis were: muscle or chewing pain (three patients), circumscribed numbness around the mouth (second patient), and persistent headache despite normalization of blood pressure. Normal findings from cranial CTAs (two patients) led to the wrong reassurance of the patient. Swelling of the optic disk (two patients) was misdiagnosed by ophthalmologists, as was a retinal branch arterial occlusion (first patient). Three patients, afraid of possible side effects caused by glucocorticoids, took ineffective alternative medications. Poor vigilance led to blindness of the fifth patient with long-standing polymyalgia rheumatica. Targeted examinations at the onset of symptoms are necessary. GCA-symptoms were mis-constructed by additional diseases that disguised the correct diagnosis. The danger of bilateral blindness is particularly great in patients of great age. A 70-year-old woman presented with new onset of left eye and facial pain. Ophthalmic and neurological examinations, magnetic resonance imaging brain, erythrocyte sedimentation rate, and C-reactive protein were unrevealing. A few days later, she developed vision loss in her left eye. Examination revealed decreased visual acuity with a relative afferent pupillary defect in the left eye and a diffuse mild swelling of the left optic nerve head. Repeat magnetic resonance imaging showed T2 hyperintensity and enhancement of the intraorbital optic nerve and surrounding tissues with no other intracranial abnormalities. Serum studies showed elevated myelin oligodendrocyte glycoprotein IgG titer. She was treated with IV methylprednisolone 1000 mg daily for 3 days and was discharged on prolonged prednisone taper with return of vision to baseline. We report a 62-year-old man who developed coma and died in a fulminant course. The patient was well until May 1, 1996 when he noted chillness, tenderness in his shoulders, and he went to bed without having his lunch and dinner. In the early morning of May 2, his families found him unresponsive and snoring; he was brought into the ER of our hospital. He had histories of hypertension, gout, and hyperlipidemia since 42 years of the age. On admission, his blood pressure was 120/70, heart rate 102 and regular, and body temperature 36.3 degrees C. His respiration was regular and he was not cyanotic. Low pitch rhonchi was heard in his right lower lung field. Otherwise general physical examination was unremarkable. Neurologic examination revealed that he was somnolent and he was only able to respond to simple questions such as opening eyes and grasping the examiner's hand, but he was unable to respond verbally. The optic discs were flat; the right pupil was slightly larger than the left, but both reacted to light. He showed ptosis on the left side, conjugate deviation of eyes to the left, and right facial paresis. The oculocephalic response and the corneal reflex were present. His right extremities were paralyzed and did not respond to pain Deep tendon reflexes were exaggerated on the right side and the plantar response was extensor on the right. No meningeal signs were present. Laboratory examination revealed the following abnormalities; WBC 18,400/ml, GOT 131 IU/l GPT 50 IU/l, CK616 IU/l, BUN 30 mg/dl, Cr 2.1 mg/ dl, glucose 339 mg/dl, and CRP 27.4 mg/dl. ECG showed sinus tachycardia and ST elevation in II, III and a VF leads and abnormal q waves in I, V5, and V6 leads. Chest X-ray revealed cardiac enlargement but the lung fields were clear. Cranial CT scan revealed low density areas in the left middle cerebral and left posterior cerebral artery territories. The patient was treated with intravenous glycerol infusion and other supportive measures. At 2: 10 AM on May 3, he developed sudden hypotension and cardiopulmonary arrest. He was pronounced dead at 3:45 AM. The patient was discussed in a neurological CPC, and the chief discussant arrived at the conclusion that the patient had acute myocardial infarction involving the inferior and the true posterior walls and left internal carotid embolism from a mural thrombus. Post mortem examination revealed occlusion of the circumflex branch of the left coronary artery due to atherom plaque rupture and myocardial infarction involving the posterior and the lateral wall with a rupture in the postero-lateral wall. Marked atheromatous changes were seen in the left internal carotid, the middle cerebral and the basilar arteries; the left internal carotid and the middle cerebral arteries were almost occluded by thrombi and blood coagulate. The territories of the left middle cerebral and the occipital arteries were infarcted; but the left thalamic area was spared. The neuropathologist concluded that the infarction was thrombotic origin not an embolic one as the atherosclerotic changes were severe. Cardiac rupture appeared to be the cause of terminal sudden hypotension and cardiopulmonary arrest. It appears likely that a vegetation which had been attached to the aortic valve induced thromboembolic occlusion of the left internal carotid artery which had already been markedly sclerotic by atherosclerosis. It is also possible that the vegetations in the aortic valve came from mural thrombi at the site of acute myocardial infarction, as no bacteria were found in those vegetations. Leber hereditary optic neuropathy (LHON) is an optic neuropathy of mitochondrial inheritance. Childhood-onset disease is relatively rare and there are limited data on this important patient subgroup. We present 3 particular presentations of LHON. Patient 1 was an 8-year-old boy admitted to the emergency department reporting a progressive bilateral visual loss and intermittent headaches. Neuro-ophthalmological examination revealed a bilateral pseudopapilledema. Lumbar puncture identified intracranial hypertension and the brain and orbits magnetic resonance imaging showed T2 hyperintensity in the posterior region of the left optic nerve and the optic chiasm. Patient 2 was a 12-year-old boy admitted to the emergency department reporting painless, progressive central vision loss in the right eye. Fundus examination revealed a hyperemic disc and vascular network papillary and peripapillary vascular microdilations. Three months later, the left eye presented visual loss. Patient 3 was a 6-year-old female child referred to the neuro-ophthalmology specialist due to painless central visual loss in both eyes. Her BCVA was 1/10 and counting fingers in right and left eye, respectively, and fundus examination revealed a pallor optic disc in the temporal sector. The phenotype of childhood-onset disease may present itself distinct from classical adult-onset LHON. The absence of classical clinical features could lead to initial misdiagnosis. There should exist a high index of suspicion in children presenting unexplained subnormal vision in order to avoid potential diagnostic delays. A 48-year-old woman was referred to the First Dept. of Int. Med., Nagasaki Univ. Sch. Med., in August, 1979, with a six-month history of recurrent episodes of right-sided painful ophthalmoplegia and diplopia. An epidode affected the right eye, lasted one to two weeks, and relapsed every month. On examination she had a complete ptosis on the right side and pain on the right eye. All extraocular muscle supplied by the 3rd nerve were paralysed. The pupils were equal in size both sides, reacting to light completely. Visual acuity was normal except myopia. All the other cranial nerves and the remainder of central nervous system was normal. Results of thyroid function tests and of lumbar puncture were normal. The glucose tolerance test showed a mild diabetic pattern. Blood and CSF cultures for bacteria, fungi, and acid-fast bacillus were negative. The skull, brain CT scan, and carotid angiogram were within normal limits. A tentative diagnosis of Tolosa-Hunt syndrome was made after an unproductive search for a cause for this woman's painful ophthalmoplegia and unsuccessful treatment of ophthalmoplegia with antibiotics or diet therapy for mild hyperglycemia. The patient was given prednisolone 30 mg daily orally when she had the 9th attack of painful ophthalmoplegia Pain, ptosis, and diplopia disappeared in 5 days and she did not show any recurrence of symptoms over the next 7 months. Defective light perception in one eye causes an asymmetrical pupillary constriction reflex called the afferent pupillary defect (APD). Arteritic PION A-PION most commonly affects Caucasian women, with an average age of 73. At onset vision loss is unilateral, but without treatment it rapidly progresses to involve both eyes. Vision loss is usually severe, ranging from counting fingers to no light perception. Associated symptoms are jaw pain exacerbated by chewing, scalp tenderness, shoulder and hip pain, headache and fatigue. Perioperative PION Vision loss is usually apparent upon waking from general anesthesia. Signs observable to a bystander include long surgery duration and facial swelling. Vision loss is usually bilateral and severe, ranging from counting fingers to no light perception. Cause A 71-year-old male presented with a history of sudden partial visual loss in the right eye with an inferior visual field defect over the past 3-4 days. He had no history of headache or of facial pain. Clinical examination confirmed that vision on the right side was reduced to 6/18 and on the left to 6/12. The right eye showed a relative afferent pupillary defect. There was no other abnormality of the anterior segment of either eye. The right retina showed a pale swollen optic disc and a provisional diagnosis of anterior ischaemic optic neuropathy (AION) was made. An urgent erythrocyte sedimentation rate (ESR) was ordered and the patient was asked to return to the eye clinic in one month. However, 16 days later--when it was first recognised that his ESR was elevated to 75 mm in the first hour--the patient was recalled immediately in order to commence systemic steroid treatment; but regrettably, by this time, his right eye had become totally blind. In this case, although the attending doctor made a correct clinical diagnosis on presentation, he failed to act upon the result of the blood test. These diagnostic criteria include: Inflammatory back pain:Chronic, inflammatory back pain is defined when at least four out of five of the following parameters are present: (1) Age of onset below 40 years old, (2) insidious onset, (3) improvement with exercise, (4) no improvement with rest, and (5) pain at night (with improvement upon getting up) Past history of inflammation in the joints, heels, or tendon-bone attachments Family history for axial spondyloarthritis or other associated rheumatic/autoimmune conditions Positive for the biomarker HLA-B27 Good response to treatment with nonsteroidal anti-inflammatory drugs (NSAIDs) Signs of elevated inflammation (C-reactive protein and erythrocyte sedimentation rate) Manifestation of psoriasis, inflammatory bowel disease, or inflammation of the eye (uveitis) If these criteria still do not give a compelling diagnosis magnetic resonance imaging (MRI) may be useful. MRI can show inflammation of the sacroiliac joint. Imaging Recurrent unbearable, paroxysmal, unilateral facial pain in the distribution of one or more branches of the trigeminal nerve often provoked by sensory stimuli is typical for idiopathic trigeminal neuralgia. The less frequent localization in the area of ophthalmic branch (5%) is particularly controversial and should be distinguished from pathological lesions in the brainstem and middle and posterior cranial fossa and from diseases of the orbit and eye. This case study presents a 79-year-old woman with typical clinical features of 1st division trigeminalgia without any neurological loss and with normal results of laryngological, ophthalmological, and stomatological examinations as well as neuroimaging CT, and MR /MRA evaluation. Only the evoked potential blink and masseter reflexes demonstrated the pathological values in the early phase of illness. After 1 year of pharmacological treatment no improvement was achieved and the pain became neuropathic and paresis of 3rd, 4th and 6th nerves developed, as observed in Tolose-Hunt syndrome. MRI of the orbit revealed a pathological mass in its apex with a connection to the superior orbital fissure. However, treatment with steroids was completely ineffective. Surgical resection of the tumor (leiomyosarcoma) only partially reversed oculomotor palsy and diminished aching. In differential diagnosis of idiopathic and symptomatic trigeminalgia, early MR and MRA imaging is the most essential and sometimes may be the best single test to evaluate lesions even in distant areas of the nervous system branches. Nonspecific orbital inflammation (NSOI) is a rare idiopathic ocular pathology characterized by unilateral, painful orbital swelling without identifiable infectious or systemic disorders, which can be complicated by optic nerve compromise. A 50-year-old man presented to the Emergency Department with recurring, progressive painless left eye swelling, decreased visual acuity, and binocular diplopia in the absence of trauma, infection, or known malignancy. His physical examination was notable for left-sided decreased visual acuity, an afferent pupillary defect, severe left eye proptosis and chemosis, and restricted extraocular movements; his dilatated funduscopic examination was notable for ipsilateral retinal folds within the macula, concerning for a disruption between the sclera and the retina. Ocular examination of the right eye was unremarkable. Laboratory data were unrevealing. Gadolinium-enhanced magnetic resonance imaging showed marked thickening of the left extraocular muscles associated with proptosis, dense inflammatory infiltration of the orbital fat, and characteristics consistent with perineuritis. The patient was diagnosed with NSOI with optic neuritis and admitted for systemic steroid therapy; he was discharged on hospital day 2 after receiving high-dose intravenous (i.v.) methylprednisolone with significant improvement. WHY SHOULD AN EMERGENCY PHYSICIAN BE AWARE OF THIS?: NSOI is a rare and idiopathic ocular emergency, with clinical mimicry resembling a broad spectrum of systemic diseases such as malignancy, autoimmune diseases, endocrine disorders, and infection. Initial work-up for new-onset ocular proptosis should include comprehensive laboratory testing and gadolinium-enhanced magnetic resonance imaging. Timely evaluation by an ophthalmologist is crucial to assess for optic nerve involvement. Signs of optic nerve compromise include decreased visual acuity, afferent pupillary defect, or decreased color saturation. Patients with optic nerve compromise require admission for aggressive anti-inflammatory therapy with i.v. steroids in an attempt to reduce risk of long-term visual sequelae. Our case demonstrates a severe presentation of this disorder and exhibits remarkable visual recovery after 48 h of systemic i.v. steroid treatment.\nHere is the question:\nNEUROLOGY: A 70-year-old woman with a history of anorexia, weight loss, discomfort in the muscles and proximal joints and pain in the temporomandibular region who comes to the emergency department for unilateral loss of vision (hand movement), sudden and painless onset (afferent pupillary defect).what test would you request first for diagnostic purposes?\nHere are the potential choices:\n1. Lumbar puncture.\n2. C Reactive Protein.\n3. Magnetic resonance angiography.\n4. Carotid ultrasound.\nThe correct answer is: ", "gold_answer": "2 C Reactive Protein.", "prediction": "You are a helpful medical expert, and your task is to answer a multi-choice medical question using the relevant documents. Please choose the answer from the provided options. Your responses will be used for research purposes only, so please have a definite answer.\nHere are the relevant documents:\n A careful neurologic examination is an essential first step in the evaluation. In most cases, patients with an abnormal examination or a history of recent-onset headache should be evaluated by a computed tomography (CT) or magnetic resonance imaging (MRI) study. As an initial screening procedure for intracranial pathology in this setting, CT and MRI methods appear to be equally sensitive. In some circumstances, a lumbar puncture (LP) is also required, unless a benign etiology can be otherwise established. A general evaluation of acute headache might include cranial arteries by palpation; cervical spine by Pain induced by bending, lifting, cough Pain associated with local tenderness, e.g., region of temporal artery the effect of passive movement of the head and by imaging; the investigation of cardiovascular and renal status by blood pressure monitoring and urine examination; and eyes by funduscopy, intraocular pressure measurement, and refraction. One recommended diagnostic approach is as follows. If any urgent red flags are present such as visual loss, new seizures, new weakness, new confusion, further workup with imaging and possibly a lumbar puncture should be done (see red flags section for more details). If the headache is sudden onset (thunderclap headache), a computed tomography test to look for a brain bleed (subarachnoid hemorrhage) should be done. If the CT scan does not show a bleed, a lumbar puncture should be done to look for blood in the CSF, as the CT scan can be falsely negative and subarachnoid hemorrhages can be fatal. If there are signs of infection such as fever, rash, or stiff neck, a lumbar puncture to look for meningitis should be considered. If there is jaw claudication and scalp tenderness in an older person, a temporal artery biopsy to look for temporal arteritis should be performed and immediate treatment should be started. Neuroimaging A 75-year-old Caucasian woman presented with sudden-onset multifocal scotomas in her right eye's central vision for 1 day. There were subtle white intraretinal foveal lesions that correlated with patchy inner retinal hyperreflectivity on optical coherence tomography, suggestive of paracentral acute middle maculopathy. Initial cerebrovascular work-up was negative. Review of systems was positive for lethargy and jaw claudication. The sedimentation rate and c-reactive protein were elevated, but platelet count was normal. The patient was started on 60 mg oral prednisone daily and underwent bilateral temporal artery that confirmed the diagnosis of giant cell arteritis. The purpose is to report a patient with primary open-angle glaucoma that developed sudden painless unilateral vision loss, a sequential ophthalmoscopic appearance with features of both central retinal artery and later central retinal vein occlusion, and objective visual system dysfunction in the form of a relative afferent pupil defect, who spontaneously recovered vision along with complete resolution of the pupillary defect over several weeks. A 50-year-old woman with a long-standing history of glaucoma presented with acute, painless vision loss in one eye, a pallid retina with a cherry red macula, diffuse retinal hemorrhages, and a relative afferent pupil defect. Spectral domain optical coherence tomography and fluorescein angiography were essentially normal with neither retinal edema nor retinal ischemia to account for the visual dysfunction. Over the course of 2 months, the patient regained vision and the relative afferent pupil defect, typically a permanent manifestation of retinal destruction, resolved. Not all retinal vaso-occlusive phenomena can be completely attributed to a central retinal vein or artery occlusion. In the patient presented, there was no objective diagnostic testing that revealed a cause for the patient's vision loss or relative afferent pupillary defect. This combined with the complete recovery of vision and resolution of the relative afferent pupillary defect underscores a lack of comprehensive understanding of retinal vaso-occlusive disease. A 71-year-old man visited our clinic with a 3-day history of severe throbbing headache and 1-day history of horizontal diplopia. He had had jaw claudication and pain in the neck and shoulder several days previously. His right eye was slightly esotropic and did not move laterally. There was no blepharoptosis, proptosis, lid edema, or conjunctival injection. The pupils were unremarkable. The remainder of the cranial nerve functions was intact. There was no limb weakness or sensory impairment. Superficial temporal arteries were swollen and tender on both sides. Laboratory examination showed elevated CRP level and high erythrocyte sedimentation rate. Cranial MR images were unremarkable. The cerebrospinal fluid was acellular with 45 mg/dl of protein. A diagnosis of temporal arteritis was made. Treatment with 50 mg of prednisolone brought about prompt disappearance of the headache. Right ocular movement fully recovered in 10 days. Temporal artery biopsy findings and response to corticosteroid were consistent with temporal arteritis. The motility pattern of the right eye was consistent with complete abducens nerve palsy, which is a rare manifestation of temporal arteritis. Although temporal arteritis is a rare cause of ophthalmoplegia in the elderly patients, swift diagnosis and treatment is necessary to avoid blindness. Leber hereditary optic neuropathy (LHON) is an inherited mitochondrial disease characterized by painless vision loss affecting both eyes. The disease usually develops in both eyes within weeks to months of onset. We report a case of LHON who presented with unilateral vision loss in childhood with an interval of more than 30 years between vision loss in the two eyes. A 43-year-old man presented with a 1-month history of vision loss in his right eye. At 9 years of age, his visual acuity in the left eye declined, and he had been treated with glaucoma eyedrops bilaterally at his eye clinic. At his first visit to our hospital, his BCVA was 0.15 in the right eye and 0.1 in the left eye, and critical flicker frequency was 16 Hz in the right eye and 15 Hz in the left eye, and he was negative for a relative afferent pupillary defect. The Goldman visual field showed central scotoma in both eyes. Fundus examination revealed slight redness of the right optic disc with meandering retinal small vessels, and the left optic disc had a slight pallor. Fluorescein angiography could not be performed because of liver dysfunction. OCT showed prominent bilateral thinning of the RNFL and retinal ganglion cell layer. Enhancement of the optic nerve was not apparent on orbital gadolinium-enhanced magnetic resonance imaging. Hematologic analysis revealed macrocytic anemia and low levels of vitamin B12 and folate. His mother had a presumptive diagnosis of LHON but did not receive genetic testing. A male cousin also had severe vision loss. Based on the likely family history of LHON, we performed genetic testing, which revealed the 11778 mitochondrial point mutation associated with this condition. We report a case of LHON with 34 years interval in vision loss in the fellow eye. LHON may develop in the second eye decades after its onset in the first. Detailed medical interviews and scrutiny, such as examination of family history, are warranted in consideration of LHON. If a 20-year-old female develops headaches after drinking red wine, think migraine. Associated symptoms/signs: Significant findings include fever or rash (consider meningitis or other infectious causes), jaw claudication (specific for temporal arteritis), or constitutional symptoms such as weight loss (associated with neoplastic, inflammatory, or infectious conditions). Photophobia, nausea, and vomiting are associated with migraine, aneurysmal SAH, and meningitis, but neck stiffness is more likely to accompany the latter two. Neurologic sequelae: Look for diplopia, mental status changes or associated symptoms (numbness, weakness, dizziness, ataxia, visual disturbances), papilledema, or pupillary abnormalities (partial CN III palsy or Horner’s syndrome). Patient risk factors: High-risk patients are > 50 years of age, immunocompromised, or with preexisting malignancy. If SAH is suspected, obtain a head CT without contrast. If CT is , LP is mandatory. Obtain a CBC. Optic neuritis is a common cause of subacute unilateral vision loss, occurring in 1-5 per 100,000 persons per year. It is more common in Caucasians, women, and those from countries with northern latitudes. Those aged 20-49 years are at greatest risk. The condition arises due to inflammation of the optic nerve. Inflammation may occur due to systemic inflammatory disorders, most commonly multiple sclerosis. A 21-year-old African-American male presented to our emergency department with a complaint of painful unilateral vision loss. On examination he was found to have a relative afferent pupillary defect and red desaturation. A bedside ultrasound suggested pseudopapilledema suggestive of optic neuritis. He was admitted to Neurology for confirmation of and treatment for optic neuritis. Magnetic resonance imaging confirmed optic neuritis. The patient was treated with i.v. steroids and discharged after improvement in visual function. WHY SHOULD AN EMERGENCY PHYSICIAN BE AWARE OF THIS?: Optic neuritis is a clinical diagnosis. The subtle historical components and examination findings make it a diagnostic challenge for the busy emergency physician. Early diagnosis may improve visual outcomes. Discovery of pseudopapilledema on bedside ultrasound may be seen in optic neuritis, and is another finding that emergency physicians may assess for in patient presenting with unilateral vision loss. Giant cell arteritis (GCA) is a polysymptomatic disease which constitutes an ophthalmic emergency because early recognition and management can prevent blindness. There is conflicting information in the literature on the validity, sensitivity, and specificity of various systemic symptoms and signs of GCA. This paper presents a review of our prospective studies on the subject, and our findings are particularly relevant to dentists. We investigated 363 patients in a prospective study. Positive temporal artery biopsy was seen in 106 patients and negative in 257 referred for diagnosis of GCA. Systemic symptoms and signs of GCA and erythrocyte sedimentation rate (Westergren-ESR) and C-reactive protein (CRP) levels were compared in these two groups of patients. The odds of having a positive temporal artery biopsy (i.e., GCA) were 9.1 times greater with jaw claudication (pain in masticatory muscles on eating), 3.4 times with neck pain, 3.2 times with CRP > 2.45 mg/dL, 2.0 times with ESR 47.107 mm/hr, 2.7 times with ESR > 107 mm/hr, and 2.0 times when the patients were aged > or = 75 years. Other signs and symptoms did not show a significant association with a positive biopsy. Our study showed that \"normal\" ESR values do not rule out GCA but that CRP is a more useful test than ESR. Since jaw claudication is one of the most important symptoms of GCA, dentists should keep this possibility in mind when older patients come complaining of jaw pain while eating. To report the case and multimodal imaging findings of a healthy young woman who developed paracentral acute middle maculopathy (PAMM) 9 weeks after COVID-19 disease. Case report. Ultra-widefield fundus photography, macular spectral-domain optical coherence tomography (SD-OCT), fluorescein angiography (FA), and OCT-angiography (OCT-A) were performed. A 36-year-old woman who developed PAMM 9 weeks after SARS-CoV-2 infection. A 36-year-old woman went to the emergency department (ED) with sudden, painless, left eye (LE) vision loss. The only relevant past medical history was COVID-19 disease 9 weeks before. Best corrected visual acuity (BCVA) was 20/200, a LE relative afferent pupillary defect (RAPD) was present and superficial hemorrhages throughout the macular area and peripheral retina were found. Nearly four hours after admission, LE BCVA recovered to 20/20 without RAPD. Five days after presentation in the ED, the patient returned with recurrent LE vision loss, with spontaneous recovery within 12 hours. Macular SD-OCT revealed hyperreflectivity of the inner plexiform and inner nuclear layers and the diagnosis of PAMM was established. The patient started oral acetylsalicylic acid and oral prednisolone. The patient did not report any new episodes of vision loss and there was a progressive resolution of abnormal fundus findings. SARS-CoV-2 infection increases the risk of vascular thrombotic events with possible involvement of the retinal circulation, and PAMM may present as a possible complication. Ophthalmologists should be able to recognize it promptly through multimodal imaging findings. In anterior ischemic optic neuropathy (AION), it is important not to miss the diagnosis of giant cell arteritis (GCA) because this requires immediate steroid treatment to prevent involvement of the second eye and possible blindness. A missed diagnosis also might lead to fatal systemic complications. Observational case report. A 79-year-old woman noticed decreased visual and visual field loss in the right eye. At presentation, right visual acuity was 10/20 (ETDRS chart 2000). There was a right relative afferent pupillary defect of 0.6 log units. Asked for symptoms of GCA she complained about temporal and occipital headache, jaw claudication combined with malaise, and myalgia of the upper limbs. Laboratory tests showed normal inflammatory markers. Repeated tests confirmed ESR and CRP to be within the normal range. GCA being suspected, ultrasound of the superficial temporal arteries and temporal artery biopsy were performed unilaterally on the right side. Histology showed a chronic inflammatory cell infiltrate consistent with active GCA. The patient was treated with high-dose corticosteroids (250 mg methylprednisolone, three times/day, initially) and symptoms rapidly resolved, but visual loss remained unchanged. The case presented here proves that GCA with typical related visual loss (AION) is possible even when both ESR and CRP are in the normal range. Therefore, in the presence of typical symptoms, the clinician must not rely solely on laboratory testing, but start steroid therapy immediately and order a temporal artery biopsy. Cancer should be suspected if there is previous history of it, unexplained weight loss, or unremitting pain. Spinal epidural abscess is more common among those with diabetes mellitus or immunocompromised or who had spinal surgery, injection or catheter; it typically causes fever, leukocytosis and increased erythrocyte sedimentation rate. If cancer or spinal epidural abscess are suspected, urgent magnetic resonance imaging is recommended for confirmation. Proximal diabetic neuropathy typically affects middle aged and older people with well-controlled type-2 diabetes mellitus; onset is sudden causing pain usually in multiple dermatomes quickly followed by weakness. Diagnosis typically involves electromyography and lumbar puncture. Shingles is more common among the elderly and immunocompromised; usually (but not always) pain is followed by appearance of a rash with small blisters along a single dermatome. Acute Lyme radiculopathy may follow a history of outdoor activities during warmer Giant cell arteritis (GCA) is often diagnosed very late, variable \"facets\" of the disease exist render the diagnosis more difficult. Follow-up observations of five very old patients are reported in whom diagnosis was made too late, resulting in blindness of both eyes. Five patients (age 76-84 years, 4 women, one man) with GCA became blind in both eyes because diagnosis had been delayed (two patients) or onset of therapy was too late (three patients). In two patients who also had arterial hypertension, the symptom \"headache\" had been misleading. Symptoms of accompanying general diseases masked the real diagnosis, particularly in the second patient who had renal insufficiency, coronary artery disease, and unilateral obstruction of the internal carotid artery. Symptoms that failed to lead to the correct diagnosis were: muscle or chewing pain (three patients), circumscribed numbness around the mouth (second patient), and persistent headache despite normalization of blood pressure. Normal findings from cranial CTAs (two patients) led to the wrong reassurance of the patient. Swelling of the optic disk (two patients) was misdiagnosed by ophthalmologists, as was a retinal branch arterial occlusion (first patient). Three patients, afraid of possible side effects caused by glucocorticoids, took ineffective alternative medications. Poor vigilance led to blindness of the fifth patient with long-standing polymyalgia rheumatica. Targeted examinations at the onset of symptoms are necessary. GCA-symptoms were mis-constructed by additional diseases that disguised the correct diagnosis. The danger of bilateral blindness is particularly great in patients of great age. A 70-year-old woman presented with new onset of left eye and facial pain. Ophthalmic and neurological examinations, magnetic resonance imaging brain, erythrocyte sedimentation rate, and C-reactive protein were unrevealing. A few days later, she developed vision loss in her left eye. Examination revealed decreased visual acuity with a relative afferent pupillary defect in the left eye and a diffuse mild swelling of the left optic nerve head. Repeat magnetic resonance imaging showed T2 hyperintensity and enhancement of the intraorbital optic nerve and surrounding tissues with no other intracranial abnormalities. Serum studies showed elevated myelin oligodendrocyte glycoprotein IgG titer. She was treated with IV methylprednisolone 1000 mg daily for 3 days and was discharged on prolonged prednisone taper with return of vision to baseline. We report a 62-year-old man who developed coma and died in a fulminant course. The patient was well until May 1, 1996 when he noted chillness, tenderness in his shoulders, and he went to bed without having his lunch and dinner. In the early morning of May 2, his families found him unresponsive and snoring; he was brought into the ER of our hospital. He had histories of hypertension, gout, and hyperlipidemia since 42 years of the age. On admission, his blood pressure was 120/70, heart rate 102 and regular, and body temperature 36.3 degrees C. His respiration was regular and he was not cyanotic. Low pitch rhonchi was heard in his right lower lung field. Otherwise general physical examination was unremarkable. Neurologic examination revealed that he was somnolent and he was only able to respond to simple questions such as opening eyes and grasping the examiner's hand, but he was unable to respond verbally. The optic discs were flat; the right pupil was slightly larger than the left, but both reacted to light. He showed ptosis on the left side, conjugate deviation of eyes to the left, and right facial paresis. The oculocephalic response and the corneal reflex were present. His right extremities were paralyzed and did not respond to pain Deep tendon reflexes were exaggerated on the right side and the plantar response was extensor on the right. No meningeal signs were present. Laboratory examination revealed the following abnormalities; WBC 18,400/ml, GOT 131 IU/l GPT 50 IU/l, CK616 IU/l, BUN 30 mg/dl, Cr 2.1 mg/ dl, glucose 339 mg/dl, and CRP 27.4 mg/dl. ECG showed sinus tachycardia and ST elevation in II, III and a VF leads and abnormal q waves in I, V5, and V6 leads. Chest X-ray revealed cardiac enlargement but the lung fields were clear. Cranial CT scan revealed low density areas in the left middle cerebral and left posterior cerebral artery territories. The patient was treated with intravenous glycerol infusion and other supportive measures. At 2: 10 AM on May 3, he developed sudden hypotension and cardiopulmonary arrest. He was pronounced dead at 3:45 AM. The patient was discussed in a neurological CPC, and the chief discussant arrived at the conclusion that the patient had acute myocardial infarction involving the inferior and the true posterior walls and left internal carotid embolism from a mural thrombus. Post mortem examination revealed occlusion of the circumflex branch of the left coronary artery due to atherom plaque rupture and myocardial infarction involving the posterior and the lateral wall with a rupture in the postero-lateral wall. Marked atheromatous changes were seen in the left internal carotid, the middle cerebral and the basilar arteries; the left internal carotid and the middle cerebral arteries were almost occluded by thrombi and blood coagulate. The territories of the left middle cerebral and the occipital arteries were infarcted; but the left thalamic area was spared. The neuropathologist concluded that the infarction was thrombotic origin not an embolic one as the atherosclerotic changes were severe. Cardiac rupture appeared to be the cause of terminal sudden hypotension and cardiopulmonary arrest. It appears likely that a vegetation which had been attached to the aortic valve induced thromboembolic occlusion of the left internal carotid artery which had already been markedly sclerotic by atherosclerosis. It is also possible that the vegetations in the aortic valve came from mural thrombi at the site of acute myocardial infarction, as no bacteria were found in those vegetations. Leber hereditary optic neuropathy (LHON) is an optic neuropathy of mitochondrial inheritance. Childhood-onset disease is relatively rare and there are limited data on this important patient subgroup. We present 3 particular presentations of LHON. Patient 1 was an 8-year-old boy admitted to the emergency department reporting a progressive bilateral visual loss and intermittent headaches. Neuro-ophthalmological examination revealed a bilateral pseudopapilledema. Lumbar puncture identified intracranial hypertension and the brain and orbits magnetic resonance imaging showed T2 hyperintensity in the posterior region of the left optic nerve and the optic chiasm. Patient 2 was a 12-year-old boy admitted to the emergency department reporting painless, progressive central vision loss in the right eye. Fundus examination revealed a hyperemic disc and vascular network papillary and peripapillary vascular microdilations. Three months later, the left eye presented visual loss. Patient 3 was a 6-year-old female child referred to the neuro-ophthalmology specialist due to painless central visual loss in both eyes. Her BCVA was 1/10 and counting fingers in right and left eye, respectively, and fundus examination revealed a pallor optic disc in the temporal sector. The phenotype of childhood-onset disease may present itself distinct from classical adult-onset LHON. The absence of classical clinical features could lead to initial misdiagnosis. There should exist a high index of suspicion in children presenting unexplained subnormal vision in order to avoid potential diagnostic delays. A 48-year-old woman was referred to the First Dept. of Int. Med., Nagasaki Univ. Sch. Med., in August, 1979, with a six-month history of recurrent episodes of right-sided painful ophthalmoplegia and diplopia. An epidode affected the right eye, lasted one to two weeks, and relapsed every month. On examination she had a complete ptosis on the right side and pain on the right eye. All extraocular muscle supplied by the 3rd nerve were paralysed. The pupils were equal in size both sides, reacting to light completely. Visual acuity was normal except myopia. All the other cranial nerves and the remainder of central nervous system was normal. Results of thyroid function tests and of lumbar puncture were normal. The glucose tolerance test showed a mild diabetic pattern. Blood and CSF cultures for bacteria, fungi, and acid-fast bacillus were negative. The skull, brain CT scan, and carotid angiogram were within normal limits. A tentative diagnosis of Tolosa-Hunt syndrome was made after an unproductive search for a cause for this woman's painful ophthalmoplegia and unsuccessful treatment of ophthalmoplegia with antibiotics or diet therapy for mild hyperglycemia. The patient was given prednisolone 30 mg daily orally when she had the 9th attack of painful ophthalmoplegia Pain, ptosis, and diplopia disappeared in 5 days and she did not show any recurrence of symptoms over the next 7 months. Defective light perception in one eye causes an asymmetrical pupillary constriction reflex called the afferent pupillary defect (APD). Arteritic PION A-PION most commonly affects Caucasian women, with an average age of 73. At onset vision loss is unilateral, but without treatment it rapidly progresses to involve both eyes. Vision loss is usually severe, ranging from counting fingers to no light perception. Associated symptoms are jaw pain exacerbated by chewing, scalp tenderness, shoulder and hip pain, headache and fatigue. Perioperative PION Vision loss is usually apparent upon waking from general anesthesia. Signs observable to a bystander include long surgery duration and facial swelling. Vision loss is usually bilateral and severe, ranging from counting fingers to no light perception. Cause A 71-year-old male presented with a history of sudden partial visual loss in the right eye with an inferior visual field defect over the past 3-4 days. He had no history of headache or of facial pain. Clinical examination confirmed that vision on the right side was reduced to 6/18 and on the left to 6/12. The right eye showed a relative afferent pupillary defect. There was no other abnormality of the anterior segment of either eye. The right retina showed a pale swollen optic disc and a provisional diagnosis of anterior ischaemic optic neuropathy (AION) was made. An urgent erythrocyte sedimentation rate (ESR) was ordered and the patient was asked to return to the eye clinic in one month. However, 16 days later--when it was first recognised that his ESR was elevated to 75 mm in the first hour--the patient was recalled immediately in order to commence systemic steroid treatment; but regrettably, by this time, his right eye had become totally blind. In this case, although the attending doctor made a correct clinical diagnosis on presentation, he failed to act upon the result of the blood test. These diagnostic criteria include: Inflammatory back pain:Chronic, inflammatory back pain is defined when at least four out of five of the following parameters are present: (1) Age of onset below 40 years old, (2) insidious onset, (3) improvement with exercise, (4) no improvement with rest, and (5) pain at night (with improvement upon getting up) Past history of inflammation in the joints, heels, or tendon-bone attachments Family history for axial spondyloarthritis or other associated rheumatic/autoimmune conditions Positive for the biomarker HLA-B27 Good response to treatment with nonsteroidal anti-inflammatory drugs (NSAIDs) Signs of elevated inflammation (C-reactive protein and erythrocyte sedimentation rate) Manifestation of psoriasis, inflammatory bowel disease, or inflammation of the eye (uveitis) If these criteria still do not give a compelling diagnosis magnetic resonance imaging (MRI) may be useful. MRI can show inflammation of the sacroiliac joint. Imaging Recurrent unbearable, paroxysmal, unilateral facial pain in the distribution of one or more branches of the trigeminal nerve often provoked by sensory stimuli is typical for idiopathic trigeminal neuralgia. The less frequent localization in the area of ophthalmic branch (5%) is particularly controversial and should be distinguished from pathological lesions in the brainstem and middle and posterior cranial fossa and from diseases of the orbit and eye. This case study presents a 79-year-old woman with typical clinical features of 1st division trigeminalgia without any neurological loss and with normal results of laryngological, ophthalmological, and stomatological examinations as well as neuroimaging CT, and MR /MRA evaluation. Only the evoked potential blink and masseter reflexes demonstrated the pathological values in the early phase of illness. After 1 year of pharmacological treatment no improvement was achieved and the pain became neuropathic and paresis of 3rd, 4th and 6th nerves developed, as observed in Tolose-Hunt syndrome. MRI of the orbit revealed a pathological mass in its apex with a connection to the superior orbital fissure. However, treatment with steroids was completely ineffective. Surgical resection of the tumor (leiomyosarcoma) only partially reversed oculomotor palsy and diminished aching. In differential diagnosis of idiopathic and symptomatic trigeminalgia, early MR and MRA imaging is the most essential and sometimes may be the best single test to evaluate lesions even in distant areas of the nervous system branches. Nonspecific orbital inflammation (NSOI) is a rare idiopathic ocular pathology characterized by unilateral, painful orbital swelling without identifiable infectious or systemic disorders, which can be complicated by optic nerve compromise. A 50-year-old man presented to the Emergency Department with recurring, progressive painless left eye swelling, decreased visual acuity, and binocular diplopia in the absence of trauma, infection, or known malignancy. His physical examination was notable for left-sided decreased visual acuity, an afferent pupillary defect, severe left eye proptosis and chemosis, and restricted extraocular movements; his dilatated funduscopic examination was notable for ipsilateral retinal folds within the macula, concerning for a disruption between the sclera and the retina. Ocular examination of the right eye was unremarkable. Laboratory data were unrevealing. Gadolinium-enhanced magnetic resonance imaging showed marked thickening of the left extraocular muscles associated with proptosis, dense inflammatory infiltration of the orbital fat, and characteristics consistent with perineuritis. The patient was diagnosed with NSOI with optic neuritis and admitted for systemic steroid therapy; he was discharged on hospital day 2 after receiving high-dose intravenous (i.v.) methylprednisolone with significant improvement. WHY SHOULD AN EMERGENCY PHYSICIAN BE AWARE OF THIS?: NSOI is a rare and idiopathic ocular emergency, with clinical mimicry resembling a broad spectrum of systemic diseases such as malignancy, autoimmune diseases, endocrine disorders, and infection. Initial work-up for new-onset ocular proptosis should include comprehensive laboratory testing and gadolinium-enhanced magnetic resonance imaging. Timely evaluation by an ophthalmologist is crucial to assess for optic nerve involvement. Signs of optic nerve compromise include decreased visual acuity, afferent pupillary defect, or decreased color saturation. Patients with optic nerve compromise require admission for aggressive anti-inflammatory therapy with i.v. steroids in an attempt to reduce risk of long-term visual sequelae. Our case demonstrates a severe presentation of this disorder and exhibits remarkable visual recovery after 48 h of systemic i.v. steroid treatment.\nHere is the question:\nNEUROLOGY: A 70-year-old woman with a history of anorexia, weight loss, discomfort in the muscles and proximal joints and pain in the temporomandibular region who comes to the emergency department for unilateral loss of vision (hand movement), sudden and painless onset (afferent pupillary defect).what test would you request first for diagnostic purposes?\nHere are the potential choices:\n1. Lumbar puncture.\n2. C Reactive Protein.\n3. Magnetic resonance angiography.\n4. Carotid ultrasound.\nThe correct answer is: 2. C Reactive Protein." }, { "question": "You are a helpful medical expert, and your task is to answer a multi-choice medical question using the relevant documents. Please choose the answer from the provided options. Your responses will be used for research purposes only, so please have a definite answer.\nHere are the relevant documents:\n Other means used in diagnosis of vertiginous epilepsy include: Electroencephalography (EEG) Magnetic resonance imaging (MRI) Positron emission tomography (PET) Neuropsychological testing The EEG measures electrical activity in the brain, allowing a physician to identify any unusual patterns. While EEGs are good for identifying abnormal brain activity is it not helpful in localizing where the seizure originates because they spread so quickly across the brain. MRIs are used to look for masses or lesions in the temporal lobe of the brain, indicating possible tumors or cancer as the cause of the seizures. When using a PET scan, a physician is looking to detect abnormal blood flow and glucose metabolism in the brain, which is visible between seizures, to indicate the region of origin. Management The most definitive test to distinguish epilepsy from PNES is long term video-EEG monitoring, with the aim of capturing one or two episodes on both video recording and EEG simultaneously (some clinicians may use suggestion to attempt to trigger an episode). Additional clinical criteria are usually considered in addition to video-EEG monitoring when diagnosing PNES. By recording the event in question on video and EEG simultaneously, a clear diagnosis can usually be obtained. Laboratory testing can detect rising blood levels of serum prolactin if samples are taken in the right time window after most tonic-clonic or complex partial epileptic seizures. However, due to false positives and variability in results, this test is relied upon less frequently. The pathogenesis of PED has also been linked to mutations in the GLUT1 glucose transporter which can result in transient energy deficits in the basal ganglia. Diagnosis Diagnosis is similar, but slightly different for each type of PD. Some types are more understood than others, and therefore have more criteria for diagnosis. PKD The guidelines for diagnosing PKD were reviewed and confirmed by Unterberger and Trinka. PKD consists of unexpected forms of involuntary movements of the body. The patient is usually diagnosed sometime before their 20s, and is more likely diagnosed during childhood than early adulthood. Almost all PKD's are idiopathic, but there have been examples of autosomal dominant inheritance as well. Physical examination and brain imaging examinations show normal results, and an EEG shows no specific abnormalities as well. However, the negative synchronous EEG results can be used to prove that PKD is not a sort of reflex epilepsy, but a different disease. To confirm diagnosis, awake and asleep EEG and magnetic resonance imaging (MRI) are performed. MRI is used to detect focal brain lesions. Ruling out other diagnoses Certain diagnoses must be ruled out before diagnosing LGS. These diagnoses are: Doose syndrome Dravet syndrome pseudo-Lennox Gastaut syndrome (atypical benign partial epilepsy) LGS is more easily distinguished from Doose syndrome by seizure type after the syndrome has progressed. Doose syndrome has more myoclonic seizures and LGS has more tonic seizures. The Doose syndromes is less likely to have cognitive disabilities. Pseudo-Lennox–Gastaut syndrome can be distinguished from LGS because pseudo-LGS has different spike-and-wave patterns on EEG. Treatment There are several treatment options, including medications, surgery, and diet. Medications In most patients with LGS, the treatment does not end seizure recurrence. There exist various morphological and biochemical changes closely associated with electrophysiological phenomena which cause epileptic seizures in the brains of epilepsy patients. Recent developments in investigation methods, not only electrophysiological(EEG and MEG), but also neuroimaging involving morphological imaging(CT and conventional MRI) and functional imaging(SPECT, PET, functional MRI and MRS) is able to demonstrate these changes. SPECT and PET can particularly clarify the changes of cerebral blood flow and glucose metabolism between interictal and ictal periods. In our experience of 423 patients who underwent epilepsy surgery for intractable seizures, these interventions provide important information to identify the epileptogenic foci. However, in practice, discordance in the results of these presurgical evaluations is recognized, and invasive intracranial recordings are needed in such cases. These problems in diagnosis were shown especially in patients with mesial temporal sclerosis and focal cortical dysplasia. To detect an epileptogenic focus more clearly, a combination of morphological and functional findings, new functional imaging such as neurotransmitter receptor imaging, EEG-triggered or neuropharmacological functional MRI, as well as, statistical parametric analysis may be needed. Diagnosis Diagnosis of PME is based on the individual’s signs and symptoms as well as failure to respond to antiepileptic drugs and therapy. Further diagnosis support includes EEG results, genetic testing, enzyme testing, and skin and muscle biopsies. Gaucher’s disease can be diagnosed through enzyme testing as it is a metabolic disease. Lafora’s disease can be diagnosed using skin biopsies. While Action myoclonus renal failure (AMRF) syndrome can only be diagnosed using genetic test. Using EEG’s as a form of diagnosis can prove difficult as patients differ in their neurophysiology. In Lafora’s disease EEGs can show slowing background activity or focal discharges as well as epileptiform discharges. In ULD EEGs show generalized epileptiform discharges and in MERRF patients show background slowing. Therefore, diagnosis is best made using a combination of different tools like signs and symptoms, age of onset, EEG, gene testing, enzyme measurements, and biopsy of skin and muscle. For adults, the testing of electrolyte, blood glucose and calcium levels is important to rule out problems with these as causes. An electrocardiogram can rule out problems with the rhythm of the heart. A lumbar puncture may be useful to diagnose a central nervous system infection but is not routinely needed. In children additional tests may be required such as urine biochemistry and blood testing looking for metabolic disorders. Together with EEG and neuroimaging, genetic testing is becoming one of the most important diagnostic technique for epilepsy, as a diagnosis might be achieved in a relevant proportion of cases with severe epilepsies, both in children and adults. For those with negative genetic testing, in some it might be important to repeat or re-analyze previous genetic studies after 2–3 years. Diagnosis The diagnosis of epilepsy is typically made based on observation of the seizure onset and the underlying cause. An electroencephalogram (EEG) to look for abnormal patterns of brain waves and neuroimaging (CT scan or MRI) to look at the structure of the brain are also usually part of the initial investigations. While figuring out a specific epileptic syndrome is often attempted, it is not always possible. Video and EEG monitoring may be useful in difficult cases. Definition Epilepsy is a disorder of the brain defined by any of the following conditions: {| cellpadding=5 style=\"border:1px solid #ccc\" |- bgcolor=\"#fafafa\" | At least two unprovoked (or reflex) seizures occurring more than 24 hours apart One unprovoked (or reflex) seizure and a probability of further seizures similar to the general recurrence risk (at least 60%) after two unprovoked seizures, occurring over the next 10 years Diagnosis of an epilepsy syndrome |} The goal of this study was to estimate the frequency of psychogenic pseudosyncope in patients with \"syncope of unknown origin.\" Twenty to thirty percent of patients referred to epilepsy centers for refractory seizures have psychogenic seizures. With syncope, about 20-30% of the cases remain unexplained after a complete evaluation, but, unlike in seizures, a psychogenic etiology is not usually investigated. We prospectively evaluated patients referred to our epilepsy center for evaluation of recurrent syncope-like episodes, that is, limp, motionless fainting. All patients had a negative syncope workup. We performed EEG-video monitoring with activation by suggestion (\"induction\"), similar to what is used for diagnosis of psychogenic seizures. Activation was performed with patients standing or sitting up. The diagnosis of psychogenic pseudosyncope required: (1) an activation procedure that triggered the habitual event; (2) a clinical event of loss of postural tone and limp, motionless unresponsiveness with eyes closed; (3) normal EEG before, during, and after the clinical event, that is, no epileptiform abnormalities, a normal alpha rhythm during unresponsiveness, and no suppression of background or slowing as is typically seen in syncope. Ten patients were recruited over an 18-month period. Habitual syncope-like episodes were triggered in 9 of 10 (90%) patients, and all 9 were shown to have psychogenic pseudosyncope (eyes closed, motionless, unresponsive with normal EEG including normal alpha rhythm). In one patient, no episode was triggered, so a diagnosis could not be made. Among the 9 patients for whom episodes were recorded, age ranged from 21 to 60 (mean=36). Five were women. Duration of symptoms ranged from 6 months to 15 years (mean=4.2 years). Event frequency ranged from four per day to two per month. Prior evaluations for syncope included ECG in all patients, two-dimensional echocardiogram in three, Holter monitoring in two, and tilt-table test in five. Four patients had undergone cardiac catheterization, and one had received a pacemaker. Neurologic tests included CT of the head in seven and MRI of the brain in eight. Many patients with \"syncope of unknown origin\" may have psychogenic pseudosyncope, but most such patients do not undergo EEG-video monitoring, which is the only way to demonstrate a psychogenic etiology. Psychogenic pseudosyncope is not simply a diagnosis of exclusion, and can be firmly diagnosed. As is usually recommended for seizure-like events, patients with syncope-like events and a negative evaluation should undergo EEG-video monitoring with induction, specifically looking for a possible psychogenic etiology. EEGs were recorded in 22 patients with medically refractory complex partial epilepsy undergoing presurgical evaluation and 11 age-matched controls while subjected to moderate levels of hypoglycemia to determine if changes activated were predictive of underlying pathology. Five patients had fasting EEGs showing focal abnormalities not seen in the non-fasting state. With hypoglycemia, EEG tracings in normal individuals showed diffuse background slowing, whereas 7 of 22 patients developed focal temporal changes, including focal spike and focal slow wave activation. The development of focal changes correlated well with clinical data concerning underlying focal pathology; focal abnormalities were not evoked in patients with multifocal disease. Hypoglycemic activation of the EEG may be a useful technique for predicting the presence of pathology in patients considered for anterior temporal lobectomy. Kamiyah Morgan is a 6 year old little girl who suffers from a very unusual set of fainting episodes that will leave her unresponsive and immobile, and they can happen up to 300 times a day. When she experiences a fainting episode she will become completely paralyzed affecting everything in her body including her lungs, her mother states that every day that passes her ability to breathe diminishes. Kamiyah’s mother said that these fainting episodes started when she was about 8 months old as she would be crawling and suddenly tip over and go limp. At first their pediatric physician said the episodes looked like she was having a seizure, but after running an EEG there was no seizure being detected during the episodes. They then tried testing with MRI for any brain tumors or malignancies but again there was nothing. They were then referred over to the NIH or the National Institute of Health where their entire purpose is to be able to research and hopefully diagnose very strange cases. Diagnosis To be diagnosed with PTE, a person must have a history of head trauma and no history of seizures prior to the injury. Witnessing a seizure is the most effective way to diagnose PTE. Electroencephalography (EEG) is a tool used to diagnose a seizure disorder, but a large portion of people with PTE may not have the abnormal \"epileptiform\" EEG findings indicative of epilepsy. In one study, about a fifth of people who had normal EEGs three months after an injury later developed PTE. However, while EEG is not useful for predicting who will develop PTE, it can be useful to localize the epileptic focus, to determine severity, and to predict whether a person will suffer more seizures if they stop taking antiepileptic medications. Magnetic resonance imaging (MRI) is performed in people with PTE, and CT scanning can be used to detect brain lesions if MRI is unavailable. However, it is frequently not possible to detect the epileptic focus using neuroimaging. We have experienced 10 cases of insulinoma during the last 10 years from 1977 to 1986. All cases had strong hypoglycemic symptoms such as disturbance of consciousness, and insulinoma still tended to be misdiagnosed as epilepsy. The diagnosis of insulinoma was easily available from serum IRI (immunoreactive insulin)/plasma glucose ratio in all of the ten cases. As preoperative procedures for the diagnosis of localization, arteriography, computed tomography and portal blood sampling were positive in 6 of 8, 4 of 6 and 2 of 2 patients, respectively. At operation, all insulinomas could be identified by digital palpation. We performed simple excision of the tumor in 6 patients and distal pancreatectomy in 4 patients. The tumors were solitary and benign in all patients, ranging in size from 1.0 cm to 4.5 cm. Three cases were presented as case reports. In these cases, portal blood sampling and/or intraoperative monitoring of plasma glucose and serum IRI were performed. Portal blood sampling was effective even for a case which was negative in image diagnostic procedures. Furthermore, simultaneous monitoring of plasma glucose and serum IRI by quick radioimmunoassay seemed to be a good guide to the completeness of resection of insulin producing tumors. Epilepsy monitoring is typically done to distinguish epileptic seizures from other types of spells, such as psychogenic non-epileptic seizures, syncope (fainting), sub-cortical movement disorders and migraine variants, to characterize seizures for the purposes of treatment, and to localize the region of brain from which a seizure originates for work-up of possible seizure surgery. Hospitals use an EEG monitor to help diagnose a seizure. They use that information to help with the treatment process as well as discovering risks. \"Many professionals have stated the importance of EEG’s when it comes to suspected seizures, for diagnosis and evaluation\". Doctors will be able to use the EEG monitoring system to help look at some treatment options as well as some risk factors. As technology advances, researchers are finding new monitors that are more accurate in regards to seizures. \"Advanced techniques with continuous EEG and simplified technique with aEEG allows clinicians to detect more We performed interictal 18F-2-fluoro-2-deoxy-D-glucose positron emission tomography (18FDG-PET) studies in 57 patients with complex partial epilepsy (CPE), not controlled by medical treatment and considered for surgical resection of their epileptic focus. A precise localization of the epileptic focus was obtained in 37 of these patients with a combination of subdural and depth electrodes. We visually inspected the metabolic images; we also measured glucose consumption in a number of brain regions and compared the values with those obtained in 17 normal controls. Eighty-two percent of the 57 patients had an area of glucose hypometabolism on the 18FDG-PET images. Six patients had a frontal epileptic focus, 3 of them had a frontal lobe hypometabolism. Twenty-six patients had a unilateral temporal lobe focus and all of them displayed a temporal lobe hypometabolism. The asymmetry was more pronounced in the lateral temporal cortex (-20%) than in the mesial part of the temporal lobe (-9.6%). In each cortical brain region on the side of the epileptic focus (except the sensorimotor cortex), glucose consumption rate was lower than in the contralateral region or than in controls. No differences could be found between patients with a seizure onset restricted to the hippocampus and patients with a seizure onset involving the hippocampus and the adjacent neocortex. Divergent metabolic patterns were obtained in 5 patients with bilateral temporal seizure foci. Combined with other non invasive techniques (EEG, neuroradiology), PET contributes increasingly to the selection of patients with CPE who could benefit from surgical treatment.(ABSTRACT TRUNCATED AT 250 WORDS) Although insulinoma constitutes almost 90% of neuroendocrine tumors localized in the pancreas, it is a rare disease. Quite commonly prior to the diagnosis there is a history of several years and misdiagnosis as neurological or cardiological disease is not infrequent. Patient, 22 years old, since 7 years experiencing multiple incidents of neuroglycopenia with concurrent hyperadrenergic reaction. Consciousness disturbances and muscle tremor together with feeling of hunger and tachycardia occurred mainly in the morning hours, or after physical exercise, and subsided after glucose intake. Increase in body weight, typical for insulinoma, was also observed. The patient was hospitalized twice in Pediatric Department and although hypoglycemia was observed, no additional testing was performed to exclude insulinoma; reported symptoms and abnormalities in EEG recording after provocation resulted in diagnosis and treatment of epilepsy. During hospitalization in the Department of Endocrinology fasting test was performed, which revealed inadequately high insulin level with glucose level of 41 mg% and signs of neuroglycopenia. The image of pancreas was normal in the acquired abdominal ultrasound and in CT a tumor was found in the tail of pancreas. The patient underwent laparoscopic operation and the clinical diagnosis was confirmed by histopathology. Antiepileptic drugs were discontinued. Total remission of symptoms was achieved. The presented case demonstrates the difficulties in correct interpretation of reported symptoms, while the results of biochemical tests and imaging studies point precisely to the diagnosis. Focal neurological signs resulting from multiple episodes of hypoglycemia may lead to misdiagnosis and treatment of epilepsia. Four patients with a well-established diagnosis of tuberous sclerosis and grand mal type epileptic seizures as their principal clinical symptom were examined by conventional surface electroencephalography (EEG), X-ray computed tomography, and positron emission tomography (PET) using the [18F]-2-fluoro-2-deoxyglucose method. The interictal EEG showed various abnormalities of poor localizing value, but no focal epileptic discharges. X-ray computed tomography demonstrated subependymal calcifications in all cases, although cortical lesions were found only twice. However, in the PET images of each patient one or two localized cortical foci with a metabolic rate for glucose more than 40% lower than in the respective contralateral region were clearly delineated. It may be assumed that those hypometabolic areas represent the epileptogenic cortical tubers, which are characteristic of the disease but usually cannot be detected in vivo by other methods. In adults, testing electrolytes, blood glucose and calcium levels is important to rule these out as causes, as is an electrocardiogram. A lumbar puncture may be useful to diagnose a central nervous system infection but is not routinely needed. Routine antiseizure medical levels in the blood are not required in adults or children. In children additional tests may be required. A high blood prolactin level within the first 20 minutes following a seizure may be useful to confirm an epileptic seizure as opposed to psychogenic non-epileptic seizure. Serum prolactin level is less useful for detecting partial seizures. If it is normal an epileptic seizure is still possible and a serum prolactin does not separate epileptic seizures from syncope. It is not recommended as a routine part of diagnosis epilepsy. Epilepsy Macropsia may present itself as a symptom of both frontal lobe epilepsy and temporal lobe epilepsy, which may actually help in the diagnosis of those diseases. Children who experience nocturnal hallucinations accompanied by macropsia may seek medical care for panic attack disorders and instead are diagnosed with forms of epilepsy. Epilepsy patients may have no memory of the seizure, but can remember the hallucinations and aura which proceed the attack. Electroencephalography, or EEG imaging, can then be utilized while the patient experiences the episode. It may be subsequently concluded that the EEG is congruent with temporal or frontal lobe seizure. Anxiety and headaches accompany the episodes of visual distortion associated with epilepsy. While Valproic acid has been used to treat this type of seizure, anti-seizure medications appropriate for focal-onset seizures, like oxcarbazapine, have also been used successfully in the treatment of epilepsy-related macropsia. Four patients with primary generalized or true petit mal epilepsy were studied with positron emission tomography using [18F]fluorodeoxyglucose (FDG). FDG studies were carried out during 10 minutes of hyperventilation before and again after medical control of spontaneous absences. Before seizures were controlled all 4 patients demonstrated frequent bilaterally synchronous three-per-second spike-and-wave discharges associated with altered consciousness. After spontaneous seizures were controlled, hyperventilation produced only electroencephalographic slowing without clinical symptoms in 3; the fourth patient had absences less frequently. Patterns of local cerebral metabolic rate for glucose (CMRGlc) were normal and identical for ictal and interictal scans; there was, however, a 2.5- to 3.5-fold diffuse ictal increase in global CMRGlc evident when ictal studies were compared with hyperventilation control studies in which no seizures occurred. The CMRGlc was similar in the two scans obtained from the patient who had absences during both studies. No anatomical substrate of petit mal epilepsy was identified. The CMRGlc in these patients during petit mal absences was higher than that recorded in other patients during partial or generalized convulsive seizures. This difference may reflect the fact that petit mal absences are not associated with postictal depression. For the diagnosis it is necessary to perform an EEG during the headache that shows epilepsy-compatible discharges coinciding with the onset and cessation of the headache. The so-called hemicrania epileptica is a variant of EH characterized by the fact that head pain and EEG paroxysms are located on the same side. MRI is necessary to establish the cause, which, as in all focal epilepsies, can be varied: malformations/dysplasia, neoplasms, encephalopathies, traumatic brain injury, vasculopathies. Therapy. It depends on the etiology. During the headache, like most seizures, i.v. benzodiazepines are usually effective. Antiepileptic drugs can be used as preventive. 2. Ictal non-epileptic headache. Rare cases are reported. It is a condition that can be differentiated with certainty from the previous one if the headache episode is also present outside the seizure, that is, before and/or after, without specific EEG abnormalities. References External links Interictal positron computed tomography (PCT) with 18F-fluorodeoxyglucose was performed on 50 patients with partial seizures disorders. Electroencephalographic (EEG) monitoring was carried out during the metabolic studies using scalp and sphenoidal electrodes in 33 patients and stereotaxically implanted depth electrodes in 17. Four patients in this series had focal abnormalities on x-ray computed tomographic scans, but these were at the site of the presumed epileptogenic lesion in only 2. One or more discrete zones of hypometabolism were identified in 35 patients, and only 1 patient appeared to show focal interictal hypermetabolism. No quantitative relationship could be demonstrated between the degree of focal hypometabolism and either the frequency of interictal EEG spikes of the presence of focal nonepileptiform EEG changes. It was concluded that metabolic and electrophysiological techniques measure different aspects of cerebral dysfunction in seizure disorders. Although interictal PCT in patients with partial epilepsy usually demonstrates zones of hypometabolism this finding, per se, does not reveal the epileptic nature of the abnormality. Other methods to diagnose Unverricht–Lundborg disease are currently being explored. While electroencephalogram (EEG) is useful in identifying or diagnosing other forms of epilepsy, the location of seizures in ULD is currently known to be generalized across the entire brain. Without a specific region to pinpoint, it is difficult to accurately distinguish an EEG reading from an individual with ULD from an individual with another type of epilepsy characterized by generalized brain seizures. However, with recent research linking ULD brain damage to the hippocampus, the usefulness of EEG as a diagnostic tool may increase. Magnetic Resonance Imaging (MRI) is also often used during diagnosis of patients with epilepsy. While MRIs taken during the onset of the disease are generally similar to those of individuals without ULD, MRIs taken once the disease has progressed show characteristic damage This study reports on the clinical, electrophysiologic, and neuroradiologic aspects of patients with epilepsy secondary to neonatal hypoglycemia. Fifteen patients with epilepsy and/or posterior cerebral lesions, and neonatal hypoglycemia were studied in the epilepsy clinic between February 1990 and March 2003. The mean age was 12 years. The different types of neonatal hypoglycemia were as follows: four patients had transitional-adaptive, seven classic transient, two secondary-associated, and two severe recurrent hypoglycemia. As to epilepsy, we recognized a larger group of 12 patients characterized by focal seizures and posterior abnormalities on the electroencephalogram, the majority of whom had a good outcome, and a second group of two patients presenting electroclinical features of encephalopathy with refractory seizures. All patients except two manifested parieto-occipital lesions on neuroradiologic images. Neurologic examination was normal in one patient. Six patients had microcephaly; eight manifested visual disturbances. Fourteen patients were mentally retarded. One had a pervasive developmental disorder. This study indicates neonatal hypoglycemia may cause posterior cerebral lesions, abnormal findings at neurologic examination, and symptomatic epilepsy, most frequently occipital lobe epilepsy, usually with a good prognosis, and occasionally epileptic encephalopathy with refractory seizures. MRI studies are essential to define the characteristics of cerebral lesions after neonatal hypoglycemia. Using the 2-[F-18]fluorodeoxyglucose method, 213 positron emission tomographic (PET) studies of local brain glucose metabolism (CMRglu) were performed in 124 patients with various forms of epilepsy. Interictal PET scans of primary epileptics typically showed some global metabolic depression and decreased functional activity of insular, basal and anterior temporal cortex. Epilepsia partialis continua Kozevnikov was characterized by hypo- or hyper-metabolism of perirolandic cortex. Tuberous sclerosis was distinguished by neocortical foci of significantly decreased glucose consumption. Even in the interictal resting state, with regard to sensitivity (greater than 90%) and accuracy of focus localization. PET was superior to other diagnostic methods in typical temporal lobe epilepsy. Averaging 23% below normal CMRglu, the majority of hypometabolic foci were found in mesial temporal structures. Improved distinction between the epileptogenic area and the surrounding tissue showing comparatively normal functional responsiveness, was achieved by psychophysical activation using emotional speech or continuous visual recognition during PET scanning. In patients who had undergone total cerebral hemispherectomy because of uncontrolled epilepsy, remarkable recruitment of association areas was observed on both motor and speech activation. Continuous partial epilepsy (CPE) is characterized by isolated, subintrant clonus focalized to a limited territory with critical focal electroencephalography in a concordant territory. CPE is observed in various cortical lesions but also in disorders of metabolism and notably decompensated diabetes mellitus. We report a case of CPE without focal lesion at MRI which revealed hyperglycaemia without ketosis. The 54-year old female patient was hospitalised for C.P.E.. Early CT and later MRI gave normal results. Biochemistry showed hyperglycaemia without kenoturia, acidosis or hyperosmolality. Insulin therapy rapidly brought glycaemia down to its normal level and the clonsism disappeared. Five months later, the patient had no other seizure and the EEG was normal. Epileptic seizures are frequent in hyperglycaemia without ketosis (25% of the cases) where they are mainly partial and motor (75 to 86% of the cases), rarely associated with a focal lesion (15% of the cases with CT scan). They are rare in patients with ketoacidosis. This apparent protective effect of ketoacidosis may be attributed to an increase of GABA bioavailability consecutive to acidosis. CPE is resistant to antiepileptic treatments. In CPE induced by hyperglycaemia without ketosis normalization of blood glucose level with insulin therapy is concomitant with a rapid cure of epilepsy. Thus glycaemia should be measured in all patients presenting with CPE, the aim being to diagnose hyperglycaemia without ketosis rapidly to avoid hyperosmolality and to prescribe an adequate treatment based exclusively on insulin and rehydration. This 71 years old women without any history of epilepsy had diabetes mellitus. She was admitted for repetitive giratory seizures in relation with non-ketotic hyperglycaemia. The EEG showed right centro-parietal paroxysmal slow activity. Symptomatology disappeared within 48 hours after insulin therapy. One month later, she presented with a left hemiplegia in relation with a right sylvian infraction. The role of focal transitory ischaemia in connection with hyperglycaemia is discussed. EEG is most often used to diagnose epilepsy, which causes abnormalities in EEG readings. It is also used to diagnose sleep disorders, depth of anesthesia, coma, encephalopathies, and brain death. EEG used to be a first-line method of diagnosis for tumors, stroke and other focal brain disorders, but this use has decreased with the advent of high-resolution anatomical imaging techniques such as magnetic resonance imaging (MRI) and computed tomography (CT). Despite limited spatial resolution, EEG continues to be a valuable tool for research and diagnosis. It is one of the few mobile techniques available and offers millisecond-range temporal resolution which is not possible with CT, PET or MRI.\nHere is the question:\nNEUROLOGY AND NEUROSURGERY: In a patient diagnosed with epilepsy who presents with episodes of unresponsiveness to external stimuli, irregular movements of all four limbs, closed eyes, crying and pelvic movements, lasting five to twenty seconds and unresponsive to treatment with antiepileptic drugs, which complementary study is most likely to clarify the diagnosis?\nHere are the potential choices:\n1. Video-EEG monitoring for diagnosis of pseudocrisis (psychogenic seizures).\n2. Holter ECG for diagnosis of arrhythmic heart disease.\n3. Routine EEG to diagnose the type of epilepsy (generalized or foc).\n4. Brain MRI to detect epileptogenic lesions (cortical dysplasia, tumor, medial temporal sclerosis).\n5. Determine capillary blood glucose for diagnosis of hypoglycemia.\nThe correct answer is: ", "gold_answer": "1 Video-EEG monitoring for diagnosis of pseudocrisis (psychogenic seizures).", "prediction": "You are a helpful medical expert, and your task is to answer a multi-choice medical question using the relevant documents. Please choose the answer from the provided options. Your responses will be used for research purposes only, so please have a definite answer.\nHere are the relevant documents:\n Other means used in diagnosis of vertiginous epilepsy include: Electroencephalography (EEG) Magnetic resonance imaging (MRI) Positron emission tomography (PET) Neuropsychological testing The EEG measures electrical activity in the brain, allowing a physician to identify any unusual patterns. While EEGs are good for identifying abnormal brain activity is it not helpful in localizing where the seizure originates because they spread so quickly across the brain. MRIs are used to look for masses or lesions in the temporal lobe of the brain, indicating possible tumors or cancer as the cause of the seizures. When using a PET scan, a physician is looking to detect abnormal blood flow and glucose metabolism in the brain, which is visible between seizures, to indicate the region of origin. Management The most definitive test to distinguish epilepsy from PNES is long term video-EEG monitoring, with the aim of capturing one or two episodes on both video recording and EEG simultaneously (some clinicians may use suggestion to attempt to trigger an episode). Additional clinical criteria are usually considered in addition to video-EEG monitoring when diagnosing PNES. By recording the event in question on video and EEG simultaneously, a clear diagnosis can usually be obtained. Laboratory testing can detect rising blood levels of serum prolactin if samples are taken in the right time window after most tonic-clonic or complex partial epileptic seizures. However, due to false positives and variability in results, this test is relied upon less frequently. The pathogenesis of PED has also been linked to mutations in the GLUT1 glucose transporter which can result in transient energy deficits in the basal ganglia. Diagnosis Diagnosis is similar, but slightly different for each type of PD. Some types are more understood than others, and therefore have more criteria for diagnosis. PKD The guidelines for diagnosing PKD were reviewed and confirmed by Unterberger and Trinka. PKD consists of unexpected forms of involuntary movements of the body. The patient is usually diagnosed sometime before their 20s, and is more likely diagnosed during childhood than early adulthood. Almost all PKD's are idiopathic, but there have been examples of autosomal dominant inheritance as well. Physical examination and brain imaging examinations show normal results, and an EEG shows no specific abnormalities as well. However, the negative synchronous EEG results can be used to prove that PKD is not a sort of reflex epilepsy, but a different disease. To confirm diagnosis, awake and asleep EEG and magnetic resonance imaging (MRI) are performed. MRI is used to detect focal brain lesions. Ruling out other diagnoses Certain diagnoses must be ruled out before diagnosing LGS. These diagnoses are: Doose syndrome Dravet syndrome pseudo-Lennox Gastaut syndrome (atypical benign partial epilepsy) LGS is more easily distinguished from Doose syndrome by seizure type after the syndrome has progressed. Doose syndrome has more myoclonic seizures and LGS has more tonic seizures. The Doose syndromes is less likely to have cognitive disabilities. Pseudo-Lennox–Gastaut syndrome can be distinguished from LGS because pseudo-LGS has different spike-and-wave patterns on EEG. Treatment There are several treatment options, including medications, surgery, and diet. Medications In most patients with LGS, the treatment does not end seizure recurrence. There exist various morphological and biochemical changes closely associated with electrophysiological phenomena which cause epileptic seizures in the brains of epilepsy patients. Recent developments in investigation methods, not only electrophysiological(EEG and MEG), but also neuroimaging involving morphological imaging(CT and conventional MRI) and functional imaging(SPECT, PET, functional MRI and MRS) is able to demonstrate these changes. SPECT and PET can particularly clarify the changes of cerebral blood flow and glucose metabolism between interictal and ictal periods. In our experience of 423 patients who underwent epilepsy surgery for intractable seizures, these interventions provide important information to identify the epileptogenic foci. However, in practice, discordance in the results of these presurgical evaluations is recognized, and invasive intracranial recordings are needed in such cases. These problems in diagnosis were shown especially in patients with mesial temporal sclerosis and focal cortical dysplasia. To detect an epileptogenic focus more clearly, a combination of morphological and functional findings, new functional imaging such as neurotransmitter receptor imaging, EEG-triggered or neuropharmacological functional MRI, as well as, statistical parametric analysis may be needed. Diagnosis Diagnosis of PME is based on the individual’s signs and symptoms as well as failure to respond to antiepileptic drugs and therapy. Further diagnosis support includes EEG results, genetic testing, enzyme testing, and skin and muscle biopsies. Gaucher’s disease can be diagnosed through enzyme testing as it is a metabolic disease. Lafora’s disease can be diagnosed using skin biopsies. While Action myoclonus renal failure (AMRF) syndrome can only be diagnosed using genetic test. Using EEG’s as a form of diagnosis can prove difficult as patients differ in their neurophysiology. In Lafora’s disease EEGs can show slowing background activity or focal discharges as well as epileptiform discharges. In ULD EEGs show generalized epileptiform discharges and in MERRF patients show background slowing. Therefore, diagnosis is best made using a combination of different tools like signs and symptoms, age of onset, EEG, gene testing, enzyme measurements, and biopsy of skin and muscle. For adults, the testing of electrolyte, blood glucose and calcium levels is important to rule out problems with these as causes. An electrocardiogram can rule out problems with the rhythm of the heart. A lumbar puncture may be useful to diagnose a central nervous system infection but is not routinely needed. In children additional tests may be required such as urine biochemistry and blood testing looking for metabolic disorders. Together with EEG and neuroimaging, genetic testing is becoming one of the most important diagnostic technique for epilepsy, as a diagnosis might be achieved in a relevant proportion of cases with severe epilepsies, both in children and adults. For those with negative genetic testing, in some it might be important to repeat or re-analyze previous genetic studies after 2–3 years. Diagnosis The diagnosis of epilepsy is typically made based on observation of the seizure onset and the underlying cause. An electroencephalogram (EEG) to look for abnormal patterns of brain waves and neuroimaging (CT scan or MRI) to look at the structure of the brain are also usually part of the initial investigations. While figuring out a specific epileptic syndrome is often attempted, it is not always possible. Video and EEG monitoring may be useful in difficult cases. Definition Epilepsy is a disorder of the brain defined by any of the following conditions: {| cellpadding=5 style=\"border:1px solid #ccc\" |- bgcolor=\"#fafafa\" | At least two unprovoked (or reflex) seizures occurring more than 24 hours apart One unprovoked (or reflex) seizure and a probability of further seizures similar to the general recurrence risk (at least 60%) after two unprovoked seizures, occurring over the next 10 years Diagnosis of an epilepsy syndrome |} The goal of this study was to estimate the frequency of psychogenic pseudosyncope in patients with \"syncope of unknown origin.\" Twenty to thirty percent of patients referred to epilepsy centers for refractory seizures have psychogenic seizures. With syncope, about 20-30% of the cases remain unexplained after a complete evaluation, but, unlike in seizures, a psychogenic etiology is not usually investigated. We prospectively evaluated patients referred to our epilepsy center for evaluation of recurrent syncope-like episodes, that is, limp, motionless fainting. All patients had a negative syncope workup. We performed EEG-video monitoring with activation by suggestion (\"induction\"), similar to what is used for diagnosis of psychogenic seizures. Activation was performed with patients standing or sitting up. The diagnosis of psychogenic pseudosyncope required: (1) an activation procedure that triggered the habitual event; (2) a clinical event of loss of postural tone and limp, motionless unresponsiveness with eyes closed; (3) normal EEG before, during, and after the clinical event, that is, no epileptiform abnormalities, a normal alpha rhythm during unresponsiveness, and no suppression of background or slowing as is typically seen in syncope. Ten patients were recruited over an 18-month period. Habitual syncope-like episodes were triggered in 9 of 10 (90%) patients, and all 9 were shown to have psychogenic pseudosyncope (eyes closed, motionless, unresponsive with normal EEG including normal alpha rhythm). In one patient, no episode was triggered, so a diagnosis could not be made. Among the 9 patients for whom episodes were recorded, age ranged from 21 to 60 (mean=36). Five were women. Duration of symptoms ranged from 6 months to 15 years (mean=4.2 years). Event frequency ranged from four per day to two per month. Prior evaluations for syncope included ECG in all patients, two-dimensional echocardiogram in three, Holter monitoring in two, and tilt-table test in five. Four patients had undergone cardiac catheterization, and one had received a pacemaker. Neurologic tests included CT of the head in seven and MRI of the brain in eight. Many patients with \"syncope of unknown origin\" may have psychogenic pseudosyncope, but most such patients do not undergo EEG-video monitoring, which is the only way to demonstrate a psychogenic etiology. Psychogenic pseudosyncope is not simply a diagnosis of exclusion, and can be firmly diagnosed. As is usually recommended for seizure-like events, patients with syncope-like events and a negative evaluation should undergo EEG-video monitoring with induction, specifically looking for a possible psychogenic etiology. EEGs were recorded in 22 patients with medically refractory complex partial epilepsy undergoing presurgical evaluation and 11 age-matched controls while subjected to moderate levels of hypoglycemia to determine if changes activated were predictive of underlying pathology. Five patients had fasting EEGs showing focal abnormalities not seen in the non-fasting state. With hypoglycemia, EEG tracings in normal individuals showed diffuse background slowing, whereas 7 of 22 patients developed focal temporal changes, including focal spike and focal slow wave activation. The development of focal changes correlated well with clinical data concerning underlying focal pathology; focal abnormalities were not evoked in patients with multifocal disease. Hypoglycemic activation of the EEG may be a useful technique for predicting the presence of pathology in patients considered for anterior temporal lobectomy. Kamiyah Morgan is a 6 year old little girl who suffers from a very unusual set of fainting episodes that will leave her unresponsive and immobile, and they can happen up to 300 times a day. When she experiences a fainting episode she will become completely paralyzed affecting everything in her body including her lungs, her mother states that every day that passes her ability to breathe diminishes. Kamiyah’s mother said that these fainting episodes started when she was about 8 months old as she would be crawling and suddenly tip over and go limp. At first their pediatric physician said the episodes looked like she was having a seizure, but after running an EEG there was no seizure being detected during the episodes. They then tried testing with MRI for any brain tumors or malignancies but again there was nothing. They were then referred over to the NIH or the National Institute of Health where their entire purpose is to be able to research and hopefully diagnose very strange cases. Diagnosis To be diagnosed with PTE, a person must have a history of head trauma and no history of seizures prior to the injury. Witnessing a seizure is the most effective way to diagnose PTE. Electroencephalography (EEG) is a tool used to diagnose a seizure disorder, but a large portion of people with PTE may not have the abnormal \"epileptiform\" EEG findings indicative of epilepsy. In one study, about a fifth of people who had normal EEGs three months after an injury later developed PTE. However, while EEG is not useful for predicting who will develop PTE, it can be useful to localize the epileptic focus, to determine severity, and to predict whether a person will suffer more seizures if they stop taking antiepileptic medications. Magnetic resonance imaging (MRI) is performed in people with PTE, and CT scanning can be used to detect brain lesions if MRI is unavailable. However, it is frequently not possible to detect the epileptic focus using neuroimaging. We have experienced 10 cases of insulinoma during the last 10 years from 1977 to 1986. All cases had strong hypoglycemic symptoms such as disturbance of consciousness, and insulinoma still tended to be misdiagnosed as epilepsy. The diagnosis of insulinoma was easily available from serum IRI (immunoreactive insulin)/plasma glucose ratio in all of the ten cases. As preoperative procedures for the diagnosis of localization, arteriography, computed tomography and portal blood sampling were positive in 6 of 8, 4 of 6 and 2 of 2 patients, respectively. At operation, all insulinomas could be identified by digital palpation. We performed simple excision of the tumor in 6 patients and distal pancreatectomy in 4 patients. The tumors were solitary and benign in all patients, ranging in size from 1.0 cm to 4.5 cm. Three cases were presented as case reports. In these cases, portal blood sampling and/or intraoperative monitoring of plasma glucose and serum IRI were performed. Portal blood sampling was effective even for a case which was negative in image diagnostic procedures. Furthermore, simultaneous monitoring of plasma glucose and serum IRI by quick radioimmunoassay seemed to be a good guide to the completeness of resection of insulin producing tumors. Epilepsy monitoring is typically done to distinguish epileptic seizures from other types of spells, such as psychogenic non-epileptic seizures, syncope (fainting), sub-cortical movement disorders and migraine variants, to characterize seizures for the purposes of treatment, and to localize the region of brain from which a seizure originates for work-up of possible seizure surgery. Hospitals use an EEG monitor to help diagnose a seizure. They use that information to help with the treatment process as well as discovering risks. \"Many professionals have stated the importance of EEG’s when it comes to suspected seizures, for diagnosis and evaluation\". Doctors will be able to use the EEG monitoring system to help look at some treatment options as well as some risk factors. As technology advances, researchers are finding new monitors that are more accurate in regards to seizures. \"Advanced techniques with continuous EEG and simplified technique with aEEG allows clinicians to detect more We performed interictal 18F-2-fluoro-2-deoxy-D-glucose positron emission tomography (18FDG-PET) studies in 57 patients with complex partial epilepsy (CPE), not controlled by medical treatment and considered for surgical resection of their epileptic focus. A precise localization of the epileptic focus was obtained in 37 of these patients with a combination of subdural and depth electrodes. We visually inspected the metabolic images; we also measured glucose consumption in a number of brain regions and compared the values with those obtained in 17 normal controls. Eighty-two percent of the 57 patients had an area of glucose hypometabolism on the 18FDG-PET images. Six patients had a frontal epileptic focus, 3 of them had a frontal lobe hypometabolism. Twenty-six patients had a unilateral temporal lobe focus and all of them displayed a temporal lobe hypometabolism. The asymmetry was more pronounced in the lateral temporal cortex (-20%) than in the mesial part of the temporal lobe (-9.6%). In each cortical brain region on the side of the epileptic focus (except the sensorimotor cortex), glucose consumption rate was lower than in the contralateral region or than in controls. No differences could be found between patients with a seizure onset restricted to the hippocampus and patients with a seizure onset involving the hippocampus and the adjacent neocortex. Divergent metabolic patterns were obtained in 5 patients with bilateral temporal seizure foci. Combined with other non invasive techniques (EEG, neuroradiology), PET contributes increasingly to the selection of patients with CPE who could benefit from surgical treatment.(ABSTRACT TRUNCATED AT 250 WORDS) Although insulinoma constitutes almost 90% of neuroendocrine tumors localized in the pancreas, it is a rare disease. Quite commonly prior to the diagnosis there is a history of several years and misdiagnosis as neurological or cardiological disease is not infrequent. Patient, 22 years old, since 7 years experiencing multiple incidents of neuroglycopenia with concurrent hyperadrenergic reaction. Consciousness disturbances and muscle tremor together with feeling of hunger and tachycardia occurred mainly in the morning hours, or after physical exercise, and subsided after glucose intake. Increase in body weight, typical for insulinoma, was also observed. The patient was hospitalized twice in Pediatric Department and although hypoglycemia was observed, no additional testing was performed to exclude insulinoma; reported symptoms and abnormalities in EEG recording after provocation resulted in diagnosis and treatment of epilepsy. During hospitalization in the Department of Endocrinology fasting test was performed, which revealed inadequately high insulin level with glucose level of 41 mg% and signs of neuroglycopenia. The image of pancreas was normal in the acquired abdominal ultrasound and in CT a tumor was found in the tail of pancreas. The patient underwent laparoscopic operation and the clinical diagnosis was confirmed by histopathology. Antiepileptic drugs were discontinued. Total remission of symptoms was achieved. The presented case demonstrates the difficulties in correct interpretation of reported symptoms, while the results of biochemical tests and imaging studies point precisely to the diagnosis. Focal neurological signs resulting from multiple episodes of hypoglycemia may lead to misdiagnosis and treatment of epilepsia. Four patients with a well-established diagnosis of tuberous sclerosis and grand mal type epileptic seizures as their principal clinical symptom were examined by conventional surface electroencephalography (EEG), X-ray computed tomography, and positron emission tomography (PET) using the [18F]-2-fluoro-2-deoxyglucose method. The interictal EEG showed various abnormalities of poor localizing value, but no focal epileptic discharges. X-ray computed tomography demonstrated subependymal calcifications in all cases, although cortical lesions were found only twice. However, in the PET images of each patient one or two localized cortical foci with a metabolic rate for glucose more than 40% lower than in the respective contralateral region were clearly delineated. It may be assumed that those hypometabolic areas represent the epileptogenic cortical tubers, which are characteristic of the disease but usually cannot be detected in vivo by other methods. In adults, testing electrolytes, blood glucose and calcium levels is important to rule these out as causes, as is an electrocardiogram. A lumbar puncture may be useful to diagnose a central nervous system infection but is not routinely needed. Routine antiseizure medical levels in the blood are not required in adults or children. In children additional tests may be required. A high blood prolactin level within the first 20 minutes following a seizure may be useful to confirm an epileptic seizure as opposed to psychogenic non-epileptic seizure. Serum prolactin level is less useful for detecting partial seizures. If it is normal an epileptic seizure is still possible and a serum prolactin does not separate epileptic seizures from syncope. It is not recommended as a routine part of diagnosis epilepsy. Epilepsy Macropsia may present itself as a symptom of both frontal lobe epilepsy and temporal lobe epilepsy, which may actually help in the diagnosis of those diseases. Children who experience nocturnal hallucinations accompanied by macropsia may seek medical care for panic attack disorders and instead are diagnosed with forms of epilepsy. Epilepsy patients may have no memory of the seizure, but can remember the hallucinations and aura which proceed the attack. Electroencephalography, or EEG imaging, can then be utilized while the patient experiences the episode. It may be subsequently concluded that the EEG is congruent with temporal or frontal lobe seizure. Anxiety and headaches accompany the episodes of visual distortion associated with epilepsy. While Valproic acid has been used to treat this type of seizure, anti-seizure medications appropriate for focal-onset seizures, like oxcarbazapine, have also been used successfully in the treatment of epilepsy-related macropsia. Four patients with primary generalized or true petit mal epilepsy were studied with positron emission tomography using [18F]fluorodeoxyglucose (FDG). FDG studies were carried out during 10 minutes of hyperventilation before and again after medical control of spontaneous absences. Before seizures were controlled all 4 patients demonstrated frequent bilaterally synchronous three-per-second spike-and-wave discharges associated with altered consciousness. After spontaneous seizures were controlled, hyperventilation produced only electroencephalographic slowing without clinical symptoms in 3; the fourth patient had absences less frequently. Patterns of local cerebral metabolic rate for glucose (CMRGlc) were normal and identical for ictal and interictal scans; there was, however, a 2.5- to 3.5-fold diffuse ictal increase in global CMRGlc evident when ictal studies were compared with hyperventilation control studies in which no seizures occurred. The CMRGlc was similar in the two scans obtained from the patient who had absences during both studies. No anatomical substrate of petit mal epilepsy was identified. The CMRGlc in these patients during petit mal absences was higher than that recorded in other patients during partial or generalized convulsive seizures. This difference may reflect the fact that petit mal absences are not associated with postictal depression. For the diagnosis it is necessary to perform an EEG during the headache that shows epilepsy-compatible discharges coinciding with the onset and cessation of the headache. The so-called hemicrania epileptica is a variant of EH characterized by the fact that head pain and EEG paroxysms are located on the same side. MRI is necessary to establish the cause, which, as in all focal epilepsies, can be varied: malformations/dysplasia, neoplasms, encephalopathies, traumatic brain injury, vasculopathies. Therapy. It depends on the etiology. During the headache, like most seizures, i.v. benzodiazepines are usually effective. Antiepileptic drugs can be used as preventive. 2. Ictal non-epileptic headache. Rare cases are reported. It is a condition that can be differentiated with certainty from the previous one if the headache episode is also present outside the seizure, that is, before and/or after, without specific EEG abnormalities. References External links Interictal positron computed tomography (PCT) with 18F-fluorodeoxyglucose was performed on 50 patients with partial seizures disorders. Electroencephalographic (EEG) monitoring was carried out during the metabolic studies using scalp and sphenoidal electrodes in 33 patients and stereotaxically implanted depth electrodes in 17. Four patients in this series had focal abnormalities on x-ray computed tomographic scans, but these were at the site of the presumed epileptogenic lesion in only 2. One or more discrete zones of hypometabolism were identified in 35 patients, and only 1 patient appeared to show focal interictal hypermetabolism. No quantitative relationship could be demonstrated between the degree of focal hypometabolism and either the frequency of interictal EEG spikes of the presence of focal nonepileptiform EEG changes. It was concluded that metabolic and electrophysiological techniques measure different aspects of cerebral dysfunction in seizure disorders. Although interictal PCT in patients with partial epilepsy usually demonstrates zones of hypometabolism this finding, per se, does not reveal the epileptic nature of the abnormality. Other methods to diagnose Unverricht–Lundborg disease are currently being explored. While electroencephalogram (EEG) is useful in identifying or diagnosing other forms of epilepsy, the location of seizures in ULD is currently known to be generalized across the entire brain. Without a specific region to pinpoint, it is difficult to accurately distinguish an EEG reading from an individual with ULD from an individual with another type of epilepsy characterized by generalized brain seizures. However, with recent research linking ULD brain damage to the hippocampus, the usefulness of EEG as a diagnostic tool may increase. Magnetic Resonance Imaging (MRI) is also often used during diagnosis of patients with epilepsy. While MRIs taken during the onset of the disease are generally similar to those of individuals without ULD, MRIs taken once the disease has progressed show characteristic damage This study reports on the clinical, electrophysiologic, and neuroradiologic aspects of patients with epilepsy secondary to neonatal hypoglycemia. Fifteen patients with epilepsy and/or posterior cerebral lesions, and neonatal hypoglycemia were studied in the epilepsy clinic between February 1990 and March 2003. The mean age was 12 years. The different types of neonatal hypoglycemia were as follows: four patients had transitional-adaptive, seven classic transient, two secondary-associated, and two severe recurrent hypoglycemia. As to epilepsy, we recognized a larger group of 12 patients characterized by focal seizures and posterior abnormalities on the electroencephalogram, the majority of whom had a good outcome, and a second group of two patients presenting electroclinical features of encephalopathy with refractory seizures. All patients except two manifested parieto-occipital lesions on neuroradiologic images. Neurologic examination was normal in one patient. Six patients had microcephaly; eight manifested visual disturbances. Fourteen patients were mentally retarded. One had a pervasive developmental disorder. This study indicates neonatal hypoglycemia may cause posterior cerebral lesions, abnormal findings at neurologic examination, and symptomatic epilepsy, most frequently occipital lobe epilepsy, usually with a good prognosis, and occasionally epileptic encephalopathy with refractory seizures. MRI studies are essential to define the characteristics of cerebral lesions after neonatal hypoglycemia. Using the 2-[F-18]fluorodeoxyglucose method, 213 positron emission tomographic (PET) studies of local brain glucose metabolism (CMRglu) were performed in 124 patients with various forms of epilepsy. Interictal PET scans of primary epileptics typically showed some global metabolic depression and decreased functional activity of insular, basal and anterior temporal cortex. Epilepsia partialis continua Kozevnikov was characterized by hypo- or hyper-metabolism of perirolandic cortex. Tuberous sclerosis was distinguished by neocortical foci of significantly decreased glucose consumption. Even in the interictal resting state, with regard to sensitivity (greater than 90%) and accuracy of focus localization. PET was superior to other diagnostic methods in typical temporal lobe epilepsy. Averaging 23% below normal CMRglu, the majority of hypometabolic foci were found in mesial temporal structures. Improved distinction between the epileptogenic area and the surrounding tissue showing comparatively normal functional responsiveness, was achieved by psychophysical activation using emotional speech or continuous visual recognition during PET scanning. In patients who had undergone total cerebral hemispherectomy because of uncontrolled epilepsy, remarkable recruitment of association areas was observed on both motor and speech activation. Continuous partial epilepsy (CPE) is characterized by isolated, subintrant clonus focalized to a limited territory with critical focal electroencephalography in a concordant territory. CPE is observed in various cortical lesions but also in disorders of metabolism and notably decompensated diabetes mellitus. We report a case of CPE without focal lesion at MRI which revealed hyperglycaemia without ketosis. The 54-year old female patient was hospitalised for C.P.E.. Early CT and later MRI gave normal results. Biochemistry showed hyperglycaemia without kenoturia, acidosis or hyperosmolality. Insulin therapy rapidly brought glycaemia down to its normal level and the clonsism disappeared. Five months later, the patient had no other seizure and the EEG was normal. Epileptic seizures are frequent in hyperglycaemia without ketosis (25% of the cases) where they are mainly partial and motor (75 to 86% of the cases), rarely associated with a focal lesion (15% of the cases with CT scan). They are rare in patients with ketoacidosis. This apparent protective effect of ketoacidosis may be attributed to an increase of GABA bioavailability consecutive to acidosis. CPE is resistant to antiepileptic treatments. In CPE induced by hyperglycaemia without ketosis normalization of blood glucose level with insulin therapy is concomitant with a rapid cure of epilepsy. Thus glycaemia should be measured in all patients presenting with CPE, the aim being to diagnose hyperglycaemia without ketosis rapidly to avoid hyperosmolality and to prescribe an adequate treatment based exclusively on insulin and rehydration. This 71 years old women without any history of epilepsy had diabetes mellitus. She was admitted for repetitive giratory seizures in relation with non-ketotic hyperglycaemia. The EEG showed right centro-parietal paroxysmal slow activity. Symptomatology disappeared within 48 hours after insulin therapy. One month later, she presented with a left hemiplegia in relation with a right sylvian infraction. The role of focal transitory ischaemia in connection with hyperglycaemia is discussed. EEG is most often used to diagnose epilepsy, which causes abnormalities in EEG readings. It is also used to diagnose sleep disorders, depth of anesthesia, coma, encephalopathies, and brain death. EEG used to be a first-line method of diagnosis for tumors, stroke and other focal brain disorders, but this use has decreased with the advent of high-resolution anatomical imaging techniques such as magnetic resonance imaging (MRI) and computed tomography (CT). Despite limited spatial resolution, EEG continues to be a valuable tool for research and diagnosis. It is one of the few mobile techniques available and offers millisecond-range temporal resolution which is not possible with CT, PET or MRI.\nHere is the question:\nNEUROLOGY AND NEUROSURGERY: In a patient diagnosed with epilepsy who presents with episodes of unresponsiveness to external stimuli, irregular movements of all four limbs, closed eyes, crying and pelvic movements, lasting five to twenty seconds and unresponsive to treatment with antiepileptic drugs, which complementary study is most likely to clarify the diagnosis?\nHere are the potential choices:\n1. Video-EEG monitoring for diagnosis of pseudocrisis (psychogenic seizures).\n2. Holter ECG for diagnosis of arrhythmic heart disease.\n3. Routine EEG to diagnose the type of epilepsy (generalized or foc).\n4. Brain MRI to detect epileptogenic lesions (cortical dysplasia, tumor, medial temporal sclerosis).\n5. Determine capillary blood glucose for diagnosis of hypoglycemia.\nThe correct answer is: 1. Video-EEG monitoring for diagnosis of pseudocrisis (psychogenic seizures)." }, { "question": "You are a helpful medical expert, and your task is to answer a multi-choice medical question using the relevant documents. Please choose the answer from the provided options. Your responses will be used for research purposes only, so please have a definite answer.\nHere are the relevant documents:\n The Wiskott-Aldrich syndrome is a primary immunodeficiency characterized by congenital microthrombocytopenia, eczema and recurrent infections. This paper reports the case of a 3-year-6-month male patient, whose maternal uncle died at the age of 3 months due to fulminant sepsis from a pulmonary infection. The patient was a product of the first pregnancy, he was born at 27 weeks' gestation and weighed 1,400 g. As a neonate he was hospitalized during the first 2 months of life because of a low gastrointestinal bleeding, thrombocytopenia and severe infections. In the next 4 months and before coming to our hospital the infant was hospitalized 54 times. On admission he presented disseminated dermatosis, enlarged neck lymph nodes and psychomotor retardation. Laboratory studies revealed hemoglobin 8.1 g/dL, platelets 31,000/uL, mean platelet volume 5.6 fL, IgM 39.3 mg/dL, IgA 67 mg/dL, IgG 1,380 mg/dL. On several occasions he received globular packages and platelet concentrates. The infusion of immunoglobulin G was started every 21 days. Bone marrow transplantation was delayed due to the complications that merited 13 hospitalizations and severe thrombocytopenia, low gastrointestinal bleeding, septic arthritis, infectious gastroenteritis, chronic suppurative otitis media and severe folliculitis. At the age of 4 years BMT of cord was performed, and 26 days after transplantation he presented septic shock and died. The prognosis of bone marrow transplantation in Wiskott-Aldrich syndrome and in other primary immunodeficiencies depends on the promptness of its performance at early stages in life. It is important that the first contact physicians be aware of the primary immunodeficiency signs and symptoms. Primary immunodeficiency diseases are not common in children. The possibility of an immunological defect should be considered in any individual with repeated infections. A definite diagnosis for immodeficiency is sometimes difficult to achieve because of overlapping clinical manifestations. Immunoglobulin subclass deficiency is an immunological deficiency disease with which, one or more IgG subclasses are deficient. T cell immunity is normal. Patients may develop recurrent bacterial and respiratory infections or could remain asymptomatic. The authors report a case of immunoglobulin G subclass deficiency presenting initially as transient hypogammaglobulinemia of infancy. A 2 month-old boy presented to Siriraj Hospital with a history of chronic protracted diarrhea, disseminated scabies and sepsis. On presentation, he had generalized scaly and maculopapular rash with no palpable lymph nodes. CBC revealed WBC 22,100 cells/cm3 with PMN 42 per cent, lymphocytes 38 per cent, Eosinophils 4 per cent, Basophil 2 per cent and platelets 254,000/cm3. The immunoglobulin levels were as follows: IgG 181 mg/dl, IgA < 6.6 mg/dl, IgM 26.3 mg/dl. Lymphocyte enumerations revealed CD4 of 2,433 cells/cm3 (N 1,460-5,160); CD8 4,682 cells/cm3 (N 650-2,450); CD19 1,588 cell/cm3 (N 500-1,500); CD16 230 cell/cm3 (N 573 +/- 264). The initial diagnosis was X-linked agammaglobulinemia vs common variable immunodeficiency disease. His diarrhea and five courses of sepsis responded well to antibiotics administration and courses of intravenous immunoglobulin (IVIG) replacement. His through IgG became normal at 2 years of age (after 12 months of IVIG). IVIG was stopped and the diagnosis was changed to transient hypogammaglobulinemia of infancy (THI). Nevertheless, during his 4 month follow-up he developed recurrent sinopulmonary infections (i.e, otitis media and pneumonia). Repeated immunoglobulin profile showed IgG 1,200 mg/dl, IgA 135 mg/dl, IgM 26 mg/dl, IgG subclass were IgG, 1,030 mg/dl (N 280-830), IgG2 30 mg/dl (N 40-2,400), IgG3 22 mg/dl (N 6-130), IgG4 3 mg/dl (N 3-120). A diagnosis of IgG2 subclass deficiency presenting early as transient hypogammaglobulinemia of infancy was then made. Treatment with monthly IVIG was reinitiated and the patient is currently doing well. The authors present a case of IgG subclass deficiency presenting as transient hypogammaglbulinemia of infancy. Follow-up of the immune profile and clinical manifestation is necessary for a definite diagnosis. X-linked hyper-IgM syndrome (XHIM) is a rare primary immunodeficiency disorder caused by mutations of the gene encoding the CD40 ligand (CD40L). It is characterized by recurrent infections with markedly decreased serum IgG, IgA and IgE levels but normal or elevated IgM levels. We report the clinical manifestations and complete immune studies in the first family with molecularly proven XHIM in Taiwan. A 5-month-old boy presented with rapidly progressive pneumonia which responded poorly to antibiotics. High levels of IgM and very low levels of IgG, IgA, and IgE were noted in his plasma specimen: IgM, 128 mg/dl; IgG, 18 mg/dl; IgA, 4 mg/dl); IgE, 1 IU/ml. Whole blood flow cytometry when he was 21 months old showed that only a small percentage (0.48%) of his in vitro-activated CD4+ T cells expressed CD40L. When he was 3 years old, repeated flow cytometry showed essentially the same result (0.4%), compared with his father's CD40L expression of over 85%. The patient's mother had moderately decreased CD40L expression (74.4%). Hyper-IgM syndrome was confirmed by CD40L mutation analysis in the boy, which revealed a Lys 96 stop (nucleotide A307T) in exon 2 of CD40L, with a truncated protein resulting in the loss of the entire TNF domain. His mother was a carrier and apparently the individual in whom the mutation originated. Eleven other family members, including the patient's father, sister, and grandmother, and the mother's sisters and their children, all had normal results on CD40L mutation analysis. The patient has remained without significant bacterial infection on a regimen of monthly IVIG infusion and oral trimethoprim-sulfamethoxazole for Pneumocystis carinii pneumonia (PCP) prophylaxis, although he has had recurrent oral ulcers and neutropenia. Bone marrow transplantation is planned. Wiskott-Aldrich syndrome (WAS) is an X-linked syndrome consisting of eczema, recurrent pyogenic infection, and thrombocytopenia with decreased platelet volume. Immunologic studies reveal normal immunoglobulin G (IgG), decreased IgM, elevated IgA and IgE levels, and decreased T-cell function. Patients with WAS often have increased susceptibility to lymphoproliferative disorders (LPDs). We report a 3-year-old boy who had persistent thrombocytopenia with bleeding, recurrent infections, and chronic eczema with frequent skin infections since birth. A blood smear revealed small platelets (50% of normal size). Immunologic studies showed normal IgG (1880 mg/dL), decreased IgM (76 mg/dL) and increased IgA (228 mg/dL) and IgE (14,282 IU/mL) levels. The relative proportions of immune cells were CD2 52.2%, CD3 41.1%, CD4 23.4%, CD8 16.8%, CD19 8.0%, CD57 7.7% and active T cells 14.6%. T-cell dysfunction was detected on the multitest for cell-mediated immunity. The WAS diagnosis was confirmed by mutation analysis which demonstrated a 4-base pair deletion in WAS protein gene exon 1. His thrombocytopenia was uncontrolled despite intravenous immunoglobulin infusions, so splenectomy was performed. The platelet count then rose to about 60,000 to 80,000/microL. However, about 2 weeks after splenectomy, he developed generalized lymphadenopathy and lymphoma was misdiagnosed based on lymph node biopsy at another hospital where he was admitted for urgent care. However, our analysis of his lymph node pathology led to the diagnosis of atypical LPD (ALPD). The lymphadenopathy regressed spontaneously 1 month later without chemotherapy. Early and correct diagnosis of WAS complicated with ALPD is important to avoid unnecessary chemotherapy. Bruton's agammaglobulinemia is a primary immunodeficiency with a disease onset during the first months of age, when the maternal serum immunoglobulin levels decrease. It is characterized by recurrent infections and agammaglobulinemia. We report the case of a 6-year-old male patient with third-degree consanguinity, product of a third pregnancy and complete immunization scheme. He had a history of oral candidiasis at the age of 3 months, chicken pox at the age of 7 months, and two episodes of complicated bronchopneumonia at the age of 1 year and 6 years. He was admitted to the hospital because of fever and cough. Examination of the chest showed rales and right basilar hypoventilation, and a blood cell count revealed leukocytosis and neutrophilia. The diagnosis of pneumonia was made. He was treated with IV antibiotics. Serum immunoglobulins were reported to be low (IgM 55 mg/dL, IgA 0.9 mg/dL, and IgG 199 mg/dL). With these findings the clinical diagnosis of X-linked agammaglobulinemia (ALX) was concluded. A molecular test was performed fining a BTK gene confirming the diagnosis of Bruton's disease. Therapy with intravenous IgG was started every 21 days. During his evolution, he presented three episodes of rhinosinusitis, one of suppurative otitis media, and four events of pneumonia that required 37 days of hospitalization. After hospital discharge, the patient was free of infections and he returned to his daily activities. In cases of recurrent and severe respiratory infections in children, we must consider primary immunodeficiency disease in the differential diagnosis, mainly antibiotic deficiency. Early diagnosis and treatment improves the survival and quality of life in these patients. Two hundred and eight children with recurrent pneumonia were studied over a 5-year period. Among these patients we found 10 cases with primary immunodeficiency disease: 6 cases of IgA deficiency, 1 case of X-linked agammaglobulinemia, 1 case of common variable immunodeficiency, 1 case of hyper IgM syndrome, and 1 case of Wiskott-Aldrich syndrome. This study describes the clinical features of these cases and assesses the usefulness of our immunodeficiency screening protocol. In this group 6 were males; the mean age at first episode of pneumonia was 3 years (range 3 months to 18 years), and the age of diagnosis ranged between 10 months and 19 years. The average number of episodes of pneumonia in each patient was 5 (range 2 to 12), and the number of hospitalizations ranged up to 13. The etiologic agents isolated from this recurrent pneumonia were S. pneumoniae, Moraxella, adenovirus, respiratory syncytial virus, and influenza B virus. Intravenous immunoglobulin was used in four cases. Two patients had chronic pulmonary damage with bronchiectasis and interstitial pneumonia. Only one patient died (Wiskott-Aldrich syndrome) during the follow-up from an intracranial hemorrhage. We found that the screening protocol applied to patients with recurrent pneumonia is a useful tool for ruling out the primary immunodeficiency disorders. Concentrations of IgD and IgE were measured in sera from 165 patients with well-defined immunodeficiency in an effort to find information possibly relevant to the roles of antibodies of these classes in host defense. Values for both immunoglobulins were generally quite low in patients who had marked deficiencies of all three major immunoglobulins, although occasional normal or high normal values for IgD were seen in hypogammaglobulinemic patients. Group mean IgD concentrations were also depressed in patients with Wiskott-Aldrich syndrome and in those with selective IgA deficiency; IgE concentrations were depressed in patients with X-linked immunodeficiency with hyper-IgM and in those with ataxia telangiectasia. IgD and IgE were both significantly elevated in patients with extreme hyperimmunoglobulinemia E and undue susceptibility to infection and in a patient with the Nezelof syndrome; none of these patients had histories suggestive of atopy. In addition, the mean IgE concentration was significantly elevated in patients with selective IgA deficiency, many of whom were atopic, and in those with the Wiskott-Aldrich syndrome. The highest IgD concentration (163 mg/100 ml) was found in serum from a boy with variable immunodeficiency who had a lifelong history of severe recurrent pharyngeal infections, primarily streptococcal in etiology. Recurrent staphylococcal infection was a feature common to many but not all patients with elevated serum IgE concentration. These data may prove useful in the future delineation of biologic roles for antibodies in these two immunoglobulin classes. Hyper IgM with low IgG and IgA is a rare humoral immunodeficiency. We presently report 12 new observations which have been clinically and immunologically studied. On one occasion the syndrome was found to be associated with congenital rubella. Since 10/12 children were male, X-linked inheritance is suggested which has been confirmed in 2 cases. In most cases (9/12), the first infections occurred within the first year of life. The syndrome is causing upper and lower respiratory tract infections due to bacteria, as well as gut infections. Lymphoid organ hyperplasia has been noted in 11/12 patients. Polyclonal hyper IgM serum contrasts with low or absent IgG, IgA and IgE. In some instances, some IgM antibody response was detected. A dysfunction of cellular immunity was not detected. Autoimmunity was detected in 3 patients. Finally, transient neutropenia occurred in 50% of the patients. Intravenous immunoglobulin G substitution treatment resulted in a significant reduction in the occurrence of infections as well as in normalization of growth rate. Immunoglobulin infusion also frequently induced correction of hyper IgM and neutropenia. The hyper-IgM syndrome (HIM) is a rare primary immunodeficiency disorder caused by defects in the CD40 ligand (CD40L)/CD40-signaling pathway. It is characterized by recurrent infections with markedly decreased IgG, IgA and IgE levels but normal or elevated serum IgM levels. A 5-month-old boy presented with rapidly progressive pneumonia which responded poorly to antibiotics. High levels of IgM and very low levels of IgG, IgE and IgA were noted in his plasma specimen (IgM, 128 mg/dl; IgG, 18 mg/dl; IgE, 1 IU/ml; IgA, 4 mg/dl). The relative proportions of immune cells were CD3 24.6%, CD4 10.3%, CD8 2.2%, CD19 30.2%, CD57 1.0% and active T cells 1.1%. After IVIG treatment, the pneumonia improved. Repeat assessment at the age of 15 months showed IgM decreased to the normal range (32 mg/dl). Whole blood flow cytometry assay for CD40L expression confirmed the diagnosis of hyper-lgM syndrome when he was 21 months old. Only a small percentage (0.48%) of the patient's in vitro activated CD4+ T cells expressed CD40L, compared with 33.54% from a healthy control. The patient's father, mother and sister all had a normal CD40L expression activation patterns (43.52%, 40.78%, 34.11%, respectively). On a regimen of monthly IVIG infusion and oral trimethoprim-sulfamethoxazole for Pneumocystis carinii pneumonia (PCP) prophylaxis, the patient has had no recurrent infections. X-linked agammaglobulinemia (XLA), caused by mutations in Bruton's tyrosine kinase (BTK), is a common form of inherited antibody deficiency. There were very few case reports of this disease that were diagnosed only based on clinical findings in China. The purpose of this study was to evaluate the clinical features of 8 Chinese cases with XLA with BTK defect which were confirmed by flow cytometry and/or gene analysis. Based on clinical findings, 8 suspected XLA patients were confirmed by detecting the expression of BTK by flow cytometry and/or gene analysis of BTK. The history and thorough physical examination and routine immunological evaluation of 8 cases were collected and reviewed. The age of onset of all the 8 male patients were from 3 months to 3 years. The mean age at diagnosis was 6 years. Recurrent upper respiratory infection and pneumonia with fever were seen in all the patients. Nasopharynx infection was mainly contributed to upper respiratory infection. Very few or no otitis (1/8) and sinusitis (0/8) were involved. Polyarthritis without evidence of infection was common (3/8). Chronic diarrhea was documented during the first 2 years after the onset of the disease in 2 cases. Two of the patients suffered from meningitis one time each. Skin infection was not serious in two patients. Osteomyelitis occurred in one case, which occurred secondary to a trauma. One case had poliomyelitis-like disease that was considered to be related to polio vaccine. Only two cases had unconfirmed maternal family history of XLA. The prominent signs at diagnosis were dystrophia, growth and developmental retardation and markedly decreased or absent tonsils and lymph nodes. Concentration of all classes of serum immunoglobulins (Igs) and the number of B cells in the peripheral circulation were dramatically decreased. The ratio of CD4/CD8 in most of the patients (6/8) was markedly inverse. The age at diagnosis of this reported group was older. Clinical symptoms displayed recurrent upper respiratory infection (nasopharynx infection but rare or no otitis or sinusitis) and pneumonia; polyarthritis was common. There were no confirmed family history of XLA. Most of the patients showed inverse ratios of CD4/CD8, the reason and potential significance are unclear. Signs and symptoms WAS occurs most often in males due to its X-linked recessive pattern of inheritance, affecting between 1 to 10 males per million. The first signs are usually petechiae and bruising, resulting from a low platelet count (i.e. thrombocytopenia). Spontaneous nose bleeds and bloody diarrhea are also common and eczema typically develops within the first month of life. Recurrent bacterial infections typically develop by three months of age. The majority of children with WAS develop at least one autoimmune disorder, and cancers (mainly lymphoma and leukemia) develop in up to a third of patients. Immunoglobulin M (IgM) levels are reduced, IgA and IgE are elevated, and IgG levels can be normal, reduced, or elevated. In addition to thrombocytopenia, WAS patients have abnormally small platelets (i.e. microthrombocytes) and ~30% also have elevated eosinophil counts (i.e. eosinophilia). Diagnosis The diagnosis can be made on the basis of clinical findings, the peripheral blood smear, and low immunoglobulin levels. Typically, IgM levels are low, IgA levels are elevated, and IgE levels may be elevated; paraproteins are occasionally observed. Skin immunologic testing (allergy testing) may reveal hyposensitivity. Individuals with Wiskott–Aldrich syndrome however are at higher risk for severe food allergies. Not all patients have a positive family history of the disorder; new mutations do occur. Often, leukemia may be suspected on the basis of low platelets and infections, and bone marrow biopsy may be performed. Decreased levels of WASp are typically observed. The current gold standard for diagnosis is DNA sequence analysis, which can detect WAS and the related disorders XLT and XLN in 95% of patients and carriers. Normal numbers of B cells with decreased IgG and IgA and increased IgM: Hyper-IgM syndromes Normal numbers of B cells with isotype or light chain deficiencies: heavy chain deletions, kappa chain deficiency, isolated IgG subclass deficiency, IgA with IgG subclass deficiency, selective immunoglobulin A deficiency Specific antibody deficiency to specific antigens with normal B cell and normal Ig concentrations Transient hypogammaglobulinemia of infancy (THI) The Wiskott-Aldrich Syndrome (WAS) is a rare X-linked immunohematological disorder characterized by eczema, profound thrombocytopenia, and progressive immunodeficiency. Severe hemorrhage, overwhelming sepsis, or lymphoreticular malignancy usually cause death in childhood. Recently, bone marrow transplantation (BMT) has been curative in some well-established cases, but there is no general agreement about the place of BMT in infants with WAS before the development of significant immunological abnormalities. We describe the successful use of early histocompatible BMT in a 10-month-old infant in whom WAS was diagnosed on the basis of eczema, thrombocytopenia, small platelets, and raised serum immunoglobulin A (Ig) and IgE, but before the development of immunodeficiency as evidenced clinically by recurrent infections, or immunologically by low serum IgM or consistently abnormal lymphocyte responses to mitogens. After an unstable period for several weeks posttransplantation when he developed marked hepatomegaly and severe interstitial pneumonitis, he made a good recovery. His eczema and thrombocytopenia resolved and he has shown no clinical or laboratory evidence of immunodeficiency. It is now over 2 years since his BMT. Because of the poor prognosis of WAS, where a histocompatible donor is available, BMT at the earliest opportunity, despite the inherent risks of such a procedure, may be the best option for an infant with WAS. To investigate the clinical and immunological laboratory features, gene mutations, treatment and prognosis in children with Wiskott-Aldrich syndrome (WAS). The clinical, laboratory characteristics, treatment and prognosis of 132 children with WAS, who visited Children's Hospital of Chongqing Medical University from April 2000 to June 2015, were analyzed retrospectively. All patients were male. The median age of disease onset was 15 days and the median age at diagnosis was 10 months. Of the 132 cases, 112 had classic WAS, 20 had X-linked thrombocytopenia (XLT). The median platelet count was 23×10(9)/L. All cases had the clinical characteristics of WAS including bleeding, eczema, and being susceptible to infection. The initial symptoms include hemorrhage (75.0%) and eczema (16.7%). Twenty-one cases had autoimmune diseases and one patient had leukemia. WAS protein (WASP) expression in 115 cases were measured by flow cytometry, 88 cases were negative, in 12 cases WASP decreased, in 5 cases it was normal, 10 cases had bimodal distribution. Eighty-one kinds of mutations were found in 122 families, including eight kinds of hot-spot mutations, which were 290 C> N / 291G> N (R86C / H / L), 665 C> T (R211X), 155 C> T (R41X), 168 C> T (T45 M), IVS1+ 1 g> t/ a, IVS6 + 5 g> a, IVS8 + 1 g> a and IVS8 + 1to + 6del gtga. Meantime, 29 kinds of novel mutations were found, which were 321T>C, 415C>A, 471C>T, 102-105delC, 521 del C, 1330 del A, IVS2-2 a>c, 168 C>A/1412 C> T, exon1-2 del/1412 C>T, and so on. The proportion of CD3(+) T cells (31.3%), helper T cells (37.3%) and cytotoxic T cells (38.6%) in the peripheral blood declined. The serum levels of IgG (51.1%), IgA (43.3%) and IgE (40.0%) increased, IgM (25.6%) decreased. Of the 132 cases, 72 remain survived, of whom 36 cases received hematopoietic stem cell transplantation (HSCT), 14 patients with classic WAS received intravenous immunoglobulin (IVIG) therapy. With regular IVIG therapy, the frequency of infections was reduced and the patients' symptoms were improved. The clinical characteristics of Wiskott-Aldrich syndrome were early age of onset, microthrombocytopenia, eczema and recurrent infections. The proportion of T lymphocyte declined, the serum levels of IgG, IgA, and IgE increased, and level of IgM decreased in a part of patients. The detection of WAS gene mutation and WAS protein detection was the key diagnostic methods. Regular IVIG can gain more time for children who will receive HSCT and improve their quality of life. The most important differential diagnosis includes the following: X-linked agammaglobulinemia characterizes by recurrent bacterial infections in boys, and genetic studies may reveal the presence of Bruton tyrosine kinase (BTK) mutations. Transient hypogammaglobulinemia of newborns presents in newborns above the age of 4 months and characterizes by recurrent pneumonia, meningitis, otitis media, and other problems that resemble Bruton disease. It is a physiological defect in the immune system caused by maternal IgG disappearance and corrected soon but requires treatment. In super-IgM syndrome, recurrent bacterial infections occur, but the cause of this illness is a mutation in the gene encoding for CD40 on T lymphocytes that causes a failure in T and B lymphocyte cooperation. Common variable immunodeficiency presents with recurrent bacterial infections, including sinopulmonary problems but later in life (second-fourth decade), and the diagnosis is made once all causes of immunodeficiency have been ruled out. [6] [7] [21] [33] of primary immunodeficiency (PID). These different forms can affect different parts of the immune system, including immunoglobulin production. Primary immunodeficiencies usually have a delay of several years between initial clinical presentation and diagnosis. Some primary immune deficiencies include ataxia-telangiectasia (A-T), autosomal recessive agammaglobulinemia (ARA), common variable immunodeficiency (CVID), hyper-IgM syndromes, IgG subclass deficiency, isolated non-IgG immunoglobulin deficiencies, severe combined immunodeficiency (SCID), specific antibody deficiency (SAD), Wiskott-Aldrich syndrome, or X-linked agammaglobulinemia. CVID is the most common form of primary immunodeficiency. SCID is considered a medical emergency and suspected cases require immediate specialist center referral for diagnosis and treatment. It is more often that hypogammaglobulinemia develops as a result of another condition, which are called secondary or acquired immune deficiencies. These Signs and symptoms Affects males 50% of the time if mother is a carrier for the gene. Children are generally asymptomatic until 6–9 months of age when maternal IgG decreases. Present with recurrent infections with Streptococcus pneumoniae, Haemophilus influenzae, Mycoplasma pneumoniae, hepatitis virus, and enterovirus CNS infections. Examination shows lymphoid hypoplasia (tonsils and adenoids, no splenomegaly or lymphadenopathy). There is significant decrease in all immunoglobulins. Genetics Most antibodies are gamma globulins. Antibodies are made mainly by plasma cells, which are daughter cells of the B cell line. The Btk enzyme plays an essential role in the maturation of B cells in the bone marrow, and when mutated, immature pro-B lymphocytes are unable to develop into pre-B lymphocytes, which normally develop into mature (naive) B cells that leave the bone marrow into the blood stream.\nHere is the question:\nGENETICS AND IMMUNOLOGY: 2-year-old boy. His personal history includes 3 episodes of acute otitis media, 1 meningococcal meningitis and 2 pneumonias (one middle lobe and one left upper lobe). She has been admitted on 3 occasions for thrombopenic purpura (on three occasions antiplatelet antibodies were negative and bone marrow showed normal megakaryocytes). Several males of the maternal family had died in childhood due to infectious processes. Physical examination showed lesions typical of atopic dermatitis. The immunological study showed a slight decrease in T-lymphocyte subpopulations; elevated IgA and IgE; decreased IgM and IgG at the lower limit of normal. What is the most likely diagnosis?\nHere are the potential choices:\n1. Wiskott-Aldrich syndrome.\n2. Hyper IgE syndrome.\n3. Transient hypogammaglobulinemia of childhood.\n4. X-linked severe combined immunodeficiency.\n5. Common variable immunodeficiency.\nThe correct answer is: ", "gold_answer": "4 X-linked severe combined immunodeficiency.", "prediction": "You are a helpful medical expert, and your task is to answer a multi-choice medical question using the relevant documents. Please choose the answer from the provided options. Your responses will be used for research purposes only, so please have a definite answer.\nHere are the relevant documents:\n The Wiskott-Aldrich syndrome is a primary immunodeficiency characterized by congenital microthrombocytopenia, eczema and recurrent infections. This paper reports the case of a 3-year-6-month male patient, whose maternal uncle died at the age of 3 months due to fulminant sepsis from a pulmonary infection. The patient was a product of the first pregnancy, he was born at 27 weeks' gestation and weighed 1,400 g. As a neonate he was hospitalized during the first 2 months of life because of a low gastrointestinal bleeding, thrombocytopenia and severe infections. In the next 4 months and before coming to our hospital the infant was hospitalized 54 times. On admission he presented disseminated dermatosis, enlarged neck lymph nodes and psychomotor retardation. Laboratory studies revealed hemoglobin 8.1 g/dL, platelets 31,000/uL, mean platelet volume 5.6 fL, IgM 39.3 mg/dL, IgA 67 mg/dL, IgG 1,380 mg/dL. On several occasions he received globular packages and platelet concentrates. The infusion of immunoglobulin G was started every 21 days. Bone marrow transplantation was delayed due to the complications that merited 13 hospitalizations and severe thrombocytopenia, low gastrointestinal bleeding, septic arthritis, infectious gastroenteritis, chronic suppurative otitis media and severe folliculitis. At the age of 4 years BMT of cord was performed, and 26 days after transplantation he presented septic shock and died. The prognosis of bone marrow transplantation in Wiskott-Aldrich syndrome and in other primary immunodeficiencies depends on the promptness of its performance at early stages in life. It is important that the first contact physicians be aware of the primary immunodeficiency signs and symptoms. Primary immunodeficiency diseases are not common in children. The possibility of an immunological defect should be considered in any individual with repeated infections. A definite diagnosis for immodeficiency is sometimes difficult to achieve because of overlapping clinical manifestations. Immunoglobulin subclass deficiency is an immunological deficiency disease with which, one or more IgG subclasses are deficient. T cell immunity is normal. Patients may develop recurrent bacterial and respiratory infections or could remain asymptomatic. The authors report a case of immunoglobulin G subclass deficiency presenting initially as transient hypogammaglobulinemia of infancy. A 2 month-old boy presented to Siriraj Hospital with a history of chronic protracted diarrhea, disseminated scabies and sepsis. On presentation, he had generalized scaly and maculopapular rash with no palpable lymph nodes. CBC revealed WBC 22,100 cells/cm3 with PMN 42 per cent, lymphocytes 38 per cent, Eosinophils 4 per cent, Basophil 2 per cent and platelets 254,000/cm3. The immunoglobulin levels were as follows: IgG 181 mg/dl, IgA < 6.6 mg/dl, IgM 26.3 mg/dl. Lymphocyte enumerations revealed CD4 of 2,433 cells/cm3 (N 1,460-5,160); CD8 4,682 cells/cm3 (N 650-2,450); CD19 1,588 cell/cm3 (N 500-1,500); CD16 230 cell/cm3 (N 573 +/- 264). The initial diagnosis was X-linked agammaglobulinemia vs common variable immunodeficiency disease. His diarrhea and five courses of sepsis responded well to antibiotics administration and courses of intravenous immunoglobulin (IVIG) replacement. His through IgG became normal at 2 years of age (after 12 months of IVIG). IVIG was stopped and the diagnosis was changed to transient hypogammaglobulinemia of infancy (THI). Nevertheless, during his 4 month follow-up he developed recurrent sinopulmonary infections (i.e, otitis media and pneumonia). Repeated immunoglobulin profile showed IgG 1,200 mg/dl, IgA 135 mg/dl, IgM 26 mg/dl, IgG subclass were IgG, 1,030 mg/dl (N 280-830), IgG2 30 mg/dl (N 40-2,400), IgG3 22 mg/dl (N 6-130), IgG4 3 mg/dl (N 3-120). A diagnosis of IgG2 subclass deficiency presenting early as transient hypogammaglobulinemia of infancy was then made. Treatment with monthly IVIG was reinitiated and the patient is currently doing well. The authors present a case of IgG subclass deficiency presenting as transient hypogammaglbulinemia of infancy. Follow-up of the immune profile and clinical manifestation is necessary for a definite diagnosis. X-linked hyper-IgM syndrome (XHIM) is a rare primary immunodeficiency disorder caused by mutations of the gene encoding the CD40 ligand (CD40L). It is characterized by recurrent infections with markedly decreased serum IgG, IgA and IgE levels but normal or elevated IgM levels. We report the clinical manifestations and complete immune studies in the first family with molecularly proven XHIM in Taiwan. A 5-month-old boy presented with rapidly progressive pneumonia which responded poorly to antibiotics. High levels of IgM and very low levels of IgG, IgA, and IgE were noted in his plasma specimen: IgM, 128 mg/dl; IgG, 18 mg/dl; IgA, 4 mg/dl); IgE, 1 IU/ml. Whole blood flow cytometry when he was 21 months old showed that only a small percentage (0.48%) of his in vitro-activated CD4+ T cells expressed CD40L. When he was 3 years old, repeated flow cytometry showed essentially the same result (0.4%), compared with his father's CD40L expression of over 85%. The patient's mother had moderately decreased CD40L expression (74.4%). Hyper-IgM syndrome was confirmed by CD40L mutation analysis in the boy, which revealed a Lys 96 stop (nucleotide A307T) in exon 2 of CD40L, with a truncated protein resulting in the loss of the entire TNF domain. His mother was a carrier and apparently the individual in whom the mutation originated. Eleven other family members, including the patient's father, sister, and grandmother, and the mother's sisters and their children, all had normal results on CD40L mutation analysis. The patient has remained without significant bacterial infection on a regimen of monthly IVIG infusion and oral trimethoprim-sulfamethoxazole for Pneumocystis carinii pneumonia (PCP) prophylaxis, although he has had recurrent oral ulcers and neutropenia. Bone marrow transplantation is planned. Wiskott-Aldrich syndrome (WAS) is an X-linked syndrome consisting of eczema, recurrent pyogenic infection, and thrombocytopenia with decreased platelet volume. Immunologic studies reveal normal immunoglobulin G (IgG), decreased IgM, elevated IgA and IgE levels, and decreased T-cell function. Patients with WAS often have increased susceptibility to lymphoproliferative disorders (LPDs). We report a 3-year-old boy who had persistent thrombocytopenia with bleeding, recurrent infections, and chronic eczema with frequent skin infections since birth. A blood smear revealed small platelets (50% of normal size). Immunologic studies showed normal IgG (1880 mg/dL), decreased IgM (76 mg/dL) and increased IgA (228 mg/dL) and IgE (14,282 IU/mL) levels. The relative proportions of immune cells were CD2 52.2%, CD3 41.1%, CD4 23.4%, CD8 16.8%, CD19 8.0%, CD57 7.7% and active T cells 14.6%. T-cell dysfunction was detected on the multitest for cell-mediated immunity. The WAS diagnosis was confirmed by mutation analysis which demonstrated a 4-base pair deletion in WAS protein gene exon 1. His thrombocytopenia was uncontrolled despite intravenous immunoglobulin infusions, so splenectomy was performed. The platelet count then rose to about 60,000 to 80,000/microL. However, about 2 weeks after splenectomy, he developed generalized lymphadenopathy and lymphoma was misdiagnosed based on lymph node biopsy at another hospital where he was admitted for urgent care. However, our analysis of his lymph node pathology led to the diagnosis of atypical LPD (ALPD). The lymphadenopathy regressed spontaneously 1 month later without chemotherapy. Early and correct diagnosis of WAS complicated with ALPD is important to avoid unnecessary chemotherapy. Bruton's agammaglobulinemia is a primary immunodeficiency with a disease onset during the first months of age, when the maternal serum immunoglobulin levels decrease. It is characterized by recurrent infections and agammaglobulinemia. We report the case of a 6-year-old male patient with third-degree consanguinity, product of a third pregnancy and complete immunization scheme. He had a history of oral candidiasis at the age of 3 months, chicken pox at the age of 7 months, and two episodes of complicated bronchopneumonia at the age of 1 year and 6 years. He was admitted to the hospital because of fever and cough. Examination of the chest showed rales and right basilar hypoventilation, and a blood cell count revealed leukocytosis and neutrophilia. The diagnosis of pneumonia was made. He was treated with IV antibiotics. Serum immunoglobulins were reported to be low (IgM 55 mg/dL, IgA 0.9 mg/dL, and IgG 199 mg/dL). With these findings the clinical diagnosis of X-linked agammaglobulinemia (ALX) was concluded. A molecular test was performed fining a BTK gene confirming the diagnosis of Bruton's disease. Therapy with intravenous IgG was started every 21 days. During his evolution, he presented three episodes of rhinosinusitis, one of suppurative otitis media, and four events of pneumonia that required 37 days of hospitalization. After hospital discharge, the patient was free of infections and he returned to his daily activities. In cases of recurrent and severe respiratory infections in children, we must consider primary immunodeficiency disease in the differential diagnosis, mainly antibiotic deficiency. Early diagnosis and treatment improves the survival and quality of life in these patients. Two hundred and eight children with recurrent pneumonia were studied over a 5-year period. Among these patients we found 10 cases with primary immunodeficiency disease: 6 cases of IgA deficiency, 1 case of X-linked agammaglobulinemia, 1 case of common variable immunodeficiency, 1 case of hyper IgM syndrome, and 1 case of Wiskott-Aldrich syndrome. This study describes the clinical features of these cases and assesses the usefulness of our immunodeficiency screening protocol. In this group 6 were males; the mean age at first episode of pneumonia was 3 years (range 3 months to 18 years), and the age of diagnosis ranged between 10 months and 19 years. The average number of episodes of pneumonia in each patient was 5 (range 2 to 12), and the number of hospitalizations ranged up to 13. The etiologic agents isolated from this recurrent pneumonia were S. pneumoniae, Moraxella, adenovirus, respiratory syncytial virus, and influenza B virus. Intravenous immunoglobulin was used in four cases. Two patients had chronic pulmonary damage with bronchiectasis and interstitial pneumonia. Only one patient died (Wiskott-Aldrich syndrome) during the follow-up from an intracranial hemorrhage. We found that the screening protocol applied to patients with recurrent pneumonia is a useful tool for ruling out the primary immunodeficiency disorders. Concentrations of IgD and IgE were measured in sera from 165 patients with well-defined immunodeficiency in an effort to find information possibly relevant to the roles of antibodies of these classes in host defense. Values for both immunoglobulins were generally quite low in patients who had marked deficiencies of all three major immunoglobulins, although occasional normal or high normal values for IgD were seen in hypogammaglobulinemic patients. Group mean IgD concentrations were also depressed in patients with Wiskott-Aldrich syndrome and in those with selective IgA deficiency; IgE concentrations were depressed in patients with X-linked immunodeficiency with hyper-IgM and in those with ataxia telangiectasia. IgD and IgE were both significantly elevated in patients with extreme hyperimmunoglobulinemia E and undue susceptibility to infection and in a patient with the Nezelof syndrome; none of these patients had histories suggestive of atopy. In addition, the mean IgE concentration was significantly elevated in patients with selective IgA deficiency, many of whom were atopic, and in those with the Wiskott-Aldrich syndrome. The highest IgD concentration (163 mg/100 ml) was found in serum from a boy with variable immunodeficiency who had a lifelong history of severe recurrent pharyngeal infections, primarily streptococcal in etiology. Recurrent staphylococcal infection was a feature common to many but not all patients with elevated serum IgE concentration. These data may prove useful in the future delineation of biologic roles for antibodies in these two immunoglobulin classes. Hyper IgM with low IgG and IgA is a rare humoral immunodeficiency. We presently report 12 new observations which have been clinically and immunologically studied. On one occasion the syndrome was found to be associated with congenital rubella. Since 10/12 children were male, X-linked inheritance is suggested which has been confirmed in 2 cases. In most cases (9/12), the first infections occurred within the first year of life. The syndrome is causing upper and lower respiratory tract infections due to bacteria, as well as gut infections. Lymphoid organ hyperplasia has been noted in 11/12 patients. Polyclonal hyper IgM serum contrasts with low or absent IgG, IgA and IgE. In some instances, some IgM antibody response was detected. A dysfunction of cellular immunity was not detected. Autoimmunity was detected in 3 patients. Finally, transient neutropenia occurred in 50% of the patients. Intravenous immunoglobulin G substitution treatment resulted in a significant reduction in the occurrence of infections as well as in normalization of growth rate. Immunoglobulin infusion also frequently induced correction of hyper IgM and neutropenia. The hyper-IgM syndrome (HIM) is a rare primary immunodeficiency disorder caused by defects in the CD40 ligand (CD40L)/CD40-signaling pathway. It is characterized by recurrent infections with markedly decreased IgG, IgA and IgE levels but normal or elevated serum IgM levels. A 5-month-old boy presented with rapidly progressive pneumonia which responded poorly to antibiotics. High levels of IgM and very low levels of IgG, IgE and IgA were noted in his plasma specimen (IgM, 128 mg/dl; IgG, 18 mg/dl; IgE, 1 IU/ml; IgA, 4 mg/dl). The relative proportions of immune cells were CD3 24.6%, CD4 10.3%, CD8 2.2%, CD19 30.2%, CD57 1.0% and active T cells 1.1%. After IVIG treatment, the pneumonia improved. Repeat assessment at the age of 15 months showed IgM decreased to the normal range (32 mg/dl). Whole blood flow cytometry assay for CD40L expression confirmed the diagnosis of hyper-lgM syndrome when he was 21 months old. Only a small percentage (0.48%) of the patient's in vitro activated CD4+ T cells expressed CD40L, compared with 33.54% from a healthy control. The patient's father, mother and sister all had a normal CD40L expression activation patterns (43.52%, 40.78%, 34.11%, respectively). On a regimen of monthly IVIG infusion and oral trimethoprim-sulfamethoxazole for Pneumocystis carinii pneumonia (PCP) prophylaxis, the patient has had no recurrent infections. X-linked agammaglobulinemia (XLA), caused by mutations in Bruton's tyrosine kinase (BTK), is a common form of inherited antibody deficiency. There were very few case reports of this disease that were diagnosed only based on clinical findings in China. The purpose of this study was to evaluate the clinical features of 8 Chinese cases with XLA with BTK defect which were confirmed by flow cytometry and/or gene analysis. Based on clinical findings, 8 suspected XLA patients were confirmed by detecting the expression of BTK by flow cytometry and/or gene analysis of BTK. The history and thorough physical examination and routine immunological evaluation of 8 cases were collected and reviewed. The age of onset of all the 8 male patients were from 3 months to 3 years. The mean age at diagnosis was 6 years. Recurrent upper respiratory infection and pneumonia with fever were seen in all the patients. Nasopharynx infection was mainly contributed to upper respiratory infection. Very few or no otitis (1/8) and sinusitis (0/8) were involved. Polyarthritis without evidence of infection was common (3/8). Chronic diarrhea was documented during the first 2 years after the onset of the disease in 2 cases. Two of the patients suffered from meningitis one time each. Skin infection was not serious in two patients. Osteomyelitis occurred in one case, which occurred secondary to a trauma. One case had poliomyelitis-like disease that was considered to be related to polio vaccine. Only two cases had unconfirmed maternal family history of XLA. The prominent signs at diagnosis were dystrophia, growth and developmental retardation and markedly decreased or absent tonsils and lymph nodes. Concentration of all classes of serum immunoglobulins (Igs) and the number of B cells in the peripheral circulation were dramatically decreased. The ratio of CD4/CD8 in most of the patients (6/8) was markedly inverse. The age at diagnosis of this reported group was older. Clinical symptoms displayed recurrent upper respiratory infection (nasopharynx infection but rare or no otitis or sinusitis) and pneumonia; polyarthritis was common. There were no confirmed family history of XLA. Most of the patients showed inverse ratios of CD4/CD8, the reason and potential significance are unclear. Signs and symptoms WAS occurs most often in males due to its X-linked recessive pattern of inheritance, affecting between 1 to 10 males per million. The first signs are usually petechiae and bruising, resulting from a low platelet count (i.e. thrombocytopenia). Spontaneous nose bleeds and bloody diarrhea are also common and eczema typically develops within the first month of life. Recurrent bacterial infections typically develop by three months of age. The majority of children with WAS develop at least one autoimmune disorder, and cancers (mainly lymphoma and leukemia) develop in up to a third of patients. Immunoglobulin M (IgM) levels are reduced, IgA and IgE are elevated, and IgG levels can be normal, reduced, or elevated. In addition to thrombocytopenia, WAS patients have abnormally small platelets (i.e. microthrombocytes) and ~30% also have elevated eosinophil counts (i.e. eosinophilia). Diagnosis The diagnosis can be made on the basis of clinical findings, the peripheral blood smear, and low immunoglobulin levels. Typically, IgM levels are low, IgA levels are elevated, and IgE levels may be elevated; paraproteins are occasionally observed. Skin immunologic testing (allergy testing) may reveal hyposensitivity. Individuals with Wiskott–Aldrich syndrome however are at higher risk for severe food allergies. Not all patients have a positive family history of the disorder; new mutations do occur. Often, leukemia may be suspected on the basis of low platelets and infections, and bone marrow biopsy may be performed. Decreased levels of WASp are typically observed. The current gold standard for diagnosis is DNA sequence analysis, which can detect WAS and the related disorders XLT and XLN in 95% of patients and carriers. Normal numbers of B cells with decreased IgG and IgA and increased IgM: Hyper-IgM syndromes Normal numbers of B cells with isotype or light chain deficiencies: heavy chain deletions, kappa chain deficiency, isolated IgG subclass deficiency, IgA with IgG subclass deficiency, selective immunoglobulin A deficiency Specific antibody deficiency to specific antigens with normal B cell and normal Ig concentrations Transient hypogammaglobulinemia of infancy (THI) The Wiskott-Aldrich Syndrome (WAS) is a rare X-linked immunohematological disorder characterized by eczema, profound thrombocytopenia, and progressive immunodeficiency. Severe hemorrhage, overwhelming sepsis, or lymphoreticular malignancy usually cause death in childhood. Recently, bone marrow transplantation (BMT) has been curative in some well-established cases, but there is no general agreement about the place of BMT in infants with WAS before the development of significant immunological abnormalities. We describe the successful use of early histocompatible BMT in a 10-month-old infant in whom WAS was diagnosed on the basis of eczema, thrombocytopenia, small platelets, and raised serum immunoglobulin A (Ig) and IgE, but before the development of immunodeficiency as evidenced clinically by recurrent infections, or immunologically by low serum IgM or consistently abnormal lymphocyte responses to mitogens. After an unstable period for several weeks posttransplantation when he developed marked hepatomegaly and severe interstitial pneumonitis, he made a good recovery. His eczema and thrombocytopenia resolved and he has shown no clinical or laboratory evidence of immunodeficiency. It is now over 2 years since his BMT. Because of the poor prognosis of WAS, where a histocompatible donor is available, BMT at the earliest opportunity, despite the inherent risks of such a procedure, may be the best option for an infant with WAS. To investigate the clinical and immunological laboratory features, gene mutations, treatment and prognosis in children with Wiskott-Aldrich syndrome (WAS). The clinical, laboratory characteristics, treatment and prognosis of 132 children with WAS, who visited Children's Hospital of Chongqing Medical University from April 2000 to June 2015, were analyzed retrospectively. All patients were male. The median age of disease onset was 15 days and the median age at diagnosis was 10 months. Of the 132 cases, 112 had classic WAS, 20 had X-linked thrombocytopenia (XLT). The median platelet count was 23×10(9)/L. All cases had the clinical characteristics of WAS including bleeding, eczema, and being susceptible to infection. The initial symptoms include hemorrhage (75.0%) and eczema (16.7%). Twenty-one cases had autoimmune diseases and one patient had leukemia. WAS protein (WASP) expression in 115 cases were measured by flow cytometry, 88 cases were negative, in 12 cases WASP decreased, in 5 cases it was normal, 10 cases had bimodal distribution. Eighty-one kinds of mutations were found in 122 families, including eight kinds of hot-spot mutations, which were 290 C> N / 291G> N (R86C / H / L), 665 C> T (R211X), 155 C> T (R41X), 168 C> T (T45 M), IVS1+ 1 g> t/ a, IVS6 + 5 g> a, IVS8 + 1 g> a and IVS8 + 1to + 6del gtga. Meantime, 29 kinds of novel mutations were found, which were 321T>C, 415C>A, 471C>T, 102-105delC, 521 del C, 1330 del A, IVS2-2 a>c, 168 C>A/1412 C> T, exon1-2 del/1412 C>T, and so on. The proportion of CD3(+) T cells (31.3%), helper T cells (37.3%) and cytotoxic T cells (38.6%) in the peripheral blood declined. The serum levels of IgG (51.1%), IgA (43.3%) and IgE (40.0%) increased, IgM (25.6%) decreased. Of the 132 cases, 72 remain survived, of whom 36 cases received hematopoietic stem cell transplantation (HSCT), 14 patients with classic WAS received intravenous immunoglobulin (IVIG) therapy. With regular IVIG therapy, the frequency of infections was reduced and the patients' symptoms were improved. The clinical characteristics of Wiskott-Aldrich syndrome were early age of onset, microthrombocytopenia, eczema and recurrent infections. The proportion of T lymphocyte declined, the serum levels of IgG, IgA, and IgE increased, and level of IgM decreased in a part of patients. The detection of WAS gene mutation and WAS protein detection was the key diagnostic methods. Regular IVIG can gain more time for children who will receive HSCT and improve their quality of life. The most important differential diagnosis includes the following: X-linked agammaglobulinemia characterizes by recurrent bacterial infections in boys, and genetic studies may reveal the presence of Bruton tyrosine kinase (BTK) mutations. Transient hypogammaglobulinemia of newborns presents in newborns above the age of 4 months and characterizes by recurrent pneumonia, meningitis, otitis media, and other problems that resemble Bruton disease. It is a physiological defect in the immune system caused by maternal IgG disappearance and corrected soon but requires treatment. In super-IgM syndrome, recurrent bacterial infections occur, but the cause of this illness is a mutation in the gene encoding for CD40 on T lymphocytes that causes a failure in T and B lymphocyte cooperation. Common variable immunodeficiency presents with recurrent bacterial infections, including sinopulmonary problems but later in life (second-fourth decade), and the diagnosis is made once all causes of immunodeficiency have been ruled out. [6] [7] [21] [33] of primary immunodeficiency (PID). These different forms can affect different parts of the immune system, including immunoglobulin production. Primary immunodeficiencies usually have a delay of several years between initial clinical presentation and diagnosis. Some primary immune deficiencies include ataxia-telangiectasia (A-T), autosomal recessive agammaglobulinemia (ARA), common variable immunodeficiency (CVID), hyper-IgM syndromes, IgG subclass deficiency, isolated non-IgG immunoglobulin deficiencies, severe combined immunodeficiency (SCID), specific antibody deficiency (SAD), Wiskott-Aldrich syndrome, or X-linked agammaglobulinemia. CVID is the most common form of primary immunodeficiency. SCID is considered a medical emergency and suspected cases require immediate specialist center referral for diagnosis and treatment. It is more often that hypogammaglobulinemia develops as a result of another condition, which are called secondary or acquired immune deficiencies. These Signs and symptoms Affects males 50% of the time if mother is a carrier for the gene. Children are generally asymptomatic until 6–9 months of age when maternal IgG decreases. Present with recurrent infections with Streptococcus pneumoniae, Haemophilus influenzae, Mycoplasma pneumoniae, hepatitis virus, and enterovirus CNS infections. Examination shows lymphoid hypoplasia (tonsils and adenoids, no splenomegaly or lymphadenopathy). There is significant decrease in all immunoglobulins. Genetics Most antibodies are gamma globulins. Antibodies are made mainly by plasma cells, which are daughter cells of the B cell line. The Btk enzyme plays an essential role in the maturation of B cells in the bone marrow, and when mutated, immature pro-B lymphocytes are unable to develop into pre-B lymphocytes, which normally develop into mature (naive) B cells that leave the bone marrow into the blood stream.\nHere is the question:\nGENETICS AND IMMUNOLOGY: 2-year-old boy. His personal history includes 3 episodes of acute otitis media, 1 meningococcal meningitis and 2 pneumonias (one middle lobe and one left upper lobe). She has been admitted on 3 occasions for thrombopenic purpura (on three occasions antiplatelet antibodies were negative and bone marrow showed normal megakaryocytes). Several males of the maternal family had died in childhood due to infectious processes. Physical examination showed lesions typical of atopic dermatitis. The immunological study showed a slight decrease in T-lymphocyte subpopulations; elevated IgA and IgE; decreased IgM and IgG at the lower limit of normal. What is the most likely diagnosis?\nHere are the potential choices:\n1. Wiskott-Aldrich syndrome.\n2. Hyper IgE syndrome.\n3. Transient hypogammaglobulinemia of childhood.\n4. X-linked severe combined immunodeficiency.\n5. Common variable immunodeficiency.\nThe correct answer is: 1. Wiskott-Aldrich syndrome." }, { "question": "You are a helpful medical expert, and your task is to answer a multi-choice medical question using the relevant documents. Please choose the answer from the provided options. Your responses will be used for research purposes only, so please have a definite answer.\nHere are the relevant documents:\n an injection of 1% lidocaine solution around this structure should attenuate this reflexive response.The most common delayed complication following carotid endarterectomy remains myocardial infarction. The possibility of a postoperative myocardial infarction should be considered as a cause of labile blood pressure and arrhythmias in high-risk patients.Thyroid and Parathyroid Glands. Surgery of the thyroid and parathyroid glands can result in hypocalcemia in the immedi-ate postoperative period. Manifestations include ECG changes (shortened P-R interval), muscle spasm (tetany, Chvostek’s sign, and Trousseau’s sign), paresthesias, and laryngospasm. Treatment includes calcium gluconate infusion and, if tetany ensues, chemical paralysis with intubation. Maintenance treat-ment is thyroid hormone replacement (after thyroidectomy) in addition to calcium carbonate and vitamin D.Recurrent laryngeal nerve (RLN) injury occurs in less than 5% of patients. Of those with injury, approximately 10% Since 1983 we have been involved in the diagnostic work-up and emergency treatment of a female patient now 48 years old who has a mitochondrial myopathy resembling Luft's disease. The syndrome was first described in 1959, and in more detail in 1962, by Luft and et al., who reported a picture of hypermetabolism with high temperature, extreme sweating, tachycardia, dyspnoea at rest, polydipsia, polyphagia and irritability but normal thyroid function. In 1971 and 1976 Haydar and Di Mauro presented a second case and proposed treatment with chloramphenicol. Our patient has the third case of the syndrome reported so far: her case was initially published in 1987. CASE REPORT. Since her 17th year of life the patient had suffered from episodes of fever, tachycardia and sweating. At the age of 32 these attacks worsened, leading to unconsciousness and apnoea. The patient then had to be intubated, ventilated and sometimes resuscitated. The diagnosis of MH susceptibility and Luft's disease was made on biochemical grounds after the first muscle biopsy in 1983. Therapy with chloramphenicol failed. Therapy with beta blockers, vitamin C and K or E, coenzyme Q10 and a high-caloric diet was started in 1985. The patient was registered with an emergency service, which flew her to our ICU whenever she had a severe crisis. For milder episodes she was supplied with an oxygen breathing mask at home. Myalgia increased with the episodes starting in 1988, and the patient needed dantrolene infusions and analgesics at home. To facilitate venepuncture a Port-A-Cath system was implanted in 1987, which had to be removed four times due to infection and sepsis. A muscle biopsy was taken in Rotterdam, which revealed differences in mitochondrial function from the biochemical findings recorded in 1983 and not in keeping with Luft's disease. Unfortunately, the patient was not able to undergo further metabolic investigations or therapeutic trials. ANAESTHESIA. The patient received three local and six general anaesthetics in our clinic. The muscle biopsies, two in 1983 and one in 1985, were performed under local infiltration with procaine and were uneventful. The general anaesthetics were carried out without MH trigger substances following pretreatment with dantrolene for the following surgical procedures: the repair of an extensive arterio-venous fistula between the brachiocephalicus trunk and the right jugular and subclavian vein, revision of the sternum cerclage, implantations and explanations of infectious Port-A-Cath systems. We used etomidate, propofol and fentanyl or alfentanil with nitrous oxide and oxygen for induction and maintenance of anaesthesia. Muscle relaxation was induced with vecuronium or atracurium. All cardiovascular, respiratory, metabolic and temperature measurements stayed in normal ranges. After the extensive vascular repair (av fistula) the patient had to be mechanically ventilated for some hours until normal body temperature was restored. At the end of all other periods of anaesthesia she was extubated in the operating theatre. In five cases the postoperative period was uneventful. Only once she developed a crisis with hyperthermia, tachycardia, sweating and dyspnoea. INTENSIVE CARE. From 1985 to 1992 the patient was treated in our ICU 21 times. On 11 occasions she was already intubated and being ventilated by the emergency service on arrival. Extubation was usually possible within 2-20 h. During the crisis, heart rate was about 160-190 per minute and temperature above 40 degrees C. Serum values of CK, glucose, BUN, electrolytes, lactate and thyroid hormones were always in the normal ranges. Blood gas controls showed a constant respiratory alkalosis, arterial pCO2 values decreasing to 20 mm Hg or less. In addition to mechanical ventilation, treatment consisted in dantrolene infusions and droperidol injections, supplemented from 1989 onward with piritramide injections because of the increased severity of myalgia. In 1991 we gave propofol by Several studies clarified the role of different interventions such as vitamine D replacement, denosumab treatment, and vertebroplasty in the prevention and management of falls and fractures. A trial tested the effectiveness of pharmaceutical assistance at the time of discharge, emphasizing the potential benefits for the patients and the health care system. Syncopal episodes frequently lead to hospital admission. A retrospective study evaluated the diagnostic yield of different tests and emphasized the importance to actively seek orthostatic hypotension in older patients. Finally, advances remain modest in the field of dementias. Post-infarction ventricular septal defect (VSD) is a rare but potentially lethal complication of acute myocardial infarction. Medical management is usually futile, so definitive surgery remains the treatment of choice but the risk surgery is very high and the optimal timing for surgery is still under debate. A 55-year-old man with no previous medical history attended the emergency-room for 12 h evolution of oppressive chest pain and strong anginal pain 7 days ago. On physical examination, blood pressure was 96/70 mmHg, pansystolic murmur over left sternal border without pulmonary crackles. An electrocardiogram revealed sinus rhythm 110 bpm, elevation ST and Q in inferior-posterior leads. Transthoracic echocardiogram showed inferoposterior akinesia, posterior-basal septal rupture (2 cm × 2 cm) with left-right shunt. Suspecting VSD in inferior-posterior acute myocardial infarction evolved, we performed emergency coronarography with 3-vessels disease and complete subacute occlusion of the mid segment of the right coronary artery. Left ventriculography demonstrated shunting of contrast from the left ventricule to the right ventricule. He was rejected for heart transplantation because of his age. Considering the high surgical risk to early surgery and his hemodynamic and clinical stability, delayed surgical treatment is decided, and 4 days after admission the patient suffered hemodynamic instability so venoarterial extracorporeal membrane oxygenation system (ECMO) is implanted as a bridge to reparative surgery. The 9th day after admission double bypass, interventricular defect repair with pericardial two-patch exclusion technique, and ECMO decannulation were performed. The patient's postoperative course was free of complications and was discharged 10 days post VSD repair surgery. Follow-up 3-month later revealed the patient to be in good functional status and good image outcome with intact interventricular septal patch without shunt. ECMO as a bridge to reparative surgery in postinfarction VSD is an adequate option to stabilize patients until surgery. An 86-year-old woman with low cardiac function was scheduled to undergo hip fracture surgery. Preoperative electrocardiogram showed complete left bundle brunch block, first degree atrioventricular block, left axis deviation and bigeminy. However, her electrocardiogram had changed to complete atrioventricular block on arrival at operating theater. ACC/AHA guideline on perioperative cardiovascular evaluation and care for non cardiac surgery indicates the assessment of both the urgency of the surgery and cardiac complications. Because complete atrioventricular block is classified to \"active cardiac conditions\", we decided to postpone the surgery for more detailed evaluation and treatment of cardiac conditions. In spite of the discontinuation of digoxin and carvegilol, complete atrioventricular block continued for a week, and the permanent pacemaker was inserted. The surgery was performed 2 weeks following the insertion of the pacemaker without any problems under combined general and lumbar epidural anesthesia. Treatment Medical therapy of aneurysm of the aortic sinus includes blood pressure control through the use of drugs, such as beta blockers. Another approach is surgical repair. The determination to perform surgery is usually based upon the diameter of the aortic root (with 5 centimeters being a rule of thumb - a normal size is 2-3 centimeters) and the rate of increase in its size (as determined through repeated echocardiography). An alternative to surgical repair or a ruptured aneurysm is percutaneous closure. In this technique, a wire is introduced via a small incision in the groin and advanced through the vascular system to the aneurysm. A closure device is advanced along the wire before being expanded to straddle the site of rupture. Hypercalcemia is not a rare event and can lead to severe consequences. Its main etiologies are primary hyperparathyroidism and neoplasic conditions. The iatrogenic etiology by vitamin D intoxication is more rarely found. A 76-year-old finish woman comes to the emergency room for chest pain. Her medical history is impossible to specify due to the language barrier and initial confusion. She has severe hypercalcaemia (4.14mmol/L), renal insufficiency, cardiac arrhythmia later complicated by an ischemic cardiac episode. Clinic and biologic examinations initially guided the research towards a hematological and neoplasic pathology. The iatrogenic etiology will be permitted by the contribution of details on its medical history and treatment learnt secondly. She was treated for post-surgical hypoparathyroidism by dihydrotachysterol, a vitamin D derivative. The cessation of substitution, treatment with hydration and biphosphonates allowed the rapid correction of hypercalcemia. Dihydrotachysterol intoxication is a rare etiology of hypercalcemia. Because of the longer half-life of this molecule, the risk of hypercalcemia seems to be greater than with other vitamin D derivatives. This molecule, withdrawn from the French market in 1982, is not detected by the dosage of 25 and 1.25 OH vitamin D. We report an original case of intoxication by dihydrotachysterol. The risk of hypercalcemia encountered with this molecule must be known. The close medical follow-up recommended in case of hypoparathyroidism seems to be particularly necessary in case of supplementation by this molecule. Amantadine hydrochloride is an antiviral medication used as therapy for parkinsonism and as a cognitive enhancer. We report 2 cases of massive, acute ingestion of amantadine hydrochloride confirmed with serial serum levels. A 47-year-old woman presented to the emergency department (ED) 30 minutes after ingesting 10 g of amantadine (150 mg/kg) by her report. Initial ECG revealed a sinus rhythm with rate of 93 bpm, and a QRS of 84 msec. While in the ED, the patient sustained a pulseless cardiac arrest and the monitor revealed ventricular tachycardia. She was successfully defibrillated. Postdefibrillation ECG showed a sinus rhythm (rate = 82 bpm), QRS of 236 msec, and QTc of 567 msec. The serum potassium was 1.0 mEq/L (1.0 mmol/L). The patient was given 300 ml (300 cc) 3% sodium chloride IV over 10 minutes. Ten minutes after completion of the hypertonic saline infusion, the patient's ECG abnormalities resolved and the QRS was 88 msec. Her potassium was repleted over the next 11 hours postpresentation, and she also received an IV bolus of 4 g of magnesium sulfate immediately after the cardiac arrest. No further hypotension, dysrhythmia, conduction delay, or ectopy was noted during the patient's hospital stay. The second case involved a 33-year-old female patient who presented 1 hour after ingesting 100 tablets of amantadine hydrochloride (100 mg/tab). Initial ECG revealed sinus tachycardia with a QRS of 113 msec, an R wave in lead aVR of 4-5 mm and a QTc of 526 msec. Her serum potassium was 3.0 mEq/L (3.0 mmol/L), her serum calcium was 9.4 mg/dl (2.35 mmol/L), and serum magnesium was 2.1 mg/dl (0.86 mmol/L) on labs drawn at initial presentation. The patient was intubated for airway protection, and her potassium was repleted and corrected over the next 9 hours. Her ECG abnormalities improved 8 hours after initial presentation and normalized at approximately 14 hours postingestion. The patient was discharged home 11 days after her ingestion. Acute amantadine toxicity manifests with life-threatening cardiotoxicity. Concurrent, often profound, hypokalemia may complicate the administration of sodium bicarbonate in the management of cardiac dysrhythmias. 130 young and middle age patients of both sexes with chronic form of coronary heart disease: functional class II-III stable exertional angina pectoris including functional class I-III chronic cardiac insufficiency were studied. In protocol 1 cured 70 patients (48 (68.6%) males and 22 (31.4%) females) 32-59 years of age (medium age was 48.4 +/- 3.25 years) with coronary heart disease. In protocol 2 (with prescription of calcium-D3) cured 60 patients (40 (66.7%) males and 20 (33.3%) females) 34-58 years of age (medium age was 47.8 +/- 3.12 years) with coronary heart disease. The groups were comparable on key parameters of disease. All patients had alimentary calcium deficit and (or) risk factors of osteoporosis, instrumental signs (X-ray filming and densitometry) of initial or evident osteoporosis. Correction of alimentary calcium deficit was realized by prescription of 1-3 tablets of calcium- Ds in different food intakes. Positive dynamics in decrease of functional class of angina pectoris and nitroglycerin requirement in both groups was noticed. Negative influence of calcium- D3 on studied indices of coronary heart disease severity was absent. The thirst and dry mouth in patients, who took furosemide, in group 1 were noticed against the background of body weight decrease (p < 0.05) and increase of diuresis. Decrease of the therapy antiarrhythmic action (p < 0.05) in patients, who took hydrochlorothiazide, was noticed too. It leaded to needs of furosemide and hydrochlorothiatide dose correction in protocol 1. In whole use of calcium- D3 together with anti-ischemic drugs in patients with chronic forms of coronary heart disease did not impair clinical course of angina pectoris and did not decrease efficiency of coronary heart disease therapy. Most hip fractures are treated surgically by implanting a prosthesis. Surgical treatment outweighs the risks of nonsurgical treatment which requires extensive bedrest. Prolonged immobilization increases risk of thromboembolism, pneumonia, deconditioning, and decubitus ulcers. Regardless, the surgery is a major stress, particularly in the elderly. Pain is also significant, and can also result in immobilization, so patients are encouraged to become mobile as soon as possible, often with the assistance of physical therapy. Skeletal traction pending surgery is not supported by the evidence. Regional nerve blocks are useful for pain management in hip fractures. Peripheral nerve blocks may reduce pain on movement and acute confusional state, may improve time to first mobilisation, and may reduce the risk of postoperative lower respiratory tract infection. Surgery can be performed under general anaesthesia or with neuraxial techniques – choice is based on surgical and patient factors, as 1. Hypo- and hypercalcemia can be explained as derangements of the calcium homeostasis. Hypocalcemic tetany usually alarming the patient tremendously is, at least in adults, rarely life-threatening. Hypercalcemia leads in 30% of the cases to clinical symptoms which may inadvertedly pass into a state of hypercalcemic crisis. This latter requires an often difficult emergency treatment. 2. Hypocalcemic tetany may be reversed by administering calcium i.v. or, in severe cases, by a calcium infusion. Only rarely are magnesium supplements necessary to let the tetany disappear. Vitamin D or dihydrotachysterol (DHT) do not correct hypocalcemia immediately, since their effects may be delayed up to 15-25 days. In order to normalize the serum calcium permanently, vitamin D or DHT treatment should be instituted as rarely as possible. 3. Initially, hypercalcemic crisis is best treated by forced intravenous fluid administration with normal saline (and furosemide) in combination with high doses of prednisone. Fluid-, sodium- and potassium balances ought to be checked during this type of treatment. A first evaluation of the effectiveness of these measures is recommended after 24 hours: treatment is continued in patients who respond favorably, while subjects who do not show a significant decrease of the serum calcium may either be given a phosphate infusion or mithramycine as a bolus. Calcitonin appears to be useful only to start treatment before institution of a phosphate infusion. A patient with a giant ovarian tumor weighing about 7 kg was successfully removed by operation. However, her ECG demonstrated ischemic changes after the operation. We report a case of ischemic heart disease due to persistent diuresis after giant ovarian tumor resection. A 75-year-old, 56.5 kg, 143.5 cm woman was admitted to our hospital for ovarian tumor resection. The preoperative ECG showed normal sinus rhythm and no ischemic changes. Both general anesthesia and epidural anesthesia were planed. An epidural catheter was inserted at T12-L1. Anesthesia was induced with propofol 100 mg, fentanyl 100 microg and vecuronium 8 mg under 100% oxygen inhalation. General anesthesia was maintained with sevoflurane while epidural anesthesia was achieved using 0.375% ropivacaine 6 ml. During the operation, blood pressure was 90-110/70-80 mmHg, with SaO2, 100% and heart rate, 70-80 beats x min(-1). The content of tumor was suctioned for 30 minutes. Surgery was successfully finished without any other incidence. After extubation, her ECG changed to atrial fibrillation from normal sinus rhythm and showed ST-T depression. And then her systolic blood pressure became 80 mmHg or below, but we found continued diuresis at about 10 ml x kg(-1) x hr(-1) for over 2 hr. The total of 7 unit vasopressin was intermittently given for vasoconstriction and antidiuresis. Her hemodynamic was immediately restored, and ECG turned to normal ST-T. The patient had uneventful postoperative recovery. 2017 highlights benefits of prevention. Better control of cardiovascular risk reduces the incidence of dementia and monthly high-dose vitamin D the incidence of respiratory infections in nursing home. Pre-operative geriatric assessment lowers by 20% the rate of delirium after hip-fracture surgery and complications in vascular surgery. Deleterious effects are also reported. High-dose vitamin D triples the rate of falls in supplemented residents and doesn't improve gait speed in sedentary men. Widely used in cardiovascular prevention, antithrombotic therapy is associated with an astonishing risk of subdural bleeding that further increases with the number of drugs combined together. Finally, the non-pharmacological management of behavioral and psychotic symptoms in advanced dementia, although effective, doesn't reduce the associated burden for proxies. Spontaneous coronary artery dissection (SCAD) is a rare cause of acute coronary syndrome (ACS), seen mostly in young females. The rarity and limited knowledge of the disease make its management challenging. Prompt diagnosis of the condition is extremely important to decrease both long- and short-term complications. Treatment options depend on hemodynamic stability and the location of the dissection- with more distal lesions treated more conservatively as opposed to proximal lesions which are treated with percutaneous coronary intervention (PCI) or coronary artery bypass graft (CABG). The following are the two cases with different presentation, management and outcomes. Our first patient was a 35-year-old woman with no medical history who presented with acute, anginal pain, diaphoresis and palpitations. She was hemodynamically stable on presentation, with work-up significant for electrocardiogram (ECG) with sinus bradycardia, ST elevation in leads V1-V6, and elevated troponin level of 4 ng/ml. There was no evidence of a pulmonary embolism on computed tomography (CT) of the chest. A coronary angiogram showed 100% dissection of the proximal to mid-left anterior descending (LAD) artery. Attempts to place a stent in the proximal to mid LAD were unsuccessful as the true lumen of the LAD was not accessible. The patient became hemodynamically unstable, and an emergent CABG was done, restoring blood flow. The patient recovered during her hospital stay and was discharged with dual antiplatelet therapy (DAPT), beta-blockers, and atorvastatin. The second patient was a 28-year-old woman, with a history of hypertension who presented with anginal chest pain. Workup showed ECG with minimal ST elevations in anteroseptal leads, with elevated troponin level to 0.71 ng/ml. Coronary angiogram showed 40-50% stenosis of the mid LAD with an aneurysmal segment. An echocardiogram showed no evidence of wall motion abnormalities, and she had a normal left ventricular ejection fraction (LVEF). She was discharged home the next day, on medical management. After two days, she returned to the hospital with similar complaints, with work-up significant for ECG with non-specific ST-T abnormality, and troponin level which peaked at 2.22 ng/ml. She was started on a heparin drip, and a repeat left heart catheterization revealed type 2 dissection of the mid to distal LAD, with intravascular ultrasound showing a fractional flow reserve of 0.76. She was discharged home on DAPT, beta-blocker, calcium channel blocker (CCB), and atorvastatin, with close cardiology follow up. These two cases highlight the importance of keeping in mind the possibility of SCAD, especially when relatively healthy young women present with anginal symptoms. Early diagnosis of the condition and prompt management are extremely important to ensure favourable outcomes. The two cases also describe the coronary angiogram findings in SCAD, and the different strategies employed in the management of this condition. 729 consecutive patients underwent thyroidectomy in 1988 in the same institution, including 477 (68%) bilateral resections and 242 (33%) total thyroidectomies. An effort was made to see and save all 4 parathyroids and their blood supply. Early post-operative hypoparathyroidism was defined at day 5, by serum calcium less than 8 mg/dl. and serum phosphate less than 4 mg/dl or by serum calcium only if greater than 7.5 mg/dl. Patients afflicted with early hypoparathyroidism were given calcium tablets without any vit D for 1 year at most. Follow-up, checking serum Ca, P and i PIH was done on a 3 months basis during 1 year. Permanent hypoparathyroidism was defined by persistence of the above-mentioned criteria after 1 year, and eventually vit D was started. 27 patients (5.6% our of 477 bilateral thyroid resections) experienced early post-op hypoparathyroidism. Inciting factors were previous thyroid surgery (4), radioiodine treatment (2), modified neck dissection (2), sternal split with mediastinal node clearance (1), visualization of 1 parathyroid gland only (3 redo cases) and autotransplantation of more than 1 parathyroid (1 case). 1 patient was lost for follow-up. 25 others recovered a normal parathyroid function. 1 is permanently hypoparathyroid (1 redo case with other risk factors). Painstaking parathyroid dissection allows a 0% rate of permanent hypoparathyroidism after primary surgery, if vit D is not given in the early post-operative period. We suggest that avoidance of early vit D prescription in cases of early post-operative hypoparathyroidism, leading to mild sustained hypocalcemia, stimulates the spared parathyroid glands (including a possible 5th) and therefore allows full recovery of the parathyroid function.\nHere is the question:\nANESTHESIOLOGY AND CRITICAL CARE: We are consulted to assess an 83-year-old woman admitted to the Trauma service for a hip fracture 6 hours ago. She has AP of hypertension, LBP, moderate dementia and lives in a nursing home. Her usual treatment is thiazide, atorvastatin, donepezil, Calcium and vitamin D. EF: Confused patient, pulse 90 bpm, respiratory rate 20 rpm, T art 170/88, jugular venous pressure normal. The CBC and chest X-ray are normal and the ECG shows sinus rhythm without ischemic alterations. Which of the following is the most correct therapeutic approach?\nHere are the potential choices:\n1. Delay surgery until the confusional picture has disappeared.\n2. Delay surgery and perform an echocardiogram.\n3. Delay surgery until good blood pressure control.\n4. Start a beta-blocker and initiate surgery.\n5. Perform closed osteosynthesis, avoiding in any case the implantation of prosthesis.\nThe correct answer is: ", "gold_answer": "4 Start a beta-blocker and initiate surgery.", "prediction": "You are a helpful medical expert, and your task is to answer a multi-choice medical question using the relevant documents. Please choose the answer from the provided options. Your responses will be used for research purposes only, so please have a definite answer.\nHere are the relevant documents:\n an injection of 1% lidocaine solution around this structure should attenuate this reflexive response.The most common delayed complication following carotid endarterectomy remains myocardial infarction. The possibility of a postoperative myocardial infarction should be considered as a cause of labile blood pressure and arrhythmias in high-risk patients.Thyroid and Parathyroid Glands. Surgery of the thyroid and parathyroid glands can result in hypocalcemia in the immedi-ate postoperative period. Manifestations include ECG changes (shortened P-R interval), muscle spasm (tetany, Chvostek’s sign, and Trousseau’s sign), paresthesias, and laryngospasm. Treatment includes calcium gluconate infusion and, if tetany ensues, chemical paralysis with intubation. Maintenance treat-ment is thyroid hormone replacement (after thyroidectomy) in addition to calcium carbonate and vitamin D.Recurrent laryngeal nerve (RLN) injury occurs in less than 5% of patients. Of those with injury, approximately 10% Since 1983 we have been involved in the diagnostic work-up and emergency treatment of a female patient now 48 years old who has a mitochondrial myopathy resembling Luft's disease. The syndrome was first described in 1959, and in more detail in 1962, by Luft and et al., who reported a picture of hypermetabolism with high temperature, extreme sweating, tachycardia, dyspnoea at rest, polydipsia, polyphagia and irritability but normal thyroid function. In 1971 and 1976 Haydar and Di Mauro presented a second case and proposed treatment with chloramphenicol. Our patient has the third case of the syndrome reported so far: her case was initially published in 1987. CASE REPORT. Since her 17th year of life the patient had suffered from episodes of fever, tachycardia and sweating. At the age of 32 these attacks worsened, leading to unconsciousness and apnoea. The patient then had to be intubated, ventilated and sometimes resuscitated. The diagnosis of MH susceptibility and Luft's disease was made on biochemical grounds after the first muscle biopsy in 1983. Therapy with chloramphenicol failed. Therapy with beta blockers, vitamin C and K or E, coenzyme Q10 and a high-caloric diet was started in 1985. The patient was registered with an emergency service, which flew her to our ICU whenever she had a severe crisis. For milder episodes she was supplied with an oxygen breathing mask at home. Myalgia increased with the episodes starting in 1988, and the patient needed dantrolene infusions and analgesics at home. To facilitate venepuncture a Port-A-Cath system was implanted in 1987, which had to be removed four times due to infection and sepsis. A muscle biopsy was taken in Rotterdam, which revealed differences in mitochondrial function from the biochemical findings recorded in 1983 and not in keeping with Luft's disease. Unfortunately, the patient was not able to undergo further metabolic investigations or therapeutic trials. ANAESTHESIA. The patient received three local and six general anaesthetics in our clinic. The muscle biopsies, two in 1983 and one in 1985, were performed under local infiltration with procaine and were uneventful. The general anaesthetics were carried out without MH trigger substances following pretreatment with dantrolene for the following surgical procedures: the repair of an extensive arterio-venous fistula between the brachiocephalicus trunk and the right jugular and subclavian vein, revision of the sternum cerclage, implantations and explanations of infectious Port-A-Cath systems. We used etomidate, propofol and fentanyl or alfentanil with nitrous oxide and oxygen for induction and maintenance of anaesthesia. Muscle relaxation was induced with vecuronium or atracurium. All cardiovascular, respiratory, metabolic and temperature measurements stayed in normal ranges. After the extensive vascular repair (av fistula) the patient had to be mechanically ventilated for some hours until normal body temperature was restored. At the end of all other periods of anaesthesia she was extubated in the operating theatre. In five cases the postoperative period was uneventful. Only once she developed a crisis with hyperthermia, tachycardia, sweating and dyspnoea. INTENSIVE CARE. From 1985 to 1992 the patient was treated in our ICU 21 times. On 11 occasions she was already intubated and being ventilated by the emergency service on arrival. Extubation was usually possible within 2-20 h. During the crisis, heart rate was about 160-190 per minute and temperature above 40 degrees C. Serum values of CK, glucose, BUN, electrolytes, lactate and thyroid hormones were always in the normal ranges. Blood gas controls showed a constant respiratory alkalosis, arterial pCO2 values decreasing to 20 mm Hg or less. In addition to mechanical ventilation, treatment consisted in dantrolene infusions and droperidol injections, supplemented from 1989 onward with piritramide injections because of the increased severity of myalgia. In 1991 we gave propofol by Several studies clarified the role of different interventions such as vitamine D replacement, denosumab treatment, and vertebroplasty in the prevention and management of falls and fractures. A trial tested the effectiveness of pharmaceutical assistance at the time of discharge, emphasizing the potential benefits for the patients and the health care system. Syncopal episodes frequently lead to hospital admission. A retrospective study evaluated the diagnostic yield of different tests and emphasized the importance to actively seek orthostatic hypotension in older patients. Finally, advances remain modest in the field of dementias. Post-infarction ventricular septal defect (VSD) is a rare but potentially lethal complication of acute myocardial infarction. Medical management is usually futile, so definitive surgery remains the treatment of choice but the risk surgery is very high and the optimal timing for surgery is still under debate. A 55-year-old man with no previous medical history attended the emergency-room for 12 h evolution of oppressive chest pain and strong anginal pain 7 days ago. On physical examination, blood pressure was 96/70 mmHg, pansystolic murmur over left sternal border without pulmonary crackles. An electrocardiogram revealed sinus rhythm 110 bpm, elevation ST and Q in inferior-posterior leads. Transthoracic echocardiogram showed inferoposterior akinesia, posterior-basal septal rupture (2 cm × 2 cm) with left-right shunt. Suspecting VSD in inferior-posterior acute myocardial infarction evolved, we performed emergency coronarography with 3-vessels disease and complete subacute occlusion of the mid segment of the right coronary artery. Left ventriculography demonstrated shunting of contrast from the left ventricule to the right ventricule. He was rejected for heart transplantation because of his age. Considering the high surgical risk to early surgery and his hemodynamic and clinical stability, delayed surgical treatment is decided, and 4 days after admission the patient suffered hemodynamic instability so venoarterial extracorporeal membrane oxygenation system (ECMO) is implanted as a bridge to reparative surgery. The 9th day after admission double bypass, interventricular defect repair with pericardial two-patch exclusion technique, and ECMO decannulation were performed. The patient's postoperative course was free of complications and was discharged 10 days post VSD repair surgery. Follow-up 3-month later revealed the patient to be in good functional status and good image outcome with intact interventricular septal patch without shunt. ECMO as a bridge to reparative surgery in postinfarction VSD is an adequate option to stabilize patients until surgery. An 86-year-old woman with low cardiac function was scheduled to undergo hip fracture surgery. Preoperative electrocardiogram showed complete left bundle brunch block, first degree atrioventricular block, left axis deviation and bigeminy. However, her electrocardiogram had changed to complete atrioventricular block on arrival at operating theater. ACC/AHA guideline on perioperative cardiovascular evaluation and care for non cardiac surgery indicates the assessment of both the urgency of the surgery and cardiac complications. Because complete atrioventricular block is classified to \"active cardiac conditions\", we decided to postpone the surgery for more detailed evaluation and treatment of cardiac conditions. In spite of the discontinuation of digoxin and carvegilol, complete atrioventricular block continued for a week, and the permanent pacemaker was inserted. The surgery was performed 2 weeks following the insertion of the pacemaker without any problems under combined general and lumbar epidural anesthesia. Treatment Medical therapy of aneurysm of the aortic sinus includes blood pressure control through the use of drugs, such as beta blockers. Another approach is surgical repair. The determination to perform surgery is usually based upon the diameter of the aortic root (with 5 centimeters being a rule of thumb - a normal size is 2-3 centimeters) and the rate of increase in its size (as determined through repeated echocardiography). An alternative to surgical repair or a ruptured aneurysm is percutaneous closure. In this technique, a wire is introduced via a small incision in the groin and advanced through the vascular system to the aneurysm. A closure device is advanced along the wire before being expanded to straddle the site of rupture. Hypercalcemia is not a rare event and can lead to severe consequences. Its main etiologies are primary hyperparathyroidism and neoplasic conditions. The iatrogenic etiology by vitamin D intoxication is more rarely found. A 76-year-old finish woman comes to the emergency room for chest pain. Her medical history is impossible to specify due to the language barrier and initial confusion. She has severe hypercalcaemia (4.14mmol/L), renal insufficiency, cardiac arrhythmia later complicated by an ischemic cardiac episode. Clinic and biologic examinations initially guided the research towards a hematological and neoplasic pathology. The iatrogenic etiology will be permitted by the contribution of details on its medical history and treatment learnt secondly. She was treated for post-surgical hypoparathyroidism by dihydrotachysterol, a vitamin D derivative. The cessation of substitution, treatment with hydration and biphosphonates allowed the rapid correction of hypercalcemia. Dihydrotachysterol intoxication is a rare etiology of hypercalcemia. Because of the longer half-life of this molecule, the risk of hypercalcemia seems to be greater than with other vitamin D derivatives. This molecule, withdrawn from the French market in 1982, is not detected by the dosage of 25 and 1.25 OH vitamin D. We report an original case of intoxication by dihydrotachysterol. The risk of hypercalcemia encountered with this molecule must be known. The close medical follow-up recommended in case of hypoparathyroidism seems to be particularly necessary in case of supplementation by this molecule. Amantadine hydrochloride is an antiviral medication used as therapy for parkinsonism and as a cognitive enhancer. We report 2 cases of massive, acute ingestion of amantadine hydrochloride confirmed with serial serum levels. A 47-year-old woman presented to the emergency department (ED) 30 minutes after ingesting 10 g of amantadine (150 mg/kg) by her report. Initial ECG revealed a sinus rhythm with rate of 93 bpm, and a QRS of 84 msec. While in the ED, the patient sustained a pulseless cardiac arrest and the monitor revealed ventricular tachycardia. She was successfully defibrillated. Postdefibrillation ECG showed a sinus rhythm (rate = 82 bpm), QRS of 236 msec, and QTc of 567 msec. The serum potassium was 1.0 mEq/L (1.0 mmol/L). The patient was given 300 ml (300 cc) 3% sodium chloride IV over 10 minutes. Ten minutes after completion of the hypertonic saline infusion, the patient's ECG abnormalities resolved and the QRS was 88 msec. Her potassium was repleted over the next 11 hours postpresentation, and she also received an IV bolus of 4 g of magnesium sulfate immediately after the cardiac arrest. No further hypotension, dysrhythmia, conduction delay, or ectopy was noted during the patient's hospital stay. The second case involved a 33-year-old female patient who presented 1 hour after ingesting 100 tablets of amantadine hydrochloride (100 mg/tab). Initial ECG revealed sinus tachycardia with a QRS of 113 msec, an R wave in lead aVR of 4-5 mm and a QTc of 526 msec. Her serum potassium was 3.0 mEq/L (3.0 mmol/L), her serum calcium was 9.4 mg/dl (2.35 mmol/L), and serum magnesium was 2.1 mg/dl (0.86 mmol/L) on labs drawn at initial presentation. The patient was intubated for airway protection, and her potassium was repleted and corrected over the next 9 hours. Her ECG abnormalities improved 8 hours after initial presentation and normalized at approximately 14 hours postingestion. The patient was discharged home 11 days after her ingestion. Acute amantadine toxicity manifests with life-threatening cardiotoxicity. Concurrent, often profound, hypokalemia may complicate the administration of sodium bicarbonate in the management of cardiac dysrhythmias. 130 young and middle age patients of both sexes with chronic form of coronary heart disease: functional class II-III stable exertional angina pectoris including functional class I-III chronic cardiac insufficiency were studied. In protocol 1 cured 70 patients (48 (68.6%) males and 22 (31.4%) females) 32-59 years of age (medium age was 48.4 +/- 3.25 years) with coronary heart disease. In protocol 2 (with prescription of calcium-D3) cured 60 patients (40 (66.7%) males and 20 (33.3%) females) 34-58 years of age (medium age was 47.8 +/- 3.12 years) with coronary heart disease. The groups were comparable on key parameters of disease. All patients had alimentary calcium deficit and (or) risk factors of osteoporosis, instrumental signs (X-ray filming and densitometry) of initial or evident osteoporosis. Correction of alimentary calcium deficit was realized by prescription of 1-3 tablets of calcium- Ds in different food intakes. Positive dynamics in decrease of functional class of angina pectoris and nitroglycerin requirement in both groups was noticed. Negative influence of calcium- D3 on studied indices of coronary heart disease severity was absent. The thirst and dry mouth in patients, who took furosemide, in group 1 were noticed against the background of body weight decrease (p < 0.05) and increase of diuresis. Decrease of the therapy antiarrhythmic action (p < 0.05) in patients, who took hydrochlorothiazide, was noticed too. It leaded to needs of furosemide and hydrochlorothiatide dose correction in protocol 1. In whole use of calcium- D3 together with anti-ischemic drugs in patients with chronic forms of coronary heart disease did not impair clinical course of angina pectoris and did not decrease efficiency of coronary heart disease therapy. Most hip fractures are treated surgically by implanting a prosthesis. Surgical treatment outweighs the risks of nonsurgical treatment which requires extensive bedrest. Prolonged immobilization increases risk of thromboembolism, pneumonia, deconditioning, and decubitus ulcers. Regardless, the surgery is a major stress, particularly in the elderly. Pain is also significant, and can also result in immobilization, so patients are encouraged to become mobile as soon as possible, often with the assistance of physical therapy. Skeletal traction pending surgery is not supported by the evidence. Regional nerve blocks are useful for pain management in hip fractures. Peripheral nerve blocks may reduce pain on movement and acute confusional state, may improve time to first mobilisation, and may reduce the risk of postoperative lower respiratory tract infection. Surgery can be performed under general anaesthesia or with neuraxial techniques – choice is based on surgical and patient factors, as 1. Hypo- and hypercalcemia can be explained as derangements of the calcium homeostasis. Hypocalcemic tetany usually alarming the patient tremendously is, at least in adults, rarely life-threatening. Hypercalcemia leads in 30% of the cases to clinical symptoms which may inadvertedly pass into a state of hypercalcemic crisis. This latter requires an often difficult emergency treatment. 2. Hypocalcemic tetany may be reversed by administering calcium i.v. or, in severe cases, by a calcium infusion. Only rarely are magnesium supplements necessary to let the tetany disappear. Vitamin D or dihydrotachysterol (DHT) do not correct hypocalcemia immediately, since their effects may be delayed up to 15-25 days. In order to normalize the serum calcium permanently, vitamin D or DHT treatment should be instituted as rarely as possible. 3. Initially, hypercalcemic crisis is best treated by forced intravenous fluid administration with normal saline (and furosemide) in combination with high doses of prednisone. Fluid-, sodium- and potassium balances ought to be checked during this type of treatment. A first evaluation of the effectiveness of these measures is recommended after 24 hours: treatment is continued in patients who respond favorably, while subjects who do not show a significant decrease of the serum calcium may either be given a phosphate infusion or mithramycine as a bolus. Calcitonin appears to be useful only to start treatment before institution of a phosphate infusion. A patient with a giant ovarian tumor weighing about 7 kg was successfully removed by operation. However, her ECG demonstrated ischemic changes after the operation. We report a case of ischemic heart disease due to persistent diuresis after giant ovarian tumor resection. A 75-year-old, 56.5 kg, 143.5 cm woman was admitted to our hospital for ovarian tumor resection. The preoperative ECG showed normal sinus rhythm and no ischemic changes. Both general anesthesia and epidural anesthesia were planed. An epidural catheter was inserted at T12-L1. Anesthesia was induced with propofol 100 mg, fentanyl 100 microg and vecuronium 8 mg under 100% oxygen inhalation. General anesthesia was maintained with sevoflurane while epidural anesthesia was achieved using 0.375% ropivacaine 6 ml. During the operation, blood pressure was 90-110/70-80 mmHg, with SaO2, 100% and heart rate, 70-80 beats x min(-1). The content of tumor was suctioned for 30 minutes. Surgery was successfully finished without any other incidence. After extubation, her ECG changed to atrial fibrillation from normal sinus rhythm and showed ST-T depression. And then her systolic blood pressure became 80 mmHg or below, but we found continued diuresis at about 10 ml x kg(-1) x hr(-1) for over 2 hr. The total of 7 unit vasopressin was intermittently given for vasoconstriction and antidiuresis. Her hemodynamic was immediately restored, and ECG turned to normal ST-T. The patient had uneventful postoperative recovery. 2017 highlights benefits of prevention. Better control of cardiovascular risk reduces the incidence of dementia and monthly high-dose vitamin D the incidence of respiratory infections in nursing home. Pre-operative geriatric assessment lowers by 20% the rate of delirium after hip-fracture surgery and complications in vascular surgery. Deleterious effects are also reported. High-dose vitamin D triples the rate of falls in supplemented residents and doesn't improve gait speed in sedentary men. Widely used in cardiovascular prevention, antithrombotic therapy is associated with an astonishing risk of subdural bleeding that further increases with the number of drugs combined together. Finally, the non-pharmacological management of behavioral and psychotic symptoms in advanced dementia, although effective, doesn't reduce the associated burden for proxies. Spontaneous coronary artery dissection (SCAD) is a rare cause of acute coronary syndrome (ACS), seen mostly in young females. The rarity and limited knowledge of the disease make its management challenging. Prompt diagnosis of the condition is extremely important to decrease both long- and short-term complications. Treatment options depend on hemodynamic stability and the location of the dissection- with more distal lesions treated more conservatively as opposed to proximal lesions which are treated with percutaneous coronary intervention (PCI) or coronary artery bypass graft (CABG). The following are the two cases with different presentation, management and outcomes. Our first patient was a 35-year-old woman with no medical history who presented with acute, anginal pain, diaphoresis and palpitations. She was hemodynamically stable on presentation, with work-up significant for electrocardiogram (ECG) with sinus bradycardia, ST elevation in leads V1-V6, and elevated troponin level of 4 ng/ml. There was no evidence of a pulmonary embolism on computed tomography (CT) of the chest. A coronary angiogram showed 100% dissection of the proximal to mid-left anterior descending (LAD) artery. Attempts to place a stent in the proximal to mid LAD were unsuccessful as the true lumen of the LAD was not accessible. The patient became hemodynamically unstable, and an emergent CABG was done, restoring blood flow. The patient recovered during her hospital stay and was discharged with dual antiplatelet therapy (DAPT), beta-blockers, and atorvastatin. The second patient was a 28-year-old woman, with a history of hypertension who presented with anginal chest pain. Workup showed ECG with minimal ST elevations in anteroseptal leads, with elevated troponin level to 0.71 ng/ml. Coronary angiogram showed 40-50% stenosis of the mid LAD with an aneurysmal segment. An echocardiogram showed no evidence of wall motion abnormalities, and she had a normal left ventricular ejection fraction (LVEF). She was discharged home the next day, on medical management. After two days, she returned to the hospital with similar complaints, with work-up significant for ECG with non-specific ST-T abnormality, and troponin level which peaked at 2.22 ng/ml. She was started on a heparin drip, and a repeat left heart catheterization revealed type 2 dissection of the mid to distal LAD, with intravascular ultrasound showing a fractional flow reserve of 0.76. She was discharged home on DAPT, beta-blocker, calcium channel blocker (CCB), and atorvastatin, with close cardiology follow up. These two cases highlight the importance of keeping in mind the possibility of SCAD, especially when relatively healthy young women present with anginal symptoms. Early diagnosis of the condition and prompt management are extremely important to ensure favourable outcomes. The two cases also describe the coronary angiogram findings in SCAD, and the different strategies employed in the management of this condition. 729 consecutive patients underwent thyroidectomy in 1988 in the same institution, including 477 (68%) bilateral resections and 242 (33%) total thyroidectomies. An effort was made to see and save all 4 parathyroids and their blood supply. Early post-operative hypoparathyroidism was defined at day 5, by serum calcium less than 8 mg/dl. and serum phosphate less than 4 mg/dl or by serum calcium only if greater than 7.5 mg/dl. Patients afflicted with early hypoparathyroidism were given calcium tablets without any vit D for 1 year at most. Follow-up, checking serum Ca, P and i PIH was done on a 3 months basis during 1 year. Permanent hypoparathyroidism was defined by persistence of the above-mentioned criteria after 1 year, and eventually vit D was started. 27 patients (5.6% our of 477 bilateral thyroid resections) experienced early post-op hypoparathyroidism. Inciting factors were previous thyroid surgery (4), radioiodine treatment (2), modified neck dissection (2), sternal split with mediastinal node clearance (1), visualization of 1 parathyroid gland only (3 redo cases) and autotransplantation of more than 1 parathyroid (1 case). 1 patient was lost for follow-up. 25 others recovered a normal parathyroid function. 1 is permanently hypoparathyroid (1 redo case with other risk factors). Painstaking parathyroid dissection allows a 0% rate of permanent hypoparathyroidism after primary surgery, if vit D is not given in the early post-operative period. We suggest that avoidance of early vit D prescription in cases of early post-operative hypoparathyroidism, leading to mild sustained hypocalcemia, stimulates the spared parathyroid glands (including a possible 5th) and therefore allows full recovery of the parathyroid function.\nHere is the question:\nANESTHESIOLOGY AND CRITICAL CARE: We are consulted to assess an 83-year-old woman admitted to the Trauma service for a hip fracture 6 hours ago. She has AP of hypertension, LBP, moderate dementia and lives in a nursing home. Her usual treatment is thiazide, atorvastatin, donepezil, Calcium and vitamin D. EF: Confused patient, pulse 90 bpm, respiratory rate 20 rpm, T art 170/88, jugular venous pressure normal. The CBC and chest X-ray are normal and the ECG shows sinus rhythm without ischemic alterations. Which of the following is the most correct therapeutic approach?\nHere are the potential choices:\n1. Delay surgery until the confusional picture has disappeared.\n2. Delay surgery and perform an echocardiogram.\n3. Delay surgery until good blood pressure control.\n4. Start a beta-blocker and initiate surgery.\n5. Perform closed osteosynthesis, avoiding in any case the implantation of prosthesis.\nThe correct answer is: 4. Start a beta-blocker and initiate surgery." }, { "question": "You are a helpful medical expert, and your task is to answer a multi-choice medical question using the relevant documents. Please choose the answer from the provided options. Your responses will be used for research purposes only, so please have a definite answer.\nHere are the relevant documents:\n Because approximately 1 in 10 women with a breast lump or abnormal mammography result will have breast cancer, a series of decisions must be taken by a primary care practitioner to exclude or establish a diagnosis of breast cancer among these women. To determine the most accurate and least invasive means to evaluate an abnormal mammography result and a palpable breast abnormality. MEDLINE search (January 1966 to March 2003) for articles and reviews describing the accuracy of clinical examination, biopsy procedures, and radiographic examination for patients with abnormal mammography results or palpable breast abnormalities. The authors reviewed abstracts and selected articles that provided relevant primary data. Studies were included if 1) mammography, fine-needle aspiration biopsy, or core-needle biopsy was performed before a definitive diagnosis was obtained; 2) the study sample included 100 or more women; and 3) breast cancer status was determined from histopathology review of excisional biopsy specimens, from linkage with a state cancer registry or the Surveillance, Epidemiology, and End Results program, or from clinical follow-up of 95% or more of the study sample. One investigator abstracted results. Methods were evaluated for major potential biases, but methodologic scoring was not performed. Likelihood ratios for first screening mammography were 0.1 for the Breast Imaging Reporting and Data System (BI-RADS) assessment category \"negative or benign finding,\" 1.2 for \"probably benign finding,\" 7 for \"need additional imaging evaluation,\" 125 for \"suspicious abnormality,\" and 2200 for \"highly suggestive of malignancy.\" For fine-needle aspiration biopsy of a palpable lump performed by formally trained physicians, the likelihood ratio was infinity for an assessment of \"malignant,\" 2.6 for \"atypical/suspicious,\" and 0.02 for \"benign.\" When diagnostic mammography was used to evaluate a palpable lump or nonpalpable breast abnormality, the positive likelihood ratios were 5.6 and 9.4, and the negative likelihood ratios were 0.15 and 0.19, respectively. Women whose screening mammography results are interpreted as \"suspicious abnormality\" or \"highly suggestive of malignancy\" have a high risk for breast cancer and should undergo core-needle biopsy or needle localization with surgical biopsy. Women whose screening mammography results are interpreted as \"need additional imaging evaluation\" have a moderate risk for breast cancer and should undergo diagnostic mammography or ultrasonography to decide whether a nonpalpable breast lesion should be biopsied. Women whose screening mammography results are interpreted as \"probably benign finding\" have a low risk for breast cancer and can undergo follow-up mammography in 6 months. Either fine-needle aspiration biopsy or ultrasonography is recommended as the first diagnostic test of a palpable breast abnormality to distinguish simple cysts from solid masses. Fine-needle aspiration biopsy also allows characterization of a solid mass. Diagnostic mammography does not help determine whether a palpable breast mass should be biopsied and should not affect the decision to perform a biopsy. The importance of these missed cancers is not clear, particularly if the woman is getting yearly mammograms. Research on a closely related situation has shown that small cancers that are not acted upon immediately, but are observed over periods of several years, will have good outcomes. A group of 3,184 women had mammograms that were formally classified as \"probably benign\". This classification is for patients who are not clearly normal but have some area of minor concern. This results not in the patient being biopsied, but rather in having early follow up mammography every six months for three years to determine whether there has been any change in status. Of these 3,184 women, 17 (0.5%) did have cancers. Most importantly, when the diagnosis was finally made, they were all still stage 0 or 1, the earliest stages. Five years after treatment, none of these 17 women had evidence of re-occurrence. Thus, small early cancers, even though not acted on immediately, were still reliably Mammography Mammography is a common screening method, since it is relatively fast and widely available in developed countries. Mammography is a type of radiography used on the breasts. It is typically used for two purposes: to aid in the diagnosis of a woman who is experiencing symptoms or has been called back for follow-up views (called diagnostic mammography), and for medical screening of apparently healthy women (called screening mammography). Mammography is not very useful in finding breast tumors in dense breast tissue characteristic of women under 40 years. In women over 50 without dense breasts, breast cancers detected by screening mammography are usually smaller and less aggressive than those detected by patients or doctors as a breast lump. This is because the most aggressive breast cancers are found in dense breast tissue, which mammograms perform poorly on. Histologically proven benign breast disease increases a woman's relative risk for subsequent cancer development. Yet follow-up guidelines for mammogram and clinical breast examination after a benign breast biopsy are lacking. Our objective was to determine if increased surveillance is indicated following a benign breast biopsy. Following institutional review board approval, a retrospective database review was conducted of prospectively gathered patients who had a benign breast biopsy (core or excisional) for an abnormality detected on mammogram, ultrasound, or clinical breast examination. Follow-up, for all subjects, was a clinical breast examination and mammogram or ultrasound at 6 months, 1 year, and 2 years after benign breast biopsy by a breast surgeon. End points were the need for additional biopsies or cancer detection. Statistical analysis was performed using chi-squared analysis. From January 2000 to July 2003, 156 patients age 18-86 years had a benign breast biopsy. During the 2 year follow-up, 20 patients (13%) required a subsequent biopsy. No significant difference was observed in mean age, race, menarche, menopause, parity, age at first live birth, use of oral contraceptives, history of prior biopsy, or the pathology of the initial lesion between those who needed a subsequent biopsy and those who did not. Seven excisional biopsies were performed (one at 6 months, four at 1 year, and two at 2 years follow-up) for growth of the benign breast biopsy lesion, and pathology remained concordant with the original diagnosis. Thirteen biopsies were done for new findings on mammogram or ultrasound. Three of these (1.9%) yielded a cancer diagnosis (one at 6 months, one at 1 year, and one at 2 years follow-up). No new lesions were identified on follow-up by clinical breast examination alone. Increased surveillance following a benign breast biopsy is necessary because of the increased need for subsequent biopsy or risk of cancer development. This should include imaging (mammography or ultrasound) and a clinical breast examination 6 months, 1 year, and 2 years after a benign breast biopsy. The final assessment includes the BI-RADS 0 to 6 categorization. A category assessment of BI-RADS 0 refers to an incomplete evaluation with further imaging required including additional mammographic views including spot compression or magnification and or ultrasound. BI-RADS 1 refers to a negative examination, meaning that there are no masses, suspicious calcifications or areas of architectural distortion. There can be no description of a finding in the report if it is categorized as a BI-RADS 1.  BI-RADS 2 is consistent with benign findings. Benign findings include secretory calcifications, simple cysts, fat-containing lesions, calcified fibroadenomas, implants and intramammary lymph nodes. BI-RADS 3 is probably benign and should have shortened interval follow-up to determine stability. The risk of malignancy is below 2%. There are very strict classifications to qualify a finding in the BI-RADS 3 category: a non-palpable, circumscribed mass on a baseline mammogram; a focal asymmetry, which becomes less dense on spot compression images, or a solitary group of punctate calcifications. Any findings other than this cannot be placed in the category 3. BI-RADS 4 is a suspicious abnormality, which can represent the chance of being malignant (in percent). The BI-RADS category 4 is subdivided into a, b, and c. The subcategory of (a) has a low probability of malignancy with a 2% to 10% chance of malignancy. The subcategory of (b) has an intermediate change of malignancy ranging from 10% to 50%. The subcategory of (c) has a high probability of malignancy ranging from 50% to 95%. BI-RADS 5 is highly suggestive of malignancy more than 95%. If something is placed in this classification and the pathology comes back as benign, the recommendation is still surgical consultation, because the pathology is discordant with the radiographic findings. The last category that was recently added is the BI-RADS 6, used for pathology proven malignancy. The accidental harm from screening mammography has been underestimated. Women who have mammograms end up with increased surgeries, chemotherapy, radiotherapy and other potentially procedures resulting from the over-detection of harmless lumps. Many women will experience important psychological distress for many months because of false positive findings. Half of suspicious findings will not become dangerous or will disappear over time. Consequently, the value of routine mammography in women at low or average risk is controversial. With unnecessary treatment of ten women for every one woman whose life was prolonged, the authors concluded that routine mammography may do more harm than good. If 1,000 women in their 50s are screened every year for ten years, the following outcomes are considered typical in the developed world: To provide information and recommendations to facilitate decision-making when a mammographic abnormality is detected by screening. References identified by use of MEDLINE, AIDSLINE, CANCERLIT and reference lists of review articles to December 1996. Where experimental evidence is lacking, recommendations are based on expert opinion. The evidence is graded accordingly in \"levels\" (page S2). Exclusion or confirmation of the presence of cancer with minimum intervention and delay. When an abnormality is detected on screening mammography, clinical evaluation and thorough radiologic work-up are needed to determine its significance. Clinical evaluation should include a history and a thorough examination of the breast, axilla and supraclavicular areas. In the radiologic work-up, diagnostic mammograms should be obtained with additional views, spot compression and magnification views as appropriate. Current mammograms should be compared with previous mammograms whenever possible. The mammographic report should include a precise description of the abnormal features visualized and an estimate of the level of suspicion of cancer they imply. Whenever there is any doubt in the interpretation of mammograms, the interpretation of 2 experienced readers should be obtained. (The following radiologic classification into 4 categories is suggested: 1--benign, not due to cancer; 2--low risk, probability of cancer under 2%; 3--intermediate risk, probability of cancer 2% to 10%; 4--high risk, probability of cancer over 10%.) Ultrasonography can be used to clarify the nature of noncalcified nodular lesions. Management decisions require close communication between the woman and her physicians. Throughout, a clinician in charge should be identified who will coordinate and transmit all decisions. Management will depend on the estimated level of risk Category 1 abnormalities require no further investigation. Category 2 abnormalities may be followed up by periodic mammographic and clinical examinations. Follow-up examination of category 2 abnormalities should be carried out at approximately 6 and 12 months. If the abnormality is stable, examination should be repeated annually for 2 to 3 years thereafter. The rationale of follow-up should be explained, and women should be made aware that it is not possible to provide complete assurance that an abnormality is benign. Category 3 abnormalities usually require image-guided fine-needle or core biopsy. Every image-guided needle biopsy should be accompanied by a full report. Category 4 abnormalities should usually be excised. This may be preceded by image-guided needle biopsy. When surgical biopsy is carried out, the margins of the resected specimen must be free of tumour. The intact pathology specimen should be examined radiographically to confirm that all mammographic abnormalities have been removed. The patient should be kept fully informed as to the reason for each test and the meaning of its results. The process, from initial detection of the mammographic abnormality to the final management decision, should be completed as rapidly as possible. Guidelines were reviewed and revised by the Writing Committee, expert primary reviewers, secondary reviewers selected from all regions of Canada and by the Steering Committee. The final document reflects a consensus of all these contributors. From 1976 to 1978 11, 197 women were examined clinically and mammographically. Biopsy material from 1,673 breasts were examined microscopically. In 536 cases, or almost every third case (32%), a carcinoma of the breast was detected. The cancer was bilateral in 19 cases and the total number of women was therefore 517. A clinically occult tumour was only found in 7.7% (40 of 517) of the cases. 5% of these patients were high risk patients and 2.7% preventive examinations. 5 women with occult carcinoma of the breast were under age 40 and 14 under age 50. Benign changes of the glandular tissue were found in 59.5% of the cases. Marked proliferative changes were found in 4.6% of the cases and carcinoma in situ was found in 3.8% of the patients. In the age group 45--54 benign and proliferative changes of the parenchyma occured almost twice as often as cancers. The ratio between benign and malignant findings was 1:1 in the age group 55--59 and was less than 1:2 in the age group over 70. A sophistication of the mammograhic technique must be obtained. A thorough microscopic examination of tissue from subcutaneous mastectomies and tissue obtained at the time of reduction mammoplasties showed occasionally unexpected malignant tissue in an unexpected location. Especially these cases are suitable for later comparison to the mammographies. We examined a 34-year-old premenopausal woman who had noticed a left-breast lump a month previously. She had no past history of malignancies but had a family history of breast and ovarian cancers. Her mother had suffered from ovarian cancer when aged 47 years and had died of the disease at age 52. The younger two of the patient's four aunts had developed breast cancer when they were 37 and 48 years old. A physical examination showed an ill-defined mass, 1.5 cm in diameter, located in the upper outer quadrant of the patient's left breast. Mammography revealed diffuse microcalcification in both breasts but ultrasonography revealed an irregular tumorous lesion only in the left breast. Aspiration breast cytology revealed adenocarcinoma of the left breast. Modified radical mastectomy of the left breast and excision of a biopsy specimen from the right breast were carried out simultaneously. Histopathologically the left-breast tumor was an atypical medullary carcinoma with cartilaginous metaplasia, of histological grade 3, and the right-breast specimen showed fibrocystic changes with atypical ductal hyperplasia. Estrogen receptors were positive, but progesterone receptor was not detected on the tumor cells, which were immunopositive for nuclear p53 although c-erbB-2 overexpression was not observed. A nonsense germline mutation of the BRCA1 gene (exon5) was detected. The patient has been well since the operation (10 months). These findings may provide useful information about the carcinogenesis and biological behavior of BRCA1-associated breast cancers. In this study, we aimed to determine the need for biopsy in patients referred from other clinics for the performance of biopsy with the suspicion of breast cancer. 112 patients were included in the study. It was decided that their biopsies be performed following examinations in other clinics and they presented to the breast radiology unit of our hospital for a second opinion. The demographic characteristics, diagnostic studies completed in the other centers, properties of lesions, decision made as a result of examinations and BI-RADS (Breast Imaging Reporting and Data Systems) categorizations were recorded on the registration forms of the study patients. In addition, the quality of examinations, reasons of repeat tests, additional tests features and the last decision of our clinic were documented. The obtained data were analyzed in terms of re-examination, additional tests and change in the biopsy decision. Changes in the biopsy decisions for patients were specifically inquired. The biopsy decisions were cancelled in our breast radiology unit for 63 out of 112 patients (56.3%) whose biopsy decisions were made at an external institute. For 42 patients, examinations made by the other clinics were deemed adequate, yet there was no need for biopsy in 22 of them. The biopsy decisions were cancelled for 27 out of 47 patients (57.4%) with repeat examination and 18 out of 28 patients (64.3%) with additional tests because of the insufficient test quality. Incorrect, inadequate breast screening and false positivity were higher at inexperienced institutes. The patient was a 41-year-old, premenopausal woman with a chief complaint of well-circumscribed palpable, right breast mass without nipple discharge. Although she noticed the lump 3 months previously, the size of the tumor (1.1 × 0.9 cm(2)) had been stable. The patient's mother suffered from gastric cancer. Her previous history of the triple different malignancies was as follows: (1) left osteosarcoma [amputation of left lower leg at 15 years old (y/o)]. After the operation, she was treated with various kinds of anticancer drugs including a total of 45 g ifosphamide and 342 g methotrexate; (2) tongue cancer (right radical neck resection; 23 y/o); and (3) thyroid cancer (right lobectomy; 40 y/o). There was no evidence of recurrence of these malignancies at the present consultation. At the time of tongue cancer operation, chromosome abnormality was investigated, but the results were normal. Physical examination showed a well-delimited, elastic-firm, mobile tumor in the central outer right breast. Regional lymph nodes were not palpable. Mammography showed a focal asymmetry in the right upper breast on the mediolateral oblique view. Ultrasonography revealed a hypoechoic mass with irregular margins. Distant metastases could not be detected by whole-body computed tomography scan. The histology of the Mammotome(®) (vacuum-assisted core needle biopsy) specimen revealed that this tumor was low-grade ductal carcinoma in situ (DCIS). She underwent breast-conserving surgery with sentinel lymph node biopsy. On permanent histopathological examination, the diagnosis of the tumor was intracystic papilloma with low-grade DCIS. Surgical margin was negative, and sentinel lymph node metastases could not be observed. Estrogen and progesterone receptor (ER/PR) were strongly positive, but human epidermal growth factor receptor-2 (HER-2) overexpression was not tested because the lesion was DCIS. She has received no adjuvant therapy and is currently disease free 3 months after surgery. BI-RADS Assessment Categories are: 0: Incomplete 1: Negative 2: Benign 3: Probably benign 4: Suspicious 5: Highly suggestive of malignancy 6: Known biopsy – proven malignancy An incomplete (BI-RADS 0) classification warrants either an effort to ascertain prior imaging for comparison or to call the patient back for additional views and/or higher quality films. A BI-RADS classification of 4 or 5 warrants biopsy to further evaluate the offending lesion. Some experts believe that the single BI-RADS 4 classification does not adequately communicate the risk of cancer to doctors and recommend a subclassification scheme: 4A: low suspicion for malignancy, about > 2% to ≤ 10% likelihood of malignancy 4B: intermediate suspicion of malignancy, about > 10% to ≤ 50% likelihood of malignancy 4C: moderate concern, but not classic for malignancy, about > 50% to < 95% likelihood of malignancy The aim of this study was to determine the features that make interval cancers apparent on the preceding screening mammogram and determine whether changes in the ways of performing the interval cancer review will affect the true interval cancer rate. This study was approved by the clinical governance committee. Mammograms of women diagnosed with an interval cancer were included in the study if they had been allocated to either the \"suspicious signs\" group or \"subtle signs\" group, during the historic interval cancer review. Three radiologists, individually and blinded to the site of interval cancer, reviewed the mammograms and documented the presence, site, characteristics and classification of any abnormality. Findings were compared with the appearances of the abnormality at the site of subsequent cancer development by a different breast radiologist. The chi-squared test was used in the analysis of the results, seeking associations between recall concordance and cancer mammographic or histological characteristics. 111/590 interval cancers fulfilled the study inclusion criteria. In 17% of the cases none of the readers identified the relevant abnormality on the screening mammogram. 1/3 readers identified the relevant lesion in 22% of the cases, 2/3 readers in 28% of cases and all 3 readers in 33% of cases. The commonest unanimously recalled abnormality was microcalcification and the most challenging mammographic abnormality to detect was asymmetric density. We did not find any statistically significant association between recall concordance and time to interval cancer, position of lesion in the breast, breast density or cancer grade. Even the simple step of performing an independent blinded review of interval cancers reduces the rate of interval cancers classified as missed by up to 39%. The triple test score (TTS) is a diagnostic tool for examining potentially cancerous breasts. Diagnostic accuracy of the triple test score is nearly 100%. Scoring includes using the procedures of physical examination, mammography and needle biopsy. If the results of a TTS are greater than five, an excisional biopsy is indicated. Scoring To obtain the triple test score, a number from 1 through 3 is assigned to each one of the procedures. A score of 1 is assigned to a benign test result, 2 applies to a suspicious test result, and 3 applies to a malignant result. The sum of the scores of all three procedures is the triple test score. A score of 3 to 4 is most likely benign, whereas a score of greater than 6 is possibly malignant. References Breast cancer New evaluation of breast ultrasound based upon review of new literature comparing ultrasound and mammography. Description and discussion of the published trials regarding breast imaging methods. Breast ultrasound is the preferable method in the case of a symptomatic patient (after clinical examination). In the case of a patient without symptoms (screening), breast ultrasound is ascribed a higher sensitivity for detecting breast cancer in women with dense breast tissue, women under the age of 50 and high-risk women. Mammographically occult cancers can be detected by sonography in 10 to 40 % of the cases depending on the patient's breast density and age. The mean size of cancers detected only by ultrasound is not significantly different to that only detected by mammography. The prevalence of breast cancers detected by ultrasound is approximately equal to the one detected by mammography, regarding the total number of examined patients. Breast ultrasound should be the preferred imaging procedure in the case of a palpable lump, leading to a definitive diagnosis itself or with an additional consecutive core needle biopsy. For women without symptoms, breast sonography should be mandatory and complementary to mammography in the case of breast density grade II (BI-RADS) or more. Application of breast ultrasound as a primary method or an alternative to mammography has not yet been evaluated sufficiently. It seems advisable in the case of women with dense breast tissue grade III and IV, women under the age of 50 and high-risk women. The implementation of breast ultrasound in this manner has to be checked by future trials. The consequences of a false-positive screening test result also need to be considered. For example, when 1000 screening mammograms are taken, only 2 to 4 new cases of cancer will be identified; this number is slightly higher (6 to 10 prevalent cancers per 1000 mammograms) for initial screen-ing mammograms.106 However, as many as 10% of screening mammograms may be potentially suggestive of an abnormal-ity, which requires further imaging (i.e., a 10% recall rate). Of those women with abnormal mammogram findings, only 5% to 10% will be determined to have a breast cancer. Among women for whom biopsy specimen is recommended, 25% to 40% will have a breast cancer. A false-positive screening result is likely to induce significant emotional distress in patients, leads to unnecessary biopsy specimens, and has cost implications for the health care system.American Cancer Society guidelines for the early detec-tion of cancer are listed in Table 10-9.96 These guidelines are updated periodically to To retrospectively correlate high-risk proliferative breast lesions (radial scar, atypical lobular hyperplasia, lobular carcinoma in situ and papillary lesions) diagnosed on core biopsy with the definitive histopathological diagnosis obtained after surgical excision or with the follow-up, in order to assess the role of core biopsy in such lesions. To discuss the management of the patient after a core biopsy diagnosis of high-risk proliferative breast lesion. We evaluated 74 out of 1776 core biopsies consecutively performed on 67 patients. The histopathologic findings were as follows: 11 radial scars (RS), 3 atypical lobular hyperplasias (ALH), 3 lobular carcinomas in situ (LCIS), 57 benign papillary lesions. All patients underwent bilateral mammography, whole-breast ultrasound with a linear-array broadband transducer, and core biopsy with a 14 Gauge needle and a mean number of samples of 5 (range 4-7). Sixty-two of 67 patients, for a total of 69/74 lesions, underwent surgical biopsy despite benign histopathologic findings, mostly because of highly suspicious imaging for malignancy (BIRADS 4-5), whereas 5 patients refused surgery and have been followed up for a least 18 months and are still being followed up (2 with RS, 1 with ADH and 2 with papillary lesions). Among the core biopsied lesions with a diagnosis of RS (n = 11) pathology revealed one ductal carcinoma in situ (DCIS) (this case was characterized by granular microcalcifications on mammography and by a mass with irregular margins on ultrasound). Also in the group of ADH (n = 3) pathology revealed one DCIS (lesion not visible on mammography but depicted as a suspicious mass on US). In the group of LCIS (n = 3) pathologists found an invasive lobular carcinoma (ILC). Among the benign papillary lesions (n = 57) histopathologic analysis of the surgical specimen revealed 7 malignant lesions (4 papillary carcinomas and 3 DCIS), whose mammographic and ultrasound findings were indistinguishable from benign lesions. Altogether there were 10 false negative results (underestimation) out of 74 core biopsies with a diagnosis of high-risk proliferative breast lesions. The high rate of histological underestimation after core biopsy (10/74) (13.5%) demands a very careful management of patents with a core biopsy diagnosis of high-risk proliferative breast lesions, especially in the case of RS, lobular neoplasia and papillary lesions. However, the high imaging suspicion for malignancy prompts surgery. It is possible to assume that, when there is a low imaging suspicion for malignancy, when enough tissue has been sampled for pathology and no atypia is found within the lesions, surgery is not mandatory but a very careful follow-up is recommended. We must underline that there is no agreement regarding the quantity of tissue to sample. Vacuum-assisted biopsy may lead to better results, although there is as yet no proof that it can actually replace surgery in this group of lesions, since it seems only to reduce but not abolish the histological underestimation. The History of mammography began in 1913, when a Berliner surgeon, A. Salomon realized a roentgeno-histological study on 3,000 mastectomies. This work is the basis of mammography. Until 1938, few articles were published but were of little help to mammography. From 1947 to 1970, the second period brought the results of roentgenologic and clinical correlation. R. Leborgne was the first accountable for the wide development of this method. Since 1951, many American and European radiologists brought their contribution. Ch. Gros is the best known. He gave this technique an acknowledgment throughout the world for the diagnosis of breast diseases. Since 1970, the third period emphasizes the value of mammography as a technique for detection of breast cancer. Some \"Screening working groups\" are being set up. The problem is mainly economical. Often women are quite distressed to be called back for a diagnostic mammogram. Most of these recalls will be false positive results. Of every 1,000 U.S. women who are screened, about 7% will be called back for a diagnostic session (although some studies estimate the number to be closer to 10% to 15%). About 10 of these individuals will be referred for a biopsy; the remaining 60 cases are found to be of benign cause. Of the 10 referred for biopsy, about 3.5 will have cancer and 6.5 will not. Of the 3.5 who have cancer, about 2 will have an early stage cancer that will be cured after treatment. If suspicious signs are identified in the image, then the woman is usually recalled for a second mammogram, sometimes after waiting six months to see whether the spot is growing, or a biopsy of the breast. Most of these will prove to be false positives, resulting in sometimes debilitating anxiety over nothing. Most women recalled will undergo additional imaging only, without any further intervention. Recall rates are higher in the U.S. than in the UK. Effectiveness On balance, screening mammography in older women increases medical treatment and saves a small number of lives. Usually, it has no effect on the outcome of any breast cancer that it detects. Screening targeted towards women with above-average risk produces more benefit than screening of women at average or low risk for breast cancer.\nHere is the question:\nONCOLOGY: A 40-year-old woman consults because she has noticed a lump in the superoexternal quadrant of the right breast for the past month. She provides a mammography report describing a BIRADS 3 lesion. What is the best course of action?\nHere are the potential choices:\n1. Reassure him, since an imaging test has already been done and malignancy has been ruled out.\n2. This classification probably implies surgery since the probability of cancer is greater than 10%. He explains it to you and refers you preferentially to the Breast Unit.\n3. This is a probably benign finding, since there is less than a 2% chance of cancer. He explains that it requires follow-up every 6-12 months until 24 months or a biopsy.\n4. The findings are of low suspicion of cancer (between 2 and 10 %) but a biopsy is necessary.\nThe correct answer is: ", "gold_answer": "3 This is a probably benign finding, since there is less than a 2% chance of cancer. He explains that it requires follow-up every 6-12 months until 24 months or a biopsy.", "prediction": "You are a helpful medical expert, and your task is to answer a multi-choice medical question using the relevant documents. Please choose the answer from the provided options. Your responses will be used for research purposes only, so please have a definite answer.\nHere are the relevant documents:\n Because approximately 1 in 10 women with a breast lump or abnormal mammography result will have breast cancer, a series of decisions must be taken by a primary care practitioner to exclude or establish a diagnosis of breast cancer among these women. To determine the most accurate and least invasive means to evaluate an abnormal mammography result and a palpable breast abnormality. MEDLINE search (January 1966 to March 2003) for articles and reviews describing the accuracy of clinical examination, biopsy procedures, and radiographic examination for patients with abnormal mammography results or palpable breast abnormalities. The authors reviewed abstracts and selected articles that provided relevant primary data. Studies were included if 1) mammography, fine-needle aspiration biopsy, or core-needle biopsy was performed before a definitive diagnosis was obtained; 2) the study sample included 100 or more women; and 3) breast cancer status was determined from histopathology review of excisional biopsy specimens, from linkage with a state cancer registry or the Surveillance, Epidemiology, and End Results program, or from clinical follow-up of 95% or more of the study sample. One investigator abstracted results. Methods were evaluated for major potential biases, but methodologic scoring was not performed. Likelihood ratios for first screening mammography were 0.1 for the Breast Imaging Reporting and Data System (BI-RADS) assessment category \"negative or benign finding,\" 1.2 for \"probably benign finding,\" 7 for \"need additional imaging evaluation,\" 125 for \"suspicious abnormality,\" and 2200 for \"highly suggestive of malignancy.\" For fine-needle aspiration biopsy of a palpable lump performed by formally trained physicians, the likelihood ratio was infinity for an assessment of \"malignant,\" 2.6 for \"atypical/suspicious,\" and 0.02 for \"benign.\" When diagnostic mammography was used to evaluate a palpable lump or nonpalpable breast abnormality, the positive likelihood ratios were 5.6 and 9.4, and the negative likelihood ratios were 0.15 and 0.19, respectively. Women whose screening mammography results are interpreted as \"suspicious abnormality\" or \"highly suggestive of malignancy\" have a high risk for breast cancer and should undergo core-needle biopsy or needle localization with surgical biopsy. Women whose screening mammography results are interpreted as \"need additional imaging evaluation\" have a moderate risk for breast cancer and should undergo diagnostic mammography or ultrasonography to decide whether a nonpalpable breast lesion should be biopsied. Women whose screening mammography results are interpreted as \"probably benign finding\" have a low risk for breast cancer and can undergo follow-up mammography in 6 months. Either fine-needle aspiration biopsy or ultrasonography is recommended as the first diagnostic test of a palpable breast abnormality to distinguish simple cysts from solid masses. Fine-needle aspiration biopsy also allows characterization of a solid mass. Diagnostic mammography does not help determine whether a palpable breast mass should be biopsied and should not affect the decision to perform a biopsy. The importance of these missed cancers is not clear, particularly if the woman is getting yearly mammograms. Research on a closely related situation has shown that small cancers that are not acted upon immediately, but are observed over periods of several years, will have good outcomes. A group of 3,184 women had mammograms that were formally classified as \"probably benign\". This classification is for patients who are not clearly normal but have some area of minor concern. This results not in the patient being biopsied, but rather in having early follow up mammography every six months for three years to determine whether there has been any change in status. Of these 3,184 women, 17 (0.5%) did have cancers. Most importantly, when the diagnosis was finally made, they were all still stage 0 or 1, the earliest stages. Five years after treatment, none of these 17 women had evidence of re-occurrence. Thus, small early cancers, even though not acted on immediately, were still reliably Mammography Mammography is a common screening method, since it is relatively fast and widely available in developed countries. Mammography is a type of radiography used on the breasts. It is typically used for two purposes: to aid in the diagnosis of a woman who is experiencing symptoms or has been called back for follow-up views (called diagnostic mammography), and for medical screening of apparently healthy women (called screening mammography). Mammography is not very useful in finding breast tumors in dense breast tissue characteristic of women under 40 years. In women over 50 without dense breasts, breast cancers detected by screening mammography are usually smaller and less aggressive than those detected by patients or doctors as a breast lump. This is because the most aggressive breast cancers are found in dense breast tissue, which mammograms perform poorly on. Histologically proven benign breast disease increases a woman's relative risk for subsequent cancer development. Yet follow-up guidelines for mammogram and clinical breast examination after a benign breast biopsy are lacking. Our objective was to determine if increased surveillance is indicated following a benign breast biopsy. Following institutional review board approval, a retrospective database review was conducted of prospectively gathered patients who had a benign breast biopsy (core or excisional) for an abnormality detected on mammogram, ultrasound, or clinical breast examination. Follow-up, for all subjects, was a clinical breast examination and mammogram or ultrasound at 6 months, 1 year, and 2 years after benign breast biopsy by a breast surgeon. End points were the need for additional biopsies or cancer detection. Statistical analysis was performed using chi-squared analysis. From January 2000 to July 2003, 156 patients age 18-86 years had a benign breast biopsy. During the 2 year follow-up, 20 patients (13%) required a subsequent biopsy. No significant difference was observed in mean age, race, menarche, menopause, parity, age at first live birth, use of oral contraceptives, history of prior biopsy, or the pathology of the initial lesion between those who needed a subsequent biopsy and those who did not. Seven excisional biopsies were performed (one at 6 months, four at 1 year, and two at 2 years follow-up) for growth of the benign breast biopsy lesion, and pathology remained concordant with the original diagnosis. Thirteen biopsies were done for new findings on mammogram or ultrasound. Three of these (1.9%) yielded a cancer diagnosis (one at 6 months, one at 1 year, and one at 2 years follow-up). No new lesions were identified on follow-up by clinical breast examination alone. Increased surveillance following a benign breast biopsy is necessary because of the increased need for subsequent biopsy or risk of cancer development. This should include imaging (mammography or ultrasound) and a clinical breast examination 6 months, 1 year, and 2 years after a benign breast biopsy. The final assessment includes the BI-RADS 0 to 6 categorization. A category assessment of BI-RADS 0 refers to an incomplete evaluation with further imaging required including additional mammographic views including spot compression or magnification and or ultrasound. BI-RADS 1 refers to a negative examination, meaning that there are no masses, suspicious calcifications or areas of architectural distortion. There can be no description of a finding in the report if it is categorized as a BI-RADS 1.  BI-RADS 2 is consistent with benign findings. Benign findings include secretory calcifications, simple cysts, fat-containing lesions, calcified fibroadenomas, implants and intramammary lymph nodes. BI-RADS 3 is probably benign and should have shortened interval follow-up to determine stability. The risk of malignancy is below 2%. There are very strict classifications to qualify a finding in the BI-RADS 3 category: a non-palpable, circumscribed mass on a baseline mammogram; a focal asymmetry, which becomes less dense on spot compression images, or a solitary group of punctate calcifications. Any findings other than this cannot be placed in the category 3. BI-RADS 4 is a suspicious abnormality, which can represent the chance of being malignant (in percent). The BI-RADS category 4 is subdivided into a, b, and c. The subcategory of (a) has a low probability of malignancy with a 2% to 10% chance of malignancy. The subcategory of (b) has an intermediate change of malignancy ranging from 10% to 50%. The subcategory of (c) has a high probability of malignancy ranging from 50% to 95%. BI-RADS 5 is highly suggestive of malignancy more than 95%. If something is placed in this classification and the pathology comes back as benign, the recommendation is still surgical consultation, because the pathology is discordant with the radiographic findings. The last category that was recently added is the BI-RADS 6, used for pathology proven malignancy. The accidental harm from screening mammography has been underestimated. Women who have mammograms end up with increased surgeries, chemotherapy, radiotherapy and other potentially procedures resulting from the over-detection of harmless lumps. Many women will experience important psychological distress for many months because of false positive findings. Half of suspicious findings will not become dangerous or will disappear over time. Consequently, the value of routine mammography in women at low or average risk is controversial. With unnecessary treatment of ten women for every one woman whose life was prolonged, the authors concluded that routine mammography may do more harm than good. If 1,000 women in their 50s are screened every year for ten years, the following outcomes are considered typical in the developed world: To provide information and recommendations to facilitate decision-making when a mammographic abnormality is detected by screening. References identified by use of MEDLINE, AIDSLINE, CANCERLIT and reference lists of review articles to December 1996. Where experimental evidence is lacking, recommendations are based on expert opinion. The evidence is graded accordingly in \"levels\" (page S2). Exclusion or confirmation of the presence of cancer with minimum intervention and delay. When an abnormality is detected on screening mammography, clinical evaluation and thorough radiologic work-up are needed to determine its significance. Clinical evaluation should include a history and a thorough examination of the breast, axilla and supraclavicular areas. In the radiologic work-up, diagnostic mammograms should be obtained with additional views, spot compression and magnification views as appropriate. Current mammograms should be compared with previous mammograms whenever possible. The mammographic report should include a precise description of the abnormal features visualized and an estimate of the level of suspicion of cancer they imply. Whenever there is any doubt in the interpretation of mammograms, the interpretation of 2 experienced readers should be obtained. (The following radiologic classification into 4 categories is suggested: 1--benign, not due to cancer; 2--low risk, probability of cancer under 2%; 3--intermediate risk, probability of cancer 2% to 10%; 4--high risk, probability of cancer over 10%.) Ultrasonography can be used to clarify the nature of noncalcified nodular lesions. Management decisions require close communication between the woman and her physicians. Throughout, a clinician in charge should be identified who will coordinate and transmit all decisions. Management will depend on the estimated level of risk Category 1 abnormalities require no further investigation. Category 2 abnormalities may be followed up by periodic mammographic and clinical examinations. Follow-up examination of category 2 abnormalities should be carried out at approximately 6 and 12 months. If the abnormality is stable, examination should be repeated annually for 2 to 3 years thereafter. The rationale of follow-up should be explained, and women should be made aware that it is not possible to provide complete assurance that an abnormality is benign. Category 3 abnormalities usually require image-guided fine-needle or core biopsy. Every image-guided needle biopsy should be accompanied by a full report. Category 4 abnormalities should usually be excised. This may be preceded by image-guided needle biopsy. When surgical biopsy is carried out, the margins of the resected specimen must be free of tumour. The intact pathology specimen should be examined radiographically to confirm that all mammographic abnormalities have been removed. The patient should be kept fully informed as to the reason for each test and the meaning of its results. The process, from initial detection of the mammographic abnormality to the final management decision, should be completed as rapidly as possible. Guidelines were reviewed and revised by the Writing Committee, expert primary reviewers, secondary reviewers selected from all regions of Canada and by the Steering Committee. The final document reflects a consensus of all these contributors. From 1976 to 1978 11, 197 women were examined clinically and mammographically. Biopsy material from 1,673 breasts were examined microscopically. In 536 cases, or almost every third case (32%), a carcinoma of the breast was detected. The cancer was bilateral in 19 cases and the total number of women was therefore 517. A clinically occult tumour was only found in 7.7% (40 of 517) of the cases. 5% of these patients were high risk patients and 2.7% preventive examinations. 5 women with occult carcinoma of the breast were under age 40 and 14 under age 50. Benign changes of the glandular tissue were found in 59.5% of the cases. Marked proliferative changes were found in 4.6% of the cases and carcinoma in situ was found in 3.8% of the patients. In the age group 45--54 benign and proliferative changes of the parenchyma occured almost twice as often as cancers. The ratio between benign and malignant findings was 1:1 in the age group 55--59 and was less than 1:2 in the age group over 70. A sophistication of the mammograhic technique must be obtained. A thorough microscopic examination of tissue from subcutaneous mastectomies and tissue obtained at the time of reduction mammoplasties showed occasionally unexpected malignant tissue in an unexpected location. Especially these cases are suitable for later comparison to the mammographies. We examined a 34-year-old premenopausal woman who had noticed a left-breast lump a month previously. She had no past history of malignancies but had a family history of breast and ovarian cancers. Her mother had suffered from ovarian cancer when aged 47 years and had died of the disease at age 52. The younger two of the patient's four aunts had developed breast cancer when they were 37 and 48 years old. A physical examination showed an ill-defined mass, 1.5 cm in diameter, located in the upper outer quadrant of the patient's left breast. Mammography revealed diffuse microcalcification in both breasts but ultrasonography revealed an irregular tumorous lesion only in the left breast. Aspiration breast cytology revealed adenocarcinoma of the left breast. Modified radical mastectomy of the left breast and excision of a biopsy specimen from the right breast were carried out simultaneously. Histopathologically the left-breast tumor was an atypical medullary carcinoma with cartilaginous metaplasia, of histological grade 3, and the right-breast specimen showed fibrocystic changes with atypical ductal hyperplasia. Estrogen receptors were positive, but progesterone receptor was not detected on the tumor cells, which were immunopositive for nuclear p53 although c-erbB-2 overexpression was not observed. A nonsense germline mutation of the BRCA1 gene (exon5) was detected. The patient has been well since the operation (10 months). These findings may provide useful information about the carcinogenesis and biological behavior of BRCA1-associated breast cancers. In this study, we aimed to determine the need for biopsy in patients referred from other clinics for the performance of biopsy with the suspicion of breast cancer. 112 patients were included in the study. It was decided that their biopsies be performed following examinations in other clinics and they presented to the breast radiology unit of our hospital for a second opinion. The demographic characteristics, diagnostic studies completed in the other centers, properties of lesions, decision made as a result of examinations and BI-RADS (Breast Imaging Reporting and Data Systems) categorizations were recorded on the registration forms of the study patients. In addition, the quality of examinations, reasons of repeat tests, additional tests features and the last decision of our clinic were documented. The obtained data were analyzed in terms of re-examination, additional tests and change in the biopsy decision. Changes in the biopsy decisions for patients were specifically inquired. The biopsy decisions were cancelled in our breast radiology unit for 63 out of 112 patients (56.3%) whose biopsy decisions were made at an external institute. For 42 patients, examinations made by the other clinics were deemed adequate, yet there was no need for biopsy in 22 of them. The biopsy decisions were cancelled for 27 out of 47 patients (57.4%) with repeat examination and 18 out of 28 patients (64.3%) with additional tests because of the insufficient test quality. Incorrect, inadequate breast screening and false positivity were higher at inexperienced institutes. The patient was a 41-year-old, premenopausal woman with a chief complaint of well-circumscribed palpable, right breast mass without nipple discharge. Although she noticed the lump 3 months previously, the size of the tumor (1.1 × 0.9 cm(2)) had been stable. The patient's mother suffered from gastric cancer. Her previous history of the triple different malignancies was as follows: (1) left osteosarcoma [amputation of left lower leg at 15 years old (y/o)]. After the operation, she was treated with various kinds of anticancer drugs including a total of 45 g ifosphamide and 342 g methotrexate; (2) tongue cancer (right radical neck resection; 23 y/o); and (3) thyroid cancer (right lobectomy; 40 y/o). There was no evidence of recurrence of these malignancies at the present consultation. At the time of tongue cancer operation, chromosome abnormality was investigated, but the results were normal. Physical examination showed a well-delimited, elastic-firm, mobile tumor in the central outer right breast. Regional lymph nodes were not palpable. Mammography showed a focal asymmetry in the right upper breast on the mediolateral oblique view. Ultrasonography revealed a hypoechoic mass with irregular margins. Distant metastases could not be detected by whole-body computed tomography scan. The histology of the Mammotome(®) (vacuum-assisted core needle biopsy) specimen revealed that this tumor was low-grade ductal carcinoma in situ (DCIS). She underwent breast-conserving surgery with sentinel lymph node biopsy. On permanent histopathological examination, the diagnosis of the tumor was intracystic papilloma with low-grade DCIS. Surgical margin was negative, and sentinel lymph node metastases could not be observed. Estrogen and progesterone receptor (ER/PR) were strongly positive, but human epidermal growth factor receptor-2 (HER-2) overexpression was not tested because the lesion was DCIS. She has received no adjuvant therapy and is currently disease free 3 months after surgery. BI-RADS Assessment Categories are: 0: Incomplete 1: Negative 2: Benign 3: Probably benign 4: Suspicious 5: Highly suggestive of malignancy 6: Known biopsy – proven malignancy An incomplete (BI-RADS 0) classification warrants either an effort to ascertain prior imaging for comparison or to call the patient back for additional views and/or higher quality films. A BI-RADS classification of 4 or 5 warrants biopsy to further evaluate the offending lesion. Some experts believe that the single BI-RADS 4 classification does not adequately communicate the risk of cancer to doctors and recommend a subclassification scheme: 4A: low suspicion for malignancy, about > 2% to ≤ 10% likelihood of malignancy 4B: intermediate suspicion of malignancy, about > 10% to ≤ 50% likelihood of malignancy 4C: moderate concern, but not classic for malignancy, about > 50% to < 95% likelihood of malignancy The aim of this study was to determine the features that make interval cancers apparent on the preceding screening mammogram and determine whether changes in the ways of performing the interval cancer review will affect the true interval cancer rate. This study was approved by the clinical governance committee. Mammograms of women diagnosed with an interval cancer were included in the study if they had been allocated to either the \"suspicious signs\" group or \"subtle signs\" group, during the historic interval cancer review. Three radiologists, individually and blinded to the site of interval cancer, reviewed the mammograms and documented the presence, site, characteristics and classification of any abnormality. Findings were compared with the appearances of the abnormality at the site of subsequent cancer development by a different breast radiologist. The chi-squared test was used in the analysis of the results, seeking associations between recall concordance and cancer mammographic or histological characteristics. 111/590 interval cancers fulfilled the study inclusion criteria. In 17% of the cases none of the readers identified the relevant abnormality on the screening mammogram. 1/3 readers identified the relevant lesion in 22% of the cases, 2/3 readers in 28% of cases and all 3 readers in 33% of cases. The commonest unanimously recalled abnormality was microcalcification and the most challenging mammographic abnormality to detect was asymmetric density. We did not find any statistically significant association between recall concordance and time to interval cancer, position of lesion in the breast, breast density or cancer grade. Even the simple step of performing an independent blinded review of interval cancers reduces the rate of interval cancers classified as missed by up to 39%. The triple test score (TTS) is a diagnostic tool for examining potentially cancerous breasts. Diagnostic accuracy of the triple test score is nearly 100%. Scoring includes using the procedures of physical examination, mammography and needle biopsy. If the results of a TTS are greater than five, an excisional biopsy is indicated. Scoring To obtain the triple test score, a number from 1 through 3 is assigned to each one of the procedures. A score of 1 is assigned to a benign test result, 2 applies to a suspicious test result, and 3 applies to a malignant result. The sum of the scores of all three procedures is the triple test score. A score of 3 to 4 is most likely benign, whereas a score of greater than 6 is possibly malignant. References Breast cancer New evaluation of breast ultrasound based upon review of new literature comparing ultrasound and mammography. Description and discussion of the published trials regarding breast imaging methods. Breast ultrasound is the preferable method in the case of a symptomatic patient (after clinical examination). In the case of a patient without symptoms (screening), breast ultrasound is ascribed a higher sensitivity for detecting breast cancer in women with dense breast tissue, women under the age of 50 and high-risk women. Mammographically occult cancers can be detected by sonography in 10 to 40 % of the cases depending on the patient's breast density and age. The mean size of cancers detected only by ultrasound is not significantly different to that only detected by mammography. The prevalence of breast cancers detected by ultrasound is approximately equal to the one detected by mammography, regarding the total number of examined patients. Breast ultrasound should be the preferred imaging procedure in the case of a palpable lump, leading to a definitive diagnosis itself or with an additional consecutive core needle biopsy. For women without symptoms, breast sonography should be mandatory and complementary to mammography in the case of breast density grade II (BI-RADS) or more. Application of breast ultrasound as a primary method or an alternative to mammography has not yet been evaluated sufficiently. It seems advisable in the case of women with dense breast tissue grade III and IV, women under the age of 50 and high-risk women. The implementation of breast ultrasound in this manner has to be checked by future trials. The consequences of a false-positive screening test result also need to be considered. For example, when 1000 screening mammograms are taken, only 2 to 4 new cases of cancer will be identified; this number is slightly higher (6 to 10 prevalent cancers per 1000 mammograms) for initial screen-ing mammograms.106 However, as many as 10% of screening mammograms may be potentially suggestive of an abnormal-ity, which requires further imaging (i.e., a 10% recall rate). Of those women with abnormal mammogram findings, only 5% to 10% will be determined to have a breast cancer. Among women for whom biopsy specimen is recommended, 25% to 40% will have a breast cancer. A false-positive screening result is likely to induce significant emotional distress in patients, leads to unnecessary biopsy specimens, and has cost implications for the health care system.American Cancer Society guidelines for the early detec-tion of cancer are listed in Table 10-9.96 These guidelines are updated periodically to To retrospectively correlate high-risk proliferative breast lesions (radial scar, atypical lobular hyperplasia, lobular carcinoma in situ and papillary lesions) diagnosed on core biopsy with the definitive histopathological diagnosis obtained after surgical excision or with the follow-up, in order to assess the role of core biopsy in such lesions. To discuss the management of the patient after a core biopsy diagnosis of high-risk proliferative breast lesion. We evaluated 74 out of 1776 core biopsies consecutively performed on 67 patients. The histopathologic findings were as follows: 11 radial scars (RS), 3 atypical lobular hyperplasias (ALH), 3 lobular carcinomas in situ (LCIS), 57 benign papillary lesions. All patients underwent bilateral mammography, whole-breast ultrasound with a linear-array broadband transducer, and core biopsy with a 14 Gauge needle and a mean number of samples of 5 (range 4-7). Sixty-two of 67 patients, for a total of 69/74 lesions, underwent surgical biopsy despite benign histopathologic findings, mostly because of highly suspicious imaging for malignancy (BIRADS 4-5), whereas 5 patients refused surgery and have been followed up for a least 18 months and are still being followed up (2 with RS, 1 with ADH and 2 with papillary lesions). Among the core biopsied lesions with a diagnosis of RS (n = 11) pathology revealed one ductal carcinoma in situ (DCIS) (this case was characterized by granular microcalcifications on mammography and by a mass with irregular margins on ultrasound). Also in the group of ADH (n = 3) pathology revealed one DCIS (lesion not visible on mammography but depicted as a suspicious mass on US). In the group of LCIS (n = 3) pathologists found an invasive lobular carcinoma (ILC). Among the benign papillary lesions (n = 57) histopathologic analysis of the surgical specimen revealed 7 malignant lesions (4 papillary carcinomas and 3 DCIS), whose mammographic and ultrasound findings were indistinguishable from benign lesions. Altogether there were 10 false negative results (underestimation) out of 74 core biopsies with a diagnosis of high-risk proliferative breast lesions. The high rate of histological underestimation after core biopsy (10/74) (13.5%) demands a very careful management of patents with a core biopsy diagnosis of high-risk proliferative breast lesions, especially in the case of RS, lobular neoplasia and papillary lesions. However, the high imaging suspicion for malignancy prompts surgery. It is possible to assume that, when there is a low imaging suspicion for malignancy, when enough tissue has been sampled for pathology and no atypia is found within the lesions, surgery is not mandatory but a very careful follow-up is recommended. We must underline that there is no agreement regarding the quantity of tissue to sample. Vacuum-assisted biopsy may lead to better results, although there is as yet no proof that it can actually replace surgery in this group of lesions, since it seems only to reduce but not abolish the histological underestimation. The History of mammography began in 1913, when a Berliner surgeon, A. Salomon realized a roentgeno-histological study on 3,000 mastectomies. This work is the basis of mammography. Until 1938, few articles were published but were of little help to mammography. From 1947 to 1970, the second period brought the results of roentgenologic and clinical correlation. R. Leborgne was the first accountable for the wide development of this method. Since 1951, many American and European radiologists brought their contribution. Ch. Gros is the best known. He gave this technique an acknowledgment throughout the world for the diagnosis of breast diseases. Since 1970, the third period emphasizes the value of mammography as a technique for detection of breast cancer. Some \"Screening working groups\" are being set up. The problem is mainly economical. Often women are quite distressed to be called back for a diagnostic mammogram. Most of these recalls will be false positive results. Of every 1,000 U.S. women who are screened, about 7% will be called back for a diagnostic session (although some studies estimate the number to be closer to 10% to 15%). About 10 of these individuals will be referred for a biopsy; the remaining 60 cases are found to be of benign cause. Of the 10 referred for biopsy, about 3.5 will have cancer and 6.5 will not. Of the 3.5 who have cancer, about 2 will have an early stage cancer that will be cured after treatment. If suspicious signs are identified in the image, then the woman is usually recalled for a second mammogram, sometimes after waiting six months to see whether the spot is growing, or a biopsy of the breast. Most of these will prove to be false positives, resulting in sometimes debilitating anxiety over nothing. Most women recalled will undergo additional imaging only, without any further intervention. Recall rates are higher in the U.S. than in the UK. Effectiveness On balance, screening mammography in older women increases medical treatment and saves a small number of lives. Usually, it has no effect on the outcome of any breast cancer that it detects. Screening targeted towards women with above-average risk produces more benefit than screening of women at average or low risk for breast cancer.\nHere is the question:\nONCOLOGY: A 40-year-old woman consults because she has noticed a lump in the superoexternal quadrant of the right breast for the past month. She provides a mammography report describing a BIRADS 3 lesion. What is the best course of action?\nHere are the potential choices:\n1. Reassure him, since an imaging test has already been done and malignancy has been ruled out.\n2. This classification probably implies surgery since the probability of cancer is greater than 10%. He explains it to you and refers you preferentially to the Breast Unit.\n3. This is a probably benign finding, since there is less than a 2% chance of cancer. He explains that it requires follow-up every 6-12 months until 24 months or a biopsy.\n4. The findings are of low suspicion of cancer (between 2 and 10 %) but a biopsy is necessary.\nThe correct answer is: 3. This is a probably benign finding, since there is less than a 2% chance of cancer. He explains that it requires follow-up every 6-12 months until 24 months or a biopsy." }, { "question": "You are a helpful medical expert, and your task is to answer a multi-choice medical question using the relevant documents. Please choose the answer from the provided options. Your responses will be used for research purposes only, so please have a definite answer.\nHere are the relevant documents:\n Bacteria can cause lung cavities in one of two ways; they can either enter the lung through the trachea (windpipe), or they can enter through the bloodstream as septic pulmonary emboli (infected blood clots). Community-acquired pneumonia is an uncommon cause of lung cavities, but cavitary pneumonia is occasionally seen with Streptococcus pneumoniae or Haemophilus influenzae infection. However, since these two species of bacteria are such common causes of pneumonia, they may cause a significant fraction of all cavitary pneumonias. The most common bacterial causes of lung cavities are Streptococcus species and Klebsiella pneumoniae. Less commonly, the bacteria Staphylococcus aureus, Pseudomonas aeruginosa, Acinetobacter, Escherichia coli, and Legionella can cause cavitation. Nocardia is a bacterium that can cause pulmonary nocardiosis and lung cavities in people who are immunocompromised (have weak immune systems), including organ transplant recipients who are on immunosuppressants, Causative organisms The most common pathogens responsible for NP are Streptococcus pneumonia, Staphylococcus aureus, Klebsiella pneumoniae. Other pathogens which are less likely to cause NP are bacteria like Haemophilus influenzae, Streptococcus anginosus group, Pseudomonas aeruginosa, Mycoplasma pneumoniae, Acinetobacter baumannii, Streptococcus pyogenes, Stenotrophomonas maltophilia, anaerobes like Fusobacterium nucleatum and Bacteroides fragilis; fungi like Aspergillus sp. and Histoplasma capsulatum; viruses like Influenza and Adenovirus. Necrotizing pneumonia (NP), also known as cavitary pneumonia or cavitatory necrosis, is a rare but severe complication of lung parenchymal infection. In necrotizing pneumonia, there is a substantial liquefaction following death of the lung tissue, which may lead to gangrene formation in the lung. In most cases patients with NP have fever, cough and bad breath, and those with more indolent infections have weight loss. Often patients clinically present with acute respiratory failure. The most common pathogens responsible for NP are Streptococcus pneumonia, Staphylococcus aureus, Klebsiella pneumoniae. Diagnosis is usually done by chest imaging, e.g. chest X-ray, CT scan. Among these CT scan is the most sensitive test which shows loss of lung architecture and multiple small thin walled cavities. Often cultures from bronchoalveolar lavage and blood may be done for identification of the causative organism(s). Causes In some studies, the bacteria found in patients with HCAP were more similar to HAP than to CAP; compared to CAP, they could have higher rates of Staphylococcus aureus (S. aureus) and Pseudomonas aeruginosa, and less Streptococcus pneumoniae and Haemophilus influenzae. In European and Asian studies, the etiology of HCAP was similar to that of CAP, and rates of multi drug resistant pathogens such as Staphylococcus aureus and Pseudomonas aeruginosa were not as high as seen in North American studies. It is well known that nursing home residents have high rates of colonization with MRSA. However, not all studies have found high rates of S. aureus and gram-negative bacteria. One factor responsible for these differences is the reliance on sputum samples and the strictness of the criteria to discriminate A. fumigatus (the most common species) infections are primary pulmonary infections and can potentially become a rapidly necrotizing pneumonia with a potential to disseminate. The organism can be differentiated from other common mold infections based on the fact that it takes on a mold form both in the environment and in the host (unlike Candida albicans which is a dimorphic mold in the environment and a yeast in the body). Aspergillosis Aspergillosis is the group of diseases caused by Aspergillus. The most common species among paranasal sinus infections associated with aspergillosis is A. fumigatus. The symptoms include fever, cough, chest pain, or breathlessness, which also occur in many other illnesses, so diagnosis can be difficult. Usually, only patients with already weakened immune systems or who suffer other lung conditions are susceptible. Opportunistic pneumonia People with weakened immune defense, such as HIV/AIDS patients, are highly susceptible to opportunistic infections affecting the lungs. Most common pathogens are Pneumocystis jiroveci, Mycobacterium avium-intracellulare complex, Streptococcus pneumoniae, Haemophilus species. Less frequent pathogens are Cryptococcus neoformans, Histoplasma capsulatum, Coccidioides immitis, cytomegalovirus (CMV), and Toxoplasma gondii. Chemotherapy-induced immunodeficiency may lead to severe lung infections. Pathogens commonly associated with lung infectioins are bacteria (like Pseudomonas aeruginosa, Stenotrophomonas maltophilia, and Nocardia species), viruses (eg, respiratory syncytial virus, parainfluenza virus, influenza virus A and influenza B, and cytomegalovirus), and fungi (eg, Aspergillus, Fusarium, and Mucorales species, and Pneumocystis jirovecii). Bacteria such as S. aureus , P. aeruginosa , Stenotrophomonas maltophilia , and Burkholderia cepacia complex, are the most common pathogens involved in pulmonary infections immediately after solid organ transplantation, especially heart and lung. ESKAPE pathogens ( Enterococcus faecium, Staphylococcus aureus, Klebsiella pneumoniae, Acinetobacter baumannii, Pseudomonas aeruginosa, and Enterobacter species) especially cause pulmonary infections after lung transplant. Due to the immunosuppression, infections from other opportunistic pathogens such as Cytomegalovirus (CMV) can also occur after solid organ transplantation. In the early phase following hematopoietic stem cell transplantation (HSCT), the majority of pulmonary infections are due to bacteria, for example, S. pneumoniae , Klebsiella , Gram-negative bacilli, and S. aureus . Up to three weeks following HSCT, which is the neutropenic phase, fungi, especially Aspergillus spp. are a common cause of infections, while CMV infections can occur up to three months following HSCT. Pneumocystis pneumonia (PCP) is uncommon after HSCT except in the setting of graft-vs-host disease. [2] [3] Following allogeneic cell transplantation, pulmonary infections due to Fusarium species can also occur, which are seen exclusively in the severely immunocompromised. [4] Bacteria have been the most commonly isolated pathogens, although viral and fungal pathogens are potentially found in immunocompromised hosts (patients on chronic immunosuppressed medications, solid organ and bone marrow transplant recipients). In general, the distribution of microbial pathogens varies among institutions, partly because of differences in patient population and local patterns of anti microbial resistance in hospitals and critical care units' Common bacterial pathogens include aerobic GNB, such as Pseudomonas aeruginosa, Acinetobacter baumanii, Klebsiella pneumoniae, Escherichia coli as well as gram-positive organisms such as Staphylococcus aureus. In patients with an early onset pneumonia (within 5 days of hospitalization), they are usually due to anti microbial-sensitive bacteria such as Enterobacter spp, E. coli, Klebsiella spp, Proteus spp, Serratia mare scans, community pathogens such as Streptococcus pneumoniae, Haemophilus influenzae, and methicillin-sensitive S. Due to their poor immune response, patients with HIV and those with immunosuppression for other causes, such as malignancy, autoimmune disease, or iatrogenic due to underlying condition or organ transplant patients, require prophylaxis for numerous pathogens. These pathogens include: Aspergillus species Candida albicans Clostridium difficile Coccidioides immitis Cryptococcus neoformans Cytomegalovirus Histoplasma capsulatum Legionella pneumophila Microsporidium Mycobacterium avium complex Mycobacterium tuberculosis Pseudomonas aeruginosa Salmonella Staphylococcus aureus Streptococcus pneumoniae Streptococcus pyogenes Toxoplasma gondii [2] [3] [5] A total of 32 patients with mycoses other than cavity-formed aspergilloma were reviewed. The main pathogenic fungi were Aspergillus in 14, Candida in 8, Cryptococcus in 4, Trichosporon in 4 and Mucor in 2. Coinfection by two species was detected in 3 cases: Trichosporon and Aspergillus in 2 and Aspergillus and Candida in 1. The underlying diseases were hematologic malignancies in all cases except 1 case of lung cancer. The hematologic malignancies were mostly leukemias of various types. Cryptococcosis developed in patients given long-term corticosteroid treatment but not in leukemic patients. All cases of aspergillosis, candidiasis and mucormycosis were due to nosocomial infection. On the other hand, 3 of 4 cases of cryptococcosis or trichosporonosis were attributable to community-acquired infection. Two of 4 trichosporonosis cases were considered to have been acquired during 2-day home stays. The diagnosis of pulmonary mycosis was made pathologically in 18 and clinically in 14 cases. Of the latter, 6 cases had an air-crescent sign on chest X-ray films and 8 cases were culture-positive. Extrapulmonary involvement was seen in all 16 cases of candidiasis, cryptococcosis and trichosporonosis but not in 10 of 14 aspergillosis cases. Severe granulocytopenia was present in all cases except 4 cases of cryptococcosis and 3 cases of aspergillosis. Chest X-ray findings of aspergillosis were of two types: one was an air-crescent sign which was noted in the recovery phase from leukopenia and the other was gradually enlarging consolidation which was bound by the interlobar fissure and progressed to lobar penumonia. A diffuse granular shadow was not characteristic of any fungus species.(ABSTRACT TRUNCATED AT 250 WORDS) Imaging modalities for evaluation of pneumonia include: Chest X-ray (CXR). This is the first-line imaging of choice in the evaluation of a suspected case of pneumonia. Posteroanterior (PA) and lateral radiographs should be obtained. However, CXR may be normal for up to 72 hours in immunocompromised patients despite having symptoms. Computed tomography (CT) of the chest should be obtained in patients with high suspicion of pneumonia and normal findings on CXR. In neutropenic patients, CXR will have minimal or no abnormalities and so performing a CT scan is highly recommended. Other investigations for specific etiologies include: Urine antigen testing for pneumococcus and Legionella ELISA and PCR for viruses such as Herpes Simplex Virus (HSV), Influenza A and B, and Cytomegalovirus (CMV) Serum antigen testing for Cryptococcus and Aspergillus Beta-D-glucan testing if a fungal pathogen is suspected. [2] [8] [17] [20] Types Bacterial pneumonia: The majority of cases related to various rod shaped gram-negative organisms (52%) and Staphylococcus aureus (19%), usually of the MRSA type. Others are Haemophilus spp. (5%). In the ICU results were S. aureus (17.4%), Pseudomonas aeruginosa (17.4%), Klebsiella pneumoniae and Enterobacter spp. (18.1%), and Haemophilus influenzae (4.9%). Viral pneumonia: influenza and respiratory syncytial virus and, in the immunocompromised host, cytomegalovirus – cause 10–20% of infections. Ventilator-associated pneumonia Recurrent pneumonia is typical, predominately due to S. aureus , and less frequently due to Streptococcus pneumoniae, and Haemophilus. It can get complicated with the development of recurrent lung abscesses, bronchiectasis, and pneumatoceles. These pneumatoceles can get colonized with Aspergillus and Pseudomonas . Superinfection with Pneumocystis carinii [5] was also reported. Neutrophil-specific granule deficiency ( previously known as lactoferrin deficiency) is a rare congenital immunodeficiency characterized by an increased risk for pyogenic infections due to defective production of specific granules and gelatinase granules in patient neutrophils. Symptoms and signs Atypical infections are the key clinical manifestation of SGD. Within the first few years of life, patients will experience repeated pyogenic infections by species such as Staphylococcus aureus, Pseudomonas aeruginosa or other Enterobacteriaceae, and Candida albicans. Cutaneous ulcers or abscesses and pneumonia and chronic lung disease are common. Patients may also develop sepsis, mastoiditis, otitis media, and lymphadenopathy. Infants may present with vomiting, diarrhea, and failure to thrive. Studying the proven and probable invasive pulmonary aspergillosis (IPA) cases of some hospitals in Shanghai to provide evidence for the improvement of IPA clinical diagnosis and therapy. Forty-nine IPA cases were retrospectively analyzed for demography data, host factors, underlying conditions, chest CT, microorganism and histopathology examination, as well as therapy and clinical outcome. Of 49 subjects including 19 (38.8%) proven and 30 (61.2%) probable IPA, 3 patients (6.1%) had no host factors, 25 patients (51.0%) had IPA associated host factors and underlying conditions, while 21 patients (42.9%) had uncertain fundamental diseases. Chest CT evaluation demonstrated that radiological lesions include nodules in 29 patients, patching in 15, mass in 12, consolidation in 10, cavitation in 34, Halo sign in 19, air bronchogram in 18, crescentic sign in 6, bilateral in 33 and multifocal lesions in 38. The yielding rate of fungus culture in sputum was 26.5% (13/49), and in bronchoalveolar lavage fluid was 66.7% (10/15). Eleven of thirty-six patients (30.6%) had positive results of serum galactomannan antigen tests. Nineteen of twenty-one patients (90.5%) were proven as IPA by lung histologic examinations. Aspergillus fumigatus was the most common pathogen 81.0% (17/21). The responding rate to initial anti-fungus therapy was 50% (21/42). Our study suggests that in IPA patients, bilateral, multifocal and nodular lesion could be the most common radiological characteristic, while Halo and crescentic sign occur occasionally. Invasive technologies are more valuable to IPA diagnosis. Defense against infections PGLYRP2 plays a limited role in host defense against infections. PGLYRP2-deficient mice are more sensitive to Pseudomonas aeruginosa-induced keratitis and Streptococcus pneumoniae-induced pneumonia and sepsis. However, PGLYRP2-deficient mice did not show a changed susceptibility to systemic Escherichia coli, Staphylococcus aureus, and Candida albicans infections or intestinal Salmonella enterica infection, although the latter was accompanied by increased inflammation in the cecum. Although PGLYRP2 is not directly bacteriolytic, it has antibacterial activity against both Gram-positive and Gram-negative bacteria and Chlamydia trachomatis. Maintaining microbiome Mouse PGLYRP2 plays a role in maintaining healthy microbiome, as PGLYRP2-deficient mice have significant changes in the composition of their intestinal microbiome, which affect their sensitivity to colitis. The aim of this study is to determine the aetiology and characteristics of pulmonary cavities that developed in patients recovering from COVID-19 infection. Between 135 mmHg, adjunct glucocorticoid therapy should be used in addition to specific antimicrobials. Overall, treatment should be continued for 21 days and followed by secondary prophylaxis. 2 Prophylaxis for PCP is indicated for any HIV-infected individual who 0 has experienced a prior bout of PCP, any patient with a CD4+ T cell count of <200/μL or a CD4 percentage <15, any patient with unex- Cefiderocol is a novel siderophore cephalosporin antibiotic with broad coverage against difficult-to-treat Gram-negative bacteria, including those resistant to carbapenems. Its activity against 35 mmHg, adjunct glucocorticoid therapy should be used in addition to specific antimicrobials. Overall, treatment should be continued for 21 days and followed by secondary prophylaxis. 2 Prophylaxis for PCP is indicated for any HIV-infected individual who 0 has experienced a prior bout of PCP, any patient with a CD4+ T cell count of <200/μL or a CD4 percentage <15, any patient with unex- Cefiderocol is a novel siderophore cephalosporin antibiotic with broad coverage against difficult-to-treat Gram-negative bacteria, including those resistant to carbapenems. Its activity against 40-50 ml/kg/24hrs) is confirmed, and urine osmolality is <800 mOsm/kg, serum sodium level has to be checked. If the serum sodium level is <135 meq, it is diagnostic of PP. If the serum sodium levels are >147, it is diagnostic of diabetes insipidus. If the serum sodium is between 135 and 147, the next step would be the water deprivation test. The traditional test that has been utilized by providers for a long time is the indirect water deprivation test that indirectly measures the activity of AVP. This test is started once hypotonic polyuria is confirmed, and serum sodium is between 135 and 147. Polyuria in primary polydipsia decreases with water deprivation (typically >8hrs), and urine osmolality increases (>800 mOsm/kg typically), this is diagnostic of PP. In diabetes insipidus, polyuria does not get better with water deprivation. If the urine osmolality remains <300 mOsm/kg after water deprivation, it is diagnostic of DI. A 12-year-old female child, with a history of polyuria and polydipsia of about three years duration, was admitted to Ethio-Swedish Paediatric Hospital, in Addis Abeba. Urine output in 24 hours averaged 5-6 litres, with a frequency of 15 times during the day and 7-8 times during the night. Random urine analysis showed an osmolality of 60 mOsm/kg, Na+ 27.1 mmol/L and K+ was 7.6 mmol/L. Basal plasma osmolality was 313 mOsm/kg with Na+ being 156 mmol/L and K+ 4.06 mmol/L. Water deprivation for nine hours failed to produce a concentrated urine, which was only 138 mOsm/kg at the end of the test, with a corresponding plasma osmolality of 336 mOsm/kg. After nine hours of water deprivation, urine Na+ increased from 27.1 to 37.3 mmol/L while K+ increased from 7.1 to 18.7 mmol/L. Lypressin, a vasopressin analogue, at a concentration of 0.3 IU/kg injected intramuscularly, resulted in a marked increase in urine osmolality to 586 mOsm/kg within two hours, associated with relief of symptoms. Urinary excretion of K+ was markedly increased during the vasopressin test while Na+ excretion was little affected. A case of central diabetes insipidus of undefined etiology is presented and the possibility of altered renal handling of electrolytes and an abnormal response to vasopressin in such cases is noted. The problem of management and the currently available treatment options are summarized. Serum and urine sodium, serum and urine osmolality, hourly urine output, 24-hour urine volume, and specific gravity must be obtained. Central DI must be distinguished from nephrogenic DI. A water deprivation test can be used with or without desmopressin injection. In central DI, the vasopressin will correct urine osmolality while in nephrogenic DI, the correction will be suboptimal. [3] [25] [26] Criteria for the diagnosis of central DI: Polyuria in two consecutive hours of > 300 ml/hr or 4 to 5 ml/kg/hr in two consecutive hours Polyuria > 3L/24 hours or > 2ml/kg/hr in 24 hours Serum osmolality > 300 mOsm/kg Urine osmolality < 300 mOsm/kg Urine/plasma osmolality <  1 The specific gravity of less than 1.005 The hyperosmolar hyperglycemic state (HHS), an acute complication of diabetes mellitus with plasma hyperosmolarity, promotes the secretion of anti-diuretic hormone (ADH) and reduces the storage of ADH. Magnetic resonance T1-weighted imaging reflects ADH storage in the posterior pituitary lobe, which disappears when the storage is depleted. Whether the HHS induces ADH depletion leading to clinical manifestations has been unclear. A 55-year-old Japanese woman was admitted to our center because of mental disturbance and hypotension. She had received lithium carbonate for bipolar disorder and presented with polydipsia and polyuria from 15 years of age. On admission, she had mental disturbance (Glasgow Coma Scale, E4V1M1), hypotension (systolic blood pressure, 50 mmHg), and tachycardia (pulse rate, 123/min). Plasma glucose was 697 mg/dL osmolality was 476 mOsm/kg•H40 mL/kg Osmolarity <300 mosm/L Basal plasma AVP >1 pg/mL <1 pg/mL Pituitary bright spot Present Absent Anatomy Pathology? GU evaluation Volume <40 mL/kg Osmolarity >300 mosm/L If MRI and/or AVP assays with the requisite sensitivity and specificity are unavailable and a fluid deprivation test is impractical or undesirable, a third way to differentiate between pituitary DI, nephrogenic DI, and primary polydipsia is a trial of desmopressin therapy. Such a trial should be conducted with very close monitoring of serum sodium as well as urine output, preferably in hospital, because desmopressin will produce hyponatremia in 8–24 h if the patient has primary polydipsia. Osmolality of blood increases with dehydration and decreases with overhydration. In normal people, increased osmolality in the blood will stimulate secretion of antidiuretic hormone (ADH). This will result in increased water reabsorption, more concentrated urine, and less concentrated blood plasma. A low serum osmolality will suppress the release of ADH, resulting in decreased water reabsorption and more concentrated plasma. Syndrome of inappropriate ADH secretion occurs when excessive release of antidiuretic hormone results in inappropriately elevated urine osmolality (>100 mOsmol/L) relative to the blood plasma, leading to hyponatraemia. This ADH secretion may occur in excessive amounts from the posterior pituitary gland, or from ectopic sources such as small-cell carcinoma of the lung. Elevation may be associated with stroke mortality. Calculated osmolarity (CO)\nHere is the question:\nENDOCRINOLOGY: A 34-year-old woman is admitted for polyuria and polydipsia. In the first 24 hours of admission a diuresis of 8.2 liters is found and a blood test shows a glycemia of 96 mg/dL, natremia of 148 mEq/L and plasma osmolality of 309 mOsm/kg with urinary osmolality of 89 mOsmlkg. What diagnostic test should be performed next?\nHere are the potential choices:\n1. Hypertonic saline infusion test for serial determination of antidiuretic hormone.\n2. Dehydration test (Miller test).\n3. Administration of desmopressin with serial monitoring of urine osmolality.\n4. Determination of antidiuretic hormone in plasma.\nThe correct answer is: ", "gold_answer": "3 Administration of desmopressin with serial monitoring of urine osmolality.", "prediction": "You are a helpful medical expert, and your task is to answer a multi-choice medical question using the relevant documents. Please choose the answer from the provided options. Your responses will be used for research purposes only, so please have a definite answer.\nHere are the relevant documents:\n Disorders of thirst and vasopressin secretion present clinically in one of three ways: as hypotonic polyuria (DI), as hypodipsic hyponatremia, and as hyponatremia. In evaluating a patient with DI, the major challenge is to differentiate between primary polydipsia and neurogenic and nephrogenic DI. This is best accomplished through a series of steps that start with simple clinical observation, and progress, as necessary, to more complicated diagnostic procedures (Fig. 1). If the diagnosis is not clear from the clinical setting and the patient's history, the first step is to measure plasma osmolality and sodium under conditions of ad libitum fluid intake. If the results are clearly above the upper limit of normal range, primary polydipsia is excluded and the work-up can proceed directly to administration of vasopressin or DDAVP and/or a measurement of plasma vasopressin levels to differentiate between neurogenic and nephrogenic DI. If basal plasma osmolality and sodium fall within normal range, the standard dehydration test should be performed. If urine osmolality does not increase above that of plasma despite evident dehydration, primary polydipsia is excluded and the effect of vasopressin or DDAVP on urine osmolality should be examined to differentiate between neurogenic and nephrogenic DI. If administration of antidiuretic hormone increases urine osmolality by more than 50 per cent, the patient has severe neurogenic DI. If the increase in urine osmolality is less than 50 per cent, the patient has nephrogenic DI. In patients who do not concentrate urine above that of plasma in response to dehydration, the best approach is to measure plasma vasopressin, osmolality, and sodium after the latter have been increased above normal range by dehydration and/or infusion of hypertonic saline. When these results are plotted on a suitable nomogram (Fig. 2), neurogenic DI can be clearly diagnosed from the relative deficiency of vasopressin. In patients with normal vasopressin levels, primary polydipsia can be differentiated from nephrogenic DI by examining the relationship of urine osmolality to plasma vasopressin (Fig. 3), obtained during dehydration and/or graded vasopressin infusion. In evaluating a patient with sustained hypernatremia, it is only necessary to assess thirst, which can be done by a simple bedside observation. In a patient without obvious neurologic or cognitive impairment, absence of thirst in the face of plasma osmolality above 305 mosm/kg (plasma sodium above 150 mEq/L) is diagnostic for hypodipsic hypernatremia. In a patient who presents with hyponatremia, the main objective is to differentiate between hyper-, hypo-, and euvolemic (SIADH) types A 20-year-old patient was evaluated because of polydipsia and polyuria; by means of the dehydration test a partial defect in the secretion of antidiuretic hormone (ADH) was demonstrated, since the urinary osmolality after the administration of exogenous vasopressin was superior by 25 percent to the maximum spontaneous urinary osmolality reached after a period of fluid restriction. Nevertheless, there was also a component of psychogenic polydipsia because the daily basal fluid intake was superior to 15 liters, and in view of the fact that the urinary osmolality could reach 600 mOsm/kg, the endocrine defect cannot totally be responsible for the enormous volume of fluid intake. This is the first case in the world literature in which the association between potomania and deficiency in the secretion of ADH is reported. Since ADH is one of the factors which regulate the behaviour of various animal species it is possible that its deficiency may be directly responsible for the psychic disorder which led to the potomania. It is also possible that an anatomical hypothalamic lesion, too small to be demonstrated, might have a simultaneous effect on the centers regulating thirst and the neurons producing vasopressin. A water deprivation test or a hypertonic saline infusion test with the measurement of plasma osmolality and plasma vasopressin are the gold standard tests in the differential diagnosis of polyuric syndromes. Because commercially available vasopressin kits are too insensitive for this approach, and the concentration of vasopressin in urine is much higher than in plasma, urinary vasopressin measurements may be an alternative to the more difficult plasma vasopressin measurement. The diagnostic value of the measurement of urinary vasopressin with a rather insensitive commercially available vasopressin kit was compared with plasma vasopressin measurement by a highly sensitive radioimmunoassay (RIA). Thirteen normal subjects and 27 patients with polyuria/polydipsia were examined by an 8-h fluid deprivation test. In all blood samples (0800 h, 1200 h, 1400 h and 1600 h) and in all urine collections (2-hourly fractions), osmolality as well as vasopressin were measured. Using plasma vasopressin measurement with a highly sensitive RIA as gold standard test, nine patients were classified as having primary polydipsia, whereas 18 had partial or complete cranial diabetes insipidus. Whereas the substitution of plasma vasopressin measurement by urinary vasopressin measurement alone did not provide 100% separation between both groups, the product of urinary vasopressin and urinary osmolality related to plasma osmolality completely separated the patients with primary polydipsia from those with diabetes insipidus. Urinary measurement of vasopressin and osmolality alone, which was recommended as a noninvasive diagnostic procedure in children, was too insensitive for exact differential diagnosis in our adult patients. The simultaneous measurement of plasma vasopressin and plasma osmolality in a dehydration test is the most powerful diagnostic tool in the differential diagnosis of polyuria/polydipsia. However, if highly sensitive assays for plasma vasopressin measurements are not available, the measurement of urinary vasopressin with commercially available, less sensitive RIAs may be a diagnostic alternative, which showed nearly the same sensitivity as plasma vasopressin measurement in our study population. Nephrogenic diabetes insipidus (NDI) is caused by the inability of renal collecting duct cells to respond to arginine vasopressin (AVP)/antidiuretic hormone (ADH). We present the case of a 7-year-old boy with a history of excretion of large amounts of dilute urine and polydipsia since infancy. The boy had several vomiting episodes with mild dehydration during the first 3 years of life. There was no evidence of headaches, dizziness or visual problems. He drinks between 2 and 3 L/day and has 24-hour diuresis of 2 liters, now. He has prepubertal appearance with appropriate weight [+0.85 standard deviation score (SDS)] and height (+0.15 SDS) for his age. His intelligence was also normal. The water deprivation test showed low urine osmolality after 8 hours of dehydration. After desmopressin administration, urine osmolality remained low. Serum osmolality was in the normal range for sex and age before and after desmopressin administration. This indicated a nephrogenic form of diabetes insipidus. Molecular analyses revealed a P286L [p.Pro(CCC)286Leu(CTC)] mutation in the AVPR2 gene, that was inherited from his mother. This patient is the first case with genetically confirmed X-linked inherited form of NDI in the Republic of Macedonia. Molecular analysis confirmed the clinical diagnosis and enabled genetic advice for this family. Hyponatremia is a frequent condition in elderly patients. In diagnostic workup, a 24-hour urine sample is used to measure urinary osmolality and urinary sodium concentration necessary to confirm the diagnosis of the syndrome of inappropriate secretion of antidiuretic hormone (SIADH). This study was undertaken to test the hypothesis that a spot urine sample would be sufficient for urinalysis. In nine patients with SIADH, morning spot and 24-hour urine samples were examined for osmolality and sodium concentration. Levels of arginine vasopressin, atrial natriuretic and brain natriuretic peptides, renin, and aldosterone were measured in the supine and upright positions of patients and compared with nine healthy age-matched control patients. The patients had low plasma osmolality (median 266 mOsm/kg) and measurable levels of arginine vasopressin (median 1.8 pg/mL). Values of osmolality in the spot urine (median 298 mOsm/kg) and in the 24-hour urine (median 215 mOsm/kg) did not differ significantly; neither did sodium concentration (medians 80 mmol/L in the spot urine versus 45 mmol/L in the 24-hour urine). Patients had significantly elevated plasma levels of brain natriuretic peptide (P = 0.007), elevated mean arterial blood pressure (P = 0.03), and lower plasma levels of creatinine (P = 0.002) compared to the controls. A spot urine sample seems to be sufficient to confirm the diagnosis of SIADH. Urine osmolality reaches the normal reference range. Urine osmolality is stable on two to three consecutive hourly measurements, even with rising plasma osmolality. Plasma osmolality is higher than 295 to 300 mOsm/kg Plasma Na greater than 145 mEq If AVP-R is suspected in newborns and young infants, the diagnostic test of choice is DDAVP (1 mcg subcutaneously or intravenously over 20 minutes, maximum dose of 0.4 mcg/kg). In children, the water deprivation test should be closely monitored. If one of the following endpoints is reached, discontinue the trial: Urine osmolality reaches the normal reference range. Plasma osmolality greater than 295 mOsm/kg to 300 mOsm/kg Plasma sodium greater than 145 meq/L Loss of 5% of body weight or signs of volume depletion A fluid or water deprivation test is a medical test which can be used to determine whether the patient has diabetes insipidus as opposed to other causes of polydipsia (a condition of excessive thirst that causes an excessive intake of water). The patient is required, for a prolonged period, to forgo intake of water completely, to determine the cause of the thirst. This test measures changes in body weight, urine output, and urine composition when fluids are withheld. Sometimes measuring blood levels of ADH (a synonym for vasopressin) during this test is also necessary. If there is no change in the water loss despite fluid deprivation, desmopressin may be administered to distinguish between the two types of diabetes insipidus which are central & nephrogenic diabetes insipidus. The time of deprivation may vary from 4 to 18 hours. The serum osmolality and urine osmolality are both measured in the test. Interpretation of WDT The conditions can be distinguished in the following way: Diabetes insipidus (DI) and primary polydipsia (PP) are characterised by polyuria and polydipsia. It is crucial to differentiate between these two disorders since the treatment is different. The aim of this study was to evaluate the diagnostic value of the short and an extended variant of the water deprivation test (WDT) and of measuring urinary vasopressin (AVP) in patients with polyuria and polydipsia. A retrospective, single-centre study based on WDTs performed between 2004 and 2014 including 104 consecutive patients with the polyuria-polydipsia syndrome. During a strict water deprivation, weight, urinary osmolality, urinary vasopressin and specific gravity were collected until one of the following was reached: i) >3% weight reduction, ii) Urinary specific gravity >1.020 or, urinary osmolality >800 mOsm/L, iii) Intolerable adverse symptoms such as excessive thirst. Out of 104 patients (67 women, 37 men), 21 (20%) were diagnosed with DI and 83 (80%) with PP. The median (interquartile range; range) test duration was 14 hours (10-16; 3-36) in patients with DI and 18 hours (14-24; 7-48) in patients with PP (P=0.011). Of those diagnosed with PP, 22 (26%) did not reach urinary specific gravity >1.020 nor urine osmolality >800 mOsm/L. Urine AVP did not overlap between patients with PP and patients with central DI. The short WDT is of limited value in the diagnostic work-up of polydipsia and polyuria and a partial DI may have been missed in every fourth patient diagnosed with PP. Urinary AVP has excellent potential in discriminating PP from central DI. A 66-year-old Japanese man presented with persistent hyponatremia without polydipsia and polyuria. Laboratory examination showed serum sodium of 117 mEq/l, plasma osmolality 239 mosm/kg, urine sodium 108 mEq/l, urine osmolality 577 mosm/kg, and normal levels (less than 2.0 pg/ml) of serum antidiuretic hormone (ADH). ADH release was regulated normally with changes in plasma osmolality. No obvious cause for the syndrome of inappropriate secretion of ADH (SIADH) could be detected. However, 20 months later, the patient had bouts of hematuria and was found to have cancer of the urinary bladder. Increased renal sensitivity to ADH was suspected as the underlying mechanism of SIADH. Combined deficits in arginine vasopressin secretion (AVP) and thirst sensation can result in life threatening hyperosmolality and hypernatremia. Complications include seizures, profound volume contraction and renal failure. Fortunately, this is an uncommon clinical condition, with approximately 70 cases reported in the literature over the past 47 years [1]. Defects in AVP secretion and/or synthesis produce central diabetes insipidus (DI), polyuria with polydipsia, hypernatremia and hyperosmolality. Most awake and alert patients with an intact thirst stimulus will \"drink\" themselves back to a normal serum sodium and osmolality. However, if there is concomitant destruction of the osmoreceptors that regulate thirst, osmolal and volume homeostasis cannot be maintained. The relationships between urine osmolarity and serum osmolarity and plasma vasopressin levels are vital for distinguishing a reset osmostat from central DI. After obtaining approval from our institutional review board, we retrospectively reviewed the medical record of a 37-year-old patient who presented to our institution with a serum sodium of 176 mEq/l. Admission laboratory examination revealed: hemoglobin 12.8 g/dl; white blood cell count 4.7 × 103/µl, with a normal differential; random serum glucose 91 mg/dl ; sodium 176 mEq/l; plasma osmolality 366 mOsm/kg; BUN 33 mg/dl; serum creatinine 1 mg/dl; calcium 9.5 mg/dl; urine specific gravity 1.032; and urine osmolality 1,172 mOsm/kg. An MRI with contrast of the sella/ pituitary revealed an enhancing mass centered within the suprasellar cistern and anterior third ventricle, measuring 3.0 × 3.9 × 3.4 cm. The lesion appeared to involve the hypothalamus and displaced the optic chiasm inferiorly. Evaluation of pituitary function revealed normal serum levels of thyroid stimulating hormone, AM cortisol, luteinizing hormone, follicle stimulating hormone and prolactin. Figure 1 illustrates the relationship between measured serum AVP levels and serum osmolality. Figure 2 shows the relationship between measured urine and serum osmolality. If the serum AVP levels were not available, it would appear as though the patient had a reset osmostat. The kidneys appear to appropriately generate maximally concentrated urine at a serum osmolality above 348 but are unable to below this value. When compared with the normal curve, our patient's AVP levels were lower than expected for the corresponding osmolality. This pattern is consistent with a partial central DI. She does not have a reset osmostat. In the presence of significant volume contraction and a reduced GFR, her kidneys produced more concentrated urine despite markedly decreased central vasopressin production. As the volume contraction abated and the GFR improved, polyuria recurred, despite persistent hyperosmolarity and hypernatremia. Hypertonic saline test is indispensable for the evaluation of posterior pituitary function. However the test is not simple, including water loading, urine sampling and at least 45 min of hypertonic saline infusion, mostly because the test relies on urinary osmolality as an index of ADH secretion. The object of this study is try to simplify the test by directly measuring plasma ADH concentration before and after 10 min of hypertonic saline infusion. Intravenous infusion of hypertonic saline (5% NaCl, 0.24 ml/kg/min, for 10 min) was performed on normal subjects, patients with diabetes insipidus and patients with renal failure under chronic hemodialysis. Venous blood samples were obtained seriously including just before and after 10 min of the infusion. ADH was extracted from plasma using Sep-Pak C18 column and assayed by specific RIA. Minimum sensitivity of the assay was 0.25 pg/ml. The hypertonic saline infusion resulted in an increase of plasma osmolality by about 8 mOsm/kg H2O and plasma sodium concentration by 4 mEq/l. Plasma ADH increased from 0.77 +/- 0.09 to 3.42 +/- 0.73 pg/ml (m +/- SE, n = 8, p less than 0.01) in normal subjects of ad lib. water drinking and from 0.55 +/- 0.33 to 2.34 +/- 0.33 (m +/- SE, n = 4, p less than 0.05) in water loaded normal subjects (20 ml/kg of water, 60 min before hypertonic saline infusion).(ABSTRACT TRUNCATED AT 250 WORDS) After polyuria (>40-50 ml/kg/24hrs) is confirmed, and urine osmolality is <800 mOsm/kg, serum sodium level has to be checked. If the serum sodium level is <135 meq, it is diagnostic of PP. If the serum sodium levels are >147, it is diagnostic of diabetes insipidus. If the serum sodium is between 135 and 147, the next step would be the water deprivation test. The traditional test that has been utilized by providers for a long time is the indirect water deprivation test that indirectly measures the activity of AVP. This test is started once hypotonic polyuria is confirmed, and serum sodium is between 135 and 147. Polyuria in primary polydipsia decreases with water deprivation (typically >8hrs), and urine osmolality increases (>800 mOsm/kg typically), this is diagnostic of PP. In diabetes insipidus, polyuria does not get better with water deprivation. If the urine osmolality remains <300 mOsm/kg after water deprivation, it is diagnostic of DI. A 12-year-old female child, with a history of polyuria and polydipsia of about three years duration, was admitted to Ethio-Swedish Paediatric Hospital, in Addis Abeba. Urine output in 24 hours averaged 5-6 litres, with a frequency of 15 times during the day and 7-8 times during the night. Random urine analysis showed an osmolality of 60 mOsm/kg, Na+ 27.1 mmol/L and K+ was 7.6 mmol/L. Basal plasma osmolality was 313 mOsm/kg with Na+ being 156 mmol/L and K+ 4.06 mmol/L. Water deprivation for nine hours failed to produce a concentrated urine, which was only 138 mOsm/kg at the end of the test, with a corresponding plasma osmolality of 336 mOsm/kg. After nine hours of water deprivation, urine Na+ increased from 27.1 to 37.3 mmol/L while K+ increased from 7.1 to 18.7 mmol/L. Lypressin, a vasopressin analogue, at a concentration of 0.3 IU/kg injected intramuscularly, resulted in a marked increase in urine osmolality to 586 mOsm/kg within two hours, associated with relief of symptoms. Urinary excretion of K+ was markedly increased during the vasopressin test while Na+ excretion was little affected. A case of central diabetes insipidus of undefined etiology is presented and the possibility of altered renal handling of electrolytes and an abnormal response to vasopressin in such cases is noted. The problem of management and the currently available treatment options are summarized. Serum and urine sodium, serum and urine osmolality, hourly urine output, 24-hour urine volume, and specific gravity must be obtained. Central DI must be distinguished from nephrogenic DI. A water deprivation test can be used with or without desmopressin injection. In central DI, the vasopressin will correct urine osmolality while in nephrogenic DI, the correction will be suboptimal. [3] [25] [26] Criteria for the diagnosis of central DI: Polyuria in two consecutive hours of > 300 ml/hr or 4 to 5 ml/kg/hr in two consecutive hours Polyuria > 3L/24 hours or > 2ml/kg/hr in 24 hours Serum osmolality > 300 mOsm/kg Urine osmolality < 300 mOsm/kg Urine/plasma osmolality <  1 The specific gravity of less than 1.005 The hyperosmolar hyperglycemic state (HHS), an acute complication of diabetes mellitus with plasma hyperosmolarity, promotes the secretion of anti-diuretic hormone (ADH) and reduces the storage of ADH. Magnetic resonance T1-weighted imaging reflects ADH storage in the posterior pituitary lobe, which disappears when the storage is depleted. Whether the HHS induces ADH depletion leading to clinical manifestations has been unclear. A 55-year-old Japanese woman was admitted to our center because of mental disturbance and hypotension. She had received lithium carbonate for bipolar disorder and presented with polydipsia and polyuria from 15 years of age. On admission, she had mental disturbance (Glasgow Coma Scale, E4V1M1), hypotension (systolic blood pressure, 50 mmHg), and tachycardia (pulse rate, 123/min). Plasma glucose was 697 mg/dL osmolality was 476 mOsm/kg•H40 mL/kg Osmolarity <300 mosm/L Basal plasma AVP >1 pg/mL <1 pg/mL Pituitary bright spot Present Absent Anatomy Pathology? GU evaluation Volume <40 mL/kg Osmolarity >300 mosm/L If MRI and/or AVP assays with the requisite sensitivity and specificity are unavailable and a fluid deprivation test is impractical or undesirable, a third way to differentiate between pituitary DI, nephrogenic DI, and primary polydipsia is a trial of desmopressin therapy. Such a trial should be conducted with very close monitoring of serum sodium as well as urine output, preferably in hospital, because desmopressin will produce hyponatremia in 8–24 h if the patient has primary polydipsia. Osmolality of blood increases with dehydration and decreases with overhydration. In normal people, increased osmolality in the blood will stimulate secretion of antidiuretic hormone (ADH). This will result in increased water reabsorption, more concentrated urine, and less concentrated blood plasma. A low serum osmolality will suppress the release of ADH, resulting in decreased water reabsorption and more concentrated plasma. Syndrome of inappropriate ADH secretion occurs when excessive release of antidiuretic hormone results in inappropriately elevated urine osmolality (>100 mOsmol/L) relative to the blood plasma, leading to hyponatraemia. This ADH secretion may occur in excessive amounts from the posterior pituitary gland, or from ectopic sources such as small-cell carcinoma of the lung. Elevation may be associated with stroke mortality. Calculated osmolarity (CO)\nHere is the question:\nENDOCRINOLOGY: A 34-year-old woman is admitted for polyuria and polydipsia. In the first 24 hours of admission a diuresis of 8.2 liters is found and a blood test shows a glycemia of 96 mg/dL, natremia of 148 mEq/L and plasma osmolality of 309 mOsm/kg with urinary osmolality of 89 mOsmlkg. What diagnostic test should be performed next?\nHere are the potential choices:\n1. Hypertonic saline infusion test for serial determination of antidiuretic hormone.\n2. Dehydration test (Miller test).\n3. Administration of desmopressin with serial monitoring of urine osmolality.\n4. Determination of antidiuretic hormone in plasma.\nThe correct answer is: 2. Dehydration test (Miller test)." }, { "question": "You are a helpful medical expert, and your task is to answer a multi-choice medical question using the relevant documents. Please choose the answer from the provided options. Your responses will be used for research purposes only, so please have a definite answer.\nHere are the relevant documents:\n Epstein-Barr virus (EBV) infection typically presents with pharyngeal symptoms and subclinical transaminitis. We present a case of a 27-year-old woman with no known past medical history who presented with painless jaundice and dark-colored urine for three days. Her review of systems was negative for fever, sore throat, nausea, vomiting, pruritus, or rash. Her last sexual contact was six months ago with a male partner, and she only drank alcohol socially. Family and surgical history were non-significant. Physical examination revealed 3+ bilateral conjunctival icterus without abdominal tenderness or organomegaly. She had elevated transaminases: alanine transaminase (ALT) of 1287U/L and aspartate aminotransferase of (AST) 1057U/L but her alkaline phosphatase (ALP) was only slightly above normal at 109U/L (normal range 35-104U/L), with a direct hyperbilirubinemia - total bilirubin 9.5mg/dl, direct bilirubin 6.8mg/dl; the abdominal ultrasound revealed non-dilated bile ducts. Hepatitis A, B, and C serology was negative, but her EBV serology showed an infection. She had incidental thalassemia minor without splenomegaly or asterixis. She was managed conservatively, and her liver enzymes trended down with supportive management. Although EBV is an uncommon cause of painless jaundice, this diagnosis should be considered, especially when other more common causes of jaundice have been ruled out. A high index of suspicion should be maintained to detect EBV hepatitis as it can easily be diagnosed through serological testing. Roddick explained that her hepatitis C was unexpectedly diagnosed in 2004, following a blood test that was part of a medical examination needed for a life insurance policy. The blood test indicated abnormal liver function and subsequent blood tests diagnosed hepatitis C. Roddick explained that she had a large blood transfusion in 1971, after the birth of her younger daughter, and that she was convinced that the transfusion had infected her with hepatitis C. This was about twenty years before blood donors in the United Kingdom were screened for hepatitis C. She reported that she had developed cirrhosis of the liver, and that her main symptoms were itching and poor concentration. She briefly mentioned that medical treatment with interferon did not suit her. Roddick explained that she kept fit and active, and that she attended biannual out-patient hospital appointments in Southampton, as well as being under review by the liver transplant team at the Addenbrooke's Hospital in Cambridge. Hepatitis is a disease of the liver that has been recognised since antiquity. Symptoms include jaundice, a yellowing of the skin, eyes and body fluids. There are numerous causes, including viruses – particularly hepatitis A virus, hepatitis B virus and hepatitis C virus. Throughout history epidemics of jaundice have been reported, mainly affecting soldiers at war. This \"campaign jaundice\" was common in the Middle Ages. It occurred among Napoleon's armies and during most of the major conflicts of the 19th and 20th centuries, including the American Civil War, where over 40,000 cases and around 150 deaths were reported. The viruses that cause epidemic jaundice were not discovered until the middle of the 20th century. The names for epidemic jaundice, hepatitis A, and for blood-borne infectious jaundice, hepatitis B, were first used in 1947, following a publication in 1946 giving evidence that the two diseases were distinct. In the 1960s, the first virus that could cause hepatitis was Approach to the Patient with Liver Disease Suspected Liver Disease Abnormal liver tests Acute < 6 months Chronic > 6 months Diagnostic evaluation 1. IgM Anti-HAV 2. HBsAg 3. IgM Anti-HBc 4. Anti-HCV 5. ANA, SMA 6. Monospot, heterophile 7. Ceruloplasmin 8. Alcohol history 9. Drug history Diagnostic evaluation 1. AMA 2. Drug history 3. Ultrasound/MRI 4. MRCP/ERCP Liver biopsy in acute liver disease: Reserved for patients in whom the diagnosis remains unclear despite medical evaluation Liver biopsy in chronic liver disease: Often valuable for diagnosis as well as staging and grading liver disease Diagnostic evaluation 1. HBsAg 2. Anti-HCV 3. Fe saturation, ferritin 4. Ceruloplasmin 5. ˜1AT 6. ANA, SMA 7. Ultrasound 8. Alcohol history Diagnostic evaluation 1. Drug history 2. AMA 3. P-ANCA 4. Ultrasound 5. MRCP/ERCP Hepatitic: °°ALT Mixed: ˛ALT, ˛AlkP Cholestatic: °°AlkP, °°gGT, ˛ALT Hepatitic: °°ALT Mixed: ˛ALT, ˛AlkP Cholestatic: °°AlkP, °°gGT, ˛ALT Her social history is significant for alcohol use (three to four glasses of wine/night). Her vital signs include the following: temperature 99.8°F, blood pressure 132/64 mm Hg, pulse 78 bpm, and respiratory rate 15/min. On physical examination, she had left upper abdominal tenderness with evidence of hepatomegaly and mild scleral icterus. Laboratory data revealed the following: alanine aminotransferase, 527 IU/L (normal 10–35 IU/L); aspartate aminotransferase, 425 IU/L (normal < 35 IU/L); and bilirubin, 2.9 mg/dL (normal 0.1–0.3 mg/dL). What medications do OTC cold and flu preparations typically contain? Which of the OTC medications might have contrib-uted to the patient’s current symptoms? KH, a 55-year-old woman, presents to the emergency department with nausea, vomiting, and complaints of new-onset flu symptoms over the past several days. Her past medical history is significant for allergic rhinitis and chronic lower back pain secondary to a work-related fall 2 years ago. Her Patients usually present with varying symptoms apart from yellowish discoloration of skin along with pruritus, thus providing clues to narrow down the etiology or can also be asymptomatic. A thorough questioning regarding the use of drugs, alcohol or other toxic substances, risk factors for hepatitis (travel, unsafe sexual practices), HIV status, personal or family history of any inherited disorders or hemolytic disorders is vital. Other important points include the duration of jaundice; and the presence of any coexisting signs and symptoms, like a joint ache, rash, myalgia, changes in urine and stool. [22] A history of arthralgias and myalgias before yellowing indicates hepatitis, either due to drugs or viral infections. Phase 2 (prodromal phase) - Patients in this phase usually present with anorexia, nausea, vomiting, malaise, pruritus, urticaria, arthralgias, and fatigue. Many times these patients are misdiagnosed as having gastroenteritis or viral infection. Phase 3 (icteric phase) - Patients in this phase present with dark-colored urine and pale-colored stool. Some patients develop jaundice and right upper quadrant pain with liver enlargement. Phase 4 (convalescent phase) - Patients typically start noticing the resolution of symptoms, and laboratory studies show liver enzymes returning to normal levels. [32] Viral infections during pregnancy can pose serious threats to mother and fetus from the time of conception to the time of delivery. These lead to congenital defects, spontaneous abortion and even death. The definitive diagnosis and management of pregnancy-related viral infections may be challenging especially in less resourced countries. We present clinical and laboratory responses to the diagnosis and management of three cases of fulminant hepatitis secondary to Hepatitis E viral infection in pregnancy.Case 1 was a 31-year-old Ghanaian woman who presented with a week's history of passing dark urine as well as yellowish discoloration of the eyes. She subsequently developed fulminant hepatitis secondary to Hepatitis E viral infection, spontaneously aborted at 24 weeks of gestation and later died.Case 2 was also a 31-year-old Ghanaian woman who was admitted with a four-day history of jaundice. She had low grade fever, but no history of abdominal pain, haematuria, pale stool or pruritus. She next developed fulminant hepatitis secondary to Hepatitis E viral infection. However, she did not miscarry but died at 28 weeks of gestation.Case 3 was a 17-year-old Ghanaian woman who was referred to the tertiary health facility on account of jaundice and anaemia. She had delivered a live male infant at maturity of 32 weeks but noticed she was jaundiced and had a presentation of active disease 3 days prior to delivery. The baby was icteric at birth and on evaluation, had elevated bilirubin (mixed type) with normal liver enzymes. Hepatitis E virus infection was confirmed in both mother and baby. However, the jaundice and the hepatomegaly resolved in mother and baby after 5 and 12 days respectively. To the best of our knowledge, these are the first documented cases of fatal fulminant hepatic failures resulting from HEV infection in Ghana. Analysis of serum copper (Cu) level dynamics during the acute phase of hepatitis acute B and the early convalescence in compliance with gravity of the acute disease course. The study included 39 patients (12 men and 27 women), aged 18 to 76 years. They were hospitalised in the Department of Infectious Diseases of Medical Academy in Lublin because of the hepatitis acute B, without coexisting diseases. The diagnosis was based on the epidemiologic anamnesis, clinical symptoms, biochemical and serological examinations. The studied group was divided in respect to sex and the course of the disease as: light, medium-weighty and weighty. In all examined patients, the serum Cu level was determined according to the following scheme: at the first, tenth, twentieth and the last day of the hospitalisation and additionally one time at four weeks after discharging from the clinic. The serum Cu level was made by atomic absorption spectrometry (AAS) at the wave length of 324.8 nm. The received data were subjected to statistical analysis according to t-Student's test and in cases of significant differences according to the variants of c-Cochran and Cox's tests. According to the SI Unit Conversion Guide, the values 11.22 to 23.58 mumol/l were taken as the normal range. The values derived from the control group of 24 healthy persons (13 men and 11 women) aged 22 to 69 years. The significant increase of serum Cu level in comparison with the control values was found both in the acute phase of hepatitis B and the early convalescence. It could be observed a correlation between serum Cu level and the course of hepatitis viralis acuta B. The statistics proved that approximately 25% of the patients with acute HCV present with jaundice, and only 10-20% develop gastrointestinal symptoms. We present the case of a 58 year-old woman, with prior antecedents of arterial hypertension and diabetes mellitus since 25 years old, hypercholesterolemia and hypertriglyceridemia, psoriasis, epilepsy and depressive syndrome. She clinically presents asthenia, anorexia, itching, jaundice and choluria. The objective examination showed an orientated patient, without flapping, hemorrhagic dyscrasia or signs of chronic hepatic disease, with icteric mucosa and skin, abdominal pain, with hepatomegaly and splenomegaly. The laboratory tests have been compatible with acute hepatitis with colestatic pattern: AST/ALT 969/798 UI/ml, FA 796 UI/ml, GGT 2476 UI/ml, BT/BD 7.39/6.10, INR 0.9. The abdominal echography showed: hepatomegaly, regular borders, hepatic steatosis, splenomegaly without ascitic fluid. The viral serological tests revealed protection for hepatitis A ( IgM neg/IgG pos), negative for HVB infection (AgHBs neg, anti-HBc neg), negative for HVE and other viruses (CMV Herpes virus, Epstein Barr, HIV), positive antibodies for HCV and positive RNA VHC (164200 UI/ml), HCV genotype 3a, IL-28B CT, negative autoimmunity. The previous HCV tests were negative, sustaining the recent infection. We assumed an acute hepatitis C. The patient was symptomatically treated with hydroxyzine for the skin itch, with vitamin K for INR correction and she was closely monitored. She had good clinical and laboratorial evolution and she was discharged after one week, maintaining hepatology consultation. She spontaneously cleared HCV infection after 3 months, maintaining negative RNA VHC 6 months after infection. The patient has cured the HCV infection with no need for antiviral treatment. 302 liver cirrhoses obtained from the post-mortem material 1970 to 1979 have been examined for the occurrence of the cuprous protein complexes by means of the orcein-staining according to Shikata. The part of the B-posthepatic cirrhoses (89%) could be simultaneously determined with the help of this method. Cuprous protein complexes have been found in bilious (100%), alcoholic (43.9%), etiologically uncertain (34.4%), and B-posthepatic (25,9%) liver cirrhoses. They prove an existing or experienced chronic cholestasis. The demonstration of copper has no principal significance for the etiologic classification of cirrhoses. In the bioptic diagnostics the demonstration of copper has a certain significance for the differentiation of the primary destructive cholangitis from chronically active hepatites of other genesis. The investigations incidentally revealed that HBsAg-containing hepatocytes could be identified by means of orcein in sections performed on archive material after destaining independent of the age of the sections and their primary staining. History The Tenpō Tsūhō came around a century after the introduction of the Hōei Tsūhō (Kyūjitai: 寳永通寳 ; Shinjitai: 宝永通宝) during the 5th year of the Hōei era (1708), which had a face value of 10 mon (while only containing 3 times as much copper as a 1 mon Kan'ei Tsūhō coin), but was discontinued shortly after it started circulating as it wasn't accepted for its nominal value. A 37-year-old woman presented with increasing abdominal pain and jaundice. Six weeks before admission, she developed persistent diarrhea and jaundice of the skin. She also bruised easily, and her gums bled. In the subsequent weeks, her appetite decreased, she was fatigued, and she had nausea, vomiting, and abdominal distension. She had a history of drinking 1 quart of vodka every day for 20 years, with brief periods of abstinence; she stopped consuming alcohol 11 days before admission because it no longer provided symptomatic relief. Her past medical history was also notable for depression, including a suicide attempt 4 years earlier. She did not smoke, use illicit drugs, or have unprotected sexual intercourse. She had received no blood transfusions and had not traveled recently. She took no medications, except for occasional ibuprofen. On physical examination, she was thin and deeply jaundiced, and she trembled and responded slowly to questions. She was afebrile but tachypneic, and she had orthostatic hypotension. Her HEENT examination was notable for scleral and sublingual icterus, as well as crusted blood on her gums and teeth. The jugular veins were flat. The cardiac examination revealed tachycardia (heart rate, 103 beats per minute) without murmurs, rubs, or gallops. The abdomen was nontender and protuberant, with hypoactive bowel sounds; the spleen was not palpable, and there was no fluid wave or caput medusae. The liver percussed to 18 cm, with a smooth edge extending 10 cm below the costal margin. She had cutaneous telangiectases on her chest and bilateral palmar erythema. There was no peripheral edema. The neurologic examination was notable for asterixis. Her stool was guaiac positive. Laboratory studies revealed the following values: hematocrit, 21.2%; white blood cells, 17,310/mm(3); ammonia, 42 micromol/L; serum creatinine, 3.9 mg/dL; serum urea nitrogen, 70 mg/dL; albumin, 2.1 g/dL; total bilirubin, 26.8 mg/dL; alanine aminotransferase, 14 U/L; aspartate aminotransferase, 77 U/L; alkaline phosphatase, 138 U/L; prothrombin time, 103 seconds (international normalized ratio, 10.6); and urinary sodium, <5 mg/dL. Urinalysis revealed an elevated specific gravity and numerous muddy granular casts. Hepatitis A, B, and C serologies were negative. On abdominal ultrasound examination, there was no ascites, and the liver was echogenic. The portal and hepatic veins were patent, and the hepatic arteries were normal. The spleen measured 14 cm. What is the diagnosis? Wilson disease is a disease of excessive copper accumulation. It is associated with psychiatric disturbances due to copper accumulation in the basal ganglia. Kayser-Fleischer rings are pathognomonic for Wilson disease but are not completely sensitive (requires an expert ophthalmologist to confirm this finding). Laboratory evaluation that favors a diagnosis of Wilson disease includes low serum ceruloplasmin levels and elevated urinary copper, and if abnormal, requires further evaluation by a hepatologist. Alcoholic hepatitis Autoimmune hepatitis Cirrhosis Drug-induced liver injury Hemochromatosis Hepatitis A Hepatitis C Hepatitis D Hepatitis E Hepatocellular carcinoma Human immunodeficiency virus Wilson disease Hydralazine is a commonly used oral antihypertensive agent. We report a rare case of hydralazine-induced hepatotoxicity in the form of subacute hepatic necrosis. A 75-year-old African American woman presented with jaundice of 7-day duration. She was started on hydralazine 100 mg 3 times a day 10 weeks before presentation. On physical examination, scleral icterus was noted. Workup revealed elevated liver transaminases, alkaline phosphatase, and conjugated bilirubin. She had no history of liver disease, and liver function tests had been normal before starting hydralazine. Other etiologies, including viruses, common toxins, drugs, autoimmune, and copper-induced hepatitis, were excluded. Abdominal imaging studies did not show any evidence of intrahepatic or extrahepatic biliary ductal dilatation, and no pathologies were seen in the liver and pancreas. The patient's liver biopsy revealed extensive lobular hepatitis, significant necrosis, mixed inflammatory infiltrate, and no significant fibrosis, supporting a diagnosis of drug-induced liver injury. Hydralazine was immediately discontinued. She showed improvement of clinical and laboratory abnormalities within 5 days after discontinuation of hydralazine. To establish the diagnosis of hydralazine-induced liver injury, we used assessment tool outlined by the Council for International Organization of Medical Sciences (CIOMS) scale that led to \"high probable\" relationship. Although rare, clinically significant, and potentially life-threatening liver injury can result from use of hydralazine. Both clinical and histological presentations in our patient suggest acute liver injury. The hydralazine-induced hepatitis seems to be reversible as discontinuation of the drug improves clinical outcomes. We highly recommend monitoring of the liver function during hydralazine treatment. Occult hepatitis B (ОHB) characterized by the absence of blood HBsAg attracts the attention of specialists of different profiles; however, its clinical morphological aspects have not been practically studied. to estimate the proportion of OHB in the structure of fatal outcomes in chronic viral hepatitis (CVH) and to characterize its clinical course and structural changes on autopsy materials. A total of 455 autopsy cases of CVH were examined for its etiology in the S.P. Botkin Clinical Hospital of Infectious Diseases in 2014-2016. An in-depth prospective clinical analysis was made to investigate 28 cases of OHB in the stage of decompensated liver cirrhosis, which had subsequently culminated in death. The criteria of inclusion were history data and clinical symptoms of CVH in the detection of markers for hepatitis A, C, and D and HIV in serum HBcAb in the absence of HBsAg. HBsAbs were also determined. Along with the traditional morphological examination, immunohistochemistry (IHC) for HBsAg and HBcAg was carried out. There were 108 CVHB cases (23.7% of the total cases of CVH), including 77 OHB cases (71.3% of those of CVHB) while HBsAg was not determined. HBsAb-negative patients were more often observed to have clinical signs of jaundice (p<0.05) and skin itching (p<0.05). Dyspepsia and hemorrhagic manifestations prevailed in patients with HBsAb (more than 10 IU/l) (p<0.05). All the cases were found to have characteristic morphological signs of CVH, including intranuclear inclusions and nuclear polymorphism in 10.7% of deaths. There was an IHC-positive reaction to VHB antigens in 28.6% of the patients and a doubtful reaction in 25.0%. Serum НВсAb may serve as a diagnostic marker for HBV infection. Clinical and morphological correlations enabled the authors to state that CVHB was present in all cases in the absence of serum HBsAg in the patients. Diagnosing NAFLD requires demonstration of increased liver fat in the absence of hazardous levels of alcohol consumption. Thresholds for potentially dangerous alcohol ingestion have been set at more than one drink per day in women and two drinks per day in men based on epidemiologic evidence that the prevalence of serum aminotransferase elevations increases when alcohol consumption habitually exceeds these levels. In those studies, one drink was defined as having 10 g of ethanol and, thus, is equivalent to one can of beer, 4 ounces of wine, or 1.5 ounces (one shot) of distilled spirits. Other causes of liver fat accumulation (particularly exposure to certain drugs; Table 364-2) and liver injury (e.g., viral hepatitis, autoimmune liver disease, iron or copper overload, α1 antitrypsin deficiency) must also be excluded. Thus, establishing the diagnosis of NAFLD does not require invasive testing: it can be accomplished by history and physical examination, liver imaging (ultrasound is an Ginseng is commonly used as a medicinal herb for memory and concentration and general well-being. Drug-induced liver injury (DILI) is one of the most challenging disorders and trending events in the United States which are related to body building and weight loss supplements. Currently, herbal and dietary supplementation is the second most common cause of DILI. Here, we report on a 45-year-old healthy Chinese woman who presented with dull intermittent left upper quadrant abdomen pain for a month. Upon thorough history taking, she had been taking ginseng tea and supplementation for her menopausal symptoms for almost 3 months. Physical examination was unremarkable except mild tenderness in left upper quadrant of the abdomen. Liver function test showed aspartate transaminase (AST) 717 U/L, alanine transaminase (ALT) 343 U/L, total bilirubin 5 mg/dL, direct bilirubin 3.3 mg/dL, alkaline phosphatase 182 U/L, with international normalized ratio (INR) 1.2. Prior liver enzymes (6 months earlier) showed AST 21 U/L, ALT 18 U/L, total bilirubin 0.8 mg/dL, direct bilirubin 0.3 mg/dL, alkaline phosphatase 34 U/L, with INR 0.7. Viral serology for acute hepatitis B, C, E, cytomegalovirus, Epstein-Barr virus, and varicella zoster virus was negative. She was immune to hepatitis A. Her antinuclear antibody was positive. Her anti-Smith antibody, anti-smooth muscle antibody, HFE gene mutation, ceruloplasmin, alpha-1 antitrypsin serologies were within normal references. An abdomen sonogram showed fatty infiltration. Liver biopsy showed moderate to severe portal inflammation and marked lobular disarray. Portal and lobular inflammatory infiltrates consisted of a mixture of histiocytes, lymphocytes, plasma cells, eosinophils, and neutrophils with centrilobular necrosis and focal bridging necrosis, and necro-inflammation. After 6 weeks of follow-up, the patient improved physically, and the abdomen pain resolved. Ginseng has been widely used in the Chinese community as medicinal herb for a variety of conditions for decades. However, proper research has never been done regarding its pharmacokinetics, efficacy, and safety issues. In our case report, the idiosyncratic DILI resulted from ingestion of ginseng as herbal supplementation for premenopausal symptoms. Physicians should be aware of and suspect DILI in any patient with acute liver injury, and patients should be reminded that all medications and supplements have a potential to cause DILI. The results of the bilirubin, enzymes, and liver function tests will direct the diagnosis towards a hepatocellular or cholestatic cause and offer some idea of the duration and severity of the disease. Further evaluation can be conducted based on the initial assessment. Hepatocellular workup: viral serologies, autoimmune antibodies, serum ceruloplasmin, ferritin. A 38-year-old woman presented with general weakness and vaginal bleeding. One month prior, she had been diagnosed with Evans syndrome (haemolytic anemia with positive Coombs test and thrombocytopenia) and was given oral steroid as maintenance therapy. Her serology examination was negative for hepatitis B, hepatitis C, and human immunodeficiency virus (HIV). Her obstetrical history was marked by miscarriage in second pregnancy and preeclampsia in third pregnancy. She used hormonal contraceptives until 5 months prior to admission. On physical examination, she had anemic conjunctiva and no organomegaly. Blood tests were significant for anemia (3.4 g/dl) and thrombocytopenia (28,000/µl). Her vaginal bleeding had ceased, however her platelet continued decreasing to 12,000/µl during first several days of hospitalization despite receiving platelet transfusion. On the tenth hospital day, she suddenly complained of severe headache and blurred vision. She had bilateral edema and erythema of palpebral, chemosis, decreased in visual acuity, and reduced ocular motility. Ear and nose examination were normal. Peripheral blood smear showed no blast. Prothrombine time (PT), INR, APTT tests were normal and D Dimer was slightly increased (3.3 mg/l; NV ≤0.5 mg/l). Urine examination revealed proteinuria with 24 hour urine protein was 1,863 mg (NV <150 mg/day). We assessed her as cavernous sinus thrombosis and treated her empirically with intravenous broad-spectrum antibiotics, morphine drip. Either digital subtraction angiography or anticoagulant was deferred due to low platelet. Further examination revealed positive for ANA, anti-SSA, and diagnosis of SLE was established. Anticardiolipin antibodies of IgG and IgM and anti-beta2 glycoprotein antibodies of IgM and IgG tests were non reactive. Methylprednisolone pulse therapy (1g/day) was given for 3 consecutive days, and then tapered to oral methylprednisolone. She additionally received azathioprine 50 mg tab BID. Meanwhile her clinical symptoms alleviated and platelet count was increased, brain MRI and MR venography finally performed suggesting cerebral venous sinus thrombosis. She got additional oral anticoagulant rivaroxaban 15 mg tab BID and eventually discharged. Cerebral venous sinus thrombosis may be the presenting symptoms or occur concomitantly within the onset of SLE. Our patient had SLE, meeting 4 of the Systemic Lupus International Collaborating Clinic classification criteria (hemolytic anemia, thrombocytopenia, renal involvement, and positive for ANA test). Vasculitis due to endothelial cell injury mediated by immune-complex deposition is proposed to be the pathogenesis of CVST in SLE. Hypercoagulable state could be other etiology factor. Antiphospholipid antibodies were absent in our case as reported in some cases, emphasizing vasculitis as the underlying mechanism. Treatment of CVST in SLE consisting of anticoagulant, steroid, and immunosuppressant. This case elicits intriguing problem: CVST and thrombocytopenia. Anticoagulant treatment is proposed as the cornerstone treatment for CVST, however it was deferred due to risk of bleeding in thrombocytopenia. Steroid plays role in treatment of CVST in SLE, owing to its anti-inflammatory property. As shown in previous cases, the patient had remarkable response to high dose steroid treatment and eventually got anticoagulant after her platelet had increased. In summary, prompt diagnosis and treatment of CVST are important for a favorable prognosis.\nHere is the question:\nDIGESTIVE: A 52-year-old woman consulted because she had noticed during the previous week a yellowish discoloration of the conjunctivae. She does not refer to risky sexual behaviors or epidemiological history of risk of viral hepatitis. She does not consume alcohol or hepatotoxic drugs. She reports a one-year history of generalized pruritus, asthenia, dry mouth and absence of lacrimation with no known cause. Rest of the anamnesis without pathological data. Physical examination showed scratching lesions, conjunctival jaundice and non-painful hepatomegaly. The patient brings a blood test carried out in his company with the following pathological results: Bilirubin 3 mg/dl, FA 400 UI/ VSG 40mm 1 hour. Indicate which would be the best recommendation to establish the etiological diagnosis of the patient's condition:\nHere are the potential choices:\n1. Anti-mitochondrial antibodies.\n2. Study of Fe metabolism.\n3. Study of copper metabolism.\n4. Hepatic MRI.\n5. Serology for B and C viruses.\nThe correct answer is: ", "gold_answer": "1 Anti-mitochondrial antibodies.", "prediction": "You are a helpful medical expert, and your task is to answer a multi-choice medical question using the relevant documents. Please choose the answer from the provided options. Your responses will be used for research purposes only, so please have a definite answer.\nHere are the relevant documents:\n Epstein-Barr virus (EBV) infection typically presents with pharyngeal symptoms and subclinical transaminitis. We present a case of a 27-year-old woman with no known past medical history who presented with painless jaundice and dark-colored urine for three days. Her review of systems was negative for fever, sore throat, nausea, vomiting, pruritus, or rash. Her last sexual contact was six months ago with a male partner, and she only drank alcohol socially. Family and surgical history were non-significant. Physical examination revealed 3+ bilateral conjunctival icterus without abdominal tenderness or organomegaly. She had elevated transaminases: alanine transaminase (ALT) of 1287U/L and aspartate aminotransferase of (AST) 1057U/L but her alkaline phosphatase (ALP) was only slightly above normal at 109U/L (normal range 35-104U/L), with a direct hyperbilirubinemia - total bilirubin 9.5mg/dl, direct bilirubin 6.8mg/dl; the abdominal ultrasound revealed non-dilated bile ducts. Hepatitis A, B, and C serology was negative, but her EBV serology showed an infection. She had incidental thalassemia minor without splenomegaly or asterixis. She was managed conservatively, and her liver enzymes trended down with supportive management. Although EBV is an uncommon cause of painless jaundice, this diagnosis should be considered, especially when other more common causes of jaundice have been ruled out. A high index of suspicion should be maintained to detect EBV hepatitis as it can easily be diagnosed through serological testing. Roddick explained that her hepatitis C was unexpectedly diagnosed in 2004, following a blood test that was part of a medical examination needed for a life insurance policy. The blood test indicated abnormal liver function and subsequent blood tests diagnosed hepatitis C. Roddick explained that she had a large blood transfusion in 1971, after the birth of her younger daughter, and that she was convinced that the transfusion had infected her with hepatitis C. This was about twenty years before blood donors in the United Kingdom were screened for hepatitis C. She reported that she had developed cirrhosis of the liver, and that her main symptoms were itching and poor concentration. She briefly mentioned that medical treatment with interferon did not suit her. Roddick explained that she kept fit and active, and that she attended biannual out-patient hospital appointments in Southampton, as well as being under review by the liver transplant team at the Addenbrooke's Hospital in Cambridge. Hepatitis is a disease of the liver that has been recognised since antiquity. Symptoms include jaundice, a yellowing of the skin, eyes and body fluids. There are numerous causes, including viruses – particularly hepatitis A virus, hepatitis B virus and hepatitis C virus. Throughout history epidemics of jaundice have been reported, mainly affecting soldiers at war. This \"campaign jaundice\" was common in the Middle Ages. It occurred among Napoleon's armies and during most of the major conflicts of the 19th and 20th centuries, including the American Civil War, where over 40,000 cases and around 150 deaths were reported. The viruses that cause epidemic jaundice were not discovered until the middle of the 20th century. The names for epidemic jaundice, hepatitis A, and for blood-borne infectious jaundice, hepatitis B, were first used in 1947, following a publication in 1946 giving evidence that the two diseases were distinct. In the 1960s, the first virus that could cause hepatitis was Approach to the Patient with Liver Disease Suspected Liver Disease Abnormal liver tests Acute < 6 months Chronic > 6 months Diagnostic evaluation 1. IgM Anti-HAV 2. HBsAg 3. IgM Anti-HBc 4. Anti-HCV 5. ANA, SMA 6. Monospot, heterophile 7. Ceruloplasmin 8. Alcohol history 9. Drug history Diagnostic evaluation 1. AMA 2. Drug history 3. Ultrasound/MRI 4. MRCP/ERCP Liver biopsy in acute liver disease: Reserved for patients in whom the diagnosis remains unclear despite medical evaluation Liver biopsy in chronic liver disease: Often valuable for diagnosis as well as staging and grading liver disease Diagnostic evaluation 1. HBsAg 2. Anti-HCV 3. Fe saturation, ferritin 4. Ceruloplasmin 5. ˜1AT 6. ANA, SMA 7. Ultrasound 8. Alcohol history Diagnostic evaluation 1. Drug history 2. AMA 3. P-ANCA 4. Ultrasound 5. MRCP/ERCP Hepatitic: °°ALT Mixed: ˛ALT, ˛AlkP Cholestatic: °°AlkP, °°gGT, ˛ALT Hepatitic: °°ALT Mixed: ˛ALT, ˛AlkP Cholestatic: °°AlkP, °°gGT, ˛ALT Her social history is significant for alcohol use (three to four glasses of wine/night). Her vital signs include the following: temperature 99.8°F, blood pressure 132/64 mm Hg, pulse 78 bpm, and respiratory rate 15/min. On physical examination, she had left upper abdominal tenderness with evidence of hepatomegaly and mild scleral icterus. Laboratory data revealed the following: alanine aminotransferase, 527 IU/L (normal 10–35 IU/L); aspartate aminotransferase, 425 IU/L (normal < 35 IU/L); and bilirubin, 2.9 mg/dL (normal 0.1–0.3 mg/dL). What medications do OTC cold and flu preparations typically contain? Which of the OTC medications might have contrib-uted to the patient’s current symptoms? KH, a 55-year-old woman, presents to the emergency department with nausea, vomiting, and complaints of new-onset flu symptoms over the past several days. Her past medical history is significant for allergic rhinitis and chronic lower back pain secondary to a work-related fall 2 years ago. Her Patients usually present with varying symptoms apart from yellowish discoloration of skin along with pruritus, thus providing clues to narrow down the etiology or can also be asymptomatic. A thorough questioning regarding the use of drugs, alcohol or other toxic substances, risk factors for hepatitis (travel, unsafe sexual practices), HIV status, personal or family history of any inherited disorders or hemolytic disorders is vital. Other important points include the duration of jaundice; and the presence of any coexisting signs and symptoms, like a joint ache, rash, myalgia, changes in urine and stool. [22] A history of arthralgias and myalgias before yellowing indicates hepatitis, either due to drugs or viral infections. Phase 2 (prodromal phase) - Patients in this phase usually present with anorexia, nausea, vomiting, malaise, pruritus, urticaria, arthralgias, and fatigue. Many times these patients are misdiagnosed as having gastroenteritis or viral infection. Phase 3 (icteric phase) - Patients in this phase present with dark-colored urine and pale-colored stool. Some patients develop jaundice and right upper quadrant pain with liver enlargement. Phase 4 (convalescent phase) - Patients typically start noticing the resolution of symptoms, and laboratory studies show liver enzymes returning to normal levels. [32] Viral infections during pregnancy can pose serious threats to mother and fetus from the time of conception to the time of delivery. These lead to congenital defects, spontaneous abortion and even death. The definitive diagnosis and management of pregnancy-related viral infections may be challenging especially in less resourced countries. We present clinical and laboratory responses to the diagnosis and management of three cases of fulminant hepatitis secondary to Hepatitis E viral infection in pregnancy.Case 1 was a 31-year-old Ghanaian woman who presented with a week's history of passing dark urine as well as yellowish discoloration of the eyes. She subsequently developed fulminant hepatitis secondary to Hepatitis E viral infection, spontaneously aborted at 24 weeks of gestation and later died.Case 2 was also a 31-year-old Ghanaian woman who was admitted with a four-day history of jaundice. She had low grade fever, but no history of abdominal pain, haematuria, pale stool or pruritus. She next developed fulminant hepatitis secondary to Hepatitis E viral infection. However, she did not miscarry but died at 28 weeks of gestation.Case 3 was a 17-year-old Ghanaian woman who was referred to the tertiary health facility on account of jaundice and anaemia. She had delivered a live male infant at maturity of 32 weeks but noticed she was jaundiced and had a presentation of active disease 3 days prior to delivery. The baby was icteric at birth and on evaluation, had elevated bilirubin (mixed type) with normal liver enzymes. Hepatitis E virus infection was confirmed in both mother and baby. However, the jaundice and the hepatomegaly resolved in mother and baby after 5 and 12 days respectively. To the best of our knowledge, these are the first documented cases of fatal fulminant hepatic failures resulting from HEV infection in Ghana. Analysis of serum copper (Cu) level dynamics during the acute phase of hepatitis acute B and the early convalescence in compliance with gravity of the acute disease course. The study included 39 patients (12 men and 27 women), aged 18 to 76 years. They were hospitalised in the Department of Infectious Diseases of Medical Academy in Lublin because of the hepatitis acute B, without coexisting diseases. The diagnosis was based on the epidemiologic anamnesis, clinical symptoms, biochemical and serological examinations. The studied group was divided in respect to sex and the course of the disease as: light, medium-weighty and weighty. In all examined patients, the serum Cu level was determined according to the following scheme: at the first, tenth, twentieth and the last day of the hospitalisation and additionally one time at four weeks after discharging from the clinic. The serum Cu level was made by atomic absorption spectrometry (AAS) at the wave length of 324.8 nm. The received data were subjected to statistical analysis according to t-Student's test and in cases of significant differences according to the variants of c-Cochran and Cox's tests. According to the SI Unit Conversion Guide, the values 11.22 to 23.58 mumol/l were taken as the normal range. The values derived from the control group of 24 healthy persons (13 men and 11 women) aged 22 to 69 years. The significant increase of serum Cu level in comparison with the control values was found both in the acute phase of hepatitis B and the early convalescence. It could be observed a correlation between serum Cu level and the course of hepatitis viralis acuta B. The statistics proved that approximately 25% of the patients with acute HCV present with jaundice, and only 10-20% develop gastrointestinal symptoms. We present the case of a 58 year-old woman, with prior antecedents of arterial hypertension and diabetes mellitus since 25 years old, hypercholesterolemia and hypertriglyceridemia, psoriasis, epilepsy and depressive syndrome. She clinically presents asthenia, anorexia, itching, jaundice and choluria. The objective examination showed an orientated patient, without flapping, hemorrhagic dyscrasia or signs of chronic hepatic disease, with icteric mucosa and skin, abdominal pain, with hepatomegaly and splenomegaly. The laboratory tests have been compatible with acute hepatitis with colestatic pattern: AST/ALT 969/798 UI/ml, FA 796 UI/ml, GGT 2476 UI/ml, BT/BD 7.39/6.10, INR 0.9. The abdominal echography showed: hepatomegaly, regular borders, hepatic steatosis, splenomegaly without ascitic fluid. The viral serological tests revealed protection for hepatitis A ( IgM neg/IgG pos), negative for HVB infection (AgHBs neg, anti-HBc neg), negative for HVE and other viruses (CMV Herpes virus, Epstein Barr, HIV), positive antibodies for HCV and positive RNA VHC (164200 UI/ml), HCV genotype 3a, IL-28B CT, negative autoimmunity. The previous HCV tests were negative, sustaining the recent infection. We assumed an acute hepatitis C. The patient was symptomatically treated with hydroxyzine for the skin itch, with vitamin K for INR correction and she was closely monitored. She had good clinical and laboratorial evolution and she was discharged after one week, maintaining hepatology consultation. She spontaneously cleared HCV infection after 3 months, maintaining negative RNA VHC 6 months after infection. The patient has cured the HCV infection with no need for antiviral treatment. 302 liver cirrhoses obtained from the post-mortem material 1970 to 1979 have been examined for the occurrence of the cuprous protein complexes by means of the orcein-staining according to Shikata. The part of the B-posthepatic cirrhoses (89%) could be simultaneously determined with the help of this method. Cuprous protein complexes have been found in bilious (100%), alcoholic (43.9%), etiologically uncertain (34.4%), and B-posthepatic (25,9%) liver cirrhoses. They prove an existing or experienced chronic cholestasis. The demonstration of copper has no principal significance for the etiologic classification of cirrhoses. In the bioptic diagnostics the demonstration of copper has a certain significance for the differentiation of the primary destructive cholangitis from chronically active hepatites of other genesis. The investigations incidentally revealed that HBsAg-containing hepatocytes could be identified by means of orcein in sections performed on archive material after destaining independent of the age of the sections and their primary staining. History The Tenpō Tsūhō came around a century after the introduction of the Hōei Tsūhō (Kyūjitai: 寳永通寳 ; Shinjitai: 宝永通宝) during the 5th year of the Hōei era (1708), which had a face value of 10 mon (while only containing 3 times as much copper as a 1 mon Kan'ei Tsūhō coin), but was discontinued shortly after it started circulating as it wasn't accepted for its nominal value. A 37-year-old woman presented with increasing abdominal pain and jaundice. Six weeks before admission, she developed persistent diarrhea and jaundice of the skin. She also bruised easily, and her gums bled. In the subsequent weeks, her appetite decreased, she was fatigued, and she had nausea, vomiting, and abdominal distension. She had a history of drinking 1 quart of vodka every day for 20 years, with brief periods of abstinence; she stopped consuming alcohol 11 days before admission because it no longer provided symptomatic relief. Her past medical history was also notable for depression, including a suicide attempt 4 years earlier. She did not smoke, use illicit drugs, or have unprotected sexual intercourse. She had received no blood transfusions and had not traveled recently. She took no medications, except for occasional ibuprofen. On physical examination, she was thin and deeply jaundiced, and she trembled and responded slowly to questions. She was afebrile but tachypneic, and she had orthostatic hypotension. Her HEENT examination was notable for scleral and sublingual icterus, as well as crusted blood on her gums and teeth. The jugular veins were flat. The cardiac examination revealed tachycardia (heart rate, 103 beats per minute) without murmurs, rubs, or gallops. The abdomen was nontender and protuberant, with hypoactive bowel sounds; the spleen was not palpable, and there was no fluid wave or caput medusae. The liver percussed to 18 cm, with a smooth edge extending 10 cm below the costal margin. She had cutaneous telangiectases on her chest and bilateral palmar erythema. There was no peripheral edema. The neurologic examination was notable for asterixis. Her stool was guaiac positive. Laboratory studies revealed the following values: hematocrit, 21.2%; white blood cells, 17,310/mm(3); ammonia, 42 micromol/L; serum creatinine, 3.9 mg/dL; serum urea nitrogen, 70 mg/dL; albumin, 2.1 g/dL; total bilirubin, 26.8 mg/dL; alanine aminotransferase, 14 U/L; aspartate aminotransferase, 77 U/L; alkaline phosphatase, 138 U/L; prothrombin time, 103 seconds (international normalized ratio, 10.6); and urinary sodium, <5 mg/dL. Urinalysis revealed an elevated specific gravity and numerous muddy granular casts. Hepatitis A, B, and C serologies were negative. On abdominal ultrasound examination, there was no ascites, and the liver was echogenic. The portal and hepatic veins were patent, and the hepatic arteries were normal. The spleen measured 14 cm. What is the diagnosis? Wilson disease is a disease of excessive copper accumulation. It is associated with psychiatric disturbances due to copper accumulation in the basal ganglia. Kayser-Fleischer rings are pathognomonic for Wilson disease but are not completely sensitive (requires an expert ophthalmologist to confirm this finding). Laboratory evaluation that favors a diagnosis of Wilson disease includes low serum ceruloplasmin levels and elevated urinary copper, and if abnormal, requires further evaluation by a hepatologist. Alcoholic hepatitis Autoimmune hepatitis Cirrhosis Drug-induced liver injury Hemochromatosis Hepatitis A Hepatitis C Hepatitis D Hepatitis E Hepatocellular carcinoma Human immunodeficiency virus Wilson disease Hydralazine is a commonly used oral antihypertensive agent. We report a rare case of hydralazine-induced hepatotoxicity in the form of subacute hepatic necrosis. A 75-year-old African American woman presented with jaundice of 7-day duration. She was started on hydralazine 100 mg 3 times a day 10 weeks before presentation. On physical examination, scleral icterus was noted. Workup revealed elevated liver transaminases, alkaline phosphatase, and conjugated bilirubin. She had no history of liver disease, and liver function tests had been normal before starting hydralazine. Other etiologies, including viruses, common toxins, drugs, autoimmune, and copper-induced hepatitis, were excluded. Abdominal imaging studies did not show any evidence of intrahepatic or extrahepatic biliary ductal dilatation, and no pathologies were seen in the liver and pancreas. The patient's liver biopsy revealed extensive lobular hepatitis, significant necrosis, mixed inflammatory infiltrate, and no significant fibrosis, supporting a diagnosis of drug-induced liver injury. Hydralazine was immediately discontinued. She showed improvement of clinical and laboratory abnormalities within 5 days after discontinuation of hydralazine. To establish the diagnosis of hydralazine-induced liver injury, we used assessment tool outlined by the Council for International Organization of Medical Sciences (CIOMS) scale that led to \"high probable\" relationship. Although rare, clinically significant, and potentially life-threatening liver injury can result from use of hydralazine. Both clinical and histological presentations in our patient suggest acute liver injury. The hydralazine-induced hepatitis seems to be reversible as discontinuation of the drug improves clinical outcomes. We highly recommend monitoring of the liver function during hydralazine treatment. Occult hepatitis B (ОHB) characterized by the absence of blood HBsAg attracts the attention of specialists of different profiles; however, its clinical morphological aspects have not been practically studied. to estimate the proportion of OHB in the structure of fatal outcomes in chronic viral hepatitis (CVH) and to characterize its clinical course and structural changes on autopsy materials. A total of 455 autopsy cases of CVH were examined for its etiology in the S.P. Botkin Clinical Hospital of Infectious Diseases in 2014-2016. An in-depth prospective clinical analysis was made to investigate 28 cases of OHB in the stage of decompensated liver cirrhosis, which had subsequently culminated in death. The criteria of inclusion were history data and clinical symptoms of CVH in the detection of markers for hepatitis A, C, and D and HIV in serum HBcAb in the absence of HBsAg. HBsAbs were also determined. Along with the traditional morphological examination, immunohistochemistry (IHC) for HBsAg and HBcAg was carried out. There were 108 CVHB cases (23.7% of the total cases of CVH), including 77 OHB cases (71.3% of those of CVHB) while HBsAg was not determined. HBsAb-negative patients were more often observed to have clinical signs of jaundice (p<0.05) and skin itching (p<0.05). Dyspepsia and hemorrhagic manifestations prevailed in patients with HBsAb (more than 10 IU/l) (p<0.05). All the cases were found to have characteristic morphological signs of CVH, including intranuclear inclusions and nuclear polymorphism in 10.7% of deaths. There was an IHC-positive reaction to VHB antigens in 28.6% of the patients and a doubtful reaction in 25.0%. Serum НВсAb may serve as a diagnostic marker for HBV infection. Clinical and morphological correlations enabled the authors to state that CVHB was present in all cases in the absence of serum HBsAg in the patients. Diagnosing NAFLD requires demonstration of increased liver fat in the absence of hazardous levels of alcohol consumption. Thresholds for potentially dangerous alcohol ingestion have been set at more than one drink per day in women and two drinks per day in men based on epidemiologic evidence that the prevalence of serum aminotransferase elevations increases when alcohol consumption habitually exceeds these levels. In those studies, one drink was defined as having 10 g of ethanol and, thus, is equivalent to one can of beer, 4 ounces of wine, or 1.5 ounces (one shot) of distilled spirits. Other causes of liver fat accumulation (particularly exposure to certain drugs; Table 364-2) and liver injury (e.g., viral hepatitis, autoimmune liver disease, iron or copper overload, α1 antitrypsin deficiency) must also be excluded. Thus, establishing the diagnosis of NAFLD does not require invasive testing: it can be accomplished by history and physical examination, liver imaging (ultrasound is an Ginseng is commonly used as a medicinal herb for memory and concentration and general well-being. Drug-induced liver injury (DILI) is one of the most challenging disorders and trending events in the United States which are related to body building and weight loss supplements. Currently, herbal and dietary supplementation is the second most common cause of DILI. Here, we report on a 45-year-old healthy Chinese woman who presented with dull intermittent left upper quadrant abdomen pain for a month. Upon thorough history taking, she had been taking ginseng tea and supplementation for her menopausal symptoms for almost 3 months. Physical examination was unremarkable except mild tenderness in left upper quadrant of the abdomen. Liver function test showed aspartate transaminase (AST) 717 U/L, alanine transaminase (ALT) 343 U/L, total bilirubin 5 mg/dL, direct bilirubin 3.3 mg/dL, alkaline phosphatase 182 U/L, with international normalized ratio (INR) 1.2. Prior liver enzymes (6 months earlier) showed AST 21 U/L, ALT 18 U/L, total bilirubin 0.8 mg/dL, direct bilirubin 0.3 mg/dL, alkaline phosphatase 34 U/L, with INR 0.7. Viral serology for acute hepatitis B, C, E, cytomegalovirus, Epstein-Barr virus, and varicella zoster virus was negative. She was immune to hepatitis A. Her antinuclear antibody was positive. Her anti-Smith antibody, anti-smooth muscle antibody, HFE gene mutation, ceruloplasmin, alpha-1 antitrypsin serologies were within normal references. An abdomen sonogram showed fatty infiltration. Liver biopsy showed moderate to severe portal inflammation and marked lobular disarray. Portal and lobular inflammatory infiltrates consisted of a mixture of histiocytes, lymphocytes, plasma cells, eosinophils, and neutrophils with centrilobular necrosis and focal bridging necrosis, and necro-inflammation. After 6 weeks of follow-up, the patient improved physically, and the abdomen pain resolved. Ginseng has been widely used in the Chinese community as medicinal herb for a variety of conditions for decades. However, proper research has never been done regarding its pharmacokinetics, efficacy, and safety issues. In our case report, the idiosyncratic DILI resulted from ingestion of ginseng as herbal supplementation for premenopausal symptoms. Physicians should be aware of and suspect DILI in any patient with acute liver injury, and patients should be reminded that all medications and supplements have a potential to cause DILI. The results of the bilirubin, enzymes, and liver function tests will direct the diagnosis towards a hepatocellular or cholestatic cause and offer some idea of the duration and severity of the disease. Further evaluation can be conducted based on the initial assessment. Hepatocellular workup: viral serologies, autoimmune antibodies, serum ceruloplasmin, ferritin. A 38-year-old woman presented with general weakness and vaginal bleeding. One month prior, she had been diagnosed with Evans syndrome (haemolytic anemia with positive Coombs test and thrombocytopenia) and was given oral steroid as maintenance therapy. Her serology examination was negative for hepatitis B, hepatitis C, and human immunodeficiency virus (HIV). Her obstetrical history was marked by miscarriage in second pregnancy and preeclampsia in third pregnancy. She used hormonal contraceptives until 5 months prior to admission. On physical examination, she had anemic conjunctiva and no organomegaly. Blood tests were significant for anemia (3.4 g/dl) and thrombocytopenia (28,000/µl). Her vaginal bleeding had ceased, however her platelet continued decreasing to 12,000/µl during first several days of hospitalization despite receiving platelet transfusion. On the tenth hospital day, she suddenly complained of severe headache and blurred vision. She had bilateral edema and erythema of palpebral, chemosis, decreased in visual acuity, and reduced ocular motility. Ear and nose examination were normal. Peripheral blood smear showed no blast. Prothrombine time (PT), INR, APTT tests were normal and D Dimer was slightly increased (3.3 mg/l; NV ≤0.5 mg/l). Urine examination revealed proteinuria with 24 hour urine protein was 1,863 mg (NV <150 mg/day). We assessed her as cavernous sinus thrombosis and treated her empirically with intravenous broad-spectrum antibiotics, morphine drip. Either digital subtraction angiography or anticoagulant was deferred due to low platelet. Further examination revealed positive for ANA, anti-SSA, and diagnosis of SLE was established. Anticardiolipin antibodies of IgG and IgM and anti-beta2 glycoprotein antibodies of IgM and IgG tests were non reactive. Methylprednisolone pulse therapy (1g/day) was given for 3 consecutive days, and then tapered to oral methylprednisolone. She additionally received azathioprine 50 mg tab BID. Meanwhile her clinical symptoms alleviated and platelet count was increased, brain MRI and MR venography finally performed suggesting cerebral venous sinus thrombosis. She got additional oral anticoagulant rivaroxaban 15 mg tab BID and eventually discharged. Cerebral venous sinus thrombosis may be the presenting symptoms or occur concomitantly within the onset of SLE. Our patient had SLE, meeting 4 of the Systemic Lupus International Collaborating Clinic classification criteria (hemolytic anemia, thrombocytopenia, renal involvement, and positive for ANA test). Vasculitis due to endothelial cell injury mediated by immune-complex deposition is proposed to be the pathogenesis of CVST in SLE. Hypercoagulable state could be other etiology factor. Antiphospholipid antibodies were absent in our case as reported in some cases, emphasizing vasculitis as the underlying mechanism. Treatment of CVST in SLE consisting of anticoagulant, steroid, and immunosuppressant. This case elicits intriguing problem: CVST and thrombocytopenia. Anticoagulant treatment is proposed as the cornerstone treatment for CVST, however it was deferred due to risk of bleeding in thrombocytopenia. Steroid plays role in treatment of CVST in SLE, owing to its anti-inflammatory property. As shown in previous cases, the patient had remarkable response to high dose steroid treatment and eventually got anticoagulant after her platelet had increased. In summary, prompt diagnosis and treatment of CVST are important for a favorable prognosis.\nHere is the question:\nDIGESTIVE: A 52-year-old woman consulted because she had noticed during the previous week a yellowish discoloration of the conjunctivae. She does not refer to risky sexual behaviors or epidemiological history of risk of viral hepatitis. She does not consume alcohol or hepatotoxic drugs. She reports a one-year history of generalized pruritus, asthenia, dry mouth and absence of lacrimation with no known cause. Rest of the anamnesis without pathological data. Physical examination showed scratching lesions, conjunctival jaundice and non-painful hepatomegaly. The patient brings a blood test carried out in his company with the following pathological results: Bilirubin 3 mg/dl, FA 400 UI/ VSG 40mm 1 hour. Indicate which would be the best recommendation to establish the etiological diagnosis of the patient's condition:\nHere are the potential choices:\n1. Anti-mitochondrial antibodies.\n2. Study of Fe metabolism.\n3. Study of copper metabolism.\n4. Hepatic MRI.\n5. Serology for B and C viruses.\nThe correct answer is: 1. Anti-mitochondrial antibodies." }, { "question": "You are a helpful medical expert, and your task is to answer a multi-choice medical question using the relevant documents. Please choose the answer from the provided options. Your responses will be used for research purposes only, so please have a definite answer.\nHere are the relevant documents:\n The aim of this retrospective study was to analyze the characteristics of delayed panfacial fractures and evaluate treatment results. Thirty-three patients with delayed panfacial fractures were treated in the Maxillofacial Trauma Center of Peking University, School and Hospital of Stomatology between 1998 and 2004. Each patient was examined by computed tomography (CT) scans before operation. For those who had no severe opening restriction, dental impressions were taken to fabricate dental casts. For those with severely comminuted fractures, 3-dimensional (3D) models of the facial skeleton were used. Re-establishing the continuity of the mandible was the first step and then used as a platform to reconstruct the maxillary fractures via maxillomandibular fixation after Le Fort I osteotomy. The third step was to restore the mid- and upper-facial width and projection by coronal approach to expose the zygomatic complex and frontal bone/sinus and/or naso-orbito-ethmoid (NOE) fractures. There were 3 types of mandibular fractures that affected the treatment plan: 1) type I, mandibular body/symphysis fracture(s) (17/33, 51.52%); 2) type II, mandibular angle and/or condylar fracture(s) (6/33, 18.18%); and 3) type III, both mandibular body/symphysis and angle/condylar fractures (10/33, 30.30%). Fourteen cases were associated with NOE fractures (42.42%) and 3 cases had frontal sinus fractures (9.1%). Twelve cases had enophthalmos (36.36%) and 3 lost 1 eyeball. The order of treatment was dependent on the mandibular fracture type. For type I fractures, reconstructing the mandibular arch was the first step. For type II fractures, repairing the angle, ascending rami, and condylar areas was the first step. For type III fractures, when both mandibular height and arch were disrupted, freeing the malunited angle or condyle was the first step before restoring the mandibular arch form. Reconstruction of the mandibular height and projection was then carried out. For all 3 types, the second step was to restore the mid- and upper facial width and projection by reducing the zygomatic complex and frontal bone/sinus or NOE fractures. Maxillary fixation across the Le Fort I level was the last step. Le Fort I osteotomy was used for all 33 cases. Bone grafts and soft tissue suspension also were used. Twenty-one cases (63.64%) had good results, 7 (21.21%) cases were acceptable, and 5 (15.15%) were not good. There were 7 cases (21.21%) that still had soft tissue problems that needed secondary operations. Reconstruction of the mandible first with Le Fort I osteotomy is a good way to treat delayed panfacial fractures. Computed tomography and 3D CT, model surgery, and occasionally 3D models are necessary aids for diagnosis and treatment. Soft tissue problems, including lacerations and asymmetries, were often the factors that caused an unfavorable outcome. A prospective study on mid-face fractures was carried out in the Department of Oral and Maxillofacial Surgery at College of Dentistry, Indore, from August 2007 to September 2009 to analyze etiology, incidence and patterns of midface fractures and associated ocular injuries. Two hundred patients were included in this study, amongst those who reported to the Department of OMFS, College of Dentistry, Indore. After confirmed diagnosis of mid face fracture all the patients were stratified according to age, sex, cause of the accident, influence of alcohol, location, type of fractures and associated ocular injuries. The study included 200 patients with a mean age of 29.6 years. The most frequently injured patients belonged to the 21-30 year-old age group. The male predilection was 76 %. Road traffic accident was the most common causative factor (64 %), followed by assault (21 %), cases of fall (9.5 %) and other causes (5.5 %). The most common fracture in this study was found to be zygomatic complex fractures (62.5 %) (more in the age group of 21-30 years). This was followed by Lefort II fractures (23 %), multiple fractures (10 %) and Lefort I fractures (6 %), Lefort III fractures (4.5 %) and Naso-ethmoidal fractures (4 %) in descending order. 84.5 % subjects were having ocular involvement. Subconjunctival hemorrhage was present mostly in 83.5 % followed by remaining as corneal injury 15 %, reduced acuity 11.5 %, diplopia 10.5 %, enophthalmos 8.5 %, telecanthus 5 %, hyphema 3.5 %, blindness 3 % and proptosis 0.5 %. Zygomatic complex fractures were the most frequent type of injury that was complicated by blindness or a serious eye injury (61 %). Collection of data regarding the epidemiology of maxillofacial fractures is important because it may assist healthcare providers to provide necessary information for the development and evaluation of preventive measures. Ocular injuries should have an early ophthalmological examination at the time of trauma to detect any kind of ocular dysfunction. The worldwide population is increasingly aging. Maxillofacial fractures of the geriatric population have been increased. Evaluation of the demographic variables, causes and the patterns of maxillofacial traumas in the elderly population is the main aim of this study. Seven hundred thirteen maxillofacial tomography images which were scanned between 2010 and 2019 were evaluated. Data from 50 patients aged 65 years old and/or older, who were treated for maxillofacial fracture at the Department of Otorhinolaryngology, Gaziantep University, were retrospectively analyzed. Two groups were created according to the facial fracture pattern. Facial fractures were reclassified into 2 groups; mandibula, orbital, zygomaticomaxillary complex group fractures and the other group of frontal, naso-orbito-ethmoid fractures and were used as a comparison. The mean age of the patients was 72.5 (min 65- max 93). The gender distribution was 17 females (34%) and 33 males (66%). The most common fractured bone was the nasal bone and the least one is the frontal bone. Approximately one-quarter of 50 fractures were seen in 70 to 79 years old. Falling is more common in females and men are more prone to work-related accidents than home-related accidents. Facial fractures in the elderly often seen in midface location. Falling is the common etiology of facial fracture in all genders at elderly. However, male dominance is seen in other etiological factors. Additional diseases in the elderly seem to increase the severity of facial fracture. To determine the incidence and types of ophthalmic complications associated with maxillofacial trauma over a period of 24 months. An institutional prospective study was conducted on 62 patients presenting with maxillofacial trauma to study the correlation between facial trauma and ophthalmic complications. Road traffic accidents were reported to be the primary etiologic factor for most trauma cases studied. Zygomaticomaxillary complex (ZMC) fracture was associated with more ophthalmic complications while fractures involving the orbital rims and walls were associated with severe complications. Maxillofacial trauma, particularly those associated with midface, including ZMC fracture, Le Fort II, Le Fort III, and naso-orbito-ethmoidal fractures, can commonly cause ophthalmic complications and blindness in rare cases. Hence, every patient with maxillofacial trauma should undergo an ophthalmic examination and should be placed under close observation for necessary treatment when required. The upper midface area comprises mainly the naso-orbito-ethmoidal (NOE) region which plays a paramount role in facial expression. Fractures of this area often result in neglected bony defects in the fragile periorbital region with major secondary impairments such as traumatic telecanthus, orbital dystopia, and/or enophthalmos. Permanent cranial nerve deficits also can occur as the result of post-traumatic/post-operative sequelae. Seventy-one patients (age range 7-78 years) with severe high midface trauma, treated from January 1989 to December 1996, were reviewed with a minimum follow-up of 2 years. The patient population has been distributed according to the fracture type in three groups: Group 1 (n = 35): Isolated NOE with/without associated central midface injury; Group 2 (n = 22): NOE associated with craniofacial injury and Group 3 (n = 14): NOE associated with orbital displacement. The estimated post-surgical parameters included qualitative and quantitative data from the long-term clinical evaluation. Persistent headache and/or concentration difficulties were mainly noted in Group 1. Smell reduction or anosmia was reported mainly in Group 2. Deficits of the trigeminal and/or the facial nerve were found in Group 3. Enophthalmos and/or telecanthus were predominantly seen with injuries associated with orbital displacement. The purpose of this study was to report on the pattern of occurrence of nasoorbitoethmoid (NOE) fractures in Odisha and the various factors that influence their distribution. The study period was from January 1, 2016 to December 15, 2017. After approval from the Institutional Ethics Committee, all patients diagnosed with naso-orbito-ethmoid fractures reporting to the department of OMFS and Level-1 trauma centers were included in the study. Sociodemographic data along with the etiology and type of fracture were mentioned. Associated injuries to other body parts were noted. Open reduction was possible only in five cases of NOE fractures. The treatment plan including the operative approach and postoperative results was evaluated. A total of 1192 patients with facial fracture were seen, of which 52 (4.36%) patients had NOE fractures. Males far outnumbered females in a ratio of 9:1. Thirty-three patients (63.46%) had unilateral NOE fracture, while the rest 19 (36.54%) had bilateral NOE fracture. Sixteen (30.76%) cases were classified as Type I, 35 (67.30%) as Type II, and 1 (1.92%) as Type III. Road traffic accidents were the most common cause of NOE fractures (69%), followed by fall (17%) and assault (10%). The most common neurological injury to be associated with NOE fractures was pneumocephalus (29%), followed by diffuse axonal injury (8%). Telecanthus (100%) was found to be the primary clinical feature in patients of NOE fracture, followed by a depressed nasal bridge (92%). Fracture of the nasal bone was invariably associated with NOE fracture. Complications observed due to untreated NOE fractures included a shortened and retruded nose, shortened palpebral fissures, telecanthus, and enophthalmos. Contemporary management of NOE complex fractures demands precise diagnosis and immediate surgical management with anatomic reduction and rigid fixation of the involved bone segments. With an improvement in socioeconomic status and increased awareness among maxillofacial surgeons, hopefully, a greater number of NOE fracture patients will avail the benefits of open reduction in future. The incidence of maxillofacial injuries is on the rise due to motor vehicle accidents and increased incidence of violence in recent times. The aim of this retrospective study was to determine the incidence, aetiology, the pattern of fractures, their management with open reduction and internal fixation (ORIF) and complications, if any. A retrospective analysis of 621 fractures in 361 patients managed by ORIF over a four year period was carried out. The average age of patients was 24.3 years with a male to female ratio of 21.2:1. Panfacial fractures comprised 4.7%, frontal bone fractures 8.9%, orbital fractures 0.7%, naso-orbito-ethmoid complex (NOE) fractures 0.7%, zygomatic complex fractures 23.5%, fracture maxilla 11.5% and mandibular fractures 52.2% of all facial fractures. All the cases were successfully managed by ORIF under general anaesthesia (GA). Complications were noticed in 6.8% of cases in the form of reactive implants in 3.6%, deranged occlusion in 1% and infection at operated site in 1% cases which were managed satisfactorily. The findings of this study reveal sharp annual increase in the number of cases of maxillofacial trauma. Road traffic accidents (RTA) were the commonest cause and the age group most affected was between 20-25 years. ORIF of these fractures was chosen for its obvious advantages of direct anatomical reduction, early return to function and minimal complications. Orbital fractures are classified as diseases usually related to common midface trauma. It represents the most challenging treatment due to the complex anatomy, physiology, and aesthetic role. A midface trauma involves also the zygomatic complex and the nose, however the orbit fracture seems to be a more frequent disease due to its anatomical features. The purpose of this work is to retrospectively evaluate and record the frequency of the midfacial traumas and orbital fractures observed in the North Eastern Sicily. The results of the present data may be useful for the clinicians in order to recognize the kind of fracture just from the first general visit having a quick diagnosis and management. In the years between 2001 and 2016, about 1200 patients with midfacial trauma and about 100 patients involving the orbital floor have been evaluated. All those patients underwent the surgical fracture reduction and a CT scan follow up control at one month, three months, six months and one year. Data showed high percentage of orbital floor, nose and mandibular body and ramus fractures; moreover the most frequent causes of fractures seem to be related to motor vehicle accident, followed by assaults, work and fall. The results have highlighted the changing trends in the causes of facial injuries, particularly the increasing incidence of assaults and the falling incidence of motor vehicle accidents in developed countries. The quick diagnosis and management proved fundamental for the successful treatment. Clinicians should be able to recognize the first symptoms in order to avoid possible complications. A considerable number of patients experiencing facial trauma are diagnosed with blowout fracture. Preoperative computed tomographic scan is often different from the actual surgical area. This study is restricted to orbital floor fracture. This study is expected to help speculating fracture site and making surgical plans according to symptoms of periorbital trauma. From March 2005 to September 2013, a total of 150 cases of orbital floor fracture surgeries have been analyzed. This study analyzed the preoperative symptoms at the certain fractured area of orbital floor, at the aspects of sagittal view of computed tomography, which is sectioned into anterior one-third, middle one-third, posterior one-third, and mixed types. Symptoms for analysis are diplopia, extraocular movement limitation, enophthalmos and other combined facial bone fractures, and the like. Fracture areas of orbital floor are 21 cases (14%) of anterior one-third, 47 cases (31%) of middle one-third, 7 cases (5%) of posterior one-third, and 75 cases (50%) of the mixed. Frequency of diplopia was 0 case, 24 cases (42.1%), 4 cases (7.0%), and 29 cases (50.9%), respectively. In the case of extraocular movement limitation, 0 case, 15 cases (39.5%), 2 cases (5.3%), and 21 cases (55.2%) were found, respectively. In the case of enophthalmos, 0 case, 5 cases (16.7%), 7 cases (23.3%), and 18 cases (60.0%) were found, respectively. The most commonly associated other facial bone fractures were nasal bone fractures. In the case of blowout fracture, diplopia, extraocular movement limitation, enophthalmos, and other symptoms are checked through physical examination. This study would help speculating fracture site and making surgical plans according to symptoms of periorbital trauma. To explore the secondary surgical reconstruction for orbital bone deformities accompanied with canthus dislocation after trauma. From June 1998 to July 2007, 37 patients with secondary orbital bone fracture deformity accompanied with medial or lateral canthal ligament dislocation posttraumatically were treated, among whom there were 22 males and 15 females, aged 13-46 years old (21 on average). There were 29 cases of traffic accident, 6 of boxing injury and 2 of beating injury by sticks. The latest reconstruction was performed on these 37 cases during 3 months to 8 years after injuries. There were 11 cases of orbital maxillary zygoma (OMZ) fracture, 15 of naso-orbito-ethmoid (NOE) fracture, 8 of OMZ and NOE fracture and 3 of frontal fracture. There were 31 patients who were reconstructed for the first time and 6 for the second time. Typical bicoronal and subciliary incisions and intra-oral approach were employed to expose all the fractured sites. According to the fractured position and the degree of deformity and dislocation, the orbito-zygomatic fracture was repositioned after osteotomy and rigid fixation, or the healed fragments were trimmed with a burr and the depressed fragments were filled with autogenous bone such as ilium, cranial outer table or Medpor in order to reconstruct orbital wall framework; the orbital walls were repaired to correct the enophthalmos with autogenous bone or Medpor after the herniated orbital contents were released. The medial canthal ligament was anchored superior-posteriorly to the lacrimal fossa with transnasal wires fixation or fixed with titanium miniplates and nails. The 36 patients' incisions obtained healing by first intention after the operation, and 1 case failed because of wound infection from maxillary sinusitis. There were 24 patients who were cured successfully with facial appearance and function improved significantly. During the follow-up for 3-6 months, no complication was found such as dislocation of the implant, rejection and infection. Two patients still showed slight enophthalmos while 3 patients with canthus dislocation regained improved appearances but not satisfactory. At 6 months after operation, the CT scan conducted in 3 patients with autogenous bone and Medpor grafting showed all fractures were fixed rigidly. Surgical reduction combined with bone grafting is a satisfactory method for the correction of secondary orbital bone deformity, and the repair of canthus dislocation and correction of enophthalmos should be considered at the same time. An ideal result could be achieved only through all-round consideration and comprehensive treatment. Trauma is the fourth major cause of mortality in the Western countries, of which approximately one half involve maxillofacial injury. Statistics reported by emergency room officials show motor vehicles cause many of the injuries and deaths that occur in Iran. Having completed a retrospective descriptive study of 200 patients who experienced maxillofacial trauma, the authors report its occurrence with respect to age, sex, trauma type, and site of injury so as to evaluate the operational functionality of the Department of Otolaryngology and Head & Neck Surgery of Hazrat-e Rasoul Akram Hospital from 2000 to 2004. Mandibular fractures (36.2%) occurred in the subsequently listed sites and at the specified frequencies: mandibular angle (9.7%), mandible body (6.9%), parasymphysis, ramus and subcondyle at 5.6% each, and symphysis at 2.8%. No condylar fractures were reported. Frontal bone fracture was observed in 9.7% of the patients with eye globe injury occurring simultaneously in 8.3% of corresponding cases. Orbital fracture (63.9%) also occurred in various cases as follows: orbital floor 39%, lateral rim 24%, inferior rim 22%, medial wall 11%, and the superior rim and orbital roof at 2% each. Motor vehicle accidents were the most common causes of trauma (42%). The most common fracture was in the zygoma (43%) with 8.3% of them being orbital injury. Fractures of mandibular bones (36.2%) and the maxilla (33%) were the most commonly seen in trauma occurring to the maxillomandibular region. Mandibular condyle fractures develop frequently and show the variable type of injury and complication. New opinions have emerged from recent investigation into condylar fractures. The author investigated 246 patients with condylar fractures who visited SNUDH from January 1980 to August, 1988, 8. with regard to clinical and treatment aspects, area and displacement of fractures, associated teeth injury and other body injury, complications. At last I have got the following results. 1. The incidence to condylar fractures in a series of 765 mandibular fractures may be as high as 32.2%. 2. The male patients are 3 times more than female patients. The highest frequency was recorded in the group 21-30 years of age. (34.1%). 3. Falls caused the greatest number of condylar fractures (45.2%) and next was in assult (25.6%), traffic accidents (22.4%). 4. Unilateral condylar fractures were present in 74.8%, giving a left: right ratio of 1.2:1. In cases of unilateral fracture, subcondylar fractures were by far the commonest (32.9%) but in cases of bilateral fracture, condylar neck fractures were by far the commonest. In children under 15 years of age, condylar neck fractures were more common but in patients over 16 years of age, subcondylar fractures were common. 5. Anteromedial fracture dislocations were by far the commonest (20.3%). In children under 15 years of age, fracture deviations were common but in patients over 16 years of age, fracture displacements were common. 6. 44.7% of patients with condylar fractures sustained the teeth injuries. Teeth fractures were by far the commonest. 7. Single condylar fractures showed a frequency of 30.5%. Of the concomitant fractures elsewhere in the mandible, symphysis fractures were by far the commonest (54.1%). 8. Associated other body injuries showed a frequency of 28.0%. Of them, head injuries were by far the commonest. 9. The mean interval from injury to treatment was 14.3 days. Of the treatment of condylar fractures, open reduction was by far the commonest (70.3%). Closed reduction comprised 19.9% and functional therapy comprised 8.5%. 10. In 67 patients with possible follow up period, the following complications were developed, two ankylosis, anterior open bite, mouth opening limitation, mouth opening deviation. Morbidity and mortality among children is usually the result of trauma. Because a child's face is retruded relative to the protecting skull, has a thicker layer of adipose tissue, more elastic bones, flexible sutures lines, the presence of tooth buds within the jaws, and the lack of pneumatisation of the sinuses, the facial bones fracture less commonly than in adults. Our aim was to assess the patterns of such fractures in children who presented to the department of Oral and Maxillofacial Surgery, King Edward Medical University/Mayo Hospital Lahore, Pakistan. All 535 eligible children between the ages of 1-16 years who presented during the two years December 2009 - December 2011 were included in the study. Facial fractures were diagnosed by clinical examination, plain radiographs, and computed tomography, and the pattern of fractures of the facial bones including the frontal bone, orbital bones, maxilla, zygoma, naso-orbito-ethmoidal complex, mandible, and dentoalveolar region was documented. The male:female ratio was 2:1 with 369 male (70%) and 166 female (31%) patients. Fall was the cause in 212 (39%), and in 167 (31%) it was road traffic accidents, while sports were the cause in 135 (25%). The naso-orboto-ethmoid complex was fractured in 37 cases (7%) while 104 children (19%) presented with isolated fractures of the zygomatic bone. The maxilla was fractured in 195 cases (36%), the mandible in 380 (71%), and dentoalveolar trauma was the cause in 256 (50%). The mandible was the bone that was most often fractured (mostly in boys and usually as a result of falls during summer vacations), with the peak occurring in those aged 8-12 years. Given the variability of the timing and order of surgeries, it is difficult to choose the best treatment for patients with complex facial fractures. Based on the clinical experiences, the authors have reviewed their experience with the timing and order of operations depending on the sites of complex facial fractures and their concurrent injuries. The current study was based on a total of 105 patients with complex facial fractures from the year 2002 to 2011. After assessing the patients' clinical records, radiological data, and clinical photographs, the following data were analyzed: patients' age and sex, causes of injury, concurrent injuries, sites of fractures, the interval between trauma and the operations, the presence of additional surgeries, and the aesthetic and functional outcomes.For most of the patients, early operation was performed (within 2 weeks in 95.2%). Additional surgeries within 1 month after injuries were performed in 22 patients. Usually, a top-to-bottom direction repair was applied when head injuries were involved, and bottom-to-top direction repair was applied when occlusal problems were involved. Of 105 patients whom we were able to follow up, 49 patients showed complications or were dissatisfied with the outcomes. However, except them, most of the patients were satisfied with the outcomes of surgical treatments. There were 14 cases of cheek asymmetry, 9 enophthalmos, 30 paresthesia, 4 malocclusion, and a single case of persistent trismus.In the current study, satisfactory results could be achievable under the following principles: a repair should be done in the early stage after the onset of the injury; supportive surgeries should be done, if necessary, within 2 weeks (no later than 4 weeks); and the order of surgical treatment should be determined by the severity of bone fracture and the systemic status. Damages to the middle third of the facial bone generally involve the orbital skeleton and can lead to eye impairment. In this study, it is attempted to determine the incidence of ophthalmic injuries in maxillofacial trauma with zygomatic bone fractures. One hundred and fifteen cases with ophthalmic (ocular) involvement after maxillofacial trauma were referred to the Shariati Hospital, Tehran, Iran, and were visited at the Ophthalmology Department between 2016 and 2018. Zygomatic fractures and resulting ocular complications were evaluated in 87 males and 28 females with the mean ages of 26 and 32 years, respectively. Subconjunctival ecchymosis was detected in 23.07% of men and 21.05% of women. Displacement of the palpebral fissure was detected in 26.5% of men and 27.6% of women. Furthermore, the unequal pupillary level was observed in 18.37% of men and 15.78% of women. Diplopia was detected in 8.9% of men and 10.5% of women. Additionally, enophthalmos was observed in 23.1% of men and 25% of women. The most common ocular presentations in midfacial trauma are diplopia and reduced visual acuity. Even after the operation, a significant number of patients experience poor vision and diplopia. Ophthalmology consultation is essential for these patients. Mental health issues, as patients with facial injuries, are at greater risk of developing post-traumatic stress disorder or anxiety-related disorders, particularly those who were victims of assault. [31] With an increased popularity of sport and active living worldwide, our study aims to explore the incidence and features of sports-related orbital fractures in Singapore. 1421 computer tomography (CT) imaging scans of the face and orbits done at the National University Hospital over a 24-month period from January 2013 and December 2014 were reviewed retrospectively for orbital fractures. We identified 483 orbital fractures of which sports injury was the fourth most common etiology (n = 65; 13.5%) after road traffic accident (n = 131; 27.1%), geriatric fall (n = 81; 16.8%) and workplace injury (n = 67; 13.9%). The three most common sport in orbital fractures were soccer (n = 20; 30.8%), bicycling (n = 11; 16.9%) and jogging (n = 8; 12.3%). The three most common fracture patterns were zygomatico-maxillary complex fractures (n = 24; 36.9%), isolated one wall blowout fractures (n = 19; 29.2%) and naso-orbito-ethmoid fractures (n = 7; 10.8%). Sports-related orbital fractures were associated with a low mean age of patients (45.9 years, range, 14-79 years), a higher proportion of males (n = 58; 89.2%) than that from geriatric falls (n = 37, 45.6%) (P < 0.01), a higher likelihood of unilaterality (n = 62; 95.4%) than that from traffic accidents (n = 99; 75.6%) (P < 0.01) and a lower likelihood of pan-facial involvement (n = 4; 6.15%) than that from traffic accident (n = 60; 45.8%) (P < 0.01). Sports-related orbital fractures are the fourth most common cause of orbital fractures. Though commonly seen in young male adults, in view of the aging population and people exercising more regularly, education of safety measures among sports users is paramount to preventing sports-related orbital fractures.\nHere is the question:\nOPHTHALMOLOGY (ECTOPIC): 20-year-old patient who comes to the emergency department after suffering a bicycle accident with facial trauma. A cranial CT scan was performed showing a fracture of the middle third of the face involving the orbito-malar region. One of the most frequent complications of this type of fracture is:\nHere are the potential choices:\n1. Temporomandibular ankylosis.\n2. Dental malocclusion.\n3. Naso-ethmoidal pseudoarthrosis.\n4. Enophthalmos.\nThe correct answer is: ", "gold_answer": "4 Enophthalmos.", "prediction": "You are a helpful medical expert, and your task is to answer a multi-choice medical question using the relevant documents. Please choose the answer from the provided options. Your responses will be used for research purposes only, so please have a definite answer.\nHere are the relevant documents:\n The aim of this retrospective study was to analyze the characteristics of delayed panfacial fractures and evaluate treatment results. Thirty-three patients with delayed panfacial fractures were treated in the Maxillofacial Trauma Center of Peking University, School and Hospital of Stomatology between 1998 and 2004. Each patient was examined by computed tomography (CT) scans before operation. For those who had no severe opening restriction, dental impressions were taken to fabricate dental casts. For those with severely comminuted fractures, 3-dimensional (3D) models of the facial skeleton were used. Re-establishing the continuity of the mandible was the first step and then used as a platform to reconstruct the maxillary fractures via maxillomandibular fixation after Le Fort I osteotomy. The third step was to restore the mid- and upper-facial width and projection by coronal approach to expose the zygomatic complex and frontal bone/sinus and/or naso-orbito-ethmoid (NOE) fractures. There were 3 types of mandibular fractures that affected the treatment plan: 1) type I, mandibular body/symphysis fracture(s) (17/33, 51.52%); 2) type II, mandibular angle and/or condylar fracture(s) (6/33, 18.18%); and 3) type III, both mandibular body/symphysis and angle/condylar fractures (10/33, 30.30%). Fourteen cases were associated with NOE fractures (42.42%) and 3 cases had frontal sinus fractures (9.1%). Twelve cases had enophthalmos (36.36%) and 3 lost 1 eyeball. The order of treatment was dependent on the mandibular fracture type. For type I fractures, reconstructing the mandibular arch was the first step. For type II fractures, repairing the angle, ascending rami, and condylar areas was the first step. For type III fractures, when both mandibular height and arch were disrupted, freeing the malunited angle or condyle was the first step before restoring the mandibular arch form. Reconstruction of the mandibular height and projection was then carried out. For all 3 types, the second step was to restore the mid- and upper facial width and projection by reducing the zygomatic complex and frontal bone/sinus or NOE fractures. Maxillary fixation across the Le Fort I level was the last step. Le Fort I osteotomy was used for all 33 cases. Bone grafts and soft tissue suspension also were used. Twenty-one cases (63.64%) had good results, 7 (21.21%) cases were acceptable, and 5 (15.15%) were not good. There were 7 cases (21.21%) that still had soft tissue problems that needed secondary operations. Reconstruction of the mandible first with Le Fort I osteotomy is a good way to treat delayed panfacial fractures. Computed tomography and 3D CT, model surgery, and occasionally 3D models are necessary aids for diagnosis and treatment. Soft tissue problems, including lacerations and asymmetries, were often the factors that caused an unfavorable outcome. A prospective study on mid-face fractures was carried out in the Department of Oral and Maxillofacial Surgery at College of Dentistry, Indore, from August 2007 to September 2009 to analyze etiology, incidence and patterns of midface fractures and associated ocular injuries. Two hundred patients were included in this study, amongst those who reported to the Department of OMFS, College of Dentistry, Indore. After confirmed diagnosis of mid face fracture all the patients were stratified according to age, sex, cause of the accident, influence of alcohol, location, type of fractures and associated ocular injuries. The study included 200 patients with a mean age of 29.6 years. The most frequently injured patients belonged to the 21-30 year-old age group. The male predilection was 76 %. Road traffic accident was the most common causative factor (64 %), followed by assault (21 %), cases of fall (9.5 %) and other causes (5.5 %). The most common fracture in this study was found to be zygomatic complex fractures (62.5 %) (more in the age group of 21-30 years). This was followed by Lefort II fractures (23 %), multiple fractures (10 %) and Lefort I fractures (6 %), Lefort III fractures (4.5 %) and Naso-ethmoidal fractures (4 %) in descending order. 84.5 % subjects were having ocular involvement. Subconjunctival hemorrhage was present mostly in 83.5 % followed by remaining as corneal injury 15 %, reduced acuity 11.5 %, diplopia 10.5 %, enophthalmos 8.5 %, telecanthus 5 %, hyphema 3.5 %, blindness 3 % and proptosis 0.5 %. Zygomatic complex fractures were the most frequent type of injury that was complicated by blindness or a serious eye injury (61 %). Collection of data regarding the epidemiology of maxillofacial fractures is important because it may assist healthcare providers to provide necessary information for the development and evaluation of preventive measures. Ocular injuries should have an early ophthalmological examination at the time of trauma to detect any kind of ocular dysfunction. The worldwide population is increasingly aging. Maxillofacial fractures of the geriatric population have been increased. Evaluation of the demographic variables, causes and the patterns of maxillofacial traumas in the elderly population is the main aim of this study. Seven hundred thirteen maxillofacial tomography images which were scanned between 2010 and 2019 were evaluated. Data from 50 patients aged 65 years old and/or older, who were treated for maxillofacial fracture at the Department of Otorhinolaryngology, Gaziantep University, were retrospectively analyzed. Two groups were created according to the facial fracture pattern. Facial fractures were reclassified into 2 groups; mandibula, orbital, zygomaticomaxillary complex group fractures and the other group of frontal, naso-orbito-ethmoid fractures and were used as a comparison. The mean age of the patients was 72.5 (min 65- max 93). The gender distribution was 17 females (34%) and 33 males (66%). The most common fractured bone was the nasal bone and the least one is the frontal bone. Approximately one-quarter of 50 fractures were seen in 70 to 79 years old. Falling is more common in females and men are more prone to work-related accidents than home-related accidents. Facial fractures in the elderly often seen in midface location. Falling is the common etiology of facial fracture in all genders at elderly. However, male dominance is seen in other etiological factors. Additional diseases in the elderly seem to increase the severity of facial fracture. To determine the incidence and types of ophthalmic complications associated with maxillofacial trauma over a period of 24 months. An institutional prospective study was conducted on 62 patients presenting with maxillofacial trauma to study the correlation between facial trauma and ophthalmic complications. Road traffic accidents were reported to be the primary etiologic factor for most trauma cases studied. Zygomaticomaxillary complex (ZMC) fracture was associated with more ophthalmic complications while fractures involving the orbital rims and walls were associated with severe complications. Maxillofacial trauma, particularly those associated with midface, including ZMC fracture, Le Fort II, Le Fort III, and naso-orbito-ethmoidal fractures, can commonly cause ophthalmic complications and blindness in rare cases. Hence, every patient with maxillofacial trauma should undergo an ophthalmic examination and should be placed under close observation for necessary treatment when required. The upper midface area comprises mainly the naso-orbito-ethmoidal (NOE) region which plays a paramount role in facial expression. Fractures of this area often result in neglected bony defects in the fragile periorbital region with major secondary impairments such as traumatic telecanthus, orbital dystopia, and/or enophthalmos. Permanent cranial nerve deficits also can occur as the result of post-traumatic/post-operative sequelae. Seventy-one patients (age range 7-78 years) with severe high midface trauma, treated from January 1989 to December 1996, were reviewed with a minimum follow-up of 2 years. The patient population has been distributed according to the fracture type in three groups: Group 1 (n = 35): Isolated NOE with/without associated central midface injury; Group 2 (n = 22): NOE associated with craniofacial injury and Group 3 (n = 14): NOE associated with orbital displacement. The estimated post-surgical parameters included qualitative and quantitative data from the long-term clinical evaluation. Persistent headache and/or concentration difficulties were mainly noted in Group 1. Smell reduction or anosmia was reported mainly in Group 2. Deficits of the trigeminal and/or the facial nerve were found in Group 3. Enophthalmos and/or telecanthus were predominantly seen with injuries associated with orbital displacement. The purpose of this study was to report on the pattern of occurrence of nasoorbitoethmoid (NOE) fractures in Odisha and the various factors that influence their distribution. The study period was from January 1, 2016 to December 15, 2017. After approval from the Institutional Ethics Committee, all patients diagnosed with naso-orbito-ethmoid fractures reporting to the department of OMFS and Level-1 trauma centers were included in the study. Sociodemographic data along with the etiology and type of fracture were mentioned. Associated injuries to other body parts were noted. Open reduction was possible only in five cases of NOE fractures. The treatment plan including the operative approach and postoperative results was evaluated. A total of 1192 patients with facial fracture were seen, of which 52 (4.36%) patients had NOE fractures. Males far outnumbered females in a ratio of 9:1. Thirty-three patients (63.46%) had unilateral NOE fracture, while the rest 19 (36.54%) had bilateral NOE fracture. Sixteen (30.76%) cases were classified as Type I, 35 (67.30%) as Type II, and 1 (1.92%) as Type III. Road traffic accidents were the most common cause of NOE fractures (69%), followed by fall (17%) and assault (10%). The most common neurological injury to be associated with NOE fractures was pneumocephalus (29%), followed by diffuse axonal injury (8%). Telecanthus (100%) was found to be the primary clinical feature in patients of NOE fracture, followed by a depressed nasal bridge (92%). Fracture of the nasal bone was invariably associated with NOE fracture. Complications observed due to untreated NOE fractures included a shortened and retruded nose, shortened palpebral fissures, telecanthus, and enophthalmos. Contemporary management of NOE complex fractures demands precise diagnosis and immediate surgical management with anatomic reduction and rigid fixation of the involved bone segments. With an improvement in socioeconomic status and increased awareness among maxillofacial surgeons, hopefully, a greater number of NOE fracture patients will avail the benefits of open reduction in future. The incidence of maxillofacial injuries is on the rise due to motor vehicle accidents and increased incidence of violence in recent times. The aim of this retrospective study was to determine the incidence, aetiology, the pattern of fractures, their management with open reduction and internal fixation (ORIF) and complications, if any. A retrospective analysis of 621 fractures in 361 patients managed by ORIF over a four year period was carried out. The average age of patients was 24.3 years with a male to female ratio of 21.2:1. Panfacial fractures comprised 4.7%, frontal bone fractures 8.9%, orbital fractures 0.7%, naso-orbito-ethmoid complex (NOE) fractures 0.7%, zygomatic complex fractures 23.5%, fracture maxilla 11.5% and mandibular fractures 52.2% of all facial fractures. All the cases were successfully managed by ORIF under general anaesthesia (GA). Complications were noticed in 6.8% of cases in the form of reactive implants in 3.6%, deranged occlusion in 1% and infection at operated site in 1% cases which were managed satisfactorily. The findings of this study reveal sharp annual increase in the number of cases of maxillofacial trauma. Road traffic accidents (RTA) were the commonest cause and the age group most affected was between 20-25 years. ORIF of these fractures was chosen for its obvious advantages of direct anatomical reduction, early return to function and minimal complications. Orbital fractures are classified as diseases usually related to common midface trauma. It represents the most challenging treatment due to the complex anatomy, physiology, and aesthetic role. A midface trauma involves also the zygomatic complex and the nose, however the orbit fracture seems to be a more frequent disease due to its anatomical features. The purpose of this work is to retrospectively evaluate and record the frequency of the midfacial traumas and orbital fractures observed in the North Eastern Sicily. The results of the present data may be useful for the clinicians in order to recognize the kind of fracture just from the first general visit having a quick diagnosis and management. In the years between 2001 and 2016, about 1200 patients with midfacial trauma and about 100 patients involving the orbital floor have been evaluated. All those patients underwent the surgical fracture reduction and a CT scan follow up control at one month, three months, six months and one year. Data showed high percentage of orbital floor, nose and mandibular body and ramus fractures; moreover the most frequent causes of fractures seem to be related to motor vehicle accident, followed by assaults, work and fall. The results have highlighted the changing trends in the causes of facial injuries, particularly the increasing incidence of assaults and the falling incidence of motor vehicle accidents in developed countries. The quick diagnosis and management proved fundamental for the successful treatment. Clinicians should be able to recognize the first symptoms in order to avoid possible complications. A considerable number of patients experiencing facial trauma are diagnosed with blowout fracture. Preoperative computed tomographic scan is often different from the actual surgical area. This study is restricted to orbital floor fracture. This study is expected to help speculating fracture site and making surgical plans according to symptoms of periorbital trauma. From March 2005 to September 2013, a total of 150 cases of orbital floor fracture surgeries have been analyzed. This study analyzed the preoperative symptoms at the certain fractured area of orbital floor, at the aspects of sagittal view of computed tomography, which is sectioned into anterior one-third, middle one-third, posterior one-third, and mixed types. Symptoms for analysis are diplopia, extraocular movement limitation, enophthalmos and other combined facial bone fractures, and the like. Fracture areas of orbital floor are 21 cases (14%) of anterior one-third, 47 cases (31%) of middle one-third, 7 cases (5%) of posterior one-third, and 75 cases (50%) of the mixed. Frequency of diplopia was 0 case, 24 cases (42.1%), 4 cases (7.0%), and 29 cases (50.9%), respectively. In the case of extraocular movement limitation, 0 case, 15 cases (39.5%), 2 cases (5.3%), and 21 cases (55.2%) were found, respectively. In the case of enophthalmos, 0 case, 5 cases (16.7%), 7 cases (23.3%), and 18 cases (60.0%) were found, respectively. The most commonly associated other facial bone fractures were nasal bone fractures. In the case of blowout fracture, diplopia, extraocular movement limitation, enophthalmos, and other symptoms are checked through physical examination. This study would help speculating fracture site and making surgical plans according to symptoms of periorbital trauma. To explore the secondary surgical reconstruction for orbital bone deformities accompanied with canthus dislocation after trauma. From June 1998 to July 2007, 37 patients with secondary orbital bone fracture deformity accompanied with medial or lateral canthal ligament dislocation posttraumatically were treated, among whom there were 22 males and 15 females, aged 13-46 years old (21 on average). There were 29 cases of traffic accident, 6 of boxing injury and 2 of beating injury by sticks. The latest reconstruction was performed on these 37 cases during 3 months to 8 years after injuries. There were 11 cases of orbital maxillary zygoma (OMZ) fracture, 15 of naso-orbito-ethmoid (NOE) fracture, 8 of OMZ and NOE fracture and 3 of frontal fracture. There were 31 patients who were reconstructed for the first time and 6 for the second time. Typical bicoronal and subciliary incisions and intra-oral approach were employed to expose all the fractured sites. According to the fractured position and the degree of deformity and dislocation, the orbito-zygomatic fracture was repositioned after osteotomy and rigid fixation, or the healed fragments were trimmed with a burr and the depressed fragments were filled with autogenous bone such as ilium, cranial outer table or Medpor in order to reconstruct orbital wall framework; the orbital walls were repaired to correct the enophthalmos with autogenous bone or Medpor after the herniated orbital contents were released. The medial canthal ligament was anchored superior-posteriorly to the lacrimal fossa with transnasal wires fixation or fixed with titanium miniplates and nails. The 36 patients' incisions obtained healing by first intention after the operation, and 1 case failed because of wound infection from maxillary sinusitis. There were 24 patients who were cured successfully with facial appearance and function improved significantly. During the follow-up for 3-6 months, no complication was found such as dislocation of the implant, rejection and infection. Two patients still showed slight enophthalmos while 3 patients with canthus dislocation regained improved appearances but not satisfactory. At 6 months after operation, the CT scan conducted in 3 patients with autogenous bone and Medpor grafting showed all fractures were fixed rigidly. Surgical reduction combined with bone grafting is a satisfactory method for the correction of secondary orbital bone deformity, and the repair of canthus dislocation and correction of enophthalmos should be considered at the same time. An ideal result could be achieved only through all-round consideration and comprehensive treatment. Trauma is the fourth major cause of mortality in the Western countries, of which approximately one half involve maxillofacial injury. Statistics reported by emergency room officials show motor vehicles cause many of the injuries and deaths that occur in Iran. Having completed a retrospective descriptive study of 200 patients who experienced maxillofacial trauma, the authors report its occurrence with respect to age, sex, trauma type, and site of injury so as to evaluate the operational functionality of the Department of Otolaryngology and Head & Neck Surgery of Hazrat-e Rasoul Akram Hospital from 2000 to 2004. Mandibular fractures (36.2%) occurred in the subsequently listed sites and at the specified frequencies: mandibular angle (9.7%), mandible body (6.9%), parasymphysis, ramus and subcondyle at 5.6% each, and symphysis at 2.8%. No condylar fractures were reported. Frontal bone fracture was observed in 9.7% of the patients with eye globe injury occurring simultaneously in 8.3% of corresponding cases. Orbital fracture (63.9%) also occurred in various cases as follows: orbital floor 39%, lateral rim 24%, inferior rim 22%, medial wall 11%, and the superior rim and orbital roof at 2% each. Motor vehicle accidents were the most common causes of trauma (42%). The most common fracture was in the zygoma (43%) with 8.3% of them being orbital injury. Fractures of mandibular bones (36.2%) and the maxilla (33%) were the most commonly seen in trauma occurring to the maxillomandibular region. Mandibular condyle fractures develop frequently and show the variable type of injury and complication. New opinions have emerged from recent investigation into condylar fractures. The author investigated 246 patients with condylar fractures who visited SNUDH from January 1980 to August, 1988, 8. with regard to clinical and treatment aspects, area and displacement of fractures, associated teeth injury and other body injury, complications. At last I have got the following results. 1. The incidence to condylar fractures in a series of 765 mandibular fractures may be as high as 32.2%. 2. The male patients are 3 times more than female patients. The highest frequency was recorded in the group 21-30 years of age. (34.1%). 3. Falls caused the greatest number of condylar fractures (45.2%) and next was in assult (25.6%), traffic accidents (22.4%). 4. Unilateral condylar fractures were present in 74.8%, giving a left: right ratio of 1.2:1. In cases of unilateral fracture, subcondylar fractures were by far the commonest (32.9%) but in cases of bilateral fracture, condylar neck fractures were by far the commonest. In children under 15 years of age, condylar neck fractures were more common but in patients over 16 years of age, subcondylar fractures were common. 5. Anteromedial fracture dislocations were by far the commonest (20.3%). In children under 15 years of age, fracture deviations were common but in patients over 16 years of age, fracture displacements were common. 6. 44.7% of patients with condylar fractures sustained the teeth injuries. Teeth fractures were by far the commonest. 7. Single condylar fractures showed a frequency of 30.5%. Of the concomitant fractures elsewhere in the mandible, symphysis fractures were by far the commonest (54.1%). 8. Associated other body injuries showed a frequency of 28.0%. Of them, head injuries were by far the commonest. 9. The mean interval from injury to treatment was 14.3 days. Of the treatment of condylar fractures, open reduction was by far the commonest (70.3%). Closed reduction comprised 19.9% and functional therapy comprised 8.5%. 10. In 67 patients with possible follow up period, the following complications were developed, two ankylosis, anterior open bite, mouth opening limitation, mouth opening deviation. Morbidity and mortality among children is usually the result of trauma. Because a child's face is retruded relative to the protecting skull, has a thicker layer of adipose tissue, more elastic bones, flexible sutures lines, the presence of tooth buds within the jaws, and the lack of pneumatisation of the sinuses, the facial bones fracture less commonly than in adults. Our aim was to assess the patterns of such fractures in children who presented to the department of Oral and Maxillofacial Surgery, King Edward Medical University/Mayo Hospital Lahore, Pakistan. All 535 eligible children between the ages of 1-16 years who presented during the two years December 2009 - December 2011 were included in the study. Facial fractures were diagnosed by clinical examination, plain radiographs, and computed tomography, and the pattern of fractures of the facial bones including the frontal bone, orbital bones, maxilla, zygoma, naso-orbito-ethmoidal complex, mandible, and dentoalveolar region was documented. The male:female ratio was 2:1 with 369 male (70%) and 166 female (31%) patients. Fall was the cause in 212 (39%), and in 167 (31%) it was road traffic accidents, while sports were the cause in 135 (25%). The naso-orboto-ethmoid complex was fractured in 37 cases (7%) while 104 children (19%) presented with isolated fractures of the zygomatic bone. The maxilla was fractured in 195 cases (36%), the mandible in 380 (71%), and dentoalveolar trauma was the cause in 256 (50%). The mandible was the bone that was most often fractured (mostly in boys and usually as a result of falls during summer vacations), with the peak occurring in those aged 8-12 years. Given the variability of the timing and order of surgeries, it is difficult to choose the best treatment for patients with complex facial fractures. Based on the clinical experiences, the authors have reviewed their experience with the timing and order of operations depending on the sites of complex facial fractures and their concurrent injuries. The current study was based on a total of 105 patients with complex facial fractures from the year 2002 to 2011. After assessing the patients' clinical records, radiological data, and clinical photographs, the following data were analyzed: patients' age and sex, causes of injury, concurrent injuries, sites of fractures, the interval between trauma and the operations, the presence of additional surgeries, and the aesthetic and functional outcomes.For most of the patients, early operation was performed (within 2 weeks in 95.2%). Additional surgeries within 1 month after injuries were performed in 22 patients. Usually, a top-to-bottom direction repair was applied when head injuries were involved, and bottom-to-top direction repair was applied when occlusal problems were involved. Of 105 patients whom we were able to follow up, 49 patients showed complications or were dissatisfied with the outcomes. However, except them, most of the patients were satisfied with the outcomes of surgical treatments. There were 14 cases of cheek asymmetry, 9 enophthalmos, 30 paresthesia, 4 malocclusion, and a single case of persistent trismus.In the current study, satisfactory results could be achievable under the following principles: a repair should be done in the early stage after the onset of the injury; supportive surgeries should be done, if necessary, within 2 weeks (no later than 4 weeks); and the order of surgical treatment should be determined by the severity of bone fracture and the systemic status. Damages to the middle third of the facial bone generally involve the orbital skeleton and can lead to eye impairment. In this study, it is attempted to determine the incidence of ophthalmic injuries in maxillofacial trauma with zygomatic bone fractures. One hundred and fifteen cases with ophthalmic (ocular) involvement after maxillofacial trauma were referred to the Shariati Hospital, Tehran, Iran, and were visited at the Ophthalmology Department between 2016 and 2018. Zygomatic fractures and resulting ocular complications were evaluated in 87 males and 28 females with the mean ages of 26 and 32 years, respectively. Subconjunctival ecchymosis was detected in 23.07% of men and 21.05% of women. Displacement of the palpebral fissure was detected in 26.5% of men and 27.6% of women. Furthermore, the unequal pupillary level was observed in 18.37% of men and 15.78% of women. Diplopia was detected in 8.9% of men and 10.5% of women. Additionally, enophthalmos was observed in 23.1% of men and 25% of women. The most common ocular presentations in midfacial trauma are diplopia and reduced visual acuity. Even after the operation, a significant number of patients experience poor vision and diplopia. Ophthalmology consultation is essential for these patients. Mental health issues, as patients with facial injuries, are at greater risk of developing post-traumatic stress disorder or anxiety-related disorders, particularly those who were victims of assault. [31] With an increased popularity of sport and active living worldwide, our study aims to explore the incidence and features of sports-related orbital fractures in Singapore. 1421 computer tomography (CT) imaging scans of the face and orbits done at the National University Hospital over a 24-month period from January 2013 and December 2014 were reviewed retrospectively for orbital fractures. We identified 483 orbital fractures of which sports injury was the fourth most common etiology (n = 65; 13.5%) after road traffic accident (n = 131; 27.1%), geriatric fall (n = 81; 16.8%) and workplace injury (n = 67; 13.9%). The three most common sport in orbital fractures were soccer (n = 20; 30.8%), bicycling (n = 11; 16.9%) and jogging (n = 8; 12.3%). The three most common fracture patterns were zygomatico-maxillary complex fractures (n = 24; 36.9%), isolated one wall blowout fractures (n = 19; 29.2%) and naso-orbito-ethmoid fractures (n = 7; 10.8%). Sports-related orbital fractures were associated with a low mean age of patients (45.9 years, range, 14-79 years), a higher proportion of males (n = 58; 89.2%) than that from geriatric falls (n = 37, 45.6%) (P < 0.01), a higher likelihood of unilaterality (n = 62; 95.4%) than that from traffic accidents (n = 99; 75.6%) (P < 0.01) and a lower likelihood of pan-facial involvement (n = 4; 6.15%) than that from traffic accident (n = 60; 45.8%) (P < 0.01). Sports-related orbital fractures are the fourth most common cause of orbital fractures. Though commonly seen in young male adults, in view of the aging population and people exercising more regularly, education of safety measures among sports users is paramount to preventing sports-related orbital fractures.\nHere is the question:\nOPHTHALMOLOGY (ECTOPIC): 20-year-old patient who comes to the emergency department after suffering a bicycle accident with facial trauma. A cranial CT scan was performed showing a fracture of the middle third of the face involving the orbito-malar region. One of the most frequent complications of this type of fracture is:\nHere are the potential choices:\n1. Temporomandibular ankylosis.\n2. Dental malocclusion.\n3. Naso-ethmoidal pseudoarthrosis.\n4. Enophthalmos.\nThe correct answer is: 4. Enophthalmos." }, { "question": "You are a helpful medical expert, and your task is to answer a multi-choice medical question using the relevant documents. Please choose the answer from the provided options. Your responses will be used for research purposes only, so please have a definite answer.\nHere are the relevant documents:\n Tendinopathy, a type of tendon disorder that results in pain, swelling, and impaired function. The pain is typically worse with movement. It most commonly occurs around the shoulder (rotator cuff tendinitis, biceps tendinitis), elbow (tennis elbow, golfer's elbow), wrist, hip, knee (jumper's knee, popliteus tendinopathy), or ankle (Achilles tendinitis). Causes may include an injury or repetitive activities. Groups at risk include people who do manual labor, musicians, and athletes. Less common causes include infection, arthritis, gout, thyroid disease, and diabetes. Diagnosis is typically based on symptoms, examination, and occasionally medical imaging. A few weeks following an injury little inflammation remains, with the underlying problem related to weak or disrupted tendon fibrils. X-rays are indicated to evaluate chronic hand pain and hip pain thought to be due to OA, as the diagnosis is often unclear without confirming radiographs. For knee pain, x-rays should be obtained if symptoms or signs are not typical of OA or if knee pain persists after inauguration of effective treatment. In OA, radiographic findings (Fig. 394-7) correlate poorly with the presence and severity of pain. Further, radiographs may be normal in early disease as they are insensitive to cartilage loss and other early findings. Although MRI may reveal the extent of pathology in an osteoarthritic joint, it is not indicated as part of the diagnostic workup. Findings such as meniscal tears andcartilage and bone lesions occurin most patients with OA in the knee, but almost never warrant a change in therapy. Knee pain affects approximately 25% of adults, and its prevalence has increased almost 65% over the past 20 years, accounting for nearly 4 million primary care visits annually. Initial evaluation should emphasize excluding urgent causes while considering the need for referral. Key aspects of the patient history include age; location, onset, duration, and quality of pain; associated mechanical or systemic symptoms; history of swelling; description of precipitating trauma; and pertinent medical or surgical history. Patients requiring urgent referral generally have severe pain, swelling, and instability or inability to bear weight in association with acute trauma or have signs of joint infection such as fever, swelling, erythema, and limited range of motion. A systematic approach to examination of the knee includes inspection, palpation, evaluation of range of motion and strength, neurovascular testing, and special (provocative) tests. Radiographic imaging should be reserved for chronic knee pain (more than six weeks) or acute traumatic pain in patients who meet specific evidence-based criteria. Musculoskeletal ultrasonography allows for detailed evaluation of effusions, cysts (e.g., Baker cyst), and superficial structures. Magnetic resonance imaging is rarely used for patients with emergent cases and should generally be an option only when surgery is considered or when a patient experiences persistent pain despite adequate conservative treatment. When the initial history and physical examination suggest but do not confirm a specific diagnosis, laboratory tests can be used as a confirmatory or diagnostic tool. There are several reports of symptomatic ganglion cysts near the anterior cruciate ligament (ACL), posterior cruciate ligament (PCL), and lateral and medial meniscus, but symptomatic ganglia arising from the anterior horn of the medial meniscus to the ACL have not been reported. Here we report the arthroscopic resection of a ganglion cyst arising from the anterior horn of the medial meniscus with a meniscal tear to the ACL. A 43-year-old female presented with a 10-year history of continuous aching pain in the right knee, but without any history of trauma. Clinical examination revealed right-sided knee pain in the medial joint line, exacerbated by end range flexion and extension, a -10°-100° active range of movement, and a -5°-110° passive range of movement。McMurray's, patellar compression, and compression rotation tests were positive. Magnetic resonance imaging (MRI) and arthroscopic examination revealed a cyst related to the ACL and medial meniscus. Histological examination confirmed the cyst to be a ganglion cyst. We present a new type of ganglion cyst, this is the first reported case of an ganglion cyst impinged between the ACL and the medial meniscus. It is hoped that this study will provide a better understanding of the condition and lead to better diagnosis and treatment. Gonalgia is a frequent reason for consultation of a primary care physician. The road leading to diagnosis is mainly clinical. A detailed medical history and physical examination are capital for establishing diagnostic hypotheses and choosing the most appropriate imaging test. Initially, a simple X-ray of the knee joint is the most common exam, even though it is not always needed, especially after a minor trauma. MRI and CT-scan allow a more detailed examination of the structures; however, they should only be ordered to answer a specific question. Most of the time, echography is reserved to extra-articular pathologies and for guiding an articular tap. Additional imaging techniques may possess greater diagnostic sensitivity and facilitate early diagnosis in a limited number of articular disorders and in selected circumstances and are indicated when conventional radiography is inadequate or nondiagnostic (Table 393-5). Ultrasonography is useful in the detection of soft tissue abnormalities, such as tendinitis, tenosynovitis, enthesitis, bursitis, and entrapment neuropathies. Wider use, lower cost, better technology, and enhanced site-specific transducers now allow for routine use in outpatient care. Owing to low cost, portability, and wider use, ultrasound use has grown and is the preferred method for the evaluation of synovial (Baker’s) cysts, rotator cuff tears, tendinitis and tendon injury, and Approach to Articular and Musculoskeletal Disorders Strongly consider synovial fluid aspiration and analysis if there is Trauma with joint effusion Monarthritis in a patient with chronic polyarthritis A thorough history and physical exam (with a focused musculoskeletal exam) should be performed on all patients, with some findings summarized above. OA is a clinical diagnosis and can be diagnosed with confidence if the following are present: 1) pain worse with activity and better with rest, 2) age more than 45 years, 3) morning stiffness lasting less than 30 minutes, 4) bony joint enlargement, and 5) limitation in range of motion. A differential diagnosis should include rheumatoid arthritis, psoriatic arthritis, crystalline arthritis, hemochromatosis, bursitis, avascular necrosis, tendinitis, radiculopathy, among other soft tissue abnormalities. [9] [10] Calcific tendinitis most commonly occurs to the shoulder, but may also involve other structures of the locomotor system. It is a rare cause of knee pain. We report a 46-year-old woman with severe medial knee pain and limitation of movement in her right knee. There was a marked tenderness site at the proximal insertion of the medial collateral ligament (MCL). Flexion was able to provoke the painful symptoms in the medial knee. The involvement of differentiated diagnoses were excluded by history, laboratory and radiograph examinations, while X-ray, CT and MRI suggested calcific tendonitis of the MCL. Due to the failure of conservative treatments, we offered her arthroscopic excision of calcific deposit which was sent for biopsy. Histopathological evaluation confirmed the diagnosis of calcific tendinitis. This patient recovered shortly afterwards with immediate resolution of symptoms following excision. Thus far, calcifications involving the MCL have been documented thrice. Calcific tendonitis of the MCL diagnosed and treated by arthroscopy has not previously been reported, which can be challenging to diagnose and treat because of its rarity. Although conservative treatment appears to be frequently satisfactory, arthroscopic excision may be a better option for the refractory or severe cases. Etymology The word patella originated in the late 17th century from the diminutive form of Latin or or paten, meaning shallow dish. See also Patellar reflex Knee pain Osteoarthritis Lateral retinaculum Lateral release References External links Knee Sesamoid bones Bones of the lower limb The unicompartimental knee prosthesis known as \"Oxford\" is a non constraint prosthesis, entrusting the whole of its stability to an intact ligamentary apparatus. Where the support surfaces of most prostheses remain limited, even punctiform, the originality of the Goodfellow prosthesis lies in the fact that the prosthetic condyle, whatever the flexion angle is, leans against a mobile prosthetic meniscus with spheric superior concavity of the same radius as the condylian radius, which increases considerably the prosthetic leaning surfaces and therefore lessens the pressure constraints. The superior surface, concave, of this prosthetic meniscus takes charge of the rolling, where the inferior plane surface realizes the gliding on the metallic tibial plate. The total conformity of the components minimizes the forces of friction.Between July 1988 and March 1993, 24 patients underwent the placing of UCP. Three patients died and 2 were lost of sight. 19 patients could be seen again or checked, corresponding of 21 operated knees. Two knees benefited from the start from UCP (medial and lateral) and 2 knees had a UCP in the first instance and then a second UCP in the compartment left safe primarily. For the 21 UCP, there are 16 medial and 3 lateral. Our mean drawback is of 3 years and 3 months, all the drawbacks being superior to 1 year and 4 months. The mean age is of 64 years. There were 17 female and 2 male patients. The mean weight is of nearly 80 kg (79,8) and nearly 52% of the operated patients have an important overweight (Body Mass Index superior to 30). Preoperative clinical analysis. It is based on a retrospective study of files using the quotation described by AUBRIOT for the «GUEPAR» group. This one establishes a gradation of four levels for each of the three criteria retained (Pain, Mobility, Instability), thus determining a global result imposed by the lowest level retained.For walking, other factors than just the state of the operated knee may intervene, this being the reason why it doesn't show in this chart. The GUEPAR group quantifies it with letters A, B, C, D.Concerning pain, all 21 knees were quoted as \"Bad\" in preoperative. Pain constitutes the decisive argument for the operative indication. In our series, only one knee had an average amplitude, all the others had a mobility superior to 89°. In 5 cases there was a flessum between 11 and 20° (penalizing of a level). Concerning walking and stability, they were taken into account, thanks to a precise questionnaire about the daily life acts. Concerning the walking perimeter, it was found as unlimited (A) in 1 case, superior to 500 m (B) in 2 cases, inferior to 500 m (C) in 17 cases and limited to home (D) in 1 case. The early after effects. At the end of the intervention, the knee is placed into a splint with limited flexion. As soon as the second day the patient is sat on the border of his bed. The first partial support at the third of the body weight is authorized between the fourth and the fifth day, when at the same time flexion exercises on electrical splint are started, as soon as the Redon draining is removed. The average hospitalization length was of a fortnight. Among secondary late complications and retakes, let us stop on meniscal luxations which constitute a specific complication of the Oxford arthroplasty. They concern 3 times the medial compartment and 4 lateral compartment. They happened in 1 case early, at D 22, in 3 cases within the 6 first months and in 3 cases after 2 years. They were treated : 3 times by reduction under general anæsthetic, no more ; 3 times changing the meniscusus for a meniscusus of superior size and once by placing a total prosthesis at the place of the UCP. The deteriorations of the opposed compartment not prosthesized occured in three cases. They were treated by unicompartmental additional arthroplasty in two cases and by total prosthesis in the third case. The clinical results on pain are very satisfactory as from the early check up onwards we have 17 successes (no pain 11 cases and occasional pains 6 cases) and as after 3 years and 5 months in average, we have 19 successes (no pain : 10 cases - occasional pain : 9 cases). At the maximal drawback, the mobility is quoted very good in 7 cases and good in 13 cases, mean in 1 case. At the latest check up, we note an excellent stability in 17 cases and good in 3 cases, that is to say 20 successes and 1 case of stability quoted as mean. At the latest check up we note 17 successes (A and B) and 4 relative failures (C) concerning the quality of walking.At the question «are you pleased with the intervention and would you advise it to a friend?» and with the nuance «very pleased» and «simply satisfied», we get 10 cases «very pleased», 8 cases «pleased» and 3 cases «moderately satisfied»; only those 3 cases advise against the intervention. The radiological results are less satisfying as they show frequent imperfections : • for the 16 medial UCP : only 9 cases hypocorrected or normo axed, but 1 case strongly hypocorrected (residual varus of 7°) and 6 hypercorrected cases. • for the 5 lateral UCP : 3 normo-axed cases, 1 case strongly hypocorrected (residual valgus of 6°) and 1 case strongly hypercorrected (10° varus). • the failures due to rapid deterioration of the non prosthetized compartment occurred on hypercorrected knees. • on 21 knees, 14 borders of tibial plate were noticed, out of which 9 had no plate displacement and 5 had a slight displacement, at the origin of a small angular loss. • accumulations of cement on the tibial side, towards the back or in medial were noticed in 8 cases, which explains a slope of the tibial plate to the back inferior to 5° in 11 cases (should be of 7°). • 4 femoral components seem to be too posterior and one shows curved.In total, only 7 cases out of 21 were estimated with no peculiarities on the radiological point of view. It seems difficult to place a UCP well. The meniscal luxations are favored by an alignment rotational defect of the tibial plate, specially for the lateral UCP, the meniscus coming to hit the lip of the tibial plate during the lifting from a sitting position. For 5 of these luxations, we must recognize the existence of a ligamentary collateral laxity which should have altered the surgical indication either to an osteotomy, or to a total arthroplasty. Conclusions. Under the condition of respecting the absolute counter indications, of thoroughly evaluating the relative counter indications and of reducing at the best the defects linked to the surgical technique, the unicompartmental arthroplasty, including that of Oxford, gives good functional results after more than three years. In our series, the result on pain is constant if we exclude the cases with risk with ligamentary laxity and that of centered gonarthrosis at obese subject, that is to say 15 successes on 15 knees thus selected retrospectively. The gain on mobility is weak, of 5° in average. The result on stability is, as for pain, excellent, if we exclude the cases with risk, as we get then also 15 successes on 15 knees. Concerning the global result according to the quotation of Aubriot-Guepar, we note 14 successes and 1 relative failure. 4 knees were bad indications and should have benefited from a total arthroplasty or from an osteotomy. Brodie's abscess is an uncommon form of subacute osteomyelitis where the main presenting symptom is mild to moderate pain of insidious onset for several months' duration. We report a case of a patient presenting with acute leg pain resembling that of a deep vein thrombosis, and a beginning leg compartment syndrome following a suspected ruptured Baker's cyst. Our case is unusual because of the acute presentation of the Brodie's abscess with acute leg pain and acute swelling without any preceding trauma; to the best of our knowledge, this presentation has not been reported before. A 17-year-old white boy presented to our out-patient clinic with a 6-month history of pain in his left knee joint of insidious onset. There was no history of trauma to the extremity. After performing physical and radiological (X-ray) examinations, we initially diagnosed medial meniscus damage. One week later he presented to our emergency department with acute sudden increase in the pain and swelling of his left knee, and pain and swelling of his left leg, without any trauma. Deep vein thrombosis and beginning leg compartment syndrome from ruptured Baker's cyst were initially diagnosed. Magnetic resonance imaging was performed and Brodie's abscess was the most probable diagnosis. We performed open surgical debridement and curettage with drainage of the abscess and administered postoperative antibiotics. He presented to our out-patient clinic 3 months postoperatively, where he was pain-free with no residual local tenderness. In cases of sudden acute increase in joint or extremity pain or swelling that has been insidiously present for months, Brodie's abscess should be considered as one of the differential diagnoses, as it may present acutely in cases with accompanying fasciitis and myositis and be clinically mistaken for deep vein thrombosis or limb compartment. Magnetic resonance imaging remains the gold standard imaging study, and surgical treatment followed by postoperative antibiotics remains the standard treatment. Popliteus tendinopathies are rare injuries that can occur from overuse, trauma, or secondary causes, such as sesamoid bones or calcifications. They present with nonspecific symptoms and should be considered in any patient with posterolateral knee pain, instability, popliteus tenderness, and a positive Garrick test. Diagnosis can be made with magnetic resonance imaging, but arthroscopy remains the criterion standard. For minor popliteus tendinopathies, initial management involves conservative treatment, including rest, activity modification, physical therapy, and quadriceps strengthening. For more severe or refractory disease, corticosteroid injections and arthroscopy should be considered. [ 1 is 90% sensitivity in predicting negative balance (> 78-mmol/day sodium excretion). Diuretic resistance: Diuretic resistance can be predicted by giving 80 mg intravenous furosemide after 3 days without diuretics and on an 80 mEq sodium/day diet. The urinary sodium excretion over 8 hours < 50 mEq/8 hours predicts resistance. Clinical use Loop diuretics are principally used in the following indications: Heart failure - Giving 2.5 times of previous oral dose twice daily for those with acute decompensated heart failure is a reasonable strategy. However, daily assessment of clinical response is needed to adjust the subsequent doses. Edema associated with liver cirrhosis, and nephrotic syndrome Cerebral edema - intravenous furosemide can be combined with mannitol to initiate rapid diuresis. However, the optimum duration of such treatment remains unknown. Frequent fluid status monitoring is required to prevent intravascular volume depletion which leads to reduced cerebral perfusion. A bolus intravenous dose of 10 or 20 mg of furosemide can be administered and then followed by intravenous bolus of 2 or 3% hypertonic saline to increase the serum sodium level. Felodipine (Plendil), a new drug, has been used in the treatment of five patients with refractory essential hypertension (WHO II-III). Their mean blood pressure at the last outpatient visit before the study was opened was 195 +/- 25/129 +/- 21 mmHg (mean +/- s.d.) (range 175-235/110-165 mmHg), despite treatment with combinations of diuretics, beta-blockers and vasodilators, including minoxidil and captopril. Felodipin is a dihydropyridine derivative, a calcium antagonist that exerts a relaxant effect on resistance vessels. The first period of the study consisted of a 5-day stay in hospital followed by 3 months during which observations were carried out at the Outpatients' Department. After the first days in hospital felodipine therapy was introduced at a dose of 25 mg three times daily, given together with diuretics, beta-blockers and, in one case, captopril. At 8.00 immediately before the first dose was given, the blood pressure was 178 +/- 19/118 +/- 19 mmHg (mean +/- s.d.); 2 h later it was 144 +/- 18/85 +/- 4 mmHg, at which level it remained throughout the rest of the study. At the 3-month follow-up the mean pressure (recorded at the Outpatients' Department) was 138 +/- 20/89 +/- 14 mmHg. Side-effects included headache, flushing, palpitations and ankle oedema (in two patients during the second part of the study); they were of a mild to moderate degree and did not interfere with the treatment. There was no evidence of general fluid retention, and the body weight remained constant.(ABSTRACT TRUNCATED AT 250 WORDS) Placebo-controlled trials have found that angiotensin-converting enzyme inhibitors (ACEIs) decrease proteinuria and slow the progression of nondiabetic nephropathies. However, head-to-head comparisons of ACEIs and calcium channel blockers (CCBs) have shown conflicting results. Indeed, a recent metaanalysis concluded that there is still uncertainty about the greater renoprotection seen with ACEIs or angiotensin II receptor blockers in nondiabetic patients with renal disease, particularly when using true glomerular filtration rate (GFR) as the primary outcome. The objective of this 3-year, randomized, multicenter, double-blind, placebo-controlled study was to compare true GFR decline (measured by yearly 51Cr-EDTA blood clearance) in nondiabetic, nonnephrotic adult hypertensive patients with estimated creatinine clearance of 20 to 60 mL/min.1.73 m(2), when randomized to a CCB (amlodipine, 5-10 mg/d) or an ACEI (enalapril, 5-20 mg/d). Patients (aged 18-80 years) entered a 4-week placebo run-in washout period and previous antihypertensive drugs were tapered off over 2 weeks. Add-on treatments were atenolol (50-100 mg/d), loop diuretics (furosemide, 20-500 mg/d or torsemide, 5-200 mg/d), alpha-blockers (prazosin, 2.5-5 mg/d or doxazosin, 1-16 mg/d), and centrally acting drugs (rilmenidine, 1-2 mg/d or methyldopa, 250-500 mg/d). The primary end point was true GFR measured by yearly (51)Cr-EDTA blood clearance. Secondary end points included a clinical composite of renal events and tolerability collected by a full clinical and laboratory evaluation at each study visit. Post hoc analyses for the change in GFR, proteinuria, and time to clinical events were also planned on baseline proteinuria subgroups (<1 and >or=1 g/d) before unblinding the database. Three hundred eighteen patients entered the run-in period and 263 patients (156 men/107 women; mean age, 58 years) were randomized to receive either amlodipine (5 mg/d, n=132) or enalapril (5 mg/d, n=131). Blood pressure declined from 165/102 mm Hg to 138/84 mm Hg and 138/85 mm Hg with amlodipine and enalapril, respectively (no between-group significance). Only 20.8% of the patients randomized to ACEI treatment received diuretics at the last observation. No statistically significant difference was found between amlodipine and enalapril in GFR decline (-4.92 and -3.98 mL/min.1.73 m(2), respectively, at last observation) and composite secondary end point after a median follow-up of 2.9 years, including in the subgroup of patients with proteinuria >1 g/d at baseline. Protein excretion rate decreased significantly from baseline in patients taking enalapril plus diuretics (median -270 mg/d; P<0.001) but not in patients taking amlodipine plus diuretics (-25 mg/d at last observation). In this cohort of nondiabetic, nonnephrotic hypertensive patients, no statistically significant difference in true GFR decline was found over 3 years between amlodipine-treated patients and enalapril-treated patients with main add-on treatment with ss-blockers, including in the subgroup of patients with proteinuria >1 g/d. The international, prospective, randomized HOT study was aimed at determining the influence of a targeted BP reduction on cardiovascular morbidity and mortality. Patients were randomly allocated to 3 DBP targets (< 80, < 85, < 90 mmHg). In addition, the impact of a coprescription of aspirin was studied. The BP target had to be reached within 3 months, according to a well-defined strategy : felodipine 5 mg o.d. as a 1st intention drug, 1, 2 or 3 additional drugs, if necessary, on the following steps. BP measurements were made, using an oscillometric automatic device (Hestia). From April 1992 to October 1994, 18,790 patients with an age range 50-80 years, coming from 26 countries, entered the study. The data collected on the 36th month were in agreement with those obtained on the 12th and the 24th months. Baseline DBP was reduced by 21, 23 and 25 mmHg in the 90, 85 and 80 mmHg target groups, respectively. The rate of patients whose DBP reached the target, obviously increased from the 3rd to the 12th month: from 43 to 56%, 60 to 70%, 74 to 83% in the 90, 85 and 80 mmHg, target groups, respectively. From the 2nd to the 3rd year, BP control was further improved, with a slightly higher rate of controlled patients in the elderly (age > 60 y), especially in the 80 mmHg target group. From inclusion to the 3rd month, one-drug treated patients decreased, whereas 2- or 3-drug treated patients increased. Felodipine-treated patients decreased on the 36th month, but remained over 80%. From the 6th to the 36th month, additional prescription of a betablocker or an ACE-inhibitor increased from 36 to 39%, and from 23 to 28%, respectively; moreover, the side-effects rate decreased from 10.5 to 3.6%, with a special decline in ankle edema from 4 to 1%. In conclusion, the BP reduction observed on the 36th month was of the same extent as that observed in the first months. It seems obviously possible to reach a targeted DBP and to maintain it over time, along with a good acceptability of the treatment. Targeted DBP could be more easily achieved in elderly patients, possibly due to a better drug compliance. Ephedrine has previously been described as a causative factor of vasculitis but myocarditis has not yet been associated with either ephedrine or its plant derivative ephedra. A 39-year-old African American male with hypertension presented to Rush Presbyterian St. Luke's Medical Center with a 1-month history of progressive dyspnea on exertion, orthopnea, and dependent edema. He was taking Ma Huang (Herbalife) 1-3 tablets twice daily for 3 months along with other vitamin supplements, pravastatin, and furosemide. Physical examination revealed a male in mild respiratory distress. The lung fields had rales at both bases without audible wheezes. Internal jugular venous pulsations were 5 cm above the sternal notch. Medical therapy with intravenous furosemide and oral enalapril was initiated upon admission. Cardiac catheterization with coronary angiography revealed normal coronary arteries, a dilated left ventricle, moderate pulmonary hypertension, and a pulmonary capillary wedge pressure of 34 mm Hg. The patient had right ventricular biopsy performed demonstrating mild myocyte hypertrophy and an infiltrate consisting predominantly of lymphocytes with eosinophils present in significantly increased numbers. Treatment for myocarditis was initiated with azothioprine 200 mg daily and prednisone 60 mg per day with a tapering course over 6 months. Anticoagulation with warfarin and diuretics was initiated and angiotensin-converting enzyme inhibition was continued. Hydralazine was added later. One month into therapy, an echocardiogram demonstrated improved left ventricular function with only mild global hypokinesis. A repeat right ventricular biopsy 2 months after the first admission showed no evidence of myocarditis. At 6 months, left ventricular ejection fraction was normal (EFN 50%) and the patient asymptomatic. Ephedra (Ma Huang) is the suspected cause of hypersensitivity myocarditis in this patient due to the temporal course of disease and its propensity to induce vasculitis. Sildenafil, a selective inhibitor of cyclic guanosine monophosphate (cGMP)-specific phosphodiesterase type 5 (PDE5), is a well-tolerated and highly effective treatment for erectile dysfunction. The mechanism of action of sildenafil depends on activation of the nitric oxide (NO)-cGMP pathway during sexual stimulation, which results in corpus cavernosal smooth muscle relaxation and penile erection. Endogenously derived NO is also involved in blood pressure regulation through its effect on basal vascular tone, which is mediated by cGMP levels. Organic nitrates and NO donors exert their therapeutic effects on blood pressure and vascular smooth muscle by the same mechanism as endogenous NO. Since both sildenafil and organic nitrates exert their pharmacologic effects via increases in cGMP concentrations, a double-blind, placebo-controlled, crossover study was undertaken to investigate the effects of sildenafil coadministered with glyceryl trinitrate on blood pressure and heart rate in healthy male subjects. The hemodynamic effects of sildenafil were also evaluated in a second placebo-controlled crossover study in men with hypertension who were taking the calcium antagonist amlodipine, which has a mechanism of action that does not involve the cGMP pathway. In the first crossover study, subjects were treated with oral sildenafil (25 mg, 3 times a day for 4 days) or placebo and then challenged on day 4 with a 40-minute, stepwise, intravenous infusion of glyceryl trinitrate (0.5 mg/mL in 5% dextrose at an initial infusion rate of 2.5 microg/min and doubling every 5 minutes to a maximum rate of 40 microg/min) 1 hour after taking sildenafil or placebo. On day 5, subjects received a sublingual glyceryl trinitrate tablet (500 microg) 1 hour after taking 25 mg of sildenafil or placebo. During sildenafil treatment, the subjects were significantly less tolerant of intravenously administered glyceryl trinitrate than during placebo treatment, based on the occurrence of a >25 mm Hg decrease in blood pressure or the incidence of symptomatic hypotension (p <0.01). When a sublingual glyceryl trinitrate tablet was administered on day 5, a 4-fold greater decrease in systolic blood pressure was observed for the subjects during the sildenafil treatment period than during the placebo treatment period. The changes in heart rate were negligible during both glyceryl trinitrate challenges. In conclusion, sildenafil potentiated the hypotensive effects of glyceryl trinitrate, an organic nitrate. Thus, sildenafil administration to patients who are using organic nitrates, either regularly and/or intermittently, in any form is contraindicated. In the second crossover study, men with hypertension, who were taking 5 or 10 mg/day of amlodipine, received a single oral dose of 100 mg sildenafil or placebo. Coadministration of sildenafil did not significantly affect the pharmacokinetics of amlodipine. In the 4 hours after dosing, differences in the mean maximum change from baseline in supine systolic and diastolic blood pressures between the sildenafil plus amlodipine and the placebo plus amlodipine treatment periods were -8 mm Hg and -7 mm Hg, respectively (p < or =0.002). The mean maximum supine heart rate increased 2.1 beats/min during sildenafil plus amlodipine treatment and decreased 1.5 beats/min during placebo plus amlodipine treatment (p <0.02). The adverse events in this study were predominantly mild or moderate and did not cause discontinuation of treatment. Adverse events considered to be related to sildenafil treatment included headache, nausea, and dyspepsia. In patients with hypertension who were taking amlodipine therapy, sildenafil produced additive, but not synergistic, reductions in blood pressure. The difference in the mean maximum change from baseline in blood pressure between sildenafil plus amlodipine and placebo plus amlodipine was comparable to the decrease in blood pressure reported for healthy men taking sildenafil alone. (ABSTRACT TRUNCATED)\nHere is the question:\nCARDIOLOGY AND VASCULAR SURGERY: A 73-year-old woman is admitted with progressive dyspnea until she becomes at rest, orthopnea and weight gain of 4 kg. Physical examination showed blood pressure of 150/84 mm Hg, heart rate 100 beats/minute, increased jugular venous pressure, crepitant in both bases and malleolar edema. Usual treatment: enalapril 5 mg every 12 hours, furosemide 80 mg per day. What is the most appropriate treatment at this time?\nHere are the potential choices:\n1. Administer fiirosemide intravenously.\n2. Increase enalapril dose according to tolerance and administer intravenous furosemide.\n3. Start a beta-blocker.\n4. Add treatment with amlodipine.\nThe correct answer is: ", "gold_answer": "2 Increase enalapril dose according to tolerance and administer intravenous furosemide.", "prediction": "You are a helpful medical expert, and your task is to answer a multi-choice medical question using the relevant documents. Please choose the answer from the provided options. Your responses will be used for research purposes only, so please have a definite answer.\nHere are the relevant documents:\n Diuretics have been accepted as first-line treatment in refractory congestive heart failure (CHF), but a lack of response to them is a frequent event. A randomized, single-blind study was performed to evaluate the effects of the combination of high-dose furosemide and small-volume hypertonic saline solution (HSS) infusion in the treatment of refractory New York Heart Association (NYHA) class IV CHF and a normosodic diet during follow-up. Materials and Methods One hundred seven patients (39 women and 68 men, age range 65-90 years) with refractory CHF (NYHA class IV) of different etiologies, who were unresponsive to high oral doses of furosemide, angiotensin-converting enzyme inhibitors, digitalis, and nitrates, were enrolled. Inclusion criteria included an ejection fraction (EF) <35%, serum creatinine level <2 mg/dL, blood urea nitrogen level < or =60 mg/dL, reduced urinary volume, and low natriuresis. The patients were randomized in 2 groups (single-blind). Patients in group 1 (20 women and 33 men) received an intravenous (IV) infusion of furosemide (500-1000 mg) plus HSS (150 mL of 1.4%-4.6% NACl) twice a day in 30 minutes. Patients in group 2 (19 women and 35 men) received an IV bolus of furosemide (500-1000 mg) twice a day, without HSS, during a period lasting 6 to 12 days. Both groups received IV KCl (20-40 mEq) to prevent hypokalemia. At study entry, all patients underwent a physical examination and measurement of body weight (BW), blood pressure (BP), and heart rate (HR), an evaluation of signs of CHF, and measurement of control levels of serum Na, K, Cl, bicarbonate, albumin, uric acid, creatinine, urea, and glycemia daily during hospitalization, and measurements of the daily output of urine for Na, K, and Cl. A chest radiograph, electrocardiogram, and echocardiogram were obtained at study entry, during hospitalization, and at the time of discharge from the hospital. During the treatment and after discharge, the daily dietary Na intake was 120 mmol in group 1 versus 80 mmol in group 2, with a fluid intake of 1000 mL daily in both groups. An assessment of BW and 24-hour urinary volume, serum, and urinary laboratory parameters were performed daily until patients reached a compensated state, when IV furosemide was replaced with oral administration (250-500 mg/d). After discharge from the hospital, patients were observed as outpatients weekly for the first 3 months and, subsequently, once a month. The groups were similar in age, sex, EF, risk factors, treatment, and etiology of CHF. All patients showed a clinical improvement. Ten patients in both groups had hyponatremia at entry. A significant increase in daily diuresis and natriuresis was observed in both groups, but it was more significant in the group receiving HSS (P <.05). The serum Na level increased in group 1 and decreased in group 2 (P <.05). The serum K level was decreased in both groups (P <.05). BW was reduced in both groups (P <.05). Group 2 had an increase in serum creatinine level. Serum uric acid levels increased in both groups. BP values decreased and HR was corrected to normal values in both groups. In the follow-up period (31 +/- 14 months), 25 patients from group 1 were readmitted to the hospital for heart failure. In group 2, 43 patients were readmitted to the hospital at a higher class than at discharge. Twenty-four patients in group 1 died during follow-up, versus 47 patients in group 2 (P <.001). This treatment is effective and well tolerated, improves the quality of life through the relief of signs and symptoms of congestion, and may delay more aggressive treatments. The effects were also beneficial in a long period for mortality reduction (55% vs 13% survival rate) and for clinical improvement. Pulmonary edema - Slow intravenous bolus dose of 40 to 80 mg furosemide at 4 mg per minute is indicated for patients with fluid overload and pulmonary odema. Such dose can be repeated after 20 minutes. After the bolus, a continuous intravenous infusion can be given at 5 to 10 mg per hour. For those with underlying renal impairment or severe heart failure, up to 160 to 200 mg bolus dose can be given. The effectiveness and safety of once-daily administration of drugs in the treatment of moderate to severe hypertension was studied. Forty men taking diuretics were randomized to atenolol (A, n = 18), 50 mg/day, or betaxolol (B, n = 22), a new B1-blocker, 20 mg/day, if their SDAP was 105 to 125 mm Hg at baseline (weeks 2 to 4). At week 6, if SDAP was greater than 95 mm Hg, minoxidil (M), 5.5 mg/day, was added. The patients were seen every two weeks to week 16 (end of drug titration) and then every four weeks to week 32. The dosages were increased to 200 mg/day for A, 80 mg/day for B, and 20 mg/day for M as needed. Physical examinations, chest x-ray films, ECGs, echocardiograms, spirometric studies, 24-h ambulatory arterial pressures (AAP), and blood chemistry analyses were done at baseline and during treatment. A and B combined with a diuretic (furosemide, F) and M decreased the arterial pressures and heart rates equally well by both clinical and AAP measurements (p less than .001). The IVS was decreased (p less than .05), whereas LVIDd, RVIDd, and cardiothoracic ratios were increased by both A and B (p less than .05, p less than .01). No changes were noted in LVPW, LVM, EF, FS, spirometric values, or blood chemistry analyses. Common side effects were weight gain, edema, and hypertrichosis. Once-daily administration of A or B in combination with F and M were effective in the treatment of moderate to severe hypertension. Although effective, prolonged use of M may lead to volume overload and cardiomegaly. The significance of these latter findings is not yet known. Nitrates and furosemide, commonly administered in the treatment of pulmonary oedema, have not been compared in a prospective clinical trial. We compared the efficacy and safety of these drugs in a randomised trial of patients with severe pulmonary oedema and oxygen saturation below 90%. Patients presenting to mobile emergency units with signs of congestive heart failure were treated with oxygen 10 L/min, intravenous furosemide 40 mg, and morphine 3 mg bolus. 110 patients were randomly assigned either to group A, who received isosorbide dinitrate (3 mg bolus administered intravenously every 5 min; n=56) or to group B, who received furosemide (80 mg bolus administered intravenously every 15 min, as well as isosorbide dinitrate 1 mg/h, increased every 10 min by 1 mg/h; n=54). Six patients were withdrawn on the basis of chest radiography results. Treatment was continued until oxygen saturation was above 96% or mean arterial blood pressure had decreased by 30% or to below 90 mm Hg. The main endpoints were death, need for mechanical ventilation, and myocardial infarction. The analyses were by intention to treat. Mechanical ventilation was required in seven (13%) of 52 group-A patients and 21 (40%) of 52 group-B patients (p=0.0041). Myocardial infarction occurred in nine (17%) and 19 (37%) patients, respectively (p=0.047). One patient in group A and three in group B died (p=0.61). One or more of these endpoints occurred in 13 (25%) and 24 (46%) patients, respectively (p=0.041). High-dose isosorbide dinitrate, given as repeated intravenous boluses after low-dose intravenous furosemide, is safe and effective in controlling severe pulmonary oedema. This treatment regimen is more effective than high-dose furosemide with low-dose isosorbide nitrate in terms of need for mechanical ventilation and frequency of myocardial infarction. The hemodynamic response of isosorbide-5-mononitrate (IS-5-MN) to the addition of the widely used therapy of diuretic drugs and the maximally tolerated dose of enalapril for heart failure was assessed in 8 patients with congestive heart failure (CHF) (New York Heart Association class II and III). The diuretic therapy was furosemide, 40 to 80 mg/day, with or without amiloride, 5 to 10 mg/day. The dose of enalapril was 5 to 20 mg/day. Four hours after the administration of the morning dose of enalapril, a Swan-Ganz catheter was positioned in the pulmonary artery. Patients received increasing doses of IS-5-MN to produce a satisfactory decrease in pulmonary capillary wedge pressure. Two of the first 3 patients studied had a large reduction in blood pressure when given 10 mg of IS-5-MN. Subsequent patients were therefore given an initial dose of 5 mg, the total dose being 5 to 20 mg over 2 hours. Results at baseline and 1 hour after the final dose of IS-5-MN are expressed as mean +/- standard deviation. Both pulmonary artery systolic and diastolic pressures decreased significantly (p less than 0.05) by 12.2 +/- 8.9/4.2 +/- 5.2 mm Hg, from 47.2 +/- 16.0/21.6 +/- 6.0 mm Hg to 35.0 +/- 15.2/17.4 +/- 9.3 mm Hg. Pulmonary capillary wedge pressure decreased by 8.6 +/- 4.4 mm Hg, from 22.1 +/- 5.4 to 13.6 +/- 7.5 mm Hg (p less than 0.01).(ABSTRACT TRUNCATED AT 250 WORDS) A recent 8-week, double-masked, placebo-controlled, 3 x 4 factorial-design study demonstrated that enalapril-felodipine extended-release (ER) combinations had statistically significant additive effects for reducing both sitting systolic blood pressure (SiSBP) and sitting diastolic blood pressure (SiDBP) and were generally well tolerated in hypertensive patients with SiDBPs ranging from 95 to 115 mm Hg. The present open-label study was undertaken to assess the long-term efficacy, tolerability, and safety of such combinations. Patients from the factorial study were eligible for the 1-year, open-label extension. Initially, all patients received enalapril 5 mg-felodipine ER 2.5 mg once daily; if SiDBP was not controlled (< 90 mm Hg) after 4 weeks of treatment, the dose was titrated upward at 2- to 4-week intervals to a maximum of enalapril 10 mg-felodipine ER 10 mg. Hydrochlorothiazide (HCTZ) 12.5 mg was added to the regimen of patients whose hypertension was not controlled at the highest enalapril-felodipine ER dose. A total of 507 patients were enrolled, of whom 502 were assessable. At their last study visit, 391 (78%) of the assessable patients were receiving only an enalapril-felodipine ER combination. The enalapril-felodipine ER combinations resulted in mean trough SiDBPs of 85 to 89 mm Hg (decreases of 13 to 16 mm Hg from baseline) and SiSBPs of 137 to 140 mm Hg (decreases of 13 to 21 mm Hg). Overall, 407 (81%) of the 502 assessable patients achieved an SiDBP < 90 mm Hg or a reduction from baseline > or = 10 mm Hg (responders); such a response was recorded in 331 patients (66%) taking a combination of enalapril-felodipine ER alone and 76 patients (15%) taking the combination with the addition of HCTZ 12.5 mg. Blood pressure reductions were maintained throughout the treatment period. Drug-related adverse events were relatively infrequent, often transient, usually mild, and apparently not dose related. The most frequently reported drug-related adverse events were edema/swelling, asthenia/fatigue, dizziness, cough, and headache. These results suggest that combination therapy with enalapril-felodipine ER is effective for long-term blood pressure reduction, has an excellent safety profile, and is generally well tolerated. Addition of low-dose HCTZ to the enalapril-felodipine ER combination appears to provide further blood pressure control without increasing drug-related adverse events. Monitoring diuresis: Diuresis can be monitored by weighing the person daily. The goal is weight loss of no more than 1.0 kg/day for people with both ascites and peripheral edema and no more than 0.5 kg/day for people with ascites alone. If daily weights cannot be obtained, diuretics can also be guided by the urinary sodium concentration. Dosage is increased until a negative sodium balance occurs. A random urine sodium-to-potassium ratio of > 1 is 90% sensitivity in predicting negative balance (> 78-mmol/day sodium excretion). Diuretic resistance: Diuretic resistance can be predicted by giving 80 mg intravenous furosemide after 3 days without diuretics and on an 80 mEq sodium/day diet. The urinary sodium excretion over 8 hours < 50 mEq/8 hours predicts resistance. Clinical use Loop diuretics are principally used in the following indications: Heart failure - Giving 2.5 times of previous oral dose twice daily for those with acute decompensated heart failure is a reasonable strategy. However, daily assessment of clinical response is needed to adjust the subsequent doses. Edema associated with liver cirrhosis, and nephrotic syndrome Cerebral edema - intravenous furosemide can be combined with mannitol to initiate rapid diuresis. However, the optimum duration of such treatment remains unknown. Frequent fluid status monitoring is required to prevent intravascular volume depletion which leads to reduced cerebral perfusion. A bolus intravenous dose of 10 or 20 mg of furosemide can be administered and then followed by intravenous bolus of 2 or 3% hypertonic saline to increase the serum sodium level. Felodipine (Plendil), a new drug, has been used in the treatment of five patients with refractory essential hypertension (WHO II-III). Their mean blood pressure at the last outpatient visit before the study was opened was 195 +/- 25/129 +/- 21 mmHg (mean +/- s.d.) (range 175-235/110-165 mmHg), despite treatment with combinations of diuretics, beta-blockers and vasodilators, including minoxidil and captopril. Felodipin is a dihydropyridine derivative, a calcium antagonist that exerts a relaxant effect on resistance vessels. The first period of the study consisted of a 5-day stay in hospital followed by 3 months during which observations were carried out at the Outpatients' Department. After the first days in hospital felodipine therapy was introduced at a dose of 25 mg three times daily, given together with diuretics, beta-blockers and, in one case, captopril. At 8.00 immediately before the first dose was given, the blood pressure was 178 +/- 19/118 +/- 19 mmHg (mean +/- s.d.); 2 h later it was 144 +/- 18/85 +/- 4 mmHg, at which level it remained throughout the rest of the study. At the 3-month follow-up the mean pressure (recorded at the Outpatients' Department) was 138 +/- 20/89 +/- 14 mmHg. Side-effects included headache, flushing, palpitations and ankle oedema (in two patients during the second part of the study); they were of a mild to moderate degree and did not interfere with the treatment. There was no evidence of general fluid retention, and the body weight remained constant.(ABSTRACT TRUNCATED AT 250 WORDS) Placebo-controlled trials have found that angiotensin-converting enzyme inhibitors (ACEIs) decrease proteinuria and slow the progression of nondiabetic nephropathies. However, head-to-head comparisons of ACEIs and calcium channel blockers (CCBs) have shown conflicting results. Indeed, a recent metaanalysis concluded that there is still uncertainty about the greater renoprotection seen with ACEIs or angiotensin II receptor blockers in nondiabetic patients with renal disease, particularly when using true glomerular filtration rate (GFR) as the primary outcome. The objective of this 3-year, randomized, multicenter, double-blind, placebo-controlled study was to compare true GFR decline (measured by yearly 51Cr-EDTA blood clearance) in nondiabetic, nonnephrotic adult hypertensive patients with estimated creatinine clearance of 20 to 60 mL/min.1.73 m(2), when randomized to a CCB (amlodipine, 5-10 mg/d) or an ACEI (enalapril, 5-20 mg/d). Patients (aged 18-80 years) entered a 4-week placebo run-in washout period and previous antihypertensive drugs were tapered off over 2 weeks. Add-on treatments were atenolol (50-100 mg/d), loop diuretics (furosemide, 20-500 mg/d or torsemide, 5-200 mg/d), alpha-blockers (prazosin, 2.5-5 mg/d or doxazosin, 1-16 mg/d), and centrally acting drugs (rilmenidine, 1-2 mg/d or methyldopa, 250-500 mg/d). The primary end point was true GFR measured by yearly (51)Cr-EDTA blood clearance. Secondary end points included a clinical composite of renal events and tolerability collected by a full clinical and laboratory evaluation at each study visit. Post hoc analyses for the change in GFR, proteinuria, and time to clinical events were also planned on baseline proteinuria subgroups (<1 and >or=1 g/d) before unblinding the database. Three hundred eighteen patients entered the run-in period and 263 patients (156 men/107 women; mean age, 58 years) were randomized to receive either amlodipine (5 mg/d, n=132) or enalapril (5 mg/d, n=131). Blood pressure declined from 165/102 mm Hg to 138/84 mm Hg and 138/85 mm Hg with amlodipine and enalapril, respectively (no between-group significance). Only 20.8% of the patients randomized to ACEI treatment received diuretics at the last observation. No statistically significant difference was found between amlodipine and enalapril in GFR decline (-4.92 and -3.98 mL/min.1.73 m(2), respectively, at last observation) and composite secondary end point after a median follow-up of 2.9 years, including in the subgroup of patients with proteinuria >1 g/d at baseline. Protein excretion rate decreased significantly from baseline in patients taking enalapril plus diuretics (median -270 mg/d; P<0.001) but not in patients taking amlodipine plus diuretics (-25 mg/d at last observation). In this cohort of nondiabetic, nonnephrotic hypertensive patients, no statistically significant difference in true GFR decline was found over 3 years between amlodipine-treated patients and enalapril-treated patients with main add-on treatment with ss-blockers, including in the subgroup of patients with proteinuria >1 g/d. The international, prospective, randomized HOT study was aimed at determining the influence of a targeted BP reduction on cardiovascular morbidity and mortality. Patients were randomly allocated to 3 DBP targets (< 80, < 85, < 90 mmHg). In addition, the impact of a coprescription of aspirin was studied. The BP target had to be reached within 3 months, according to a well-defined strategy : felodipine 5 mg o.d. as a 1st intention drug, 1, 2 or 3 additional drugs, if necessary, on the following steps. BP measurements were made, using an oscillometric automatic device (Hestia). From April 1992 to October 1994, 18,790 patients with an age range 50-80 years, coming from 26 countries, entered the study. The data collected on the 36th month were in agreement with those obtained on the 12th and the 24th months. Baseline DBP was reduced by 21, 23 and 25 mmHg in the 90, 85 and 80 mmHg target groups, respectively. The rate of patients whose DBP reached the target, obviously increased from the 3rd to the 12th month: from 43 to 56%, 60 to 70%, 74 to 83% in the 90, 85 and 80 mmHg, target groups, respectively. From the 2nd to the 3rd year, BP control was further improved, with a slightly higher rate of controlled patients in the elderly (age > 60 y), especially in the 80 mmHg target group. From inclusion to the 3rd month, one-drug treated patients decreased, whereas 2- or 3-drug treated patients increased. Felodipine-treated patients decreased on the 36th month, but remained over 80%. From the 6th to the 36th month, additional prescription of a betablocker or an ACE-inhibitor increased from 36 to 39%, and from 23 to 28%, respectively; moreover, the side-effects rate decreased from 10.5 to 3.6%, with a special decline in ankle edema from 4 to 1%. In conclusion, the BP reduction observed on the 36th month was of the same extent as that observed in the first months. It seems obviously possible to reach a targeted DBP and to maintain it over time, along with a good acceptability of the treatment. Targeted DBP could be more easily achieved in elderly patients, possibly due to a better drug compliance. Ephedrine has previously been described as a causative factor of vasculitis but myocarditis has not yet been associated with either ephedrine or its plant derivative ephedra. A 39-year-old African American male with hypertension presented to Rush Presbyterian St. Luke's Medical Center with a 1-month history of progressive dyspnea on exertion, orthopnea, and dependent edema. He was taking Ma Huang (Herbalife) 1-3 tablets twice daily for 3 months along with other vitamin supplements, pravastatin, and furosemide. Physical examination revealed a male in mild respiratory distress. The lung fields had rales at both bases without audible wheezes. Internal jugular venous pulsations were 5 cm above the sternal notch. Medical therapy with intravenous furosemide and oral enalapril was initiated upon admission. Cardiac catheterization with coronary angiography revealed normal coronary arteries, a dilated left ventricle, moderate pulmonary hypertension, and a pulmonary capillary wedge pressure of 34 mm Hg. The patient had right ventricular biopsy performed demonstrating mild myocyte hypertrophy and an infiltrate consisting predominantly of lymphocytes with eosinophils present in significantly increased numbers. Treatment for myocarditis was initiated with azothioprine 200 mg daily and prednisone 60 mg per day with a tapering course over 6 months. Anticoagulation with warfarin and diuretics was initiated and angiotensin-converting enzyme inhibition was continued. Hydralazine was added later. One month into therapy, an echocardiogram demonstrated improved left ventricular function with only mild global hypokinesis. A repeat right ventricular biopsy 2 months after the first admission showed no evidence of myocarditis. At 6 months, left ventricular ejection fraction was normal (EFN 50%) and the patient asymptomatic. Ephedra (Ma Huang) is the suspected cause of hypersensitivity myocarditis in this patient due to the temporal course of disease and its propensity to induce vasculitis. Sildenafil, a selective inhibitor of cyclic guanosine monophosphate (cGMP)-specific phosphodiesterase type 5 (PDE5), is a well-tolerated and highly effective treatment for erectile dysfunction. The mechanism of action of sildenafil depends on activation of the nitric oxide (NO)-cGMP pathway during sexual stimulation, which results in corpus cavernosal smooth muscle relaxation and penile erection. Endogenously derived NO is also involved in blood pressure regulation through its effect on basal vascular tone, which is mediated by cGMP levels. Organic nitrates and NO donors exert their therapeutic effects on blood pressure and vascular smooth muscle by the same mechanism as endogenous NO. Since both sildenafil and organic nitrates exert their pharmacologic effects via increases in cGMP concentrations, a double-blind, placebo-controlled, crossover study was undertaken to investigate the effects of sildenafil coadministered with glyceryl trinitrate on blood pressure and heart rate in healthy male subjects. The hemodynamic effects of sildenafil were also evaluated in a second placebo-controlled crossover study in men with hypertension who were taking the calcium antagonist amlodipine, which has a mechanism of action that does not involve the cGMP pathway. In the first crossover study, subjects were treated with oral sildenafil (25 mg, 3 times a day for 4 days) or placebo and then challenged on day 4 with a 40-minute, stepwise, intravenous infusion of glyceryl trinitrate (0.5 mg/mL in 5% dextrose at an initial infusion rate of 2.5 microg/min and doubling every 5 minutes to a maximum rate of 40 microg/min) 1 hour after taking sildenafil or placebo. On day 5, subjects received a sublingual glyceryl trinitrate tablet (500 microg) 1 hour after taking 25 mg of sildenafil or placebo. During sildenafil treatment, the subjects were significantly less tolerant of intravenously administered glyceryl trinitrate than during placebo treatment, based on the occurrence of a >25 mm Hg decrease in blood pressure or the incidence of symptomatic hypotension (p <0.01). When a sublingual glyceryl trinitrate tablet was administered on day 5, a 4-fold greater decrease in systolic blood pressure was observed for the subjects during the sildenafil treatment period than during the placebo treatment period. The changes in heart rate were negligible during both glyceryl trinitrate challenges. In conclusion, sildenafil potentiated the hypotensive effects of glyceryl trinitrate, an organic nitrate. Thus, sildenafil administration to patients who are using organic nitrates, either regularly and/or intermittently, in any form is contraindicated. In the second crossover study, men with hypertension, who were taking 5 or 10 mg/day of amlodipine, received a single oral dose of 100 mg sildenafil or placebo. Coadministration of sildenafil did not significantly affect the pharmacokinetics of amlodipine. In the 4 hours after dosing, differences in the mean maximum change from baseline in supine systolic and diastolic blood pressures between the sildenafil plus amlodipine and the placebo plus amlodipine treatment periods were -8 mm Hg and -7 mm Hg, respectively (p < or =0.002). The mean maximum supine heart rate increased 2.1 beats/min during sildenafil plus amlodipine treatment and decreased 1.5 beats/min during placebo plus amlodipine treatment (p <0.02). The adverse events in this study were predominantly mild or moderate and did not cause discontinuation of treatment. Adverse events considered to be related to sildenafil treatment included headache, nausea, and dyspepsia. In patients with hypertension who were taking amlodipine therapy, sildenafil produced additive, but not synergistic, reductions in blood pressure. The difference in the mean maximum change from baseline in blood pressure between sildenafil plus amlodipine and placebo plus amlodipine was comparable to the decrease in blood pressure reported for healthy men taking sildenafil alone. (ABSTRACT TRUNCATED)\nHere is the question:\nCARDIOLOGY AND VASCULAR SURGERY: A 73-year-old woman is admitted with progressive dyspnea until she becomes at rest, orthopnea and weight gain of 4 kg. Physical examination showed blood pressure of 150/84 mm Hg, heart rate 100 beats/minute, increased jugular venous pressure, crepitant in both bases and malleolar edema. Usual treatment: enalapril 5 mg every 12 hours, furosemide 80 mg per day. What is the most appropriate treatment at this time?\nHere are the potential choices:\n1. Administer fiirosemide intravenously.\n2. Increase enalapril dose according to tolerance and administer intravenous furosemide.\n3. Start a beta-blocker.\n4. Add treatment with amlodipine.\nThe correct answer is: 2. Increase enalapril dose according to tolerance and administer intravenous furosemide." }, { "question": "You are a helpful medical expert, and your task is to answer a multi-choice medical question using the relevant documents. Please choose the answer from the provided options. Your responses will be used for research purposes only, so please have a definite answer.\nHere are the relevant documents:\n Inguinal lymphadenopathy causes swollen lymph nodes in the groin area. It can be a symptom of infective or neoplastic processes. Infective aetiologies include Tuberculosis, HIV, non-specific or reactive lymphadenopathy to recent lower limb infection or groin infections. Another notable infectious cause is Lymphogranuloma venereum, which is a sexually transmitted infection of the lymphatic system. Neoplastic aetiologies include lymphoma, leukaemia and metastatic disease from primary tumours in the lower limb, external genitalia or perianal region and melanoma. References Further reading Inflammations Diseases of veins, lymphatic vessels and lymph nodes Overview A genital ulcer may be located on the vulva, penis, perianal region, or anus. Globally, the incidence of genital ulcers is estimated to be approximately 20 million cases annually. The most likely cause of a genital ulcer varies depending on the characteristics of a population and location. The most common cause of genital ulcers in the United States is herpes simplex infections, with syphilis the second most common cause, and chancroid the third. These common causes of genital ulcer disease (HSV-1, HSV-2 and treponema pallidum) can all be efficiently transmitted through oral sex. Important signs associated with genital ulcers that may assist in the diagnosis of the cause of the genital ulcer may include the presence of tender or non-tender enlarged lymph nodes in the groin area, a painful or non-painful genital ulcer, or the presence of vesicular lesions, which are small, painful, elevated blisters. Syphilis (lues) is a sexually transmitted infection caused by the spirochete Treponema pallidum. In adolescents, the diagnosis of primary syphilis can be made promptly by taking a sexual medical history and inspecting the glans penis. A 17-year-old male was referred to the paediatric oncology centre for additional diagnostics due to inguinal lymphadenopathy, with a strong suspicion of a malignant lymphoma. None of the physicians took a sexual medical history or investigated the glans penis, as a result of which essential information was lacking. The combination of inguinal lymphadenopathy and the ultrasound findings for the inguinal region made the physicians only consider a malignancy. However, it actually concerned a reactive lymphadenopathy associated with primary syphilis. This case demonstrates that a full medical history and thorough physical examination can prevent the need for costly and invasive diagnostics. Clinical significance The presence of swollen inguinal lymph nodes is an important clinical sign because lymphadenopathy (swelling) may indicate an infection, or spread as a metastasis from cancers, such as anal cancer and vulvar cancer. Inguinal lymph nodes may normally be up to 2 cm. The cut-off value for normal sized inguinal nodes is up to 10 mm. Additional images References Lymphatic organ anatomy Swollen lymph nodes usually indicate infection from bacteria or viruses. Swollen inguinal lymph nodes could indicate an infection of areas of the lower body. One of the more concerning causes of inguinal lymphadenopathy is sexually transmitted infections. Sexually transmitted infections that commonly presents with inguinal lymphadenopathy are lymphogranuloma venereum, secondary syphilis, and chancroid caused by Chlamydia trachomatis (L1-L3), Treponema pallidum , and Haemophilus ducreyi , respectively. Lymphogranuloma venereum Diagnosis Although most genital ulcerations cannot be diagnosed confidently on clinical grounds alone, clinical findings (Table 163-7) FIGURE 163-5 Chancroid: multiple, painful, punched-out ulcers with undermined borders on the labia occurring after autoinoculation. Sexually Transmitted Infections: Overview and Clinical Approach FIGURE 163-6 Genital herpes. A relatively mild, superficial ulcer is typically seen in episodic outbreaks. (Courtesy of Michael Remington, University of Washington Virology Research Clinic.) FIGURE 163-7 Lymphogranuloma venereum (LGV): striking ten-der lymphadenopathy occurring at the femoral and inguinal lymph nodes, separated by a groove made by Poupart’s ligament. This “sign-of-the-groove” is not considered specific for LGV; for example, lym-phomas may present with this sign. InITIAL MAnAgEMEnT of gEnITAL oR PERIAnAL uLCER The secondary stage most often occurs 10–30 days later, but can present up to six months later. The infection spreads to the lymph nodes through lymphatic drainage pathways. The most frequent presenting clinical manifestation of LGV among males whose primary exposure was genital is unilateral (in two-thirds of cases) lymphadenitis and lymphangitis, often with tender inguinal and/or femoral lymphadenopathy because of the drainage pathway for their likely infected areas. Lymphangitis of the dorsal penis may also occur and resembles a string or cord. If the route was anal sex, the infected person may experience lymphadenitis and lymphangitis noted above. They may instead develop proctitis, inflammation limited to the rectum (the distal 10–12 cm) that may be associated with anorectal pain, tenesmus, and rectal discharge, or proctocolitis, inflammation of the colonic mucosa extending to 12 cm above the anus and associated with symptoms of proctitis plus diarrhea or abdominal cramps. Several clinical presentations are highly suggestive of specific diagnoses: 1. A painless and minimally tender ulcer, not accompanied by inguinal lymphadenopathy, is likely to be syphilis, especially if the ulcer is indurated. A nontreponemal rapid plasma reagin (RPR) test, or venereal disease research laboratory (VDRL) test, and a confirmatory treponemal test—fluorescent treponemal antibody absorption (FTA ABS) or microhemagglutinin–T. pallidum (MHA TP)—should be used to diagnose syphilis presumptively. Some laboratories screen samples with treponemal enzyme immunoassay (EIA) tests, the results of which should be confirmed with nontreponemal tests. The results of nontreponemal tests usually correlate with disease activity and should be reported quantitatively. 2. Treponema pallidum : Syphilis infections result from Treponema pallidum . This infection usually manifests as a painless chancre in the primary stage. If the disease is left untreated, it will progress to the secondary stage. In the secondary stage, it manifests as fever, widespread maculopapular skin rashes involving the palms and soles,  widespread lymphadenopathy (epitrochlear node is pathognomic), and genital lesions similar to genital warts (condylomata lata- has a rounder surface when compared with condylomata acuminata). If there is still no treatment during the secondary stage, the infection will progress into the tertiary stage. The tertiary stage causes necrotic lesions called Gummas, neurological symptoms such as tabes dorsalis, Argyll Robertson pupils, and general paresis, cardiac symptoms such as aortitis. The treatment of syphilis is with the use of penicillin. Source: From RM Ballard, in KK Holmes et al (eds): Sexually Transmitted Diseases, 4th ed. New York, McGraw-Hill, 2008. HSV-1 from HSV-2 has prognostic implications, because the latter causes more frequent genital recurrences. Painless, nontender, indurated ulcers with firm, nontender inguinal adenopathy suggest primary syphilis. If results of dark-field examination and a rapid serologic test for syphilis are initially negative, presumptive therapy should be provided on the basis of the individual’s risk. For example, with increasing rates of syphilis among MSM in the United States, most experts would not withhold therapy for this infection pending watchful waiting and/or subsequent detection of seroconversion. Repeated serologic testing for syphilis 1 or 2 weeks after treatment of seronegative primary syphilis usually demonstrates seroconversion. Diagnosis The diagnosis usually is made serologically (through complement fixation) and by exclusion of other causes of inguinal lymphadenopathy or genital ulcers. Serologic testing has a sensitivity of 80% after two weeks. Serologic testing may not be specific for serotype (has some cross reactivity with other chlamydia species) and can suggest LGV from other forms because of their difference in dilution, 1:64 more likely to be LGV and lower than 1:16 is likely to be other chlamydia forms (emedicine). For identification of serotypes, culture is often used. Culture is difficult. Requiring a special medium, cycloheximide-treated McCoy or HeLa cells, and yields are still only 30-50%. DFA, or direct fluorescent antibody test, PCR of likely infected areas and pus, are also sometimes used. DFA test for the L-type serovar of C. trachomatis is the most sensitive and specific test, but is not readily available. Demonstration of H. ducreyi by culture (or by PCR, where available) is most useful when ulcers are painful and purulent, especially if inguinal lymphadenopathy with fluctuance or overlying erythema is noted; if chancroid is prevalent in the community; or if the patient has recently had a sexual exposure elsewhere in a chancroid-endemic area (e.g., a developing country). Enlarged, fluctuant lymph nodes should be aspirated for culture or PCR to detect H. ducreyi as well as for Gram’s staining and culture to rule out the presence of other pyogenic bacteria. Syphilis is a bacterial infection caused by the spirochete Treponema pallidum. Clinical presentation of secondary syphilis can present with diffuse lymphadenopathy along with fever, skin rashes, and condylomata lata. [7] Chancroid Lymphadenopathy may be an incidental finding in patients being examined for various reasons, or it may be a presenting sign or symptom of the patient’s illness. The physician must eventually decide whether the lymphadenopathy is a normal finding or one that requires further study, up to and including biopsy. Soft, flat, submandibular nodes (<1 cm) are often palpable in healthy children and young adults; healthy adults may have palpable inguinal nodes of up to 2 cm, which are considered normal. Further evaluation of these normal nodes is not warranted. In contrast, if the physician believes the node(s) to be abnormal, then pursuit of a more precise diagnosis is needed. APPROACH TO THE PATIENT: Approach to the Patient with an Infectious Disease regions (e.g., popliteal, inguinal, epitrochlear, axillary, multiple cervical regions), with notation of the location, size (normal, <1 cm), presence or absence of tenderness, and consistency (soft, firm, or shotty) and of whether the nodes are matted (i.e., connected and moving together). Of note, palpable epitrochlear nodes are always pathologic. Of patients presenting with lymphadenopathy, 75% have localized findings, and the remaining 25% have generalized lymphadenopathy (i.e., that involving more than one anatomic region). Localized lymphadenopathy in the head and neck region is found in 55% of patients, inguinal lymphadenopathy in 14%, and axillary lymphadenopathy in 5%. Determining whether the patient has generalized versus localized lymphadenopathy can help narrow the differential diagnosis, as various infections present differently. Females and males: Signs and symptoms: Patients will present with highly vascularized lesions over the genitals and perineum that tend to be painless. [37] [38] It can cause severe scarring. Physical Exam: Typical findings include ulcer-like lesions that are beefy red, consistent with high vascularization that bleeds easily with manipulation. Subcutaneous granulomas may be present, but lymphadenopathy is uncommon. The lesions tend to be relatively large and irregular. It is often found to be associated with secondary infections. Four main lesions can be seen on examination: 1. Ulcerovegetative: large painless ulcer on the patient's physical exam. 2. Nodular: soft and erythematous that tend to ulcerate throughout the infectious process. 3. Cicatricial: dry ulcerations that tend to transition into plaques. 4. Hypertrophic: lesions are thick and painless. [37] [39] node is an enlarged left supraclavicular node infiltrated with metastatic cancer from a gastrointestinal primary. Metastases to supraclavicular nodes also occur from lung, breast, testis, or ovarian cancers. Tuberculosis, sarcoidosis, and toxoplasmosis are nonneoplastic causes of supraclavicular adenopathy. Axillary adenopathy is usually due to injuries or localized infections of the ipsilateral upper extremity. Malignant causes include melanoma or lymphoma and, in women, breast cancer. Inguinal lymphadenopathy is usually secondary to infections or trauma of the lower extremities and may accompany sexually transmitted diseases such as lymphogranuloma venereum, primary syphilis, genital herpes, or chancroid. These nodes may also be involved by lymphomas and metastatic cancer from primary lesions of the rectum, genitalia, or lower extremities (melanoma). Serological tests cannot distinguish yaws from the closely related syphilis; no test distinguishing yaws from syphilis is widely available. The two genomes differ by about 0.2%. PCR and DNA sequencing can distinguish the two. There are also no common blood tests which distinguish among the four treponematoses: syphilis (Treponema pallidum pallidum), yaws (Treponema pallidum pertenue), bejel (Treponema pallidum endemicum), and pinta (Treponema carateum). Haemophilus ducreyi infections can cause skin conditions that mimic primary yaws. People infected with Haemophilus ducreyi lesions may or may not also have latent yaws, and thus may or may not test positive on serological tests. This was discovered in the mid 2010s. It seems that a recently diverged strain of Haemophilus ducreyi has evolved from being a sexually transmitted infection to being a skin ulcer pathogen that looks like yaws. Yaws has been reported in nonendemic countries. Females and males: Signs and symptoms: Patients will present with painful lymphadenopathy localized to the inguinal area. Patients may note the initial presentation of a pustule that gradually progresses to large painful ulceration. [41] Men tend to present with early or acute stages, while women typically present much later. [19] Physical Exam: Lymphogranuloma venereum presents with two stages: Primary phase is a small painless papule/pustule that will ulcerate and can be visualized throughout the affected genital area. During the secondary phase, patients present with unilateral lymphadenopathy that is fluctuant with palpation or may be suppurative in a presentation known as buboes. [38] Buboes tend to rupture in the acute phase and progress to a thickened mass. [42] A 36-year-old male sustained fracture of first lumbar vertebra, splenic tear and paraplegia in a motorcycle accident in 2001; splenectomy was performed. In 2008, he presented with temperature and feeling rough. With a diagnosis of urine infection, he was prescribed ciprofloxacin, followed by trimethoprim, amoxicillin, and gentamicin, as temperature did not subside. White cell count was 21.2 x 109/L; lymphocytes were 13.05 x 109/L (1.00 - 4.00). Therefore, computerised tomography (CT) of chest and abdomen was performed. Thrombus was present in pulmonary arteries bilaterally involving the lobar and segmental branches. Enlarged lymph nodes were seen in axillae, chest, abdomen and inguinal regions. Radiological diagnosis was lymphoma. Cell marker showed an excess of large granular lymphocytes and activated lymphocytes. The Glandular Fever Slide Test was positive. Subsequently, Paul Bunnell test was also positive. Epstein Barr virus serology was consistent with recent Epstein Barr virus infection. Antibiotic was omitted; enoxaparin was prescribed for pulmonary artery thrombosis. Learning points from this case: (1) Although routine administration of antibiotic to a spinal cord injury patient with pyrexia may be acceptable in outpatient setting, other possibilities such as infection by multi-drug resistant organism, viral infection, venous or, arterial thrombosis should be considered if a patient does not respond promptly to antibacterial therapy. (2) When full blood count showed lymphocytosis (comprising > 50% of white blood cells) with atypical morphology, lymphocyte surface markers, Paul Bunnell test, and Epstein Barr virus serology should be performed. These tests would have led to a diagnosis of infectious mononucleosis, and abdominal imaging studies could have been avoided. (3) Lymphoid hyperplasia is the hallmark of infectious mononucleosis; therefore, we should have suspected glandular fever rather than lymphoma when CT scan revealed enlarged lymph nodes in abdomen, mediastinum, axillae and inguinal regions in this patient, who had lymphocytosis with atypical morphology. (4) A soft tissue mass, situated inferior to left hemidiaphragm in this asplenic patient, was misinterpreted as lymph nodes; review of CT led to the correct diagnosis of splenunculus. (5) Acute infection with Epstein Barr virus may lead to transient induction of anti-phospholipid antibodies, which can cause vascular thrombosis. (6) This case illustrates the value of reviewing test results and discussion with senior doctors, as these measures help to recognize medical errors and improve patient care. In a private room or space, the patient will partially undress. The clinician may inspect the patient's: Throat and lymph nodes of the neck for inflammation Pubic hair for lice Lymph nodes of the groin for swelling Genitals, anus, and surrounding areas for sores and warts The clinician may swab the patient's: Throat to test for gonorrhea and possibly chlamydia Cheek, inside, to diagnose HIV Sores of the genitals, anus, and surrounding areas to test for herpes Urethra to test for gonorrhea and possibly chlamydia Vagina to test for chlamydia and possibly gonorrhea Cervix to test for cervical intraepithelial neoplasia (a Pap test) Rectum to test for gonorrhea and possibly chlamydia The clinician may take small blood samples by pricking a finger or from a vein to test for HIV, syphilis, and possibly herpes and hepatitis C. Dear Editor, Syphilis is an infection caused by Treponema pallidum. Without treatment, it goes through the following stages: primary, secondary, latent, and tertiary (1). The clinical picture of secondary syphilis is very variable (2,3). We present two rare cases of secondary syphilis, one with nodular lesions initially considered to be lymphoma and second with periostitis, which was initially interpreted as an osteoma. To date, only 15 cases with nodular lesions and 10 cases with periostitis in secondary syphilis have been reported in the literature. The first patient was a 59 year old man who presented in a private practice with nodular lesions on the face and axillary and inguinal folds (Figure 1, a, b). The initial diagnostic consideration was lymphoma. A biopsy specimen was taken, and the histopathological features revealed epidermal hyperplasia with papillomatosis, minimal spongiosis with many neutrophils and with a marked inflammatory infiltrate in dermis, consisting of lymphocytes, plasma cells, and neutrophils; the diagnosis of interfaced dermatitis was established (Figure 1, d, e). After one month, the patient presented to our clinic with numerous nodular lesions, some of them painful, located on the trunk and intertriginous folds, including the intergluteal cleft - the lesions in this area being suggestive of condylomata lata (Figure 1, c). The diagnosis of secondary syphilis was taken into consideration, and screening serum tests were performed and found reactive: a Venereal Diseases Research Laboratory (VDRL) titer of 1:64 and Treponema pallidum Hemaglutination Assay (TPHA) titer of 1:80. Hepatitis and anti-human immunodeficiency virus (HIV) antibodies serology was negative. The biopsy was repeated and showed the same histopathological changes. In addition, Warthin-Starry staining was performed, revealing the presence of some spiral micro-organisms in the dermis corresponding to Treponema pallidum (Figure 1, f). A diagnosis of secondary syphilis was established, and the patient was treated with benzathine penicillin G 2.4 million units by intramuscular injection once a week for 2 consecutive weeks. The skin lesions regressed within 1 month, and serological tests showed a VDRL titer of 1:8 3 months after treatment. The second patient was a homosexual male, 35 years old, diagnosed with HIV infection, stage B2. He presented with bone pain in the calves and forearms, with insidious onset. He also presented with an associated erythematous maculo-papular rash on the trunk and limbs and generalized lymphadenopathy (Figure 2, a, b). The tibial crest and radius were sensitive to palpation. A right leg radiography was performed, raising suspicion of osteoid osteoma. The CT scan excluded the diagnosis of osteoma; taking into account the epidemiological context, the diagnosis of syphilis was suspected. The diagnosis was confirmed by leg ultrasound examination (2D US) which showed thickening of the compact tibial bone associated with subperiosteal destructive and proliferative changes (Figure 2, c, d) and by serology for syphilis: the VDRL titer was 1:32 and the TPHA titer was 1:80. The patient was treated with benzathine penicillin 2.4 million units, once a week, for 2 consecutive weeks, with clinical improvement. Syphilis continues to be a serious public health problem worldwide, even if it is a controllable disease due to diagnostic tests and effective and accessible treatment. According to the World Health Organization in 2008, the estimated number of new cases of sexually transmitted diseases in adults with syphilis is 10.6 million cases (4). The cases presented in this paper were characterized by unusual manifestations, requiring good collaboration between the dermatologist and other specialties. In the first case, the diagnosis of secondary syphilis was confirmed by positive serological, clinical, and histopathological findings. The main differential diagnosis of nodular syphilis includes lymphoma, sarcoidosis, Kaposi's sarcoma, atypical mycobacteriosis, deep fungal infections, leprosy, tuberculosis, leishmaniasis, and lymphomatoid papulosis (5). Another important differential diagnosis is between secondary and tertiary syphilis, especially when ulcerating nodules are present. Tertiary syphilis is characterized by unilateral, deep ulcerating nodules with necrotizing granulomas (6). Bone involvement during syphilis is mainly represented by polyarthritis, synovitis, osteitis, and periostitis (7,8). Syphilitic periostitis is characterized by localized or diffuse pain, particularly during the night, which is relieved by movement. The skull, the shoulder girdle, and the long bones are the most common sites of involvement (9). In conclusion, we presented two different cases of secondary syphilis that contribute to the clinical experience of rare cases presented in the literature, raising the awareness of dermatologists and other specialists about less specific clinical aspects of syphilis. adenopathy that is sometimes mistaken for malignancy; syphilis, LGV, HSV infection, and chancroid involving the anus can produce inguinal adenopathy because anal lymphatics drain to inguinal lymph nodes. Cervical lymphadenopathy is a sign or a symptom, not a diagnosis. The causes are varied, and may be inflammatory, degenerative, or neoplastic. In adults, healthy lymph nodes can be palpable (able to be felt), in the axilla, neck and groin. In children up to the age of 12 cervical nodes up to 1 cm in size may be palpable and this may not signify any disease. If nodes heal by resolution or scarring after being inflamed, they may remain palpable thereafter. In children, most palpable cervical lymphadenopathy is reactive or infective. In individuals over the age of 50, metastatic enlargement from cancers (most commonly squamous cell carcinomas) of the aerodigestive tract should be considered. Classification Cervical lymphadenopathy can be thought of as local where only the cervical lymph nodes are affected, or general where all the lymph nodes of the body are affected. Causes Sexually transmitted infections are the first to consider in the differential diagnosis of LGV. The exclusion of diseases which cause genital ulceration and inguinal adenopathy including herpes simplex, syphilis, chancroid, herpes, and granuloma inguinale will help narrow the diagnosis. HIV and lymphoma can also cause generalized lymphadenopathy. Dermatological conditions and trauma which can cause genital ulcerations should also be in the differential diagnosis. Lymph node enlargement is recognized as a common sign of infectious, autoimmune, or malignant disease. Examples may include: Reactive: acute infection (e.g., bacterial, or viral), or chronic infections (tuberculous lymphadenitis, cat-scratch disease). The most distinctive sign of bubonic plague is extreme swelling of one or more lymph nodes that bulge out of the skin as \"buboes.\" The buboes often become necrotic and may even rupture. Infectious mononucleosis is an acute viral infection usually caused by Epstein-Barr virus and may be characterized by a marked enlargement of the cervical lymph nodes. It is also a sign of cutaneous anthrax and Human African trypanosomiasis Toxoplasmosis, a parasitic disease, gives a generalized lymphadenopathy (Piringer-Kuchinka lymphadenopathy). Plasma cell variant of Castleman's disease - associated with HHV-8 infection and HIV infection A 38-year-old man who had sex with men, presented at the outpatient department for Sexually Transmitted Diseases in Amsterdam with a painful, red, fluctuating swelling in the left groin and general discomfort. He had been sexually active in the population of men who have sex with men, in which an anorectal lymphogranuloma venereum (LGV) epidemic has recently been discovered. Unlike other cases where there was anorectal involvement, this patient was the first case of LGV with the classical inguinal presentation although he had not visited the tropics where the inguinal form of LGV occurs as an STD. Routine investigation using PCR on material from urethra and rectum and from the urine, repeatedly failed to detect LGV. However, PCR on pus aspirated from the enlarged lymph node demonstrated Chlamydia trachomatis serovar type L2. Treatment with doxycycline 100 mg twice daily was started. This case illustrates that routine analysis from urethra and rectum and of urine may fail to detect LGV. Furthermore, this case of a patient who probably had LGV initially in the urethra may be the missing link in explaining the route of transmission of the anorectal LGV epidemic. The presentation of chancroid does not usually include all of the typical clinical features and is sometimes atypical. Multiple ulcers can coalesce to form giant ulcers. Ulcers can appear and then resolve, with inguinal adenitis (Fig. 182-2) and suppuration following 1–3 weeks later; this clinical picture can be confused with that of lymphogranuloma venereum (Chap. 213). Multiple small ulcers can resemble folliculitis. Other differential diagnostic considerations include the various infections causing genital ulceration, such as primary syphilis, secondary syphilis (condyloma latum), genital herpes, and donovanosis. In rare cases, chancroid lesions become secondarily infected with bacteria; the result is extensive inflammation. FIGURE 182-2 Chancroid with characteristic penile ulcers and associated left inguinal adenitis (bubo). A 33-year-old man presented with a two-week history of an asymptomatic ulcer of the oropharynx and submandibular lymph nodes swelling. Laboratory examinations were normal, but serological tests revealed positivity for rapid plasma reagin, Treponema pallidum haemagglutination assay and anti-T. pallidum IgM antibodies. Since the patient denied any homosexual relationship, a biopsy of the lesion was performed, which confirmed primary syphilis. The patient received an intramuscular injection of Benzathine Penicillin G (2.4 MU) with complete resolution of the lesion. Extragenital chancres occur in at least 5% of patients with primary syphilis, and the oral mucosa is the most frequent location as a consequence of orogenital/oroanal contact with an infectious lesion. Because of their transient nature, these oral ulcerations are often underestimated by the patient or by any unsuspecting clinician. Health professionals should consider the recent sexual history of their patients and should be prepared to recognise oral and systemic manifestations of sexually transmitted infections. \nHere is the question:\nINFECTIOUS DISEASES: A 24-year-old woman consults after noticing inguinal lymphadenopathy. The interrogation does not reveal the presence of any local discomfort or data suggestive of sexually transmitted infection. The examination revealed two lymphadenopathies, one in each groin, 1 cm in diameter, soft, mobile, non-painful. There are no skin lesions on the lower limbs, anus or perineum. Which test do you consider essential?\nHere are the potential choices:\n1. A lues serology since it is most likely a Treponema pallidum infection.\n2. A gynecological examination to rule out ovarian cancer.\n3. By the clinical characteristics it seems to be normal lymph nodes and complementary explorations should not be done.\n4. A Paul-Bunnell test should be performed in order to rule out infectious mononucleosis.\nThe correct answer is: ", "gold_answer": "3 By the clinical characteristics it seems to be normal lymph nodes and complementary explorations should not be done.", "prediction": "You are a helpful medical expert, and your task is to answer a multi-choice medical question using the relevant documents. Please choose the answer from the provided options. Your responses will be used for research purposes only, so please have a definite answer.\nHere are the relevant documents:\n Inguinal lymphadenopathy causes swollen lymph nodes in the groin area. It can be a symptom of infective or neoplastic processes. Infective aetiologies include Tuberculosis, HIV, non-specific or reactive lymphadenopathy to recent lower limb infection or groin infections. Another notable infectious cause is Lymphogranuloma venereum, which is a sexually transmitted infection of the lymphatic system. Neoplastic aetiologies include lymphoma, leukaemia and metastatic disease from primary tumours in the lower limb, external genitalia or perianal region and melanoma. References Further reading Inflammations Diseases of veins, lymphatic vessels and lymph nodes Overview A genital ulcer may be located on the vulva, penis, perianal region, or anus. Globally, the incidence of genital ulcers is estimated to be approximately 20 million cases annually. The most likely cause of a genital ulcer varies depending on the characteristics of a population and location. The most common cause of genital ulcers in the United States is herpes simplex infections, with syphilis the second most common cause, and chancroid the third. These common causes of genital ulcer disease (HSV-1, HSV-2 and treponema pallidum) can all be efficiently transmitted through oral sex. Important signs associated with genital ulcers that may assist in the diagnosis of the cause of the genital ulcer may include the presence of tender or non-tender enlarged lymph nodes in the groin area, a painful or non-painful genital ulcer, or the presence of vesicular lesions, which are small, painful, elevated blisters. Syphilis (lues) is a sexually transmitted infection caused by the spirochete Treponema pallidum. In adolescents, the diagnosis of primary syphilis can be made promptly by taking a sexual medical history and inspecting the glans penis. A 17-year-old male was referred to the paediatric oncology centre for additional diagnostics due to inguinal lymphadenopathy, with a strong suspicion of a malignant lymphoma. None of the physicians took a sexual medical history or investigated the glans penis, as a result of which essential information was lacking. The combination of inguinal lymphadenopathy and the ultrasound findings for the inguinal region made the physicians only consider a malignancy. However, it actually concerned a reactive lymphadenopathy associated with primary syphilis. This case demonstrates that a full medical history and thorough physical examination can prevent the need for costly and invasive diagnostics. Clinical significance The presence of swollen inguinal lymph nodes is an important clinical sign because lymphadenopathy (swelling) may indicate an infection, or spread as a metastasis from cancers, such as anal cancer and vulvar cancer. Inguinal lymph nodes may normally be up to 2 cm. The cut-off value for normal sized inguinal nodes is up to 10 mm. Additional images References Lymphatic organ anatomy Swollen lymph nodes usually indicate infection from bacteria or viruses. Swollen inguinal lymph nodes could indicate an infection of areas of the lower body. One of the more concerning causes of inguinal lymphadenopathy is sexually transmitted infections. Sexually transmitted infections that commonly presents with inguinal lymphadenopathy are lymphogranuloma venereum, secondary syphilis, and chancroid caused by Chlamydia trachomatis (L1-L3), Treponema pallidum , and Haemophilus ducreyi , respectively. Lymphogranuloma venereum Diagnosis Although most genital ulcerations cannot be diagnosed confidently on clinical grounds alone, clinical findings (Table 163-7) FIGURE 163-5 Chancroid: multiple, painful, punched-out ulcers with undermined borders on the labia occurring after autoinoculation. Sexually Transmitted Infections: Overview and Clinical Approach FIGURE 163-6 Genital herpes. A relatively mild, superficial ulcer is typically seen in episodic outbreaks. (Courtesy of Michael Remington, University of Washington Virology Research Clinic.) FIGURE 163-7 Lymphogranuloma venereum (LGV): striking ten-der lymphadenopathy occurring at the femoral and inguinal lymph nodes, separated by a groove made by Poupart’s ligament. This “sign-of-the-groove” is not considered specific for LGV; for example, lym-phomas may present with this sign. InITIAL MAnAgEMEnT of gEnITAL oR PERIAnAL uLCER The secondary stage most often occurs 10–30 days later, but can present up to six months later. The infection spreads to the lymph nodes through lymphatic drainage pathways. The most frequent presenting clinical manifestation of LGV among males whose primary exposure was genital is unilateral (in two-thirds of cases) lymphadenitis and lymphangitis, often with tender inguinal and/or femoral lymphadenopathy because of the drainage pathway for their likely infected areas. Lymphangitis of the dorsal penis may also occur and resembles a string or cord. If the route was anal sex, the infected person may experience lymphadenitis and lymphangitis noted above. They may instead develop proctitis, inflammation limited to the rectum (the distal 10–12 cm) that may be associated with anorectal pain, tenesmus, and rectal discharge, or proctocolitis, inflammation of the colonic mucosa extending to 12 cm above the anus and associated with symptoms of proctitis plus diarrhea or abdominal cramps. Several clinical presentations are highly suggestive of specific diagnoses: 1. A painless and minimally tender ulcer, not accompanied by inguinal lymphadenopathy, is likely to be syphilis, especially if the ulcer is indurated. A nontreponemal rapid plasma reagin (RPR) test, or venereal disease research laboratory (VDRL) test, and a confirmatory treponemal test—fluorescent treponemal antibody absorption (FTA ABS) or microhemagglutinin–T. pallidum (MHA TP)—should be used to diagnose syphilis presumptively. Some laboratories screen samples with treponemal enzyme immunoassay (EIA) tests, the results of which should be confirmed with nontreponemal tests. The results of nontreponemal tests usually correlate with disease activity and should be reported quantitatively. 2. Treponema pallidum : Syphilis infections result from Treponema pallidum . This infection usually manifests as a painless chancre in the primary stage. If the disease is left untreated, it will progress to the secondary stage. In the secondary stage, it manifests as fever, widespread maculopapular skin rashes involving the palms and soles,  widespread lymphadenopathy (epitrochlear node is pathognomic), and genital lesions similar to genital warts (condylomata lata- has a rounder surface when compared with condylomata acuminata). If there is still no treatment during the secondary stage, the infection will progress into the tertiary stage. The tertiary stage causes necrotic lesions called Gummas, neurological symptoms such as tabes dorsalis, Argyll Robertson pupils, and general paresis, cardiac symptoms such as aortitis. The treatment of syphilis is with the use of penicillin. Source: From RM Ballard, in KK Holmes et al (eds): Sexually Transmitted Diseases, 4th ed. New York, McGraw-Hill, 2008. HSV-1 from HSV-2 has prognostic implications, because the latter causes more frequent genital recurrences. Painless, nontender, indurated ulcers with firm, nontender inguinal adenopathy suggest primary syphilis. If results of dark-field examination and a rapid serologic test for syphilis are initially negative, presumptive therapy should be provided on the basis of the individual’s risk. For example, with increasing rates of syphilis among MSM in the United States, most experts would not withhold therapy for this infection pending watchful waiting and/or subsequent detection of seroconversion. Repeated serologic testing for syphilis 1 or 2 weeks after treatment of seronegative primary syphilis usually demonstrates seroconversion. Diagnosis The diagnosis usually is made serologically (through complement fixation) and by exclusion of other causes of inguinal lymphadenopathy or genital ulcers. Serologic testing has a sensitivity of 80% after two weeks. Serologic testing may not be specific for serotype (has some cross reactivity with other chlamydia species) and can suggest LGV from other forms because of their difference in dilution, 1:64 more likely to be LGV and lower than 1:16 is likely to be other chlamydia forms (emedicine). For identification of serotypes, culture is often used. Culture is difficult. Requiring a special medium, cycloheximide-treated McCoy or HeLa cells, and yields are still only 30-50%. DFA, or direct fluorescent antibody test, PCR of likely infected areas and pus, are also sometimes used. DFA test for the L-type serovar of C. trachomatis is the most sensitive and specific test, but is not readily available. Demonstration of H. ducreyi by culture (or by PCR, where available) is most useful when ulcers are painful and purulent, especially if inguinal lymphadenopathy with fluctuance or overlying erythema is noted; if chancroid is prevalent in the community; or if the patient has recently had a sexual exposure elsewhere in a chancroid-endemic area (e.g., a developing country). Enlarged, fluctuant lymph nodes should be aspirated for culture or PCR to detect H. ducreyi as well as for Gram’s staining and culture to rule out the presence of other pyogenic bacteria. Syphilis is a bacterial infection caused by the spirochete Treponema pallidum. Clinical presentation of secondary syphilis can present with diffuse lymphadenopathy along with fever, skin rashes, and condylomata lata. [7] Chancroid Lymphadenopathy may be an incidental finding in patients being examined for various reasons, or it may be a presenting sign or symptom of the patient’s illness. The physician must eventually decide whether the lymphadenopathy is a normal finding or one that requires further study, up to and including biopsy. Soft, flat, submandibular nodes (<1 cm) are often palpable in healthy children and young adults; healthy adults may have palpable inguinal nodes of up to 2 cm, which are considered normal. Further evaluation of these normal nodes is not warranted. In contrast, if the physician believes the node(s) to be abnormal, then pursuit of a more precise diagnosis is needed. APPROACH TO THE PATIENT: Approach to the Patient with an Infectious Disease regions (e.g., popliteal, inguinal, epitrochlear, axillary, multiple cervical regions), with notation of the location, size (normal, <1 cm), presence or absence of tenderness, and consistency (soft, firm, or shotty) and of whether the nodes are matted (i.e., connected and moving together). Of note, palpable epitrochlear nodes are always pathologic. Of patients presenting with lymphadenopathy, 75% have localized findings, and the remaining 25% have generalized lymphadenopathy (i.e., that involving more than one anatomic region). Localized lymphadenopathy in the head and neck region is found in 55% of patients, inguinal lymphadenopathy in 14%, and axillary lymphadenopathy in 5%. Determining whether the patient has generalized versus localized lymphadenopathy can help narrow the differential diagnosis, as various infections present differently. Females and males: Signs and symptoms: Patients will present with highly vascularized lesions over the genitals and perineum that tend to be painless. [37] [38] It can cause severe scarring. Physical Exam: Typical findings include ulcer-like lesions that are beefy red, consistent with high vascularization that bleeds easily with manipulation. Subcutaneous granulomas may be present, but lymphadenopathy is uncommon. The lesions tend to be relatively large and irregular. It is often found to be associated with secondary infections. Four main lesions can be seen on examination: 1. Ulcerovegetative: large painless ulcer on the patient's physical exam. 2. Nodular: soft and erythematous that tend to ulcerate throughout the infectious process. 3. Cicatricial: dry ulcerations that tend to transition into plaques. 4. Hypertrophic: lesions are thick and painless. [37] [39] node is an enlarged left supraclavicular node infiltrated with metastatic cancer from a gastrointestinal primary. Metastases to supraclavicular nodes also occur from lung, breast, testis, or ovarian cancers. Tuberculosis, sarcoidosis, and toxoplasmosis are nonneoplastic causes of supraclavicular adenopathy. Axillary adenopathy is usually due to injuries or localized infections of the ipsilateral upper extremity. Malignant causes include melanoma or lymphoma and, in women, breast cancer. Inguinal lymphadenopathy is usually secondary to infections or trauma of the lower extremities and may accompany sexually transmitted diseases such as lymphogranuloma venereum, primary syphilis, genital herpes, or chancroid. These nodes may also be involved by lymphomas and metastatic cancer from primary lesions of the rectum, genitalia, or lower extremities (melanoma). Serological tests cannot distinguish yaws from the closely related syphilis; no test distinguishing yaws from syphilis is widely available. The two genomes differ by about 0.2%. PCR and DNA sequencing can distinguish the two. There are also no common blood tests which distinguish among the four treponematoses: syphilis (Treponema pallidum pallidum), yaws (Treponema pallidum pertenue), bejel (Treponema pallidum endemicum), and pinta (Treponema carateum). Haemophilus ducreyi infections can cause skin conditions that mimic primary yaws. People infected with Haemophilus ducreyi lesions may or may not also have latent yaws, and thus may or may not test positive on serological tests. This was discovered in the mid 2010s. It seems that a recently diverged strain of Haemophilus ducreyi has evolved from being a sexually transmitted infection to being a skin ulcer pathogen that looks like yaws. Yaws has been reported in nonendemic countries. Females and males: Signs and symptoms: Patients will present with painful lymphadenopathy localized to the inguinal area. Patients may note the initial presentation of a pustule that gradually progresses to large painful ulceration. [41] Men tend to present with early or acute stages, while women typically present much later. [19] Physical Exam: Lymphogranuloma venereum presents with two stages: Primary phase is a small painless papule/pustule that will ulcerate and can be visualized throughout the affected genital area. During the secondary phase, patients present with unilateral lymphadenopathy that is fluctuant with palpation or may be suppurative in a presentation known as buboes. [38] Buboes tend to rupture in the acute phase and progress to a thickened mass. [42] A 36-year-old male sustained fracture of first lumbar vertebra, splenic tear and paraplegia in a motorcycle accident in 2001; splenectomy was performed. In 2008, he presented with temperature and feeling rough. With a diagnosis of urine infection, he was prescribed ciprofloxacin, followed by trimethoprim, amoxicillin, and gentamicin, as temperature did not subside. White cell count was 21.2 x 109/L; lymphocytes were 13.05 x 109/L (1.00 - 4.00). Therefore, computerised tomography (CT) of chest and abdomen was performed. Thrombus was present in pulmonary arteries bilaterally involving the lobar and segmental branches. Enlarged lymph nodes were seen in axillae, chest, abdomen and inguinal regions. Radiological diagnosis was lymphoma. Cell marker showed an excess of large granular lymphocytes and activated lymphocytes. The Glandular Fever Slide Test was positive. Subsequently, Paul Bunnell test was also positive. Epstein Barr virus serology was consistent with recent Epstein Barr virus infection. Antibiotic was omitted; enoxaparin was prescribed for pulmonary artery thrombosis. Learning points from this case: (1) Although routine administration of antibiotic to a spinal cord injury patient with pyrexia may be acceptable in outpatient setting, other possibilities such as infection by multi-drug resistant organism, viral infection, venous or, arterial thrombosis should be considered if a patient does not respond promptly to antibacterial therapy. (2) When full blood count showed lymphocytosis (comprising > 50% of white blood cells) with atypical morphology, lymphocyte surface markers, Paul Bunnell test, and Epstein Barr virus serology should be performed. These tests would have led to a diagnosis of infectious mononucleosis, and abdominal imaging studies could have been avoided. (3) Lymphoid hyperplasia is the hallmark of infectious mononucleosis; therefore, we should have suspected glandular fever rather than lymphoma when CT scan revealed enlarged lymph nodes in abdomen, mediastinum, axillae and inguinal regions in this patient, who had lymphocytosis with atypical morphology. (4) A soft tissue mass, situated inferior to left hemidiaphragm in this asplenic patient, was misinterpreted as lymph nodes; review of CT led to the correct diagnosis of splenunculus. (5) Acute infection with Epstein Barr virus may lead to transient induction of anti-phospholipid antibodies, which can cause vascular thrombosis. (6) This case illustrates the value of reviewing test results and discussion with senior doctors, as these measures help to recognize medical errors and improve patient care. In a private room or space, the patient will partially undress. The clinician may inspect the patient's: Throat and lymph nodes of the neck for inflammation Pubic hair for lice Lymph nodes of the groin for swelling Genitals, anus, and surrounding areas for sores and warts The clinician may swab the patient's: Throat to test for gonorrhea and possibly chlamydia Cheek, inside, to diagnose HIV Sores of the genitals, anus, and surrounding areas to test for herpes Urethra to test for gonorrhea and possibly chlamydia Vagina to test for chlamydia and possibly gonorrhea Cervix to test for cervical intraepithelial neoplasia (a Pap test) Rectum to test for gonorrhea and possibly chlamydia The clinician may take small blood samples by pricking a finger or from a vein to test for HIV, syphilis, and possibly herpes and hepatitis C. Dear Editor, Syphilis is an infection caused by Treponema pallidum. Without treatment, it goes through the following stages: primary, secondary, latent, and tertiary (1). The clinical picture of secondary syphilis is very variable (2,3). We present two rare cases of secondary syphilis, one with nodular lesions initially considered to be lymphoma and second with periostitis, which was initially interpreted as an osteoma. To date, only 15 cases with nodular lesions and 10 cases with periostitis in secondary syphilis have been reported in the literature. The first patient was a 59 year old man who presented in a private practice with nodular lesions on the face and axillary and inguinal folds (Figure 1, a, b). The initial diagnostic consideration was lymphoma. A biopsy specimen was taken, and the histopathological features revealed epidermal hyperplasia with papillomatosis, minimal spongiosis with many neutrophils and with a marked inflammatory infiltrate in dermis, consisting of lymphocytes, plasma cells, and neutrophils; the diagnosis of interfaced dermatitis was established (Figure 1, d, e). After one month, the patient presented to our clinic with numerous nodular lesions, some of them painful, located on the trunk and intertriginous folds, including the intergluteal cleft - the lesions in this area being suggestive of condylomata lata (Figure 1, c). The diagnosis of secondary syphilis was taken into consideration, and screening serum tests were performed and found reactive: a Venereal Diseases Research Laboratory (VDRL) titer of 1:64 and Treponema pallidum Hemaglutination Assay (TPHA) titer of 1:80. Hepatitis and anti-human immunodeficiency virus (HIV) antibodies serology was negative. The biopsy was repeated and showed the same histopathological changes. In addition, Warthin-Starry staining was performed, revealing the presence of some spiral micro-organisms in the dermis corresponding to Treponema pallidum (Figure 1, f). A diagnosis of secondary syphilis was established, and the patient was treated with benzathine penicillin G 2.4 million units by intramuscular injection once a week for 2 consecutive weeks. The skin lesions regressed within 1 month, and serological tests showed a VDRL titer of 1:8 3 months after treatment. The second patient was a homosexual male, 35 years old, diagnosed with HIV infection, stage B2. He presented with bone pain in the calves and forearms, with insidious onset. He also presented with an associated erythematous maculo-papular rash on the trunk and limbs and generalized lymphadenopathy (Figure 2, a, b). The tibial crest and radius were sensitive to palpation. A right leg radiography was performed, raising suspicion of osteoid osteoma. The CT scan excluded the diagnosis of osteoma; taking into account the epidemiological context, the diagnosis of syphilis was suspected. The diagnosis was confirmed by leg ultrasound examination (2D US) which showed thickening of the compact tibial bone associated with subperiosteal destructive and proliferative changes (Figure 2, c, d) and by serology for syphilis: the VDRL titer was 1:32 and the TPHA titer was 1:80. The patient was treated with benzathine penicillin 2.4 million units, once a week, for 2 consecutive weeks, with clinical improvement. Syphilis continues to be a serious public health problem worldwide, even if it is a controllable disease due to diagnostic tests and effective and accessible treatment. According to the World Health Organization in 2008, the estimated number of new cases of sexually transmitted diseases in adults with syphilis is 10.6 million cases (4). The cases presented in this paper were characterized by unusual manifestations, requiring good collaboration between the dermatologist and other specialties. In the first case, the diagnosis of secondary syphilis was confirmed by positive serological, clinical, and histopathological findings. The main differential diagnosis of nodular syphilis includes lymphoma, sarcoidosis, Kaposi's sarcoma, atypical mycobacteriosis, deep fungal infections, leprosy, tuberculosis, leishmaniasis, and lymphomatoid papulosis (5). Another important differential diagnosis is between secondary and tertiary syphilis, especially when ulcerating nodules are present. Tertiary syphilis is characterized by unilateral, deep ulcerating nodules with necrotizing granulomas (6). Bone involvement during syphilis is mainly represented by polyarthritis, synovitis, osteitis, and periostitis (7,8). Syphilitic periostitis is characterized by localized or diffuse pain, particularly during the night, which is relieved by movement. The skull, the shoulder girdle, and the long bones are the most common sites of involvement (9). In conclusion, we presented two different cases of secondary syphilis that contribute to the clinical experience of rare cases presented in the literature, raising the awareness of dermatologists and other specialists about less specific clinical aspects of syphilis. adenopathy that is sometimes mistaken for malignancy; syphilis, LGV, HSV infection, and chancroid involving the anus can produce inguinal adenopathy because anal lymphatics drain to inguinal lymph nodes. Cervical lymphadenopathy is a sign or a symptom, not a diagnosis. The causes are varied, and may be inflammatory, degenerative, or neoplastic. In adults, healthy lymph nodes can be palpable (able to be felt), in the axilla, neck and groin. In children up to the age of 12 cervical nodes up to 1 cm in size may be palpable and this may not signify any disease. If nodes heal by resolution or scarring after being inflamed, they may remain palpable thereafter. In children, most palpable cervical lymphadenopathy is reactive or infective. In individuals over the age of 50, metastatic enlargement from cancers (most commonly squamous cell carcinomas) of the aerodigestive tract should be considered. Classification Cervical lymphadenopathy can be thought of as local where only the cervical lymph nodes are affected, or general where all the lymph nodes of the body are affected. Causes Sexually transmitted infections are the first to consider in the differential diagnosis of LGV. The exclusion of diseases which cause genital ulceration and inguinal adenopathy including herpes simplex, syphilis, chancroid, herpes, and granuloma inguinale will help narrow the diagnosis. HIV and lymphoma can also cause generalized lymphadenopathy. Dermatological conditions and trauma which can cause genital ulcerations should also be in the differential diagnosis. Lymph node enlargement is recognized as a common sign of infectious, autoimmune, or malignant disease. Examples may include: Reactive: acute infection (e.g., bacterial, or viral), or chronic infections (tuberculous lymphadenitis, cat-scratch disease). The most distinctive sign of bubonic plague is extreme swelling of one or more lymph nodes that bulge out of the skin as \"buboes.\" The buboes often become necrotic and may even rupture. Infectious mononucleosis is an acute viral infection usually caused by Epstein-Barr virus and may be characterized by a marked enlargement of the cervical lymph nodes. It is also a sign of cutaneous anthrax and Human African trypanosomiasis Toxoplasmosis, a parasitic disease, gives a generalized lymphadenopathy (Piringer-Kuchinka lymphadenopathy). Plasma cell variant of Castleman's disease - associated with HHV-8 infection and HIV infection A 38-year-old man who had sex with men, presented at the outpatient department for Sexually Transmitted Diseases in Amsterdam with a painful, red, fluctuating swelling in the left groin and general discomfort. He had been sexually active in the population of men who have sex with men, in which an anorectal lymphogranuloma venereum (LGV) epidemic has recently been discovered. Unlike other cases where there was anorectal involvement, this patient was the first case of LGV with the classical inguinal presentation although he had not visited the tropics where the inguinal form of LGV occurs as an STD. Routine investigation using PCR on material from urethra and rectum and from the urine, repeatedly failed to detect LGV. However, PCR on pus aspirated from the enlarged lymph node demonstrated Chlamydia trachomatis serovar type L2. Treatment with doxycycline 100 mg twice daily was started. This case illustrates that routine analysis from urethra and rectum and of urine may fail to detect LGV. Furthermore, this case of a patient who probably had LGV initially in the urethra may be the missing link in explaining the route of transmission of the anorectal LGV epidemic. The presentation of chancroid does not usually include all of the typical clinical features and is sometimes atypical. Multiple ulcers can coalesce to form giant ulcers. Ulcers can appear and then resolve, with inguinal adenitis (Fig. 182-2) and suppuration following 1–3 weeks later; this clinical picture can be confused with that of lymphogranuloma venereum (Chap. 213). Multiple small ulcers can resemble folliculitis. Other differential diagnostic considerations include the various infections causing genital ulceration, such as primary syphilis, secondary syphilis (condyloma latum), genital herpes, and donovanosis. In rare cases, chancroid lesions become secondarily infected with bacteria; the result is extensive inflammation. FIGURE 182-2 Chancroid with characteristic penile ulcers and associated left inguinal adenitis (bubo). A 33-year-old man presented with a two-week history of an asymptomatic ulcer of the oropharynx and submandibular lymph nodes swelling. Laboratory examinations were normal, but serological tests revealed positivity for rapid plasma reagin, Treponema pallidum haemagglutination assay and anti-T. pallidum IgM antibodies. Since the patient denied any homosexual relationship, a biopsy of the lesion was performed, which confirmed primary syphilis. The patient received an intramuscular injection of Benzathine Penicillin G (2.4 MU) with complete resolution of the lesion. Extragenital chancres occur in at least 5% of patients with primary syphilis, and the oral mucosa is the most frequent location as a consequence of orogenital/oroanal contact with an infectious lesion. Because of their transient nature, these oral ulcerations are often underestimated by the patient or by any unsuspecting clinician. Health professionals should consider the recent sexual history of their patients and should be prepared to recognise oral and systemic manifestations of sexually transmitted infections. \nHere is the question:\nINFECTIOUS DISEASES: A 24-year-old woman consults after noticing inguinal lymphadenopathy. The interrogation does not reveal the presence of any local discomfort or data suggestive of sexually transmitted infection. The examination revealed two lymphadenopathies, one in each groin, 1 cm in diameter, soft, mobile, non-painful. There are no skin lesions on the lower limbs, anus or perineum. Which test do you consider essential?\nHere are the potential choices:\n1. A lues serology since it is most likely a Treponema pallidum infection.\n2. A gynecological examination to rule out ovarian cancer.\n3. By the clinical characteristics it seems to be normal lymph nodes and complementary explorations should not be done.\n4. A Paul-Bunnell test should be performed in order to rule out infectious mononucleosis.\nThe correct answer is: 3. By the clinical characteristics it seems to be normal lymph nodes and complementary explorations should not be done." }, { "question": "You are a helpful medical expert, and your task is to answer a multi-choice medical question using the relevant documents. Please choose the answer from the provided options. Your responses will be used for research purposes only, so please have a definite answer.\nHere are the relevant documents:\n A systematic review and cost comparison were conducted to determine the optimal treatment of active herpes zoster ophthalmicus (HZO) in immunocompetent adults. A literature search of MEDLINE, EMBASE, CINAHL, Cochrane Library, BIOSIS Previews and Web of Science, ClinicalTrials.gov, International Clinical Trials Registry Platform, Networked Digital Library of Theses and Dissertations, and Canadian Health Research Collection was performed. The search period was from January 1990 to March 2017. Collectively, 516 immunocompetent patients with active HZO treated with oral antivirals were included. Randomized controlled trials (RCTs) investigating treatment of active HZO in immunocompetent adults, with one oral acyclovir monotherapy arm, were included. Studies fulfilling inclusion criteria were subjected to quality assessment and data extraction. Provincial drug formularies were consulted to extrapolate cost comparison for investigated treatment regimens. A total of 1515 titles and abstracts and 9 full-text articles were assessed. Three RCTs met the inclusion criteria. Treatment with oral acyclovir (800 mg 5 times daily for 10 days) was superior to placebo in the prevention of ocular manifestations. Oral famciclovir (500 mg 3 times daily for 7 days) and valacyclovir (1000 mg 3 times daily for 7 days) resulted in comparable rates of ocular manifestations relative to oral acyclovir (800 mg 5 times daily for 7 days). According to provincial drug formulary data, famciclovir and valacyclovir are more affordable across Canada with the recommended dosing schedules. Oral famciclovir and valacyclovir are reasonable alternatives to oral acyclovir for treatment of active HZO in immunocompetent individuals. Their simpler dosing schedules are associated with a cost benefit that is consistent across Canada. Zoster ophthalmicus Treatment for zoster ophthalmicus is similar to standard treatment for shingles at other sites. A trial comparing acyclovir with its prodrug, valacyclovir, demonstrated similar efficacies in treating this form of the disease. The significant advantage of valacyclovir over acyclovir is its dosing of only three times/day (compared with acyclovir's five times/day dosing), which could make it more convenient for people and improve adherence with therapy. Treatment Treatment is usually with antivirals such as acyclovir, valacyclovir, or famcyclovir by mouth. There is uncertainty as to the difference in effect between these three antivirals. Antiviral eye drops have not been found to be useful. These medications work best if started within 3 days of the start of the rash. Cycloplegics prevent synechiae from forming. References External links Varicella zoster virus-associated diseases Ophthalmology Wikipedia medicine articles ready to translate Secondary prevention A 2013 Cochrane meta-analysis of 6 randomized controlled trials (RCTs) investigating oral antiviral medications given within 72 hours after the onset of herpes zoster rash in immunocompetent people for preventing postherpetic neuralgia (PHN) found no significant difference between placebo and acyclovir. Additionally, there was no significant difference in preventing the incidence of PHN found in the one RCT included in the meta-analysis that compared placebo to oral famciclovir treatment within 72 hours of HZ rash onset. Studies using valacyclovir treatment were not included in the meta-analysis. PHN was defined as pain at the site of the dermatomic rash at 120 days after the onset of rash, and incidence was evaluated at 1, 4, and 6 months after rash onset. Patients who are prescribed oral antiviral agents after the onset of rash should be informed that their chances of developing PHN are no different than those not taking oral antiviral agents. Evaluation of evidence-based strategies for managing herpes zoster (HZ) and the pain of postherpetic neuralgia (PHN). Approximately 20% of the world's population suffers from herpes zoster at least once in a lifetime, with 10% to 20% having ophthalmic involvement. Treatment of the acute disease with oral antivirals may reduce the incidence and severity of complications but does not reliably prevent PHN or postherpetic itch (PHI). The acute pain abates as the acute phase resolves; the long-term pain of PHN or PHI may be severe and difficult to manage. Although many therapeutic agents have efficacy in the management of these complications, relief is frequently partial for months to the remainder of the lifetime. Literature review was performed using the resources of the Harvard Medical School/Massachusetts Eye and Ear Infirmary Ophthalmic library as well as the National Library of Medicine and the National Institutes of Health PubMed service searching by pertinent topics, authors, and journals. If started within 72 hours of the onset of the acute HZ rash, the oral antiviral agents acyclovir, valacyclovir, and famciclovir significantly shorten the periods of acute pain, virus shedding, rash, acute and late-onset anterior segment complications, and, in the case of valacyclovir and famciclovir, the incidence and severity of PHN. However, these medications do not prevent PHN, which remains a common and debilitating complication of HZ in older patients, requiring assiduous pain management. Tricyclic antidepressants, antiseizure drugs, opioids, and topical analgesics all offer some pain relief, and may be combined. Options are available to manage HZ and reduce the pain of PHN. However, prevention, now possible with the HZ vaccine, is preferable to treatment. The mutation rate for synonymous and nonsynonymous mutation rates among the herpesviruses have been estimated at 1 × 10−7 and 2.7 × 10−8 mutations/site/year, respectively, based on the highly conserved gB gene. Treatment Within the human body it can be treated by a number of drugs and therapeutic agents including acyclovir for the chicken pox, famciclovir, valaciclovir for the shingles, zoster-immune globulin (ZIG), and vidarabine. Acyclovir is frequently used as the drug of choice in primary VZV infections, and beginning its administration early can significantly shorten the duration of any symptoms. However, reaching an effective serum concentration of acyclovir typically requires intravenous administration, making its use more difficult outside of a hospital. Vaccination Approximately 10 to 30% of the population will suffer from herpes zoster (HZ) during their lifetime. Prompt treatment of acute HZ with acyclovir, valacyclovir or famciclovir is recommend, if patients are over 50 years old or have severe or moderate pain or severe or moderate rash or they are immonocompromised or suffer from herpes zoster ophtalmicus. Zoster lesions contain high concentrations of Varicella zoster virus that can spread, and cause chicken pox. There is no universal recommendations for varicella vaccination. It has been shown that zoster vaccine markedly reduced morbidity from herpes zoster and postherpetic neuralgia among older adults. Antiviral agents: Ideally, treatment with systemic antiviral agents should begin within 72 hours of disease onset. Initiation should not be delayed while awaiting definitive diagnosis or ophthalmology follow-up. Topical antiviral agents may be considered, but there is limited evidence regarding their utility in managing HZO. Immunocompetent adult dosing (choose single agent): Acyclovir 800 mg orally five times per day for at least 7 days Valacyclovir 1000 mg orally every eight hours for at least 7 days (may require renal dosing) Famciclovir 500 mg orally three times per day for at least 7 days (may require renal dosing) Immunocompromised adult dosing (choose a single agent): Acyclovir 10 mg/kg of ideal body weight (IBW) intravenously (IV) every eight hours for at least 7 days Foscarnet 90 mg/kg IV every 12 hours (typically reserved for severe or acyclovir-resistant disease) The objective of this study was to assess the status and trends of antiviral treatment in outpatients with herpes zoster in China. Prescription data on antiviral drugs were extracted from the database of the Hospital Prescription Analysis Program of China according to the inclusion criteria. Yearly prescriptions and costs were calculated, and trends were analyzed. The trends were further stratified by age, sex, and specific drug use. The distribution of defined daily costs (DDCs) of valaciclovir and famciclovir were analyzed, and trends in the median DDCs were identified. A total of 132,911 prescriptions from 49 hospitals located in six major areas of China were included in the analysis. The yearly prescriptions containing antivirals increased from 8,819 in 2010 to 16,361 in 2019. The percentage of prescriptions for patients aged 65 years and above also increased (27.7% in 2010 to 31.0% in 2019), and the number of prescriptions for females was higher than those for males (3 BP drugs, drug-resistant) or Family history of early-onset hypertension or cerebrovascular events at < 40 years of age Measurement of aldosterone-renin ratio (ARR) on current blood pressure medication (stop spironolactone for 4 wks) and with hypokalemia corrected (ARR screen positive if ARR >750 pmol/L: ng/ml/h and aldosterone >450 pmol/L) (consider repeat off ˜-blockers for 2 wks if results are equivocal) Negative E.g., saline infusion test (2 liters physiologic saline over 4 h IV), oral sodium loading, fludrocortisone suppression Confirmation of diagnosis Rare: Both renin and Aldo suppressed The diagnosis of primary hyperaldosteronism (PHPA) has progressively increased over the last years and some authors consider it as the main cause of secondary hypertension. We studied the prevalence of PHPA in hypertensive patients followed at the Hypertension Unit from July 1999 to July 2017. A total of 2500 patients were included and diagnosis of PHPA was done in 79 of them (3.2%). It was more frequent in women (55.7%) with an increased incidence in the elderly, as compared to previous studies (27.8%). Initial diagnosis was suspected upon the presence of inappropriate kaliuria and metabolic alkalosis, associated to an aldosterone/plasma renin activity ratio > 30 (ng/dl)/(ng/ml/h). After confirmation of the presence of PA, imaging techniques to determine the etiology were performed. In this way, 29 cases (36.8%) of aldosterone-producing adenoma and 5 cases of bilateral adrenal hyperplasia with nodules were identified. Computed tomography identified the adenomas and hyperplasias with bilateral cortical nodules in all patients. Adrenalectomy and/or antialdosteronics were efficient in controlling blood pressure in 69.9% of cases. Of note in this series was the remission of stage 3 chronic renal failure in two cases, the high prevalence of hypercalciuric urinary lithiasis and a case of breast carcinoma after prolonged treatment with spironolactone. Primary hyperaldosteronism has a number of causes. About 33% of cases are due to an adrenal adenoma that produces aldosterone, and 66% of cases are due to an enlargement of both adrenal glands. Other uncommon causes include adrenal cancer and an inherited disorder called familial hyperaldosteronism. Some recommend screening people with high blood pressure who are at increased risk, while others recommend screening all people with high blood pressure for the disease. Screening is usually done by measuring the aldosterone-to-renin ratio in the blood (ARR) whilst off interfering medications and a serum potassium over 4, with further testing used to confirm positive results. While low blood potassium is classically described in primary hyperaldosteronism, this is only present in about a quarter of people. To determine the underlying cause, medical imaging is carried out.\nHere is the question:\nNEPHROLOGY: A 58-year-old man with a 6-year history of hypertension consults for poor blood pressure control despite treatment with an angiotensin-converting enzyme inhibitor, a diuretic and a calcium antagonist. On consultation she presented with blood pressure of 149/100 mmHg. Laboratory tests: creatinine 1.2 mg/dl, potassium 2.2 mEq/l and compensated metabolic alkalosis; the rest of the biochemical study, blood count, coagulation and urinary sediment were normal. Point out the correct statement:\nHere are the potential choices:\n1. The origin of hypertension in this case is excessive secretion of aldosterone caused by autonomic hyperfunction of the adrenal medulla.\n2. In most cases the anatomical substrate is a bilateral hyperplasia of the adrenal cortex.\n3. CT scan is part of the diagnostic study in case of biochemical confirmation.\n4. Spironolactone is contraindicated in the management of this pathology.\nThe correct answer is: ", "gold_answer": "3 CT scan is part of the diagnostic study in case of biochemical confirmation.", "prediction": "You are a helpful medical expert, and your task is to answer a multi-choice medical question using the relevant documents. Please choose the answer from the provided options. Your responses will be used for research purposes only, so please have a definite answer.\nHere are the relevant documents:\n Primary hyperaldosteronism is only rarely caused by unilateral adrenal hyperplasia. A 73-year-old hypertensive Greek man (on 10 mg amlodipine for the last ten years) presented in the emergency department with severe muscle weakness of all limbs. The initial physical and laboratory examination revealed normal blood pressure, muscle weakness, severe hypokalemia, sinus rhythm and U wave, rhabdomyolysis and metabolic alkalosis. The patient was immediately treated with intravenous administration of potassium-rich solutions, 25 mg spironolactone with progressive dose titration up to 100 mg. Because of high arterial blood pressure, irbesartan was added. On day 6, muscle weakness was completely restored with decrease of arterial blood pressure and further improvement of laboratory tests. The combination of hypokalemia with arterial hypertension raised the suspicion of primary hyperaldosteronism; therefore, we performed abdomen computed tomography scan, which revealed a nodular mass (15 mm in diameter) in the left adrenal gland. Plasma renin activity was in the lower normal range with a three-fold increase of plasma aldosterone concentration. We performed total resection of the left adrenal gland and the histopathological examination revealed hyperplasia of the left adrenal gland. This report presents a rare case of an elderly patient under antihypertensive treatment the last ten years for essential hypertension, who admitted to our emergency department with hypokalemia - induced myopathy as first manifestation of primary hyperaldosteronism due to unilateral adrenal hyperplasia. Primary aldosteronism occurs in 1-10% of hypertensive patients and is classified in adenomas or bilateral adrenal hyperplasia. Computed tomography (CT) or magnetic resonance imaging can be used to discriminate these subtypes and in guiding treatment selection. This case report describes a 65-year-old man with hypertension and hypokalaemia during 25 years. Bilateral adrenal hyperplasia was diagnosed based on a CT, and an oral sodium-loading test with measurement of renin and aldosterone confirmed the diagnosis. Blood pressure and potassium in plasma was normalized during treatment with the mineralocorticoid receptor antagonist eplerenon. Primary aldosteronism (PA) is the most common form of secondary hypertension (HTN), with an estimated prevalence of 4% of hypertensive patients in primary care and around 10% of referred patients. Patients with PA have higher cardiovascular morbidity and mortality than age- and sex-matched patients with essential HTN and the same degree of blood pressure elevation. PA is characterized by an autonomous aldosterone production causing sodium retention, plasma renin supression, HTN, cardiovascular damage, and increased potassium excretion, leading to variable degrees of hypokalemia. Aldosterone-producing adenomas (APAs) account for around 40% and idiopathic hyperaldosteronism for around 60% of PA cases. The aldosterone-to-renin ratio is the most sensitive screening test for PA. There are several confirmatory tests and the current literature does not identify a \"gold standard\" confirmatory test for PA. In our institution, we recommend starting case confirmation with the furosemide test. After case confirmation, all patients with PA should undergo adrenal CT as the initial study in subtype testing to exclude adrenocortical carcinoma. Bilateral adrenal vein sampling (AVS) is the gold standard method to define the PA subtype, but it is not indicated in all cases. An experienced radiologist must perform AVS. Unilateral laparoscopic adrenalectomy is the preferential treatment for patients with APAs, and bilateral hyperplasia should be treated with mineralocorticoid antagonist (spironolactone or eplerenone). Cardiovascular morbidity caused by aldosterone excess can be decreased by either unilateral adrenalectomy or mineralocorticoid antagonist. In this review, we address the most relevant issues regarding PA screening, case confirmation, subtype classification, and treatment. Based on two patients, we discuss the difficulties in diagnosing and managing primary aldosteronism in pregnancy, which derive from changes of the renin-angiotensin-aldosterone axis, from the uncertainty regarding blood pressure control along gestation and postpartum, and from the contraindication to the use of spironolactone. The first case is a 27 years old woman with a long standing refractory hypertension, a hemorrhagic stroke with left brachial hemiplegia and crural hemiparesia, two miscarriages, one stillbirth and one offspring with intrauterine growth retardation. Due to hypokalemia, a plasma aldosterone/renin activity ratio of 91, and a negative genetic screening for glucocorticoid remediable aldosteronism (GRA), a primary hyperaldosteronism with normal adrenals in CT scan was diagnosed, and good blood pressure control was attained with spironolactone. After two and a half years of normotension, a fifth pregnancy, managed with methyldopa evolved with satisfactory blood pressures, plasma potassium, fetal growth, uterine and umbilical arterial resistance indexes, and maternal endothelial function. At 37 1/2 weeks of pregnancy the patient delivered a healthy newborn weighing 2,960 g. Blood pressure rose during the 48 hours of postpartum in the absence of proteinuria and required i.v. hydralazine. The second patient is a 37 years old woman, with known refractory hypertension for 7 years, hypokalemia, plasma aldosterone/renin activity ratio greater than 40, normal adrenals in the CAT scan, and a negative genetic screening for GRA. She had normotensive pregnancies 5 and 3 years prior to the detection of hypertension, with hypertensive crisis in both postpartum periods, retrospectively considered as expressions of primary hyperaldosteronism. The prevalence of primary hyperaldosteronism is 5-10% of all hypertensive patients, and clearly above the estimated prevalence in the past. In nearly 30% of patients with therapy resistant hypertension, primary hyperaldosteronism is detected if they are investigated thoroughly. This will result in 1.5 to 2.5 million people in Germany suffering from primary hyperaldosteronism. Besides efficient diagnostic procedures, an effective treatment is of increasing importance. The aldosterone-producing adenoma (Conn's syndrome) is primarily cured by operation, in most cases performed endoscopically. Bilateral hyperplasia, which is found in two-thirds of primary hyperaldosteronism, is treated primarily by mineralocorticoid receptor antagonist: 12.5-50 mg/day spironolactone (in case of anti-androgenic side-effects alternatively by 50-100 mg/day eplerenone). If the blood pressure can not be lowered by this first-line treatment, an additional treatment with potassium-sparing diuretics, calcium-antagonists, ACE-inhibitors or angiotensin-2-antagonists is necessary. The start of medication should be closely monitored by serum electrolyte and creatinine controls. We report a case of a renin secreting tumor, which is a very rare cause of secondary high blood pressure. A 22-year-old woman was hospitalised for exploration of high blood pressure (160/110 mmHg) with severe hypokaliemia (2,7 mmol/l) and secondary hyperaldosteronism. Physical examination was normal except the high blood pressure. Bioassays show increased kaliuresis (66 mmol/24h), plasma renin (89 pg/ml in clinostastism--108 pg/ml in orthostatism), pro-renin (1207 pg/ml in clinostastism--1412 pg/ml in orthostatism) and aldosterone (210 pg/ml in clinostastism--566 pg/ml in orthostatism). The rest of the endocrine tests were normal (cortisol and ACTH at 8:00 am, urinary free cortisol, overnight 1 mg dexamethasone suppression test). Doppler ultrasound method, performed by an experienced radiologist, did not show renal artery stenosis. Abdominal computerized tomography showed a nodular formation at the upper pole of the right kidney, isodense to renal medullary. The size tumor was 15 mm. The renal vein sampling shows high values of renin on both sides whereas, for the pro-renin, the values were higher on the tumor side. In spite of treatment with CEI (Converting Enzyme Inhibitors) and calcium antagonists, the blood pressure was not controlled. Hypokaliemia persisted (3 mmol/l) in spite of high daily potassium intake (64 mmol/l of potassium chloride). After tumor resection, reninoma was diagnosed by the pathology examination and blood pressure, plasma rennin, plasma aldosterone level returned to normal. Diagnosis Other conditions such as Liddle's Syndrome can mimic the clinical features of AME, so diagnosis can be made by calculating the ratio of free urinary cortisol to free urinary cortisone. Since AME patients create less cortisone, the ratio will much be higher than non-affected patients. Alternatively, one could differentiate between the two syndromes by administering a potassium-sparing diuretic. Patients with Liddle's syndrome will only respond to a diuretic that binds the ENaC channel, whereas those with AME will respond to a diuretic that binds to ENaC or the mineralcorticoid receptor. Treatment The treatment for AME is based on the blood pressure control with Aldosterone antagonist like Spironolactone which also reverses the hypokalemic metabolic alkalosis and other anti-hypertensives. Renal transplant is found curative in almost all clinical cases.AME is exceedingly rare, with fewer than 100 cases recorded worldwide. Primary hyperaldosteronism (PHA) represents less than 1 to 2% of all causes of hypertension (HT). We report 2 cases of primary hyperaldosteronism which emphasize the difficulty of distinguishing neoplastic PHA from idiopathic PHA, observed in a 60-year-old woman and a 42-year old woman, respectively. In both cases, the diagnosis of PHA was suggested by marked hypokalaemia with inappropriate potassium excretion and was confirmed by hyperaldosteronaemia and low and poorly stimulated renin activity. In the first case, computed tomography showed nodular hyperplasia of the 2 adrenal glands. The patient was treated with spironolactone and calcium channel blockers which controlled blood pressure and serum potassium. In the second case, computed tomography and magnetic resonance imaging revealed an adrenocortical adenoma confirmed by pathological examination after the operation. The diagnosis of primary hyperaldosteronism is based on three steps: detection, positive diagnosis and aetiological diagnosis. Detection is essentially based on demonstration of hypokalaemia. Positive diagnosis is based on demonstration of elevated aldosterone secretion with inhibited renin secretion. The aetiological diagnosis is dominated by the differentiation between Conn's adenoma and bilateral adrenal hyperplasia, which has therapeutic implications. To investigate the clinical characteristics, differential diagnosis, and surgery outcome of unilateral nodular adrenal hyperplasia (UNAH). The clinical data of 145 patients with primary aldosteronism, 67 males and 78 females, aged 37.9 (19-60), including 78 cases of aldosterone-producing adenoma (APA), 14 cases of UNAH, and 55 cases of idiopathic bilateral adrenal hyperplasia (BAH), were collected. Radioimmunoassay was used to examine the blood and urine aldosterone and plasma rennin activity. Automatic biochemical apparatus was used to examine the blood and urine electrolytes, renal functions, and urine microalbumin. Twelve-lead electrocardiography, echocardiography, and plain scanning of enhanced CT scanning of the bilateral adrenals were conducted. Adrenal venous sampling (AVS) was conducted in 62 patients to collect blood samples from vena cava and bilateral suprarenal veins to detect the levels of aldosterone and cortisol. All UNAH patients and 3 BAH patients underwent unilateral adrenalectomy and three APA patients underwent unilateral adrenalectomy or adenoma resection. Then the patients were followed up for 39.2 months. The incidence of UNAH is 9.7% in the primary aldosteronism patients. There were no significant differences in age, gender, duration of hypertension, blood pressure (SBP, DBP), and indexes indicating damages in target organs of hypertension (left ventricular hypertrophy rate, blood creatinine, urine microalbumin, etc) among these three groups. The level of serum potassium of the APA group was significantly lower than that of the BAH group (P < 0.01), and the levels of plasma and urine aldosterone of the APA group were significantly higher than those of the BAH group (P < 0.05 and P < 0.01). The serum potassium of the UNAH group was higher than that of the APA group and lower than that of the BAH group, and the levels of plasma and urine aldosterone of the UNAH group were both higher than those of the APA group and lower than those of the BAH group, however all not significantly (all P > 0.05). The coincidence rate of CT was 50% (7/14) in the UNAH group. The accuracy of AVS for diagnosis of UNAH was 85.7% (12/14). After operation, the serum potassium and plasma aldosterone concentrations returned normal in all the UNAH patients. Blood pressure returned to normal in 50% (7/14) of the UNAH patients, and was improved in the other 50% (7/14) patients. UNAH can be cured by adrenal surgery. The diagnostic values of clinical examination and adrenal CT are limited. AVS is essential in diagnosing UNAH patients. Pseudohyperaldosteronism (also pseudoaldosteronism) is a medical condition which mimics the effects of elevated aldosterone (hyperaldosteronism) by presenting with high blood pressure (hypertension), low blood potassium levels (hypokalemia), metabolic alkalosis, and low levels of plasma renin activity (PRA). However, unlike hyperaldosteronism, this conditions exhibits low or normal levels of aldosterone in the blood. Causes include genetic disorders (e.g. Apparent mineralocorticoid excess syndrome, Liddle's syndrome, and types of Congenital adrenal hyperplasia), acquired conditions (e.g. Cushing's syndrome and mineralocorticoid-producing adrenal tumors), metabolic disorders, and dietary imbalances including excessive consumption of licorice. Confirmatory diagnosis depends on the specific root cause and may involve blood tests, urine tests, or genetic testing; however, all forms of this condition exhibit abnormally low concentrations of both plasma renin activity (PRA) and plasma Primary hyperaldosteronism (PHA) is characterized by an increased Aldosterone synthesis which is independent of the Renin-Angiotensin-Aldosterone-System (RAAS). The prevalence of PHA in patients who present in specialized hypertension centers is approx. 10 %. Besides patients with the classical symptoms known as \"Conn-Trias\" (hypertension, hypokalemia, metabolic alkalosis), the more frequent normokalemic patients with PHA also show a worse outcome compared to patients with essential hypertension. Identifying these patients is an important task in the evaluation of hypertension since targeted treatment options are available. Screening for PHA using the Aldosterone-Renin-Ratio (ARR) should be performed in patients with hypokalemic, severe or resistant hypertension. In addition, young patients with early onset of severe hypertension and/or positive family history should be screened. A positive screening result should be followed by a confirmatory test. The saline infusion test is the preferred clinical test for confirming a suspected PHA since it is accessible and time efficient. Other confirmatory tests are not used on a regular basis. After any confirmatory test, CT- or MRI-imaging and adrenal vein sampling (AVS) is used in order to differentiate between a unilateral adenoma, a bilateral hyperplasia or another cause of PHA. CT or MRI usually cannot discriminate smaller tumors form hyperplasia. Therefore AVS is used to detect lateralization of autonomous aldosterone production. Lateralization of aldosterone production indicates a unilateral adenoma. In these cases, laparoscopic adrenalectomy is the therapeutic option of choice with a hypertension cure rate of up to 60 %. If no lateralization is detectable, bilateral hyperplasia as the underlying cause of PHA is likely. Pharmacological inhibition of the mineralocorticoid receptor is the preferred treatment option in these cases. If Spironolactone is not well tolerated, Eplerenone and potassium-sparing diuretics should be prescribed. Often, however, in order to fully control hypertension, additional antihypertensive therapy is necessary. Great strides have been made in our understanding of the pathophysiology of primary aldosteronism syndrome since Conn's description of the clinical presentation of a patient with an aldosterone-producing adenoma (APA) more than 50 years ago. It is now recognized that the APA is just one of the seven subtypes of primary aldosteronism. APA and bilateral idiopathic hyperaldosteronism (IHA) are the most common subtypes of primary aldosteronism. Although most clinicians had thought primary aldosteronism to be a rare form of hypertension for more than three decades, it is now recognized to be the most common form of secondary hypertension. Using the plasma aldosterone to plasma renin activity ratio as a case-finding test, followed by aldosterone suppression confirmatory testing, has resulted in much higher prevalence estimates of 5-13% of all patients with hypertension. In addition, there has been a new recognition of the aldosterone-specific cardiovascular morbidity and mortality associated with aldosterone excess. Although thought to be daunting and complex in the past, the diagnostic approach to primary aldosteronism is straightforward and can be considered in three phases: case-finding tests, confirmatory tests and subtype evaluation tests. Patients with hypertension and hypokalaemia (regardless of presumed cause), treatment-resistant hypertension (three antihypertensive drugs and poor control), severe hypertension (>or= 160 mmHg systolic or >or= 100 mmHg diastolic), hypertension and an incidental adrenal mass, onset of hypertension at a young age or patients being evaluated for other forms of secondary hypertension should undergo screening for primary aldosteronism. In patients with suspected primary aldosteronism, screening can be accomplished by measuring a morning (preferably between 0800 and 1000 h) ambulatory paired random plasma aldosterone concentration (PAC) and plasma renin activity (PRA). An increased PAC:PRA ratio is not diagnostic by itself, and primary aldosteronism must be confirmed by demonstrating inappropriate aldosterone secretion. Aldosterone suppression testing can be performed with orally administered sodium chloride and measurement of urinary aldosterone or with intravenous sodium chloride loading and measurement of PAC. Unilateral adrenalectomy in patients with APA or unilateral adrenal hyperplasia results in normalization of hypokalaemia in all these patients; hypertension is improved in all and is cured in approximately 30-60% of them. In bilateral adrenal forms of primary aldosteronism, unilateral or bilateral adrenalectomy seldom corrects the hypertension and they should be treated medically with a mineralocorticoid receptor antagonist. Primary Primary aldosteronism (hyporeninemic hyperaldosteronism) was previously thought to be most commonly caused by an adrenal adenoma, termed Conn's syndrome. However, recent studies have shown that bilateral idiopathic adrenal hyperplasia is the cause in up to 70% of cases. Differentiating between the two is important, as this determines treatment. Also, see congenital adrenal hyperplasia. Adrenal carcinoma is an extremely rare cause of primary hyperaldosteronism. Two familial forms have been identified: type I (dexamethasone suppressible), and type II, which has been linked to the 7p22 gene. Features Hypertension Hypokalemia (e.g., may cause muscle weakness) Alkalosis Investigations High serum aldosterone Low serum renin High-resolution CT abdomen Management Adrenal adenoma: surgery Bilateral adrenocortical hyperplasia: aldosterone antagonist, e.g., spironolactone To show clinical, biochemical, and morphological data of 12 patients with primary hyperaldosteronism: eight with an aldosterone-producing adenoma and four with adrenal hyperplasia. To compare clinical and biochemical parameters of the patients with adenoma and hyperplasia. For those with adenoma, to verify clinical and biochemical modifications after adrenalectomy. In the 12 patients with hyperaldosteronism, retrospective analysis of clinical (age, sex, blood pressure), biochemical (plasmatic and urinary potassium, plasmatic aldosterone, plasma renin activity, and plasmatic aldosterone/renin activity ratio), and morphological (computed tomography, magnetic resonance, and norcholesterol scintigraphy) data was performed. 1--In the 12 patients with hyperaldosteronism (seven female), the age was 51.0 +/- 10.2 years (mean +/- standard deviation), the systolic pressure 200.9 +/- 34.5 mm Hg and the diastolic pressure 120.0 +/- 12.3 mm Hg. Hypertension was diagnosed 12.0 +/- 10.1 years before. As biochemical evidence, we found kalaemia of 3.06 +/- 0.28 and urinary potassium of 63.4 +/- 16.5 mEq/l, renin activity 0.98 +/- 1.02 ng/ml/h, plasmatic aldosterone of 49.4 +/- 36.0 ng/dl, aldosterone/renin activity > 30 in 83% of the cases. As morphological evidence, computed tomography allowed diagnosis in nine patients, suggested it in two, being doubtful in one. Performed on four patients, resonance confirmed the tomography in three and was not contributive in one. The scintigraphy performed in four patients visualized two adenomas, was negative in one adenoma and in one hyperplasia. 2--In the eight patients with adenoma (six female), the youngest age and the highest diastolic pressure compared with patients with hyperplasia were statistically significant (p < 0.01 and 0.05). In the adenomas, the biochemical changes were more pronounced, but not statistically significant. The plasmatic aldosterone/renin activity ratio was also higher in the adenoma cases. 3--After the adrenalectomy, blood pressure became normal in five patients and was more easily therapeutically controlled in three. The average systolic and diastolic pressures decreased and the biochemical parameters became normal in all patients. The pre/post surgical modification of these parameters had statistical significance (systolic pressure decrease, p < 0.01; diastolic pressure decrease, p < 0.01; kalaemia increase, p < 0.001; renin activity increase, p < 0.01; aldosterone decrease, p < 0.02). The plasmatic aldosterone/renine activity ratio normalized in all patients. In diagnosing primary hyperaldosteronism, biochemical (kalaemia, urinary potassium, plasmatic aldosterone, renin activity, aldosterone plasmatic/renin activity) and tomography studies were important. On comparing the patients with hyperplasia with those with adenoma, we found that the latter are younger and exhibit higher diastolic pressure, both findings with statistical significance. After adenoma surgery, blood pressure became normal in five patients and improved in three, these findings, and the improvement of the kalaemia, plasmatic aldosterone, and renin activity parameters were statistically significant. In patients with normal adrenal morphology and family history of early-onset, severe hypertension, a diagnosis of GRA should be Disorders of the Adrenal Cortex Clinical suspicion of mineralocorticoid excess Patients with hypertension and Severe hypertension (>3 BP drugs, drug-resistant) or Family history of early-onset hypertension or cerebrovascular events at < 40 years of age Measurement of aldosterone-renin ratio (ARR) on current blood pressure medication (stop spironolactone for 4 wks) and with hypokalemia corrected (ARR screen positive if ARR >750 pmol/L: ng/ml/h and aldosterone >450 pmol/L) (consider repeat off ˜-blockers for 2 wks if results are equivocal) Negative E.g., saline infusion test (2 liters physiologic saline over 4 h IV), oral sodium loading, fludrocortisone suppression Confirmation of diagnosis Rare: Both renin and Aldo suppressed The diagnosis of primary hyperaldosteronism (PHPA) has progressively increased over the last years and some authors consider it as the main cause of secondary hypertension. We studied the prevalence of PHPA in hypertensive patients followed at the Hypertension Unit from July 1999 to July 2017. A total of 2500 patients were included and diagnosis of PHPA was done in 79 of them (3.2%). It was more frequent in women (55.7%) with an increased incidence in the elderly, as compared to previous studies (27.8%). Initial diagnosis was suspected upon the presence of inappropriate kaliuria and metabolic alkalosis, associated to an aldosterone/plasma renin activity ratio > 30 (ng/dl)/(ng/ml/h). After confirmation of the presence of PA, imaging techniques to determine the etiology were performed. In this way, 29 cases (36.8%) of aldosterone-producing adenoma and 5 cases of bilateral adrenal hyperplasia with nodules were identified. Computed tomography identified the adenomas and hyperplasias with bilateral cortical nodules in all patients. Adrenalectomy and/or antialdosteronics were efficient in controlling blood pressure in 69.9% of cases. Of note in this series was the remission of stage 3 chronic renal failure in two cases, the high prevalence of hypercalciuric urinary lithiasis and a case of breast carcinoma after prolonged treatment with spironolactone. Primary hyperaldosteronism has a number of causes. About 33% of cases are due to an adrenal adenoma that produces aldosterone, and 66% of cases are due to an enlargement of both adrenal glands. Other uncommon causes include adrenal cancer and an inherited disorder called familial hyperaldosteronism. Some recommend screening people with high blood pressure who are at increased risk, while others recommend screening all people with high blood pressure for the disease. Screening is usually done by measuring the aldosterone-to-renin ratio in the blood (ARR) whilst off interfering medications and a serum potassium over 4, with further testing used to confirm positive results. While low blood potassium is classically described in primary hyperaldosteronism, this is only present in about a quarter of people. To determine the underlying cause, medical imaging is carried out.\nHere is the question:\nNEPHROLOGY: A 58-year-old man with a 6-year history of hypertension consults for poor blood pressure control despite treatment with an angiotensin-converting enzyme inhibitor, a diuretic and a calcium antagonist. On consultation she presented with blood pressure of 149/100 mmHg. Laboratory tests: creatinine 1.2 mg/dl, potassium 2.2 mEq/l and compensated metabolic alkalosis; the rest of the biochemical study, blood count, coagulation and urinary sediment were normal. Point out the correct statement:\nHere are the potential choices:\n1. The origin of hypertension in this case is excessive secretion of aldosterone caused by autonomic hyperfunction of the adrenal medulla.\n2. In most cases the anatomical substrate is a bilateral hyperplasia of the adrenal cortex.\n3. CT scan is part of the diagnostic study in case of biochemical confirmation.\n4. Spironolactone is contraindicated in the management of this pathology.\nThe correct answer is: 3. CT scan is part of the diagnostic study in case of biochemical confirmation." }, { "question": "You are a helpful medical expert, and your task is to answer a multi-choice medical question using the relevant documents. Please choose the answer from the provided options. Your responses will be used for research purposes only, so please have a definite answer.\nHere are the relevant documents:\n Erythematous scaly papules on the palms and soles are a common manifestation of secondary syphilis. We report a case of 19-year-old male who presented with erythematous, scaly, psoriasiform lesions over the palms and glans penis. The papules over the palms showed tenderness on blunt vertical pressure. There was a history of sexual contact and ulcer over the glans around 2 months back, which resolved on its own. Venereal Disease Research Laboratory test was positive in 1:32 dilution. 75 y - 4 points Hematocrit level <39% for men and <35% for women - 3 points Diabetes mellitus- 3 points Contrast media volume - 1 point for each 100 mL Decreased kidney function: Serum creatinine level >1.5 g/dL - 4 points or Estimated Glomerular filtration rate (online calculator) 2 for 40–60 mL/min/1.73 m2 4 for 20–40 mL/min/1.73 m2 6 for < 20 mL/min/1.73 m2 Although chronic kidney disease is a risk factor for cardiovascular disease it is unclear whether diabetic patients with a reduced glomerular filtration rate (GFR), independent of (micro)albuminuria, carry an increased risk of stroke. We therefore investigated the independent effect of estimated GFR (eGFR) on stroke events in patients with type 2 diabetes mellitus (T2DM). We studied T2DM patients with an eGFR >or=15 ml min(-1) per 1.73 m(2), who had no history of stroke. Patients were divided into four categories by the eGFR at baseline for comparison: >or=90, 60-89, 30-59 and 15-29 ml min(-1) per 1.73 m(2). The end point was an incident stroke event. The Cox proportional hazard model was used to calculate the hazard ratio (HR) and 95% confidence interval (CI). The study included a total of 1300 T2DM patients (546 women and 754 men) with a mean (+/-s.d.) age of 63+/-13 years. During a mean follow-up period of 3.7+/-1.4 years, 91 patients experienced an incident stroke event. Although a lower eGFR was associated with an increased stroke risk using a univariate model, statistical significance disappeared after adjusting for other risk factors including albuminuria. The HR (95% CI) was 0.75 (0.40-1.41, P=0.373), 0.99 (0.50-1.95, P=0.964) and 0.91 (0.36-2.28, P=0.844) for patients with eGFRs of 60-89, 30-59 and 15-29 ml min(-1) per 1.73 m(2), respectively, compared with patients with an eGFR >or=90. Clinical albuminuria remained a significant risk factor for stroke, and the adjusted HR compared with normoalbuminuria was 2.40 (1.46-3.95, P=0.001). In conclusion, the association between reduced GFR and stroke events in patients with T2DM is likely to be mediated by albuminuria. Serum creatinine is also utilized in GFR estimating equations such as the Modified Diet in Renal Disease (MDRD) and the CKD-EPI (Chronic Kidney Disease Epidemiology Collaboration) equation. These eGFR equations are superior to serum creatinine alone since they include race, age, and gender variables. GFR is classified into the following stages based on kidney disease. Kidney Disease Improving Global Outcomes (KDIGO) stages of chronic kidney disease (CKD): Stage 1 GFR greater than 90 ml/min/1.73 m² Stage 2 GFR-between 60 to 89 ml/min/1.73 m² Stage 3a  GFR 45 to 59 ml/min/1.73 m² Stage 3b GFR 30 to 44 ml/min/1.73 m² Stage 4 GFR of 15 to 29 ml/min/1.73 m² Stage 5-GFR less than 15 ml/min/1.73 m² (end-stage renal disease) Although much emphasis has been placed on screening for albuminuria in type II diabetic patients, less attention has been focused on the role of glomerular filtration rate (GFR) in the assessment of risk. Herein, we examined the association between GFR and vascular complications in a consecutive cohort of 5174 type II diabetic patients between 1995 and 2000. Renal function was assessed by GFR (estimated by Modification of Diet in Renal Disease equation). The frequency of chronic kidney disease (CKD) as defined by GFR <60 ml/min/1.73 m(2), micro- and macrovascular complications, and their associations were analyzed. In this study cohort, 6% had serum creatinine > or =150 micromol/l and 15.8% had CKD. After adjustment for potential confounders, including urinary albumin excretion, odds ratios [95% confidence interval (CI)] across different stages of estimated GFR (> or =90, 60-89, 30-59, 15-29, <15 ml/min/1.73 m(2)) for macrovascular disease were 1.00, 1.42 [1.12-1.80], 1.80 [1.32-2.45], 2.74 [1.64-4.56], and 4.05 [1.77-9.26], respectively (P for trend <0.001); for retinopathy were 1.00, 1.23 [1.04-1.46], 1.80 [1.40-2.30], 2.05 [1.25-3.37], and 4.12 [1.56-10.90], respectively (P for trend <0.001); for sensory neuropathy were 1.00, 1.53[1.27-1.85], 2.09 [1.58-2.76], 4.32 [2.41-7.77], and 3.16 [1.25-8.02], respectively (P for trend <0.001); and for microalbumuria (with GFR <15 ml/min/1.73 m(2) excluded from the analysis) were 1.00, 1.51 [1.30-1.75], 5.80 [4.52-7.44], and 52.5 [16.4-168.2] respectively (P for trend <0.001). Measurement of serum creatinine alone without GFR may underestimate renal impairment in type II diabetic patients. Decreasing GFR was significantly associated with increasing frequency of micro- and macrovascular complications. There are several different techniques used to calculate or estimate the glomerular filtration rate (GFR or eGFR). The above formula only applies for GFR calculation when it is equal to the Clearance Rate. The normal range of GFR, adjusted for body surface area, is 100–130 average 125 (mL/min)/(1.73 m2) in men and 90–120 (mL/min)/(1.73 m2) in women younger than the age of 40. In children, GFR measured by inulin clearance is 110 (mL/min)/(1.73 m2) until 2 years of age in both sexes, and then it progressively decreases. After age 40, GFR decreases progressively with age, by 0.4–1.2 mL/min per year. Estimated GFR (eGFR) is now recommended by clinical practice guidelines and regulatory agencies for routine evaluation of GFR whereas measured GFR (mGFR) is recommended as a confirmatory test when more accurate assessment is required. Protein in the urine is regarded as an independent marker for worsening of kidney function and cardiovascular disease. Hence, British guidelines append the letter \"P\" to the stage of chronic kidney disease if protein loss is significant. Stage 1: Slightly diminished function; kidney damage with normal or relatively high GFR (≥90 ml/min/1.73 m2) and persistent albuminuria. Kidney damage is defined as pathological abnormalities or markers of damage, including abnormalities in blood or urine tests or imaging studies. Stage 2: Mild reduction in GFR (60–89 ml/min/1.73 m2) with kidney damage. Kidney damage is defined as pathological abnormalities or markers of damage, including abnormalities in blood or urine tests or imaging studies. Stage 3: Moderate reduction in GFR (30–59 ml/min/1.73 m2):. British guidelines distinguish between stage 3A (GFR 45–59) and stage 3B (GFR 30–44) for purposes of screening and referral. Evidence shows that apart from positive effects on glycemic levels, canagliflozin also reduces the risk of heart attacks and heart failures. SGLT2 inhibitors, including canagliflozin, reduce the likelihood of hospitalization for congestive heart failure or progression of renal disease in persons with diabetes mellitus type 2 and reduce the likelihood of stroke and heart attack in persons with diabetes mellitus type 2 who have known atherosclerotic vascular disease. Contraindications Canaglifozin is contraindicated in: Type 1 diabetes Diabetic ketoacidosis Severe renal impairment (estimated glomerular filtration rate <30 mL/min/1.73 m2), end-stage renal disease Patients on dialysis The normal range of GFR, adjusted for body surface area, is 100–130 average 125 mL/min/1.73m2 in men and 90–120 mL/min/1.73m2 in women younger than the age of 40. In children, GFR measured by inulin clearance is 110 mL/min/1.73 m2 until 2 years of age in both sexes, and then it progressively decreases. After age 40, GFR decreases progressively with age, by 0.4–1.2 mL/min per year. Estimated GFR (eGFR) is now recommended by clinical practice guidelines and regulatory agencies for routine evaluation of GFR whereas measured GFR (mGFR) is recommended as a confirmatory test when more accurate assessment is required. To determine the renal and cardiovascular prognosis and all-cause mortality of Japanese patients with type 2 diabetes showing a reduced estimated glomerular filtration rate (eGFR) without albuminuria. A population of 675 patients with type 2 diabetes was prospectively observed for 4 years to determine the renal and cardiovascular outcomes and mortality. The subjects were divided into the four groups: those with a preserved eGFR and no albuminuria (n = 306), a preserved eGFR and albuminuria (n = 151), a reduced eGFR and no albuminuria (n = 96), and a reduced eGFR and albuminuria (n = 122). The Cox proportional hazard model and Fine and Gray method were used to assess between-group differences in the risk of mortality and cardiovascular events. In the group with a reduced eGFR, the eGFR value did not significantly change in the subjects without albuminuria (0 ± 8 mL/min/1.73 m75 y - 4 points Hematocrit level <39% for men and <35% for women - 3 points Diabetes mellitus- 3 points Contrast media volume - 1 point for each 100 mL Decreased kidney function: Serum creatinine level >1.5 g/dL - 4 points or Estimated Glomerular filtration rate (online calculator) 2 for 40–60 mL/min/1.73 m2 4 for 20–40 mL/min/1.73 m2 6 for < 20 mL/min/1.73 m2 Although chronic kidney disease is a risk factor for cardiovascular disease it is unclear whether diabetic patients with a reduced glomerular filtration rate (GFR), independent of (micro)albuminuria, carry an increased risk of stroke. We therefore investigated the independent effect of estimated GFR (eGFR) on stroke events in patients with type 2 diabetes mellitus (T2DM). We studied T2DM patients with an eGFR >or=15 ml min(-1) per 1.73 m(2), who had no history of stroke. Patients were divided into four categories by the eGFR at baseline for comparison: >or=90, 60-89, 30-59 and 15-29 ml min(-1) per 1.73 m(2). The end point was an incident stroke event. The Cox proportional hazard model was used to calculate the hazard ratio (HR) and 95% confidence interval (CI). The study included a total of 1300 T2DM patients (546 women and 754 men) with a mean (+/-s.d.) age of 63+/-13 years. During a mean follow-up period of 3.7+/-1.4 years, 91 patients experienced an incident stroke event. Although a lower eGFR was associated with an increased stroke risk using a univariate model, statistical significance disappeared after adjusting for other risk factors including albuminuria. The HR (95% CI) was 0.75 (0.40-1.41, P=0.373), 0.99 (0.50-1.95, P=0.964) and 0.91 (0.36-2.28, P=0.844) for patients with eGFRs of 60-89, 30-59 and 15-29 ml min(-1) per 1.73 m(2), respectively, compared with patients with an eGFR >or=90. Clinical albuminuria remained a significant risk factor for stroke, and the adjusted HR compared with normoalbuminuria was 2.40 (1.46-3.95, P=0.001). In conclusion, the association between reduced GFR and stroke events in patients with T2DM is likely to be mediated by albuminuria. Serum creatinine is also utilized in GFR estimating equations such as the Modified Diet in Renal Disease (MDRD) and the CKD-EPI (Chronic Kidney Disease Epidemiology Collaboration) equation. These eGFR equations are superior to serum creatinine alone since they include race, age, and gender variables. GFR is classified into the following stages based on kidney disease. Kidney Disease Improving Global Outcomes (KDIGO) stages of chronic kidney disease (CKD): Stage 1 GFR greater than 90 ml/min/1.73 m² Stage 2 GFR-between 60 to 89 ml/min/1.73 m² Stage 3a  GFR 45 to 59 ml/min/1.73 m² Stage 3b GFR 30 to 44 ml/min/1.73 m² Stage 4 GFR of 15 to 29 ml/min/1.73 m² Stage 5-GFR less than 15 ml/min/1.73 m² (end-stage renal disease) Although much emphasis has been placed on screening for albuminuria in type II diabetic patients, less attention has been focused on the role of glomerular filtration rate (GFR) in the assessment of risk. Herein, we examined the association between GFR and vascular complications in a consecutive cohort of 5174 type II diabetic patients between 1995 and 2000. Renal function was assessed by GFR (estimated by Modification of Diet in Renal Disease equation). The frequency of chronic kidney disease (CKD) as defined by GFR <60 ml/min/1.73 m(2), micro- and macrovascular complications, and their associations were analyzed. In this study cohort, 6% had serum creatinine > or =150 micromol/l and 15.8% had CKD. After adjustment for potential confounders, including urinary albumin excretion, odds ratios [95% confidence interval (CI)] across different stages of estimated GFR (> or =90, 60-89, 30-59, 15-29, <15 ml/min/1.73 m(2)) for macrovascular disease were 1.00, 1.42 [1.12-1.80], 1.80 [1.32-2.45], 2.74 [1.64-4.56], and 4.05 [1.77-9.26], respectively (P for trend <0.001); for retinopathy were 1.00, 1.23 [1.04-1.46], 1.80 [1.40-2.30], 2.05 [1.25-3.37], and 4.12 [1.56-10.90], respectively (P for trend <0.001); for sensory neuropathy were 1.00, 1.53[1.27-1.85], 2.09 [1.58-2.76], 4.32 [2.41-7.77], and 3.16 [1.25-8.02], respectively (P for trend <0.001); and for microalbumuria (with GFR <15 ml/min/1.73 m(2) excluded from the analysis) were 1.00, 1.51 [1.30-1.75], 5.80 [4.52-7.44], and 52.5 [16.4-168.2] respectively (P for trend <0.001). Measurement of serum creatinine alone without GFR may underestimate renal impairment in type II diabetic patients. Decreasing GFR was significantly associated with increasing frequency of micro- and macrovascular complications. There are several different techniques used to calculate or estimate the glomerular filtration rate (GFR or eGFR). The above formula only applies for GFR calculation when it is equal to the Clearance Rate. The normal range of GFR, adjusted for body surface area, is 100–130 average 125 (mL/min)/(1.73 m2) in men and 90–120 (mL/min)/(1.73 m2) in women younger than the age of 40. In children, GFR measured by inulin clearance is 110 (mL/min)/(1.73 m2) until 2 years of age in both sexes, and then it progressively decreases. After age 40, GFR decreases progressively with age, by 0.4–1.2 mL/min per year. Estimated GFR (eGFR) is now recommended by clinical practice guidelines and regulatory agencies for routine evaluation of GFR whereas measured GFR (mGFR) is recommended as a confirmatory test when more accurate assessment is required. Protein in the urine is regarded as an independent marker for worsening of kidney function and cardiovascular disease. Hence, British guidelines append the letter \"P\" to the stage of chronic kidney disease if protein loss is significant. Stage 1: Slightly diminished function; kidney damage with normal or relatively high GFR (≥90 ml/min/1.73 m2) and persistent albuminuria. Kidney damage is defined as pathological abnormalities or markers of damage, including abnormalities in blood or urine tests or imaging studies. Stage 2: Mild reduction in GFR (60–89 ml/min/1.73 m2) with kidney damage. Kidney damage is defined as pathological abnormalities or markers of damage, including abnormalities in blood or urine tests or imaging studies. Stage 3: Moderate reduction in GFR (30–59 ml/min/1.73 m2):. British guidelines distinguish between stage 3A (GFR 45–59) and stage 3B (GFR 30–44) for purposes of screening and referral. Evidence shows that apart from positive effects on glycemic levels, canagliflozin also reduces the risk of heart attacks and heart failures. SGLT2 inhibitors, including canagliflozin, reduce the likelihood of hospitalization for congestive heart failure or progression of renal disease in persons with diabetes mellitus type 2 and reduce the likelihood of stroke and heart attack in persons with diabetes mellitus type 2 who have known atherosclerotic vascular disease. Contraindications Canaglifozin is contraindicated in: Type 1 diabetes Diabetic ketoacidosis Severe renal impairment (estimated glomerular filtration rate <30 mL/min/1.73 m2), end-stage renal disease Patients on dialysis The normal range of GFR, adjusted for body surface area, is 100–130 average 125 mL/min/1.73m2 in men and 90–120 mL/min/1.73m2 in women younger than the age of 40. In children, GFR measured by inulin clearance is 110 mL/min/1.73 m2 until 2 years of age in both sexes, and then it progressively decreases. After age 40, GFR decreases progressively with age, by 0.4–1.2 mL/min per year. Estimated GFR (eGFR) is now recommended by clinical practice guidelines and regulatory agencies for routine evaluation of GFR whereas measured GFR (mGFR) is recommended as a confirmatory test when more accurate assessment is required. To determine the renal and cardiovascular prognosis and all-cause mortality of Japanese patients with type 2 diabetes showing a reduced estimated glomerular filtration rate (eGFR) without albuminuria. A population of 675 patients with type 2 diabetes was prospectively observed for 4 years to determine the renal and cardiovascular outcomes and mortality. The subjects were divided into the four groups: those with a preserved eGFR and no albuminuria (n = 306), a preserved eGFR and albuminuria (n = 151), a reduced eGFR and no albuminuria (n = 96), and a reduced eGFR and albuminuria (n = 122). The Cox proportional hazard model and Fine and Gray method were used to assess between-group differences in the risk of mortality and cardiovascular events. In the group with a reduced eGFR, the eGFR value did not significantly change in the subjects without albuminuria (0 ± 8 mL/min/1.73 m6.0kPa). The basic defect in type 2 respiratory failure is characterized by: {| class=\"wikitable\" |PaO2 || decreased (< )or normal |- | PaCO2 || increased (> ) |- | PA-aO2 || normal |- |pH || <7.35 |} Intubation may be necessary for a patient with decreased oxygen content and oxygen saturation of the blood caused when their breathing is inadequate (hypoventilation), suspended (apnea), or when the lungs are unable to sufficiently transfer gasses to the blood. Such patients, who may be awake and alert, are typically critically ill with a multisystem disease or multiple severe injuries. Examples of such conditions include cervical spine injury, multiple rib fractures, severe pneumonia, acute respiratory distress syndrome (ARDS), or near-drowning. Specifically, intubation is considered if the arterial partial pressure of oxygen (PaO2) is less than 60 millimeters of mercury (mm Hg) while breathing an inspired O2 concentration (FIO2) of 50% or greater. In patients with elevated arterial carbon dioxide, an arterial partial pressure of CO2 (PaCO2) greater than 45 mm Hg in the setting of acidemia would prompt intubation, especially if a series of measurements demonstrate a worsening Respiratory failure results from inadequate gas exchange by the respiratory system, meaning that the arterial oxygen, carbon dioxide, or both cannot be kept at normal levels. A drop in the oxygen carried in the blood is known as hypoxemia; a rise in arterial carbon dioxide levels is called hypercapnia. Respiratory failure is classified as either Type 1 or Type 2, based on whether there is a high carbon dioxide level, and can be acute or chronic. In clinical trials, the definition of respiratory failure usually includes increased respiratory rate, abnormal blood gases (hypoxemia, hypercapnia, or both), and evidence of increased work of breathing. Respiratory failure causes an altered mental status due to ischemia in the brain. The typical partial pressure reference values are oxygen Pa O2 more than 80 mmHg (11 kPa) and carbon dioxide Pa CO2 less than 45 mmHg (6.0 kPa). Cause Causes Restricted oxygen flow to the body's tissue that leads to hypoxia can be caused by a variety of situations or other underlying conditions. High altitude (above 3048 metres/10,000 feet) Hypoventilation – failure of respiratory pump due to any cause (fatigue, barbiturate poisoning, pneumothorax, etc.) Ventilation perfusion mismatch Obstructed airway Drowning Abnormal pulmonary function Chronic obstructive pulmonary diseases (COPD) Neuromuscular diseases or interstitial lung disease Constrained blood flow to a tissue (such as atherosclerosis or vasoconstriction) Blockage in blood flow like a sickle cell crisis Low or no blood flow caused by bleeding or heart attack Malformed vascular system such as an anomalous coronary artery Limited oxygen transportation due to anemia While respiring in the open air at high altitudes, the human body experiences altitude sickness and hypoxemia due to a low partial pressure of oxygen, decreasing the carriage of oxygen by hemoglobin. Because A–a gradient is approximated as: (150 − 5/4(P)) – at sea level and on room air (0.21x(760-47) = 149.7 mmHg for the alveolar oxygen partial pressure, after accounting for the water vapor), the direct mathematical cause of a large value is that the blood has a low , a low Pa, or both. is very easily exchanged in the lungs and low Pa directly correlates with high minute ventilation; therefore a low arterial Pa indicates that extra respiratory effort is being used to oxygenate the blood. A low indicates that the patient's current minute ventilation (whether high or normal) is not enough to allow adequate oxygen diffusion into the blood. Therefore, the A–a gradient essentially demonstrates a high respiratory effort (low arterial Pa) relative to the achieved level of oxygenation (arterial ). A high A–a gradient could indicate a patient breathing hard to achieve normal oxygenation, a patient breathing normally and attaining low oxygenation, or a patient breathing hard and still Asthma and COPD Acute exacerbations of chronic respiratory diseases, mainly asthma and chronic obstructive pulmonary disease (COPD), are assessed as emergencies and treated with oxygen therapy, bronchodilators, steroids or theophylline, have an urgent chest X-ray and arterial blood gases and are referred for intensive care if necessary. Noninvasive ventilation in the ED has reduced the requirement for tracheal intubation in many cases of severe exacerbations of COPD. Special facilities, training, and equipment An ED requires different equipment and different approaches than most other hospital divisions. Patients frequently arrive with unstable conditions, and so must be treated quickly. They may be unconscious, and information such as their medical history, allergies, and blood type may be unavailable. ED staff are trained to work quickly and effectively even with minimal information. When 100% oxygen (1.00 Fi) is used initially for an adult, it is easy to calculate the next Fi to be used, and easy to estimate the shunt fraction. The estimated shunt fraction refers to the amount of oxygen not being absorbed into the circulation. In normal physiology, gas exchange of oxygen and carbon dioxide occurs at the level of the alveoli in the lungs. The existence of a shunt refers to any process that hinders this gas exchange, leading to wasted oxygen inspired and the flow of un-oxygenated blood back to the left heart, which ultimately supplies the rest of the body with de-oxygenated blood. When using 100% oxygen, the degree of shunting is estimated as 700 mmHg - measured Pa. For each difference of 100 mmHg, the shunt is 5%. A shunt of more than 25% should prompt a search for the cause of this hypoxemia, such as mainstem intubation or pneumothorax, and should be treated accordingly. If such complications are not present, other causes must be sought after, and positive Berlin Criteria: as stated on UpToDate (2020) Timing: onset of respiratory symptoms within one week of a injury/insult. Chest Imaging: either chest x-ray or CT scan, must show bilateral opacities that cannot be fully explained by other conditions such as effusion, lung/lobar collapse, or lung nodules. Origin of Edema: respiratory failure that cannot be fully explained by cardiac failure or fluid overload, this needs objective assessment such as an echocardiogram. Impaired Oxygenation: this can be determined by looking at the ratio of arterial oxygen tension to fraction of inspired oxygen (PaO2/FiO2) that can be obtained based on an arterial blood gas test. Note: all PaO2/FiO2 ratios used in the determination of the severity of ARDS require that the patient be on a ventilator at a setting that includes 5 cm H2O or more of positive end-expiratory pressure (PEEP) or continuous positive airway pressure (CPAP). The typical partial pressure reference values are oxygen Pa O2 more than 80 mmHg (11 kPa) and carbon dioxide Pa CO2 less than 45 mmHg (6.0 kPa). Cause Several types of conditions can potentially result in respiratory failure: Conditions that reduce the flow of air into and out of the lungs, including physical obstruction by foreign bodies or masses and reduced breathing due to drugs or changes to the chest. Conditions that impair the lungs' blood supply. These include thromboembolic conditions and conditions that reduce the output of the right heart, such as right heart failure and some myocardial infarctions. Conditions that limit the ability of the lung tissue to exchange oxygen and carbon dioxide between the blood and the air within the lungs. Any disease which can damage the lung tissue can fit into this category. The most common causes are (in no particular order) infections, interstitial lung disease, and pulmonary oedema. Diagnosis Intermittent positive pressure breathing (IPPB) is a respiratory therapy treatment for people who are hypoventilating. While not a preferred method due to cost, IPPB is used to expand the lungs, deliver aerosol medications, and in some circumstances ventilate the patient. Indications IPPB may be indicated for patients who are at risk for developing atelectasis and who are unable or unwilling to breathe deeply without assistance. In patients with severe lung hyperinflation, IPPB may decrease dyspnea and discomfort during nebulized therapy. Contraindications Most contraindications are relative, such as nausea, hemodynamic instability, tracheal fistula, singulation and hemoptysis. Untreated tension pneumothorax is an absolute contraindication. IMPLEMENTATION When treating atelectasis - Therapy should be volume-oriented 2. Tidal volumes(VT) must be measured 3. VT goals must be set 4. VT goal of 10-15mL/kg ofbody weight A 68-year-old man with severe dyspnea was admitted as an emergency case. He had no past history of any respiratory or neuromuscular diseases. Immediately after insufflation of oxygen, respiratory arrest occurred. The blood gas analysis showed hypoxemia and severe hypercapnia (PaO2; 32 mmHg, PaCO2; 127 mmHg). We diagnosed as CO2 narcosis, and he was treated with a respirator in the ICU. He showed nonflaccid bilateral diaphragmatic paralysis and muscle atrophy of the upper extremities. As the EMG showed giant spikes of neurogenic pattern, he was diagnosed as ALS. Weaning from the respirator failed because of his respiratory muscle fatigue. He was given rehabilitation during the day time and ventilatory support with the respirator during the night. We conclude that if we meet with an emergency patient with CO2 narcosis without any pulmonary disorder, we have to suspect neuromuscular diseases, e.q. ALS. In some of such cases, mechanical ventilation supports social rehabilitation. We present a case of postural hypoxemia with a final diagnosis of myasthenia gravis (MG). A 62-year-old man experienced double vision in his left eye from December 2008 and received a diagnosis of diabetic neuropathy. From mid-December he began to experience breathing difficulties at night when in a supine position and was admitted to our hospital. Bilateral diaphragmatic elevation was observed on a chest X-ray film, and lower lung atelectasis and an anterior mediastinal tumor were observed on chest CT. However, his breathing difficulties only occurred when he was in a supine position. Therefore, we performed blood gas analysis in supine and sitting positions. Hypoxemia, hypercapnia and an increase in A-aDO2 were observed in the supine position, leading to a diagnosis of postural hypoxemia. Due to the exacerbation of his double vision, the patient was referred to the ophthalmology and neurology departments where he tested positive for anti-acetylcholine receptor antibodies and also on a tensilon test, resulting in a final diagnosis of MG. During the tensilon test, the patient's breathing difficulties in the supine position improved, and therefore his postural hypoxemia was thought to have resulted from diaphragmatic muscle weakness as a result of MG. MG respiratory failure is typically of the acute fulminating type and is considered to be a critical condition. However, it should be noted that there are cases, such as the present one, in which MG presents as postural hypoxemia. The authors report a rare, recently diagnosed and atypical mishap during one-lung ventilation (OLV) via a double lumen tube (DLT) and left-sided thoracotomy: an ipsilateral pneumothorax during ventilation of the right lung. This occurred in a 63-year-old patient with chronic obstructive airway disease who was scheduled for urgent repair of a descending thoracic aortic aneurysm. Anaesthesia and surgery were uneventful until aortic cross-clamping release. The common presentation of increased intrathoracic extrapleural pressure owing to a pneumothorax in patients with mechanically ventilated lungs is a rapid decrease in oxygen saturation, followed or paralleled by haemodynamic deterioration. Although the above presentation could be seen in this case, the diagnosis of a tension pneumothorax was delayed twice. First, symptoms were initially obscured by haemodynamic changes resulting from a head-down tilt and aortic declamping. Second, since the lack of consolidation after aortic declamping focused attention on the airway problems, complications resulting from the use of a DLT were primarily considered. In particular, since breathing sounds were detectable initially, malposition or torsion of the DLT had to be excluded by fibre-optic bronchoscopy, which involved a further delay. Finally, two observations led to the diagnosis of a right-sided tension pneumothorax: (1) bullae of the contralateral lung, detected during thoracotomy; (2) the finding that ventilation of both lungs and the left lung subsequently increased arterial (SaO2) and mixed venous oxygen saturation (SvO2) and the circulatory status, but ventilation of the right lung caused a deterioration. Chest radiography and insertion of a chest tube with drainage of air, thereafter, validated our hypothesis. The time course of oxygen desaturation during OLV and tension pneumothorax was as severe as expected; the time course of haemodynamic deterioration, however, appeared quicker and had more impact than expected. Assuming that mediastinal deviation was not hindered by contralateral intrathoracic pressure during thoracotomy, we believed that circulation should be depressed later or to a lesser extent in patients with an intraoperative pneumothorax. Yet, during thoracotomy, decrease in cardiac filling and output during tension pneumothorax in OLV obviously results primarily from the immovability of the mediastinum owing to mediastinal fixation and is at least as decisive as the contralateral intrathoracic pressure in closed-chest patients. In summary, a tension pneumothorax during one-lung ventilation and thoracotomy is a rare, but disastrous complication during the use of a DLT, which has not, to our knowledge, been reported previously. We recommend that tension pneumothorax be added to the list of complications and problems during OLV by the use of a DLT, especially in patients with structural lung diseases. Respiratory compromise describes a deterioration in respiratory function with a high likelihood of rapid progression to respiratory failure and death. Respiratory failure occurs when inadequate gas exchange by the respiratory system occurs, with a low oxygen level or a high carbon dioxide level. Causes Patients in acute care hospitals, particularly those with respiratory conditions, are at risk for developing respiratory compromise. Respiratory failure requiring emergency mechanical ventilation occurs in over 40,000 patients per year in the United States. In postoperative patients in the United States, the National Surgical Quality Improvement Program reports that 1.03% of all surgical patients require an unplanned intubation postoperatively. General principles of respiratory evaluation in patients with PNS involvement, regardless of cause, include assessment of pulmonary mechanics, such as maximal inspiratory force (MIF) and vital capacity (VC), and evaluation of strength of bulbar muscles. Regardless of the cause of weakness, endotracheal intubation should be considered when the MIF falls to <–25 cmH2O or the VC is <1 L. Also, patients with severe palatal weakness may require endotracheal intubation in order to prevent acute upper airway obstruction or recurrent aspiration. Arterial blood gases and oxygen saturation from pulse oximetry are used to follow patients with potential respiratory compromise from PNS dysfunction. However, intubation and mechanical ventilation should be undertaken based on clinical assessment rather than waiting until oxygen saturation drops or CO2 retention develops from hypoventilation. Noninvasive mechanical ventilation may be considered initially in lieu of endotracheal intubation but is If Pa:Fi < 300 mmHg (40 kPa), then the definitions recommended a classification as \"acute lung injury\" (ALI). Note that according to these criteria, arterial blood gas analysis and chest X-ray were required for formal diagnosis. Limitations of these definitions include lack of precise definition of acuity, nonspecific imaging criteria, lack of precise definition of hypoxemia with regards to PEEP (affects arterial oxygen partial pressure), arbitrary Pa thresholds without systematic data. are usually necessary (see further on). However, a fairly severe impairment of ventilation may occur before the first sign of dyspnea appears and before there is elevation of arterial carbon dioxide content. Incipient respiratory failure may be evident by tachypnea and a decrease in arterial oxygen tension (Po2 less than 85 mm Hg) reflecting pulmonary atelectasis. When respiratory failure arises gradually as the patient weakens over days, there is slight tachycardia, diaphoresis, restlessness, and tachypnea. Attempts to forestall intubation and positive-pressure ventilation by using negative-pressure cuirass-type devices have been unsatisfactory in our experience. Patients with oropharyngeal weakness require intubation even earlier so as to prevent aspiration, but full mechanical ventilation is not always necessary at the same time. Patients in these circumstances should obviously be admitted to an intensive care unit staffed by personnel skilled in maintaining ventilation and airway A 33-year-old woman visited the emergency department presenting with fever and dyspnea. She was pregnant with gestational age of 31 weeks and 6 days. She had dysuria for 7 days, and fever and dyspnea for 1 day. The vital signs were as follows: blood pressure 110/70 mmHg, heart rate 118 beats/minute, respiratory rate 28/minute, body temperature 38.7℃, and oxygen saturation by pulse oximetry 84% during inhalation of 5 liters of oxygen by nasal prongs. Crackles were heard over both lung fields. There were no signs of uterine contractions. Chest X-ray and chest computed tomography scan showed multiple consolidations and air bronchograms in both lungs. According to urinalysis, there was pyuria and microscopic hematuria. She was diagnosed with community-acquired pneumonia and urinary tract infection (UTI) that progressed to severe sepsis and acute respiratory failure. We found extended-spectrum beta-lactamase producing 6.0kPa). The basic defect in type 2 respiratory failure is characterized by: {| class=\"wikitable\" |PaO2 || decreased (< )or normal |- | PaCO2 || increased (> ) |- | PA-aO2 || normal |- |pH || <7.35 |} Intubation may be necessary for a patient with decreased oxygen content and oxygen saturation of the blood caused when their breathing is inadequate (hypoventilation), suspended (apnea), or when the lungs are unable to sufficiently transfer gasses to the blood. Such patients, who may be awake and alert, are typically critically ill with a multisystem disease or multiple severe injuries. Examples of such conditions include cervical spine injury, multiple rib fractures, severe pneumonia, acute respiratory distress syndrome (ARDS), or near-drowning. Specifically, intubation is considered if the arterial partial pressure of oxygen (PaO2) is less than 60 millimeters of mercury (mm Hg) while breathing an inspired O2 concentration (FIO2) of 50% or greater. In patients with elevated arterial carbon dioxide, an arterial partial pressure of CO2 (PaCO2) greater than 45 mm Hg in the setting of acidemia would prompt intubation, especially if a series of measurements demonstrate a worsening Respiratory failure results from inadequate gas exchange by the respiratory system, meaning that the arterial oxygen, carbon dioxide, or both cannot be kept at normal levels. A drop in the oxygen carried in the blood is known as hypoxemia; a rise in arterial carbon dioxide levels is called hypercapnia. Respiratory failure is classified as either Type 1 or Type 2, based on whether there is a high carbon dioxide level, and can be acute or chronic. In clinical trials, the definition of respiratory failure usually includes increased respiratory rate, abnormal blood gases (hypoxemia, hypercapnia, or both), and evidence of increased work of breathing. Respiratory failure causes an altered mental status due to ischemia in the brain. The typical partial pressure reference values are oxygen Pa O2 more than 80 mmHg (11 kPa) and carbon dioxide Pa CO2 less than 45 mmHg (6.0 kPa). Cause Causes Restricted oxygen flow to the body's tissue that leads to hypoxia can be caused by a variety of situations or other underlying conditions. High altitude (above 3048 metres/10,000 feet) Hypoventilation – failure of respiratory pump due to any cause (fatigue, barbiturate poisoning, pneumothorax, etc.) Ventilation perfusion mismatch Obstructed airway Drowning Abnormal pulmonary function Chronic obstructive pulmonary diseases (COPD) Neuromuscular diseases or interstitial lung disease Constrained blood flow to a tissue (such as atherosclerosis or vasoconstriction) Blockage in blood flow like a sickle cell crisis Low or no blood flow caused by bleeding or heart attack Malformed vascular system such as an anomalous coronary artery Limited oxygen transportation due to anemia While respiring in the open air at high altitudes, the human body experiences altitude sickness and hypoxemia due to a low partial pressure of oxygen, decreasing the carriage of oxygen by hemoglobin. Because A–a gradient is approximated as: (150 − 5/4(P)) – at sea level and on room air (0.21x(760-47) = 149.7 mmHg for the alveolar oxygen partial pressure, after accounting for the water vapor), the direct mathematical cause of a large value is that the blood has a low , a low Pa, or both. is very easily exchanged in the lungs and low Pa directly correlates with high minute ventilation; therefore a low arterial Pa indicates that extra respiratory effort is being used to oxygenate the blood. A low indicates that the patient's current minute ventilation (whether high or normal) is not enough to allow adequate oxygen diffusion into the blood. Therefore, the A–a gradient essentially demonstrates a high respiratory effort (low arterial Pa) relative to the achieved level of oxygenation (arterial ). A high A–a gradient could indicate a patient breathing hard to achieve normal oxygenation, a patient breathing normally and attaining low oxygenation, or a patient breathing hard and still Asthma and COPD Acute exacerbations of chronic respiratory diseases, mainly asthma and chronic obstructive pulmonary disease (COPD), are assessed as emergencies and treated with oxygen therapy, bronchodilators, steroids or theophylline, have an urgent chest X-ray and arterial blood gases and are referred for intensive care if necessary. Noninvasive ventilation in the ED has reduced the requirement for tracheal intubation in many cases of severe exacerbations of COPD. Special facilities, training, and equipment An ED requires different equipment and different approaches than most other hospital divisions. Patients frequently arrive with unstable conditions, and so must be treated quickly. They may be unconscious, and information such as their medical history, allergies, and blood type may be unavailable. ED staff are trained to work quickly and effectively even with minimal information. When 100% oxygen (1.00 Fi) is used initially for an adult, it is easy to calculate the next Fi to be used, and easy to estimate the shunt fraction. The estimated shunt fraction refers to the amount of oxygen not being absorbed into the circulation. In normal physiology, gas exchange of oxygen and carbon dioxide occurs at the level of the alveoli in the lungs. The existence of a shunt refers to any process that hinders this gas exchange, leading to wasted oxygen inspired and the flow of un-oxygenated blood back to the left heart, which ultimately supplies the rest of the body with de-oxygenated blood. When using 100% oxygen, the degree of shunting is estimated as 700 mmHg - measured Pa. For each difference of 100 mmHg, the shunt is 5%. A shunt of more than 25% should prompt a search for the cause of this hypoxemia, such as mainstem intubation or pneumothorax, and should be treated accordingly. If such complications are not present, other causes must be sought after, and positive Berlin Criteria: as stated on UpToDate (2020) Timing: onset of respiratory symptoms within one week of a injury/insult. Chest Imaging: either chest x-ray or CT scan, must show bilateral opacities that cannot be fully explained by other conditions such as effusion, lung/lobar collapse, or lung nodules. Origin of Edema: respiratory failure that cannot be fully explained by cardiac failure or fluid overload, this needs objective assessment such as an echocardiogram. Impaired Oxygenation: this can be determined by looking at the ratio of arterial oxygen tension to fraction of inspired oxygen (PaO2/FiO2) that can be obtained based on an arterial blood gas test. Note: all PaO2/FiO2 ratios used in the determination of the severity of ARDS require that the patient be on a ventilator at a setting that includes 5 cm H2O or more of positive end-expiratory pressure (PEEP) or continuous positive airway pressure (CPAP). The typical partial pressure reference values are oxygen Pa O2 more than 80 mmHg (11 kPa) and carbon dioxide Pa CO2 less than 45 mmHg (6.0 kPa). Cause Several types of conditions can potentially result in respiratory failure: Conditions that reduce the flow of air into and out of the lungs, including physical obstruction by foreign bodies or masses and reduced breathing due to drugs or changes to the chest. Conditions that impair the lungs' blood supply. These include thromboembolic conditions and conditions that reduce the output of the right heart, such as right heart failure and some myocardial infarctions. Conditions that limit the ability of the lung tissue to exchange oxygen and carbon dioxide between the blood and the air within the lungs. Any disease which can damage the lung tissue can fit into this category. The most common causes are (in no particular order) infections, interstitial lung disease, and pulmonary oedema. Diagnosis Intermittent positive pressure breathing (IPPB) is a respiratory therapy treatment for people who are hypoventilating. While not a preferred method due to cost, IPPB is used to expand the lungs, deliver aerosol medications, and in some circumstances ventilate the patient. Indications IPPB may be indicated for patients who are at risk for developing atelectasis and who are unable or unwilling to breathe deeply without assistance. In patients with severe lung hyperinflation, IPPB may decrease dyspnea and discomfort during nebulized therapy. Contraindications Most contraindications are relative, such as nausea, hemodynamic instability, tracheal fistula, singulation and hemoptysis. Untreated tension pneumothorax is an absolute contraindication. IMPLEMENTATION When treating atelectasis - Therapy should be volume-oriented 2. Tidal volumes(VT) must be measured 3. VT goals must be set 4. VT goal of 10-15mL/kg ofbody weight A 68-year-old man with severe dyspnea was admitted as an emergency case. He had no past history of any respiratory or neuromuscular diseases. Immediately after insufflation of oxygen, respiratory arrest occurred. The blood gas analysis showed hypoxemia and severe hypercapnia (PaO2; 32 mmHg, PaCO2; 127 mmHg). We diagnosed as CO2 narcosis, and he was treated with a respirator in the ICU. He showed nonflaccid bilateral diaphragmatic paralysis and muscle atrophy of the upper extremities. As the EMG showed giant spikes of neurogenic pattern, he was diagnosed as ALS. Weaning from the respirator failed because of his respiratory muscle fatigue. He was given rehabilitation during the day time and ventilatory support with the respirator during the night. We conclude that if we meet with an emergency patient with CO2 narcosis without any pulmonary disorder, we have to suspect neuromuscular diseases, e.q. ALS. In some of such cases, mechanical ventilation supports social rehabilitation. We present a case of postural hypoxemia with a final diagnosis of myasthenia gravis (MG). A 62-year-old man experienced double vision in his left eye from December 2008 and received a diagnosis of diabetic neuropathy. From mid-December he began to experience breathing difficulties at night when in a supine position and was admitted to our hospital. Bilateral diaphragmatic elevation was observed on a chest X-ray film, and lower lung atelectasis and an anterior mediastinal tumor were observed on chest CT. However, his breathing difficulties only occurred when he was in a supine position. Therefore, we performed blood gas analysis in supine and sitting positions. Hypoxemia, hypercapnia and an increase in A-aDO2 were observed in the supine position, leading to a diagnosis of postural hypoxemia. Due to the exacerbation of his double vision, the patient was referred to the ophthalmology and neurology departments where he tested positive for anti-acetylcholine receptor antibodies and also on a tensilon test, resulting in a final diagnosis of MG. During the tensilon test, the patient's breathing difficulties in the supine position improved, and therefore his postural hypoxemia was thought to have resulted from diaphragmatic muscle weakness as a result of MG. MG respiratory failure is typically of the acute fulminating type and is considered to be a critical condition. However, it should be noted that there are cases, such as the present one, in which MG presents as postural hypoxemia. The authors report a rare, recently diagnosed and atypical mishap during one-lung ventilation (OLV) via a double lumen tube (DLT) and left-sided thoracotomy: an ipsilateral pneumothorax during ventilation of the right lung. This occurred in a 63-year-old patient with chronic obstructive airway disease who was scheduled for urgent repair of a descending thoracic aortic aneurysm. Anaesthesia and surgery were uneventful until aortic cross-clamping release. The common presentation of increased intrathoracic extrapleural pressure owing to a pneumothorax in patients with mechanically ventilated lungs is a rapid decrease in oxygen saturation, followed or paralleled by haemodynamic deterioration. Although the above presentation could be seen in this case, the diagnosis of a tension pneumothorax was delayed twice. First, symptoms were initially obscured by haemodynamic changes resulting from a head-down tilt and aortic declamping. Second, since the lack of consolidation after aortic declamping focused attention on the airway problems, complications resulting from the use of a DLT were primarily considered. In particular, since breathing sounds were detectable initially, malposition or torsion of the DLT had to be excluded by fibre-optic bronchoscopy, which involved a further delay. Finally, two observations led to the diagnosis of a right-sided tension pneumothorax: (1) bullae of the contralateral lung, detected during thoracotomy; (2) the finding that ventilation of both lungs and the left lung subsequently increased arterial (SaO2) and mixed venous oxygen saturation (SvO2) and the circulatory status, but ventilation of the right lung caused a deterioration. Chest radiography and insertion of a chest tube with drainage of air, thereafter, validated our hypothesis. The time course of oxygen desaturation during OLV and tension pneumothorax was as severe as expected; the time course of haemodynamic deterioration, however, appeared quicker and had more impact than expected. Assuming that mediastinal deviation was not hindered by contralateral intrathoracic pressure during thoracotomy, we believed that circulation should be depressed later or to a lesser extent in patients with an intraoperative pneumothorax. Yet, during thoracotomy, decrease in cardiac filling and output during tension pneumothorax in OLV obviously results primarily from the immovability of the mediastinum owing to mediastinal fixation and is at least as decisive as the contralateral intrathoracic pressure in closed-chest patients. In summary, a tension pneumothorax during one-lung ventilation and thoracotomy is a rare, but disastrous complication during the use of a DLT, which has not, to our knowledge, been reported previously. We recommend that tension pneumothorax be added to the list of complications and problems during OLV by the use of a DLT, especially in patients with structural lung diseases. Respiratory compromise describes a deterioration in respiratory function with a high likelihood of rapid progression to respiratory failure and death. Respiratory failure occurs when inadequate gas exchange by the respiratory system occurs, with a low oxygen level or a high carbon dioxide level. Causes Patients in acute care hospitals, particularly those with respiratory conditions, are at risk for developing respiratory compromise. Respiratory failure requiring emergency mechanical ventilation occurs in over 40,000 patients per year in the United States. In postoperative patients in the United States, the National Surgical Quality Improvement Program reports that 1.03% of all surgical patients require an unplanned intubation postoperatively. General principles of respiratory evaluation in patients with PNS involvement, regardless of cause, include assessment of pulmonary mechanics, such as maximal inspiratory force (MIF) and vital capacity (VC), and evaluation of strength of bulbar muscles. Regardless of the cause of weakness, endotracheal intubation should be considered when the MIF falls to <–25 cmH2O or the VC is <1 L. Also, patients with severe palatal weakness may require endotracheal intubation in order to prevent acute upper airway obstruction or recurrent aspiration. Arterial blood gases and oxygen saturation from pulse oximetry are used to follow patients with potential respiratory compromise from PNS dysfunction. However, intubation and mechanical ventilation should be undertaken based on clinical assessment rather than waiting until oxygen saturation drops or CO2 retention develops from hypoventilation. Noninvasive mechanical ventilation may be considered initially in lieu of endotracheal intubation but is If Pa:Fi < 300 mmHg (40 kPa), then the definitions recommended a classification as \"acute lung injury\" (ALI). Note that according to these criteria, arterial blood gas analysis and chest X-ray were required for formal diagnosis. Limitations of these definitions include lack of precise definition of acuity, nonspecific imaging criteria, lack of precise definition of hypoxemia with regards to PEEP (affects arterial oxygen partial pressure), arbitrary Pa thresholds without systematic data. are usually necessary (see further on). However, a fairly severe impairment of ventilation may occur before the first sign of dyspnea appears and before there is elevation of arterial carbon dioxide content. Incipient respiratory failure may be evident by tachypnea and a decrease in arterial oxygen tension (Po2 less than 85 mm Hg) reflecting pulmonary atelectasis. When respiratory failure arises gradually as the patient weakens over days, there is slight tachycardia, diaphoresis, restlessness, and tachypnea. Attempts to forestall intubation and positive-pressure ventilation by using negative-pressure cuirass-type devices have been unsatisfactory in our experience. Patients with oropharyngeal weakness require intubation even earlier so as to prevent aspiration, but full mechanical ventilation is not always necessary at the same time. Patients in these circumstances should obviously be admitted to an intensive care unit staffed by personnel skilled in maintaining ventilation and airway A 33-year-old woman visited the emergency department presenting with fever and dyspnea. She was pregnant with gestational age of 31 weeks and 6 days. She had dysuria for 7 days, and fever and dyspnea for 1 day. The vital signs were as follows: blood pressure 110/70 mmHg, heart rate 118 beats/minute, respiratory rate 28/minute, body temperature 38.7℃, and oxygen saturation by pulse oximetry 84% during inhalation of 5 liters of oxygen by nasal prongs. Crackles were heard over both lung fields. There were no signs of uterine contractions. Chest X-ray and chest computed tomography scan showed multiple consolidations and air bronchograms in both lungs. According to urinalysis, there was pyuria and microscopic hematuria. She was diagnosed with community-acquired pneumonia and urinary tract infection (UTI) that progressed to severe sepsis and acute respiratory failure. We found extended-spectrum beta-lactamase producing 18 mmHg) suggests fluid volume overload or cardiac failure rather than ARDS. Pneumonia caused by viruses or by This type of respiratory failure is caused by conditions that affect oxygenation, such as: Low ambient oxygen (e.g. at high altitude) Ventilation-perfusion mismatch (parts of the lung receive oxygen but not enough blood to absorb it, e.g. pulmonary embolism) Alveolar hypoventilation (decreased minute volume due to reduced respiratory muscle activity, e.g. in acute neuromuscular disease); this form can also cause type 2 respiratory failure if severe. Diffusion problem (oxygen cannot enter the capillaries due to parenchymal disease, e.g. in pneumonia or ARDS) Shunt (oxygenated blood mixes with non-oxygenated blood from the venous system, e.g. right to left shunt) Type 2 Hypoxemia (PaO2 <8kPa or normal) with hypercapnia (PaCO2 >6.0kPa). The basic defect in type 2 respiratory failure is characterized by: {| class=\"wikitable\" |PaO2 || decreased (< )or normal |- | PaCO2 || increased (> ) |- | PA-aO2 || normal |- |pH || <7.35 |} This type of respiratory failure occurs with alveolar flooding and subsequent intrapulmonary shunt physiology. Alveolar flooding may be a consequence of pulmonary edema, pneumonia, or alveolar hemorrhage. Pulmonary edema can be further categorized as occurring due to elevated pulmonary microvascular pressures, as seen in heart failure and intravascular volume overload or ARDS (“low-pressure pulmonary edema,” Chap. 322). This syndrome is defined by acute onset (≤1 week) of bilateral opacities on chest imaging that are not fully explained by cardiac failure or fluid overload and of shunt physiology requiring positive end-expiratory pressure (PEEP). Type I respiratory failure occurs in clinical settings such as sepsis, gastric aspiration, pneumonia, near-drowning, multiple blood transfusions, and pancreatitis. The mortality rate among patients with ARDS was traditionally very high (50–70%), although changes in patient care have led to mortality rates closer to 30% (see below). Equations Abbreviated alveolar air equation PAO2, PEO2, and PIO2 are the partial pressures of oxygen in alveolar, expired, and inspired gas, respectively, and VD/Vt is the ratio of physiologic dead space over tidal volume. Medicine In medicine, the FIO2 is the assumed percentage of oxygen concentration participating in gas exchange in the alveoli. Uses The FIO2 is used in the APACHE II (Acute Physiology and Chronic Health Evaluation II) severity of disease classification system for intensive care unit patients. For FIO2 values equal to or greater than 0.5, the alveolar–arterial gradient value should be used in the APACHE II score calculation. Otherwise, the PaO2 will suffice. The ratio between partial pressure of oxygen in arterial blood (PaO2) and FIO2 is used as an indicator of hypoxemia per the American-European Consensus Conference on lung injury. A high FIO2 has been shown to alter the ratio of PaO2/FIO2. Cardiovascular dysfunction a) Shock b) Cardiac Arrest c) Severe hypoperfusion (lactate >5 mmol/L or >45 mg/dL) d) Severe acidosis (pH<7.1) e) Use of continuous vasoactive drugs f) Cardio-pulmonary resuscitation Respiratory dysfunction g) Acute cyanosis h) Gasping i) Severe tachypnea (respiratory rate>40 breaths per minute) j) Severe bradypnea (respiratory rate<6 breaths per minute) k) Severe hypoxemia (O2 saturation <90% for ≥60min or PAO2/FiO2<200) l) Intubation and ventilation not related to anaesthesia Renal dysfunction m) Oliguria non responsive to fluids or diuretics n) Severe acute azotemia (creatinine >300 μmol/ml or >3.5 mg/dL) o) Dialysis for acute renal failure Coagulation dysfunction p) Failure to form clots q) Severe acute thrombocytopenia (<50,000 platelets/ml) r) Massive transfusion of blood or red cells (≥ 5 units) Hepatic dysfunction s) Jaundice in the presence of pre-eclampsia Noninvasive estimation techniques have been proposed. Clinical significance Because of the large compliance of pulmonary circulation, it provides an indirect measure of the left atrial pressure. For example, it is considered the gold standard for determining the cause of acute pulmonary edema; this is likely to be present at a PWP of >20mmHg. It has also been used to diagnose severity of left ventricular failure and mitral stenosis, given that elevated pulmonary capillary wedge pressure strongly suggests failure of left ventricular output. Traditionally, it was believed that pulmonary edema with normal PWP suggested a diagnosis of acute respiratory distress syndrome (ARDS) or non cardiogenic pulmonary edema (as in opiate poisoning). However, since capillary hydrostatic pressure exceeds wedge pressure once the balloon is deflated (to promote a gradient for forward flow), a normal wedge pressure cannot conclusively differentiate between hydrostatic pulmonary edema and ARDS. Removal of the extrinsic cause or treatment of underlying infection if identifed. Corticosteroid treatment may be used if no cause is identif ed. Causes include ventilation-perfusion (V/Q) mismatch, right-to-left shunt, hypoventilation, low inspired O2 content (important at altitudes), and diffusion impairment. Findings depend on the etiology. ↓HbO2 saturation, cyanosis, tachypnea, shortness of breath, pleuritic chest pain, and altered mental status may be seen. Pulse oximetry: Demonstrates ↓ HbO2 saturation. CXR: To rule out ARDS, atelectasis, or an infltrative process (e.g., pneumonia) and to look for signs of pulmonary embolism. ABGs: To evaluate PaO2 and to calculate the alveolar-arterial (A-a) oxygen gradient ([(Patm − 47) × FiO2 − (PaCO2/0.8)] − PaO2). An ↑ A-a gradient suggests a V/Q mismatch or a diffusion impairment. Figure 2.15-4 summarizes the approach toward hypoxemic patients. Is PaCO2 increased? Hypoventilation Yes Yes No No Is PAO2 − PaO2 increased? Twenty-nine patients, divided into three groups: 1) chronic obstructive pulmonary disease; 2) acute or chronic pulmonary disease with left heart failure; 3) respiratory insufficiency after peritonitis, pancreatitis, and/or sepsis, were studied during respirator treatment with regard to gas exchange, breathing mechanics and central circulation. The dead space ventilation was somewhat greater in group 1 than in the other groups. The alveolar-arterial oxygen tension difference was least in group 1, greater in group 2 and extremely high in group 3. Neither dynamic compliance of the thorax nor inspiratory resistance showed any significant differences between the groups. The cardiac output had the highest values in group 3. The venous admixture was generally small in group 1 and extremely large in group 3. The pulmonary artery pressures were highest in group 2. Three variables proved to be valuable when assessing the prognosis of a patient: a large venous admixture; a large alveolar-arterial oxygen tension difference, and a high pulmonary artery pressure indicated a less favourable prognosis. 1. The use of an oxygen chamber in the treatment of pneumonia patients makes it possible to administer this gas for long periods of time under exactly known conditions. The medical and nursing care of the patient is greatly facilitated. 2. Prolonged inhalation of oxygen varying from 40 to 60 per cent appears to be without harm. 3. Oxygen administered to intensely anoxemic patients almost immediately clears up this anoxemia. Cyanosis disappears with the anoxemia. 4. The removal of patients from the high oxygen while they are still sick and while examination shows that there are still extensive edema and infiltration of the lung results in a return of the intense anoxemia. 5. It is sometimes impossible to clear up the anoxemia, even when as high as 60 per cent of oxygen is given, especially when there are considerable edema and infiltration of the lungs. 6. Five cases in which the prognosis was grave recovered. Three cases, one of tuberculosis, one with a Pneumococcus Type III infection, and a third with a pneumonia superimposed on a chronic pulmonary condition, died. 7. In all cases there appeared to be an improvement in the patient's condition. In one case, particularly, with an intense degree of anoxemia, the patient became moribund and pulseless. Following the administration of 60 per cent of oxygen there was a lowering of the heart rate from 160 to 120, the return of the pulse to the radial artery, the color became bright pink, and there was a remarkable change in the clinical condition. 8. The anoxemia of pneumonia is due, in large measure, to an impairment of the respiratory surface of the lungs. The greater the lung involvement the greater the anoxemia. Especially is this so when the pneumonic process extends throughout the lungs so that there are many patches of bronchopneumonia, with accompanying bronchitis and edema, as evidenced by the presence of râles throughout the lungs. 9. Rapid and shallow breathing of the degree observed in pneumonia is, as far as present evidence shows, of less importance in the production of anoxemia.\nHere is the question:\nANESTHESIOLOGY, CRITICAL CARE AND EMERGENCIES: A patient admitted for acute pancreatitis starts with tachypnea, tachycardia, sweating and progressive cyanosis. PaO2 is 55 mm Hg (PaO2/FiO2 ratio<200). CXR shows bilateral alveolar pulmonary infiltrates. Pulmonary capillary wedge pressure is normal. Oxygen therapy does not improve the situation. What is the most probable diagnosis:\nHere are the potential choices:\n1. Nosocomial pneumonia.\n2. Cardiac failure.\n3. Carcinomatous lymphangitis.\n4. Pulmonary thromboembolism.\n5. Respiratory distress.\nThe correct answer is: ", "gold_answer": "5 Respiratory distress.", "prediction": "You are a helpful medical expert, and your task is to answer a multi-choice medical question using the relevant documents. Please choose the answer from the provided options. Your responses will be used for research purposes only, so please have a definite answer.\nHere are the relevant documents:\n In contrast to pulmonary parenchyma metastases or lymphangitic carcinomatosis, neoplastic emboli of small pulmonary arteries and capillaries frequently go unrecognized and are only discovered at autopsy. Five patients (48 +/- 12 years old) were admitted to 3 intensive care units for severe acute respiratory failure and died between the first and the tenth day following hospitalization. Each patient had a history of rapidly progressive dyspnea, and physical examination showed clinical evidence of right ventricular failure. The lungs were clear on chest X-rays and the ECG revealed sinus tachycardia with a right QRS axis. The mean partial pressures of oxygen (PaO2) and carbon dioxide (PaCO2) were, respectively, 50.8 +/- 9.1 mm Hg and 22.2 +/- 2.4 mm Hg. A swan-Ganz catheter, inserted into 4 patients, revealed pulmonary arterial hypertension (55, 43, 37, 28) with capillary wedge pressure within the normal limits and cardiac output normal or low (3.0, 3.8, 4.4, 5.0 l/min). Pulmonary angiograms from each patient showed decreased distal lung perfusion without any proximal defects suggestive of pulmonary embolism. The inferior vena cava always appeared clear. Malignant cells were found upon autopsy (4 cases) in the lumina of the pulmonary arterioles and the primary site of the cancer was determined in 3 patients (2 hepatomas and 1 pancreatic carcinoma). The last patient had a known breast cancer with bone marrow metastases and clinical, hemodynamic and angiographic evidence of neoplastic emboli. The clinical course of neoplastic emboli can suggest acute pulmonary embolism, but the diagnosis can only be advanced after pulmonary angiography, especially if the patient is to have a cancer.(ABSTRACT TRUNCATED AT 250 WORDS) To report clinical application of Extracorporeal membrane oxygenation for severe acute respiratory and heart failure in a child with severe pneumonia. A seven-year old male patient with severe pneumonia complicated with heart and lung function failure was admitted to PICU in 28th of December, 2008.Veno-artery access was set up via euthyphoria cannulation in operative incision. Blood was drained from the right atrium through a cannula introduced via femoral veins, and returned via femoral artery. The inter-surface of the ECMO equipment system was completely coated with heparin-coating technique. Anticoagulation was maintained with heparin to keep the activated clotting time (ACT) between 150 and 200 seconds and heparin usage dose was 10 U/(kg.h), mean blood flow was 1/2-2/3 of 80-120 ml/(kg.min) during ECMO assistant period. During ECMO, ventilator settings were gradually reduced to allow lung rest, i.e. peak inspiratory pressure less than 25 cm H2O (1 cm H2O=0.098 kPa), end expiratory pressure 8-10 cm H2O, rate 10-15 breaths per minute and FiO2 30%-40%. In management of ECMO, the incipient blood flow was set at 0.8 L/min, the radio of oxygen and blood flow was 1:1, FiO2 60%. After ten minutes of ECMO working, the blood oxygen saturation of radial artery increased from 40 mm Hg (1 mm Hg=0.133 kPa) to 177 mm Hg, Lac decreased from 3.5 mmol/L to 2.8 mmol/L. Four hours later, blood gas analysis of radial artery showed PaO2 202 mm Hg, PCO2 44 mm Hg, Lac 1.5 mmol/L, blood flow was set at 0.6 L/min, FiO2 60%, PaO2 kept above 150 mm Hg. 96 hours after ECMO supporting, the blood flow was set at 0.4 L/min [20 ml/(kg.min)], the results of blood gas analysis of radial artery was PaO2 190 mm Hg, PaCO2 36 mm Hg, SaO2 100%, Lac 0.9 mmol/L, then the child weaned off successfully from ECMO. Two days later, the child was successfully extubated. After two weeks treatment, the patient was discharged. The main complication associated with extracorporeal membrane oxygenation were bleeding. ECMO is an effective mechanical assistant therapy method for severe pulmonary and cardiac failure in a child. Acute Respiratory Distress Syndrome (ARDS) is a potential complication of cardiac surgery, given that patients undergoing CABG frequently have hypoxemia and pulmonary dysfunction during initial hours after surgery. Thus, ARDS criteria in these patients are more likely to be positive while these criteria may not match the patient`s clinical picture. We aimed to investigate frequency of rapid onset hypoxemia in Pressure of Arterial Oxygen to Fractional Inspired Oxygen Concentration (PaO2/FiO2) less than 200 and diffuse pulmonary infiltrates as two diagnostic criteria forwards and compared these criteria with the clinical picture of the patients after Coronary Artery Bypass Graft (CABG) in this study. The study was prospective case series which carried out in about six months. All patients admitted to intensive care unit of Tehran Heart Center, who had undergone CABG on cardiopulmonary pump (CPB) recruited in the study. After considering inclusion criteria, age, sex, duration of intubation, arterial blood gas and chest radiography, on 24 hours and 48 hours after admission to the ICU were recorded. Then, patients with rapid onset of hypoxemia (PaO2/FiO2≤200mmHg) and diffuse pulmonary infiltrates and without sign or symptoms of obvious heart failure (probable positive ARDS cases) criteria were recorded and comparison between these probable positive cases with clinician`s clinical diagnosis (blinded to the study) was performed. In this study, a total of 300 patients after on-pump coronary artery bypass surgery were included. Postoperatively, 2 (0.66 %) in the 24 hours and 4 (1.33%) patients in 48 hours after surgery were positive for the two ARDS criteria according to the checklists, but; nobody had saved persistently ARDS criteria persistently during 48 hours after surgery. At the same time, clinician did not report any case of ARDS among 300 patients. In this study patients with ARDS criteria had no significant differences in age (P.value=0.937) and sex (P.value=0.533). Duration of intubation in patients with ARDS (14.26 ± 4.25 hours) in the first 48 hours was higher but not statistically different from the group without ARDS (11.60 ± 5.45 hours) (P.value=0.236). ARDS diagnosis based on rapid onset of hypoxemia (PaO2/FiO2≤200 mmHg) and diffuse pulmonary infiltrates and without signs or symptoms of obvious heart failure criteria in patients undergoing CABG could lead to overdiagnosis or misdiagnosis in less than 24 hours follow up. We recommend following patients for more than 24 hours and revise the current ARDS criteria for CABG patients. We evaluated the frequency and cause of acute respiratory failure in renal transplant recipients. Our single-center retrospective observational study included consecutive renal transplant recipients who were admitted to an intensive care unit for acute respiratory failure between 2011 and 2017. Acute respiratory failure was defined as oxygen saturation < 92% or partial pressure of oxygen in arterial blood < 60 mm Hg on room air and/or requirement of noninvasive or invasive mechanical ventilation. Of 187 renal transplant recipients, 35 (18.71%) required intensive care unit admission; 11 of these patients (31.4%) were admitted to the intensive care unit with acute respiratory failure. Six of these patients (54.5%) had pneumonia and had shown infiltrates on chest radiography, which were shown in a minimum of 3 zones of the lung (2 with Klebsiella pneumonia, 1 with Acinetobacter species, 1 with Proteus mirabilis, 2 with no microorganisms). The other reasons for acute respiratory failure were cardiogenic pulmonary edema (2 patients), acute respiratory distress syndrome (2 patients, due to acute pancreatitis and acute cerebrovascular thromboembolism), and exacerbation of chronic obstructive pulmonary disease (1 patient). Six patients (54.5%) needed invasive mechanical ventilation because of pneumonia (3 patients), cardiogenic pulmonary edema (2 patients), and cerebrovascular thromboembolism (1 patient). Hemodialysis was administered in 5 patients (45%). Six of 11 patients died due to pneumonia (3 p atients), cardiogenic pulmonary edema (2 patients), and cerebrovascular thromboembolism (1 patient). Among the 5 survivors, 3 (60%) had recovered previous graft function. Acute respiratory failure is associated with high mortality and morbidity in renal transplant recipients. Main causes of acute respiratory failure were bacterial pneumonia and cardiogenic pulmonary edema in our study population. Extended chemoprophylaxis for bacterial and fungal infection and early intensive care unit admission of patients with acute respiratory failure may improve outcomes. To assess whether patients breathing spontaneously under standard oxygen could be recognized early as acute respiratory distress syndrome patients according to the current Berlin definition. A post hoc analysis from two prospective studies. Twenty-three French ICUs. All patients admitted for acute hypoxemic respiratory failure and treated with noninvasive ventilation were analyzed. Patients with cardiogenic pulmonary edema, acute exacerbation of chronic obstructive pulmonary disease, or hypercapnia were excluded. None. The PaO2/FIO2 ratio was estimated at admission under standard oxygen and then under noninvasive ventilation 1 hour after initiation and within the first 24 hours. Among the 219 patients treated with noninvasive ventilation for acute hypoxemic respiratory failure, 180 (82%) had bilateral infiltrates including 161 patients with PaO2/FIO2 less than or equal to 300 mm Hg under standard oxygen. Among them, 127 were treated with positive end-expiratory pressure of at least 5 cm H2O, and 120 (94%) fulfilled criteria for acute respiratory distress syndrome within the first 24 hours. The mortality rate of patients with bilateral infiltrates and PaO2/FIO2 less than or equal to 300 mm Hg under standard oxygen was 29%, a rate very close to that of intubated patients with acute respiratory distress syndrome in the Berlin definition. Almost all patients with pulmonary bilateral infiltrates and a PaO2/FIO2 less than or equal to 300 mm Hg under standard oxygen fulfilled the acute respiratory distress syndrome criteria under noninvasive ventilation within the first 24 hours. Their mortality rate was similar to that reported in the Berlin definition of acute respiratory distress syndrome. Therefore, spontaneous breathing patients with the acute respiratory distress syndrome criteria could be identified early without positive pressure ventilation. Berlin Criteria: as stated on UpToDate (2020) Timing: onset of respiratory symptoms within one week of a injury/insult. Chest Imaging: either chest x-ray or CT scan, must show bilateral opacities that cannot be fully explained by other conditions such as effusion, lung/lobar collapse, or lung nodules. Origin of Edema: respiratory failure that cannot be fully explained by cardiac failure or fluid overload, this needs objective assessment such as an echocardiogram. Impaired Oxygenation: this can be determined by looking at the ratio of arterial oxygen tension to fraction of inspired oxygen (PaO2/FiO2) that can be obtained based on an arterial blood gas test. Note: all PaO2/FiO2 ratios used in the determination of the severity of ARDS require that the patient be on a ventilator at a setting that includes 5 cm H2O or more of positive end-expiratory pressure (PEEP) or continuous positive airway pressure (CPAP). Causes may include sepsis, pancreatitis, trauma, pneumonia, and aspiration. The underlying mechanism involves diffuse injury to cells which form the barrier of the microscopic air sacs of the lungs, surfactant dysfunction, activation of the immune system, and dysfunction of the body's regulation of blood clotting. In effect, ARDS impairs the lungs' ability to exchange oxygen and carbon dioxide. Adult diagnosis is based on a PaO2/FiO2 ratio (ratio of partial pressure arterial oxygen and fraction of inspired oxygen) of less than 300 mm Hg despite a positive end-expiratory pressure (PEEP) of more than 5 cm H2O. Cardiogenic pulmonary edema, as the cause, must be excluded. To examine the relationship between age and mortality in ARDS patients and evaluate the importance of factors that increase the mortality of older ARDS patients. Prospective inception cohort study. Community-based referral hospital. Two hundred fifty-six ARDS patients identified from May 1987 to December 1990. ARDS was defined by the following: (1) PaO2/PAO2 < or = 0.2; (2) pulmonary capillary wedge pressure < or = 15 mm Hg; (3) total static thoracic compliance < or = 50 mL/cm H2O; (4) bilateral infiltrates on chest radiograph; and (5) an appropriate clinical setting for ARDS. Comparison of organ failure, incidence of sepsis, patient demographics, arterial oxygenation, and level of support in those 55 years and younger and those older than 55 years of age. Withdrawal of support in patients who died. Seventy-two of 112 patients older than 55 years (64%) died vs 65 of 144 patients 55 years and younger (45%) (p = 0.002). Examination of patient groups using age identified older than 55 years as a \"cutpoint\" above which mortality was greater (p = 0.002). Older nonsurvivors did not differ from nonsurvivors 55 years or younger with respect to gender, smoking history, ARDS risk factors, ARDS identifying characteristics, APACHE II (acute physiology and chronic health evaluation), number of organ failures, or the incidence of sepsis. In the 48 h prior to death, nonsurvivors 55 years and younger had more organ failure (3.4 +/- 0.2 vs 2.8 +/- 0.2; p = 0.03), higher fraction of inspired oxygen (0.82 +/- 0.03 vs 0.68 +/- 0.03; p = 0.008), and higher positive end-expiratory pressure levels (13 +/- 1 vs 8 +/- 1; p = 0.001) than older nonsurvivors. Despite more severe expression of disease, only 32 (50%) nonsurvivors 55 years and younger had support withdrawn. Significantly more nonsurvivors older than 55 years (73%) had support withdrawn (p = 0.009). Even in the absence of chronic disease states, withdrawal was more likely for patients older than 55 years (21/51) than in those 55 years and younger (3/32; p < 0.001). Mortality is significantly higher for patients with ARDS older than 55 years. Decisions to withdraw support are made more often in ARDS patients older than 55 years. These data suggest that age bias may influence decisions to withdraw support. Phase 1 (acute injury): Normal physical exam; possible respiratory alkalosis. Phase 2 (6–48 hours): Hyperventilation, hypocapnia, widening A-a oxygen gradient. Phase 3: Acute respiratory failure, tachypnea, dyspnea, ↓ lung compliance, scattered rales, diffuse chest infltrates on CXR (see Figure 2.15-5). F IGU R E 2.1 5-5. AP CXR showing a diffuse alveolar filling pattern 2° to ARDS. (Reproduced, with permission, from Kasper DL et al. Harrison’s Principles of Internal Medicine, 16th ed. New York: McGraw-Hill, 2005: 1497.) ■ Phase 4: Severe hypoxemia unresponsive to therapy; ↑ intrapulmonary shunting; metabolic and respiratory acidosis. The criteria for ARDS diagnosis (according to the American-European Consensus Conference defnition) are as follows: Acute onset of respiratory distress. PaO2/FiO2 ratio ≤ 200 mmHg. Bilateral pulmonary infltrates on CXR. No evidence of cardiac origin (capillary wedge pressure < 18 mmHg or no clinical evidence of elevated left atrial pressure). To define the occurrence rate of acute respiratory distress syndrome (ARDS) using established criteria in a well-defined general patient population, to study the clinical course of ARDS when patients were ventilated using a \"lung-protective\" strategy, and to define the total costs of care. A 3-yr (1993 through 1995) retrospective descriptive analysis of all patients with ARDS treated in Kuopio University Hospital. Intensive care unit in the university hospital. Fifty-nine patients fulfilled the definition of ARDS: Pao2/Fio2 <200 mm Hg (33.3 kPa) during mechanical ventilation and bilateral infiltrates on chest radiograph. None. With a patient data management system, the day-by-day data of hemodynamics, ventilation, respiratory mechanics, gas exchange, and organ failures were collected during the period that Pao2/Fio2 ratio was <200 mm Hg (33.3 kPa). The frequency of ARDS was 4.9 cases/100,000 inhabitants/yr. Pneumonia and sepsis were the most common causes of ARDS. Mean age was 43+/-2 yrs. At the time of lowest Pao2/Fio2, the nonsurvivors had lower arterial and venous oxygen saturations and higher arterial lactate than survivors, whereas there were no differences between the groups in other parameters. Multiple organ dysfunction preceded the worst oxygenation in both the survivors and nonsurvivors. The intensive care mortality was 37%; hospital mortality and mortality after a minimum 8 months of follow-up was 42%. The most frequent cause of death was multiple organ failure. The effective costs of intensive care per survivor were US $73,000. The outcome of ARDS is unpredictable at the time of onset and also at the time of the worst oxygenation. Keeping the inspiratory pressures low (30-35 cm H2O [2.94 to 3.43 kPa]) reduces the frequency of pneumothorax, and might lower the mortality. Most patients are young, and therefore the costs per saved year of life are low. MAJOR COMPLICATIONS Cardiopulmonary Complications Ventilation-perfusion mismatching produces a fall in arterial Po2 early in the course. Increasing alveolar epithelial injury and capillary permeability result in increased pulmonary water content, which decreases pulmonary compliance and interferes with oxygen exchange. In the absence of pneumonia or heart failure, progressive diffuse pulmonary infiltrates and arterial hypoxemia occurring within 1 week of a known insult indicate the development of mild acute respiratory distress syndrome (ARDS) (200 mmHg < Pao2/Fio2 ≤ 300 mmHg), moderate ARDS (100 mmHg < Pao2/Fio2 ≤ 200 mmHg), or severe ARDS (Pao2/Fio2 ≤100 mmHg). Acute lung injury or ARDS develops in ~50% of patients with severe sepsis or septic shock. Respiratory muscle fatigue can exacerbate hypoxemia and hypercapnia. An elevated pulmonary capillary wedge pressure (>18 mmHg) suggests fluid volume overload or cardiac failure rather than ARDS. Pneumonia caused by viruses or by This type of respiratory failure is caused by conditions that affect oxygenation, such as: Low ambient oxygen (e.g. at high altitude) Ventilation-perfusion mismatch (parts of the lung receive oxygen but not enough blood to absorb it, e.g. pulmonary embolism) Alveolar hypoventilation (decreased minute volume due to reduced respiratory muscle activity, e.g. in acute neuromuscular disease); this form can also cause type 2 respiratory failure if severe. Diffusion problem (oxygen cannot enter the capillaries due to parenchymal disease, e.g. in pneumonia or ARDS) Shunt (oxygenated blood mixes with non-oxygenated blood from the venous system, e.g. right to left shunt) Type 2 Hypoxemia (PaO2 <8kPa or normal) with hypercapnia (PaCO2 >6.0kPa). The basic defect in type 2 respiratory failure is characterized by: {| class=\"wikitable\" |PaO2 || decreased (< )or normal |- | PaCO2 || increased (> ) |- | PA-aO2 || normal |- |pH || <7.35 |} This type of respiratory failure occurs with alveolar flooding and subsequent intrapulmonary shunt physiology. Alveolar flooding may be a consequence of pulmonary edema, pneumonia, or alveolar hemorrhage. Pulmonary edema can be further categorized as occurring due to elevated pulmonary microvascular pressures, as seen in heart failure and intravascular volume overload or ARDS (“low-pressure pulmonary edema,” Chap. 322). This syndrome is defined by acute onset (≤1 week) of bilateral opacities on chest imaging that are not fully explained by cardiac failure or fluid overload and of shunt physiology requiring positive end-expiratory pressure (PEEP). Type I respiratory failure occurs in clinical settings such as sepsis, gastric aspiration, pneumonia, near-drowning, multiple blood transfusions, and pancreatitis. The mortality rate among patients with ARDS was traditionally very high (50–70%), although changes in patient care have led to mortality rates closer to 30% (see below). Equations Abbreviated alveolar air equation PAO2, PEO2, and PIO2 are the partial pressures of oxygen in alveolar, expired, and inspired gas, respectively, and VD/Vt is the ratio of physiologic dead space over tidal volume. Medicine In medicine, the FIO2 is the assumed percentage of oxygen concentration participating in gas exchange in the alveoli. Uses The FIO2 is used in the APACHE II (Acute Physiology and Chronic Health Evaluation II) severity of disease classification system for intensive care unit patients. For FIO2 values equal to or greater than 0.5, the alveolar–arterial gradient value should be used in the APACHE II score calculation. Otherwise, the PaO2 will suffice. The ratio between partial pressure of oxygen in arterial blood (PaO2) and FIO2 is used as an indicator of hypoxemia per the American-European Consensus Conference on lung injury. A high FIO2 has been shown to alter the ratio of PaO2/FIO2. Cardiovascular dysfunction a) Shock b) Cardiac Arrest c) Severe hypoperfusion (lactate >5 mmol/L or >45 mg/dL) d) Severe acidosis (pH<7.1) e) Use of continuous vasoactive drugs f) Cardio-pulmonary resuscitation Respiratory dysfunction g) Acute cyanosis h) Gasping i) Severe tachypnea (respiratory rate>40 breaths per minute) j) Severe bradypnea (respiratory rate<6 breaths per minute) k) Severe hypoxemia (O2 saturation <90% for ≥60min or PAO2/FiO2<200) l) Intubation and ventilation not related to anaesthesia Renal dysfunction m) Oliguria non responsive to fluids or diuretics n) Severe acute azotemia (creatinine >300 μmol/ml or >3.5 mg/dL) o) Dialysis for acute renal failure Coagulation dysfunction p) Failure to form clots q) Severe acute thrombocytopenia (<50,000 platelets/ml) r) Massive transfusion of blood or red cells (≥ 5 units) Hepatic dysfunction s) Jaundice in the presence of pre-eclampsia Noninvasive estimation techniques have been proposed. Clinical significance Because of the large compliance of pulmonary circulation, it provides an indirect measure of the left atrial pressure. For example, it is considered the gold standard for determining the cause of acute pulmonary edema; this is likely to be present at a PWP of >20mmHg. It has also been used to diagnose severity of left ventricular failure and mitral stenosis, given that elevated pulmonary capillary wedge pressure strongly suggests failure of left ventricular output. Traditionally, it was believed that pulmonary edema with normal PWP suggested a diagnosis of acute respiratory distress syndrome (ARDS) or non cardiogenic pulmonary edema (as in opiate poisoning). However, since capillary hydrostatic pressure exceeds wedge pressure once the balloon is deflated (to promote a gradient for forward flow), a normal wedge pressure cannot conclusively differentiate between hydrostatic pulmonary edema and ARDS. Removal of the extrinsic cause or treatment of underlying infection if identifed. Corticosteroid treatment may be used if no cause is identif ed. Causes include ventilation-perfusion (V/Q) mismatch, right-to-left shunt, hypoventilation, low inspired O2 content (important at altitudes), and diffusion impairment. Findings depend on the etiology. ↓HbO2 saturation, cyanosis, tachypnea, shortness of breath, pleuritic chest pain, and altered mental status may be seen. Pulse oximetry: Demonstrates ↓ HbO2 saturation. CXR: To rule out ARDS, atelectasis, or an infltrative process (e.g., pneumonia) and to look for signs of pulmonary embolism. ABGs: To evaluate PaO2 and to calculate the alveolar-arterial (A-a) oxygen gradient ([(Patm − 47) × FiO2 − (PaCO2/0.8)] − PaO2). An ↑ A-a gradient suggests a V/Q mismatch or a diffusion impairment. Figure 2.15-4 summarizes the approach toward hypoxemic patients. Is PaCO2 increased? Hypoventilation Yes Yes No No Is PAO2 − PaO2 increased? Twenty-nine patients, divided into three groups: 1) chronic obstructive pulmonary disease; 2) acute or chronic pulmonary disease with left heart failure; 3) respiratory insufficiency after peritonitis, pancreatitis, and/or sepsis, were studied during respirator treatment with regard to gas exchange, breathing mechanics and central circulation. The dead space ventilation was somewhat greater in group 1 than in the other groups. The alveolar-arterial oxygen tension difference was least in group 1, greater in group 2 and extremely high in group 3. Neither dynamic compliance of the thorax nor inspiratory resistance showed any significant differences between the groups. The cardiac output had the highest values in group 3. The venous admixture was generally small in group 1 and extremely large in group 3. The pulmonary artery pressures were highest in group 2. Three variables proved to be valuable when assessing the prognosis of a patient: a large venous admixture; a large alveolar-arterial oxygen tension difference, and a high pulmonary artery pressure indicated a less favourable prognosis. 1. The use of an oxygen chamber in the treatment of pneumonia patients makes it possible to administer this gas for long periods of time under exactly known conditions. The medical and nursing care of the patient is greatly facilitated. 2. Prolonged inhalation of oxygen varying from 40 to 60 per cent appears to be without harm. 3. Oxygen administered to intensely anoxemic patients almost immediately clears up this anoxemia. Cyanosis disappears with the anoxemia. 4. The removal of patients from the high oxygen while they are still sick and while examination shows that there are still extensive edema and infiltration of the lung results in a return of the intense anoxemia. 5. It is sometimes impossible to clear up the anoxemia, even when as high as 60 per cent of oxygen is given, especially when there are considerable edema and infiltration of the lungs. 6. Five cases in which the prognosis was grave recovered. Three cases, one of tuberculosis, one with a Pneumococcus Type III infection, and a third with a pneumonia superimposed on a chronic pulmonary condition, died. 7. In all cases there appeared to be an improvement in the patient's condition. In one case, particularly, with an intense degree of anoxemia, the patient became moribund and pulseless. Following the administration of 60 per cent of oxygen there was a lowering of the heart rate from 160 to 120, the return of the pulse to the radial artery, the color became bright pink, and there was a remarkable change in the clinical condition. 8. The anoxemia of pneumonia is due, in large measure, to an impairment of the respiratory surface of the lungs. The greater the lung involvement the greater the anoxemia. Especially is this so when the pneumonic process extends throughout the lungs so that there are many patches of bronchopneumonia, with accompanying bronchitis and edema, as evidenced by the presence of râles throughout the lungs. 9. Rapid and shallow breathing of the degree observed in pneumonia is, as far as present evidence shows, of less importance in the production of anoxemia.\nHere is the question:\nANESTHESIOLOGY, CRITICAL CARE AND EMERGENCIES: A patient admitted for acute pancreatitis starts with tachypnea, tachycardia, sweating and progressive cyanosis. PaO2 is 55 mm Hg (PaO2/FiO2 ratio<200). CXR shows bilateral alveolar pulmonary infiltrates. Pulmonary capillary wedge pressure is normal. Oxygen therapy does not improve the situation. What is the most probable diagnosis:\nHere are the potential choices:\n1. Nosocomial pneumonia.\n2. Cardiac failure.\n3. Carcinomatous lymphangitis.\n4. Pulmonary thromboembolism.\n5. Respiratory distress.\nThe correct answer is: 5. Respiratory distress." }, { "question": "You are a helpful medical expert, and your task is to answer a multi-choice medical question using the relevant documents. Please choose the answer from the provided options. Your responses will be used for research purposes only, so please have a definite answer.\nHere are the relevant documents:\n Assessment of the etiology of urinary tract infections and pathogen drug sensitivity in hospitalized children. We analyzed 156 medical records of patients admitted to the Clinical Department of Pediatrics, Bielański Hospital in Warsaw in 2012, with a suspected UTI. Positive urine culture results were found in 113 (72.4%) children (68; 60.2% of girls and 45; 39.8% of boys), aged from 2 months to 17.9 years (the average age was 2 years and 3 months). E. coli was the most frequent isolated pathogen - 92.0% of patients (104/113). The greatest sensitivity of pathogens showed to cephalosporins of the second and third generation (80.5-90.3%). The sensitivity to amoxicillin with clavulanic acid was 71.7% and 41.6% for ampicillin. The length of hospital stay and treatment ranged from 2 to 16 days (average 8.6 days). In 60.2% (68/113) of patients were treated with second cephalosporin, in 17.7% (20/113) with third generation cephalosporins. Only 11.5% of them (13/113) received amoxicillin with clavulanic acid. Before the treatment, 69.9% (79/113) of children had a fever from 38 up to 41,7ºC, and the fever persisted for the average of 2.5 days (1-8 days). We found significantly higher levels of CRP in children aged between 2-4 in comparison to other age groups (p= 0.0290). In 44.2% (50/113) of children the cystourethrography was performed and in 22% (11/50) cases we recognized a unilateral or bilateral vesicoureteral-ureter of a I to IV degree, on one or both sides. The most common etiological agent of UTIs in children remains E. coli. The sensitivity of urinary pathogens to the commonly used antibiotics is still high, however, finds a large percentage of strains resistant to ampicillin and to amoxicillin with clavulanic acid. The antibiotic recommended for empiric therapy of UTIs in children should be cephalosporins, if there is such a possibility, the treatment should be based on drug sensitivity tests of the organisms grown. Because of the relatively long hospitalization of children with UTIs and the possibility of hospital complications, sequential treatment should also be considered sequential. A first urinary tract infection (UTI) in childhood is more prevalent in females < 5-years-old. Circumcision generally protects males from UTI, however, during the month following the procedure, the prevalence of infection increases up to 12 times in circumcised boys when compared with those not circumcised. Almost all the infections are caused by aerobic Gram-negative bacteria of which E. coli are responsible for 70-90% of the cases. Signs and symptoms of UTI vary in different age groups. Factors associated with the likelihood of UTI are: non-circumcised male, fever > 40 degrees C, and a fever > 39 degrees C for more than 48 hours with no other focus of infection on physical examination. Urinalysis and urine microscopy are screening tests for UTI. In children with clinical symptoms and signs suggesting UTI, the results of these tests have a positive predictive value (if both are positive), or negative predictive value (if both are negative) approximating 100%. The definitive diagnosis of UTI is based on the urine culture. Bag urine culture is associated with a very high rate of contamination. Therefore, in non-toilet trained children, urine culture should be obtained directly from the urinary bladder either by supra pubic aspiration or in and out transurethral catheterization. Mid stream clean voided urine specimens obtained from circumcised males in the first months of life are also acceptable. Depending on the clinical presentation, oral therapy can begin from as early as two months of age, and the recommended empiric drugs for first febrile UTI are cefuroxime axetil, or amoxicillin clavulanate. Cephlexin is recommended for cystitis. During the course of antibiotic treatment, serial white blood cell count and temperature are closely monitored. Typically, the intravenous antibiotics are continued until the person has no fever for at least 24 to 48hours, then equivalent antibiotics by mouth can be given for a total of twoweek duration of treatment. Intravenous fluids may be administered to compensate for the reduced oral intake, insensible losses (due to the raised temperature) and vasodilation and to optimize urine output. Percutaneous nephrostomy or ureteral stent placement may be indicated to relieve obstruction caused by a stone. Children with acute pyelonephritis can be treated effectively with oral antibiotics (cefixime, ceftibuten and amoxicillin/clavulanic acid) or with short courses (2 to 4days) of intravenous therapy followed by oral therapy. If intravenous therapy is chosen, single daily dosing with aminoglycosides is safe and effective. Infants less than 3 months of age with urinary tract infection are usually hospitalized. Recent studies show that a less aggressive management for those patients aged ≥ 29 days may be feasible. To determine the complication rate in infants<3 months of age with urinary tract infection, and to identify the causative agents and their antibiotic susceptibility. A retrospective study was conducted on infants<3 months of age with positive urinalysis results, together with a positive urine culture from a catheterized specimen and seen in the Emergency Department from 2007 to 2012. Demographics, clinical and microbiology (microorganism isolated and antibiotic susceptibility) data were collected. The complications rate (bacteremia, bacterial meningitis, renal abscess, surgical intervention, Intensive Care Unit admission, or death) were calculated for the overall sample and for different age groups (<29, 29-60, and 61-90 days). A total of 460 patients are included; 137 (29.8%)<29, 166 (36.1%) 29-60, and 157 (34.1%) 61-90 days of age. Twenty four (5.4%; 95% CI: 3.6-7.8) had bacteremia; 15 (10.9%; 95% CI: 6.7-17.3) were<29 days; 8 (4.9%; 95% CI: 2.5-9.4) were 29-60 days, and one (0.7%; 95% CI: 0.1-3.7) was 61-90 days of age (P<.001). One neonate (0.8%; 95% CI: 0.1-4.1) had bacterial meningitis, and 2, renal abscess. Escherichia coli was the common pathogen identified (87.2%) in the urine culture, with a susceptibility to amoxicillin-clavulanate, gentamicin, and cefixime of 89.2, 97.0, and 96.0%, respectively. Complications are low in infants<3 months of age with UTI, especially in those ≥ 29 days of age. The identification of patients at very low risk for complications would allow a less aggressive management. Escherichia coli antibiotic susceptibility remains stable, but continuing careful surveillance is essential to optimize empirical antibiotic treatment. The infectious disease is the main source of care demand in Pediatric Emergency Departments (PED) and is a frequent cause of hospital admission with antibiotics. Our objectives are: 1) to determine the diseases that are seen in PED that required admission with antibiotics; 2) to determine the microbiological methods used and, 3) to analyze the characteristics of the indicated treatment. A prospective multicenter study was conducted in 22 Spanish hospitals. We included patients younger than 18 years seen in PED on day 14 of each month between June 2009 and May 2010 who required hospitalization with systemic antibiotics. Patients admitted to Intensive Care Unit were excluded. There were 30,632 consultations in the PED during the study period. A total of 1,446 (4.7%) patients were hospitalized, 395 (27.3%) of them with antibiotics. Ninety-five patients (24.1%) had received antibiotics before admission. Three hundred twenty (81%) children underwent at least one microbiological test, with blood culture (69.9%) and urine culture (30.9%) being the most requested ones. The main diagnoses at admission were pneumonia (29.4%), urinary tract infection (15.4%), and fever without source (12.1%). Twenty five different antibiotics were prescribed, with cefotaxime (27.8%) and amoxicillin-clavulanate (23.4%) being the most prescribed ones. A single antibiotic was prescribed to 80.8% of patients, and parenteral administration was the most indicated (93.7%). Antibiotic therapy was prescribed in one in every 4 patients who required admission to hospital. Pneumonia was the most common source. Blood culture was the most frequent microbiological test requested in the PED. A limited number of beta-lactam antibiotics represented the majority of antibiotic prescriptions. Urinary tract infections (UTIs) are a common diagnosis within the pediatric emergency department (ED). Because of the necessary delay in obtaining urine culture results, clinicians must decide whether to prescribe antibiotics for a suspected UTI before urine culture results. The primary objective of this study was to identify the proportion of children given empiric antibiotics who subsequently did not meet consensus definition of an UTI. The secondary objective was to identify factors associated with return visits to the ED after an index visit for UTI. We also attempted to identify predictors of prescription of empiric antibiotics for children who did not have a UTI. This was a retrospective chart review of all patients between the ages of 2 months and 18 years diagnosed with a UTI between July 2016 and June 2017 in the ED of a single urban quaternary care center. Patients were excluded for the following reasons: use of bag for urine collection, subsequent admission to the hospital, receipt of antibiotics within the previous 3 days, use of antibiotics for an indication other than a UTI, and urine culture obtained at an outside facility. Of 404 included patients, 389 (96.2%) were discharged on antibiotics and 243 (62.4%) did not have a UTI. On the multivariate analysis, age ≧ 36 months was associated with increased odds of receiving antibiotics and not having a UTI while both ≥1+ leukocyte esterase and ≥1+ nitrites on urinalysis were associated with decreased odds of receiving antibiotics and not meeting UTI criteria. Sixty-two patients revisited the ED within 30 days of the initial visit, 24 (38.7%) of which met criteria for UTI during the index visit. Prescription of antibiotics at the time of the index visit was associated with decreased odds of reutilization, whereas an extended-spectrum β-lactamase producing organism cultured from urine at the index visit was associated with increased odds of reutilization. A high number of patients discharged on empiric antibiotics did not meet criteria for a UTI. We did not identify clinically useful factors that predicted prescription of empiric antibiotics for children who do not have a UTI. We believe that unnecessary antibiotic prescriptions could be substantially decreased by decreasing empiric use of antibiotics coupled with reliable follow-up for positive urine cultures. Diagnosis Laboratory examination Analysis of the urine may show signs of urinary tract infection. Specifically, the presence of nitrite and white blood cells on a urine test strip in patients with typical symptoms are sufficient for the diagnosis of pyelonephritis, and are an indication for empirical treatment. Blood tests such as a complete blood count may show neutrophilia. Microbiological culture of the urine, with or without blood cultures and antibiotic sensitivity testing are useful for establishing a formal diagnosis, and are considered mandatory. Diagnosis In straightforward cases, a diagnosis may be made and treatment given based on symptoms alone without further laboratory confirmation. In complicated or questionable cases, it may be useful to confirm the diagnosis via urinalysis, looking for the presence of urinary nitrites, white blood cells (leukocytes), or leukocyte esterase. Another test, urine microscopy, looks for the presence of red blood cells, white blood cells, or bacteria. Urine culture is deemed positive if it shows a bacterial colony count of greater than or equal to 103 colony-forming units per mL of a typical urinary tract organism. Antibiotic sensitivity can also be tested with these cultures, making them useful in the selection of antibiotic treatment. However, women with negative cultures may still improve with antibiotic treatment. As symptoms can be vague and without reliable tests for urinary tract infections, diagnosis can be difficult in the elderly. Assess H and P for “low risk” status Age? <1 mo >1 mo Low risk Not low risk CBC with differential, blood culture, urine culture All parameters normal? No Yes All parameters normal? NoYes outpatients without empirical antibiotic treatment or, alternatively, treated with intramuscular ceftriaxone. Regardless of antibiotic treatment, close follow-up for at least 72 hours, including re-evaluation in 24 hours or immediately with any clinical change, is essential. Children with a positive blood culture require immediate re-evaluation, repeat blood culture, consideration for lumbar puncture, and empirical antibiotic treatment. Urine dipsticks have to be used more frequently for the screening of urinary tract infections (UTI) in febrile infants and children (grade A). Confirmation of the UTI by urine culture should prefer other methods of sampling than the urine bag: sampling jet, urethral catheterization, or pubic puncture (grade A). The percentage of Escherichia coli producing extended-spectrum beta-lactamases (ESBL) in children accounts for less than 10 % in France and does not justify revising the 2007 recommendations (grade B). An increase in the use of carbapenems in first-line treatment is a major environmental hazard and exposes the patient to the risk of untreatable infections. For febrile UTI, the expert group recommended: (1) recover the results of susceptibility testing as soon as possible to quickly adapt treatment for possible resistant strains; (2) favor initial treatment with aminoglycosides (particularly amikacin) which remain active in the majority of ESBL strains for patients seen in the pediatric emergency department and/or hospital; (3) ceftriaxone (IV or IM) remains an appropriate treatment for patients seen in the emergency department or outpatient clinic because the percentage of ESBL-producing enterobacteria strains remains low; (4) use oral cefixime (grade B) in nonsevere cases and low-risk patients defined as age>3 months, general condition preserved, disease duration of fever<4 days, no associated comorbidity, and no history of urinary tract infection, uropathy, or prior antibiotic therapy in the last 3 months; (5) oral relay for parenteral treatment is guided by in vitro susceptibility testing, in an attempt to reduce the use of oral cephalosporins to limit the selection of resistant bacterial strains. The total duration of treatment recommended is usually 10 days. Except for special circumstances, there is no need to prescribe retrograde cystography or antibiotic prophylaxis after a first febrile urinary tract infection. For cystitis, the panel recommends systematic urinalysis and initial prescription before the results of the urine culture of one of the three following oral antibiotics: amoxicillin-clavulanate, cotrimoxazole, cefixime. The total duration of antibiotic treatment is 5days to tailor treatment based on clinical progression and antibiotic susceptibility. Urinary tract infection is a common cause of serious bacterial infection in young children. The non-specific presentation has implications for misdiagnosis and the potential for long-term complications. To determine if a negative dipstick urinalysis is adequate to exclude urinary tract infection in children aged 0-10 years. Data was subdivided into two age groups: 0-2 years and 2-10 years. Retrospective case note review over an 8-month period. Cases included required a printed urinalysis recorded from the Clinitek 50 (Bayer) machine and a printed microscopy and culture result. We defined a negative urinalysis as being negative for all of blood, protein, leucocytes and nitrites. A total of 375 cases were included for statistical calculation. Three hundred and seventy-five cases gave a prevalence of 10.7% with a sensitivity of 92.5%, specificity of 39.4% and a negative predictive value of 97.8%. In the 0-2-year-old group, we demonstrated a prevalence of 15%, a sensitivity of 87.5%, specificity of 39.7% and a negative predictive value of 94.7%. This compares to the older group (2-10 years) with a prevalence of 7.0%, a sensitivity of 100%, specificity of 39.7% and a negative predictive value of 100%. Prevalence of urinary tract infection varied with age with a higher prevalence in the 0-2 years age group. The lower negative predictive value and the higher clinical importance in this age group means that dipstick urinalysis is inadequate to exclude urinary tract infection. Conversely, we believe that children in the 2-10 years age group can adequately have urinary tract infection excluded with a negative dipstick urinalysis. Acute fever of unknown origin (FUO) in children under 29 days is a worrying situation because of the risk of serious bacterial infection (SBI). to study the main clinical and laboratory characteristics of a group of hospitalized children under 29 days with diagnosis of FUO. Retrospective study of children under 29 days hospitalized due to FUO. The clinical records of the patients were reviewed, recording age, sex, history of fever before consultation, temperature at admission, estimated severity at admission and discharge, discharge diagnoses, laboratory tests, and indicated treatments. Patients were classified according to the severity of the discharge diagnosis, as severe (S) and non-severe (NS). The inclusion criteria were term newborn, age less than 29 days, fe ver > 38°C registered at home or admission, and history of < 4 days. 468 children with FUO were admitted. Concordance between severity at admission and discharge was low (Kappa = 0.125; p = 0.0007). 26.1% of children were S and 73.9% NS. In the S group, urinary tract infection domínate (70.5%) and in the NS, FUO (67.6%). The cut-off levels for leukocytes/mm3, C-reactive protein, and neutrophils/mm3 showed negative predictive values to rule out severe bacterial infection. Conclu sions: Most of the newborns presented mild severity at admission, but 24% of them had SBI, thus hospitalization and close clinical observation are always necessary. Laboratory tests, such as CRP, white blood cell and neutrophils count are not good predictors of SBI. Early treatment with antibio tics for patients who meet the low-risk criteria is debatable. Most episodes of fever in children younger than 3 years of age have a demonstrable source of infection elicited by history, physical examination, or a simple laboratory test. In this age group, the most commonly identified serious bacterial infection is a UTI. A blood culture to evaluate for occult bacteremia, and urinalysis and urine culture to evaluate for a UTI, should be considered for all children younger than 3 years of age with fever without localizing signs. Stool culture should be obtained in those with diarrhea marked by blood or mucous. Ill-appearing children should be admitted to the hospital and treated with empirical antibiotics. Approximately 0.2% of well-appearing febrile children 3 to 36 months of age vaccinated against S. pneumoniae and To investigate antibiotic resistance among pathogens isolated from urines in a tertiary care children's hospital in Italy. Retrospective analysis of prospectively collected data on antibiotic susceptibility of Gram-negatives isolated from urines at the Istituto Giannina Gaslini, Genoa - Italy from 2007 to 2014. Antibiotic susceptibility was evaluated. By means of CLSI criteria from 2007 to 2010, while from 2011 EUCAST criteria were adopted. Data on susceptibility to amoxicillin-clavulanate, co-trimoxazole, cefuroxime, nitrofurantoin, fosfomycin and ciprofloxacin were evaluated for Escherichia coli, while for other Enterobacteriaceae data were collected for amoxicillin-clavulanate, co-trimoxazole and ciprofloxacin and for ciprofloxacin against Pseudomonas aeruginosa. Univariate and multivariable analyses were performed for risk factors associated with resistance. A total of 4596 Gram-negative strains were observed in 3364 patients. A significant increase in the proportion of resistant strains was observed for E.coli against amoxicillin-clavulanate, cefuroxime and ciprofloxacin and for others Enterobacteriaceae against co-trimoxazole and ciprofloxacin. Resistance to nitrofurantoin and fosfomycin was very infrequent in E.coli. Logistic regression analysis showed that repeated episode of urinary tract infections was a risk factor for E.coli resistance to amoxicillin-clavulanate, co-trimoxazole and cefuroxime, while admission in one of the Units usually managing children with urinary tract malformations was significantly associated to resistance to amoxicillin-clavulanate and cefuroxime. In conclusion the present study shows an increase in antibiotic resistance in pediatric bacteria isolated from urines in children, especially in presence of repeated episodes and/or urinary tract malformations. This resistance is worrisome for beta-lactams and cotrimoxazole, and start to increase also for fluoroquinolones while nitrofurantoin and fosfomycin still could represent useful drugs for oral treatment of these infections. • Infections are frequent in patients with urinary tract malformations • Antibiotic prophylaxis can select for resistant pathogens What is New: • The increase in the resistance to β-lactams, co-trimoxazole or fluoroquinolones in pathogens causing urinary tract infections cause a reduction of drugs with oral formulations available for therapy • Old drugs like nitrofurantoin and fosfomycin can represent attractive compounds for oral treatment of urinary tract infections in children presence of resistance to other drug classes. Acute fever of unknown origin (AFUO) is established when the anamnesis and physical examination cannot identify the cause. In infants less than 3 months-old this is situation for concern, due to the risk of a serious bacterial infection. To describe the clinical and laboratory variable of patients with AFUO, in order to look for clues in order to base studies on the decisions arising drom this problem. A report is presented on a retrospective study conducted on a cohort of children less than three months-old admitted to the Hospital Roberto del Río (2007-2011) due to an AFUO. Clinical histories were reviewed and the patients were grouped, according to the severity of the admission diagnosis, into severe and non-severe. They were compared in strata determined by the variables of clinical interest. A total of 550 children were admitted with AFUO during the study period. There was low agreement between the severity on admission and at discharge (kappa=0.079; P=.26). There were 23.8% of children in the severe group and 76.2% in the non-severe group. Urinary tract infection predominated in the severe group (68.7%) and 40.7% with acute febrile syndrome in the non-severe group. The cut-off levels for C-reactive protein, white cells, and neutrophils per mm(3), to calculate the fixed and variable indices, only showed negative predictive values of some use for ruling out serious bacterial infection. The ROC curves with white cell and neutrophil counts and C-reactive protein, did not provide andy fixed indices of clinical use. More than one-third (34.6%) of lumbar punctures were traumatic or failures. According to the results of this study, there is an obvious excess of hospital admissions, little usefulness in the examinations to identify serious bacterial infection, a high percentage lumbar punctures traumatic and lumbar punctures failures, and an excess of antibiotic treatments. A review of clinical criteria and procedures is needed. Ewingella americana (Ea) is a Gram-negative, lactose-fermenting, oxidase-negative and catalase-positive bacterium that was first described in 1983 as a new genus and species in the family Enterobacteriaceae. It is not known whether Ea is a true pathogen or simply an opportunistic infectious agent, as most of the cases have been described in patients at risk. A 4-year-old girl described here was hospitalized due to a productive cough over the previous 3 weeks and a fever > 38 °C associated with tachypnea over the previous 2 days. Her familial and personal medical histories were negative for relevant diseases, including respiratory infections. At admission, she was febrile (axillary temperature 39.2 °C) and had dyspnea with retractions, grunting and nasal flaring. A chest examination revealed fine crackling rales in the left upper field associated with bilateral wheezing. A chest X-ray revealed segmental consolidation of the lingula of the left lung. Laboratory tests revealed leukocytosis (15.,800 white blood cells/mm1 mo Low risk Not low risk CBC with differential, blood culture, urine culture All parameters normal? No Yes All parameters normal? NoYes outpatients without empirical antibiotic treatment or, alternatively, treated with intramuscular ceftriaxone. Regardless of antibiotic treatment, close follow-up for at least 72 hours, including re-evaluation in 24 hours or immediately with any clinical change, is essential. Children with a positive blood culture require immediate re-evaluation, repeat blood culture, consideration for lumbar puncture, and empirical antibiotic treatment. Urine dipsticks have to be used more frequently for the screening of urinary tract infections (UTI) in febrile infants and children (grade A). Confirmation of the UTI by urine culture should prefer other methods of sampling than the urine bag: sampling jet, urethral catheterization, or pubic puncture (grade A). The percentage of Escherichia coli producing extended-spectrum beta-lactamases (ESBL) in children accounts for less than 10 % in France and does not justify revising the 2007 recommendations (grade B). An increase in the use of carbapenems in first-line treatment is a major environmental hazard and exposes the patient to the risk of untreatable infections. For febrile UTI, the expert group recommended: (1) recover the results of susceptibility testing as soon as possible to quickly adapt treatment for possible resistant strains; (2) favor initial treatment with aminoglycosides (particularly amikacin) which remain active in the majority of ESBL strains for patients seen in the pediatric emergency department and/or hospital; (3) ceftriaxone (IV or IM) remains an appropriate treatment for patients seen in the emergency department or outpatient clinic because the percentage of ESBL-producing enterobacteria strains remains low; (4) use oral cefixime (grade B) in nonsevere cases and low-risk patients defined as age>3 months, general condition preserved, disease duration of fever<4 days, no associated comorbidity, and no history of urinary tract infection, uropathy, or prior antibiotic therapy in the last 3 months; (5) oral relay for parenteral treatment is guided by in vitro susceptibility testing, in an attempt to reduce the use of oral cephalosporins to limit the selection of resistant bacterial strains. The total duration of treatment recommended is usually 10 days. Except for special circumstances, there is no need to prescribe retrograde cystography or antibiotic prophylaxis after a first febrile urinary tract infection. For cystitis, the panel recommends systematic urinalysis and initial prescription before the results of the urine culture of one of the three following oral antibiotics: amoxicillin-clavulanate, cotrimoxazole, cefixime. The total duration of antibiotic treatment is 5days to tailor treatment based on clinical progression and antibiotic susceptibility. Urinary tract infection is a common cause of serious bacterial infection in young children. The non-specific presentation has implications for misdiagnosis and the potential for long-term complications. To determine if a negative dipstick urinalysis is adequate to exclude urinary tract infection in children aged 0-10 years. Data was subdivided into two age groups: 0-2 years and 2-10 years. Retrospective case note review over an 8-month period. Cases included required a printed urinalysis recorded from the Clinitek 50 (Bayer) machine and a printed microscopy and culture result. We defined a negative urinalysis as being negative for all of blood, protein, leucocytes and nitrites. A total of 375 cases were included for statistical calculation. Three hundred and seventy-five cases gave a prevalence of 10.7% with a sensitivity of 92.5%, specificity of 39.4% and a negative predictive value of 97.8%. In the 0-2-year-old group, we demonstrated a prevalence of 15%, a sensitivity of 87.5%, specificity of 39.7% and a negative predictive value of 94.7%. This compares to the older group (2-10 years) with a prevalence of 7.0%, a sensitivity of 100%, specificity of 39.7% and a negative predictive value of 100%. Prevalence of urinary tract infection varied with age with a higher prevalence in the 0-2 years age group. The lower negative predictive value and the higher clinical importance in this age group means that dipstick urinalysis is inadequate to exclude urinary tract infection. Conversely, we believe that children in the 2-10 years age group can adequately have urinary tract infection excluded with a negative dipstick urinalysis. Acute fever of unknown origin (FUO) in children under 29 days is a worrying situation because of the risk of serious bacterial infection (SBI). to study the main clinical and laboratory characteristics of a group of hospitalized children under 29 days with diagnosis of FUO. Retrospective study of children under 29 days hospitalized due to FUO. The clinical records of the patients were reviewed, recording age, sex, history of fever before consultation, temperature at admission, estimated severity at admission and discharge, discharge diagnoses, laboratory tests, and indicated treatments. Patients were classified according to the severity of the discharge diagnosis, as severe (S) and non-severe (NS). The inclusion criteria were term newborn, age less than 29 days, fe ver > 38°C registered at home or admission, and history of < 4 days. 468 children with FUO were admitted. Concordance between severity at admission and discharge was low (Kappa = 0.125; p = 0.0007). 26.1% of children were S and 73.9% NS. In the S group, urinary tract infection domínate (70.5%) and in the NS, FUO (67.6%). The cut-off levels for leukocytes/mm3, C-reactive protein, and neutrophils/mm3 showed negative predictive values to rule out severe bacterial infection. Conclu sions: Most of the newborns presented mild severity at admission, but 24% of them had SBI, thus hospitalization and close clinical observation are always necessary. Laboratory tests, such as CRP, white blood cell and neutrophils count are not good predictors of SBI. Early treatment with antibio tics for patients who meet the low-risk criteria is debatable. Most episodes of fever in children younger than 3 years of age have a demonstrable source of infection elicited by history, physical examination, or a simple laboratory test. In this age group, the most commonly identified serious bacterial infection is a UTI. A blood culture to evaluate for occult bacteremia, and urinalysis and urine culture to evaluate for a UTI, should be considered for all children younger than 3 years of age with fever without localizing signs. Stool culture should be obtained in those with diarrhea marked by blood or mucous. Ill-appearing children should be admitted to the hospital and treated with empirical antibiotics. Approximately 0.2% of well-appearing febrile children 3 to 36 months of age vaccinated against S. pneumoniae and To investigate antibiotic resistance among pathogens isolated from urines in a tertiary care children's hospital in Italy. Retrospective analysis of prospectively collected data on antibiotic susceptibility of Gram-negatives isolated from urines at the Istituto Giannina Gaslini, Genoa - Italy from 2007 to 2014. Antibiotic susceptibility was evaluated. By means of CLSI criteria from 2007 to 2010, while from 2011 EUCAST criteria were adopted. Data on susceptibility to amoxicillin-clavulanate, co-trimoxazole, cefuroxime, nitrofurantoin, fosfomycin and ciprofloxacin were evaluated for Escherichia coli, while for other Enterobacteriaceae data were collected for amoxicillin-clavulanate, co-trimoxazole and ciprofloxacin and for ciprofloxacin against Pseudomonas aeruginosa. Univariate and multivariable analyses were performed for risk factors associated with resistance. A total of 4596 Gram-negative strains were observed in 3364 patients. A significant increase in the proportion of resistant strains was observed for E.coli against amoxicillin-clavulanate, cefuroxime and ciprofloxacin and for others Enterobacteriaceae against co-trimoxazole and ciprofloxacin. Resistance to nitrofurantoin and fosfomycin was very infrequent in E.coli. Logistic regression analysis showed that repeated episode of urinary tract infections was a risk factor for E.coli resistance to amoxicillin-clavulanate, co-trimoxazole and cefuroxime, while admission in one of the Units usually managing children with urinary tract malformations was significantly associated to resistance to amoxicillin-clavulanate and cefuroxime. In conclusion the present study shows an increase in antibiotic resistance in pediatric bacteria isolated from urines in children, especially in presence of repeated episodes and/or urinary tract malformations. This resistance is worrisome for beta-lactams and cotrimoxazole, and start to increase also for fluoroquinolones while nitrofurantoin and fosfomycin still could represent useful drugs for oral treatment of these infections. • Infections are frequent in patients with urinary tract malformations • Antibiotic prophylaxis can select for resistant pathogens What is New: • The increase in the resistance to β-lactams, co-trimoxazole or fluoroquinolones in pathogens causing urinary tract infections cause a reduction of drugs with oral formulations available for therapy • Old drugs like nitrofurantoin and fosfomycin can represent attractive compounds for oral treatment of urinary tract infections in children presence of resistance to other drug classes. Acute fever of unknown origin (AFUO) is established when the anamnesis and physical examination cannot identify the cause. In infants less than 3 months-old this is situation for concern, due to the risk of a serious bacterial infection. To describe the clinical and laboratory variable of patients with AFUO, in order to look for clues in order to base studies on the decisions arising drom this problem. A report is presented on a retrospective study conducted on a cohort of children less than three months-old admitted to the Hospital Roberto del Río (2007-2011) due to an AFUO. Clinical histories were reviewed and the patients were grouped, according to the severity of the admission diagnosis, into severe and non-severe. They were compared in strata determined by the variables of clinical interest. A total of 550 children were admitted with AFUO during the study period. There was low agreement between the severity on admission and at discharge (kappa=0.079; P=.26). There were 23.8% of children in the severe group and 76.2% in the non-severe group. Urinary tract infection predominated in the severe group (68.7%) and 40.7% with acute febrile syndrome in the non-severe group. The cut-off levels for C-reactive protein, white cells, and neutrophils per mm(3), to calculate the fixed and variable indices, only showed negative predictive values of some use for ruling out serious bacterial infection. The ROC curves with white cell and neutrophil counts and C-reactive protein, did not provide andy fixed indices of clinical use. More than one-third (34.6%) of lumbar punctures were traumatic or failures. According to the results of this study, there is an obvious excess of hospital admissions, little usefulness in the examinations to identify serious bacterial infection, a high percentage lumbar punctures traumatic and lumbar punctures failures, and an excess of antibiotic treatments. A review of clinical criteria and procedures is needed. Ewingella americana (Ea) is a Gram-negative, lactose-fermenting, oxidase-negative and catalase-positive bacterium that was first described in 1983 as a new genus and species in the family Enterobacteriaceae. It is not known whether Ea is a true pathogen or simply an opportunistic infectious agent, as most of the cases have been described in patients at risk. A 4-year-old girl described here was hospitalized due to a productive cough over the previous 3 weeks and a fever > 38 °C associated with tachypnea over the previous 2 days. Her familial and personal medical histories were negative for relevant diseases, including respiratory infections. At admission, she was febrile (axillary temperature 39.2 °C) and had dyspnea with retractions, grunting and nasal flaring. A chest examination revealed fine crackling rales in the left upper field associated with bilateral wheezing. A chest X-ray revealed segmental consolidation of the lingula of the left lung. Laboratory tests revealed leukocytosis (15.,800 white blood cells/mm